Вы находитесь на странице: 1из 595

Aligned on Class 11 th & 12 th Syllabus

38 JEE ADVANCED
+ 14 Years
Years (2002-2015) JEE MAIN
1978 - 2015

PHYSICS
Topic-wise Solved Paper

Er. Sunil Batra


• Head Office : B-32, Shivalik Main Road, Malviya Nagar, New Delhi-110017

• Sales Office : B-48, Shivalik Main Road, Malviya Nagar, New Delhi-110017
Tel. : 011-26691021 / 26691713

Typeset by Disha DTP Team

DISHA PUBLICATION
ALL RIGHTS RESERVED

© Reserved
No part of this publication may be reproduced in any form without prior permission of the publisher. The author and the
publisher do not take any legal responsibility for any errors or misrepresentations that might have crept in. We have tried
and made our best efforts to provide accurate up-to-date information in this book.

For further information about the books from DISHA,


Log on to www.dishapublication.com or email to info@dishapublication.com
Index
• JEE Advanced 2015 Solved Paper 2015-1-16

• JEE Main 2015 Solved Paper 1-8


• JEE Advanced 2014 Solved Paper 2014-1-15

• JEE Main 2014 Solved Paper 1-8

CHAPTERS BASED ON CLASS 11TH SYLLABUS

1. Units & Measurements 1 - 10

2. Motion 11 - 30

3. Laws of Motion 31 - 54

4. Work, Energy & Power 55 - 72

5. Momentum and Impulse 73 - 86

6. Rotational Motion 87 - 132

7. Gravitation 133 - 144

8. Mechanical Properties of Solids & Fluids 145 - 166

9. Heat & Thermodynamics and Gases 167 - 220

10. Simple Harmonic Motion (Oscillations) 221 - 240

11. Waves 241 - 272


CHAPTERS BASED ON CLASS 12TH SYLLABUS
12. Electrostatics 273 - 324

13. Current Electricity 325 - 360

14. Moving Charges & Magnetism 361 - 402

15. Electromagnetic Induction & Alternating Current 403 - 432

16. Ray & Wave Optics 433 - 490

17. Modern Physics 491 - 544

* The chapters have been divided as per the Class 11th & 12th syllabus followed by the NCERT books. Some of the chapters which
are split in the class 11th & 12th syllabus in NCERT have been combined. There might be certain topics/ chapters which are not
covered in NCERT but are a part of JEE Advanced/IIT-JEE syllabus.
JEE ADVANCED 2015 PHYSICS
PAPER - 1
SECTION - I
This section contains 8 questions. Each question, when worked
out will result in one integer from 0 to 9 (both inclusive).
1. Two spherical stars A and B emit blackbody radiation. The 15cm
radius of A is 400 times that of B and A emits 104 times the

æ lA ö
power emitted from B. The ratio ç l ÷ of their wavelengths 50cm
è Bø
5. An infinitely long uniform line charge distribution of charge
lA and lB at which the peaks occur in their respective
per unit length l lies parallel to the y-axis in the y – z plane at
radiation curves is
2. A nuclear power plant supplying electrical power to a village 3
z= a (see figure). If the magnitude of the flux of the
uses a radioactive material of half life T years as the fuel. The 2
amount of fuel at the beginning is such that the total power electric field through the rectangular surface ABCD lying in
requirement of the village is 12.5% of the electrical power lL
available from the plant at that time. If the plant is able to the x – y plane with its centre at the origin is ne
meet the total power needs of the village for a maximum period 0
of nT years, then the value of n is (e0 = permittivity of free space), then the value of n is
3. A Young's double slit interference arrangement with slits S1 z
4
and S2 is immersed in water (refractive index = ) as shown
3 L
D 3
in the figure. The positions of maximum on the surface of C a
x 2
water are given by x 2 = p2 m2l2 – d2 , where l is the a y
wavelength of light in air (refractive index = 1), 2d is the O
separation between the slits and m is an integer. The value of A
p is B

6. Consider a hydrogen atom with its electron in the nth orbital.


S1 An electromagnetic radiation of wavelength 90 nm is used to
d
Air
ionize the atom. If the kinetic energy of the ejected electron is
x
10.4 eV, then the value of n is (hc = 1242 eV nm)
d
7. A bullet is fired vertically upwards with velocity v from the
S2 Water
surface of a spherical planet. When it reaches its maximum
1 th
height, its acceleration due to the planet's gravity is of
4
4. Consider a concave mirror and a convex lens (refractive index its value of the surface of the planet. If the escape velocity
= 1.5) of focal length 10 cm each, separated by a distance of
from the planet is vesc = v N , then the value of N is (ignore
50 cm in air (refractive index = 1) as shown in the figure. An
object is placed at a distance of 15 cm from the mirror. Its energy loss due to atmosphere)
erect image formed by this combination has magnification 8. Two identical uniform discs roll without slipping on two
M1. When the set-up is kept in a medium of refractive index different surfaces AB and CD (see figure) starting at A and C
with linear speeds v1 and v2, respectively, and always remain
7 M in contact with the surfaces. If they reach B and D with the
, the magnification becomes M2. The magnitude 2 is
6 M1 same linear speed and v1 = 3 m/s then v2 in m/s is (g = 10 m/s2)
2-2015 JEE Advanced 2015 Solved Paper

M
v1 = 3m/s two point masses each of mass at rest at O. These masses
A 8
30m can move radially outwards along two massless rods fixed
on the ring as shown in the figure. At some instant the angular
B
8
speed of the system is w and one of the masses is at a
9
v2
C
3
27m distance of R from O. At this instant the distance of the
5
D
other mass from O is

w
SECTION - II
This section contains 10 multiple choice questions. Each question
has 4 choices (a), (b), (c) and (d) out of which ONE or MORE O
THAN ONE are correct.
9. Planck’s constant h, speed of light c and gravitational constant
G are used to form a unit of length L and a unit of mass M.
Then the correct option(s) is(are) 2 1
(a) R (b) R
3 3
(a) Mµ c (b) M µ G

(c) Lµ (d) L µ 3 4
h G (c) R (d) R
5 5
10. Two independent harmonic oscillators of equal mass are
oscillating about the origin with angular frequencies w1 and 12. The figures below depict two situations in which two infinitely
w2 and have total energies E1 and E2, respectively. The long static line charges of constant positive line charge
variations of their momenta p with positions x are shown in density l are kept parallel to each other. In their resulting
electric field, point charges q and –q are kept in equilibrium
a a between them. The point charges are confined to move in the
the figures. If = n2 and = n, then the correct equation(s)
b R x direction only. If they are given a small displacement about
is(are) their equilibrium positions, then the correct statement(s) is(are)

p P
l l l l
Energy = E1 Energy = E2

b x x
x x +q –q
a R

(a) Both charges execute simple harmonic motion


w2 (b) Both charges will continue moving in the direction of
(a) E1w1 = E2w2 (b) 2
w1 = n their displacement
(c) Charge +q executes simple harmonic motion while
E1 E2 charge –q continues moving in the direction of its
(c) w1w2 = n2 (d) =
w1 w2 displacement
11. A ring of mass M and radius R is rotating with angular speed (d) Charge –q executes simple harmonic motion while
w about a fixed vertical axis passing through its centre O with charge +q continues moving in the direction of its
displacement
JEE Advanced 2015 Solved Paper 2015-3

13. Two identical glass rods S1 and S2 (refractive index = 1.5)


have one convex end of radius of curvature 10 cm. They are
placed with the curved surfaces at a distance d as shown in
the figure, with their axes (shown by the dashed line) aligned. Q
When a point source of light P is placed inside rod S1 on its
axis at a distance of 50 cm from the curved face, the light rays
emanating from it are found to be parallel to the axis inside
Al 50mm
S2. The distance d is
Fe
S1 S2
P 2mm P

d 7mm
50 cm
2475 1875
(a) 60 cm (b) 70 cm (a) µW (b) µW
64 64
(c) 80 cm (d) 90 cm
14. A conductor (shown in the figure) carrying constant current 1875 2475
(c) µW (d) µW
r 49 132
I is kept in the x-y plane in a uniform magnetic field B . If F is
the magnitude of the total magnetic force acting on the 17. For photo-electric effect with incident photon wavelength l,
conductor, then the correct statement(s) is(are) the stopping potential is V0. Identify the correct variation(s)
of V0 with l and 1/l.
y
R R V0
V0
l p/6 p/4 x
L R R L (a) (b)
r l
(a) If B is along $z , F µ (L + R) l
r
(b) If B is along x̂ , F = 0 V0
r (c) (d) V0
(c) If B is along ŷ , F µ (L + R)
r
(d) If B is along $z , F = 0 1/l 1/l
15. A container of fixed volume has a mixture of one mole of 18. Consider a Vernier callipers in which each 1 cm on the main
hydrogen and one mole of helium in equilibrium at temperature scale is divided into 8 equal divisions and a screw gauge
T. Assuming the gases are ideal, the correct statement(s) is with 100 divisions on its circular scale. In the Vernier callipers,
(are) 5 divisions of the Vernier scale coincide with 4 divisions on
(a) The average energy per mole of the gas mixture is 2RT the main scale and in the screw gauge, one complete rotation
(b) The ratio of speed of sound in the gas mixture to that in of the circular scale moves it by two divisions on the linear
scale. Then :
helium gas is 6/5
(a) If the pitch of the screw gauge is twice the least count
(c) The ratio of the rms speed of helium atoms to that of of the Vernier callipers, the least count of the screw
hydrogen molecules is 1/2 gauge is 0.01 mm
(d) The ratio of the rms speed of helium atoms to that of (b) If the pitch of the screw gauge is twice the least count
1 of the Vernier callipers, the least count of the screw
hydrogen molecules is gauge is 0.005 mm
2
(c) If the least count of the linear scale of the screw gauge
16. In an aluminium (Al) bar of square cross section, a square is twice the least count of the Vernier callipers, the
hole is drilled and is filled with iron (Fe) as shown in the least count of the screw gauge is 0.01 mm
figure. The electrical resistivities of Al and Fe are 2.7 × 10–8
(d) If the least count of the linear scale of the screw gauge
W m and 1.0 × 10–7 W m, respectively. The electrical resistance
is twice the least count of the Vernier callipers, the
between the two faces P and Q of the composite bar is
least count of the screw gauge is 0.005 mm
4-2015 JEE Advanced 2015 Solved Paper
SECTION - III Column I Column II
This section contains TWO questions. Each question contains 2
two columns, Column I and Column II. Column I has four entries U0 é æ x ö2 ù
(A) U1(x) = ê1 - ç ÷ ú (P) The force acting on the
(A), (B), (C) and (D). Column II has five entries (P), (Q), (R), (S) 2 êë è a ø úû
and (T). Match the entries in Column I with the entries in Column
II. One or more entries in Column I may match with one or more particle is zero at x = a
entries in Column II. 2
U0 æxö
19. Match the nuclear processes given in column I with the (B) U2(x) = ç ÷ (Q) The force acting on the
2 èaø
appropriate option(s) in column II.
particle is zero at x = 0
Column I Column II
(A) Nuclear fusion (P) Absorption of thermal U 0 æ x ö2
(C) U3(x) = ç ÷ (R) The force acting on the
2 èaø
neutrons by 235
92 U
é æ x ö2 ù
(B) Fission in a nuclear (Q) 60 Co nucleus exp ê- ç ÷ ú particle is zero at x = – a
27 êë è a ø úû
reactor
(C) b-decay (R) Energy production in U0
(D) U4(x) = (S) The particle experiences
2
stars via hydrogen
conversion to helium é x 1 æ x ö3 ù
ê – ç ÷ ú an attractive force towards
(D) g-ray emission (S) Heavy water êë a 3 è a ø úû
(T) Neutrino emission
x = 0 in the region |x| < a
20. A particle of unit mass is moving along the x-axis under the (T) The particle with total
influence of a force and its total energy is conserved. Four
possible forms of the potential energy of the particle are given U0
energy can oscillate
in column I (a and U0 constants). Match the potential energies 4
in column I to the corresponding statement(s) in column II. about the point x = – a

PAPER - 2
SECTION - I
This section contains 8 questions. Each question, when worked
out will result in one integer from 0 to 9 (both inclusive).
1. For a radioactive material, its activity A and rate of change of 60° q
dN dA
its activity R are defined as A = - and R = – , where
dt dt
N(t) is the number of nuclei at time t. Two radioactive sources
P (mean life t) and Q (mean life 2t) have the same activity at 3. In the following circuit, the current through the resistor
t = 0. Their rates of change of activities at t = 2t are RP and R (= 2 W) is I amperes. The value of I is
RP n
RQ, respectively. If = , then the value of n is (=2W)
RQ e
2. The monochromatic beam of light is incident at 60° on one
face of an equilateral prism of refractive index n and emerges
from the opposite face making an angle q(n) with the normal
dq
(see the figure). For n = 3 the value of q is 60° and = m.
dn
The value of m is
JEE Advanced 2015 Solved Paper 2015-5
r uruur
4. An electron is an excited state of Li2+ ion has angular cavity at position r is E (r), then the correct statement(s)
momentum 3h/2p. The de Broglie wavelength of the electron is(are)
in this state is pp a0 (where a0 is the Bohr radius). The value
of p is
5. A large spherical mass M is fixed at one position and two
identical point masses m are kept on a line passing through
the centre of M (see figure). The point masses are connected
by a rigid massless rod of length l and this assembly is free
to move along the line connecting them. All three masses
interact only through their mutual gravitational interaction.
When the point mass nearer to M is at a distance r = 3l from
æ M ö ur
M, the tension in the rod is zero for m = k ç ÷ . The value (a) E is uniform, its magnitude is independent of R2 but
è 288 ø r
its
ur direction depends on r
of k is (b) E is uniform, its magnitude depends on R2 and its
r
direction depends on r
ur
(c) E is uniform, its magnitude is independent of a but its
r
direction depends on a
6. The energy of a system as a function of time t is given as E(t)
ur
(d) E is uniform and both its magnitude and direction
= A2 exp(–at,) where a = 0.2 s–1. The measurement of A has r
an error of 1.25%. If the error in the measurement of time is depend on a
1.50%, the percentage error in the value of E(t) at t = 5 s is 11. In plotting stress versus strain curves for two materials P
7. The densities of two solid spheres A and B of the same radii and Q,a student by mistake puts strain on the y-axis and
stress on the x-axis as shown in the figure. Then the correct
R vary with radial distance r as rA(r) = k æç ö÷ and rB(r) =
r statement(s) is (are)
è Rø
5
ærö
k ç ÷ , respectively, where k is a constant. The moments
è Rø
of inertia of the individual spheres about axes passing through
IB n
their centres are IA and IB, respectively. If , = , the
I A 10
value of n is
8. Four harmonic waves of equal frequencies and equal
p 2p
intensities I0 have phase angles 0, , and p. When they (a) P has more tensile strength than Q
3 3 (b) P is more ductile than Q
are superposed, the intensity of the resulting wave is nI0. (c) P is more brittle than Q
The value of n is (d) The Young's modulus of P is more than that of Q
12. A spherical body of radius R consists of a fluid of constant
SECTION - II density and is in equilibrium under its own gravity. If P(r) is
This section contains 8 multiple choice questions. Each question the pressure at r(r < R), then the correct option(s) is (are)
has 4 choices (a), (b), (c) and (d) out of which ONE or MORE P (r = 3R /4) 63
(a) P(r = 0) = 0 (b) =
THAN ONE are correct. P (r = 2R /3) 80
9. In terms of potential difference V, electric current I, permittivity P (r = 3R /5) 16 P (r = R /2) 20
e0, permeability m0 and speed of light c, the dimensionally (c) = (d) =
P (r = 2R /5) 21 P (r = R /3) 27
correct equation(s) is(are)
(a) m0I2 = e0V2 (b) m0I = m0V 13. A parallel plate capacitor having plates of area S and plate
(c) I = e0cV (d) m0cI = e0V separation d, has capacitance C1 in air. When two dielectrics
10. Consider a uniform spherical charge distribution of radius of different relative primitivities (e1 = 2 and e2 = 4) are
R1 centred at the origin O. In this distribution, a spherical introduced between the two plates as shown in the figure,
cavity of radius R 2, centred at P with distance OP = a C2
= R1 – R2 (see figure) is made. If the electric field inside the the capacitance becomes C2. The ratio C is
1
6-2015 JEE Advanced 2015 Solved Paper
8.5 MeV and 8.5 MeV, respectively. Considering different
conservation laws, the correct option(s) is(are)
(a) x = n, y = n, KSr = 129 MeV, KXe = 86 MeV
(b) x = p, y = e–, KSr = 129 MeV, KXe = 86 MeV
(c) x = p, y = n, KSr = 129 MeV, KXe = 86 MeV
(d) x = n, y = n, KSr = 86 MeV, KXe = 129 MeV
16. Two spheres P and Q of equal radii have densities r1 and r2,
respectively. The spheres are connected by a massless string
and placed in liquids L1 and L2 of densities s1 and s2 and
viscosities h1 and h2, respectively. They float in equilibrium
with the sphere P in L1 and sphere Q in L2 and the string
being taut (see figure). If sphere P alone in L2 has terminal
ur ur
velocity VP and Q alone in L1 has terminal velocity V Q ,
then
(a) 6/5 (b) 5/3
(c) 7/5 (d) 7/3
14. An ideal monoatomic gas is confined in a horizontal cylinder L1
by a spring loaded piston (as shown in the figure). Initially
the gas is at temperature T1, pressure P1 and volume V1 and
the spring is in its relaxed state. The gas is then heated very L2
slowly to temperature T2, pressure P2 and volume V2. During
this process the piston moves out by a distance x. Ignoring
the friction between the piston and the cylinder, the correct
statement(s) is (are)
ur ur
VP h VP h2
(a) ur = 1 (b) ur = h
V Q h2 VQ 1
ur ur ur ur
(c) V P .V Q > 0 (d) V P .V Q < 0

SECTION - III
This section contains 2 paragraphs, each describing theory,
experiments, data etc. four questions related to the two paragraphs
with two questions on each paragraph. Each question has one or
(a) If V2 = 2V1 and T2 = 3T1, then the energy stored in the more than one correct answer(s) among the four given options
(a), (b), (c) and (d).
1 PARAGRAPH 1
spring is PV
4 1 1 Light guidance in an optical fibre can be understood by considering
(b) If V2 = 2V1 and T2 = 3T1, then the change in internal a structure comprising of thin solid glass cylinder of refractive
energy is 3P1V1 index n1 surrounded by a medium of lower refractive index n2. The
(c) If V2 = 3V1 and T2 = 4T1, then the work done by the gas light guidance in the structure takes place due to successive total
7 internal reflections at the interface of the media n1 and n2 as shown
is PV in the figure. All rays with the angle of incidence i less than a
3 1 1
particular value im are confined in the medium of refractive index
(d) If V2 = 3V1 and T2 = 4T1, then the heat supplied to the
n1. The numerical aperture (NA) of the structure is defined as
17 sin im.
gas is PV
6 1 1
236
15. A fission reaction is given by 92 U ® 140
54 Xe +
94
38 Sr + x + y,
236
where x and y are two particles. Considering 92 U to be at
rest, the kinetic energies of the products are denoted by KXe,
KSr, Kx(2 MeV) and Ky(2 MeV), respectively. Let the binding
236 140 94
energies per nucleon of 92 U, 54 Xe and 38 Sr be 7.5 MeV,
JEE Advanced 2015 Solved Paper 2015-7

accumulation continues until the magnetic force is balanced by


17. For two structure namely S1 with n1 = 45 / 4 and n2 = 3/2,
the electric force. The current is assumed to be uniformly
and S2 with n1 = 8/5 and n2 = 7/5 and taking the refractive distributed on the cross-section of the strip and carried by
index of water to be 4/3 and that of air to be 1, the correct electrons.
option(s) is(are)
(a) NA of S1 immersed in water is the same as that of S2
16
immersed in a liquid of refractive index
3 15

6
(b) NA of S1 immersed in liquid of refractive index is
15
the same as that of S2 immersed in water 19. Consider two different metallic strips (1 and 2) of the same
(c) NA of S1 placed in air is the same as that of S2 immersed material. Their lengths are the same, widths are w1 and w2
4 and thicknesses are d1 and d2 respectively. Two points K
in liquid of refractive index and M are symmetrically located on the opposite faces parallel
15
to the x-y plane (see figure). V1 and V2 are the potential
(d) NA of S1 placed in air is the same as that of S2 placed in differences between K and M in strips 1 and 2, respectively.
water Then, for a given current I flowing through them in a given
18. If two structure of same cross-sectional area, but different magnetic field strength B, the correct statement(s) is(are)
numerical apertures NA1 and NA2(NA2 < NA1) are joined (a) If w1 = w2 and d1 = 2d2, then V2 = 2V1
longitudinally, the numerical aperture of the combined
(b) If w1 = w2 and d1 = 2d2, then V2 = V1
structure is
(c) If w1 = 2w2 and d1 = d2, then V2 = 2V1
NA1 NA2 (d) If w1 = 2w2 and d1 = d2, then V2 = V1
(a) (b) NA1 + NA2
NA1 + NA2
20. Consider two different metallic strips (1 and 2) of same
(c) NA 1 (d) NA 2 dimensions (length l, width w and thickness d) with carrier
densities n1 and n2, respectively. Strip 1 is placed in magnetic
PARAGRAPH 2
field B1 and strip 2 is placed in magnetic field B2, both along
In a thin rectangular metallic strip a constant current I flows along positive y-directions. Then V1 and V2 are the potential
the positive x-direction, as shown in the figure. The length, width differences developed between K and M in strips 1 and 2,
and thickness of the strip are l, w and d, respectively.
su respectively. Assuming that the current I is the same for
A uniform magnetic field B is applied on the strip along the both the strips, the correct option(s) is(are)
positive y-direction. Due to this, the charge carriers experience a (a) If B1 = B2 and n1 = 2n2, then V2 = 2V1
net deflection along the z-direction. This results in accumulation
(b) If B1 = B2 and n1 = 2n2, then V2 = V1
of charge carriers on the surface PQRS and appearance of equal
and opposite charges on the face opposite to PQRS. A potential (c) If B1 = 2B2 and n1 = n2, then V2 = 0.5V1
difference along the z-direction is thus developed. Charge (d) If B1 = 2B2 and n1 = n2, then V2 = V1
8-2015 JEE Advanced 2015 Solved Paper

SOLUTIONS
Paper - 1
fm = –10cm fl = +10cm
PA AA TA4 AA l 4B
1. (2) = = ´
PB AB TB4 AB l 4A
1 1 1
l éA P ù 4 é R 2 P ù 4 é 400 ´ 400 ù 4
\ A ê A ´ B ú =ê A ´ B ú =ê ú
l B ë AB PA û 2
êë RB PA úû ë 10 4 û 15cm
lA
\
lB = 2
30cm
2. (3) T No T No T No
No ¾¾
® ¾¾
® ¾¾
® 50cm
2 4 8 For convex lens u = |2fl|
100% 50% 25% 12.5% Therefore image will have a magnification of 1.
Three half life are required. Therefore n = 3 When the set – up is kept in a medium
3. (3) For maxima The focal length of the lens will change
Path defference = ml
1 æ nl ö é1.5 ù
\ S2A – S1A = ml ç –1÷
S1 fl è n s ø f l¢ ê 1 –1ûú
ë
= Þ =
1 æ nl ö 10 é 1.5 ù
ç – 1 ÷ ê –1ú
x 2 + d2 f l¢ ç n¢ ÷ ë7 / 6 û
d è s ø

x Þ fl¢ = 17.5 cm.


A
1 1 1
Applying lens formula – =
v u f l¢
1 1 1
S2 \ – = Þ v = 140 cm.
v –20 17.5
é (n - 1) d 2 + x 2 + d 2 + x 2 ù – d 2 - x 2
\ = ml v 140
ëê ûú M l¢ = Magnification by lens = = = –7
u –20
\ (n – 1) (d 2 + x 2 ) = ml
M2 Mmirror ´ M l¢
æ4 ö 2 2 Now M = Mmirror ´ M = 7
\ ç 3 –1÷ d + x = ml 1 l
è ø
5. (6)
\ d 2 + x 2 = 3ml cylinder
\ d2 + x2 = 9m2l2
\ x2 = 9m2l2 – d2
\ p2 = 9 Þ p=3
4. (7) Applying mirror formula line charge
1 1 1 +
+ =
v u f q q
1 1 1 1 1 3a
= – = + 2
v f u –10 15
a/2 a/2
1 –15 + 10 –5 –1
\ = = =
v 150 150 30 a/2 1
\ v = –30cm tanq = =
3a / 2 3
\ q = 30°
JEE Advanced 2015 Solved Paper 2015-9

The flux through the dotted cylinder by Gauss’s law is Given VB = VD. Therefore from (i) and (ii)
q inlL 3 3
f cylinder = =e m (3)2 + mg × 30 = m (v2)2 + mg × 27
e0 0
4 4
\ V2 = 7
lL 9. (a, c, d)
\ For 360° angle the flux is e L µ hx cy Gz
0
Dimensionally
lL
\ For 60° angle the flux will be 6e [M 0 L1 T 0 ] = [ML2T –1 ]x [LT –1 ]y [M –1L3T –2 ]z
0
Therefore n = 6 M 0 L1 T 0 = Mx – z L2x + y + 3z T–x – y – 2z
hc 13.6 \ x–z=0 Þ x=z
6. (2) = 2 + 10.2 \ 2x + y + 3z = 1 and –x – y – 2z = 0
l n On solving we get
1242 13.6 1 3 1
\ = 2 + 10.2 x= ,y= – ,z=
90 n 2 2 2
\ n2 = 4 \ Lµ h
\ n=2
Lµ G
7. (2) Let h be the height to which the bullet rises
C, D are correct options
–2 M µ hxcyGz
æ hö
then, g1 = g ç1 + ÷ 2 –1 x –1 y –1 3 –2 Z
è Rø M ¢L °T ° µ [ML T ] [LT ] [M L T ]
\ x – z L2x + y + 3z T– x – y –2z
M ¢L °T ° µ M
–2
g æ hö \ x–z=1
Þ = g ç1 + ÷ 2x + y + 3z = 0
4 è R ø
–x – y – 2z = 0
Þ h=R
On solving we get
2GM 1 1 1
We know that ve = = v N (given) ...(i) x= ,y= ,z=–
R 2 2 2
Now applying conservation of energy for the throw \ Mµ C
Loss of kinetic energy = Gain in gravitaional potential energy
A is the correct option.
1 2 GMm æ GMm ö 10. (b, d)
\ –ç–
R ÷ø
mv = – Maximum linear momentum in case 1 is (p1)max = mnmax
2 2R è
b = m [aw1] ...(i)
GM Maximum linear momentum in case 2 is (p2)max = mnmax
\ v= ...(ii) R = m [Rw2]
R
Comparing (i) & (ii) N=2 \ 1 = mw2 ...(ii)

1 1 b aw1
Dividing (i) & (ii) =
8. (7) Total kinetic energy of a rolling disc = mv2 + Iw2 1 w2
2 2
w1 b 1
1 2 1 æ 1 mR 2 ö æç v ö
2
÷ \ w =a= 2 \ B is a correct option.
= mv + ç ÷ n
ø çè R 2 ÷ 2
2 2 è2 ø
1
3 Also E1 = m w12 a 2
K.E = mv2 2
4
1
For surface AB E2 = m w 22 R 2
k.Ei + loss in gravitational potential energy = K.Ef 2
2 2 2 2
3 3 E1 w1 a w1 1 w1 w2 w1
m (3)2 + mg(30) = m VB2 ...(i) \ E = 2´ 2 = 2´ = 2´ =
4 4 2 w2 R w2 n w2 w1 w2
For surface CD
E1 E 2
3 3 \ w = w D is the correct option
m (v2)2 + mg (27) = m VD2 ...(ii) 1 2
4 4
10-2015 JEE Advanced 2015 Solved Paper
11. (d) Applying conservation of angular mumentum about
the axis 1 1
\ =
d - 50 20
8w M 9R2 8w M 2 8w \ d = 70 cm.
MR2 × w = MR2 × + × × + r ×
9 8 25 9 8 9 B is the correct option.
4R 14. (a, b, c)
Þ r= ur uur ur
5
D is the correct option
( )
F = I éë ò dl ´ B ùû
12. (c) Force on change q when it is given a small displacement ur r ur
x is Fnet = F1 – F2 If B is along z then F = I éë(2 L + 2R )$i ´ B x$ ùû
l l
1 l 1 l option [A] is correct
Fnet = 2pe d – x – 2pe d + x ur r ur
0 0 If B is along x then F = I éë(2L + 2R )i$ ´ Bi$ùû = 0
F1 +q F2
l éd + x – d + x ù x ur ur ur
\ Fnet = 2
pe0 êë d 2 - x 2 úû If B is along y then F = I éë(2L + 2R)$i ´ ˆj ùû
x d–x
Option (b) and (c) are also correct
l 2x d
\ Fnet = 2pe 2 15. (a, b, d)
2
0 d –x
2d
line charge 1 line charge 2 3 5
When x << d then Total energy = RT + RT = 4RT
2 2
l
Fnet = pe x and is directed towards the mean position 4RT
0 \ Average energy per mole = = 2RT
therefore the charge +q will execute SHM. 2

l l gRT
We know that Vsound =
M

n1 + n2 n n
= 1 + 2
–q g mi x –1 g1 - 1 g 2 –1
F2
F1
2 1 1
Þ = +
g mix - 1 5 7
-1 -1
3 5

2 3 5
= +
line charge 1 line charge 2 g mix - 1 2 2 = 4
In case of charge (–q)
F2 > F1 therefore the charge –q continues to move in the 1 3
\ g mix - 1 = \ g mix =
direction of its displacement. 2 2
[C] is the correct option.
13. (b) For refraction in S1 (Vs )mix g mix M He
= ´
n1 n2 n2 – n1 (Vs ) He M mix g He
– + =
u v R
1.5 1 1 - 1.5 3
´4
– + = 2 é 1 ´ 2 + 1´ 4 ù
–50 V –10 =
5 êQ M mix = 2
= 3ú
3´ ë û
Þ v = 50 cm.
3
For refraction in S2
n1 n2 n2 – n1 6
– + = =
u v R 5
1 1.5 1.5 –1
– + = 3RT
–(d – 50) µ 10 We know that Vrms =
M
JEE Advanced 2015 Solved Paper 2015-11

(b) is a correct option


(Vrms ) He M H2 2 1 Now if the least count of the linear scale of the screw gauge
\ (V ) = M HE
=
4
=
2
rms H 2 is twice the least count of venier callipers then.
L.C of linear scale of screw gauge = 2 × 0.025 = 0.05cm.
\ options [A], [B] and [C] are correct.
Then pitch = 2 × 0.05 = 0.1cm.
rFe ´ lFe 10–7 ´ 50 ´10 –3 25
16. (b) RFe = = = ´10–4 0.1
AFe 4 ´ 10 –6 2 Then L.C of screw gauge =
100
cm = 0.001cm = 0.01mm.

r Al ´ l Al 2.7 ´ 10 –8 ´ 50 ´ 10 –3 2.7 ´ 50 (c) is a correct option.


RAl = = = × 10–5 19. A ® R, T ; B ® P, S ; C ® P, Q, R, T; D ® P, Q, R, T
AAl (49 – 4) ´ 10 –6 45
Based on facts
= 0.3 × 10–4 20. A ® P, Q, R, T; B ® Q, S; C ® P, Q, R, S; D ® P, R, T
RFe ´ RAl 12.5 ´10 –4 ´ 0.3 ´ 10–4 For A
Rtotal = = » 29µW
RFe + RAl 12.8 ´10 –4 2
dU – d éU 0 æ æ x ö2 ù
(B) is the correct option. Fx = – dx = dx ê 2 ç1 - ç ÷ ú
ç èaø ú
17. (a, c) êë è û
hC hc W
We know that – W = eV0 Þ – = V0 -2U 0
l el e = ( x - a) x ( x + a)
a3
1
For V0 versus we should get a straight line with negative
l U U
slope and positive intercept. U0
For V0 vesus l, we will get a hyperbola. As l decreases V0 2
increases.
(a) and (c) are the correct options
18. (b, c)
–a +a x x
Vernier callipers
U U
1cm
1 MSD = = 0.125cm
8
5 VSD = 4MSD
1
\ 5VSD = 4 × cm = 0.5cm +a x –a +a x
8
\ 1 VSD = 0.1cm
L.C = 1MSD – 1VSD
= 0.125cm – 0.1cm
= 0.025cm
Screw gauge – dU æ xö
For B Fx = = -U 0 ç ÷
One complete revolution = 2M.S.D dx èaø
If the pitch of screw gauge is twice the L.C of vernier callipers
e- x / a
2 2
then pitch = 2 × 0.025 = 0.05cm. – dU
L.C of screw Gauge For C Fx = = U0 x( x - a)( x + a)
dx a3
pitch
= Total no. of divisions of circular scale dU U
For D Fx = - = - 03 [( x - a )( x + a)]
dx 2a
0.05
= cm = 0.0005cm = 0.005 mm.
100
12-2015 JEE Advanced 2015 Solved Paper

Paper - 2

1. (2) R=–
dA
=– – = =
2
d é dN ù d 2 N d N o e
–l t
( )
dt dt êë dt úû dt 2 dt 2 2W 1W
\ R = No l e = (Nol) l e = Aole–lt
2 –lt –lt Balanced
2W
[Q Ao = Nol] I wheatstone
8W
6W 2W bridge
2t 4W
l t 6.5V
RP l P e – l Pt lP e Q 2t e 2t
2
= = ´ = = 10W
\ RQ l – l Qt l
lQ e P t t 2 t e
Q e
et 12W 4W
\ n=2
2. (2) Here ÐMPQ + ÐMQP = 60°. If ÐMPQ = r then ÐMQP
= 60 – r The equivalent resistance of balanced wheat stone
Applying Snell’s law at P bridge is
sin60° = n sin r ...(i)
Differentiating w.r.t ‘n’ we get 6 ´ 18 9
Req= = W
dr 24 2
O = sin r + n cos r × ...(ii) 6.5
dn \ I= = 1A
2 + 4.5

60° 2W Balanced
wheat stone
P Q I bridge
60° r q 6W 2W
60°–rr
6.5V 10W
M

12W 4W
Applying Snell’s law at Q
sin q = n sin (60° – r) ...(iii)

Differentiating the above equation w.r.t ‘n’ we get


3h
dq é dr ù 4. (2) Given mvr = Þn=3
cos q
dn
= sin (60° – r) + n cos (60° – r) êë – dn úû 2p

hr 3h é h ù
dq é tan r ù \
l
=
2p êQ l = mv ú
\cos q = sin (60° – r) – n cos (60° – r) ê– n ú ë û
dn ë û
[from (ii)] 2pr 2 é n2 ù é n2 ù
\ l = 3 = 3 p ê a0 z ú êQ r = a0 ú
dq 1 êë ûú ëê z úû
\ = [sin (60° – r) + cos (60° – r) tan r] ...(iv)
dn cos q
2 é3 ´ 3ù
From eq. (i), substituting n = 3 we get r = 30° \ l = p a0 ê ú = 2pa0
3 ë 3 û
From eq (iii), substituting n = 3 , r = 30° we get q = 60° \ p=2
5. (7) For the tension in the rod to be zero, the force on both
On substituting the values of r and q in eq (iv) we get
the masses m and m should be equal in magnitude and
dq 1 direction. Therefore
=
dn cos 60° [sin 30° + cos 30° tan 30°] = 2 M
3. (1) The equivalent resistance of balanced wheatstone
m m
bridge is
3´ 6
Req = = 2W 3l l
3+ 6
JEE Advanced 2015 Solved Paper 2015-13

GMm Gmm GMm Gmm IB 6


+ = – \ = Þn=6
2 2 2 2
(4l) l (3l) l I A 10

é1 1 ù é p 2p ù
\ 2m = M ê – ú 8. (3) y = I 0 êsin O + sin + sin + sin pú
ë 9 16 û ë 3 3 û
7M é 3 3ù
\ m= y = I0 ê + ú = 3 I0
288
ë 2 2 û
K=7 \ Ir = y2 = 3I0 Þ n=3
6. (4) E = A2 e–0.2t 9. (a, c)We know that
\ loge E = 2 loge A –0.2t
On differentiating we get 1 m0
C= and R =
dE dA dt m0e 0 e0
=2 – 0.2 ´ t
E A t Now, m0 I2 = e0 V2
As errors always add up therefore
m0 V 2
dE æ dA ö æ dt ö \ = = R2 Þ Option A is correct
´ 100 = 2 ç ´ 100 ÷ + 0.2t ç ´ 100 ÷ e0 I 2
E è A ø è t ø Now, e0 I = m0 V
dE m0 I 1
\ × 100 = 2 × 1.25% + 0.2 × 5 × 1.5% = =
E \ e0 V R
Þ Option B is incorrect
dE Now, I = e0C V
\ × 100 = 4%
E 1 V
R \ = =R
e 0C I
7. (6) I = ò (dm)r 2
0 1
\ =R
1
e0
R m0 e0

m0
\ =R Þ Option C is correct
r dr e0
Now, mo C I = e0 V
m0 V R m0 1
\ = = = ´ = µ0
e0 I C C e0 1
R m 0 e0
\ I = ò r ´ 4 pr 2 dr ´ r 2
Þ Option (d) is incorrect
0
10. (d) Assume the cavity to contain similar charge distribution
R of positive and negative charge as the rest of sphere.
\ I = 4 p ò r r 4 dr Electric field at M due to uniformly distributed charge
0 of the whole sphere of radius R1

R R
r 4 pK
\ I A = 4p ò k ´ r 4 dr = òr
5
dr M
R R P
0 0 r
a
4pK æ R6 ö R5
= ç ÷ = 4pK O
R ç 6 ÷ 6
è ø
R 5
ærö
I B = 4p ò K ç ÷ r 4 dr
0
èRø

4pK R10 R5 ur r r
= ´ = 4pK E= r
R5 10 10 3e
14-2015 JEE Advanced 2015 Solved Paper
Electric field at M due the negative charge distribution Here
in the cavity
GMr
ur r uuur P 4pr2 – (P + dP) 4pr2 = r (4pr2) dr
E2 = MP R3
3e
P r
\ The total electric field at M is GM r
\ – ò dp = ò rdr
ur ur ur r r r uuur R3 R
E = E1 + E 2 = r+ MP 0
3e 3e
GM r é 2 2 ù
ur r r r r r r uuur r \ P= R -r
\ E= r+
3e 3e ë( )
a – r éQ r + MP = a ù
û
2R 3 ë û

ur r r é 2 9R 2 ù 2
\ E= a êR – ú 7R
3e P (r = 3R/ 4) ëê 16 úû 63
\ = = 16 =
(d) is the correct option P (r = 2 R/ 3) é 2 4 R 2 ù 5R 2 80
11. (a, b) The maximum stress that P can withstand before êR – ú
êë 9 úû 9
breaking is greater than Q. Therefore (A) is a correct
option.
é 2 9 R2 ù
Strain êR – ú
P (r = 3R/ 5) ëê 25 úû 16
and = =
P (r = 2 R/ 5) é 2 4R 2 ù 21
Max êR – ú
(strain P) ëê 25 úû

B and C are correct options.


13. (d)
Max d/2
(strain Q)

C1 C2
e1 = 2 e2 = 4
Max s/2
stress
Stress of P –
Maximum
stress of Q + C3
e1 = 2
The strain of P is more than Q therefore P is more ductile. s/2
Therefore (B) is a correct option.
stress
Y=
strain 2 e0 s/2 4 e0 s/2
C1 = C2 =
For a given strain, stress is more for Q. Therefore d/2 d/2
YQ > YP.
12. (b, c) Let us consider an elemental mass dm shown in the
shaded portion.

2 e 0 s/2 e0s
C3 = =
dr d d
r
2e 0 s 4 e 0 s
p ´
p + dp C1 ´ C2
+ C3 = d d + e0 s
Ceq = 6e 0 s
C1 + C2 d
d
4 e 0 s e0 s
= +
3 d d
JEE Advanced 2015 Solved Paper 2015-15

7 e0 s 7 é e0 s ù é PV P2V2 4 P1 ù
êQ C1 = d ú
= C1 1 1
\ Ceq =
3 d 3 ë û ê Here on applying T = T we get P2 = 3 ú
ê 1 2 ú
14. (b) Applying combined gas law ê 2V1 ú
ê and V2 = V1 + Ax Þ x = [Q V2 = 3V1 ] ú
PV
1 1 PV ë A û
= 2 2
T1 T2
1 P1 A 2V1 7
If V2 = 2 V1 and T2 = 3T1 then \ W = 2P1V1 + ´ ´ = PV
1 1
2 3 A 3
PV
1 1 P ´ 2V1 2 C is correct option
= 2 Þ P1 = P2
T1 3T1 3 Heat supplied
Now change in internal energy Q = W + DU

f f 7 PV
1 1 3
= + ( P2V2 – PV
1 1)
DU = [nR (T2 – T1)] = [P2V2 – P1V1] 3 2
2 2
For monoatomic gas f = 3 7 PV
1 1 3 é4 ù 41
= + ê P1 3V1 – PV
1 1ú = PV
1 1
3 é3 ù 3 2 ë3 û 6
DU = ê P1 ´ 2V1 – PV 1 1 ú = 3P1 V1
2 ë2 û 236
® 140 94
15. (a) 92 U 54 Xe + 38 Sr + x + y
\ (b) is the correct option.
The number of proton in reactants is equal to the
Now assuming that the pressure on the piston on the
products (leaving x and y) and mass number of product
right hand side (not considering the affect of spring)
(leaving x and y) is two less than reactants
remains the same throughout the motion of the piston
\ x = p, y = e– is ruled out [B] is incorrect
then,
and x = p, y = n is ruled out [C] is incorrect
kx kx Total energy loss = (236 × 7.5) – [140 × 8.5 + 94 × 8.5]
Pressure of gas = P1 + Þ P2 = P1 +
= 219 MeV
A A
where k is spring constant and A = area of piston The energies of kx and ky together is 4MeV
The energy remain is distributed by Sr and Xe which is
1 2 equal to 219 – 4 = 215 MeV
Energy stored = kx
2 \ A is the correct option
kx Also momentum is conserved
P2 = P1 +
A 1
\ K .E. µ . Therefore K.Esr > K.Exe
m
3 kx
P1 = P1 + 16. (a, d) From the figure it is clear that
2 A
(a) s2 > s1
P1 kx (b) r2 > s2 [As the string is taut]
= (c) r1 < s1 [As the string is taut]
2 A
\ r1 < s1 < s2 < r2
P1 A When P alone is in L2
\ kx =
2
Also, 2pr 2 (r1 – s 2 ) g
VP = is negative as r1 < s2
V2 = V1 + Ax 9h2
V1 = Ax Where r is radius of sphere.
V When Q alone is in L1
\ x= 1
A 2pr 2 (r2 – s1 ) g
VQ = is positive as r2 > s1
1 P1 A V1 1 9h1
\ Energy = ´ = PV
1 1
2 2 A 4 ur ur
\ A is correct Therefore V P . V Q < O option (d) is correct
Now
æ kx ö kx
W = ò PdV = ò ç P1 + ÷ dV = ò P1dV + ò dV s1
è A ø A
kx T
\ W = ò P1dV + ò ´ (dx) A T
A
s2
kx 2
\ W = P1 (V2 – V1 ) +
2
16-2015 JEE Advanced 2015 Solved Paper
VP r1 – s 2 h1 15 45 9 3 15
Also = ´ ...(i) NA = - =
VQ r2 – s1 h2 6 16 4 24
For equilibrium of Q For S2 (in water)
4 3 4 3 64 49 3 15
T+ pr s 2 g = pr 3 r 2 g ...(ii) NA = – =
3 3 4 25 25 4 5
For equilibrium of P For S2 (in air)
4 3 4 64 49 15
T+ pr r1 g = pr 3 s1 g ...(iii) NA = – =
3 3 25 25 5
(iii) – (ii) gives
r1 – s2 = s1 – r2 ...(iv) 4
For S2 (in ns = )
From (i) and (iv) 15
VP h VP h1 15 64 49 3
=– 1 \ = NA = – =
VQ h2 VQ h2 4 25 25 4
\ A is also a correct option æ 16 ö
17. (a, c) For S2 ç in ns = ÷
Applying Snell’s law at P; ns sin im = n1 sin (90° – C) è 3 15 ø
ns = Refractive index of surrounding
3 15 64 49 9
NA = – =
Q n2 16 25 25 16
ns n1
C (a), (c) are correct options
P 90° – C
1
im 18. (d) NA = n12 – n22
ns
Here
NA2 < NA1
\ the NA of combined structure is equal to the
n2 smaller value of the two numerical apertures.
Also sin C =
n1 (d) is the correct option.
19. (a, d) When megnetic force balances electric force
FB = FE
n1 q vd B = q E

n2 V
\ vd B = [Q V = E × w]
w
C
é I ù
Now \ V = wvdB = w ê ú´B
ë newd û
n1 n1 n22 é I I ù
NA = sin im = n cos C = n 1 – 2 êvd = neA = newd ú
s s n1 ë û
I
n12 – n22 \ V = ´B
\ NA = ned
ns
1
For S1 (in air) \ Vµ Þ V1d1 = V2d2
d
45 9 3 when d1 = 2d2, V2 = 2V1
NA = – = and when d1 = d2, V2 = V1
16 4 4
For S1 (in water) (a), (d) are correct options
20. (a, d) Here
3 45 9 9 B Vn V n
NA = – = Vµ Þ 11= 2 2
4 16 4 16 n B1 B2
æ 6 ö If B1 = B2 and n1 = 2n2 Þ V2 = 2V1
For s1 ç in ns = ÷
è 15 ø and of B1 = 2B2 and n1 = n2 Þ V2 = 0.5V1
A and C are the correct options.
JEE MAIN 2015 PHYSICS
(Held on 4th April-2015)
1. Distance of the centre of mass of a solid uniform cone from 6. When 5V potential difference is applied across a wire of length
its vertex is z0. If the radius of its base is R and its height is h 0.1 m, the drift speed of electrons is 2.5 × 10–4 ms–1. If the
then z0 is equal to : electron density in the wire is 8 × 1028 m–3, the resistivity of
5h 3h 2 the material is close to :
(a) (b)
8 8R (a) 1.6 × 10–6 Wm (b) 1.6 × 10–5 Wm
2 3h
h (c) 1.6 × 10–8 Wm (d) 1.6 × 10–7 Wm
(c) (d)
4R 4
7. Two long current carrying thin wires, both with current I, are
2. A red LED emits light at 0.1 watt uniformly around it. The held by insulating threads of length L and are in equilibrium
amplitude of the electric field of the light at a distance of 1 m as shown in the figure, with threads making an angle 'q' with
from the diode is : the vertical. If wires have mass l per unit length then the
(a) 5.48 V/m (b) 7.75 V/m value of I is :
(c) 1.73 V/m (d) 2.45 V/m
3. A pendulum made of a uniform wire of cross sectional area (g = gravitational acceleration)
A has time period T. When an additional mass M is added to
its bob, the time period changes to TM. If the Young's modulus
1
of the material of the wire is Y then is equal to :
Y
(g = gravitational acceleration)

é æ T ö2 ù A é æ T ö2 ù A L
(b) ê1 - ç T ÷ ú Mg
ê M
(a) 1 - ç ÷ ú q
ë è T ø û Mg êë è M ø úû

éæ T ö2 ù A éæ T ö2 ù Mg
M
(c) êç ÷ - 1ú (d) êç M ÷ - 1ú
ëè T ø û Mg ëè T ø û A
I I
4. For a simple pendulum, a graph is plotted between its kinetic
energy (KE) and potential energy (PE) against its displacement d.
Which one of the following represents these correctly?
pgL plgL
(graphs are schematic and not drawn to scale) (a) 2 tan q (b) tan q
µ0 µ0
E
E KE PE plgL plgL
(c) sin q (d) 2sin q
µ0 cos q µ0 cos q
(a) d (b) KE 8. In the circuit shown, the current in the 1W resistor is :

PE 6V P 2W
E PE
E 1W 9V
KE

PE KE
(c) d
(d) d 3W W 3W
(a) 0.13 A, from Q to P (b) 0.13 A, from P to Q
5. A train is moving on a straight track with speed 20 ms–1. It is (c) 1.3A from P to Q (d) 0A
blowing its whistle at the frequency of 1000 Hz. The 9. Assuming human pupil to have a radius of 0.25 cm and a
percentage change in the frequency heard by a person comfortable viewing distance of 25 cm, the minimum
standing near the track as the train passes him is (speed of separation between two objects that human eye can resolve
sound = 320 ms–1) close to : at 500 nm wavelength is :
(a) 18% (b) 24% (a) 100 µm (b) 300 µm
(c) 6% (d) 12% (c) 1 µm (d) 30 µm
2 PHYSICS
10. An inductor (L = 0.03 H) and a resistor (R = 0.15 kW) are
Charge Charge
connected in series to a battery of 15V EMF in a circuit shown
below. The key K1 has been kept closed for a long time. Then Q2 Q2
at t = 0, K1 is opened and key K2 is closed simultaneously. At (a) (b)
t = l ms, the current in the circuit will be : ( e5 @ 150 ) 1µF 3µF
C
1µF 3µF
C

0.03 H 0.15 kW Charge Charge

Q2 Q2
K2
(c) (d)
C C
1µF 3µF 1µF 3µF
15V K1
13. From a solid sphere of mass M and radius R a cube of maximum
possible volume is cut. Moment of inertia of cube about an
(a) 6.7 mA (b) 0.67 mA axis passing through its center and perpendicular to one of
(c) 100 mA (d) 67 mA its faces is :
4MR 2 4MR 2
11. An LCR circuit is equivalent to a damped pendulum. In an (a) (b)
LCR circuit the capacitor is charged to Q0 and then connected 9 3p 3 3p
to the L and R as shown below : MR 2 MR 2
(c) (d)
32 2p 16 2p
R L
L
14. The period of oscillation of a simple pendulum is T = 2p .
g
Measured value of L is 20.0 cm known to 1 mm accuracy and
C time for 100 oscillations of the pendulum is found to be 90 s
using a wrist watch of 1s resolution. The accuracy in the
If a student plots graphs of the square of maximum charge determination of g is :
( Q2Max ) on the capacitor with time(t) for two different values (a) 1%
(c) 2%
(b) 5%
(d) 3%
L1 and L2 (L1 > L2) of L then which of the following represents 15. On a hot summer night, the refractive index of air is smallest
this graph correctly ? (plots are schematic and not drawn to near the ground and increases with height from the ground.
scale) When a light beam is directed horizontally, the Huygens'
principle leads us to conclude that as it travels, the light
beam :
L1
2 2 (a) bends downwards
QMax QMax Q0 (For both L1 and L2)
(a) L2 (b) (b) bends upwards
t t (c) becomes narrower
(d) goes horizontally without any deflection
16. A signal of 5 kHz frequency is amplitude modulated on a
carrier wave of frequency 2 MHz. The frequencies of the
L1 L2
2
Q
2 resultant signal is/are :
QMax
(c) L2 (d) Max L1 (a) 2005 kHz, 2000 kHz and 1995 kHz
t t (b) 2000 kHz and 1995 kHz
(c) 2 MHz only
12. In the given circuit, charge Q2 on the 2µF capacitor changes (d) 2005 kHz and 1995 kHz
as C is varied from 1µF to 3µF. Q2 as a function of 'C' is given 17. A solid body of constant heat capacity 1 J/°C is being heated
properly by: (figures are drawn schematically and are not by keeping it in contact with reservoirs in two ways :
to scale) (i) Sequentially keeping in contact with 2 reservoirs such
that each reservoir supplies same amount of heat.
(ii) Sequentially keeping in contact with 8 reservoirs such
1µF
that each reservoir supplies same amount of heat.
C In both the cases body is brought from initial temperature
2µF
100°C to final temperature 200°C. Entropy change of the body
in the two cases respectively is :
E (a) ln2, 2ln2 (b) 2ln2, 8ln2
(c) ln2, 4ln2 (d) ln2, ln2
JEE MAIN 2015 Solved Paper 3
18. Consider a spherical shell of radius R at temperature T. The 21. Monochromatic light is incident on a glass prism of angle A.
black body radiation inside it can be considered as an ideal gas If the refractive index of the material of the prism is µ, a ray,
U incident at an angle q, on the face AB would get transmitted
of photons with internal energy per unit volume u = µ T4
V through the face AC of the prism provided :
1æUö
and pressure p = ç ÷ . If the shell now undergoes an
3è V ø
adiabatic expansion the relation between T and R is : A
1 1
(a) T µ (b) T µ 3 q
R R
(c) T µ e –R (d) T µ e–3R
19. Two stones are thrown up simultaneously from the edge of a
B C
cliff 240 m high with initial speed of 10 m/s and 40 m/s
respectively. Which of the following graph best represents
é æ æ 1 öù
the time variation of relative position of the second stone (a) q > cos -1 êµsin ç A + sin -1 ç ÷ ú
with respect to the first ? êë è è µ ø úû
(Assume stones do not rebound after hitting the ground and
neglect air resistance, take g = 10 m/ s2) -1 é æ -1 æ 1 ö ù
(The figures are schematic and not drawn to scale) (b) q < cos êµsin ç A + sin ç ÷ ú
ëê è è µ ø ûú
(y2 – y1) m
240 -1 é æ -1 æ 1 ö ù
(c) q > sin êµsin ç A - sin ç ÷ ú
(a) ëê è è µ ø ûú

é æ æ 1 öù
t(s) (d) q < sin -1 êµsin ç A - sin -1 ç ÷ ú
8 12 êë è è µ ø úû
22. A rectangular loop of sides 10 cm and 5 cm carrying a current
(y2 – y1 ) m
240 1 of 12 A is placed in different orientations as shown in the
figures below :
(b)
z
I
t(s) B
8 12
I I
(A) y
(y2 – y1 ) m I
240 x
(c)
z

t(s) B
t® 8 12
(B) I
I y
(y2 – y1 ) m I
240 x I
(d) z

I B
t(s)
12 I
20. A uniformly charged solid sphere of radius R has potential (C) I y
V0 (measured with respect to ¥) on its surface. For this sphere I
x
3V0 5V0 3V0
the equipotential surfaces with potentials , ,
2 4 4 z
V0
and have radius R1, R2, R3 and R4 respectively. Then B
4
(a) R1 = 0 and R2 < (R4 – R3) I
(D) I y
(b) 2R < R4
I
(c) R1 = 0 and R2 > (R4 – R3) x I
(d) R1 ¹ 0 and (R2 – R1) > (R4 – R3)
4 PHYSICS
If there is a uniform magnetic field of 0.3 T in the positive z the blocks is 0.1 and between block B and the wall is 0.15, the
direction, in which orientations the loop would be in (i) stable frictional force applied by the wall on block B is:
equilibrium and (ii) unstable equilibrium ?
(a) (B) and (D), respectively
(b) (B) and (C), respectively
(c) (A) and (B), respectively F
A B
(d) (A) and (C), respectively
23. Two coaxial solenoids of different radius carry current I in the
uur
same direction. F1 be the magnetic force on the inner solenoid
uur
due to the outer one and F2 be the magnetic force on the
outer solenoid due to the inner one. Then :
uur uur
(a) F1 is radially inwards and F2 = 0
uur uur
(b) F1 is radially outwards and F2 = 0 (a) 120 N (b) 150 N
uur uur (c) 100 N (d) 80 N
(c) F1 = F2 = 0
28. A long cylindrical shell carries positive surface charge s in
uur uur
(d) F1 is radially inwards and F2 is radially outwards the upper half and negative surface charge - s in the lower
half. The electric field lines around the cylinder will look like
24. A particle of mass m moving in the x direction with speed 2v
figure given in : (figures are schematic and not drawn to
is hit by another particle of mass 2m moving in the y direction
scale)
with speed v. If the collision is perfectly inelastic, the
percentage loss in the energy during the collision is close to :
(a) 56% (b) 62% (a) (b)
(c) 44% (d) 50%
25. Consider an ideal gas confined in an isolated closed chamber.
As the gas undergoes an adiabatic expansion, the average
time of collision between molecules increases as Vq, where V (c) (d)

æ Cp ö
is the volume of the gas. The value of q is : ç g = ÷ 29. As an electron makes a transition from an excited state to the
è Cv ø ground state of a hydrogen - like atom/ion :
(a) kinetic energy decreases, potential energy increases but
g +1 g -1
(a) (b) total energy remains same
2 2
(b) kinetic energy and total energy decrease but potential
3g + 5 3g - 5 energy increases
(c) (d)
6 6 (c) its kinetic energy increases but potential energy and total
26. From a solid sphere of mass M and radius R, a spherical energy decrease
portion of radius R/2 is removed, as shown in the figure. (d) kinetic energy, potential energy and total energy
Taking gravitational potential V = 0 at r = ¥, the potential at decrease
the centre of the cavity thus formed is : 30. Match List - I (Fundamental Experiment) with List - II (its
(G = gravitational constant) conclusion) and select the correct option from the choices
given below the list:
Lis t-I Lis t-II
A . Fran ck-Hertz (i) Particle n atu re of
Exp eriment ligh t
B. Pho to -electric (ii) Dis crete en erg y
experimen t levels of atom
-2GM -2GM C. Davis on -Germer (iii) W av e natu re o f
(a) (b) experimen t electro n
3R R
(iv ) Structu re o f ato m
-GM -GM
(c) (d) (a) (A)-(ii); (B)-(i); (C)-(iii)
2R R
27. Given in the figure are two blocks A and B of weight 20 N and (b) (A)-(iv); (B)-(iii); (C)-(ii)
100 N, respectively. These are being pressed against a wall (c) (A)-(i); (B)-(iv); (C)-(iii)
by a force F as shown. If the coefficient of friction between (d) (A)-(ii); (B)-(iv); (C)-(iii)
JEE MAIN 2015 Solved Paper 5

SOLUTIONS
l
ycm =
ò ydm 6. (b) V = IR = (neAvd )r
1. (d) y a A
ò dm r h V
h \ r = V lne
2
ò pr dyr ´ y
3h
R d

Here V = potential difference


0
= =
1 2 4 l = length of wire
pR hr
3 n = no. of electrons per unit volume of conductor.
1 e = no. of electrons
2. (d) Using Uav = e0 E 2 Placing the value of above parameters we get resistivity
2
P 5
But U av = r=
2
4 pr ´ c 8 ´ 10 ´ 1.6 ´ 10 ´ 2.5 ´ 10-4 ´ 0.1
28 -19

P 1 = 1.6 × 10–5Wm
\ = e0 E 2 ´ c
4 pr 2
2 7. (d) Let us consider 'l' length of current carrying wire.
At equilibrium
2P 2 ´ 0.1 ´ 9 ´ 109
E 20 = = T cos q = lgl
4 pr 2 e 0 c 1 ´ 3 ´ 108
\ E0 = 6 = 2.45V/m q L
l T
3. (c) As we know, time period, T = 2p T cos q
g
When additional mass M is added then Lsin q Lsin q FB
l + Dl T sin q
TM = 2p (ll)g
g
m 0 I ´ Il é FB m0 2 I ´ I ù
l + Dl æ TM ö
2
l + Dl and T sin q = êQ =
TM = or ç ÷ = 2 p 2L sin q ë l 4p 2l sin q úû
l è T ø l
T
2 plgL
æ TM ö Mg é Mgl ù Therefore, I = 2 sin q
or, ç
è T ø÷ =1+ êQ Dl = Ay ú u 0 cos q
Ay ë û
8. (a) From KVL
1 éæ T ö
2 ù
A – 6 + 3I1 + 1 (Ii – I2) = 0
\ = êç M ÷ - 1ú
y ëêè T ø ûú Mg
6V P 2W
1
4. (d) K.E = k ( A2 - d 2 )
2
9V
1 2 1W
and P.E. = kd
2
At mean position d = 0. At extrement positions d = A 3W q 4W
é v ù 320
5. (d) f1 = f ê ú= f´ Hz 6 = 3 I 1 + I1 – I2
ë v - vsû 300
4I1 – I2 = 6 ...(1)
é v ù 320 – 9 + 2I2 – (I1 – I2) + 3I2 = 0
f2 = f ê ú= f ´ Hz – I1 + 6I2 = 9 ...(2)
ë v + vs û 340
On solving (1) and (2)
æ f2 ö æ 300 ö I1 = 0.13A
çè f - 1÷ø ´ 100 = çè 340 - 1÷ø ´ 100 ; 12% Direction Q to P, since I1 > I2.
1
6 PHYSICS
Alternatively 11. (c) From KVL at any time t
p
3 di
V R L
6V 9V 8 + dt –
1W 1W
i
3W 5W 15 W I
8

d
E1 E 2 6 9
+ - + –
r1 r2 3 5= 3 q c
Eq = 1 1 =
1 1 8V
+ + q di
r1 r2 3 5 - iR - L = 0
c dt
3 dq q dq Ld 2q
8 = 3 i=- Þ + R+ 2 =0
dt c dt dt
\ I = 15 23 = 0.13A
+1
8 d 2q R dq q
2
+ + =0
Considering potential at P as 0V and at Q as x volt, then dt L dt Lc
2W From damped harmonic oscillator, the amplitude is
6V O
dt
P given by A = Aoe -
2m
1W
9V
d2x b dx k
Double differential equation 2
+ + x=0
dt m dt m
Rt Rt
3W Q 3W Qmax = Q o e - Þ Q 2max = Qo2e -
2L L
x-6 x-0 x+9
+ + =0 Hence damping will be faster for lesser self inductance.
3 1 5
Q1 1mF
2 12. (d)
\ x= Q
23
x-0 2 C
\ i= = =0.13A
1 23 Q2
From Q to P 2mF
0.25 1
9. (d) sin q = =
25 100
2 2
From figure, Q2 = Q = Q
2 +1 3

0.25cm q æ C ´ 3ö
Q = E çè ÷
C + 3ø

2 æ 3CE ö 2CE
\ Q2 = ç ÷ =
25cm 3 èC +3ø C +3
1.22 l
Resolving power = 2 m sin q = 30 mm. Therefore graph d correctly dipicts.

15 ´ 100 Charge
10. (b) I(0) = = 0.1A
0.15 ´ 103
I (¥) = 0
–t
I (t) = [I (0) – I (¥)] e L / R + i( ¥ )
–t R
I(t) = 0.1 e L / R = 0.1 e L
0.15 ´1000 C
I(t) = 0.1 e 1mF 3mF
0.03 = 0.67mA
JEE MAIN 2015 Solved Paper 7
2 19. (b) y1 = 10t – 5t2 ; y2 = 40t – 5t2
13. (a) Here a = R
3 for y1 = – 240m, t = 8s
4 3 \ y2 – y1 = 30t for t < 8s.
pR
M for t > 8s,
Now, = 3 3
M¢ a 1
y2 – y1 = 240 – 40t – gt2
2
4 3
pR Kq
= 3 3
= p. a 20. (a,b) We know, V0 = = Vsurface
3 R
æ 2 ö 2
Kq
çè R ÷ø Now, Vi = (3R 2 – r 2 ) [For r < R]
3 2R 3

2M At the centre of sphare r = 0. Here


M¢ =
3p 3
V = V0
Moment of inertia of the cube about the given axis, 2
M¢ a 2 5 Kq Kq
I= Now, = (3R 2 – r 2 )
6 4 R 2R 3

2 R
2M æ 2 ö R2 =
´ç R÷ 2
3p è 3 ø 4MR 2
= =
6 9 3p 3 Kq Kq
= 3
4 R R
2 l
14. (d) As, g = 4 p 1 Kq Kq
T2 =
4 R R4
Dg Dl DT
So, ´ 100 = ´ 100 + 2 ´ 100 R4 = 4R
g l T
Also, R1 = 0 and R2 < (R4 – R3)
0.1 1 21. (c) When r2 = C, ÐN2Rc = 90°
= ´ 100 + 2 ´ ´ 100 = 2.72 ; 3%
20 90
Where C = critical angle
Plane WF 1
µ increases As sin C = = sin r2
15. (b) v
(Light bends Vel decreases A
upwards)
Refracted
WF N1

16. (a) Amplitude modulated wave consists of three q N2


frequencies are wc + wm, w,wc – wm Q r1
r2 R
i.e. 2005 KHz, 2000KHz, 1995 KHz
P
17. (d) The entropy change of the body in the two cases is
same as entropy is a state function.
B C
1æUö
18. (a) As, P = ç ÷
3èVø
Applying snell's law at ‘R’
U
But = KT 4 µ sin r2 = 1 sin90° ...(i)
V
Applying snell's law at ‘Q’
1 4
So, P = KT 1 × sin q = µ sin r 1 ...(ii)
3
But r1 = A – r2
uRT 1 So, sin q = µ sin (A – r 2)
or = KT 4 [As PV = u RT]
V 3 sin q = µ sin A cos r 2 – cos A ...(iii) [using (i)]
4 From (1)
pR 3T 3 = constant
3
1 2 1
Therefore, Tµ cos r2 = 1 – sin r2 = 1– ...(iv)
R µ2
8 PHYSICS
By eq. (iii) and (iv)
-GM æ 11R 2 ö GM
1 = ç ÷ = -11
sin q = µsin A 1 - - cos A 2R 3 è 4 ø 8R
µ2
on further solving we can show for ray not to transmitted Solid
through face AC sphere
é –1 æ 1 ö ù
q = sin–1 ê u sin(A – sin çè µ ÷ø ú
ë û
So, for transmission through face AC P
é –1 æ 1 ö ù
Cavity
q > sin –1 ê u sin(A – sin çè µ ÷ø ú
ë û
r r
22. (a) For stable equilibrium M || B Due to cavity part potential at point P
r r
For unstable equilibrium M || (–B) GM
r r 3 8 3GM
23. (c) F1 = F2 = 0 Vcavity = - =-
because of action and reaction pair 2 R 8R
2
Y
So potential at the centre of cavity
pf = 3 m V
11GM æ 3 GM ö -GM
= Vsphere - Vcavity = - - ç- =
m
2v pi 45° 8R è 8 R ÷ø R
24. (a) X
f1 f2

v
2m F N
27. (a) A B
Initial momentum of the system
pi = [m(2V)2 ´ m(2V)2 ] f1
20N 100N
= 2m ´ 2V
Assuming both the blocks are stationary
Final momentum of the system = 3mV
By the law of conservation of momentum N= F
f1 = 20 N
2 2mv = 3mV f2 = 100 + 20 = 120N
2 2v f
Þ = Vcombined
3
Loss in energy
1 1 1
DE = m1V12 + m2V22 - (m1 + m2 )Vcombined
2
2 2 2
4 5
DE = 3mv2 - mv 2 = mv 2 = 55.55%
3 3
Percentage loss in energy during the collision ; 56%
120N
1
25. (a) t = Considering the two blocks as one system and due to
æ N ö 3RT equilibrium f = 120N
2pd2 ç ÷
èVø M 28. (c) Field lines originate perpendicular from positive charge
V and terminate perpendicular at negative charge. Further
tµ this system can be treated as an electric dipole.
T
As, TVg–1 = K ze 2 k ze 2
29. (c) U = –K ; T.E = –
So, t µ Vg + 1/2 r 2 r
g +1
Therefore, q = k ze 2
2 K.E = . Here r decreases
26. (d) Due to complete solid sphere, potential at point P 2 r
30. (a) Frank-Hertz experiment - Discrete energy levels of atom
- GM é 2 æ R ö

Vsphere = Photoelectric effect - Particle nature of light
ê3R - ç ÷ ú
2R 3 êë è2ø úû Davison - Germer experiment - wave nature of electron.
JEE Advanced 2014 Solved Paper 2014-1

JEE ADVANCED 2014 PHYSICS


PAPER - 1
SECTION - I (Useful information: 167 RT = 640J1/ 2 mole -1/ 2 ;
This section contains 10 multiple choice questions. Each question
140 RT = 590J1/ 2 mole -1/ 2 . The molar masses M in grams
has 4 choices (a), (b), (c) and (d) out of which ONE or MORE
THAN ONE are correct. 10
are given in the options. Take the values of for each
1. Let E1 (r), E2(r) and E3(r) be the respective electric field at a M
distance r from a point charge Q, an infinitely long wire with gas as given there.)
constant linear charge density l, and an infinite plane with
uniform surface charge density s. If E1(r0) = E2(r0) = E3(r0) æ 10 7ö
(a) Neon ç M = 20, = ÷
at a given distance r0, then è 20 10 ø
(a) Q = 4spr02
æ 10 3 ö
Nitrogen ç M = 28, =
28 5 ÷ø
l (b)
(b) r0 = è
2 ps
(c) E1 ( r0 / 2) = 2 E2 ( r0 / 2) (c)
æ
Oxygen ç M = 32,
10 9ö
= ÷
è 32 16 ø
(d) E2 ( r0 / 2) = 4 E3 ( r0 / 2 )
2. Heater of an electric kettle is made of a wire of length L and æ 10 17 ö
Argon ç M = 36, =
36 32 ÷ø
diameter d. It takes 4 minutes to raise the temperature of (d)
0.5 kg water by 40 K. This heater is replaced by a new heater è
having two wires of the same material, each of length L and 5. In the figure, a ladder of mass m is shown leaning against a
diameter 2d. The way these wires are connected is given in wall. It is in static equilibrium making an angle q with the
the options. How much time in minutes will it take to raise the horizontal floor. The coefficient of friction between the wall
temperature of the same amount of water by 40 K? and the ladder is m1 and that between the floor and the ladder
(a) 4 if wires are in parallel (b) 2 if wires are in series is m2. The normal reaction of the wall on the ladder is N1 and
(c) 1 if wires are in series (d) 0.5 if wires are in parallel that of the floor is N2. If the ladder is about to slip, then
3. A transparent thin film of uniform thickness and refractive
index n1 = 1.4 is coated on the convex spherical surface of
m1
radius R at one end of a long solid glass cylinder of refractive
index n2 = 1.5, as shown in the figure. Rays of light parallel to
the axis of the cylinder traversing through the film from air to
glass get focused at distance f1 from the film, while rays of
light traversing from glass to air get focused at distance f2
from the film, Then
n1
m2

mg
Air n2 (a) m1 = 0, m 2 ¹ 0 and N2 tanq =
2
mg
(b) m1 ¹ 0, m2 = 0 and N1 tan q =
2
(a) f1 = 3R (b) f1 = 2.8R
mg
(c) f2 = 2 R (d) f2 = 1.4R (c) m1 ¹ 0, m2 ¹ 0 and N 2 = 1 + m m
1 2
4. A student is performing an experiment using a resonance
column and a tuning fork of frequency 244 s–1. He is told that mg
the air in the tube has been replaced by another gas (assume (d) m1 = 0, m 2 ¹ 0 and N1 tan q =
2
that the column remains filled with the gas). If the minimum
6. Two ideal batteries of emf V1 and V2 and three resistances
height at which resonance occurs is ( 0.350 ± 0.005 ) m, the R1, R2 and R3 are connected as shown in the figure. The
gas in the tube is current in resistance R2 would be zero if
2-2014 Physics
9. One end of a taut string of length 3 m along the x-axis is fixed
at x = 0. The speed of the waves in the string is 100 ms –1. The
other end of the string is vibrating in the y-direction so that
V1 R1 stationary waves are set up in the string. The possible
R2 waveform (s) of these stationary waves is(are)
px 50 pt
(a) y ( t ) = A sin cos
V2 6 3
R3 px 100pt
(b) y ( t ) = A sin cos
3 3
(a) V1 = V2 and R1 = R2 = R3 5px 250pt
(c) y ( t ) = A sin cos
(b) V1 = V2 and R1 = 2R2 = R3 6 3
(c) V1 = 2V2 and 2R1 = 2R2 = R3
5 px
(d) 2V1 = V2 and 2R1 = R2 = R3 (d) y ( t ) = A sin cos 250pt
7. A light source, which emits two wavelength l1 = 400 nm and 2
l2 = 600 nm, is used in a Young’s double slit experiment. If 10. A parallel plate capacitor has a dielectric slab of dielectric
recorded fringe widths for l1 and l2 are b1 and b2 and the constant K between its plates that covers 1/3 of the area of
number of fringes for them within a distance y on one side of its plates, as shown in the figure. The total capacitance of the
the central maximum are m1 and m2 respectively, then capacitor is C while that of the portion with dielectric in
(a) b2 > b1 between is C1. When the capacitor is charged, the plate area
covered by the dielectric gets charge Q1 and the rest of the
(b) m1 > m2
area gets charge Q2. The electric field in the dielectric is E1
(c) Form the central maximum, 3rd maximum of l2 overlaps and that in the other portion is E2. Choose the correct option/
with 5th minimum of l1 options, ignoring edge effects.
(d) The angular separation of fringes for l1 is greater
than l2.
8. At time t = 0, terminal A in the circuit shown in the figure is Q1 E1
connected to B by a key and an alternating current
I(t) = I0cos (wt), with I0 = 1 A and w = 500 rad s–1 starts
flowing in it with the initial direction shown in the figure. At
7p
t= , the key is switched from B to D. Now onwards only E2
6w Q2
A and D are connected. A total charge Q flows from the
battery to charge the capacitor fully. If C = 20 mF, R = 10 W
and the battery is ideal with emf of 50 V, identify the correct E1 E1 1
statement(s). (a) =1 (b) =
E2 E2 K
B D Q1 3 C 2+ K
(c) = (d) =
A Q2 K C1 K

50 V SECTION - II
C = 20 m F
This section contains 10 questions. Each question, when worked
out will result in one integer from 0 to 9 (both inclusive).
R = 10 W 11. A galvanometer gives full scale deflection with 0.006 A
(a) Magnitude of the maximum charge on the capacitor current. By connecting it to a 4990 W resistance, it can be
converted into a voltmeter of range 0 – 30 V. If connected to
7p
before t = is 1 × 10–3 C 2n
W resistance, it becomes an ammeter of range 0 – 1.5A.
6w a
249
(b) The current in the left part of the circuit just before The value of n is
7p 12. To find the distance d over which a signal can be seen clearly
t= is clockwise in foggy conditions, a railways-engineer uses dimensions
6w
and assumes that the distance depends on the mass density
(c) Immediately after A is connected to D, the current in R r of the fog, intensity (power/area) S of the light from the
is 10 A signal and its frequency f. The engineer finds that d is
(d) Q = 2 × 10–3 C proportional to S1/n. The value of n is
JEE Advanced 2014 Solved Paper 2014-3

13. During Searle’s experiment, zero of the Vernier scale lies 17. A thermodynamic system is taken from an initial state i with
between 3.20 × 10–2 m and 3.25 × 10–2 m of the main scale. internal energy Ui = 100 J to the final state f along two different
The 20th division of the Vernier scale exactly coincides with paths iaf and ibf, as schematically shown in the figure. The
one of the main scale divisions. When an additional load of work done by the system along the paths af, ib and bf are
2 kg is applied to the wire, the zero of the Vernier scale still Waf = 200 J, Wib = 50 J and Wbf =100 J respectively. The heat
lies between 3.20 × 10–2 m and 3.25 × 10–2 m of the main scale supplied to the system along the path iaf, ib and bf are Qiaf,
but now the 45th division of Vernier scale coincides with one Qib and Qbf respectively. If the internal energy of the system
of the main scale divisions. The length of the thin metallic
wire is 2 m and its cross-sectional area is 8 × 10–7 m2. The Qbf
least count of the Vernier scale is 1.0 × 10–5 m. The maximum in the state b is Ub = 200 J and Qiaf = 500 J, The ratio is
Qib
percentage error in the Young’s modulus of the wire is
14. A horizontal circular platform of radius 0.5 m and mass a f
0.45 kg is free to rotate about its axis. Two massless spring
P
toy-guns, each carrying a steel ball of mass 0.05 kg are
attached to the platform at a distance 0.25 m from the centre
on its either sides along its diameter (see figure). Each gun i
simultaneously fires the balls horizontally and perpendicular b
to the diameter in opposite directions. After leaving the V
platform, the balls have horizontal speed of 9 ms–1 with
18. Two parallel wires in the plane of the paper are distance X0
respect to the ground. The rotational speed of the platform in
apart. A point charge is moving with speed u between the
rad s–1 after the balls leave the platform is
wires in the same plane at a distance X1 from one of the wires.
When the wires carry current of magnitude I in the same
direction, the radius of curvature of the path of the point
charge is R1. In contrast, if the currents I in the two wires
have directions opposite to each other, the radius of curvature
15. A uniform circular disc of mass 1.5 kg and radius 0.5 m is X0 R
initially at rest on a horizontal frictionless surface. Three forces of the path is R2. If = 3 , the value of 1 is
X1 R2
of equal magnitude F = 0.5 N are applied simultaneously
along the three sides of an equilateral triangle XYZ with its 19. Airplanes A and B are flying with constant velocity in the
vertices on the perimeter of the disc (see figure). One second same vertical plane at angles 30° and 60° with respect to the
after applying the forces, the angular speed of the disc in rad horizontal respectively as shown in figure. The speed of A is
s–1 is F 100 3 m/s. At time t = 0 s, an observer in A finds B at a
X distance of 500 m. The observer sees B moving with a
constant velocity perpendicular to the line of motion of A. If
at t = t0, A just escapes being hit by B, t0 in seconds is
O A

Y F
Z
B
F
16. Consider an elliptical shaped rail PQ in the vertical plane with 30° 60°
OP = 3 m and OQ = 4 m. A block of mass 1 kg is pulled along
the rail from P to Q with a force of 18 N, which is always 20. A rocket is moving in a gravity free space with a constant
parallel to line PQ (see the figure given). Assuming no acceleration of 2 m/s2 along +x direction (see figure). The
frictionless losses, the kinetic energy of the block when it length of a chamber inside the rocket is 4 m. A ball is thrown
reaches Q is (n × 10) joules. The value of n is (take acceleration from the left end of the chamber in +x direction with a speed
due to gravity = 10 ms–2) of 0.3 m/s relative to the rocket. At the same time, another ball
Q is thrown in –x direction with a speed of 0.2 m/s from its right
end relative to the rocket. The time in seconds when the two
balls hit each other is
4m 2
0.3 m/s 0.2 m/s a = 2 m/s
x

90°
4m
O 3m P
4-2014 Physics

PAPER - 2
SECTION - I (a) always radially outwards
(b) always radially inwards
This section contains 10 multiple choice questions. Each question (c) radially outwards initially and radially inwards later
has four choices (a), (b), (c) and (d) out of which ONLY ONE (d) radially inwards initially and radially outwards later
option is correct.
1
6. A planet of radius R = ´ ( radius of Earth ) has the same
1. If lCu is the wavelength of Ka X-ray line of copper (atomic 10
number 29) and lMo is the wavelength of the Ka X-ray line of R
molybdenum (atomic number 42), then the ratio lCu/lMo is mass density as Earth. Scientists dig a well of depth
5
on it
close to and lower a wire of the same length and a linear mass density
(a) 1.99 (b) 2.14 10–3 kg m–1 into it. If the wire is not touching anywhere, the
(c) 0.50 (d) 0.48 force applied at the top of the wire by a person holding it in
2. A metal surface is illuminated by light of two different
place is (take the radius of Earth = 6 × 106 m and the
wavelengths 248 nm and 310 nm. The maximum speeds of the
acceleration due to gravity on Earth is 10 ms–2)
photoelectrons corresponding to these wavelengths are
(a) 96 N (b) 108 N
u1 and u2, respectively. If the ratio u1 : u2 = 2 : 1 and hc = 1240
(c) 120 N (d) 150 N
eV nm, the work function of the metal is nearly
7. A glass capillary tube is of the shape of a truncated cone
(a) 3.7 eV (b) 3.2 eV
with an apex angle a so that its two ends have cross sections
(c) 2.8 eV (d) 2.5 eV
of different radii. When dipped in water vertically, water rises
3. Parallel rays of light of intensity I = 912 Wm–2 are incident on
in it to a height h, where the radius of its cross section is b. If
a spherical black body kept in surroundings of temperature
300 K. Take Stefan-Boltzmann constant s = 5.7 × 10–8 the surface tension of water is S, its density is r, and its
Wm–2K–4 and assume that the energy exchange with the contact angle with glass is q, the value of h will be (g is the
surroundings is only through radiation. The final steady state acceleration due to gravity)
temperature of the black body is close to
(a) 330 K (b) 660 K
(c) 990 K (d) 1550 K
4. During an experiment with a metre bridge, the galvanometer
shows a null point when the jockey is pressed at 40.0 cm
using a standard resistance of 90 W, as shown in the figure.
h
The least count of the scale used in the metre bridge is 1mm.
The unknown resistance is

R 90 W

2S 2S
40.0 cm (a) cos ( q - a ) (b) cos ( q + a )
brg brg
(a) 60 ± 0.15 W (b) 135 ± 0.56 W
2S 2S
(c) 60 ± 0.25 W (d) 135 ± 0.23 W (c) cos ( q - a / 2) (d) cos ( q + a / 2)
brg brg
5. A wire, which passes through the hole in a small bead, is 8. Charges Q, 2Q and 4Q are uniformly distributed in three
bent in the form of quarter of a circle. The wire is fixed vertically dielectric solid spheres 1, 2 and 3 of radii R/2, R and 2R
on ground as shown in the figure. The bead is released from respectively, as shown in figure. If magnitude of the electric
near the top of the wire and it slides along the wire without fields at point P at a distance R from the centre of sphere 1, 2
friction. As the bead moves from A to B, the force it applies and 3 are E1, E2 and E3 respectively, then
on the wire is
A P
P
R R
2Q
Q

90° R/2
B Sphere 1 Sphere 2
JEE Advanced 2014 Solved Paper 2014-5

P
(c)
R
4Q t
K

2R
(d)
Sphere 3
t
(a) E1 > E2 > E3 (b) E3 > E1 > E2
(c) E2 > E1 > E3 (d) E3 > E2 > E1 SECTION - II
9. A point source S is placed at the bottom of a transparent This section contains 3 paragraphs, each describing theory,
block of height 10 mm and refractive index 2.72. It is immersed experiments, data etc. Six questions related to the three paragraphs
in a lower refractive index liquid as shown in the figure. It is with two questions on each paragraph. Each question has only
one correct answer among the four given options (a), (b), (c) and
found that the light emerging from the block to the liquid
(d).
forms a circular bright spot of diameter 11.54 mm on the top
of the block. The refractive index of the liquid is Paragraph for Questions 11 and 12
The figure shows a circular loop of radius a with two long parallel
wires (numbered 1 and 2) all in the plane of the paper. The distance
Liquid of each wire from the centre of the loop is d. The loop and the wire
are carrying the same current I.The current in the loop is in the
counterclockwise direction if seen from above.
Block Q S
d d

S
Wire 1 a Wire 2
(a) 1. 21 (b) 1. 30
(c) 1. 36 (d) 1. 42
P R
10. A tennis ball is dropped on a horizontal smooth surface. It
bounces back to its original position after hitting the surface. 11. When d » a but wires are not touching the loop, it is found
The force on the ball during the collision is proportional to that the net magnetic field on the axis of the loop is zero at a
the length of compression of the ball. Which one of the height h above the loop. In that case
following sketches describes the variation of its kinetic energy (a) current in wire 1 and wire 2 in the direction PQ and RS,
K with time t most appropriately? The figure are only respectively and h » a
illustrative and not to the scale. (b) current in wire 1 and wire 2 in the direction PQ and SR,
respectively and h » a
K
(c) current in wire 1 and wire 2 in the direction PQ and SR,
respectively and h » 1.2a
(d) current in wire 1 and wire 2 in the direction PQ and RS,
(a) respectively and h » 1.2a
12. Consider d >> a, and the loop is rotated about its diameter
t parallel to the wires by 30° from the position shown in the
figure. If the currents in the wires are in the opposite
directions, the torque on the loop at its new position will be
K
(assume that the net field due to the wires is constant over
the loop).
m0 I 2a2 m0 I 2a2
(b) (a) (b)
d 2d

t 3m 0 I 2 a 2 3m 0 I 2 a 2
(c) (d)
d 2d
6-2014 Physics
Paragraph for Questions 13 and 14 16. If the density of air is ra, and that of the liquid rl, then for a
In the figure, a container is shown to have a movable (without given piston speed the rate (volume per unit time) at which
friction) piston on top. The container and the piston are all made the liquid is sprayed will be proportional to
of perfectly insulated material allowing no heat transfer between
outside and inside the container. The container is divided into ra
(a) rl (b) ra rl
two compartments by a rigid partition made of a thermally
conducting material that allows slow transfer of heat. The lower
compartment of the container is filled with 2 moles of an ideal rl
monatomic gas at 700 K and the upper compartment is filled with (c) ra (d) rl
2 moles of an ideal diatomic gas at 400 K. The heat capacities per
3 5 SECTION - III
mole of an ideal monatomic gas are CV = R , CP = R , and
2 2 This section contains 4 questions, each having two matching
lists. Choices for the correct combination of elements from List-I
5 7
those for an ideal diatomic gas are CV = R , CP = R . and List-II are given as options (a), (b), (c) and (d), out of which
2 2 one is correct.
17. A person in lift is holding a water jar, which has a small hole
at the lower end of its side. When the lift is at rest, the water
jet coming out of the hole hits the floor of the lift at a distance
d of 1.2 m from the person. In the following, state of the lift’s
motion is given in List-I and the distance where the water jet
hits the floor of the lift is given in List-II. Match the statements
from List-I with those in List-IIand select the correct answer
using the code given below the lists.
List - I List - II
13. Consider the partition to be rigidly fixed so that it does not P. Lift is accelerating 1. d = 1.2 m
move. When equilibrium is achieved, the final temperature of vertically up
the gases will be Q. Lift is accelerating 2. d > 1.2 m
(a) 550 K (b) 525 K vertically down with
(c) 513 K (d) 490 K an acceleration less
14. Now consider the partition to be free to move without friction than the gravitational
so that the pressure of gases in both compartments is the acceleration
same. The total work done by the gases till the time they R. Lift is moving vertically 3. d < 1.2 m
achieve equilibrium will be up with constant speed
(a) 250R (b) 200 R S. Lift is falling freely 4. No water leaks out of the
(c) 100 R (d) – 100 R jar
Code:
Paragraph for Questions 15 and 16
(a) P-2, Q-3, R-2, S-4 (b) P-2, Q-3, R-1, S-4
A spray gun is shown in the figure where a piston pushes air out
(c) P-1, Q-1, R-1, S-4 (d) P-2, Q-3, R-1, S-1
of a nozzle. A thin tube of uniform cross section is connected to
18. Four charges Q1, Q2, Q3 and Q4 of same magnitude are fixed
the nozzle. The other end of the tube is in a small liquid container.
along the x axis at x = – 2a, – a, + a and + 2a, respectively. A
As the piston pushes air through the nozzle, the liquid from the
positive charge q is placed on the positive y axis at a distance
container rises into the nozzle and is sprayed out. For the spray
b > 0. Four options of the signs of these charges are given in
gun shown, the radii of the piston and the nozzle are 20 mm and
List-I. The direction of the forces on the charge q is given in
1 mm respectively. The upper end of the container is open to the
List-II. Match List-I with List-II and select the correct answer
atmosphere.
using the code given below the lists.

q(0, b)

15. If the piston is pushed at a speed of 5 mms–1, the air comes


out of the nozzle with a speed of Q1 Q2 Q3 Q4
(a) 0.1 ms–1 (b) 1 ms–1 (– 2a, 0) (– a, 0) (+ a, 0) (+ 2a, 0)
(c) 2 ms –1 (d) 8 ms–1
JEE Advanced 2014 Solved Paper 2014-7

List - I List - II
P. Q1, Q2, Q3, Q4 all 1. + x
positive
Q. Q1, Q2 positive; 2. – x S. 4. r
Q3, Q4 negative
R. Q1, Q4 positive; 3. + y
Code:
Q2, Q3 negative
(a) P-1, Q-2, R-3, S-4 (b) P-2, Q-4, R-3, S-1
S. Q1, Q3 positive; 4. – y
(c) P-4, Q-1, R-2, S-3 (d) P-2, Q-1, R-3, S-4
Q2, Q4 negative
20. A block of mass m1 = 1 kg another mass m2 = 2 kg, are placed
Code: together (see figure) on an inclined plane with angle of
(a) P-3, Q-1, R-4, S-2 (b) P-4, Q-2, R-3, S-1 inclination q. Various values of q are given in List-I. The
(c) P-3, Q-1, R-2, S-4 (d) P-4, Q-2, R-1, S-3 coefficient of friction between the block m1 and plane is
19. Four combinations of two thin lenses are given in List-I. The always zero. The coefficient of static and dynamic friction
radius of curvature of all curved surfaces is r and the refractive between the block m2 and the plane are equal to m = 0.3. In
index of all the lenses is 1.5. Match lens combinations in List-II expressions for the friction on block m2 are given.
List-I with their focal length in List-II and select the correct Match the correct expression of the friction in List-II with the
answer using the code given below the lists. angles given in List-I, and choose the correct option. The
List - I List - II acceleration due to gravity is denoted by g.
[Useful information: tan (5.5°) » 0.1; tan (11.5°) » 0.2;
tan (16.5°) » 0.3]

P. 1. 2r
m1
m2

q
r
Q. 2.
2
List-I List-II
P. q = 5° 1. m2g sinq
Q. q = 10° 2. (m1 + m2)g sinq
R. q = 15° 3. mm2g cosq
S. q = 20° 4. m(m1 + m2)g cosq
R. 3. –r
Code:
(a) P-1, Q-1, R-1, S-3 (b) P-2, Q-2, R-2, S-3
(c) P-2, Q-2, R-2, S-4 (d) P-2, Q-2, R-3, S-3
8-2014 Physics

SOLUTIONS
Paper - 1
When resistances are connected in series
r0
1. (c) 1 Q é 4rl ù R R
+Q E1 = Total resistance = R1 + R2 = 2 ê 2ú
= 2´ =
4pÎ0 r 2 ë 4pd û 4 2
0

+ r0 1 2l V2
+ E2 =
4pÎ0 r0 \ H= ´ t2 ...(ii)
+ R/2
From (i) and (ii) t2 = 2 min. Therefore (b) is correct.
r0 s
++ E3 = When resistance are connected in parallel
++ 2Î0
++ R1R 2 R2 R / 4 R
Total resistance = = 1 = =
R1 + R 2 2R1 2 8
E1 = E2 (Given)
V2
1 Q 1 2l \ H= ´ t2 ...(iii)
= R /8
4pÎ0 r0 4p Î0 r0
2
From (i) and (iii) t2 = 0.5 min
\ Q = 2lr0 ...(1) \ (d) is correct
E2 = E3 (Given) 3. (a, c)
There will be no effect of the transparent thin film of
1 2l s l
= Þ r0 = uniform thickness and refractive index n 1 = 1.4
4pÎ0 r0 2Î0 sp
n1 n 2 n 2 - n1
\ (b) is incorrect Therefore, - + = [For case (i)]
u v R
E1 = E3 (Given)
1 1.5 1.5 - 1
1 Q s Þ - + = Þ f1 = 3R
\ = Þ Q = 2psr02 ¥ f1 R
4pÎ0 r0 2Î0
2
\ (a) is a correct option
\ (a) is incorrect n1 n 2 n 2 - n1
Again applying - + = [For case (ii)]
1 4Q u v R
Now E1 (r0/2) =
4p Î0 r02 1.5 1 1 - 1.5
- + = Þ f 2 = 2R
4 ´ 2lr0 ¥ f2 -R
1 1 8l
= ´ = \ (c) is a correct option.
4p Î0 2
r0 4p Î0 r0
v gRT 1
é 1 4l ù 4. (d) Here, n = = ´ Þ v = n × 4l
1 8l 4l M ´ 10 -3 4l
and 2E2 (r0/2) = 2 ê ú=
ë 4 pÎ0 0 û
r 4 pÎ0 r0 Þ v = 336.7 m/s to 346.5 m/s
\ (c) is correct For monatomic gas g =1.67

1 2l 1 4l l gRT 10
E2 (r0/2) = = = v= = 100 gRT ´
4p Î0 r0 / 2 4p Î0 r0 pÎ0 r0 -3
M ´ 10 M

4s 2s 2 l 10 10
4E3 (r0/2) = = = ´ = 167RT ´ = 640
2 Î0 Î0 Î0 pr0 M M
\ (d) is incorrect. 10 7
For Neon M = 20 and =
2 20 10
V
2. (b, d) H = ´4 ...(i)
R 7
\ v = 640 ´ = 448 ms–1
4rl 10
where R =
pd2 \ (a) is incorrect
JEE Advanced 2014 Solved Paper 2014-9

10 17 O P
= 6. (a, b, d) i
For Argon M = 36,
36 32
V1 R1
17
\ v = 640 ´ = 340 ms–1 R2
32
\ (d) is the correct option. N Q
For diatomic gas g = 1.4
V2
10 10 i
v = 140RT = 590 ´ R3
M M
M A
10 9
For Oxygen = Applying KVL in MNOPQAM
32 16
V1 - iR1 + V2 - iR 3 = 0
9
\ v = 590 ´ = 331.87 ms–1 V1 + V2
16 \i= ... (i)
R1 + R 3
\ (c) is incorrect
Applying KVL in NOPQN
10 3
For Nitrogen = v1 - iR1 = 0 \ i =
V
...(ii)
28 5 R1
3 V1 V1 + V2
\ v = 590 ´ = 354 ms–1 From (i) & (ii) R = R + R
5
1 1 3
\ (b) is incorrect \ V1R1 + V1R3 = V1R1 + V2R1
5. (c, d) When m1 ¹ 0 and m2 ¹ 0 Þ V1R3 = V2R1
If V1 = V2 then R1 = R3 = R2\ (a) is correct option.
If V1 = V2 then R1 = R3 = 2R2\ (b) is correct option.
(R2 can have any value as there is no current flowing
N1 f1 = m1N1 through d)
If V1 = 2V2 then 2R3 = R1 \ (c) is incorrect option.
m1 If 2V1 = V2 then R3 = 2R1 = R2 \ (d) is correct option.
7. (a, b, c)
lD
N2 We know that b =
m2 mg d
q
As l2 > l1 \ b2 > b1 \ (a) is correct option.
P f2 = m2N2 Therefore m1 > m2 \ (b) is correct option.
l 2 D ( 2 ´ 5 - 1) l1 D
As 3 ´ =
N1 = m2N2 [... horizontal equilibrium] d 2 d
mg = N2 + m1N1 [... vertical equilibrium] 3 × 600 = 4.5 × 400 \ (c) is correct option.
Solving the above equation we get l
The angular width = \ (d) is incorrect option.
mg d
N2 =
1 + m1m2 8. (c, d) I = cos 500 t

\ (c) is the correct option. I


When m1 = 0
Taking torque about P we get 1
l 7p
mg ´ cos q = N1 ´ l sin q 6w
2
p p 2p
t
mg
w w
2w
N1 tan q =
2
\ (d) is correct –1
10-2014 Physics
B D
10. (a, d) C1
A
K
50 V A/3 Q1 E1
C = 20 m F

R = 10 W E2
2A/3 Q2
7p p
Till t = , the charge will be maximum at C2 = C – C1
6w 2w
d
p / 2w p / 2w
é sin 500t ù
Q¢ = ò cos 500t dt = ê
ë 500 û 0
ú This is a combination of two capacitors in parallel.
Therefore
0
C = C1 + C2 \ C2 = C – C1
1 æ p ö 1
= sin ç 500 ´ ÷= C kA 2A
500 è 2 ´ 500 ø 500 where C1 = and C - C1 =
3Î0 d 3Î0 d
\ (a) is incorrect
C - C1 2
7p \ =
From the graph it is clear that just before t = , the C1 k
6w
current is in anticlockwise direction. C 2
\ -1 =
\ (b) is incorrect C1 k
7p
At t = , the charge on the upper plate of capacitor C 2
6w \ = +1
C1 k
is
C 2
7p = +1
6w C1 k
1 æ 7p ö
ò cos 500t dt = 500 sin çè 500 ´ 6 ´ 500 ÷ø \ (d) is a correct option.
0
kA
Now, Q1 = C1V = ×V
=-
1 1
´ = -10 -3 C 3Î0 d
500 2
2A
Now applying KVL (when A is just connected to D) and Q2 = (C – C1)V = ´V
3Î0 d
10 -3
50 + - i ´ 10 = 0 Þ i = 10 A Q1 k
20 ´ 10 -6 \ = \ (c) is incorrect
Q2 2
\ (c) is the correct option.
The maximum charge on C is Q = CV = 20 × 10–6 × 50 Also V = E × d
=10–3C V
Therefore, the total charge flown = 2 × 10 –3 C \ E= = E1 = E2 \ (a) is a correct option
d
\ (d) is the correct option.
9. (a, c, d) æ I - Ig ö V
11. (5) ç ÷S= -R
Clearly in the given situation a displacement node is è g ø
I I g
present at x = 0 and a displacement antinode is present
at x = 3 m. Therefore, y = 0 at x = 0 and y = ± A at 1.5 - 0.006 2n 30
´ = - 4990
x = 3 m. 0.006 249 0.006
\ n»5
w
The velocity v = = 100 ms–1.
k 12. (3) d µ rx S y f z
a, c and d are the correct options which satisfy the
above conditions. M 0 L1T 0 = M x L-3x M yT - yT - z
JEE Advanced 2014 Solved Paper 2014-11

M 0 L1T 0 = M x + y L-3xT - y - z µ0 é 1 1 ù
\ x + y = 0, – 3x = 1 ´ 2I ê + ú
R1 4p ë X1 X 0 - X1 û
=
1 1 R2 µ0 é 1 1 ù
\ x=- and y = ´ 2I ê - ú
3 3 4p ë X1 X 0 - X1 û
\n=3
X 0 - X1 + X 1 X0
Y=
FL = X - X - X = X - 2X
13. (4) 0 1 1 0 1
a ´l
Here F, a and L are accurately known. X0
1.0 ´10-5
R1 X1 3
DY DL \ = = =3
´ 100 = ´ 100 = ´ 100 = 4% R2 X0 3- 2
Y l 25 ´10-5 -2
X1
14. (4) By conservation of angular momentum
2 (mvr) = Iw 19. (5) vA
1
2 × 0.05 × 9 × 0.25 = × 0.45 × (0.5)2 × w
2 vB cos 30°
vB
\ w = 4 rad s–1 vB sin 30°
30°
é 1ù 30° 60°
15. (2) 3 êF ´ r ´ ú = I a
ë 2û
Here
1 1 vA = vB cos 30°
3 ´ 0.5 ´ 0.5 ´ = ´ 1.5 ´ 0.5 ´ 0.5 ´ a
2 2
3
Þ a = 2 rad s–1 \ 100 3 = vB ´
2
w = w0 + at Þ w = 0 + 2 × 1 = 2 rad s–1
\ vB = 200 ms–1
16. (5) Work done = Increase in potential energy
+ gain in kinetic energy displacement
Time =
F × d = mgh + gain in K.E. velocity
18 × 5 = 1 × 10 × 4 + gain in K.E.
500 500
\ Gain in K.E. = 50 J = 10n \ t0 = = = 5 sec
vB sin 30° 200 ´ sin 30°
\n=5
17. (2) Applying first law of thermodynamics to path iaf 20. (8)
Qiaf = DUiaf + Wiaf 0.3 ms –1 0.2 ms –1 a = 2 ms –2
x
A
500 = DUiaf + 200 \ DUiaf = 300 J B
Now,
Qibf = DUibf + Wib + Wbf 4m
= 300 + 50 + 100 For ball A
Qib + Qbf = 450 J ...(1) u1 = 0.3 ms -1 , a1 = -2ms -2 , s1 = x, t1 = t
Also Qib = DU ib + Wib
1
\ Qib = 100 + 50 = 150 J ...(2) \ s1 = u1t1 + a1t12
2
Qbf x = 0.3t – t 2 ...(1)
300
From (1) & (2) = =2
Qib 150 For ball B
u2 = 0.2 ms–1, a2 = 2ms–2, s2 = 4 – x, t2 = t
mv
18. (3) R= \ s2 = u2t2 +
1
a 2 t 22
qB 2
4 – x = 0.2 t + t2 ...(2)
R1 B2
= [... m, q, v are the same] From (1) and (2) t = 8 sec
R2 B1
12-2014 Physics

Paper - 2
DR 0.1 0.1
lCu ( Z Mo - 1)
2 2 2
æ 42 - 1ö æ 41ö \ = +
1. (b) = =ç ÷ = çè ÷ø = 2.14 60 40 60
l Mo ( Z - 1) 2 è 29 - 1ø 28
Cu \ DR = 0.25W
Therefore, R = ( 60 ± 0.25 ) W
é ù
êQ n = ( Z - b ) here b = 1ú
ê ú 5. (d) h
ê n = ( Z - 1)
2 N
ú
ê 1 ú v
ê µ ( Z - 1) ú
2
ë l û q mg sinq
mg cosq mg
hC 1
2. (a) - W = mu12
l1 2 q
hC 1
and - W = mu22
l2 2
As the bead is moving in the circular path
Dividing the above two equations, we get
mv 2
hC \ mg cos q - N =
-W R
l1 u2
= 1 mv 2
hC 2
- W u2 \ N = mg cos q - ...(1)
l2 R
1 2
By energy conservation, mv = mg [ R - R cos q]
1240 2
-W
248 4
\ = v2
1240
-W
1 \ = 2 g (1 - cos q ) ..(2)
310 R
From (1) and (2)
1240 4 ´ 1240 N = mg cos q - m [ 2 g - 2 g cos q]
\ -W = - 4W
248 310
N = mg cos q - 2mg + 2mg cos q
\ W = 3.7 eV
3. (a) In steady state N = 3mg cos q - 2mg
Energy lost = Energy gained
Þ N = mg ( 3cos q - 2 )
(
s T 4
- T04 ) ´ 4pR 2
( )
= I pR 2
Clearly N is positive (acts radially outwards) when
2
\ 5.7 ´ 10 -8 êéT 4 - ( 300 ) úù ´ 4 = 912
4 cos q >
ë û 3
\ T = 330 K 2
Similarly, N acts radially inwards if cos q <
4. (c) In case of a meter bridge 3
R X
= Here X = 90 W, l = 40.0 cm 6. (b)
l 100 - l Rp
Rp
Xl 10 5
\ R= M
100 - l
Rp
For finding the value of R
90 ´ 40
R= = 60W
60
For finding the value of DR
DR Dl D (100 - l )
Re
Rp = = 6 ´ 105 m
= + 10
R l 100 - l
JEE Advanced 2014 Solved Paper 2014-13

The mass of the wire = 10–3 × 1.2 × 105 = 120 kg Clearly E2 > E1 > E3
Let gpM be the acceleration due to gravity at point M where Q/2 is the charge enclosed in a sphere of radius
which is the mid point of the wire and is at a depth of R concentric with the given sphere.
Rp é ù
. ê 4Q
10 Q' ú
ê4 = ú
Let gp be the acceleration due to gravity at the surface ê p ( 2 R )3 4 pR 3 ú
of the planet. ë3 3 û

4
gp = prGRP 9. (c)
3 n2
4 11.54 mm
ge = prGRE 2
3
P
gp 1 Rp q
\ = = 10 mm
ge RE 10
n1 q
10
\ gp = = 1 ms –2
10
Applying Snell’s law at point P
é R p /10 ù n1 sin q = n2sin 90°
ú = 1[1 - 0.1] = 0.9 ms
-2
and g pM = g p ê1 -
êë R p úû 11.54 / 2
\ n2 = 2.72 ´
2
\
(10 )2 + æç
Force = mass of wire × gpM = 120 × 0.9 = 108 N 11.54 ö
÷
è 2 ø
\ n2 = 1.36
7. (d) C 1 2 1 2 1
mv = m [ u + at ] = m [ 0 + gt ]
2
R a 10. (b) K.E. =
2 2 2
2
q+a q 1
M \ K .E = mgt 2 \ K .E µ t 2 ...(1)
b 2 P 2
First the kinetic energy will increase as per eq (1). As
the balls touches the ground it starts deforming and
loses its K.E. (K.E. converting into elastic potential
h energy). When the deformation is maximum, K.E. = 0.
The ball then again regain its shape when its elastic
potential energy changes into K.E. As the ball moves
up it loses K.E. and gain gravitational potential energy.
These characteristics are according to graph B.

11. (c) B
æ aö b
In DPCM cos ç q + ÷ = ...(1)
è 2ø R B2 sin q
B1 sin q
æ 2S ö
Also ç P0 - ÷ + hrg = P0 B2 q q B1
è R ø
(90º – q)
2S 2S
h= = cos ( q + a 2 ) B1cos q + B2 cos q
Rrg brg
a +h2 2
1 Q h
8. (c) E1 = · ;
4pÎ0 R 2
q
Wire 1 a a
1 2Q 1 Q/2 Wire 2
E2 = · ; E3 = ·
4pÎ0 R 2 4pÎ0 R 2
14-2014 Physics
Magnetic field due to current carrying loop = 15. (c) From principle of continuity,
Magnetic field due to straight wires a1v1 = a2v2
B = B1cos q + B2cos q = 2 B1cos q
pr12 v1 = pr22 v2
m0 Ia 2 é m0 I ù a
= 2ê ú´ ( 20 )2 ´ 5 = (1)2 ´ v2
(
2 a +h2
)
2 3/ 2
ëê 2p a + h
2 2
ûú a 2 + h2
\ v2= 2000 mms–1 = 2 ms–1
Þ h » 1.2a
16. (a) A B
The current is from P to Q in wire 1 and from R to S in
wire 2. C
12. (b) We know that torque
® ® ®
t = M´ B
t = MB sin q 1
PA - PB = ra va2
2
( é
)
m Iù
= I ´ pa 2 ´ ê2 ´ 0 ú sin 30°
ë 2 pd û 1
PC - PB = rl vl2
2
m0 I 2a2 But PC = PA
=
2d
1 1 ra
13. (d) Heat lost by monatomic gas at constant volume
\ rl vl2 = ra va2 Þ vl = ´ va
= Heat gained by diatomic gas at constant pressure 2 2 rl

\ nCv1 ( 700 - T ) = nC p2 (T - 400)


ra
\ Volume flow rate µ
3 7 rl
R ( 700 - T ) = R (T - 400)
2 2
17. (c)
Þ 2100 - 3T = 7T - 2800
Þ 10T = 4900
\ T = 490 K h2
14. (d) In this case both the gases are at constant pressure.
v = 2gh2
\ nC p1 ( 700 - T ) = nC p2 (T - 400 )

5 7 h1
R ( 700 - T ) = R (T - 400 )
2 2
3500 – 5T = 7T – 2800
2gh2 × 2h1
Þ 12T = 6300 g
\ T = 525 K
= 2 h1h2
Applying first law of thermodynamics
DW1 + DU1 = DQ1 If geff > g
geff = g
and DW2 + DU 2 = DQ2 geff < g
As the gas two system is thermally insulated, therefore
In all the three cases d = 2 h1h2 = 1.2 m
DQ1 + DQ2 = 0
If geff = 0, then no water leaks out
- ( DW1 + DW2 ) = DU1 + DU 2 18. (a) If Q1, Q2, Q3 and Q4 are all positive, then the force will
be along + y-direction.
= nCv1 ( 525 - 700 ) + n2 Cv2 ( 525 - 400 )
If Q1, Q2 are positive and Q3, Q4 are negative the force
3R 5R will act along + x-direction.
= -2 ´ ´ 175 + 2 ´ ´125 If Q1, Q4 are positive and Q2, Q3 are negative then
2 2
attractive force will dominate repulsive force and the
= –525R + 625R = – 100 R
force will be along – y direction.
Therefore, total work done = –100 R
JEE Advanced 2014 Solved Paper 2014-15

20. (d) If (m1 + m2) sin q < mN2 the bodies will be at rest
1 é1 1 ù
19. (b) For (P) = ( m - 1) ê - ú i.e., (m1 + m2)g sin q < mm2 g cos q
f ë 1
R R2û

é2ù 1
= (1.5 - 1) ê ú = Þ f = r f = mN2
ër û r N2

For the combination


q
1 1 1 1 1 2 (m1 + m2)g cos q
= + = + = (m1 + m2) sin q
F f1 f 2 r r r (m1 + m2)g
q
r
\ F=
2
mm2 g
tan q <
For (Q)
1 é1
= ( m - 1) ê - ú
1 ù ( m1 + m2 ) g
f ë R1 R2 û
mm2
Þ tan q <
é 1 1 ù 0.5 1 m1 + m2
= (1.5 - 1) ê - ú = =
ë ¥ -r û r 2r
0.3 ´ 2
\ f = 2r Þ tan q <
1+ 2
For the combination
Þ tan q < 0.2
1 1 1 1 1 2 1 i.e., If the angle q < 11.5° the frictional force is less than
= + = + = =
F f1 f 2 2r 2r 2r r mN2 = mm2g = 0.3 × 2 × g = 0.6 g
\F=r and is equal to (m1 + m2)g sin q
Similarly, we can either find or do not find the remaining At q = 11.5° the bodies are on the verge of moving,
options (b) is the correct option. f = 0.6 g
At q > 11.5° the bodies start moving and f = 0.6 g
The above relationship is true for (d).
JEE MAIN 2014 PHYSICS
(Held on 6th April-2014)
1. The current voltage relation of a diode is given by 5. When a rubber-band is stretched by a distance x, it exerts
I = (e1000V T
- 1) mA, where the applied voltage V is in volts restoring force of magnitude F = ax + bx 2 where a and b are
constants. The work done in stretching the unstretched
and the temperature T is in degree kelvin. If a student makes
rubber-band by L is:
an error measuring ±0.01 V while measuring the current of
5 mA at 300 K, what will be the error in the value of current in
mA? (a) aL2 + bL3 (b)
1
2
(
aL2 + bL3 )
(a) 0.2 mA (b) 0.02 mA
(c) 0.5 mA (d) 0.05 mA
aL2 bL3 1 æ aL2 bL3 ö
2. From a tower of height H, a particle is thrown vertically (c) + (d) 2 çç 2 + 3 ÷÷
upwards with a speed u. The time taken by the particle, to hit 2 3 è ø
the ground, is n times that taken by it to reach the highest
6. A bob of mass m attached to an inextensible string of length
point of its path. The relation between H, u and n is:
l is suspended from a vertical support. The bob rotates in a
(a) 2gH = n 2 u 2 (b) gH = ( n - 2 ) u 2
2
horizontal circle with an angular speed w rad/s about the vertical.
About the point of suspension:
(c) 2gH = nu 2 ( n - 2 ) (d) gH = ( n - 2 ) u 2
(a) angular momentum is conserved.
3. A mass ‘m’ is supported by a massless string wound around
(b) angular momentum changes in magnitude but not in
a uniform hollow cylinder of mass m and radius R. If the
string does not slip on the cylinder, with what acceleration direction.
will the mass fall or release? (c) angular momentum changes in direction but not in
magnitude.
(d) angular momentum changes both in direction and
magnitude.
7. Four particles, each of mass M and equidistant from each
other, move along a circle of radius R under the action of
R their mutual gravitational attraction. The speed of each particle
m is:

GM GM
(a) (b) 2 2
R R
m
(c)
GM
R
(
1+ 2 2 ) (d)
1 GM
2 R
(
1+ 2 2 )
2g g
(a) (b) 8. The pressure that has to be applied to the ends of a steel wire
3 2
of length 10 cm to keep its length constant when its
5g temperature is raised by 100ºC is:
(c) (d) g
6
(For steel Young’s modulus is 2 ´ 1011 Nm -2 and coefficient
4. A block of mass m is placed on a surface with a vertical cross of thermal expansion is 1.1 ´ 10-5 K -1 )
x3 (a) 2.2 ´ 108 Pa (b) 2.2 ´ 10 9 Pa
section given by y = . If the coefficient of friction is 0.5,
6
the maximum height above the ground at which the block can (c) 2.2 ´ 10 7 Pa (d) 2.2 ´ 10 6 Pa
be placed without slipping is: 9. There is a circular tube in a vertical plane. Two liquids which
1 2 do not mix and of densities d1 and d2 are filled in the tube.
(a) m (b) m Each liquid subtends 90º angle at centre. Radius joining their
6 3
1 1 d1
(c) m (d) m interface makes an angle a with vertical. Ratio is:
3 2 d2
2 PHYSICS
12. One mole of a diatomic ideal gas undergoes a cyclic process
ABC as shown in figure. The process BC is adiabatic. The
temperatures at A, B and C are 400 K, 800 K and 600 K
respectively. Choose the correct statement:

a d2 B
800 K

P
d1
600 k
1 + sin a 1 + cos a A C
(a) (b) 400 K
1 - sin a 1 - cos a
V
1 + tan a 1 + sin a
(c) (d) (a) The change in internal energy in whole cyclic process is
1 - tan a 1 - cos a 250 R.
10. On heating water, bubbles being formed at the bottom of the (b) The change in internal energy in the process CA is 700 R.
vessel detach and rise. Take the bubbles to be spheres of (c) The change in internal energy in the process AB is -350 R.
radius R and making a circular contact of radius r with the (d) The change in internal energy in the process BC is
bottom of the vessel. If r << R and the surface tension of -500 R.
water is T, value of r just before bubbles detach is: 13. An open glass tube is immersed in mercury in such a way
that a length of 8 cm extends above the mercury level. The
(density of water is r w ) open end of the tube is then closed and sealed and the tube
is raised vertically up by additional 46 cm. What will be length
of the air column above mercury in the tube now?
(Atmospheric pressure = 76 cm of Hg)
(a) 16 cm (b) 22 cm
(c) 38 cm (d) 6 cm
14. A particle moves with simple harmonic motion in a straight
line. In first t s, after starting from rest it travels a distance a,
and in next t s it travels 2a, in same direction, then:
(a) amplitude of motion is 3a
R (b) time period of oscillations is 8t
(c) amplitude of motion is 4a
(d) time period of oscillations is 6t
2r 15. A pipe of length 85 cm is closed from one end. Find the number
of possible natural oscillations of air column in the pipe whose
frequencies lie below 1250 Hz. The velocity of sound in air is
2 rw g 2 rw g 340 m/s.
(a) R (b) R
3T 6T (a) 12 (b) 8
(c) 6 (d) 4
rw g 3rw g r
(c) R
2
(d) R
2 16. Assume that an electric field E = 30x 2 ˆi exists in space. Then
T T
the potential difference VA - VO , where VO is the potential
11. Three rods of Copper, Brass and Steel are welded together to
form a Y shaped structure. Area of cross - section of each rod at the origin and VA the potential at x = 2 m is:
(a) 120 J/C (b) -120 J/C
= 4cm 2 . End of copper rod is maintained at 100°C where as
(c) -80 J/C (d) 80 J/C
ends of brass and steel are kept at 0°C. Lengths of the copper,, 17. A parallel plate capacitor is made of two circular plates
brass and steel rods are 46, 13 and 12 cms respectively. The separated by a distance 5 mm and with a dielectric of dielectric
rods are thermally insulated from surroundings excepts at constant 2.2 between them. When the electric field in the
ends. Thermal conductivities of copper, brass and steel are dielectric is 3 ´ 104 V m the charge density of the positive
0.92, 0.26 and 0.12 CGS units respectively. Rate of heat flow
plate will be close to:
through copper rod is:
(a) 1.2 cal/s (b) 2.4 cal/s (a) 6 ´ 10 -7 C m 2 (b) 3 ´ 10 -7 C m 2
(c) 4.8 cal/s (d) 6.0 cal/s (c) 3 ´ 10 4 C m 2 (d) 6 ´ 10 4 C m 2
JEE MAIN 2014 Solved Paper 3
18. In a large building, there are 15 bulbs of 40 W, 5 bulbs of 22. During the propagation of electromagnetic waves in a medium:
100 W, 5 fans of 80 W and 1 heater of 1 kW. The voltage of (a) Electric energy density is double of the magnetic energy
electric mains is 220 V. The minimum capacity of the main fuse density.
of the building will be: (b) Electric energy density is half of the magnetic energy
(a) 8 A (b) 10 A density.
(c) 12 A (d) 14 A (c) Electric energy density is equal to the magnetic energy
19. A conductor lies along the z-axis at -1.5 £ z < 1.5 m and density.
carries a fixed current of 10.0 A in -â z direction (see figure). (d) Both electric and magnetic energy densities are zero.
r
For a field B = 3.0 ´10-4 e -0.2x aˆ y T, find the power required æ 3ö
23. A thin convex lens made from crown glass ç m = ÷ has focal
to move the conductor at constant speed to x = 2.0 m, y = 0 m è 2ø
in 5 ´10-3 s. Assume parallel motion along the x-axis. length f. When it is measured in two different liquids having
4 5
z refractive indices and , it has the focal lengths f1 and f2
3 3
1.5 respectively. The correct relation between the focal lengths
I is:
(a) f1 = f2 < f
(b) f1 > f and f2 becomes negative
B y (c) f2 > f and f1 becomes negative
2.0 (d) f1 and f2 both become negative
24. A green light is incident from the water to the air - water
x
interface at the critical angle (q). Select the correct statement.
–1.5 (a) The entire spectrum of visible light will come out of the
water at an angle of 90º to the normal.
(a) 1.57 W (b) 2.97 W (b) The spectrum of visible light whose frequency is less
than that of green light will come out to the air medium.
(c) 14.85 W (d) 29.7 W
(c) The spectrum of visible light whose frequency is more
20. The coercivity of a small magnet where the ferromagnet gets than that of green light will come out to the air medium.
demagnetized is 3 ´ 103 Am-1. The current required to be (d) The entire spectrum of visible light will come out of the
passed in a solenoid of length 10 cm and number of turns 100, water at various angles to the normal.
so that the magnet gets demagnetized when inside the
25. Two beams, A and B, of plane polarized light with mutually
solenoid, is: perpendicular planes of polarization are seen through a
(a) 30 mA (b) 60 mA polaroid. From the position when the beam A has maximum
(c) 3 A (d) 6 A intensity (and beam B has zero intensity), a rotation of polaroid
21. In the circuit shown here, the point ‘C’ is kept connected to through 30° makes the two beams appear equally bright. If
point ‘A’ till the current flowing through the circuit becomes the initial intensities of the two beams are IA and IB
constant. Afterward, suddenly, point ‘C’ is disconnected from
point ‘A’ and connected to point ‘B’ at time t = 0. Ratio of the IA
respectively, then equals:
voltage across resistance and the inductor at t = L/R will be IB
equal to:
3
A C R (a) 3 (b)
2

1
L (c) 1 (d)
3
26. The radiation corresponding to 3 ® 2 transition of hydrogen
B
atom falls on a metal surface to produce photoelectrons. These
electrons are made to enter a magnetic field of 3 × 10–4 T. If
e the radius of the largest circular path followed by these
(a) (b) 1 electrons is 10.0 mm, the work function of the metal is close
1- e
to:
1- e (a) 1.8 eV (b) 1.1 eV
(c) –1 (d)
e (c) 0.8 eV (d) 1.6 eV
4 PHYSICS

27. Hydrogen ( 1 H1 ) , Deuterium ( 1 H 2 ) , singly ionised Helium


29. Match List - I (Electromagnetic wave type) with List - II (Its
association/application) and select the correct option from

( He4 ) and doubly ionised lithium ( Li 6 )


+ ++ the choices given below the lists:
2 3 all have one
List 1 List 2
electron around the nucleus. Consider an electron transition 1. Infrared waves (i) To treat muscular
from n = 2 to n = 1. If the wavelengths of emitted radiation are strain
l1 , l 2 , l3 and l4 respectively then approximately which 2. Radio waves (ii) For broadcasting
one of the following is correct? 3. X-rays (iii) To detect fracture of
bones
(a) 4l1 = 2l 2 = 2l3 = l 4 (b) l1 = 2l 2 = 2l3 = l 4
4. Ultraviolet rays (iv) Absorbed by the ozone
(c) l1 = l 2 = 4l3 = 9l 4 (d) l1 = 2l 2 = 3l3 = 4l 4 layer of the atmosphere
28. The forward biased diode connection is: 1 2 3 4
(a) (iv) (iii) (ii) (i)
+2V –2V
(a) (b) (i) (ii) (iv) (iii)
(c) (iii) (ii) (i) (iv)
–3V –3V
(b) (d) (i) (ii) (iii) (iv)
30. A student measured the length of a rod and wrote it as 3.50
2V 4V cm. Which instrument did he use to measure it?
(c)
(a) A meter scale.
(d)
–2V +2V (b) A vernier calliper where the 10 divisions in vernier scale
matches with 9 division in main scale and main scale has
10 divisions in 1 cm.
(c) A screw gauge having 100 divisions in the circular scale
and pitch as 1 mm.
(d) A screw gauge having 50 divisions in the circular scale
and pitch as 1 mm.
JEE MAIN 2014 Solved Paper 5

SOLUTIONS
1. (a) The current voltage relation of diode is
I = (e1000 V /T - 1) mA (given) m
y
q
When, I = 5mA, e1000 V /T = 6mA

1000
Also, dI = (e1000 V /T

T
(By exponential function)
1000 Q Coefficient of friction m = 0.5
= (6 mA) ´ ´ (0.01)
300
x2
= 0.2 mA \ 0.5 =
u 2
2. (c) Speed on reaching ground v = u 2 + 2 gh Þ x=+1
Now, v = u + at H x3 1
Now, y = = m
Þ u 2 + 2 gh = -u + gt 6 6
5. (c) Work done in stretching the rubber-band by a distance
u dx is
Time taken to reach highest point is t = ,
g dW = F dx = (ax + bx2)dx
Integrating both sides,
u + u 2 + 2 gH nu L L
Þ t= = (from question) aL2 bL3
g g W = ò axdx + ò bx 2 dx = +
2 3
Þ 2gH = n(n –2)u2 0 0
3. (b) 6. (c) Torque working on th e bob of mass m is,
t = mg ´ l sin q . (Direction parallel to plane of rotation
of particle)
a
R
q
l l
T
m mg
T
ur
As t is perpendicular to L , direction of L changes
m a
mg but magnitude remains same.
Mv 2
From figure, 7. (d) 2 F cos 45° + F ' = (From figure)
R
Acceleration a = Ra …(i)
and mg – T = ma …(ii) GM 2 GM 2
From equation (i) and (ii) Where F = and F ' =
( 2 R) 2 4R 2
æ aö
T × R = mR2a = mR2 çè ÷ø
R
F
or T = ma M M
F'
Þ mg – ma = ma
F
g R
Þ a=
2 o
4. (a) At limiting equilibrium, m = tanq
dy x 2
tanq = m = = (from question) M
dx 2 M
6 PHYSICS

2 ´ GM 2 GM 2 Mv 2 2T 4p R 3
Þ + = (p r 2 ) = rw g
2( R 2)2 4 R2 R R 3T

GM 2 é 1 1 ù 2 R 4 rw g
Þ ê + ú = Mv 2 Þ r2 =
R ë4 2û 3T

2rw g
Gm æ 2 + 4 ö 1 Gm Þ r = R2
\ v= ç ÷= (1 + 2 2) 3T
R çè 4 2 ÷ø 2 R
11. (c) Rate of heat flow is given by,
stress KA(q1 - q 2 )
8. (a) Young's modulus Y = Q=
strain l
stress = Y ´ strain
Where, K = coefficient of thermal conductivity
Stress in steel wire = Applied pressure l = length of rod and A = Area of cross-section of rod
Pressure = stress = Y × strain
100°C
DL
Strain = = α D T (As length is constant)
L Copper
= 2 × 1011 × 1.1 × 10–5 × 100 T
= 2.2 × 108 Pa B Brass
Steel
9. (c) Pressure at interface A must be same from both the 0°C 0°C
sides to be in equilibrium.
If the junction temperature is T, then
QCopper = QBrass + QSteel
0.92 ´ 4(100 - T ) 0.26 ´ 4 ´ (T - 0)
= +
R d2 46 13
Rsina
a
0.12 ´ 4 ´ (T - 0)
a Rcosa R
12
Rsin a – Rsin a
A Þ 200 – 2T = 2T + T
d1 Þ T = 40°C

\ ( R cos a + R sin a )d2 g = ( R cos a - R sin a )d1 g 0.92 ´ 4 ´ 60


\ QCopper = = 4.8 cal/s
46
d1 cos a + sin a 1 + tan a 12. (d) In cyclic process, change in total internal energy is
Þ = =
d 2 cos a - sin a 1 - tan a zero.
DUcyclic = 0
10. None of the given option is correct.
5R
When the bubble gets detached, DUBC = nCv DT = 1 ´ DT
Buoyant force = force due to surface tension 2
Where, Cv = molar specific heat at constant volume.
For BC, DT = –200 K
\ DUBC = –500R

13. (a) (54–x)


R P
q 54 cm
8 cm x
rq
T×dl

Force due to excess pressure = upthrust Hg


2T
Access pressure in air bubble =
R
JEE MAIN 2014 Solved Paper 7
Length of the air column above mercury in the tube is, 17. (a) Electric field in presence of dielectric between the two
P + x = P0 plates of a parallel plate capacitor is given by,
Þ P = (76 – x)
s
Þ 8 × A × 76 = (76 – x) × A × (54 – x) E=
K e0
\ x = 38
Thus, length of air column = 54 – 38 = 16 cm. Then, charge density
14. (d) In simple harmonic motion, starting from rest, s = K e0 E
At t = 0 , x = A
= 2.2 × 8.85 × 10–12 × 3 × 104 » 6 × 10–7 C/m2
x = Acoswt ..... (i)
When t = t , x = A – a 18. (c) Total power consumed by electrical appliances in the
building, Ptotal = 2500W
When t = 2 t , x = A –3a
Watt = Volt × ampere
From equation (i)
A – a = Acosw t ......(ii) Þ 2500 = V × I
A – 3a = A cos2w t ......(iii) Þ 2500 = 220 I
As cos2w t = 2 cos2 w t – 1 …(iv) 2500
From equation (ii), (iii) and (iv) Þ I= = 11.36 » 12A
220
2
A - 3a æ A-a ö (Minimum capacity of main fuse)
= 2ç ÷ -1 19. (b) Work done in moving the conductor is,
A è A ø
2
A - 3a 2 A2 + 2a 2 - 4 Aa - A2 W = ò Fdx
0
Þ =
A A2 2
Þ A2 – 3aA = A2 + 2a2 – 4Aa = ò 3.0 ´ 10 -4 e -0.2 x ´ 10 ´ 3dx
0
Þ 2a2 = aA
Þ A = 2a
a 1
Þ = l=3m
A 2 I = 10 A
Now, A – a = A coswt z
A- a
Þ cos wt = x
A
1
Þ cos wt = -3 2
-0.2 x
2 = 9 ´ 10 ò e dx
0
2p p
or, t=
T 3 9 ´ 10-3
Þ T= 6t = [-e -0.2 ´ 2 + 1] B = 3.0 ´ 10-4 e-0.2 x
0.2
15. (c) Length of pipe = 85 cm = 0.85m
(By exponential function)
Frequency of oscillations of air column in closed organ
pipe is given by, 9 ´ 10-3
= ´ [1 - e -0.4 ]
(2n - 1)u 0.2
f =
4L
(2n - 1)u = 9 × 10–3 × (0.33) = 2.97 × 10–3J
f = £ 1250 Power required to move the conductor is,
4L
(2n - 1) ´ 340 W
Þ £ 1250 P=
0.85 ´ 4 t
Þ 2n – 1 < 12.5 » 6 2.97 ´ 10-3
P= = 2.97 W
(0.2) ´ 5 ´ 10-3
16. (c) Potential difference between any two points in an
electric field is given by,
ur uur 20. (c) Magnetic field in solenoid B = m0n i
dV = – E.dx
VA 2 B
= ni
ò dV = - ò 30 x 2 dx Þ m0
VO 0
(Where n = number of turns per unit length)
VA - VO = -[10 x3 ]20 = -80 J / C
8 PHYSICS
Now, IA' = I A cos230º
B Ni
Þ =
m0 L I B ' = I B cos 2 60º
100i
Þ 3 ´ 103 = As I A ' = I B '
10 ´ 10-2
Þ i = 3A 3 1
21. (c) Applying Kirchhoff's law of voltage in closed loop Þ IA ´ = IB ´
4 4
V
–VR –VC = 0 Þ R = -1
VC IA 1
=
VR IB 3
A C R
26. (b) Radius of circular path followed by electron is given
by,
L VL mu 2meV 1 2m
r= = = V
B qB eB B e

B2 r 2 e
Þ V= = 0.8V
1 2m
22. (c) E0 = CB0 and C = m e For transition between 3 to 2.
0 0
æ 1 1 ö 13.6 ´ 5
1 E = 13.6 ç - ÷ = = 1.88eV
Electric energy density = e 0 E02 = m E è 4 9ø 36
2
Work function = 1.88 eV – 0.8 eV
2
1 Bo = 1.08 eV » 1.1eV
Magnetic energy density = = mB
2 m0
1 é 1 1 ù
Thus, mE = mB 27. (c) Wave number = RZ 2 ê - ú
Energy is equally divided between electric and magnetic l êë n12 n 2 úû
field
23. (b) By Lens maker's formula for convex lens 1
Þ lµ
1 æ m öæ 2 ö Z2
=ç - 1÷ ç ÷
f è mL øè R ø
By question n = 1 and n 1 = 2
Then, l1 = l2 = 4l3 = 9l4
4
for, m L1 = , f1 = 4 R 28. (a) P n
3
For forward bias, p-side must be at higher potential
5
for m L 2 = , f 2 = -5 R than n-side. DV = (+ )Ve
3
Þ f2 = (–) ve 29. (d)
24. (b) For critical angle qc, (1) Infrared rays are used to treat muscular strain because
these are heat rays.
1
sin qc = (2) Radio waves are used for broadcasting because these
m
waves have very long wavelength ranging from few
centimeters to few hundred kilometers
air (3) X-rays are used to detect fracture of bones because
they have high penetrating power but they can't
q water penetrate through denser medium like bones.
(4) Ultraviolet rays are absorbed by ozone of the
For greater wavelength or lesser frequency m is less. atmosphere.
So, critical angle would be more. So, they will not suffer 30. (b) Measured length of rod = 3.50 cm
reflection and come out at angles less then 90°. For vernier scale with 1 Main Scale Division = 1 mm
25. (d) According to malus law, intensity of emerging beam is 9 Main Scale Division = 10 Vernier Scale Division,
given by, Least count = 1 MSD –1 VSD
I = I0cos2q = 0.1 mm
1 Units & Measurements

FILL IN THE BLANKS : 6. Which of the following set have different dimensions?
1. Planck’s constant has dimension ________. (a) Pressure, Young’s modulus , Stress (2005S)
(1985 - 2 Marks) (b) EMF, Potential difference, Electric potential
2. In the formula X = 3YZ 2, X and Z have dimensions of (c) Heat, Work done, Energy
capacitance and magnetic induction respectively. The (d) Dipole moment, Electric flux, Electric field
dimensions of Y in MKSQ sytem are _________, ________.
7. In a screw gauge, the zero of mainscale coincides with fifth
(1988 - 2 Marks)
division of circular scale in figure (i). The circular division of
3. The dimension of electrical conductivity is _________.
(1997C - 1 Mark) screw gauge are 50. It moves 0.5 mm on main scale in one
4. The equation of state for real gas is given by rotation. The diameter of the ball in figure (ii) is
(2006 - 3M, –1)
æ a ö
çè P + 2 ÷ø (V - b) = RT . The dimensions of the constant
V 0 10
a is _________. (1997 - 2 Marks) 5
0

MCQ's WITH ONE CORRECT ANSWER :

The dimension of æç ö÷ e 0 E 2 ( e 0 : permittivity of free space,


1
1.
è 2ø
Figure (i)
E electric field) (2000S)
(a) MLT–1 (b) ML2T–2
(c) ML–1T–2 (d) ML2T–1 0 30
25
DV
2. A quantity X is given by e 0 L where Î0 is the 20
Dt
permittivity of the free space, L is a length, DV is a potential
difference and Dt is a time interval. The dimensional formula
for X is the same as that of (2001S)
(a) resistance (b) charge Figure (ii)
(c) voltage (d) current
3. A cube has a side of length 1.2 × 10–2m. Calculate its volume. (a) 2.25 mm (b) 2.20 mm
(a) 1.7 × 10–6 m3 (b) 1.73 × 10–6 m3 (2003S) (c) 1.20 mm (d) 1.25 mm
(c) 1.70 × 10 m–6 3 (d) 1.732 × 10–6 m3 8. A student performs an experiment for determination of
a æ az ö æ 4p 2 l ö
4. Pressure depends on distance as, P = exp ç - ÷ , where
b è kq ø gç= ÷ . The error in length l is Dl and in time T is DT
è T2 ø
a, b are constants, z is distance, k is Boltzman’s constant
and q is temperature. The dimension of b are (2004S) and n is number of times the reading is taken. The
(a) M0L0T0 (b) M–1L–1T –1 measurement of g is most accurate for (2006 - 3M, –1)
(c) M0L2T0 (d) M–1L1T 2 Dl DT n
5. A wire of length l = 6 ± 0.06 cm and radius r = 0.5 ± 0.005 cm (a) 5 mm 0.2 sec 10
and mass m = 0.3 ± 0.003 gm. Maximum percentage error in
(b) 5mm 0.2 sec 20
density is (2004S)
(c) 5 mm 0.1 sec. 10
(a) 4 (b) 2
(c) 1 (d) 6.8 (d) 1mm 0.1 sec 50
P-2 Topic-wise Solved Papers - PHYSICS
9. A student performs an experiment to determine the Young's (a) due to the errors in the measurements of d
modulus of a wire, exactly 2 m long, by Searle's method. In a and l are the same.
particular reading, the student measures the extension in (b) due to the error in the measurement of d is twice that
the length of the wire to be 0.8 mm with an uncertainty due to the error in the measurement of l .
of ± 0.05 mm at a load of exactly 1.0 kg. The student also (c) due to the error in the measurement of l is twice that
measures the diameter of the wire to be 0.4 mm with an due to the error in the measurement of d.
uncertainty of ± 0.01 mm. Take g = 9.8 m/s2 (exact). The
(d) due to the error in the measurement of d is four times
Young's modulus obtained from the reading is (2007)
that due to the error in the measurement of l.
(a) (2.0 ± 0.3) × 1011 N/m2 (b) (2.0 ± 0.2) × 1011 N/m2
14. The diameter of a cylinder is measured using a Vernier
(c) (2.0 ± 0.1) × 1011 N/m2 (d) (2.0 ± 0.05) × 1011 N/m2
callipers with no zero error. It is found that the zero of the
10. Students I, II and III perform an experiment for measuring
Vernier scale lies between 5.10 cm and 5.15 cm of the main
the acceleration due to gravity (g) using a simple pendulum.
They use different lengths of the pendulum and /or record scale. The Vernier scale has 50 divisions equivalent to 2.45
time for different number of oscillations. The observations cm. The 24th division of the Vernier scale exactly coincides
are shown in the table. (2008) with one of the main scale divisions. The diameter of the
Least count for length = 0.1 cm cylinder is (JEE Adv. 2013)
Least count for time = 0.1 s (a) 5.112 cm (b) 5.124 cm
(c) 5.136 cm (d) 5.148 cm
Student Length of the No. of Total time Time
pendulum oscillations for (n) period
(cm) (n) oscillations (s) 1. The dimensions of the quantities in one (or more) of the
(s) following pairs are the same. Identify the pair (s)
I 64.0 8 128.0 16.0 (a) Torque and Work (1986 - 2 Marks)
II 64.0 4 64.0 16.0
(b) Angular momentum and Work
III 20.0 4 36.0 9.0
(c) Energy and Young’s modulus
If EI, EII and EIII are the percentage errors in g, i.e., (d) Light year and Wavelength
æ Dg ö 2. The pairs of physical quantities that have the same
çè g ´ 100÷ø for students I, II and III, respectively, then dimensions is (are) : (1995S)
(a) Reynolds number and coefficient of friction
(a) EI = 0 (b) EI is minimum (b) Curie and frequency of a light wave
(c) EI = EII (d) EII is maximum
(c) Latent heat and gravitational potential
11. A vernier calipers has 1 mm marks on the main scale. It has 20
(d) Planck’s constant and torque
equal divisions on the Vernier scale which match with 16 main
scale divisions. For this Vernier calipers, the least count is 3. The SI unit of inductance, the henry can be written as
(a) 0.02 mm (b) 0.05 mm (2010) (1998 - 2 Marks)
(c) 0.1 mm (d) 0.2 mm (a) weber/ampere (b) volt-sec/amp
12. The density of a solid ball is to be determined in an (c) Joule/(ampere)2 (d) ohm-second
experiment. The diameter of the ball is measured with a screw
4. Let [e 0 ] denote the dimensional formula of the permittivity
gauge, whose pitch is 0.5 mm and there are 50 divisions on
the circular scale. The reading on the main scale is of the vacuum, and [m 0 ] that of the permeability of the
2.5 mm and that on the circular scale is 20 divisions. If the vacuum. If M = mass, L = length, T = time and I = electric
measured mass of the ball has a relative error of 2 %, the current, (1998 - 2 Marks)
relative percentage error in the density is (2011)
(a) 0.9 % (b) 2.4 % (a) [e 0 ] = M -1 L-3 T 2 Ι (b) [e 0 ] = M -1 L-3 T 4 Ι 2
(c) 3.1 % (d) 4.2 %
(c) [m 0 ] = M LT -2 Ι -2 (d) [m 0 ] = M L2T -1 Ι
æ 4 MLg ö
13. In the determination of Young’s modulus ç Y = ÷ by 5. A student uses a simple pendulum of exactly 1m length to
è pld 2 ø determine g, the acceleration due to gravity. He uses a stop
using Searle’s method, a wire of length L = 2 m and diameter watch with the least count of 1 sec for this and records 40
d = 0.5 mm is used. For a load M = 2.5 kg, an extension l = seconds for 20 oscillations. For this observation, which of
0.25 mm in the length of the wire is observed. Quantities d the following statement(s) is (are) true? (2010)
and l are measured using a screw gauge and a micrometer, (a) Error DT in measuring T, the time period, is 0.05 seconds
respectively. They have the same pitch of 0.5 mm. The (b) Error DT in measuring T, the time period, is 1 second
number of divisions on their circular scale is 100. The
(c) Percentage error in the determination of g is 5%
contributions to the maximum probable error of the Y
measurement (2012) (d) Percentage error in the determination of g is 2.5%
UNITS & MEASUREMENTS P-3
6. Using the expression 2d sin q = l, one calculates the values 5. Column-I gives three physical quantities. Select the
of d by measuring the corresponding angles q in the range 0 appropriate units for the choices given in Column-II. Some
to 90°. The wavelength l is exactly known and the error in q of the physical quantities may have more than one choice
is constant for all values of q. As q increases from 0° correct : (1990 - 3 Marks)
(JEE Adv. 2013) Column I Column II
(a) The absolute error in d remains constant Capacitance (i) ohm-second
(b) The absolute error in d increases Inductance (ii) coulomb2–Joule–1
(c) The fractional error in d remains constant Magnetic Induction (iii) coulomb (volt)–1
(d) The fractional error in d decreases (iv) Newton (amp-metre)–1
(v) volt-second (ampere)–1
SUBJECTIVE PROBLEMS :
6. If nth division of main scale coincides with (n+1)th divisions
1. Give the MKS units for each of the following quantities. of vernier scale. Given one main scale division is equal to ‘a’
(i) Young’s modulus (1980) units. Find the least count of the vernier. (2003 - 2 Marks)
(ii) Magnetic Induction 7. A screw gauge having 100 equal divisions and a pitch of
length 1 mm is used to measure the diameter of a wire of
(iii) Power of a lens
length 5.6 cm. The main scale reading is 1 mm and 47th circular
2. A gas bubble, from an explosion under water, oscillates with
division coincides with the main scale. Find the curved
a period T proportional to p a d b E c . Where ‘P’ is the static
surface area of wire in cm2 to appropriate significant figure.
pressure, ‘d’ is the density of water and ‘E’ is the total energy
of the explosion. Find the values of a, b and c. 22
(use p = ). (2004 - 2 Marks)
(1981- 3 Marks) 7
3. Write the dimensions of the following in terms of mass, 8. In Searle’s experiment, which is used to find Young’s
time, length and charge (1982 - 2 Marks) Modulus of elasticity, the diameter of experimental wire is D
(i) magnetic flux (ii) rigidity modulus = 0.05 cm (measured by a scale of least count 0.001 cm) and
4. Match the physical quantities given in column I with length is L = 110 cm (measured by a scale of least count 0.1
dimensions expressed in terms of mass (M), length (L), time cm). A weight of 50 N causes an extension of X = 0.125 cm
(T), and charge (Q) given in column II and write the correct (measured by a micrometer of least count 0.001cm). Find
answer against the matched quantity in a tabular form in maximum possible error in the values of Young’s modulus.
your answer book. (1983 - 6 Marks) Screw gauge and meter scale are free from error.
Column I Column II (2004 - 2 Marks)
Angular momentum ML2T–2 9. The side of a cube is measured by vernier callipers (10
Latent heat ML2Q–2 divisions of a vernier scale coincide with 9 divisions of main
scale, where 1 division of main scale is 1 mm). The main
Torque ML2T–1
scale reads 10 mm and first division of vernier scale coincides
Capacitance ML3T–1Q–2
with the main scale. Mass of the cube is 2.736 g. Find the
Inductance M –1 L –2 T 2 Q2 density of the cube in appropriate significant figures.
Resistivity L2T –2 (2005 - 2 Marks)

MATCH THE FOLLOWING :


MUTLIPLE CHOICE QUESTIONS WITH ONE CORRECT
DIRECTIONS (Q. No. 1) : Each question contains statements given in two columns, which have to be matched. The statements in
Column-I are labelled A, B, C and D, while the statements in Column-II are labelled p, q, r and s. Any given statement in Column-
I can have correct matching with ONE OR MORE statement(s) in Column-II. The appropriate bubbles corresponding to the
answers to these questions have to be darkened as illustrated in the following example :
If the correct matches are A-p, s and t; B-q and r; C-p and q; and D-s then the correct darkening of bubbles will look like the given.

p q r s t
A p q r s t
B p q r s t
C p q r s t
D p q r s t
P-4 Topic-wise Solved Papers - PHYSICS
1. Some physical quantities are given in Column I and some possible SI units in which these quantities may be expressed are given
in Column II. Match the physical quantities in Column I with the units in Column II and indicate your answer by darkening
appropriate bubbles in the 4 × 4 matrix given in the ORS. (2007)
Column I Column II
(A) GMeMs (p) (volt) (coulomb)(metre)
G – universal gravitational constant,
Me – mass of the earth, Ms – mass of the Sun
3RT
(B) (q) (kilogram) (metre)3(second)–2
M
R – universal gas constant,
T – absolute temperature, M – molar mass

F2
(C) (r) (metre)2 (second)–2
q2 B 2
F – Force, q – charge, B – magnetic field
GM e
(D) (s) (farad) (volt)2 (kg)–1
Re
G – universal gravitational constant,
Me – mass of the earth, Re – radius of the earth

DIRECTIONS (Q. No. 2) : Following question has matching lists. The codes for the lists have choices (a), (b), (c) and (d) out of which
ONLY ONE is correct.
2. Match List I with List II and select the correct answer using the codes given below the lists: (JEE Adv. 2013)
List I List II
P. Boltzmann constant 1. [ML2T-1]
Q. Coefficient of viscosity 2. [ML–1T–1]
R. Planck constant 3. [MLT–3K–1]
S. Thermal conductivity 4. [ML2T–2K–1]
Codes:
P Q R S
(a) 3 1 2 4
(b) 3 2 1 4
(c) 4 2 1 3
(d) 4 1 2 3

1. Taking the electronic charge as ‘e’ and the permittivity as


COMPREHENSION BASED Q UESTIONS :
‘e0’. Use dimensional analysis to determine the correct
expression for wp.
A dense collection of equal number of electrons and positive ions
is called neutral plasma. Certain solids containing fixed positive
Ne me0
ions surrounded by free electrons can be treated as neutral plasma. (a) (b)
me 0 Ne
Let ‘N’ be the number density of free electrons, each of mass ‘m’.
When the electrons are subjected to an electric field, they are
displaced relatively away from the heavy positive ions. If the Ne 2 Ne 2
electric field becomes zero, the electrons begin to oscillate about (c) (d)
me0 me0
the positive ions with a natural angular frequency ‘wp’ which is
called the plasma frequency. To sustain the oscillations, a time 2. Estimate the wavelength at which plasma reflection will occur
varying electric field needs to be applied that has an angular for a metal having the density of electrons
frequency w, where a part of the energy is absorbed and a part of N » 4 ´ 1027 m–3. Taking e0 = 10–11 and mass m » 10–30,
it is reflected. As w approaches wp all the free electrons are set to where these quantities are in proper SI units.
resonance together and all the energy is reflected. This is the (a) 800 nm (b) 600 nm
explanation of high reflectivity of metals. (2011)
(c) 300 nm (d) 200 nm
UNITS & MEASUREMENTS P-5

1. Identify the pair whose dimensions are equal [2002] 9. In an experiment the angles are required to be measured
(a) torque and work (b) stress and energy using an instrument, 29 divisions of the main scale exactly
(c) force and stress (d) force and work coincide with the 30 divisions of the vernier scale. If the
smallest division of the main scale is half- a degree (= 0.5°),
1 then the least count of the instrument is : [2009]
2. Dimension of , where symbols have their usual
m0 e0 (a) half minute (b) one degree
meaning, are [2003] (c) half degree (d) one minute
10. The respective number of significant figures for the numbers
(a) [L-1T] (b) [L-2 T 2 ]
23.023, 0.0003 and 2.1 × 10–3 are [2010]
(c) [L2 T -2 ] (d) [LT -1 ] (a) 5, 1, 2 (b) 5, 1, 5
(c) 5, 5, 2 (d) 4, 4, 2
3. The physical quantities not having same dimensions are
11. A screw gauge gives the following reading when used to
(a) torque and work [2003]
measure the diameter of a wire.
(b) momentum and planck’s constant
Main scale reading : 0 mm
(c) stress and young’s modulus
Circular scale reading : 52 divisions
(d) speed and (m o e o ) -1 / 2 Given that 1mm on main scale corresponds to 100 divisions
4. Which one of the following represents the correct of the circular scale. The diameter of wire from the above
dimensions of the coefficient of viscosity? [2004] data is: [2011]
(a) 0.052 cm (b) 0.026 cm
(a) ML-1T -1 (b) MLT -1
(c) 0.005 cm (d) 0.52 cm
(c) ML-1T -2 (d) ML-2 T -2 12. Resistance of a given wire is obtained by measuring the
5. Out of the following pair , which one does NOT have identical current flowing in it and the voltage difference applied across
dimensions is [2005] it. If the percentage errors in the measurement of the current
(a) impulse and momentum and the voltage difference are 3% each, then error in the
(b) angular momentum and planck’s constant value of resistance of the wire is : [2012]
(c) work and torque (a) 6% (b) zero
(d) moment of inertia and moment of a force (c) 1% (d) 3%
(towards north-west) 13. A spectrometer gives the following reading when used to
6. The dimension of magnetic field in M, L, T and C (coulomb) measure the angle of a prism. [2012]
is given as [2008] Main scale reading : 58.5 degree
(a) MLT C –1 –1 (b) MT C2 –2 Vernier scale reading : 09 divisions
(c) MT–1 C–1 (d) MT–2 C–1 Given that 1 division on main scale corresponds to 0.5 degree.
7. A body of mass m = 3.513 kg is moving along the x-axis with Total divisions on the Vernier scale is 30 and match with 29
a speed of 5.00 ms–1. The magnitude of its momentum is divisions of the main scale. The angle of the prism from the
recorded as [2008] above data :
(a) 17.6 kg ms –1 (b) 17.565 kg ms –1 (a) 58.59 degree (b) 58.77 degree
(c) 17.56 kg ms–1 (d) 17.57 kg ms–1 (c) 58.65 degree (d) 59 degree
8. Two full turns of the circular scale of a screw gauge cover a 14. Let [ Î0 ] denote the dimensional formula of the permittivity of
distance of 1mm on its main scale. The total number of
vacuum. If M = mass, L = length, T = time and A = electric
divisions on the circular scale is 50. Further, it is found that
current, then: [JEE Main 2013]
the screw gauge has a zero error of – 0.03 mm. While
measuring the diameter of a thin wire, a student notes the (a) Î0 = [M–1 L–3 T2 A]
main scale reading of 3 mm and the number of circular scale
divisions in line with the main scale as 35. The diameter of (b) Î0 = [M1 L3 T5 A2]
the wire is [2008] (c) Î0 = [M1 L2 T1 A2]
(a) 3.32 mm (b) 3.73 mm
(c) 3.67 mm (d) 3.38 mm (d) Î0 = [M1 L2 T1 A]
P-6 Topic-wise Solved Papers - PHYSICS

Solutions & Explanations


Section-A : JEE Advanced/ IIT-JEE
A 1. ML 2 T – 1 2. M –3 L –2 T 4 Q 4 3. M – 1 L – 3 T 3 A 2 4. ML 5 T – 2
C 1. (c) 2. (d) 3. (a) 4. (c) 5. (a) 6. (d)
7. (c) 8. (d) 9. (b) 10. (b) 11. (d) 12. (c)
13. (a) 14. (b)
D 1. (a, d) 2. (a, b, c) 3. (a, b, c, d) 4. (b, c) 5. (a, c) 6. (d)
E 1. (i) N/m2; (ii) Tesla; (iii) Dioptre; 2. a = – 5/6, b = 1/2, c = 1/3 3. (i) [M1L2T–1Q–1] (ii) [ML–1T–2]
4. Angular Momentum – [ML2T–1]
Latent heat – [L2T –2]
Torque – [ML2T–2]
Capacitance – [M–1L–2T2Q2]
Inductance – [ML2Q–2]
Resistivity – [ML3T–1Q–2]
5. Capacitance coulomb-volt–1, coulomb2-joule–1
Inductance ohm-sec, volt-second (ampere)–1
Magnetic Induction newton (ampere-metre)–1
1
Q (a) q = CV; U = CV 2
2
(b) Refer to solution of Q. 3, type D
(c) F = I l B
a
6. units 7. 2.6 cm2 8. 1.09 ´ 1010 Nm -2 9. 2.66 gm/cm3
n +1
F 1. (A) ® p, q ; (B) ® r, s ; (C) ® r, s ; (D) ® r,, s 2. (c)
G 1. (c) 2. (b)

Section-B : JEE Main/ AIEEE


1. (a) 2. (c) 3. (b) 4. (a) 5. (d) 6. (c)
7. (a) 8. (d) 9. (d) 10. (a) 11. (a) 12. (a)
13. (c) 14. (b)

FILL IN THE BLANKS : 1 l l Il Il I lq t


3. s= = = = = = ´
r RA V VA W WA t
A A
I q
é 2 -2 ù
E ë ML T û é 2 -1 ù é A2 LT 1 ù
1. E = hv h = v = é -1 ù = ë ML T û I 2l t ë û
T =
= WA é 2 -2 2 ù = [M –1 L –3 T 3 A 2 ]
ë û
ML T L
ë û
2. [X] = [C] = [M –1 L–2 T 2 Q2] 4. In the equation
[Z] = [B] = [MT –1 Q –1 ] æ a ö é a ù
çè P + 2 ÷ø (V - b) = RT , [P] = ê 2 ú
V ëV û
[M -1L-2T 2 Q 2 ]
\ [Y] = = [M -3 L-2T 4 Q 4 ] MLT -2
-1 -1 2
[ MT Q ] or [a] = [PV 2 ] = L6 = ML5 T –2
L2
UNITS & MEASUREMENTS P-7

MCQ's WITH ONE CORRECT ANSWER : m


5. (a) r =
l p r2
(c) Note : Here æç ö÷ e 0 E 2 represents energy per unit
1
1.
è 2ø Dr Dm 2Dr Dl
volume. = + + NOTE THIS STEP
r m r l
[e 0 ] [ E 2 ] =
[ Energy ] = ML2T -2 = ML-1T -2 Putting the values
[ Volume] L3 Dl = 0.06 cm, l = 6cm; Dr = 0.005 cm; r = 0.5 cm,
2. (d) Dimensionally e0 L = C m = 0.3 gm; Dm = 0.003 gm
where C = capacitance Dr 4
we get =
Now the given expression reduces to C DV Dt . r 100
Dimensionally C DV = q
Dr
where q is charge. \ ´ 100 = 4%.
r
Again the given expression reduces to q Dt . ur ur
q 6. (d) KEY CONCEPT : Electric flux f E = E . S
Dimensionally =I
Dt \ Dimensionally f E ¹ E
where I is current
0.5
3. (a) V = l3 = (1.2 × 10–2 m)3 = 1.728 × 10–6 m3 7. (c) Least count = = 0.01mm
Þ V = 1.7 × 10–6 m3. 50
Zero error = 5 × L.C = 5 × 0.01 mm = 0.05 mm
Note : l has two significant figures. Hence V will also Diameter of ball = [Reading on main scale] + [Reading
have two significant figures. on circular scale × L . C] – Zero error
4. (c) Unit of k is joules per kelvin or dimensional formula of = 0.5 × 2 + 25 × 0.01 – 0.05
k is [ML2T–2 q -1 ] = 1.20 mm
Note : The power of an exponent is a number. Dg Dl DT
az 8. (d) = +2
Therefore, dimensionally = M ° L °T ° g l T
kq
kq Dl and DT are least and number of readings are
\ a= × a dimension less quantity maximum in option (d), therefore the measurement of g
z is most accurate with data used in this option.
\ Dimensional formula of
mg L
[ML2T -2 q -1 ][q] é 9. (b) We know that Y = ´
a= = MLT -2 ù D 2 l
[ L] ë û p
Also, dimensional formula of P = [ML–1 T–2] 4
a a 4mgL 4 ´ 1 ´ 9.8 ´ 2
and dimensionally P = Þb= Þ Y= =
b P
( ) ´ (0.8 ´ 10 )
2 2
pD l p 0.4 ´ 10 -3 -3
-2
MLT
\ [b] = = M 0 L2T 0
ML-1T -2 = 2.0 ´ 1011 N/m 2
ALTERNATE SOLUTIONS
az Now DY = 2DD + Dl
= M 0 L0T 0 Y D l
kq [Q the value of m, g and L are exact]
kq
\ a= × a dimension less quantity 0.01 0.05
z = 2´ + = 2 × 0.025 + 0.0625
a 0.4 0.8
Also, dimensionally P = = 0.05 + 0.0625 = 0.1125
b Þ DY = 2 × 1011 × 0.1125 = 0.225 × 1011
a 1 ék qù = 0.2 ´ 1011 N/m 2
\ b= = ....(i)
P P êë z úû Note : We can also take the value of Y from options
Also for one molecule of a gas given without calculating it as it is same in all options.
æ R ö \ Y = ( 2 ± 0.2 ) ´ 1011 N/m 2
PV = k q çQ k =
è N A ÷ø 10. (b) The time period of a simple pendulum is given by
kq l l l
\ V= ....(ii) T = 2p \ T 2 = 4p 2 Þ g = 4p 2 2
P g g
From (i) & (ii) T

V L3 Dg Dl DT
b= = = L2 = M 0 L2T 0 Þ ´ 100 = ´ 100 + 2 ´ 100
z L g l T
P-8 Topic-wise Solved Papers - PHYSICS
Case (i)
Dl = 0.1 cm, l = 64cm, DT = 0.1s, T = 128s :
Dg 1. (a, d) t = F × r × sin q; W = F × d × cosq
\ ´ 100 = 0.3125
g Dimensionally, light year = wavelength = [L]
Case (ii) 2. (a, b, c)
Dl = 0.1 cm, l = 64cm, DT = 0.1s, T = 64s Reynold’s number
Dg = Coefficient of friction = [M0L0T0]
\ ´ 100 = 0.46875 Note : Curie is the unit of radioactivity (number of
g atoms decaying per second) and frequency also has
Case (iii) the unit per second.
Dl = 0.1 cm, l = 20cm, DT = 0.1s, T = 36s
Q W
Dg Latent heat = and Gravitation potential = .
\ ´ 100 = 1.055 m m
g 3. (a, b, c, d)
Dg f æ dI ö 2U
Clearly, the value of ´ 100 will be least in case (i). L = ; L = -e ç ÷ ; L = ; L=R×t
g è dt ø
I I2
11. (d) 20 divisions on the vernier scale f weber
= 16 divisions of main scale L= =
\ 1 division on the vernier scale I ampere
16 16 -e volt volt-sec
= divisions of main scale = ´ 1mm = 0.8 mm L= = =
20 20 dI / dt ampere/sec ampere
We know that least count = 1MSD – 1VSD 2U joule
= 1 mm – 0.8 mm L=
2
=
I (ampere) 2
= 0.2 mm
12. (c) Diameter D = M.S.R. + (C.S.R) × L.C. L = R ´ t = ohm - sec
0.5 Q1Q2 F µ0 I1I 2
D = 2.5 + 20 × 4. (b, c) By definition F = and =
50 (4pe 0 ) r 2 l 2pL
D = 2.70 mm
The uncertainty in the measurement of diameter [Q ]2 I 2T 2
Hence, [e0] = = = M–1L–3 T 4 I 2
DD = 0.01 mm. [ F ][ r 2 ] MLT -2 .L2
We know that
Mass M M [F ] MLT - 2
r= = = [µ0] = = = MLT -2 I -2
Volume V 4 æ Dö
3 [ I ]2 I2
πç ÷ 5. (a,c) As the length of the string of simple pendulum is exactly
3 è 2ø
l m (given), therefore the error in length Dl = 0.
Dr DM DD Further the possibility of error in measuring time is 1s
\ ´ 100 = ´ 100 + 3 ´ 100
r M D in 40s.
0.01 Dt DT 1
=2+3× ´ 100 = 3.1% \ = =
2.70 t T 40
13. (a) The maximum possible error in Y due to l and d are 40
The time period T = = 2 seconds
DY Dl 2 Dd 20
= + DT 1 DT 1
Y l d \ = Þ = Þ DT = 0.05sec
The least count T 40 2 40
Pitch l l
= We know that T = 2p Þ T 2 = 4p 2
Number of divisions on circular scale g g
0.5 l
= mm = 0.005 mm \ g = 4p2
100 T2
Dl 0.005 mm 1 Dg Dl DT
= = \ ´ 100 = ´ 100 + 2 ´100
Error contribution of l = g l T
l 0.25 mm 50
Dg æ 1 ö
2 Dd 2 × 0.005 mm 1 \ ´100 = 0 + 2 ç ÷ ´100 = 5
Error contribution of d = = = g è 40 ø
d 0.5 mm 50 6. (d) Given 2d sin q = l
14. (b) Reading = M.S.R + No of division of V.S matching the
l
main scale division (1MSD – 1VSD) \ d = cosec q ... (i)
2
æ 2.45 ö Differentiating the above equation w.r.t 'q' we get
= 5.10 + 24 ç 0.05 - ÷
è 50 ø d(d) l
= 5.124 cm Option (b) is correct. = [ - cosec q cot q]
dq 2
UNITS & MEASUREMENTS P-9
l Surface Area = pDl
\ d (d) = – cosecq cotq dq ... (ii) 22
2
= × 1.47 × 56 mm2 = 2.58724 cm2
on dividing (i) and (ii) we get 7
d(d) = 2.6 cm2 (Rounding off to two significant figures)
\ = cot q d q W L
d 8. Y= ´
2 X
As q increases from 0° to 90°, cot q decreases and pD
d(d) 4
therefore decreases option (d) is correct KEY CONCEPT : Maximum error in Y is given by
d
l cos q æ DY ö æ DD ö DX DL
çè ÷ = 2ç + +
From (ii) |d (d)| = 2 Y ø max è D ÷ø X L
2 sin q
cos q æ 0.001ö æ 0.001ö æ 0.1 ö
= 2ç + + = 0.0489
This value of
sin 2 q
decreases as q increases from è 0.05 ÷ø çè 0.125 ÷ø çè 110 ÷ø
0° to 90° It is given that W = 50 N; D = 0.05 cm = 0.05 × 10–2m;
X = 0.125 cm = 0.125 × 10–2m;
SUBJECTIVE PROBLEMS :
L = 110 cm = 110 × 10–2m
1. The M.K.S. unit of Young’s modulus is Nm–2.
The M.K.S. unit of magnetic induction is Tesla. 50 ´ 4 ´ 110 ´ 10-2
\ Y= = 2.24 × 1011N/m2
The M.K.S. unit of power of lens is Dioptre. 3.14(0.05 ´ 10-2 ) ´ (0.125 ´ 10-2 )
2. Given that T µ PadbEc \ Maximum possible error in the value of
Þ [M0L0T1] = [ML–1T–2]a [ML–3]b [ML2T–2]c
Y = DY = 0.0489 × 2.24 × 1011
\ [M0L0T1] = [Ma + b + c L–a –3b + 2c T–2a – 2c] = 1.09 × 1010 N/m2
\ a + b + c = 0, – a – 3b + 2c = 0 9. L.C. = 1 MSD – 1 VSD
– 2a – 2c = 1 9
= 1MSD - MSD
On solving, we get 10
a = – 5/6, b = 1/2, c = 1/3 æ 9ö 1 æ 1 ö
3. Magnetic Flux = [M1L2T–1Q–1] = çè1 - ÷ø MSD = MSD = ç ´ 1÷ mm = 0.1 mm
è 10 ø
10 10
Modulus of Rigidity = [ML–1T–2] The side of cube = 10 mm + 1 × 0.1 mm = 10.1 mm = 1.01 cm
4. Angular Momentum [ML2T–1] mass 2.736 g
Latent heat [L2T–2] Now, density = = = 2.66g / cm3
volume (1.01)3
Torque [ML2T–2] (Rounding off to 3 significant figures)
Capacitance [M–1 L–2T2Q2 ]
Inductance [ML2Q–2] MATCH THE FOLLOWING :
Resistivity [ML3T–1Q–2 ] 1. A: p®q
5. Capacitance coulomb-volt–1, coulomb2- m
Reason : Unit of GMeMs = Fr2 = Nm 2 = kg × m2
joule–1 s2
Inductance ohm-sec, volt-second = kg m3s–2
(ampere)–1 Also (volt) (coulomb) (metre) = (joule) (metre)
Magnetic Induction newton (ampere-metre)–1
= (N - m) (m) = Nm2 = kg m3s–2
1 B:r®s
Q (a) q = CV; U = CV 2
2 3RT 3 RT
2
(b) Refer to solution of Q. 3, type D Reason : vrms = Þ vrms =
M M
(c) F = I l B
3RT
6. (n + 1) divisions of vernier scale = n divisions of main scale. Þ Unit of is m2 s–2
M
n Also (farad) (volt)2 (kg)–1 = (joule) kg–1
\ One vernier division = main scale division
n +1 = N-m kg–1 = kg ms–2 m kg–1 = m2s–2
C:r®s
æ n ö
Least count = 1 M.S.D – 1VS.D = çè1 - ÷ MSD F2
n + 1ø Reason : F = qvB Þ v2 =
q 2 B2
1 a
= M.S.D. = units [ Q 1 MSD = a units] \ Unit of v2 is m2s–2 which is further equal to FV2 kg–1.
n +1 n +1 D:r®s
1mm 2GM 2GM
7. Least Count = = 0.01mm Reason : Escape velocity ve = Þ ve2 =
100 R R
Diameter = MSR + CSR × (least count) GM
= 1 mm + 47 × (0.01) mm = 1.47 mm \ The unit of is m2 s–2.
R
P-10 Topic-wise Solved Papers - PHYSICS
We do not want Ampere [A] in the expression. This is
R PV ML-1T -2 ´ L3
2. (c) Boltzmannn constant = = = only possible when Î0 occurs as square. Therefore
N nTN K options a and b are incorrect.
= ML2 T -2 K -1
Ne 2 L-3 A2T 2
F MLT -2 = = T -2 = T -1
Coefficient of viscosity = = = ML-1T -1 m Îo MM -1 -3 2 4
L A T
6prv L ´ LT -1
E ML2 T -2 Ne 2 c
Planck constant = = = ML-2T -1 2. (b) wp = = 2pn = 2p
v T -1 m Îo l
Hl ML2 T -2 ´ L m Îo
Thermal conductivity = = l = 2pc
tADT T ´ L2 ´ K Ne2
= MLT–3K–1
(c) is the correct option.
22 10-30 ´ 10 -11
= 2´ ´ 3 ´ 108 = 600 nm
COMPREHENSION BASED Q UESTIONS : 7 4 ´ 10 27 ´ (1.6 ´10 -19 ) 2
1. (c) e = [AT], w = [T–1]
N = [L–3], Îo = [M–1 L–3 A2 T4]

r r
1. (a) W = F × s = Fs cos q 29
Therefore 1 V.S.D = MSD
= [ MLT -2 ][ L ] = [ ML2T -2 ] ; 30
r r r 29
t = r ´ F Þ t = rF sin q Least count = 1 MSD – 1VSD = 1 MSD - MSD
30
= [ L ] [ MLT - 2 ] = [ ML2T -2 ] 1 1
2. (c) We know that the velocity of light in vacuum is given = MSD = ´ 0.5° = 1 minute.
30 30
by 10. (a) Number of significant figures in 23.023= 5
1 1 Number of significant figures in 0.0003 = 1
c= \ = c2 = L2T–2
Number of significant figures in 2.1 × 10–3 = 2
mo eo m e
o o
1
3. (b) Momentum = mv = [MLT–1] 11. (a) L.C. = mm
100
E ML2T –2
Planck’s constant, h = = = ML2T -1 1
n T –1 Diameter of wire = MSR + CSR × L.C. = 0 +
100
× 52
F = 0.52 mm = 0.052 cm
4. (a) From stokes law F = 6phrv Þ h =
6prv V V ± DV
MLT -2 12. (a) R= Þ R ± DR =
\h = I I ± DI
Þ h = [ ML-1T -1 ]
[ L ][ LT -1 ]
æ ö
5. (d) Moment of Inertia, I = Mr 2 V ç 1 ± DV / V ÷
æ DR ö
Rç1 ± ÷= ç DI ÷
[I] = [ ML2 ] è R ø I ç 1 ± ÷
r uur uur è I ø
Moment of force, t = r ´ F
r æ DR ö æ DV ö + æ DI ö
é t ù = [ L ][ MLT -2 ] = [ ML2T -2 ] ç ÷ =ç ÷ ç ÷ = (3 + 3)% = 6%
ë û è R ø è V ø è I ø
6. (c) We know that F = q v B 13. (c) Q Reading of Vernier = Main scale reading
F MLT -2 + Vernier scale reading × least count.
\ B= = = MT -1C -1 Main scale reading = 58.5
qv C ´ LT -1
Vernier scale reading = 09 division
7. (a) Momentum, p = m × v least count of Vernier = 0.5°/30
= (3.513) × (5.00) = 17.565 kg m/s
= 17.6 (Rounding off to get three significant figures) 0.5°
Thus R = 58.5° + 9 ×
0.5 30
8. (d) Least count of screw gauge = mm = 0.01mm R = 58.65
50
\ Reading = [Main scale reading + circular scale 1 q1q 2 q1q 2
reading × L.C] – (zero error) 14. (b) As we know, F = Þ e0 =
= [3 + 35 × 0.01] – (–0.03) = 3.38 mm 4pe 0 R 2 4pFR 2
9. (d) 30 Divisions of vernier scale coincide with 29 divisions C2 [AT]2
of main scales Hence, e 0 = 2
= -2 2
= [M -2 L-3T4 A 2 ]
N.m MLT .L
2 Motion

FILL IN THE BLANKS : MCQ's WITH ONE CORRECT ANSWER :


1. A particle moves in a circle of radius R. In half the period of 1. A river is flowing from west to east at a speed of 5 metres
revolution its displacement is ____________and distance per minute. A man on the south bank of the river, capable of
covered is ________. (1983 - 2 Marks) swimming at 10 metres per minute in still water, wants to
2. Four persons K, L, M, N are initially at the four corners of a swim across the river in the shortest time. He should swim
square of side d. Each person now moves with a uniform in a direction (1983 - 1 Mark)
speed v in such a way that K always moves directly towards (a) due north (b) 30° east of north
L, L directly towards M, M directly towards N, and N directly (c) 30° west of north (d) 60° east of north
towards K. The four persons will meet at a time ..............
(1984- 2 Marks) 2. A boat which has a speed of 5 km/hr in still water crosses a
river of width 1 km along the shortest possible path in 15
3. Spotlight S rotates in a horizontal plane with constant angular
minutes. The velocity of the river water in km/hr is
velocity of 0.1 radian/second. The spot of light P moves
along the wall at a distance of 3 m. The velocity of the spot (a) 1 (b) 3 (1988 - 1 Mark)
P when q = 45° (see fig.) is ................. m/s (c) 4 (d) 41
(1987 - 2 Marks)
3. In 1.0 s, a particle goes from point A to point B, moving in a
semicircle of radius 1.0 m (see Figure). The magnitude of the
average velocity (1999S - 2 Marks)
A

1.0m

4. The trajectory of a projectile in a vertical plane is


y = ax - bx 2 , where a, b are constants, and x and y are
respectively the horizontal and vertical distances of the B
projectile from the point of projection. The maximum height (a) 3.14 m/s (b) 2.0 m/s
attained is _______ and the angle of projection from the (c) 1.0 m/s (d) Zero
horizontal is_________ (1997C - 1 Mark) 4. A ball is dropped vertically from a height d above the ground.
It hits the ground and bounces up vertically to a height d/2.
TRUE / FALSE : Neglecting subsequent motion and air resistance, its velocity
v varies with the height h above the ground as (2000S )
1. Two balls of different masses are thrown vertically upwards
with the same speed. They pass through the point of
projection in their downward motion with the same speed
(Neglect air resistance). (1983 - 2 Marks)
(a) (b)
2. A projectile fired from the ground follows a parabolic path.
The speed of the projectile is minimum at the top of its path.
(1984 - 2 Marks)
3. Two identical trains are moving on rails along the equator
on the earth in opposite directions with the same speed.
They will exert the same pressure on the rails. (c) (d)
(1985 - 3 Marks)
P-12 Topic-wise Solved Papers - PHYSICS
5. A particle starts sliding down a frictionless inclined plane.
If Sn is the distance travelled by it from time t = n – 1 sec to
t = n sec, the ratio Sn/Sn+1 is (2004S)
(a)
2n - 1 2n + 1
(a) (b)
2n + 1 2n
2n
(c) (d) 2n + 1
2n + 1 2n - 1
6. A body starts from rest at time t = 0, the acceleration time (b)
graph is shown in the figure. The maximum velocity attained
by the body will be (2004S)

(c)

(d)
(a) 110 m/s (b) 55 m/s
(c) 650 m/s (d) 550 m/s 9. Consider a disc rotating in the horizontal plane with a
7. The velocity-displacement graph of a particle moving constant angular speed w about its centre O. The disc has a
along a straight line is shown (2005S) shaded region on one side of the diameter and an unshaded
region on the other side as shown in the figure. When the
disc is in the orientation as shown, two pebbles P and Q are
simultaneously projected at an angle towards R. The velocity
of projection is in the y-z plane and is same for both pebbles
with respect to the disc. Assume that (i) they land back on
the disc before the disc has completed 1/8 rotation, (ii) their
range is less than half the disc radius, and (iii) w remains
The most suitable acceleration-displacement graph will be constant throughout. Then (2012)

(a) (b)

(a) P lands in the shaded region and Q in the unshaded


region.
(b) P lands in the unshaded region and Q in the shaded
region.
(c) Both P and Q land in the unshaded region.
(c) (d)
(d) Both P and Q land in the shaded region.

8. Two identical discs of same radius R are rotating about their 1. A particle is moving eastwards with a velocity of 5 m/s. In
axes in opposite directions with the same constant angular 10s the velocity changes to 5 m/s northwards. The average
speed w. The discs are in the same horizontal plane. At time acceleration in this time is (1982 - 3 Marks)
t = 0, the points P and Q are facing each other as shown in (a) zero
the figure. The relative speed between the two points P and (b) 1/Ö2 m/s2 towards north-west
Q is vr. In one time period (T) of rotation of the discs, vr as a
function of time is best represented by (2012) (c) 1 / 2 m/s2 towards north-east
1
(d) m / s 2 towards north-west
2
1
(e) m / s 2 towards north
2
MOTION P-13
2. A particle of mass m moves on the x-axis as follows : it starts 4. Particles P and Q of mass 20 gm and 40 gm respectively are
from rest at t = 0 from the point x = 0, and comes to rest at simultaneously projected from points A and B on the ground.
t = 1 at the point x = 1. NO other information is available about The initial velocities of P and Q make 45° and 135° angles
its motion at intermediate times (0 < t< 1). If a denotes the respectively with the horizontal AB as shown in the figure.
instantaneous acceleration of the particle, then: Each particle has an initial speed of 49 m/s. The separation
(1993-2 Marks) AB is 245 m. (1982 - 8 Marks)
(a) a cannot remain positive for all t in the interval
0 £ t £ 1.
(b) |a| cannot exceed 2 at any point in its path.
(c) |a| must be ³ 4 at some point or points in its path.
(d) a must change sign during the motion, but no other
assertion can be made with the information given.
3. The coordinates of a particle moving in a plane are given by Both particle travel in the same vertical plane and undergo a
x(t) = a cos (pt) and y (t) = b sin (pt) where a, b (< a) and p are collision After the collision, P retraces its path, Determine
positive constants of appropriate dimensions. Then the position of Q when it hits the ground. How much time
(1999S - 3 Marks) after the collision does the particle Q take to reach the
(a) the path of the particle is an ellipse ground? Take g = 9.8 m/s2.
5. A body falling freely from a given height ‘H’ hits an inclined
(b) the velocity and acceleration of the particle are normal
plane in its path at a height ‘h’ . As a result of this impact the
to each other at t = p/(2p)
direction of the velocity of the body becomes horizontal.
(c) the acceleration of the particle is always directed
For what value of (h/H) the body will take maximum time to
towards a focus
reach the ground ? (1986 - 6 Marks)
(d) the distance travelled by the particle in time interval 6. Two towers AB and CD are situated a distance d apart as
t = 0 to t = p/(2p) is a shown in figure. (1994 - 6 Marks)
AB is 20 m high and CD is 30 m high from the ground. An
SUBJECTIVE PROBLEMS :
object of mass m is thrown from the top of AB horizontally
1. A car accelerates from rest at a constant rate a for some time with a velocity of 10 m/s towards CD.
after which it decelerates at a constant rate b to come to
rest. If the total time lapse is t seconds, evaluate. (1978)
(i) maximum velocity reached, and
(ii) the total distance travelled.
2. The displacement x of particle moving in one dimension,
under the action of a constant force is related to the time t
by the equation (1979)
t= x +3
where x is in meters and t in seconds.
Find Simultaneously another object of mass 2 m is thrown from
(i) The displacement of the particle when its velocity is the top of CD at an angle of 60° to the horizontal towards
zero, and AB with the same magnitude of initial velocity as that of the
first object. The two objects move in the same vertical plane,
(ii) The work done by the force in the first 6 seconds.
collide in mid-air and stick to each other.
3. Answer the following giving reasons in brief :
(i) Calculate the distance ‘d’ between the towers and,
Is the time variation of position, shown in the figure observed (ii) Find the position where the objects hit the ground.
in nature? (1979) 7. Two guns, situated on the top of a hill of height 10 m, fire
one shot each with the same speed 5 3 m s–1 at some
interval of time. One gun fires horizontally and other fires
upwards at an angle of 60° with the horizontal. The shots
collide in air at a point P. Find (i) the time-interval between
the firings, and (ii) the coordinates of the point P. Take origin
of the coordinate system at the foot of the hill right below
the muzzle and trajectories in x-y plane. (1996 - 5 Marks)
P-14 Topic-wise Solved Papers - PHYSICS
8. A large, heavy box is sliding without friction down a smooth an angle of 45° with the x-axis, a ball is thrown along the
plane of inclination q. From a point P on the bottom of the surface from the origin O. Its velocity makes an angle f
box, a particle is projected inside the box. The initial speed with the x-axis and it hits the trolley.
of the particle with respect to the box is u, and the direction
of projection makes an angle a with the bottom as shown in y
Figure. (1998 - 8 Marks) A

a Q 45°
P O x

q (a) The motion of the ball is observed from the frame of


the trolley. Calculate the angle q made by the velocity
(a) Find the distance along the bottom of the box between vector of the ball with the x-axis in this frame.
the point of projection P and the point Q where the (b) Find the speed of the ball with respect to the surface,
particle lands. (Assume that the particle does not hit if f = 4q/3. (2002 - 5 Marks)
any other surface of the box. Neglect air resistance.)
ASSERTION & REASON TYPE QUESTIONS :
(b) If the horizontal displacement of the particle as seen
by an observer on the ground is zero, find the speed of 1. STATEMENT-1 : For an observer looking out through the
the box with respect to the ground at the instant when window of a fast moving train, the nearby objects appear to
particle was projected. move in the opposite direction to the train, while the distant
9. An object A is kept fixed at the point x = 3 m and y = 1.25 m objects appear to be stationary.
on a plank P raised above the ground. At time t = 0 the plank
and
starts moving along the + x direction with an acceleration
STATEMENT-2 : If the observer and the object are moving
1.5 m/s2. At the same instant a stone is projected from the
origin with a velocity u as shown. A stationary person on at velocities v 1 and v 2 respectively with reference to a
the ground observes the stone hitting the object during its
laboratory frame, the velocity of the object with respect to
downward motion at an angle of 45° to the horizontal. All
the motions are in the X–Y plane. Find u and the time after the observer is v 2 – v 1 . (2008)
which the stone hits the object. Take g = 10 m/s2.
(a) STATEMENT-1 is True, STATEMENT-2 is True;
(2000 - 10 Marks)
STATEMENT-2 is a correct explanation for
STATEMENT-1
(b) STATEMENT-1 is True, STATEMENT-2 is True;
STATEMENT-2 is NOT a correct explanation for
STATEMENT-1
(c) STATEMENT -1 is True, STATEMENT-2 is False
(d) STATEMENT -1 is False, STATEMENT-2 is True

INTEGER VALUE CORRECT TYPE :

1. A train is moving along a straight line with a constant


acceleration ‘a’. A boy standing in the train throws a ball
10. On a frictionless horizontal surface, assumed to be the x-y forward with a speed of 10 m/s, at an angle of 60° to the
plane, a small trolley A is moving along a straight line parallel horizontal. The boy has to move forward by 1.15 m inside
to the y-axis (see figure) with a constant velocity of the train to catch the ball back at the initial height. The
acceleration of the train, in m/s2, is (2011)
( 3 - 1) m/s. At a particular instant, when the line OA makes
MOTION P-15

1. A ball whose kinetic energy is E, is projected at an angle of


7. If A ´ B = B ´ A, then the angle between A and B is [2004]
45° to the horizontal. The kinetic energy of the ball at the
highest point of its flight will be [2002] p p
(a) (b)
(a) E (b) E/ 2 2 3
(c) E/2 (d) zero. p
2. From a building two balls A and B are thrown such that A is (c) p (d)
4
thrown upwards and B downwards (both vertically with the
8. A projectile can have the same range ‘R’ for two angles of
same speed). If vA and vB are their respective velocities on
projection. If ‘T1’ and ‘T2’ to be time of flights in the two
reaching the ground, then [2002]
cases, then the product of the two time of flights is directly
(a) vB > vA proportional to. [2004]
(b) vA = vB
(c) vA > vB 1
(a) R (b)
(d) their velocities depend on their masses. R
3. A car, moving with a speed of 50 km/hr, can be stopped by 1
brakes after at least 6 m. If the same car is moving at a speed (c) (d) R2
of 100 km/hr, the minimum stopping distance is [2003] R2
(a) 12 m (b) 18 m (c) 24 m (d) 6 m 9. Which of the following statements is FALSE for a particle
4. A boy playing on the roof of a 10 m high building throws a moving in a circle with a constant angular speed ?
ball with a speed of 10m/s at an angle of 30º with the [2004]
horizontal. How far from the throwing point will the ball be (a) The acceleration vector points to the centre of the circle
at the height of 10 m from the ground ? [2003] (b) The acceleration vector is tangent to the circle
(c) The velocity vector is tangent to the circle
1 3
[ g = 10m/s2 , sin 30o = , cos 30o = ] (d) The velocity and acceleration vectors are perpendicular
2 2 to each other.
(a) 5.20m (b) 4.33m 10. An automobile travelling with a speed of 60 km/h, can brake
(c) 2.60m (d) 8.66m to stop within a distance of 20m. If the car is going twice as
5. The co-ordinates of a moving particle at any time ‘t’are given fast i.e., 120 km/h, the stopping distance will be
by x = a t 3 and y = b t 3 . The speed of the particle at time (a) 60 m (b) 40 m [2004]
‘t’ is given by [2003] (c) 20 m (d) 80 m
11. A ball is thrown from a point with a speed ' v0 ' at an
(a) 3t a 2 + b2 (b) 3t 2 a 2 + b2
elevation angle of q. From the same point and at the same

(c) t 2 a 2 + b2 (d) a 2 + b2 ' v0 '


instant, a person starts running with a constant speed
2
6. A ball is released from the top of a tower of height h meters.
to catch the ball. Will the person be able to catch the ball? If
It takes T seconds to reach the ground. What is the position
yes, what should be the angle of projection q ?
T (a) No (b) Yes, 30° [2004]
of the ball at second [2004]
3 (c) Yes, 60° (d) Yes, 45°
12. A car, starting from rest, accelerates at the rate f through a
8h
(a) meters from the ground distance S, then continues at constant speed for time t and
9
f
7h then decelerates at the rate to come to rest. If the total
(b) meters from the ground 2
9 distance traversed is 15 S , then [2005]
h 1 2
(c) meters from the ground (a) S = ft (b) S = f t
9 6
17 h 1 2 1 2
(d) meters from the ground (c) S = ft (d) S = ft
18 4 72
P-16 Topic-wise Solved Papers - PHYSICS
19. Consider a rubber ball freely falling from a height h = 4.9 m
13. A particle is moving eastwards with a velocity of 5 ms -1 . In
onto a horizontal elastic plate. Assume that the duration of
10 seconds the velocity changes to 5 ms -1 northwards. collision is negligible and the collision with the plate is totally
The average acceleration in this time is [2005] elastic.
Then the velocity as a function of time and the height as a
1 -2 function of time will be : [2009]
(a) ms towards north
2
v y
1
ms - 2 towards north - east
(b) +v1
h
2 (a) O t
–v1
1 t
(c) ms - 2 towards north - west
2 v y
(d) zero +v1
h
2
14. The relation between time t and distance x is t = ax + bx (b) O t1 2t1 4t1 t
where a and b are constants. The acceleration is –v1
t
3 2
(a) 2bv (b) -2abv [2005]
y
(c) 2 av 2 (d) -2av3
h
15. A particle located at x = 0 at time t = 0, starts moving along O t
with the positive x-direction with a velocity 'v' that varies as (c) t1 2t1

t
v = a x . The displacement of the particle varies with time
as [2006] v y
(a) t 2 (b) t v1 h
(c) t 1/2 (d) t 3
16. A particle is projected at 60o to the horizontal with a kinetic (d) O t t
energy K. The kinetic energy at the highest point is
(a) K/2 (b) K [2007]
(c) Zero (d) K/4 20. A particle has an initial velocity of iˆ + ˆj and an
17. The velocity of a particle is v = v0 + gt + ft2. If its position is
acceleration of . iˆ + . ˆj . Its speed after 10 s is : [2009]
x = 0 at t = 0, then its displacement after unit time (t = 1) is
(a) v0 + g/2 + f (b) v0 + 2g + 3f [2007] (a) 7 2 units (b) 7 units
(c) v0 + g/2 + f/3 (d) v0 + g + f (c) 8.5 units (d) 10 units
18. A body is at rest at x = 0. At t = 0, it starts moving in the 21. A particle is moving with velocity n = k ( yiˆ + xjˆ) , where k
positive x-direction with a constant acceleration. At the same
instant another body passes through x = 0 moving in the is a constant. The general equation for its path is [2010]
positive x-direction with a constant speed. The position of (a) y = x2 + constant (b) y2 = x + constant
the first body is given by x1(t) after time ‘t’; and that of the (c) xy = constant (d) y2 = x2 + constant
second body by x2(t) after the same time interval. Which of 22. A point P moves in counter-clockwise direction on a circular
the following graphs correctly describes (x1 – x 2) as a path as shown in the figure. The movement of 'P' is such
function of time ‘t’? [2008] that it sweeps out a length s = t3 + 5, where s is in metres and
t is in seconds. The radius of the path is 20 m. The
(x1 – x2) (x1 – x 2) acceleration of 'P' when t = 2 s is nearly. [2010]

(a) t (b) O
t B
O
P(x,y)

(x1 – x2) m
(x1 – x 2)
20

x
O A
(c) t (d) t
O O (a) 13m/s2 (b) 12 m/s2
(c) 7.2 ms2 (d) 14m/s2
MOTION P-17
23. For a particle in uniform circular motion, the acceleration a 27. A particle of mass ‘m’ is projected with a velocity v making
at a point P(R,q) on the circle of radius R is (Here q is measured an angle of 30° with the horizontal. The magnitude of
from the x-axis ) [2010] angular momentum of the projectile about the point of
projection when the particle is at its maximum height ‘H’ is
n2 n2
(a) - cos q iˆ + sin q ˆj mv3
R R (a) zero (b) [2011RS]
2g
n2 n2
(b) - sin q iˆ + cos q ˆj 3 mv3 3 mv 2
R R (c) (d)
16 g 2 g
n2 n2
(c) - cos q iˆ - sin q ˆj 28. A boy can throw a stone up to a maximum height of 10 m.
R R The maximum horizontal distance that the boy can throw
the same stone up to will be : [2012]
n2 ˆ n2 ˆ
(d) i+ j (a) 20 2 m (b) 10 m
R R
24. A small particle of mass m is projected at an angle q with the (c) 10 2 m (d) 20 m
x-axis with an initial velocity n0 in the x-y plane as shown in
29. Two cars of mass m1 and m2 are moving in circles of radii r1
n sin q and r2, respectively. Their speeds are such that they make
the figure. At a time t < 0 , the angular momentum of complete circles in the same time t. The ratio of their
g
centripetal acceleration is :
the particle is [2010] (a) m1r1 : m2r2 (b) m1 : m2
y (c) r1 : r2 (d) 1 : 1
30. A particle of mass m is at rest at the origin at time
v0 t = 0. It is subjected to a force F(t) = F0e–bt in the x direction.
Its speed v(t) is depicted by which of the following curves?

q
x (a)

ˆ
(a) - mg n 0 t 2 cos q ˆj (b) mg n 0t cos q k

1 1
(c) - mgn 0 t 2 cos q kˆ (d) mgn0t 2 cos q iˆ
2 2 (b)
where iˆ, ˆj and k̂ are unit vectors along x, y and z-axis
respectively.
25. An object, moving with a speed of 6.25 m/s, is decelerated
at a rate given by: [2011]
(c)
dv
= -2.5 v where v is the instantaneous speed. The time
dt
taken by the object, to come to rest, would be:
(a) 2 s (b) 4 s
(c) 8 s (d) 1 s (d)
26. A water fountain on the ground sprinkles water all around
it. If the speed of water coming out of the fountain is v, the
total area around the fountain that gets wet is : [2011] 31. A projectile is given an initial velocity of (iˆ + 2 ˆj ) m/s, where
v4 p v4
(a) p (b) 2 2 iˆ is along the ground and ĵ is along the vertical. If g = 10
2
g g
m/s2 , the equation of its trajectory is : [JEE-Main 2013]

v2 v2 (a) y = x - 5 x2 (b) y = 2 x - 5 x2
(c) p (d) p
g2 g
(c) 4 y = 2 x - 5 x2 (d) 4 y = 2 x - 25 x 2
P-18 Topic-wise Solved Papers - PHYSICS

Solutions & Explanations


Section-A : JEE Advanced/ IIT-JEE
A 1. 2R, pR 2. d/v 3. 0.6 m/s 4. a2 / 4b, q = tan–1a
B 1. T 2. T 3. F
C 1. (a) 2. (b) 3. (b) 4. (a) 5. (a)
6. (b) 7. (a) 8. (a) 9. (c)
D 1. (b) 2. (a, c, d) 3. (a, b, c)

ab 1 ab 2
E 1. t; t 2. (i) 0 ; (ii) 0 3. No 4. mid point of AB, 3.53 sec.
a+b 2 a+b

h 1
5. = 6. 17.32, 11.547 m from B 7. 1 sec, (5 3 , 5) in metres
H 2

u 2 sin 2a u cos( a + q)
8. (a)
g cos q
(b)
cos q
9. u = (3.75î + 6.25ˆj) m / s , t = 1 sec. 10. 45°, 2 m/sec.

H 1. (b)
I 1. 5

Section-B : JEE Main/ AIEEE


1. (c) 2. (b) 3. (c) 4. (d) 5. (b) 6. (a)
7. (c) 8. (a) 9. (b) 10. (d) 11. (c) 12. (d)
13. (c) 14. (d) 15. (a) 16. (d) 17. (c) 18. (b)
19. (b) 20. (a) 21. (d) 22. (d) 23. (c) 24. (c)
25. (a) 26. (a) 27. (c) 28. (d) 29. (c) 30. (c)
31. (b)

v
FILL IN THE BLANKS : N M
distance C
1. Displacement = AOB = 2R
R v
O
Distance = ACB = pR A O B
v
v cos 45o
displacement
2. As K, L, M and N move, each person takes a curved path as 45o
shown in the figure and meet at O, the centre of the square. K v L
The instantaneous position of K, L, M and N are shown in ALTERNATE SOLUTION
the figure and are joined by dotted lines. We get a square of The relative velocity of K w.r.t L along the line KL is
smaller edge and is rotated as compared to the original r r r r r
vK L = vK - vL = vK + (-vL )
square. As the time passes, the rotation of square increases
and the edge decreases and the square vanishes when K, L, =v
M and N meet at O. (Q the component of velocity of L along KL is zero)
The velocity of K throughout the motion towards the centre v
of the square is v cos 45° and the displacement is KO.
KO ( 2 d / 2) d K v L
\ t= = = d
v cos 45° v/ 2 v
MOTION P-19
The displacement of K till K and L meet is d. 2 2
u sin q
d Maximum height attained H =
\ Time taken for K and L to meet will be = v 2g

ga 2 a2
3. From (iv) H = =
2b ´ 2 g 4b
Also the angle of projection, q = tan -1 ( a )
ALTERNATE SOLUTION
dy
= a - 2bx = 0 (For Maximum Height)
dx
a
Þ x=
2b
Substituting the value of x in y = ax – bx2 to find maximum
height
dx
The velocity (v) of spot = æ aö æ a 2 ö a2 a 2 a 2
dt H= aç ÷ - bç 2 ÷ = - =
è 2b ø è 4b ø 2b 4b 4b
df
and the angular speed (w) of spot light = (b) Slope at x = 0 gives the value of tan q , where q is the
dt
From D SOP, angle of projection

x æ dy ö
çè ÷ø = a = tan q \ q = tan–1 a
tan f = \ x = h tan f dx x = 0
h
dx df TRUE / FALSE :
\ = h sec 2 f \ v = (h sec2 f) w
dt dt 1. KEY CONCEPT
\ v = 3 sec2 45° × 0.1 [Q q + f = 90° ] When the two balls are thrown vertically upwards with the
\ v = 3 × 2 × 0.1 = 0.6 m/s same speed u then their final speed v at the point of projection
4. y = ax – bx2 is v2 – u2 = 2 × g × s
Comparing it with equation of trajectory of a projectile Here, s = 0
\ v = u for both the cases
gx 2 Thus, we find that final velocity is independent of mass.
y = x tan q - Þ tan q = a ... (i)
2u 2 cos 2 q 2. P v2= vX
v1
g vY 2
and =b ... (ii) 1 vX
2u 2 cos 2 q vX
3
From (i) v =Ö vX2 +vy2 >vx vy
1
2 2
v3
v3=Ö vX +vy >vx
As shown in the figure, the velocity at 1 and 3 i.e., at any
arbitrary points before and after the top most point is greater
than vx.
ALTERNATE SOLUTION
T.E. = P.E. + K.E.
T.E. = Constant
At P, K.E. is minimum and P.E. is maximum. Since K.E. is
From (ii) minimum speed is also minimum.
é ù 3. The pressure exerted will be different because one train is
g g (a 2 + 1) êQ cos q =
1
ú moving in the direction of earth’s rotation and other in the
u2 = = ... (iii)
2b cos 2 q 2b êë a 2 + 1 úû opposite direction.

g g MCQ's WITH ONE CORRECT ANSWER :


From (ii) = u 2 cos 2 q Þ = u 2 - u 2 sin 2 q
2b 2b 1. (a) N
g vs vs cos q
2 2 2
Þ u sin q = u - d
2b W E
q
2 2
g (a + 1) g ga
Þ u 2 sin 2 q = - = ... (iv) S
2b 2b 2b vr
P-20 Topic-wise Solved Papers - PHYSICS
4. (a) KEY CONCEPT
d
Time taken to cross the river t = Before hitting the ground, the velocity v is given by
vs cos q v2 = 2 gd (quadratic equation and hence parabolic path)
NOTE : For time to be minimum, cos q = maximum Downwards direction means negative velocity. After
Þ q = 0° collision, the direction become positive and velocity
The swimmer should swim due north. decreases.
2. (b) Displacement = 1 km
ædö
15 1 Further, v '2 = 2 g ´ ç ÷ = gd ;
= hr è 2ø
t=
60 4
æ vö v
velocity along AB =
AB \ çè ÷ø = 2 or v = v ' 2 Þ v ' = 2
t v'
As the direction is reversed and speed is decreased
1 graph (a) represents these conditions correctly.
\ Vb cos q = = 4 km / h
1/ 4 a
5. (a) sn = (2n – 1);
2
a a
sn + 1 = [2(n + 1) – 1] = (2n + 1)
2 2
sn 2n - 1
=
sn +1 2n + 1
6. (b) Change in velocity = area under the graph
4 4 3 1
Þ cos q = = Þ sin q = Area under the graph = ´ 10 ´ 11 = 55 m / s
vb 5 5 2
By velocity triangle ABC
vr vr 3
sin q = \ =
vb vb 5

vr 3
Þ = Þ vr = 3 km/h
5 5
ALTERNATE SOLUTION
r Since, initial velocity is zero, final velocity is 55 m/s.
Shortest route corresponds to vb perpendicular to river ALTERNATE SOLUTION
®
flow vr From the given graph, a – t equation is
d d æ 10 ö
\t= = ® a = ç - ÷ t + (10) or dv = æ - 10 ö t + (10)
vb ® v d è 11 ø ç ÷
vbr - vr2
2 vbr b
dt è 11ø
-10 ö
or dv = æç t dt + (10) dt
or
1
=
1
Þ vr = 3 km/h è 11 ÷ø
4 25 - vr2 Integrating,
æ 10 ö
3. (b) | Average velocity | =
| displacement | ò dv = çè - 11 ÷ø ·ò t dt + 10ò dt
time
2
2r 1 æ 10 ö t
= = 2 ´ = 2 m/s. v = ç - ÷ . + 10t
t 1 è 11ø 2
For v to be maximum,
dv
= 0 Þ - 10 t + 110 = 0
dt
t = 11s
æ -10 ö 11 ´ 11
\ vmax = ç . + 110 = (–55 + 110) m/s = 55 m/s
è 11 ÷ø 2
7. (a) The equation for the given v-x graph is
v
v = – 0 x + v0 ... (i)
x0
MOTION P-21
Differentiating the above equation w.r.t x, we get This horizontal distance travelled will be greater than
v any point on the disc between O and P. Therefore the
dv
=- 0 landing will be in unshaded area. In the same way, the
dx x0 horizontal distance travelled by Q is always less than
Multiplying the above equation in both sides by v, we that of any point between O and R. Therefore the landing
get will be in unshaded area.
dv v v é v ù
v = - 0 ´ v = - 0 ê - 0 x + v0 ú From (i)
dx x0 x0 ë x0 û 1. (b) Average acceleration
uur ur uur ur r
v02 v02 r v f - vi v f + (-vi ) D v
é dv ù a= = =
\ a=
x02
x -
x0
.... (ii) êëQ a = v dx úû t t t
uur ur
To find the resultant of v f and -vi , we draw a diagram

N
®
vf
®
Dv
W E

On comparing the equation (ii) with equation of a


straight line S -vi
y = mx + c
r
v02 | Dv |= v 2f + vi2 = 52 + 52 = 5 2 m/s
we get m = 2 = + ve, i.e. tan q = + ve, i.e., q is acute.
x0 r 5 2 1
a = =
v02 10 2
Also c = - , i.e., the y-intercept is negative uur uuur
x02 Since, | v f | = | - vi |
The above conditions are satisfied in graph (a). r
Therefore, v is directed towards N – W..
8. (a) At t = 0, the relative velocity will be zero.
2. (a, c, d) Note : a cannot remain positive for all t in the
T interval 0 < t < 1. This is because since the body starts
At t = , the relative velocity will be maximum in
4 from rest, it will first accelerate. Finally it stops therefore
magnitude. a will become negative. Therefore a will change its
direction. Options (a) and (d) are correct.
T
At t = , the relative velocity will be zero. t=0 t=1
2 A C
3T x=0 B1 B B2 x=1
At t = , the relative velocity will be maximum in
4 Let the particle accelerate uniformly till half the distance
magnitude (A to B) and then retard uniformly in the remaining half
At t = T, the relative velocity again becomes zero. distance (B to C).
The total time is 1 sec. Therefore the time taken from
w A to B is 0.5 sec.
9. (c) R For A to B
R
1 2 1
Q Q S = ut + at 0.5 = 0 + ´ a ´ (0.5) 2
2 2
O O \ a = 4 m/s2
45° \ VB = 0 + 4 × 0.5 = 2 m/s2
Note : Now, if the particle accelerates till B2 then for
P P
covering the same total distance in same time,
t=
p acceleration should be less than 4 m/s2 but |deceleration|
t =0 4w should be greater than 4 m/s2. And if the particle
accelerates till B1, then for covering the same total
p pR distance in the same time, the acceleration should be
The x-coordinate of P = vx × t = wR × = greater than 4 m/s2 and | deceleration | < 4 m/s2.
4w 4
P-22 Topic-wise Solved Papers - PHYSICS
r
At t = t, position of the particle r (t ) = xi$ + y $j
The same is depicted by the graph.

= a cos pt $i + b sin pt ˆj and acceleration of the particle


is
r
a(t ) = ax $i + a y $j = - p 2 [a cos pt $i + b sin pt $j ]
r
= – p2 [ xi$ + y $j ] = - p 2 r (t )
Therefore, acceleration of the particle is always directed
towards origin.
p
So, the | acceleration | must be greater than or equal to At t = 0, particle is at (a, 0) and at t = , particle is at
2p
4 m/s2 at some point or points in the path.
(0, b). Therefore, the distance covered is one fourth of
x the elliptical path and not a.
3. (a, b, c) x = a cos pt Þ cos (pt) = ... (1)
a
y SUBJECTIVE PROBLEMS :
y = b sin pt Þ sin (pt) = ... (2)
b 1. From A to B
Squaring and adding (1) and (2), we get, s1 s2
2 2
x y a1 = a B a2 = b
+ =1 A C
2 2
a b u1 = 0 t1 = t1 v1 t2 = (t – t1) v2 = 0
\ The path of the particle is an ellipse. u1 = 0, t1 = t1, a1 = a, s1 = s1
1 2
s1 = at1 .... (i)
2
v12 = 2as1 Þ v1 = 2as1 .... (ii)
v1 = at1 .... (iii)
From B to C
u2 = v1, v2 = 0, a2 = –b, t2 = t – t1
(0)2 – v12 = 2 (– b)s2
From the given equations we can find, Þ 2as1 = 2bs2 Þ as1 = bs2 .... (iv)
2 v1 = b(t – t1) .... (v)
dx d x
= v x = - ap sin pt ; 2
= ax = - ap 2 cos pt From (iii) and (v)
dt dt b
at1 = bt – bt1 Þ t1 = t
dy d2y a +b
= v y = pb cos pt and = a y = -bp 2 sin pt Substituting in (iii), we get
dt dt 2
ab
p p Max. velocity v1 = ´t
At time t = or pt = a+b
2p 2 Total distance travelled
1 2 a
= s1 + s2 = at + s
2 1 b 1
1 2 a 1 2
= at + ´ at
2 1 b 2 1

1 æ a ö b2 1 æ ab ö 2
a ç1 + ÷
= t2 = ç t
2 è b ø (a + b ) 2 2 è a + bø÷
ALTERNATE SOLUTION
p
ax and v y become zero (because cos = 0 ). Only
2
vx and ay are left,
or we can say that velocity is along negative x-axis and
acceleration along negative y-axis.
p
Hence, at t = , velocity and acceleration of the
2p
particle are normal to each other.
MOTION P-23
Distance travelled = area of D ABC Applying conservation of linear momentum in the horizontal
direction with the information that P retraces its path
1 1 therefore, its momentum will be same in magnitude but
= ´ base ´ altitude = ´ t ´ vmax
2 2 different in direction.
® ¬
1 ab 1 æ ab ö 2 m p v p mQ vQ ® ¬¾¾
¾¾
´t ´ t = ç t + -
2 è a + bø÷
=
2 a+b Before collision
®
2. x = t - 3 Þ x = t2 + 9 – 6 t ®
m p v p mQ v 'Q
dx After collision
v= = 2t - 6
dt Momentum of system before collision = Momentum of
(i) For velocity to be zero, 2t – 6 = 0 Þ t = 3 sec. system after collision.
The displacement is x = 9 + 9 – 6 × 3 = 0 \ mPvP – mQvQ = – mPvP + mQv'Q
2mP vP - mQ vQ
dv \ v'Q =
(ii) a = =2 mQ
dt
\ At t = 0, v = – 6 ms–1
2 ´ 0.02 ´ (49 / 2) - 0.040(49 / 2)
At t = 6 sec, v = 6 ms–1 =
\ Work done = Change in K.E. = [K.Ef – K.Ei] 0.040

1 1
49 é 0.04 ù 49
m(6)2 - m(6)2 = 0 = ê - 1ú = ´0 = 0
= 2 ë 0.040 û 2
2 2
New Path of Q after Collision
3. Since, the vQ' = 0, therefore, the particle Q falls down vertically
and will strike the mid point of AB.
Considering vertical motion of Q
uy = 0; sy = – 61.25; ay = – 9.8; ty = ?
1 2 1 2
s = ut + at = ´ ( -9.8) ´ t = (- 61.25)
2 2
\ t = 3.53 sec
ALTERNATE SOLUTION
As shown, at a given instant of time, the body is at two If a body drops from a height H above the ground then
different positions A and B which is not possible. the time taken by it to reach the ground
4. mP = 20 g, mQ = 40 g 2H 2 ´ 61.25
The horizontal velocities of both the particles are same and t= \ t= = 3.53 s
g 9.8
since both are projected simultaneously, these particle will
meet exactly in the middle of AB (horizontally). 5. For A to B
u = 0; s = – (H – h); a = – g; t = ?
49 m/s 49 m/s 1 2 1 2
Ö2 Ö2 s = ut + at Þ – (H – h) = ( - g )t
P Q 2 2
49m/s
49m/s
45O 135O
A 49/Ö 2m/s 49/Ö 2m/s B
245m

The range of the particles P and Q is

u 2 sin 2q (49)2 sin 2 ´ 45°


R= = = 245 m.
g 9.8
1/ 2
This range is equal to the distance between the particles. é 2( H - h ) ù
Þ t= ê ú
Since, the initial velocity and angle of projection are same, ë g û
we can conclude that the particles will strike at their highest For B to C Vertical Motion
points when the vertical velocities are zero. uy = 0; sy = – h; ay = – g
2 1 2
æ 1 ö s = ut + at
(49)2 ´ ç 2
u 2 sin 2 q è 2 ÷ø
Maximum height = = 2h
2g 2 ´ 9.8 1
–h= ( - g )t '2 t' =
= 61.25 m 2 g
P-24 Topic-wise Solved Papers - PHYSICS
Total time of fall T = t + t' From (ii) and (vi)
1/ 2 1/ 2 1 2 1 2
é 2( H - h) ù é 2h ù gt + 10 = 5 3 t + gt 2 Þ t = sec
T= ê ú +ê ú 2 2 3
ë g û ëgû
2
Note : For finding the maximum time, using the concept of \ BD = PM + MQ = 10 t + 5 t = 15 t = 15 ×
differentiation 3
1/ 2 1/ 2 = 10 3 = 17.32 m
dT d é 2( H - h ) ù d é 2h ù
we have =0 Þ + ê ú =0 ALTERNATE SOLUTION
dh dt êë g ú
û dt ë g û
Let us assume A to be at rest. Then the condition for collision
of objects at A and C will be that the relative velocity of the
1 é 2( H - h) ù
-1/ 2
æ -2 ö 1 é 2 h ù
-1/ 2
æ 2ö r
´ ç ÷+ ê ú object at C with respect to the object at A, i.e., vCA should
Þ
2 êë g ú è g ø 2ë g û çè g ÷ø = 0 uuur
û be along CA . For this condition to be satisfied, we must
-1/ 2 -1/ 2 have
é 2 ( H - h) ù é 2h ù
Þ ê ú = ê ú ( vCA ) y CT
ë g û ëgû = (= tan q)
( vCA ) x AT
2( H - h) 2h h 1 r r r
Þ = Þ H–h=h Þ = Here, vCA = vC - v A
g g H 2
= (10cos 60°)(-iˆ) + (10sin 60°)( - ˆj ) - 10(iˆ)
6. (i) Let t be the time taken for collision.
For mass m thrown horizontally from A. = 5( -iˆ) + 5 3( - ˆj ) - 10(iˆ) = -15 iˆ - 5 3 ˆj
For horizontal motion r r
PM = 10 t ... (i) \ (vCA ) x = 15 , (vCA ) y = 5 3
For vertical motion
uy = 0; sy = y; ay = g; ty = t 5 3 10 10 ´ 15
Þ = Þ d= = 17.32 m
15 d 5 3
1 2
\ y= gt ... (ii) v(CA) x
2 C
vy = uy + ay t = gt ... (iii) vCA v(CA) y
10m
A q T
30m

20m

B D
d
(ii) KEY CONCEPT
Applying conservation of linear momentum (during collision
of the masses at M) in the horizontal direction
m × 10 – 2 m 10 cos 60° = 3 m × vx
For mass 2m thrown from C Þ 10 m – 10 m = 3 m × vx Þ vx = 0
For horizontal motion Since, the horizontal momentum comes out to be zero, the
QM = [10 cos 60°] t combination of masses will drop vertically downwards and
QM = 5 t ... (iv) fall at E.
For vertical motion
2
vy = 10 sin 60° = 5 3 ; ay = g BE = PM = 10 t = 10 × = 11.547 m
3
sy = y + 10; ty = t 7. For Bullet A. Let t be the time taken by bullet A to reach P.
Now, vy = 5 3 + gt ... (v) Vertical Motion
uy = 0; sy = 10 – y; ay = 10 m/s2; ty = t
1 2
and (sy ) = uy t + a t 1 2
2 y sy = uy t + a t
2 y
1 2
Þ y + 10 = 5 3 t + gt ... (vi) 10 – y = 5t 2 ... (i)
2
MOTION P-25
X - direction motion (Taking relative terms w.r.t. box)
ux = + u cos a ; ax = 0, tx = t, sx = sx

1 2 2u sin a u 2 sin 2a
sx = u xt + a xt Þ sx= u cos a × =
2 g cos q g cos q
(b) For the observer (on ground) to see the horizontal
displacement to be zero, the distance travelled by the box in
æ 2u sin a ö
time ç should be equal to the range of the particle.
è g cos q ø÷
Let the speed of the box at the time of projection of particle
be U. Then for the motion of box with respect to ground.
Horizontal Motion
x = 5 3t ... (ii)
For bullet B.
Let (t + t ') be the time taken by bullet B to reach P
Vertical Motion
Let us consider upward direction negative and downward
positive. Then
uy = - 5 3 sin 60° = –7.5 m/s, ay = + 10 m/s2
1 2
sy = + (10 – y); ty = t + t ', sy = uy t + a y t
2 2u sin a -u 2 sin 2a
10 – y = – 7.5 (t + t ') + 5 (t + t')2 ... (iii) ux = – U ; ax= – g sin q ; t y = ; sx =
g cos q g cos q
Horizontal Motion
x = (5 3 cos 60°) (t + t ') 1 2
sx = u xt + axt
2
Þ 5 3 t + 5 3 t ' = 2x ... (iv) 2
Substituting the value of x from (ii) in (iv), we get -u 2 sin 2a æ 2u sin a ö 1 æ 2u sin a ö
= -U ç - g sin q ç
g cos q è g cos q ø÷ 2 è g cos q ø÷
5 3 t + 5 3 t ' = 10 3 t
Þ t = t' u cos (a + q)
Putting t = t ' in eq. (iii) On solving we get U =
cos q
y – 10 = 15 t – 20 t2 ... (v)
Adding (i) and (v) 9. Let 't' be the time after which the stone hits the object and q
r
0 = 15 t – 15t2 Þ t = 1 sec. be the angle which the velocity vector u makes with
(ii) Putting t = 1 in eq. (ii), we get x = 5 3 horizontal.
Putting t = 1 in eq. (i), we get y = 5
Therefore, the coordinates of point P are ( 5 3 , 5) in metres.
8. (a) u is the relative velocity of the particle with respect to
the box. Resolve u.
ux is the relative velocity of particle with respect to the box
in x-direction.
uy is the relative velocity with respect to the box in
y-direction.
Since, there is no velocity of the box in the y-direction,
therefore this is the vertical velocity of the particle with
respect to ground also.
Y-direction motion (Taking relative terms w.r.t. box)
uy = + u sin a According to question, we have following three conditions.
ay = – g cos q (i) Vertical displacement of stone is 1.25 m.
sy = 0 (activity is taken till the time the particle comes 1 2
back to the box.) Therefore, 1.25 = ( u sin q ) t – gt
2
ty = t
where g = 10 m/s2
1 2 1
sy = u y t + a y t Þ 0 = (u sin a) t – g cos q ´ t 2 or (u sin q ) t = 1.25 + 5 t 2 ... (i)
2 2 (ii) Horizontal displacement of stone
2u sin a = 3 + displacement of object A.
Þ t = 0 or t =
g cos q
P-26 Topic-wise Solved Papers - PHYSICS
1 2 (b) Here q = 45°
Therefore, (u cos q )t= 3 + at 4q 4 ´ 45
2 \ f= = = 60°
where a = 1.5 m/s2 3 3
or (u cos q )t = 3 + 0.75 t2 ... (ii) In DOMA,
(iii) Horizontal component of velocity of stone = vertical q = 45° Þ ÐOAM = 45°
component (because velocity vector is inclined at 45° \ ÐOAB = 135°
with horizontal.) Also ÐBOA = 60° – 45° = 15°
Using sine law in DOBA
Therefore (u cos q ) = gt – (u sin q ) ... (iii)
v BG v
(The right hand side is written gt – u sin q because the = TG Þ vBG = 2 m/s
sin135° sin15°
stone is in its downward motion. Therefore, gt > u sin q .
In upward motion usin q > gt). Multiplying equation ASSERTION & REASON TYPE QUESTIONS :
(iii) with t we can write, 1. (b) Statement-1 is true. For a moving observer, the near by
(u cos q ) t + (u sin q ) t = 10 t2 ... (iv) objects appear to move in the opposite direction at a
Now, (iv) – (ii) – (i) gives 4.25 t2 – 4.25 = 0 or t = 1s large speed. This is because the angular speed of the
Substituting t = 1s in (i) and (ii), we get, near by object w.r.t observer is large. As the object moves
u sin q = 6.25 m/s or uy = 6.25 m/s away the angular velocity decreases and therefore its
speed seems to be less. The distant object almost
and u cos q = 3.75 m/s.
r remains stationary.
or u = 3.75 m/s therefore u = u $i + u $j
x x y
Statement-2 is the concept of relative velocity which
r states that
or u = (3.75 $i + 6.25 $j ) m/s r r r
v 21 = v 2 G – v1G
10. (a) where G is the laboratory frame.
B Thus both the statement are true but statement-2 is not
vTG the correct explanation of statement-1.
v BG
INTEGER VALUE CORRECT TYPE :
A
1. 5
From the perspective of observer A, considering vertical
motion of the ball from the point of throw till it reaches back
at the initial height.
45°
O M Uy = + 5 3 m/s, Sy = 0
ay = – 10m/s2, t = ?
Let the ball strike the trolley at B. Let
r 5 3 m/s 10m/s
v BG = velocity of ball w.r.t.ground
r A
B
vTG = velocity of trolley w.r.t.ground
u
\ Velocity of ball w.r.t. trolley
r r r 5m/s
v BT = v BG - vTG ... (i)
1 2
From triangle OAB Applying S = ut + at
uuur uuur uuur uuur r r 2
OA + AB = OB \ OA + vTG = v BG
uuur r r 0 = 5 3t - 5t 2
\ OA = v BG - vTG ... (ii)
uuur r \ t = 3 sec
From (i) and (ii) OA = v BT Considering horizontal motion from the perspective of
Þ velocity of ball w.r.t. trolley makes an angle of 45° observer B. Let u be the speed of train at the time of throw.
with the X-axis The horizontal distance travelled by the ball = (u + 5) 3 .
B The horizonal distance travelled by the boy
v BG
vTG é 1 2ù
= êu 3 + a ( 3) ú + 1.15
ë 2 û
135° 45° As the boy catches the ball therefore
45° A 3
15°
(u + 5) 3 = u 3 + a + 1.15
2
f
q = 45° \ 5 3 = 1.5a + 1.15 \ 7.51 = 1.5a
O M \ a » 5 m/s2
MOTION P-27

1. (c) Let u be the speed with which the ball of mass m is Dividing (i) and (ii) we get
projected. Then the kinetic energy (E) at the point of
100 ´ 100 2 ´ a ´ s
projection is = Þ s = 24m
50 ´ 50 2´a´6
u 4. (d) From the figure it is clear that range is required

u 2 u 2 sin 2q (10)2 sin(2 ´ 30°)


45° R= = =5 3
g 10
1
E= mu 2 ...(i) u
2 30°
When the ball is at the highest point of its flight, the Range R
u

10m

10m
speed of the ball is (Remember that the horizontal
2 Tower
component of velocity does not change during a
projectile motion).
\ The kinetic energy at the highest point
5. (b) x = at 3 and y = bt 3
2 2
1 æ u ö 1 mu E
= m = = [From (i)]
2 çè 2 ÷ø 2 2 2 vx =
dx
= 3at 2 and v y =
dy
= 3bt 2
dt dt
2. (b) Ball A is thrown upwards from the
building. During its downward 2 2 2
u \ v = v2x + v2y = 9a 2t 4 + 9b2 t 4 = 3t a + b
journey when it comes back to the A u
point of throw, its speed is equal to 1 2
the speed of throw. So, for the 6. (a) We know that s = ut + gt ,
h 2
journey of both the balls from point
A to B . 1 2
B or h = gT (Q u = 0)
We can apply v2 – u2 = 2gh. 2
As u, g, h are same for both the balls, vA = vB now for T/3 second, vertical distance moved is given
by
5
3. (c) Case-1 : u = 50 ´ m / s, 1 æTö
2
1 gT 2 h
18 h' = g ç ÷ Þ h' = ´ =
v = 0,s = 6m, a = a 2 è 3ø 2 9 9

h 8h
æ 5ö
2 \ position of ball from ground = h - =
2 2 2 9 9
v - u = 2as Þ 0 - ç 50 ´ ÷ = 2 ´ a ´ 6
è 18 ø r r r r r r r r
7. (c) A´ B - B ´ A = 0 Þ A´ B + A´ B = 0
2 r r
æ 5ö \ A´ B = 0
Þ - ç 50 ´ ÷ = 2 ´ a ´ 6 ....(i)
è 18 ø Angle between them is 0, p, or 2 p
from the given options, q = p
5 8. (a) The angle for which the ranges are same is
Case-2 : u = 100 km/hr = 100 ´ m/sec
18 complementary.
v = 0, s = s, a = a \ v 2 - u 2 = 2as Let one angle be q, then other is 90° – q
2u sin q 2u cos q
2 T1 = , T2 =
2 æ 5ö g g
Þ 0 - ç100 ´ ÷ = 2as
è 18 ø
4u 2 sin q cos q u 2 sin 2 q
2 T1T2 = = 2 R (Q R = )
æ 5ö g g
Þ - ç100 ´ ÷ = 2as … (ii)
è 18 ø Hence it is proportional to R.
P-28 Topic-wise Solved Papers - PHYSICS
9. (b) Only option (b) is false since acceleration vector is
always radial (i.e . towards the center) for uniform 14. (d) t = ax 2 + bx ; Diff. with respect to time (t)
circular motion.
d d dx dx dx
5 50 (t ) = a ( x 2 ) + b = a.2 x + b.
10. (d) Speed, u = 60 ´ m/s = m/s dt dt dt dt dt
18 3
1 = 2axv + bv = v (2ax + b)
5 100
d = 20m, u' = 120 ´ = m/s
18 3 1
2ax + b = .
Let declaration be a then (0)2 – u2 = –2ad v
or u2 = 2ad … (1)
Again differentiating,
and (0)2 – u'2 = –2ad' or u '2 = 2 ad ' …(2)
dx 1 dv
d' 2a +0=- 2
(2) divided by (1) gives, 4 = Þ d ' = 4 ´ 20 = 80m dt v dt
d
11. (c) Yes, the person can catch the ball when horizontal
dv 3 æ dv ö
velocity is equal to the horizontal component of ball’s Þ
dt
= f = - 2av çQ = f = acc ÷
velocity, the motion of ball will be only in vertical è dt ø
direction with respect to person for that,
vo dx dx
15. (a) v=a x , =a xÞ = a dt
= vo cos q or q = 60° dt x
2
1 2
12. (d) Distance from A to B = S = f t1 Þ ft12 = 2 S x
dx
t
2
Distance from B to C = ( f t1) t
ò x
= a ò dt
0 0
2 2
u ( ft1)
Distance from C to D = = = ft12 = 2S x
2 a 2( f / 2) é2 x ù a2 2
ú = a[t ]0 Þ 2 x = at Þ x = 4 t
t
ê
A f B C f /2 D ë 1 û0
t1 t 2t 1 16. (d) Let u be the velocity with which the particle is thrown
15 S and m be the mass of the particle. Then
Þ S + f t1t + 2 S = 15 S Þ f t1t = 12 S 1
K= mu 2 . ... (1)
1 2 2
But f t1 = S
2 At the highest point the velocity is u cos 60° (only the
t horizontal component remains, the vertical component
On dividing the above two equations, we get t1 = being zero at the top-most point). Therefore kinetic
6
energy at the highest point.
2
1 ætö f t2
Þ S= fç ÷ = 1 1 K
2 è 6ø 72 K ' = m(u cos 60°)2 = mu 2 cos2 60° = [From 1]
2 2 4
13. (c) Average acceleration
change in velocity dx
= v2 17. (c) We know that, v = Þ dx = v dt
time interval dt
N
uur D v = v 2 + ( -v 1 ) x t
Dv
= Integrating, ò dx = ò v dt
t 0 0
uur uur
v1 = 5iˆ, v2 = 5 ˆj q
W E t t
r 5 ˆj - 5iˆ ˆj - iˆ - v1 v1 é 2 gt 2 ft 3 ù
\ a= = or x = ò (v0 + gt + ft ) dt = êv0 t + + ú
10 2 ëê 2 3 úû
0 0

12 + (-1) 2 2 1
\ a= = = ms -2 gt 2 ft 3
2 2 2 S or, x = v0 t + +
2 3
v2 5
tan q = = = 1 \ q = 45° g f
v1 5 At t = 1, x = v0 + + .
Therefore the directon is North-west. 2 3
MOTION P-29
18. (b) For the body starting from rest r
21. (d) v = k ( y iˆ + x ˆj )
1 2 x1 – x2 x-component of v = ky
x1 = 0 + at
2
dx
Þ = ky ...(1)
1 2 dt
Þ x1 = at
2 y-component of v = kx
For the body moving dy
with constant speed Þ = kx ...(2)
v/a dt
x2 = vt t
dy x
From (1) and (2) , =
dx y
1 a ( x1 - x2 )
\ x1 - x2 = at 2 - vt Þ = at - v Þ y dy = x dx Þ y 2 = x 2 + constant
2 dt
at t = 0, x1–x2 = 0 22. (d) s = t3 + 5
v ds
For t < ; the slope is negative Þ velocity, v = = 3t 2
a dt
v dv
For t = ; the slope is zero Tangential acceleration a t = = 6t
a dt

v
For t > ; the slope is positive
v 2 9t 4
Radial acceleration ac = =
a R R
These characteristics are represented by graph (b). At t = 2s, at = 6 ´ 2 = 12 m/s2
19. (b) For downward motion v = –gt
The velocity of the rubber ball increases in downward 9 ´ 16
ac = = 7.2 m/s2
direction and we get a straight line between v and t 20
with a negative slope. \ Resultant acceleration
1 2 at2 + ac2 =
Also applying y - y0 = ut + at = (12) 2 + (7.2) 2 = 144 + 51.84
2
= 2
195.84 = 14 m/s
1 2 1 2
We get y - h = - gt Þ y = h - gt r
2 2 23. (c) Clearly a = ac cos q(-iˆ) +ac sin q(- ˆj )
The graph between y and t is a parabola with y = h at
-v 2 v2
t = 0. As time increases y decreases. = cos q iˆ - sin q ˆj
For upward motion. R R
The ball suffer elastic collision with the horizontal Y
elastic plate therefore the direction of velocity is
reversed and the magnitude remains the same. P( R, q)
Here v = u – gt where u is the velocity just after collision.
As t increases, v decreases. We get a straight line R
q
between v and t with negative slope. X
O
1 2
Also y = ut - gt
2
All these characteristics are represented by graph (b). r r r
r r 24. (c) L = m( r ´ v )
20. (a) Given u = 3iˆ + 4 ˆj , a = 0.4iˆ + 0.3 ˆj , t = 10 s
r
L = m éêv0 cos qt iˆ + (v0 sin qt - gt 2 ) ˆj ùú
1
r r r ë 2 û
v = u + at = 3iˆ + 4 ˆj + (0.4iˆ + 0.3 ˆj) ´ 10 = 7iˆ + 7 ˆj
´ éë v0 cos q iˆ + (v0 sin q - gt ) ˆj ùû
r
\ | v |= 72 + 72 = 7 2 units
é 1 ùˆ 1
= mv0 cos qt ê - gt ú k = - mgv0 t cos qkˆ
2
ë 2 û 2
P-30 Topic-wise Solved Papers - PHYSICS

dv dv
25. (a) = -2.5 v Þ = – 2.5 dt u 2 sin 2q u2
dt v R= Þ Rmax =
(g ) g
Integrating,
10 ´ g ´ 2
0 -½ t Rmax =
ò6.25 v dv = -2.5ò dt
0 g
= 20 meter

0
é v +½ ù v12 v22
= -2.5 [ t ]0
t
Þ ê ú 29. (c) a1 = … (1) a2 = … (2)
ëê (½) ûú r1 r2
6.25

Þ – 2(6.25)½ = – 2.5t 2 pr1 2pr2


t1 = and t2 =
Þ t = 2 sec v1 v2
26. (a) Total area around fountain
Þ Putting v1 and v2 in equations (1) and (2) in terms of t.
2
A = pRmax
4p2 r1 4p2 r2
a1 = ; a2 =
v 2 sin 2q v 2 sin 90° v 2
t t
Where Rmax = = =
g g g
a1 r1
Now, a = r
v4 2 2
\ A=p
g2
30. (c) Given that F(t) = F0 e -bt

27. (c) dv
Þ m = F0 e -bt
dt
v dv F0 -bt
v cos q = e
H dt m
q
v t
F0 - bt
Angular momentum
ò dv = mò
e dt
0 0
L0 = pr^
t
é e -bt ù
( Q linear momentum p = mv cos q and r^ = H) F
v= 0
m
ê ú =
F0 é
êë -b úû 0 mb ë (
- e - bt - e -0 ù
û)
Þ L0 = mv cos qH

F0 é
3 v 2 sin 2 30° 3 mv3 Þ v= 1 - e -bt ù
= mv · = mb ë û
2 2g 16 g
r
31. (b) From equation, v = iˆ + 2 ˆj
u2 sin 2 q u2 sin 2 q
28. (d) R= ,H= Þ x=t … (i)
g 2g
1
Hmax at 2q =90 y = 2t - (10t 2 ) … (ii)
2
u2
Hmax = From (i) and (ii), y = 2 x - 5 x2
2g

u2
= 10 Þ u 2 = 10 g ´ 2
2g
Laws of Motion
3
FILL IN THE BLANKS : MCQ's WITH ONE CORRECT ANSWER :

1. A block of mass 1 kg lies on a horizontal surface in a truck. 1. A ship of mass 3 × 107 kg initially at rest, is pulled by a force
The coefficient of static friction between the block and the of 5 × 104 N through a distance of 3m. Assuming that the
surface is 0.6. If the acceleration of the truck is 5 m/s2, the resistance due to water is negligible, the speed of the ship is
(1980)
frictional force acting on the block is ..... newtons.
(a) 1.5 m/sec. (b) 60 m/sec.
(1984 - 2 Marks)
(c) 0.1 m/sec. (d) 5 m/sec.
2. A uniform rod of length L and density r is being pulled 2. A block of mass 2 kg rests on a rough inclined plane making
along a smooth floor with a horizontal acceleration a (see an angle of 30° with the horizontal. The coefficient of static
Fig.) The magnitude of the stress at the transverse cross- friction between the block and the plane is 0.7. The frictional
section through the mid- point of the rod is ......... force on the block is
(1993 - 1 Mark) (a) 9.8 N (b) 0.7 ´ 9.8 ´ 3N
L (c) 9.8 ´ 3N (d) 0.7 × 9.8 N (1980)
3. A block of mass 0.1 is held against a wall applying a horizontal
a force of 5 N on the block. If the coefficient of friction between
the block and the wall is 0.5, the magnitude of the frictional
force acting on the block is : (1994 - 1 Mark)
(a) 2.5 N (b) 0.98 N
TRUE / FALSE : (c) 4.9 N (d) 0.49 N
1. A rocket moves forward by pushing the surrounding air 4. A small block is shot into each of the four tracks as shown
below. Each of the tracks rises to the same height. The
backwards. (1980)
speed with which the block enters the track is the same in all
2. When a person walks on a rough surface, the frictional force
cases. At the highest point of the track, the normal reaction
exerted by the surface on the person is opposite to the is maximum in (2001S)
direction of his motion. (1981 - 2 Marks)
3. A simple pendulum with a bob of mass m swings with an
angular amplitude of 40°. When its angular displacement is (a) (b)
v v
20°, the tension in the string is greater than mg cos 20°.
(1984 - 2 Marks)
4. The pulley arrangements of Figs. (a) and (b) are identical.
The mass of the rope is negligible. In (a) the mass m is lifted (c) v (d) v
up by attaching a mass 2m to the other end of the rope. In
5. An insect crawls up a hemispherical surface very slowly
(b) , m is lifted up by pulling the other end of the rope with
(see fig.). The coefficient of friction between the insect and
a constant downward force F = 2 mg. The acceleration of m
the surface is 1/3. If the line joining the center of the
is the same in both cases (1984 - 2 Marks) hemispherical surface to the insect makes an angle a with
the vertical, the maximum possible value of a is given by
(2001S)

m F = 2 mg
m
(a) cot a = 3 (b) tan a = 3
2m
(c) sec a = 3 (d) cosec a = 3
(a) (b)
P-32 Topic-wise Solved Papers - PHYSICS
6. The pulleys and strings shown in the figure are smooth and 11. Two particles of mass m each are tied at the ends of a light
of negligible mass. For the system to remain in equilibrium, string of length 2a. The whole system is kept on a frictionless
the angle q should be (2001S) horizontal surface with the string held tight so that each mass
is at a distance 'a' from the centre P (as shown in the figure).
Now, the mid-point of the string is pulled vertically upwards
with a small but constant force F. As a result, the particles
q move towards each other on the surface. The magnitude of
acceleration, when the separation between them becomes
Ö2m 2x, is (2007)
F

m m
(a) 0o (b) 30o m m
(c) 45o (d) 60o P
7. A string of negligible mass going over a a a
clamped pulley of mass m supports a block m
of mass M as shown in the figure. The F a F x
(a) (b)
force on the pulley by the clamp is given 2m a 2 - x 2 2m a 2 - x 2
by (2001S)
F x F a2 - x2
M (c) (d)
(a) 2 Mg (b) 2 mg 2m a 2m x
(c) ( M + m) 2 + m 2 g (d) (M + m)2 + M2 g 12. A particle moves in the X-Y plane under the influence of a
force such that its linear momentum is
8. What is the maximum value of the force F such that the r
block shown in the arrangement, does not move? p(t )= A [iˆ cos(kt ) - ˆj sin(kt )], where A and k are constants.
(2003S) The angle between the force and the momentum is
F
(a) 0° (b) 30° (2007)
1
m= (c) 45° (d) 90°
60º 2 3
m = 3kg 13. A block of base 10 cm × 10 cm and height 15 cm is kept on an
inclined plane. The coefficient of friction between them is
3. The inclination q of this inclined plane from the
(a) 20N (b) 10N horizontal plane is gradually increased from 0°. Then
(c) 12N (d) 15N
(2009)
9. A block P of mass m is placed on a horizontal frictionless
plane. A second block of same mass m is placed on it and is (a) at q = 30°, the block will start sliding down the plane
connected to a spring of spring constant k, the two blocks (b) the block will remain at rest on the plane up to certain q
are pulled by distance A. Block Q oscillates without slipping. and then it will topple
What is the maximum value of frictional force between the (c) at q = 60°, the block will start sliding down the plane
two blocks. (2004S) and continue to do so at higher angles
(d) at q = 60°, the block will start sliding down the plane
k and on further increasing q, it will topple at certain q.
Q
ms 14. A block of mass m is on an inclined plane of angle q. The
P coefficient of friction between the block and the plane is m
(a) kA/2 (b) kA and tan q > m. The block is held stationary by applying a
(c) ms mg (d) zero force P parallel to the plane. The direction of force pointing
10. The string between blocks of mass m up the plane is taken to be positive. As P is varied from
and 2m is massless and inextensible. P1 = mg(sinq – m cosq ) to P2 = mg(sinq + m cosq), the
The system is suspended by a frictional force f versus P graph will look like (2010)
massless spring as shown. If the string f f
is cut find the magnitudes of
accelerations of mass 2m and m
2m
(immediately after cutting) (a) (b)
(2006 - 3M, –1)
m
f f
g
(a) g, g (b) g,
2
g g g (c) (d)
(c) ,g (d) ,
2 2 2
LAWS OF MOTION P-33
15. A ball of mass (m) 0.5 kg is attached to 4. A particle P is sliding down a frictionless hemispherical bowl.
the end of a string having length (L) It passes the point A at t = 0. At this instant of time, the
0.5 m. The ball is rotated on a horizontal component of its velocity is v. A bead Q of the
horizontal circular path about vertical
same mass as P is ejected from A at t = 0 along the horizontal
axis. The maximum tension that the
string can bear is 324 N. The maximum L string AB, with the speed v. Friction between the bead and
possible value of anguar velocity of the string may be neglected. Let tP and tQ be the respective
times taken by P and Q to reach the point B. Then :
ball (in radian/s) is (2011) (1993-2 Marks)
m
(a) 9 (b) 18 Q
A B
(c) 27 (d) 36
16. The image of an object, formed by a plano-convex lens at a
distance of 8 m behind the lens, is real is one-third the size of P C
2
the object. The wavelength of light inside the lens is
3 (a) t P < tQ (b) t P = tQ
times the wavelength in free space. The radius of the curved
tP length of arc ACB
surface of the lens is (JEE Adv. 2013) t P > tQ =
(c) (d) tQ length of arc AB
(a) 1 m (b) 2 m
(c) 3 m (d) 6 m 5. A small block of mass of 0.1 kg lies on a fixed inclined plane
PQ which makes an angle q with the horizontal. A horizontal
force of 1 N acts on the block through its centre of mass as
1. In the arrangement shown in the Fig, the ends P and Q of an
shown in the figure. (2012)
unstretchable string move downwards with uniform speed
The block remains stationary if (take g = 10 m/s ) 2
U. Pulleys A and B are fixed. (1982 -3 Marks)

A B

q q
P Q (a) q = 45°
(b) q > 45° and a frictional force acts on the block towards
P.
M (c) q > 45° and a frictional force acts on the block towards
Q.
Mass M moves upwards with a speed
(d) q < 45° and a frictional force acts on the block towards
(a) 2Ucos q (b) U/ cos q (c) 2U / cos q (d)U cos q
2. A reference frame attached to the earth (1986 - 2 Marks) Q.
(a) is an inertial frame by definition. SUBJECTIVE PROBLEMS :
(b) cannot be an inertial frame because the earth is
revolving round the sun. 1. In the diagram shown, the blocks A, B and C weight, 3 kg,
(c) is an inertial frame because Newton’s laws are applicable
4 kg and 5 kg respectively. The coefficient of sliding friction
in this frame.
(d) cannot be an inertial frame because the earth is rotating between any two surface is 0.25. A is held at rest by a
about its own axis. massless rigid rod fixed to the wall while B and C are
3. A simple pendulum of length L and mass (bob) M is connected by a light flexible cord passing around a
oscillating in a plane about a vertical line between angular frictionless pulley. Find the force F necessary to drag C
limit – f and + f . For an angular displacement q ( | q | < f ), along the horizontal surface to the left at constant speed.
the tension in the string and the velocity of the bob are T Assume that the arrangement shown in the diagram, B on C
and V respectively. The following relations hold good under and A on B, is maintained all through. (g = 9.8 m/s2) (1978)
the above conditions : (1986 - 2 Marks)
(a) T cos q = Mg.
MV 2
(b) T – Mg cos q =
L
(c) The magnitude of the tangenial acceleration of the bob A
| aT | = g sin q B
f C
(d) T = Mg cos q
P-34 Topic-wise Solved Papers - PHYSICS
2. Two cubes of masses m1 and m2 be on two frictionless slopes 7. Two blocks of mass 2.9 kg and 1.9 kg are suspended from a
of block A which rests on a horizontal table. The cubes are rigid support S by two inextensible wires each of length 1
connected by a string which passes over a pulley as shown meter, see fig. The upper wire has negligible mass and the
in the figure. To what horizontal acceleration f should the lower wire has a uniform mass of 0.2 kg/m. The whole system
of blocks wires and support have an upward acceleration of
whole system (that is blocks and cubes) be subjected so
0.2 m/s2. Acceleration due to gravity is 9.8 m/s2.
that the cubes do not slide down the planes. What is the (1989 - 6 Marks)
tension of the string in this situation? (1978)
S

m2

b f
2.9 kg
m1
A
a

3. A horizontal uniform rope of length L, resting on a


frictionless horizontal surface, is pulled at one end by force 1.9 kg
F. What is the tension in the rope at a distance l from the (i) Find the tension at the mid-point of the lower wire.
end where the force is applied? (1978) (ii) Find the tension at the mid-point of the upper wire.
4. Two blocks connected by a massless string slide down an 8. A hemispherical bowl of radius R = 0.1 m is rotating about
inclined plane having an angle of inclination of 37°. The its own axis (which is vertical ) with an angular velocity w. A
masses of the two blocks are M1 = 4 kg and M2 = 2 kg particle of mass 10–2 kg on the frictionless inner surface of
respectively and the coefficients of friction of M1 and M2 the bowl is also rotating with the same w. The particle is at a
with the inclined plane are 0.75 and 0.25 respectively. height h from the bottom of the bowl. (1993 - 3+2 Marks)
(i) Obtain the relation between h and w. What is the
Assuming the string to be taut, find (i) the common
minimum value of w needed, in order to have a non-
acceleration of two masses and (ii) the tension in the string.
zero value of h?
(sin 37° = 0.6, cos 37° = 0.8) (1979) (ii) It is desired to measure g (acceleration due to gravity)
5. Masses M1, M2 and M3 are connected by strings of negligible using the set-up, by measuring h accurately. Assuming
mass which pass over massless and friction less pulleys P1 that R and w are known precisely, and that the least
and P2 as shown in fig The masses move such that the count in the measurement of h is 10–4 m, what is the
portion of the string between P1 and P2 in parallel to the minimum possible error D g in the measured value of g?
inclined plane and the portion of the string between P2 and 9. A smooth semicircular wire-track of radius R is fixed in a
M3 is horizontal. The masses M2 and M3 are 4.0 kg each and vertical plane. One end of a massless spring of natural length
the coefficient of kinetic friction between the masses and 3R/4 is attached to the lowest point O of the wire-track. A
the surfaces is 0.25. The inclined plane makes an angle of small ring of mass m, which can slide on the track, is attached
37° with the horizontal. (1981- 6 Marks) to the other end of the spring. The ring is held stationary at
p point P such that the spring makes an angle of 60° with the
1
vertical. The spring constant K= mg/R. Consider the instant
2 p when the ring is released, and (i) draw the free body diagram
2
of the ring, (ii) determine the tangential acceleration of the
3
ring and the normal reaction. (1996 - 5 Marks)
1
37°
If the mass M1 moves downwards with a uniform velocity,
find
(i) the mass of M1
(ii) The tension in the horizontal portion of the string
(g = 9.8 m/sec2, sin 37° - ~ 3/5)
6. A particle of mass m rests on a horizontal floor with which it
has a coefficient of static friction m . It is desired to make the
body move by applying the minimum possible force F. Find
the maguitude of F and the direction in which it has to be
applied. (1987 - 7 Marks)
LAWS OF MOTION P-35
10. Block A of mass m and block B of mass 2m are placed on a (a) Draw a free body diagram for mass M, clearly showing
fixed triangular wedge by means of a massless, in extensible all the forces.
string and a frictionless pulley as shown in figure. The wedge (b) Let the magnitude of the force of friction between m1
is inclined at 45° to the horizontal on both sides. The and M be f1 and that between m2 and ground be f2. For
coefficient of friction between block A and the wedge is 2/3 a particular F it is found that f1 = 2f2. Find f1 and f2.
and that between block B and the wedge is 1/3. If the system Write equations of motion of all the masses. Find F,
A and B is released from rest, find (1997C - 5 Marks) tension in the string and acceleration of the masses.
14. Two block A and B of equal masses are placed on rough
A B
inclined plane as shown in figure. When and where will the
two blocks come on the same line on the inclined plane
m 45° 45° 2m if they are released simultaneously? Initially the block A
is 2 m behind the block B. Co-efficient of kinetic
(i) the acceleration of A, friction for the blocks A and B are 0.2 and 0.3 respectively
(ii) tension in the string, and (g =10 m/s2). (2004 - Marks)
(iii) the magnitude and direction of the force of friction
acting on A 2m
11. Two blocks of mass m1 = 10 kg and m2 = 5 kg, connected to A
each other by a massless inextensible string of length 0.3 m
B
are placed along a diameter of a turn table. The coefficient A
of friction between the table and m1 is 0.5 while there is no
B
friction between m2 and the table. The table is rotating with 45º
an angular velocity of 10 rad/s about a vertical axis passing
through its centre O. The masses are placed along the 15. A circular disc with a groove along its diameter is placed
diameter of the table on either side of the centre O such that horizontally on a rough surface. A block of mass 1 kg is
the mass m1 is at a distance of 0.124 m from O. The masses placed as shown. The co-efficient of friction between the
are observed to be at rest with respect to an observer on the block and all surfaces of groove and horizontal surface in
turn table. (1997 - 5 Marks) 2
(a) Calculate the frictional force on m1. contact is m = . The disc has an acceleration of 25 m/s2
(b) What should be the minimum angular speed of the 5
turn table so that the masses will slip from this position? towards left. Find the acceleration of the block with respect
(c) How should the masses be placed with the string 4 3
remaining taut, so that there is no frictional force acting to disc. Given cos q = , sin q = . (2006 - 6M)
5 5
on the mass m1?
12. A particle of mass 10–2 kg is moving along the positive x
axis under the influence of a force F(x) = – K/(2x2) where
K = 10–2 N m2. At time t = 0 it is at x = 1.0 m and its velocity
is v = 0. (1998 - 8 Marks) 2
25 m/s
(a) Find its velocity when it reaches x = 0.50 m.
(b) Find the time at which it reaches x = 0.25 m. q
13. In the figure masses m1, m2 and M are 20 kg, 5 kg and 50 kg
respectively. The coefficient of friction between M and
ground is zero. The coefficient of friction between m1 and M
and that between m2 and ground is 0.3. The pulleys and the
strings are massless. The string is perfectly horizontal ASSERTION & REASON TYPE QUESTIONS :
between P1 and m1 and also between P2 and m2. The string
1. STATEMENT-1 (2007)
is perfectly vertical between P1 and P2. An external horizontal
force F is applied to the mass M. Take g = 10 m/s2. A cloth covers a table. Some dishes are kept on it. The cloth
(2000 - 10 Marks) can be pulled out without dislodging the dishes from the
table.
because
P1 m1
STATEMENT-2
For every action there is an equal and opposite reaction.
(a) Statement-1 is True, Statement-2 is True; Statement-2
is a correct explanation for Statement-1
M F (b) Statement-1 is True, Statement-2 is True; Statement-2
P2
is NOT a correct explanation for Statement-1
m2 (c) Statement-1 is True, Statement-2 is False
(d) Statement-1 is False, Statement-2 is True.
P-36 Topic-wise Solved Papers - PHYSICS
2. STATEMENT-1 : It is easier to pull a heavy object than to
push it on a level ground and INTEGER VALUE CORRECT TYPE :
STATEMENT-2 : The magnitude of frictional force depends
on the nature of the two surfaces in contact. (2008) 1. A block is moving on an inclined plane making an angle 45°
(a) STATEMENT-1 is True, STATEMENT-2 is True;
STATEMENT-2 is a correct explanation for with the horizontal and the coefficient of friction is m. The
STATEMENT-1 force required to just push it up the inclined plane is 3 times
(b) STATEMENT-1 is True, STATEMENT-2 is True; the force required to just prevent it from sliding down. If we
STATEMENT-2 is NOT a correct explanation for define N = 10 m, then N is (2011)
STATEMENT-1
(c) STATEMENT -1 is True, STATEMENT-2 is False
(d) STATEMENT -1 is False, STATEMENT-2 is True

1. If a body looses half of its velocity on penetrating 3 cm in a 8. One end of a massless rope, which passes over a massless
wooden block, then how much will it penetrate more before and frictionless pulley P is tied to a hook C while the other
coming to rest? [2002] end is free. Maximum tension that the rope can bear is 360
(a) 1 cm (b) 2 cm N. With what value of maximum safe acceleration (in ms–2)
(c) 3 cm (d) 4 cm. can a man of 60 kg climb on the rope? [2002]
2. A lift is moving down with acceleration a. A man in the lift
drops a ball inside the lift. The acceleration of the ball as P
observed by the man in the lift and a man standing stationary
on the ground are respectively [2002] C
(a) g, g (b) g – a, g – a
(c) g – a, g (d) a, g
3. When forces F1, F2, F3 are acting on a particle of mass m
such that F2 and F3 are mutually perpendicular, then the (a) 16 (b) 6
particle remains stationary. If the force F1 is now removed (c) 4 (d) 8.
then the acceleration of the particle is [2002] 9. A spring balance is attached to the ceiling of a lift. A man
hangs his bag on the spring and the spring reads 49 N,
(a) F1/m (b) F2F3/mF1
when the lift is stationary. If the lift moves downward with
(c) (F2 – F3)/m (d) F2/m.
4. Two forces are such that the sum of their magnitudes is 18 N an acceleration of 5 m / s 2 , the reading of the spring balance
and their resultant is 12 N which is perpendicular to the will be [AIEEE-2003]
smaller force. Then the magnitudes of the forces are (a) 24 N (b) 74 N
(a) 12 N, 6 N (b) 13 N, 5 N [2002] (c) 15 N (d) 49 N
(c) 10 N, 8 N (d) 16N, 2N. 10. Three forces start acting simultaneously
r
5. Speeds of two identical cars are u and 4u at the specific on a particle moving with velocity, v . C
instant. The ratio of the respective distances in which the These forces are r epresented in
two cars are stopped from that instant is [2002] magnitude and direction by the three
(a) 1 : 1 (b) 1 : 4 sides of a triangle ABC. The particle
(c) 1 : 8 (d) 1 : 16. will now move with velocity
[AIEEE-2003]
6. A light string passing over a smooth light pulley connects A B
two blocks of masses m 1 and m2 (vertically). If the r
acceleration of the system is g/8, then the ratio of the masses (a) less than v
r
is [2002] (b) greater than v
(a) 8 : 1 (b) 9 : 7 (c) v in the direction of the largest force BC
(c) 4 : 3 (d) 5 : 3. (d) vr , remaining unchanged
7. Three identical blocks of masses m = 2 kg are drawn by a 11. A horizontal force of 10 N is
force F = 10. 2 N with an acceleration of 0. 6 ms–2 on a necessary to just hold a block
frictionless surface, then what is the tension (in N) in the stationary against a wall. The
string between the blocks B and C? [2002] coefficient of friction between
the block and the wall is 0.2. 10N
C B A F
The weight of the block is
(a) 9.2 (b) 3.4 (a) 20 N (b) 50 N [AIEEE-2003]
(c) 4 (d) 9.8 (c) 100 N (d) 2 N
LAWS OF MOTION P-37
12. A marble block of mass 2 kg lying on ice when given a 18. A smooth block is released at rest on a 45° incline and then
velocity of 6 m/s is stopped by friction in 10 s. Then the slides a distance ‘d’. The time taken to slide is ‘n’ times as
coefficient of friction is [AIEEE-2003] much to slide on rough incline than on a smooth incline.
(a) 0.02 (b) 0.03 The coefficient of friction is [2005]
(c) 0.04 (d) 0.06 1 1
13. A block of mass M is pulled along a horizontal frictionless (a) mk = 1 – 2 (b) mk = 1-
surface by a rope of mass m. If a force P is applied at the free
n n2
end of the rope, the force exerted by the rope on the block is 1 1
(c) ms = 1 - (d) m s = 1 -
Pm Pm n2 n2
(a) (b) [AIEEE-2003] 19. A parachutist after bailing out falls 50 m without friction.
M +m M -m
When parachute opens, it decelerates at 2 m / s 2 . He
PM reaches the ground with a speed of 3 m/s. At what height,
(c) P (d)
M +m did he bail out ? [2005]
14. A light spring balance hangs from the hook of the other (a) 182 m (b) 91 m
light spring balance and a block of mass M kg hangs from (c) 111 m (d) 293 m
the former one. Then the true statement about the scale 20. A bullet fired into a fixed target loses half of its velocity after
reading is [AIEEE-2003] penetrating 3 cm. How much further it will penetrate before
(a) Both the scales read M kg each coming to rest assuming that it faces constant resistance to
(b) The scale of the lower one reads M kg and of the upper motion ? [2005]
one zero (a) 2.0 cm (b) 3.0 cm
(c) The reading of the two scales can be anything but the (c) 1.0 cm (d) 1.5 cm
sum of the reading will be M kg 21. An annular ring with inner and outer radii R1 and R2 is
(d) Both the scales read M/2 kg each rolling without slipping with a uniform angular speed. The
ratio of the forces experienced by the two particles situated
15. A rocket with a lift-off mass 3.5 ´ 10 4 kg is blasted upwards
with an initial acceleration of 10m/s2. Then the initial thrust F1
of the blast is [AIEEE-2003] on the inner and outer parts of the ring , F is
2
(a) 3.5 ´ 10 5 N (b) 7.0 ´ 10 5 N 2 R2
æ R1 ö
(c) 5 (d) 1.75 ´ 10 5 N (a) çè R ÷ø (b) R1 [2005]
14.0 ´ 10 N 2
16. Two masses m1 = 5kg and m2 = 4.8 kg tied to a string R1
(c) R2 (d) 1
are hanging over a light frictionless pulley. What is the
acceleration of the masses when left free to move ? 22. The upper half of an inclined plane with inclination f is
2 perfectly smooth while the lower half is rough. A body
( g = 9.8m / s ) [2004]
starting from rest at the top will again come to rest at the
bottom if the coefficient of friction for the lower half is given
by [2005]
(a) 2 cos f (b) 2 sin f
(c) tan f (d) 2 tan f
23. A particle of mass 0.3 kg subject to a force F = – kx with
k = 15 N/m . What will be its initial acceleration if it is released
from a point 20 cm away from the origin ? [2005]
2 2
(a) 15 m / s (b) 3 m / s
(c) 10 m / s 2 (d) 5 m / s 2
24. A block is kept on a frictionless inclined surface with angle
(a) 5 m / s2 (b) 9.8 m / s 2 of inclination ‘ a ’ . The incline is given an acceleration ‘a’ to
keep the block stationary. Then a is equal to [2005]
(c)0.2 m / s 2 (d) 4.8 m / s 2
17. A block rests on a rough inclined plane making an angle of
30° with the horizontal. The coefficient of static friction
between the block and the plane is 0.8. If the frictional force
on the block is 10 N, the mass of the block (in kg) is
a
(take g = 10 m / s 2 ) [2004] a
(a) 1.6 (b) 4.0 (a) g cosec a (b) g / tan a
(c) 2.0 (d) 2.5 (c) g tan a (d) g
P-38 Topic-wise Solved Papers - PHYSICS
25. Consider a car moving on a straight road with a speed of 30. A block of mass m is connected to another block of mass M
100 m/s . The distance at which car can be stopped is by a spring (massless) of spring constant k. The block are
kept on a smooth horizontal plane. Initially the blocks are at
[ m k = 0.5 ] [2005]
rest and the spring is unstretched. Then a constant force F
(a) 1000 m (b) 800 m starts acting on the block of mass M to pull it. Find the force
(c) 400 m (d) 100 m of the block of mass m. [2007]
26. A mass of M kg is suspended by a weightless string. The mF
MF
horizontal force that is required to displace it until the string (a) (b)
(m + M ) M
makes an angle of 45° with the initial vertical direction is
( M + m) F mF
(a) Mg ( 2 + 1) (b) Mg 2 [2006] (c) (d)
m (m + M )
Mg 31. Two fixed frictionless inclined planes making an angle 30°
(c) (d) Mg ( 2 - 1) and 60° with the vertical are shown in the figure. Two blocks
2
A and B are placed on the two planes. What is the relative
27. A ball of mass 0.2 kg is thrown vertically upwards by applying vertical acceleration of A with respect to B ? [2010]
a force by hand. If the hand moves 0.2 m while applying the
force and the ball goes upto 2 m height further, find the A
magnitude of the force. (Consider g = 10 m/s2).
B
(a) 4 N (b) 16 N [2006]
(c) 20 N (d) 22 N
28. A player caught a cricket ball of mass 150 g moving at a rate
of 20 m/s. If the catching process is completed in 0.1s, the
force of the blow exerted by the ball on the hand of the
player is equal to [2006] 60° 30°
(a) 150 N (b) 3 N (a) 4.9 ms–2 in horizontal direction
(c) 30 N (d) 300 N (b) 9.8 ms–2 in vertical direction
29. A coin is placed on a horizontal platform which undergoes (c) Zero
vertical simple harmonic motion of angular frequency w. (d) 4.9 ms–2 in vertical direction
The amplitude of oscillation is gradually increased. The coin 32. The minimum force required to start pushing a body up
will leave contact with the platform for the first time rough (frictional coefficient m) inclined plane is F1 while
(a) at the mean position of the platform [2006] the minimum force needed to prevent it from sliding down
is F2. If the inclined plane makes an angle q from the
g
(b) for an amplitude of F1
w2 horizontal such that tan q = 2m then the ratio is
F2
g2 [2011RS]
(c) for an amplitude of 2
w (a) 1 (b) 2
(d) at the highest position of the platform (c) 3 (d) 4
LAWS OF MOTION P-39

Solutions & Explanations


Section-A : JEE Advanced/ IIT-JEE

A 1. 5 2. rLa / 2
B 1. F 2. F 3. T 4. F
C 1. (c) 2. (a) 3. (b) 4. (a) 5. (a)
6. (c) 7. (d) 8. (a) 9. (a) 10. (c)
11. (b) 12. (d) 13. (b) 14. (a) 15. (d)
16. (c)
D 1. (b) 2. (b, d) 3. (b, c) 4. (a) 5. (a, c)

(m1 sin a + m2 sin b) g m m g sin(a - b)


E 1. 71.05 N 2. f = ; T= 1 2
m1 cos a + m2 cos b m1 cos a + m2 cos b

æ lö
3. T = F ç1 - ÷ 4. (i) 1.306 ms–2 ; (ii) 5.224 N 5. 4.2 Kg, 9.8 N
è Lø

g g
6. mgsinq, tan–1m 7. 20N, 50N 8. (i) h = R - 2
, w> (ii) ± 9.8 × 10–3 m/s2
w R

5 3 3mg 2 2 mg
9. g, 10. (i) acceleration = 0 (ii) mg (iii) downward,
8 8 3 3 2

æp 3 ö÷
ç
11. (a) 36 N (b) 11.67 radian/sec (c) 0.1 m, 0.2 m 12. (a) -1m / s (b) ç 3 + 4 ÷ sec
è ø
13. (b) F = 60 N; T = 18 N

3
a= m/s2 , f1 = 15 N, f2 = 30 N
5

14. 8 2m, 7 2 m, 2 sec . 15. 10 m/s2


H 1. (b) 2. (b)
I 1. 5

Section-B : jee MAIN/ AIEEE

1. (a) 2. (c) 3. (a) 4. (b) 5. (d) 6. (b)


7. (b) 8. (c) 9. (a) 10. (d) 11. (d) 12. (d)
13. (d) 14. (a) 15. (b) 16. (c) 17. (c) 18. (b)
19. (d) 20. (c) 21. (c) 22. (d) 23. (c) 24. (c)
25. (a) 26. (d) 27. (d) 28. (c) 29. (b) 30. (d)
31. (a) 32. (c)
P-40 Topic-wise Solved Papers - PHYSICS

FILL IN THE BLANKS : TRUE / FALSE :

1. The maximum frictional force that can act on the mass of 1. KEY CONCEPT : The rocket moves forward when the
one kg will be exhaust gases are thrown backward.
= µN = µ mg = 0.6 × 1 × 9.8 = 5.88 N. Here exhaust gases thrown backwards is action and rocket
moving forward is reaction.
The truck is accelerating at 5 m/s2.
Reaction
N

mg
The pseudo force acting on the mass as seen by the observer
on the truck is m × a = 1× 5 = 5 N. The frictional force will try
to oppose the movement of the mass by this force. Therefore Exhaust gases
the frictional force acting will be 5 N.
ALTERNATE SOLUTION :As seen by the observer on Action
the ground, the frictional force is responsible to move the Note : This phenomenon takes place in the absence of air as
mass with an acceleration of 5 m/s2. well.
Therefore, frictional force = m × a = 1 × 5 = 5 N. 2. KEY CONCEPT : Friction force opposes the relative motion
2. Let A be the area of cross-section of the rod. Therefore, of the surface of contact.
When a person walks on a rough surface, the foot is the
a A surface of contact. When he pushes the foot backward, the
motion of surface of contact tends to be backwards.
F Therefore the frictional force will act forward (in the direction
f of motion of the person)

m
r= ; mass of rod, m = rAL
AL
f
The rod is moving with an acceleration a under the action
of force F. f
\ F = rALa ... (i) 3. As the angular amplitude of the pendulum is 40°, the bob
will be in the mid of the equilibrium position and the extreme
L position as shown in the figure
Consider the half rod of length . Let the force acting on
2
the mid portion be f. Applying Fnet = ma for the shaded 40°
portion. 20°
rAL T
F–f= a ... (ii) l
2
From (i) and (ii),
L rAL
rALa – f = rA a Þ f= a 20°
2 2 mg cos 20°
f rLa mg
Þ Stress at mid point =
A 2 mv 2
Note : For equilibrium of the bob, T – mg cos 20° = ,
ALTERNATE SOLUTION : l
where l is the length of the pendulum and is the velocity of
Consider the back half portion of the rod. the bob.
rAL mv 2
Mass of half portion of the rod = \ T = mg cos 20° +
2 l
The force responsible for its acceleration is
mv 2
rAL f rLa is always a positive quantity..
f= ´a \ Stress = = l
2 A 2 Hence, T > mg cos 20°.
LAWS OF MOTION P-41
4. Case (i) For mass m 3. (b) Limiting frictional force, fl = µsN = 0.5 × 5 = 2.5 N. But
T – mg = ma ... (i) force tending to produce relative motion is the weight
For mass 2m (W) of the block which is less than fl. Therefore, the
2mg – T = 2ma ... (ii) frictional force is equal to the weight, the magnitude of
the frictional force f has to balance the weight 0.98 N
acting downwards.
m=0.5

T f
a T
5N
a 5N
mg 5N
2 mg
From (i) and (ii) 0.1 × 9.8
= 0.98 N = W
a = g/3
Case (ii) T – mg = ma' Therefore the frictional force = 0.98 N.
4. (a) Since the body presses the surface with a force N hence
2mg – mg = ma' [Q T = 2mg]
according to Newton's third law the surface presses
\ a' = g
the body with a force N. The other force acting on the
body is its weight mg.
N

N
T T=2mg
a' mg
For circular motion to take place, a centripetal force is
mg F=2mg required which is provided by (mg + N).
Hence, a < a'
mv 2
\ mg + N =
MCQ's WITH ONE CORRECT ANSWER : r
where r is the radius of curvature at the top.
1. (c) F = ma If the surface is smooth then on applying conservation
F 5 ´ 104 5 of mechanical energy, the velocity of the body is always
Þ a= = = ´ 10-3 ms -2 same at the top most point. Hence, N and r have inverse
m 3 ´ 107 3 relationship. From the figure it is clear that r is minimum
Also, v2 – u2 = 2as for first figure, therefore N will be maximum.
5 Note : If we do not assume the surface to be smooth,
Þ v2 – 02 = 2 ´ ´ 10 -3 ´ 3 = 10-2 we cannot reach to a conclusion.
3 5. (a) KEY CONCEPT :
Þ v = 0.1 ms–1 For the maximum possible value of a,
2. (a) The force acting on the block along the incline to shift mg sin a will also be maximum and equal to the frictional
the block downwards force.
N f In this case f is the limiting friction. The two forces
acting on the insect are mg and N. Let us resolve mg
into two components.
mg cos a balances N.
mg sin a is balanced by the frictional force.
mg sin 30° mg cos 30°
mg
m=1/3 a

30° f N

= mg sin q = 2 × 9.8 sin 30° = 9.8 N


mgcosa
The limiting frictional force a

3 mg mgsina
fl = µs mg cos q = 0.7 × 2 × 9.8 × = 11.8 N \ N = mg cos a
2
Note : The frictional force is never greater than the force f = mg sin a But f = µN = µ mg cos a
tending to produce relative motion. \ µ mg cos a = mg sin a
Therefore the frictional force is 9.8 N 1
Þ cot a = Þ cot a = 3
µ
P-42 Topic-wise Solved Papers - PHYSICS
6. (c) The tension in both strings will be same due to 9. (a) The forces acting on the masses are shown.
symmetry. Applying Newton's second law on mass Q, we get
F – f = ma ... (i)
where a is the acceleration at the extreme position. Now
Tcosq Tcos q
applying Newton's second law on mass P
f = ma ... (ii)
q q [acceleration is same as no slipping occurs between Q
T T
and P]
T Tsinq B Tsinq T From (i) and (ii)
A C a
Ö2 mg
Q
mg mg F m
For equilibrium in vertical direction for body B we have f
f m
2 mg = 2T cos q P
A
\ 2 mg = 2(mg ) cos q [Q T = mg, (at equilibrium]
1 (Mean position) (Extreme
\ cos q = Þ q = 45° position)
2
F kA
7. (d) At equilibrium T = Mg F = 2ma Þ a = = [Q F = kA]
2m 2 m
T Substituting this value of a in eq. (ii), we get
kA kA
f=m× =
2m 2
T
mg ALTERNATE SOLUTION : Let w be the angular frequency
T of the system.
The maximum acceleration of the system,

Mg é k k ù
2 æ k ö ê w = = ú
T=Mg a =w A =ç A
F.B.D. of pulley è 2m ÷ø ë m+m 2m û
F1 = (m + M) g The force of friction provides this acceleration.
The resultant force on pulley is
æ kA ö kA
\ f = ma = m ç =
F= F12 +T 2
F è 2m ÷ø 2
F1=(m+M)g 10. (c) Just before the string is cut by equilibrium of mass m,
2 2
F = [ (m + M ) + M ] g T ' = mg ... (i)
8. (a) The forces acting on the block are shown. Since the By equilibrium of mass 2m, T = 2mg + T' ... (ii)
block is not moving forward for the maximum force F From (i) and (ii), T = 2mg + mg = 3mg ... (iii)
applied, therefore
F cos 60° = f = µN ... (i) (Horizontal Direction)
Note : For maximum force F, the frictional force is the
limiting friction = µN]
and F sin 60° + mg = N... (ii)
From (i) and (ii)
T T
N 2m 2m
2mg
T'
F cos 60o 2mg
o T'
mg 60 m m
f

F mg mg
Fsin60o Situation 1 Situation 2
F cos 60° = µ [F sin 60° + mg]
m mg When the string is cut :
Þ F= For mass m :
cos 60° - m sin 60° Fnet = mam Þ mg = mam Þ am = g (downwards)
1 For mass 2m :
´ 3 ´ 10 Fnet = 2ma2m Þ 2mg – T = 2ma2m
2 3 5
= = = 20 N
1 1 3 1 -g
- ´ Þ 2mg – 3mg = 2ma2m Þ a2m =
2 2 3 2 4 2
LAWS OF MOTION P-43
The negative sign indicates that the acceleration is in PM 5 cm 2
upwards direction. In D POM , tan q = = =
ALTERNATE SOLUTION :In situation 1, the tension OM 7.5 cm 3
T has to hold both the masses 2m and m therefore, For this, q < 60°. From this we can conclude that the
T = 3mg block will topple at lesser angle of inclination. Thus
the block will remain at rest on the plane up to a certain
In situation 2, when the string is cut, the mass m is a anlgle q and then it will topple.
freely falling body and its acceleration due to gravity 14. (a) As tan q > m, the block has a tendency to move down
is g. the incline. Therfore a force P is applied upwards along
For mass 2m, just after the string is cut, T remains 3mg the incline. Here, at equilibrium P + f = mg sin q Þ f =
because of the extension of string. mg sin q - P
g N
\ 3mg – 2mg = 2m × a \ =a f
2
11. (b) The acceleration of mass m is due to the force T cos q P
q
T cos q mg sinq mg cosq
\ T cos q = ma Þ a= ... (i)
m
F mg
also, F = 2T sin q Þ T= ... (ii)
2 sin q
q
F
Now as P increases, f N
decreases linearly with
T T respect to P.
a
a When P = mg sin q, f = 0. P
Tsinq Tsinq
When P is increased further, q
T f mg cosq
the block has a tendency to mg sinq
q Tcosq move upwards along the mg
x x incline. q
From (i) and (ii)
Therefore the frictional force acts downwards along the
æ F ö cos q F F x incline.
a= ç ÷ = =
è 2sin q ø m 2m tan q 2m a 2 - x 2 Here, at equilibrium P = f + mg sin q
\ f = P – mg sin q
é a 2 - x2 ù Now as P increases, f increases linearly w.r.t P.
êQ tan q ú This is represented by graph (a) .
ê x ú 15. (d) Here, the horizontal component of tension provides
ë û
ur the necessary centripetal force.
12. (d) p (t ) = A [i$ cos (kt ) - $j sin (kt )] \ T sin q = mrw2 ...(i)
ur r
ur d p Also sin q = ...(ii)
F= = Ak [-$i sin (kt ) - $j cos (kt )] L
dt
r r ur ur
Here, F . P = 0 But F . p = Fp cos q
\ cos q = 0 Þ q = 90°. q
13. (b) For the block to slide, the angle of inclination should L
be equal to the angle of repose, i.e.,
tan -1 m = tan -1 3 = 60°. Therefore, option (a) is T
T cos q
wrong.
For the block to topple, the condition of the block will
be as shown in the figure. q
T sinq
N r
f
O 15 c
m mg
q
cm
10 From (i) and (ii)
P
M r
w T ´ = mr w2
L
q T 324 18
\w= = = = 36 rad/s
mL 0.5 ´ 0.5 0.5
P-44 Topic-wise Solved Papers - PHYSICS
16. (c) For a plano convex lens
Mv 2
1 (µ - 1) 1 1 \ T – Mg cos q =
= = - ...(i) l
f R v u
la l 3 q
Here µ = = a = = 1.5
lm 2 l 2
3 a

Where l a = wavelength of light in air q


gsinq gcosq
lm = wavelength of light in water g

Also, the tangential acceleration acting on the mass is


R g sin q .
4. (a) At A the horizontal speeds of both the masses is the
same. The velocity of Q remains the same in horizontal
as no force is acting in the horizontal direction. But in
case of P as shown at any intermediate position, the
horizontal velocity first increases (due to N sin q ),
v = 8m reaches a max value at O and then decreases. Thus it
v 1 always remains greater than v. Therefore tP < tQ.
Also m = =- \ u = -24 cm.
u 3
Q q
1.5 - 1 1 æ 1 ö 1 1 1 A v B
From (i) = -ç ÷= + = Ncosq
R 8 è -24 ø 8 24 6 q
N
\ R = 3m option (c) is correct P
Nsinq

mg O
5. (a, c) The forces are resolved as shown in the figure.
1. (b) This is a problem based on constraint motion. The When q = 45º, sinq = cosq
motion of mass M is constraint with the motion of P
Q
and Q. Let AN = x, NO = z. Then velocity of mass is
dz dl cos q
. Also, let OA = l . then =U R
dt dt
q
From DANO, using pythagorous theorem 1N
2 A x N B
\ x2 + z2 = l sin q q
Here x is a constant. Z sin q
cos q
Differentiating the above l
P U qq U 1N
equation w.r.t to t Q
O
dz dl FIXED q
0 + 2z = 2l Þ zvM = lU M P
dt dt
The block will remain stationary and the frictional
l U U æ zö force is zero.
Þ vM = U = = çèQ cos q = ÷ø When q > 45º, sinq > cosq
z z / l cos q l
Therefore a frictional force acts towards Q.
2. (b, d) Since earth is an accelerated frame and hence, cannot
be an inertial frame. SUBJECTIVE PROBLEMS :
Note : Strictly speaking Earth is accelerated reference 1. When force F is applied on C, the block C will move towards
frame. Earth is treated as a reference frame for practical left.
examples and Newton's laws are applicable to it only
as a limiting case.
3. (b, c) Since the body is moving
in a circular path therefore q A
it needs centripetal force l
T
æ Mv ö2 B
T
ç ÷. C
è l ø F T
v q
Mgsinq
The F.B.D. for mass C is
Mg Mgcos q
LAWS OF MOTION P-45

f2 = m (m A + mB ) g æ T cos a + m1 f ö
F C T çè ÷ø cos a + T sin a = m1 g
sin a
f1 = m (mA + mB + mC ) g
Þ T + m1f cos a = m1g sin a ... (ii)
As C is moving with constant speed F = f1 + f2 + T ... (i) On adding (i) and (ii), we get
F.B.D. for mass B is T + m1 f cos a + m2 f cos b
mm A g = f3 = m1g sin a + T + m2g sin b
B T
m (m A + mB ) g = f 2 (m1 sin a + m2 sin b) g
f=
m1 cos a + m2 cos b
As B is moving with constant speed f2 + f3 = T (ii)
Subtracting (ii) from (i) Substituting the value of f in equation (i) and solving, we
F – (f2 + f3) = f1 + f2 + T – T = f1 + f2 get
Þ F = f1 + 2f2 + f3 = µ (mA + mB + mC) g + m1m2 g sin(a - b)
T=
2µ (mA + mB) g + µ mA g m1 cos a + m2 cos b
F = µ (4 mA + 3 mB + mC) g ALTERNATE SOLUTION : Without Pseudo Force
= 0.25 [4 × 3 + 3 × 4 + 5] × 9.8 = 71.05 N F.B.D for mass m2
2. With Pseudo Force N2 cos b = T sin b + m2g ... (i)
The free body diagram for mass m2 is given. m2 f is the and (N2 sin b + T cos b) = m2 f ... (ii)
pseudo force acting. Since the mass is at rest with respect
to the observer, therefore N2 cos b
N2 cos b N2

N2
b N2 sin b

)
b ) b
T cos b T cos b
)

m2 f )
b N2 sin b
T sin b T
T sin b m2g
m 2g b FBD for mass m1
m2 f = T cos b + N2 sin b and N2 cos b = T sin b + m2 g N1 cos a + T sin a = m1g ... (iii)
Eliminating N2, we get and (N1 sin a – T cos a) = m1 f ... (iv)

æ m2 f - T cos bö N1 cos a
çè ´ cos b = T sin b + m2 g
sin b ø÷ T T sin a N1

Þ m2 f cos b – T cos2 b = Tsin2 b + m2g sin b


Þ m2 f cos b = T + m2g sin b ... (i) a ) a N1 sin b
)

The free body diagram for mass m1 is given m1 f is the pseudo T cos a
force. Since the mass m1 is at rest with respect to the
observer, we get
m1g
N1 cos a
T sin a
On solving the four equations, we get the above results.
T N1 3. Let a be the acceleration of the rope and M be its total mass.
a
Then
T cos a a N1 sin a a a
)

)
m1 f l
F
T F
L L-l T l

M
m 1g T= ( L - l) a ... (i)
L
N1 cos a + T sin a = m1g M
and F – T = ´la ... (ii)
and T cos a + m1f = N1sin a L
On eliminating N1, we get Dividing (i) and (ii)
P-46 Topic-wise Solved Papers - PHYSICS
F -T l 5. (a) For equilibrium of mass M1
= Þ F (L – l ) – T (L – l ) = T l T = M1g ... (i)
T L-l
For equilibrium of mass M2
Þ F (L – l ) = T (L – l + l ) = T × L
T = T ' + M2g sin q + f = T ' + M2g sin q + µM2 g cos q
æ lö [Q f = µN = µM2 g cos q ] ... (ii)
Þ T = F ç1 - ÷
è Lø
ALTERNATE SOLUTION : P1 v
M
From equation (i) T = ( L - l) a T M2 N
L
T æ L - lö æ lö T
Also, F = Ma \ =ç ÷ Þ T = F ç1 - ÷ T'
F è L ø è Lø nq
gsi P2 v
v M1 M2
4. For mass M1 = 4kg M2 g cosq f N'
M1 g T
The component of weight acting along the incline M2 g
= M1g sin 37° = 4 × 9.8 × 0.6 = 23.52 N f'
The maximum frictional force = µM1g cos 37° M3g
= 0.75 × 4 × 9.8 × 0.8 = 23.52 N
Note : Since the maximum frictional force is equal to the Þ T ' = f ' = µN ' = µM3g ... (iii)
component of weight, so if M1 kept below M2, it won't move Putting the value of T and T ' from (i) and (iii) in (ii), we get
and the string will become slack. M1g = µM3g + M2g sin q + µM2g cos q
For the string to remain taut the arrangement is as shown in M1 = 0.25 × 4 + 4 × sin 37° + 0.25 × 4 × cos 37° = 4.2 kg
the fig. The tension in the horizontal string will be
N1 T ' = µM3g
f1 = 0.25 × 4 × 9.8
N2 o T ' = 9.8 N
37
in ALTERNATE SOLUTION : If M 1 , M 2 and M 3 are
gs M1g cos 37o
M1
f2 M1g considered as a system, then the force responsible to more
o them is M1g and the retarding force is (M2g sin q + µM2g
7
n3
g si cos q + µM3g). These two should be equal as the system is
M g cos37 o
M2 ) 37o M2g 2
moving with constant velocity.
For mass M1 6. Let F be the force applied to move the body at an angle q to
T + M1g sin 37° – f1 = M1a the horizontal.
T + M1g sin 37° – µ1M1g cos 37° = M1a ... (i)
Fsinq
For mass M2 F
N
M2g sin 37° – T – f2 = M2a
µN q
M2g sin 37° – T – µ2M2g cos 37° = M2a ... (ii) Fcosq
Adding (i) and (ii)
T + M1g sin 37° – µ1M1g cos 37°
+ M2g sin 37° – T – µ2M2g cos 37° = (M1 + M2) a mg
Þ (M1 + M2) g sin 37° – (µ1M1g + µ2M2g) cos 37°
The body will move when
= (M1 + M2) a
F cos q = µ N ... (i)
Þ (4 + 2) 9.8 × 0.6 – [0.75 × 4 × 9.8 + 0.25 × 2 × 9.8] × 0.8
Applying equilibrium of forces in the vertical direction we
= (4 + 2) a
get
35.28 – [29.4 + 4.9] × 0.8 = 6 a
F sin q + N = mg
7.84
a= = 1.306 ms -2 Þ N = mg – F sin q ... (ii)
6 Þ From (i) and (ii)
Substituting this value in equation (i)
T + 4 × 9.8 × 0.6 – 0.75 × 4 × 9.8 × 0.8 = 4 × 1.306 m mg
F= ... (iii)
T + 23.52 – 23.52 = 5.244 Þ T = 5.224 N cos q + m sin q
LAWS OF MOTION P-47
Differentiating the above equation w.r.t. q , we get

dF m mg T'
= [- sin q + m cos q] = 0 a
d q (cos q + m sin q ) 2
m2
Þ q = tan–1µ
This is the angle for minimum force. 2l
2l
To find the minimum force substituting these values in
equation (iii) m1

(m1+m2+2ll)g
Fig. (2)
T '- (m2 g + l ´ 2 lg + m1 g ) = (m1 + l 2 l + m2 ) a
m
+1

\ T ' = (m1 + l 2 l + m2 ) (a + g )
Öm

= (1.9 + 0.2 × 1 + 2.9) (10) = 5 × 10


q = 50 N.
1 Alternatively considering m1, m2 and lower wire as a system
T' – 5 g = 5 a
µ 1 8. Let m be the mass of the particle and N the normal reaction
sin q = , cos q = acting on it. Resolving N into two rectangular components.
µ2 + 1 µ2 + 1
w

µ mg O
F=
1 m R q
+ ´m Ncosq R-h
2 2 N
µ +1 µ +1 q
Nsinq r M
P
2 h
µ mg ( µ + 1) µ mg mg
Þ F= =
µ2 + 1 µ2 + 1 N cos q = mg
Þ F = mg sin q r w2
N sin q = mrw2 \ tan q =
7. Let l be the mass per unit length of lower wire. g
Let us consider the dotted portion as a system and the
tension T accelerates the system upwards r rw2 g
\ = Þ = R-h
R -h g w2
\ T - (m1 + l l) g = (m1 + l l) a
g
\ T = (m1 + l l) (a + g ) Þ h= R- 2 ... (i)
w
= (1.9 + 0.2 × 0.5) (9.8 + 0.2)
g g
= 2 × 10 = 20 N when h = 0, R - 2 = 0 Þ w =
w R
when w is greater than this value then h has a positive
value.
g
a=0.2m/s2 Therefore w > , in order to have a non-zero positive
R
m2
value of h.
T l
g
l 2
= R - h Þ g = Rw2 – hw2 Þ Dg = ± [w2 Dh]
w
m1
Þ Dg = ± 98 × 10–4 = ± 9.8 × 10–3 m/s2
m1g+l lg
é g 9.8 2 ù
Fig. (1) êQ w min = = Þ w min = 98ú
ë R 0.1 û
To find tension T ' Let us consider the dotted portion given Dg corresponds to the minimum value as we have used the
minimum value of w.
in figure (2)
P-48 Topic-wise Solved Papers - PHYSICS
9. The forces trying to stop this motion (Maximum frictional
force)
= fA + fB = µA NA + µB NA
2 mg 1 2mg 4 mg
= ´ + ´ = ´
3 2 3 2 3 2
N Since the stopping force is more therefore the system will
Fsin30o not move.
C 60 o Þ acceleration of the system is zero.
R mgsin30o (ii) Taking equilibrium of B into consideration
F 30o
o
30
R N 2mg é 1 2mg ù
M
F=kx P O
Fcos30 mg o Since, > µB N B ê µB N B = 3 ´ ú
2 ë 2 û
60o mgcos30o
mg \ The difference of this force is provided by the force of
O tension.
Fig (i) tangential Fig (ii)
2mg 1 2 mg 2 2mg 2 2 mg
T= - = ´ =
In DOCP, OC = CP = R 2 3 2 3 2 3
\ ÐCOP = ÐCPO = 60° Þ ÐOCP = 60° (iii) For max. m
\ DOCP is an equilateral triangle Þ OP = R
2 2 mg
3R R T= in upwards direction; weight component
\ Extension of string = R - = =x 3
4 4
The forces acting are shown in the figure (i) mg mg
in downward direction. Since T >
The free body diagram of the ring is shown in fig. (ii) 2 2
Force in the tangential direction \ The frictional force will act in downward direction
= F cos 30° + mg cos 30
= [kx + mg] cos 30° mg 2 2 mg 4-3 mg
\ f=T– = mg - = mg =
2 3 2 3 2 3 2
5 mg
Ft = 3 \ Ft = mat 11.
8 w
5 3
Þ at = g
8
Also, when the ring is just released m1 T f T m2
N + F sin 30° = mg sin 30°
0.124m 0.176m
Þ N = (mg – F) sin 30° 0.3m

æ mg ö 1 3mg
= çè mg - ÷´ =
4 ø 2 8
2 1
10. µAW = ; µBW = (a) Note : The tension in the string will remain the same.
3 3 The tension T acting on m2 is responsible in moving
the mass m2 in circular motion.
\ T = m2r2w2 = 5 × 0.176 × 10 × 10 = 88 N
T T Since, m1 is also moving in a circular path, the
A B
NA centripetal force required by m1 will be
NB
= m1r1w2 = 10 × 0.124 × 10 × 10 = 124 N
mg mg 2mg 2mg Note : Out of this 124 N required, 88 N will be provided
Ö2 mg Ö2 Ö2 2mg Ö2 by the tension T in the string. The rest will be provided
by frictional force.
45o 45o Therefore, frictional force acting on
m1 = 124 – 88 = 36 N
Diagram shows the various forces acting on the masses (b) Let w' be the maximum angular speed for which no
and their resolution in the direction of motion (except slipping of masses occurs (or we may say that w' is the
frictional force) minimum angular speed for which slipping occurs)
Let us consider the two masses to be a system. The forces The maximum frictional force that can act on mass m1 is
trying to move the system, such that A moves upwards and = µ m1g
B moves downwards B fmax= 0.5 × 10 × 9.8 = 49 N
The equation for m1 to move in circular motion is
2mg mg mg T ' + fmax = m1r1w'2
= - =
2 2 2 where T ' = m2r2w'2 (for mass m2)
LAWS OF MOTION P-49
\ m2r2w'2 + fmax = m1r1w'2 Put x = sin2 q , dx = 2 sin q cos q d q
f max p/6
ò p / 2 2sin
2
Þ w' = So, q d q = -t
m1r1 - m2 r2
cos 2q = 1 - 2sin 2 q ; 2sin 2 q = 1 - cos 2q
49
Þ w' = = 11.67 rad / s p/6 p/6
10 ´ 0.124 - 5 ´ 0.176 é 2q ù
(c) For no frictional force acting on mass m1 òp/2
(1 - cos 2q) d q = - t; êq - sin ú
ë 2 û p /2
= -t
The tension should be sufficient to provided centripetal p 1 p p 1
force for both the masses. Then - sin - - sin p = -t
T = m1r1w2 and T = m2r2w2 6 2 3 2 2
Þ m1r1w2 = m2r2w2 æp 3ö
r1 m2 5 1 \ t= ç + ÷ sec.
Þ = = = .... (i) è3 4 ø
r2 m1 10 2 13. Given m1 = 20 kg, m2 = 5 kg, M = 50 kg, µ = 0.3 and
But r1+ r2 = length of thread Þ r1 + r2 = 0.3 m g = 10 m/s2
Solving equation (i) and (ii), we get (A) Free body diagram of mass M is :
r1 = 0.1 m and r2 = 0.2 m. T N1
12. m = 10–2 kg, motion is along positive X-axis
v=0 T f1
T
Mg F
0 x
F T
1m
t=0 N
K (B) The maximum value of f1 is
F (x) = – 2 , K = 10–2 Nm2; At t = 0, x = 1.0 m (f1)max = (0.3) (20) (10) = 60 N
2x
The maximum value of f2 is
and V = 0
(f2)max = (0.3) (5) (10) = 15 N
-K æ dV ö K Forces on m1 and m2 in horizontal direction are as
(a) F (x) = or m ç V=- 2
2 x2 è dx ÷ø 2x
follows :
v x K T m1 f1 f2 m2 T
or
0
ò
m VdV = -
1 2 x2
dx ò Note : There are only two possibilities.
2 x (1) Either both m1 and m2 will remain stationary (w.r.t.
or
mV éK ù K æ1 ö
ground) or (2) both m1 and m2 will move (w.r.t.
= ê ú = ç - 1÷
2 ë 2x û 1 2 è x ø ground). First case is possible when.
T < (f1)max or T < 60 N
K æ1 ö K æ1 ö and T < (f2)max or T < 15 N
or V2 = ç - 1÷ or | V | = ±
ç - 1÷ ... (i)
mèx ø mèx ø These conditions will be satisfied when T < 15 N say T
= 14 then f1 = f2 = 14 N.
Initially the particle was moving in + X direction at x = 1.
Therefore the condition f1 = 2f2 will not be satisfied.
When the particle is at x = 0.5, obviously its velocity will be
in –X direction. The force acting in –X direction first Thus m1 and m2 both can't remain stationary.
decreases the speed of the particle, bring it momentarily at In the second case, when m1 and m2 both move
rest and then changes the direction of motion of the particle. f2 = (f2) max = 15 N
Therefore, f1 = 2f2 = 30 N
Kæ 1 ö Note : Since f1 < (f1)max, there is no relative motion
When x = 0.5 m : | V | = - çè - 1÷ between m1 and M, i.e., all the masses move with same
m 0.5 ø
acceleration, say 'a'.
Free body diagrams and equations of motion are as
K 10-2 follows:
=- =- = -1m / s
m 10-2 a f1=30N a
K T
m1 m2 M
(b) As = 1 m/s, hence from (i) T F
m
f1=30N f2=15N
dx 1- x For m1 : 30 – T = 20 a ... (i)
V= =-
dt x For m2 : T – 15 = 5 a ... (ii)
Note : We have chosen –ve sign because force tends For M : F – 30 = 50 a ... (iii)
to decrease the displacement with time Solving these three equations, we get,
0.25 t 3 2
x x F = 60 N, T = 18 N and a = m / s .
1- x
dx = - dt ; ò1 1- x
dx = ò0 -dt 5
P-50 Topic-wise Solved Papers - PHYSICS

14. mN = f K ASSERTION & REASON TYPE QUESTIONS :


N = mg cos q
1. (b) Statement 1 : Cloth can be pulled out without
dislodging the dishes from the table because of inertia.
a Therefore, statement – 1 is true.
q Statement 2 : This is Newton's third law and hence
mgsinq true. But statement 2 is not a correct explanation of
mgcosq
statement 1.
mg 2. (b) It is easier to pull a heavy object than to push it on a
level ground. Statement-1 is true. This is because the
mg sin q - m k mg cos q normal reaction in the case of pulling is less as compared
a=
m by pushing. (f = m N). Therefore the frictional force is
\ aA = g sin q – µk,A g cos q ... (i) small in case of pulling.
and aB = g sin q – µk,B g cos q ... (ii) statement-2 is true but is not the correct explanation of
statement-1.
Putting values we get
INTEGER VALUE CORRECT TYPE :
aA = 4 2 m / s 2 and aB = 3.5 2 m / s 2
Let aAB is relative acceleration of A w.r.t. B. Then 1. 5
aAB = aA – aB
L= 2m
[where L is the relative distance between A and B] F1 q
1 mg sin q mg cos q
2
Then L = a AB t f mg
2
2L 2L 45°
or t2 = = The pushing force F1 = mg sinq + f
a AB a A - aB
Putting values we get, t2 = 4 or t = 2s. \ F1 = mg sin q + m mg cos q = mg (sin q + m cos q)
Distance moved by B during that time is given by
1 1
S= aB t 2 = ´ 3.5 2 ´ 4 = 7 2 m f = mN
2 2
Similarly for A = 8 2 m.
F2
15. Applying pseudo force ma and resolving it. mg sin q
Applying Fnet = mar
ma cos q – (f1 + f2) = mar
ma cos q – µN1 –µN2 = mar
ma cos q – µma sin q – µ mg = mar The force required to just prevent it from sliding down
Þ ar = a cos q – µa sin q – µg F2 = mg sin q – mN = mg (sin q – m cos q)
Given , F1 = 3F2
4 2 3 2 \ sin q + mcos q = 3(sin q – m cos q )
= 25 ´ - ´ 25 ´ - ´ 10 = 10 m / s 2
5 5 5 5 \ 1 + m = 3(1 – m) [Q sin q = cos q ]
\ 4m = 2 \ m = 0.5
\ N = 10 m = 5

1. (a) Activity A to B
u2 3 2
u Þ - u 2 = 0.06a Þ - u = 0.06a
u1 = u ; v1 = , s1 = 0.03 m, a1 = ? 4 4
2 -3 2
Þa = u
v12 - u12 = 2a1s1 ...(i) 4 ´ 0.06
Activity B to C: Assuming the same retardation
u/2 speed = 0
u -3
u2 = u /2 ; v2 = 0 ; s2 = ? ; a2 = u2
A 3 cm B C 4 ´ 0.06
v22 - u 22 = 2a2 ´ s2 ...(ii)
2
æ uö
\ ç ÷ - u 2 = 2 ´ a ´ 0.03 u2 æ -3 u 2 ö 1
è 2ø \ 0- = 2ç ÷ ´ s2 Þ s2 = 100 m = 1 cm
4 è 4 ´ 0.06 ø
LAWS OF MOTION P-51
Alternatively, dividing (i) and (ii),
v12 - u12 2a ´ s1
=
v22 - u22 2a ´ s2
2
æ uö 2 T T
çè ÷ø - u0.03 a m2
2
Þ = Þ s2 = 1 cm. m1 a
2 s2
æ uö m2g
0-ç ÷
è 2ø m1g
2. (c) • For the man standing in the left, the acceleration (m1 - m2 ) g
of the ball Adding the equations we get a =
r r r m1 + m2
abm = ab - am Þ abm = g – a
Where 'a' is the acceleration of the mass (because g
Here a =
the acceleration of the lift is 'a' ) 8
• For the man standing on the ground, the
m1
acceleration of the ball -1
r r r 1 m2 m m m 9
abm = ab - am Þ abm = g – 0 = g \ = Þ 1 +1 = 8 1 - 8 Þ 1 =
8 m1 m2 m2 m2 7
3. (a) When F1 , F2 and F3 are acting on a particle then the +1
particle remains stationary. This means that the m2
resultant of F1 F2 and F3 is zero. When F1 is removed,
10.2
F2 and F3 will remain. But the resultant of F2 and F3 7. (b) F = ( m + m + m) ´ a \a = m / s2
r r r 6
should be equal and opposite to F1. i.e. | F2 + F3 | = | F1 |
r r 10.2
| F2 + F3 | F1 \ T2 = ma = 2 ´ = 3.4N
\ a= Þ a= 6
m m F
4. (b) Let the two forces be F1 and F2 and let F2 < F1 . R is the T2
resultant force. C B A
T2 T1 T1
Given F1 + F2 = 18 ...(i)
From the figure F22 + R 2 = F12 8. (c) mg – T = ma
T 360
F12 - F22 = R 2 \ a=g- = 10 - = 4m / s 2
m 60
\ F12 - F22 = 144 ...(ii)
T
Only option (b) follows equation (i) and (ii).
F1
a
®

F2 R
mg
mg
F1 9. (a) For the bag accelerating down
5. (d) For car 1 mg – T = ma
u1 = u, v1 = 0, a1 = – a, s1 = s1
\ v12 - u12 = 2a1s1 Þ – u2 = – 2as1
Þ u2 = 2as1 ...(i)
For car 2 T
u2 = 4u, v1 = 0, a2 = – a, s2 = s2 Bag a
\ v22 - u22 = 2a2 s2 Þ – (4u)2 = 2(–a) s2 mg
Þ 16 u2 = 2as2 ...(ii)
Dividing (i) and (ii),
49
u22as1 1 s1 \ T = m( g – a ) = (10 – 5) = 24.5 N
2
=
2 as
Þ 16 = s 10
16u 2 2 10. (d) As shown in the figure, the three forces are represented
6. (b) For mass m1 by the sides of a triangle taken in the same order.
m1g – T = m1a r r
For mass m2 Therefore the resultant force is zero. Fnet = ma.
T–m2g = m2a Therefore acceleration is also zero ie velocity remains
unchanged.
P-52 Topic-wise Solved Papers - PHYSICS
11. (d) For the block to remain stationary with the wall
f=W æ m - m2 ö
16. (c) Acceleration a = ç 1 g
f = mN è m1 + m2 ÷ø

(5 - 4.8) ´ 9.8
= m / s2 = 0.2 m/s2
10N 10N 10N (5 + 4.8)
17. (c) fs
N
W
mN = W 0.2 × 10 = W Þ W = 2 N
mg
12. (d) u = 6 m/s, v = 0, t = 10s, a = ?
v = u + at Þ 0 = 6 + a ´ 10 30°
-6 mg sinq = f s ( for body to be at rest)
Þ a= = -0.6m / s2
10
Þ m ´ 10 ´ sin 30° = 10
mg
Þ m ´ 5 = 10 Þ m = 2.0 kg
f = mN N 18. (b) g sin q - mg cos q
The retardation is due to the frictional force
\ f = ma Þ mN = ma Þ mmg = ma d q
n
g si d
a 0.6 45° 45°
Þm= = = 0.06
g 10 smooth rough
13. (d) Taking the rope and the block as a system When surface is When surface is
a smooth rough
M m 1 1 2
T P d = ( g sin q)t12 , d = ( g sin q - mg cos q) t2
2 2
we get P = (m + M) a
2d 2d
P t1 = , t2 =
\ a= g sin q g sin q - mg cos q
m+M According to question, t 2 = nt1
Taking the block as a system, we get T = Ma
MP 2d 2d
\ T= n =
m+M g sin q g sin q - mg cos q
14. (a) The Earth pulls the block by a force Mg. The block in m , as applicable here, is coefficient of kinetic friction
turn exerts a force Mg on the spring of spring balance
as the block moves over the inclined plane.
S1 which therefore shows a reading of M kgf.
The spring S1 is massless. Therefore it exerts a force of 1
n= æ 1 ö
Mg on the spring of spring balance S2 which shows ççQ cos 45° = sin 45° = ÷÷
the reading of M kgf. 1 - mk 2ø
è
1 1 1
n2 = or 1 - m k = 2 or m k = 1 - 2
s2 Mkgf 1 - mk n n
19. (d) The velocity of parachutist when parachute opens is
u= 2gh = 2 ´ 9.8 ´ 50 = 980
s1 Mkgf

M
50 m
Mg
15. (b) As shown in the figure F – mg = ma u
Thrust (F) a = - 2 m / s2

3m /s = v
The velocity at ground, v = 3m/s
a v2 - u 2 32 - 980
\ S= = » 243 m
2 ´ ( -2) –4
Initially he has fallen 50 m.
mg \ Total height from where
\ F = m ( g + a) = 3.5 × 104 ( 10+10) = 7 × 105 N he bailed out = 243 + 50 = 293 m
LAWS OF MOTION P-53
20. (c) ALTERNATE SOLUTION :The force experienced by
any particle is only along radial direction.
3 cm x Force experienced by the particle, F = mw2R
u B C F R
A u v=0 \ 1 = 1
F2 R2
2 22. (d) Acceleration of block while sliding down upper half
Case I : A to B v2 – u2 = 2as = g sin f ;
2 retardation of block while sliding down lower half
æ uö 2
çè ÷ø - u = 2. a .3 = – ( g sin f - mg cos f)
2 For the block to come to rest at the bottom,
3u 2 u2 acceleration in I half = retardation in II half.
or – = 2. a. 3 Þ a = –
4 8 g sin f = -( g sin f - mg cos f) Þ m = 2 tan f
Case II : B to C v2 – u2 = 2as ALTERNATE SOLUTION : According to work-energy
2 theorem, W = DK = 0
æ uö (Since initial and final speeds are zero)
0 - ç ÷ = 2. a. x
è 2ø \ Workdone by friction + Work done by gravity = 0
æ u2 ö l
u2 i.e., - ( µ mg cos f ) + mg l sin f = 0
or – = 2.ç - ÷ × x Þ x = 1 cm 2
4 è 8ø µ
or cos f = sin f or µ = 2 tan f
ALTERNATE SOLUTION : 2
Let K be the initial kinetic energy and F be the resistive 23. (c) Mass (m) = 0.3 kg Þ F = m.a = 15x
force. Then according to work-energy theorem,
W = DK 15 150
a=– x= x = 50 x a = 50 × 0.2 = 10 m / s 2
2 0.3 3
1 2 1 æ vö 24. (c) From diagram,
i.e., 3F = mv - m ç ÷
2 2 è 2ø mag cos
1 æ 1ö N
3F = mv 2 ç1 - ÷
2 è 4ø a
3æ1 ö (pseudo ma a
3F = ç mv 2 ÷ ...(1) force) mg cos a
è
4 2 ø + ma sina mg mg sin a
For B to C : Work done by resistive force = DK For block to remain stationary,
2
1 æ vö 1 mg sin a = ma cos a Þ a = g tan a
and Fx = m ç ÷ - m(0)2
2 è 2ø 2 25. (a) v 2 - u 2 = 2as or 02 - u 2 = 2( -m k g ) s
1 æ 1 2ö
Þ Fx = ç mv ÷ø ...(2) 1
4è2 -1002 = 2 ´ - ´ 10 ´ s s = 1000 m
2
x 1 26. (d) Work done by tension + Work done by force (applied)
Dividing eqns. (1) and (2) we get = + Work done by gravitational force = change in kinetic
3 3
or x = 1 cm energy
Work done by tension is zero
21. (c) a2 O
R2

R1 v 2 = wR 2 45°
a1 l
v1 = wR 1
B
A F
C F
v 2 w 2 R12 Þ 0 + F ´ AB - Mg ´ AC = 0
a1 = 1 = = w 2 R1
R1 R1 é 1 ù
æ AC ö ê1 - ú
v22 Þ F = Mgç ÷ = Mg ê
2 ú
a2 = = w 2 R2 è AB ø ê 1 ú
R2
Taking particle masses equal ëê 2 ûú
l
F1 ma1 a1 R1 [Q AB = l sin 45° =
= = = 2
F2 ma2 a2 R2
P-54 Topic-wise Solved Papers - PHYSICS
Now, force acting on the block of mass m is
æ 1 ö
and AC = OC - OA = l - l cos 45° = l ç1 - ÷
è 2ø æ F ö = mF
ma = m ç .
where l = length of the string.] è M + m ÷ø m + M

Þ F = Mg ( 2 - 1) 31. (a) mg sin q = ma


27. (d) Let the velocity of the ball just when it leaves the hand \ a = g sin q
is u then applying, where a is along the inclined plane
v2 – u2 = 2as for upward journey \ vertical component of acceleration is g sin2 q
Þ -u 2 = 2( -10) ´ 2 Þ u 2 = 40 \ relative vertical acceleration of A with respect to B is
Again applying v2 – u2 = 2as for the upward journey of
the ball, when the ball is in the hands of the thrower, g
g (sin 2 60 - sin 2 30] = = 4.9 m/s 2 in vertical
v2 – u2 = 2as 2
Þ 40 - 0 = 2 (a) 0.2 Þ a = 100 m/s2 direction
\ F = ma = 0.2 ´ 100 = 20 N
Þ N - mg = 20 Þ N = 20 + 2 = 22 N 32. (c) N1 F1
ALTERNATE SOLUTION : Whand + Wgravity = DK
Þ F (0.2) – (0.2)(10)(2.2) = 0 Þ F = 22 N

28. (c) m(v - u ) 0.15(0 - 20)


F= = = 30 N
t 0.1 mg sin q
29. (b) For block A to move in SHM. f1 mg cos q
q mg
N
A

N2
x

2
F
mg mean
f2
position
mg – N = mw2x
where x is the distance from mean position
For block to leave contact N = 0 mg sin q
mg cos q
g q mg
2
Þ mg = mw x Þ x =
w2
30. (d) Drawing free body-diagrams for m & M,
For the upward motion of the body
M
m mg sin q + f1 = F1
K
F
or, F1 = mg sin q + mmg cos q
N N
For the downward motion of the body,
a
T T M mg sin q – f 2 = F2
m F
or F2 = mg sin q – mmg cos q
mg Mg
we get T = ma and F – T = Ma F1 sin q + m cos q
\ =
where T is force due to spring F2 sin q - m cos q
Þ F – ma = Ma or,, F = Ma + ma
F tan q + m 2m + m 3m
\ a= . Þ = = =3
M +m tan q - m 2m - m m
4 Work, Energy & Power

7. An ideal spring with spring-constant k is hung from the


MCQ's WITH ONE CORRECT ANSWER :
ceiling and a block of mass M is attached to its lower end.
1. If a machine is lubricated with oil (1980) The mass is released with the spring initially unstretched.
(a) the mechanical advantage of the machine increases. Then the maximum extension in the spring is (2002S )
(b) the mechanical efficiency of the machine increases.
(c) both its mechanical advantage and efficiency increase. 4 Mg 2 Mg
(d) its efficiency increases, but its mechanical advantage (a) (b)
k k
decreases.
2. Two masses of 1 gm and 4 gm are moving with equal kinetic Mg Mg
energies. The ratio of the magnitudes of their linear momenta (c) (d)
k 2k
is (1980)
(a) 4 : 1 (b) 2 :1 (c) 1 : 2 (d) 1 : 16 8. If W1, W2 and W3 represent the work done in moving a
3. A particle of mass m is moving in a circular path of constant particle from A to B along three different paths 1,2 and 3
radius r such that its centripetal acceleration ac is varying respectively (as shown) in the gravitational field of a point
with time t as ac = k2rt2 where k is a constant. The power mass m, find the correct relation between W1, W2 and W3
delivered to the particles by the force acting on it is: B (2003S)
(1994 - 1 Mark)
2 2 2 2 m
(a) 2p mk r t (b) mk r t
(mk 4 r 2 t 5 ) 1 2
(c) (d) zero
3 3
4. A spring of force-constant k is cut into two pieces such that
one piece is double the length of the other. Then the long
piece will have a force-constant of (1999S - 2 Marks)
(a) (2/3)k (b) (3/2)k (c) 3 k (d) 6 k A
5. A wind-powered generator converts wind energy into (a) W1> W2 > W3 (b) W1= W2 = W3
electrical energy. Assume that the generator converts a fixed
fraction of the wind energy intercepted by its blades into (c) W1< W2 < W3 (d) W2> W1 > W3
electrical energy. For wind speed v, the electrical power 9. A particle is acted by a force F = kx, where k is a +ve
output will be proportional to (2000S ) constant. Its potential energy at x = 0 is zero. Which curve
(a) v (b) v 2 (c) v 3 (d) v 4 correctly represents the variation of potential energy of
6. A particle, which is constrained to move along the x-axis, the block with respect to x (2004S)
is subjected to a force in the same direction which
varies with the distance x of the particle from the origin as U
F(x) = –kx + ax3. Here k and a are positive constants. For U
x ³ 0 , the functional form of the potential energy U(x) of
the particle is (2002S ) (a) x (b) x

U(x) U(x)
(a) X (b) X
U U

U(x) U(x) (c) x (d) x


(c) X (d) X
P-56 Topic-wise Solved Papers - PHYSICS
10. A block (B) is attached to two unstretched springs S1 and 14. The work done on a particle of mass m by a force,
S2 with spring constants k and 4k, respectively (see fig. I).
The other ends are attached to identical supports M1 and é x y ù
Kê iˆ + ˆj ú
M2 not attached to the walls. The springs and supports 2 2 32
êë ( x + y ) 2 2 32
(x + y ) úû
have negligible mass. There is no friction anywhere. The (K being a constant of appropriate dimensions), when the
block B is displaced towards wall 1 by a small distance x particle is taken from the point (a, 0) to the point (0, a) along
(figure II) and released. The block returns and moves a a circular path of radius a about the origin in the x – y plane
maximum distance y towards wall 2. Displacements x and y is (JEE Adv. 2013)
are measured with respect to the equilibrium position of the 2K p Kp Kp
block B. The ratio y/x is – (2008) (a) (b) (c) (d) 0
a a 2a
2 1
M2 S M1
2 S
B
1 I 1. A body is moved along a straight line by a machine delivering
x constant power. The distance moved by the body in time t
2 1 is proportional to (1984- 2 Marks)
M2 M1 (a) t 1/2 (b) t 3/4 (c) t 3/2 (d) t 2
S2 S1
B II 2. A uniform chain of length L and mass M is lying on a smooth
table and one third of its length is hanging vertically down
x over the edge of the table. If g is acceleration due to gravity,
(a) 4 (b) 2 (c) 1/2 (d) 1/4 the work required to pull the hanging part on to the table is
11. Two small particles of equal masses start moving in opposite (1985 - 2 Marks)
directions from a point A in a horizontal circular orbit. Their (a) MgL (b) MgL/3 (c) MgL/9 (d) MgL/18
tangential velocities are v and 2v, respectively, as shown in 3. A particle is acted upon by a force of constant magnitude
the figure. Between collisions, the particles move with which is always perpendicular to the velocity of the particle.
constant speeds. After making how many elastic collisions, The motion of the particle takes place in a plane. It follows
other than that at A, these two particles will again reach the that : (1987 - 2 Marks)
point A? (2009) (a) its velocity is constant
A (b) its acceleration is constant
v 2v
(c) its kinetic energy is constant.
(d) it moves in a circular path.
4. A force F = - K ( yiˆ + xjˆ) (where K is a positive constant) acts
on a particle moving in the xy plane. Starting from the origin,
the particle is taken along the positive x axis to the point (a, 0),
and then parallel to the y axis to the point (a, a), The total work
done by the force F on the particle is (1998S - 2 Marks)
(a) 4 (b) 3 (c) 2 (d) 1 (a) – 2Ka2 (b) 2Ka2 (c) – Ka2 (d) Ka 2
12. A piece of wire is bent in the shape of a parabola y = kx2 5. A stone tied to a string of length L is whirled in a vertical
(y-axis vertical) with a bead of mass m on it. The bead can circle with the other end of the string at the centre. At a
slide on the wire without friction. It stays at the lowest point certain instant of time, the stone is at its lowest position,
of the parabola when the wire is at rest. The wire is now and has a speed u. The magnitude of the change in its
accelerated parallel to the x-axis with a constant acceleration velocity as it reaches a position where the string is horizontal
a. The distance of the new equilibrium position of the bead, is (1998S - 2 Marks)
where the bead can stay at rest with respect to the wire, (a) u 2 - 2 gL (b) 2gL
from the y-axis is (2009)
(c) u 2 - gL (d) 2(u 2 - gL)
a a 2a a 6. A small ball starts moving from A over a fixed track as shown
(a) (b) (c) (d)
gk 2 gk gk 4 gk in the figure. Surface AB has friction. From A to B the ball rolls
13. A block of mass 2 kg is free to move along the x-axis. It is at without slipping. Surface BC is frictionless. KA, KB and KC
rest and from t = 0 onwards it is subjected to a time-dependent are kinetic energies of the ball at A, B and C, respectively.
force F(t) in the x direction. The force F(t) varies with t as Then (2006 - 5M, –1)
shown in the figure. The kinetic energy of the block after 4.5 C
seconds is (2010) A
F(t)
hC
4N hA

4.5s
O 3s t B

(a) hA > hC ; K B > KC (b) hA > hC ; KC > K A


(a) 4.50 J (b) 7.50 J (c) 5.06 J (d) 14.06 J (c) hA = hC ; K B = KC (d) hA < hC ; K B > KC
WORK, ENERGY AND POWER P-57
The coefficient of friction between the surfaces of blocks is
SUBJECTIVE PROBLEMS : 0.2. Force constant of the spring is 1960 newtons/m. If mass
1. A bullet is fired from a rifle. If the rifle recoils freely, determine of block A is 2 Kg., calculate the mass of block B and the
whether the kinetic energy of the rifle is greater than, equal energy stored in the spring. (1982 - 5 Marks)
or less than that of the bullet. (1978) 9. A 0.5 kg block slides from the point A (see Fig) on a horizontal
2. A spring of force constant k is cut into three equal parts. track with an initial speed of 3 m/s towards a weightless
What is force constant of each part? (1978) horizontal spring of length 1 m and force constant 2 Newton/
3. A ball falls under gravity from a height of 10 metres with an m. The part AB of the track is frictionless and the part BC
initial downward velocity v0. It collides with the ground, has the coefficients of static and kinetic friction as 0.22 and
loses 50 percent of its energy in collision and then rises 0 2 respectively. If the distances AB and BD are 2 m and 2.14
back to the same height. Find (i) the initial velocity v0 and m respectively, find the total distance through which the
(ii) the height to which the ball would rise, after collision, if block moves before it comes to rest completely.
the initial velocity v0 was directed upward instead of (Take g = 10m / s2) (1983 - 7 Marks)
downward. (1979)
4. A 20 gm bullet pierces through a plate of mass M1 = 1 kg and
then comes to rest inside a second plate of mass M2 = 2.98
kg. as shown. It is found that the two plates initially at rest,
now move with equal velocities. Find the percentage loss in A B D C
the initial velocity of the bullet when it is between M1 and
M2. Neglect any loss of material of the plates due to the 10. A string, with one end fixed on a rigid wall, passing over a
action of the bullet. (1979) fixed frictionless pulley at a distance of 2m from the wall,
has a point mass M = 2kg attached to it at a distance of 1m
from the wall. A mass m = 0.5 kg attached at the free end is
held at rest so that the
string is horizontal between the wall
and the pulley and vertical beyond M
M1 M2 the pulley. What will be the speed with
5. When a ball is thrown up, the magnitude of its momentum which the mass M will hit the wall
decreases and then increases. Does this violate the when the mass m is released ?
conservation of momentum principle? (1979) (1985 - 6 Marks)
m
A D
6. 11. A simple pendulum is suspended from a peg on a vertical
G wall. The pendulum is pulled away from the wall to a
horizontal position (see fig.) and released. The ball hits the
2
wall, the coefficient of restitution being .
B C E F 5
(a) (b) (1987 - 7 Marks)
In the figures (a) and (b) AC, DG and GF are fixed inclined l
planes, BC = EF = x and AB = DE = y. A small block of mass
M is released from the point A. It slides down AC and reaches
C with a speed Vc. The same block is released from rest from
the point D. It slides down DGF and reaches the point F
with speed VF. The coefficients of kinetic frictions between
the block and both the surface AC and DGF are m. (1980)
Calculate VC and VF.
7. A body of mass 2 kg is being dragged with a uniform
velocity of 2 m/sec on a rough horizontal plane. The
coefficient of friction between the body and the surface is
0.20, J = 4.2 J/cal and g = 9.8 m/sec2. (1980)
Calculate the amount of heat generated in 5 sec. What is the minimum number of collisions after which the
8. Two blocks A an d B are amplitude of oscillations becomes less than 60 degrees ?
connected to each other by a 12. Two blocks of mass 2 kg and M are at rest on an inclined
string and a spring ; the string plane and are separated by a distance of 6.0 m as shown in
passes over a frictionless pulley Figure. The coefficient of friction between each of the blocks
as shown in the figure. Block B B and the inclined plane is 0.25. The 2 kg block is given a
slides over the horizontal top velocity of 10.0 m/s up the inclined plane. It collides with M,
surface of a stationary block C comes back and has a velocity of 1.0 m/s when it reaches its
and the block A slides along the initial position. The other block M after the collision moves
C
vertical side of C, both with the 0.5 m up and comes to rest. Calculate the coefficient of
same uniform speed. A restitution between the blocks and the mass of the block M.
P-58 Topic-wise Solved Papers - PHYSICS
1. The speed of the block at point B immediately after it strikes
[Take sin q » tan q = 0.05 and g = 10m / s 2 . ] the second incline is –
(1999 - 10 Marks)
A
M
3m
2kg 60° B
6.0 m v
3m 30°
q
30° C
13. A spherical ball of mass m is kept at the highest point in the
space between two fixed, concentric spheres A and B (see 3m 3 3m
figure). The smaller sphere A has a radius R and the space
(a) 60 m/s (b) 45 m/s
between the two spheres has a width d. The ball has a
diameter very slightly less than d. All surfaces are (c) 30 m/s (d) 15 m/s
frictionless. The ball is given a gentle push (towards the 2. The speed of the block at point C, immediately before it
right in the figure). The angle made by the radius vector of leaves the second incline is
the ball with the upward vertical is denoted by q (shown in
the figure). (2002 - 5 Marks) (a) 120 m/s (b) 105 m/s
(c) 90 m/s (d) 75 m/s
3. If collision between the block and the incline is completely
elastic, then the vertical (upward) component of the velocity
Sphere B of the block at point B, immediately after it strikes the second
incline is –
q (a) 30 m/s (b) 15 m/s
d (c) 0 (d) - 15 m/s
O
R
PASSAGE-2
A small block of mass 1 kg is released from rest at the top of a
Sphere A rough track. The track is a circular arc of radius 40 m. The block
slides along the track without toppling and a frictional force acts
on it in the direction opposite to the instantaneous velocity. The
(a) Express the total normal reaction force exerted by the
sphere on the ball as a function of angle q. work done in overcoming the friction up to the point Q, as shown
in the figure below, is 150 J.
(b) Let NA and NB denote the magnitudes of the normal
reaction forces on the ball exerted by the sphere A and (Take the acceleration due to gravity, g = 10 ms–2)
B, respectively. Sketch the variations of NA and NB as
y
functions of cos q in the range 0 £ q £ p by drawing
two separate graphs in your answer book, taking cos
q on the horizontal axes.
COMPREHENSION BASED Q UESTIONS : R P
30°
PASSAGE-1
A small block of mass M moves on a frictionless surface of an
inclined plane, as shown in figure. The angle of the incline suddenly
changes from 60° to 30° at point B. The block is initially at rest at Q R
A. Assume that collisions between the block and the incline are
totally inelastic (g = 10 m/s2). (2008)
M x
A O
v
60° B (JEE Adv. 2013)
4. The magnitude of the normal reaction that acts on the block
at the point Q is
(a) 7.5 N (b) 8.6 N
30° C (c) 11.5 N (d) 22.5 N
5. The speed of the block when it reaches the point Q is
3m 3 3m (a) 5 ms–1 (b) 10 ms–1
(c) 10 3 ms -1 (d) 20 ms–1
WORK, ENERGY AND POWER P-59
2. Three objects A, B and C are kept in a straight line on a
ASSERTION & REASON TYPE QUESTIONS : frictionless horizontal surface. These have masses m, 2m
1. STATEMENT–1 : A block of mass m starts moving on a and m, respectively. The object A moves towards B with a
rough horizontal surface with a velocity v. It stops due to speed 9 m/s and makes an elastic collision with it. There
friction between the block and the surface after moving after, B makes completely inelastic collision with C. All
through a certain distance. The surface is now tilted to an motions occur on the same straight line. Find the final speed
angle of 30° with the horizontal and the same block is made (in m/s) of the object C.
to go up on the surface with the same initial velocity v. The
decrease in the mechanical energy in the second situation
is smaller than that in the first situation. m 2m m
STATEMENT–2 : The coefficient of friction between the
block and the surface decreases with the increase in the A B C
angle of inclination. (2007)
(a) Statement–1 is True, Statement–2 is True; Statement– (2009)
2 is a correct explanation for Statement–1 3. A block of mass 0.18 kg is attached to a spring of force-
(b) Statement–1 is True, Statement–2 is True; Statement– constant 2 N/m. The coefficient of friction between the block
2 is NOT a correct explanation for Statement–1 and the floor is 0.1. Initially the block is at rest and the
(c) Statement–1 is True, Statement–2 is False spring is un-stretched. An impulse is given to the block as
(d) Statement–1 is False, Statement–2 is True shown in the figure. The block slides a distance of 0.06 m
and comes to rest for the first time. The initial velocity of the
INTEGER VALUE CORRECT TYPE :
block in m/s is V = N/10. Then N is (2011)
1. A light inextensible string that goes over a
smooth fixed pulley as shown in the figure
connects two blocks of masses 0.36 kg and
0.72 kg. Taking g = 10 m/s2, find the work
4. A particle of mass 0.2 kg is moving in one dimension under a
done (in joules) by the string on the block
force that delivers a constant power 0.5 W to the particle. If
of mass 0.36 kg during the first second after
the system is released from rest. (2009) the initial speed (in ms–1) of the particle is zero, the speed
(in ms–1) after 5 s is (JEE Adv. 2013)

1. Consider the following two statements : [2003] 4. A body is moved along a straight line by a machine
A. Linear momentum of a system of particles is zero delivering a constant power. The distance moved by the
body in time ‘t’ is proportional to [2003]
B. Kinetic energy of a system of particles is zero. 3/4 3/2
(a) t (b) t
Then
(c) t 1/4 (d) t 1/2
(a) A does not imply B and B does not imply A
5. A particle moves in a straight line with retardation
(b) A implies B but B does not imply A proportional to its displacement. Its loss of kinetic energy
(c) A does not imply B but B implies A for any displacement x is proportional to [2004]
(d) A implies B and B implies A (a) x (b) e x

2. A wire suspended vertically from one of its ends is stretched (c) x2 (d) loge x
by attaching a weight of 200N to the lower end. The weight 6. A uniform chain of length 2 m is kept on a table such that a
stretches the wire by 1 mm. Then the elastic energy stored length of 60 cm hangs freely from the edge of the table. The
in the wire is [2003] total mass of the chain is 4 kg. What is the work done in
(a) 0.2 J (b) 10 J pulling the entire chain on the table ? [2004]
(a) 12 J (b) 3.6 J
(c) 20 J (d) 0.1 J
(c) 7.2 J (d) 1200 J
3. A spring of spring constant 5 × 103 N/m is stretched initially r r r r
7. A force F = (5i + 3 j + 2k ) N is applied over a particle
by 5cm from the unstretched position. Then the work r r r
required to stretch it further by another 5 cm is [2003] which displaces it from its origin to the point r = (2i - j )m.
(a) 12.50 N-m (b) 18.75 N-m The work done on the particle in joules is [2004]
(c) 25.00 N-m (d) 6.25 N-m (a) +10 (b) +7
(c) –7 (d) +13
P-60 Topic-wise Solved Papers - PHYSICS
8. A body of mass ‘m’, accelerates uniformly from rest to ‘v1’
3
in time ‘t1’. The instantaneous power delivered to the body (a) (b)
2 2
as a function of time ‘t’ is [2004]

mv1t 2 mv12t 1
(a) (b) (c) (d) 2
t1 t12 2
15. A 2 kg block slides on a horizontal floor with a speed of 4m/s.
mv1t mv12t It strikes a uncompressed spring, and compresses it till the
(c) t1 (d) block is motionless. The kinetic friction force is 15N and
t1
spring constant is 10,000 N/m. The spring compresses by
9. A Particle is acted upon by a force of constant magnitude (a) 8.5 cm (b) 5.5 cm [2007]
which is always perpendicular to the velocity of the particle, (c) 2.5 cm (d) 11.0 cm
the motion of the particles takes place in a plane. It follows 16. An athlete in the olympic games covers a distance of 100 m
that [2004] in 10 s. His kinetic energy can be estimated to be in the
(a) its kinetic energy is constant range [2008]
(b) its acceleration is constant (a) 200 J - 500 J (b) 2 × 105 J - 3 × 105 J
(c) its velocity is constant
(c) 20, 000 J - 50,000 J (d) 2,000 J - 5, 000 J
(d) it moves in a straight line
17. A block of mass 0.50 kg is moving with a speed of 2.00 ms–1
10. The block of mass M moving on the frictionless horizontal
on a smooth surface. It strikes another mass of 1.00 kg and
surface collides with the spring of spring constant k and
then they move together as a single body. The energy loss
compresses it by length L. The maximum momentum of
during the collision is [2008]
the block after collision is [2005]
(a) 0.16 J (b) 1.00 J
M (c) 0.67 J (d) 0.34 J
18. The potential energy function for the force between two
atoms in a diatomic molecule is approximately given by
kL2 a b
(a) (b) Mk L U(x) = 12 - 6 , where a and b are constants and x is the
2M x x
distance between the atoms. If the dissociation energy of
ML2
(c) (d) zero the molecule is D = éëU ( x = ¥) - U at equilibrium ùû , D is
k
11. A spherical ball of mass 20 kg is stationary at the top of a hill
of height 100 m. It rolls down a smooth surface to the ground, b2 b2
(a) (b) [2010]
then climbs up another hill of height 30 m and finally rolls 2a 12a
down to a horizontal base at a height of 20 m above the
ground. The velocity attained by the ball is [2005] b2 b2
(a) 20 m/s (b) 40 m/s (c) (d)
4a 6a
(c) 10 30 m/s (d) 10 m/s 19. At time t = 0 a particle starts moving along the x-axis. If its
12. A body of mass m is accelerated uniformly from rest to a kinetic energy increases uniformly with time ‘t’, the net
speed v in a time T. The instantaneous power delivered to force acting on it must be proportional to [2011RS]
the body as a function of time is given by [2005] (a) constant (b) t
1
mv 2 mv 2 (c) (d) t
(a) .t 2 (b) .t t
2 2
T T
20. This question has Statement 1 and Statement 2. Of the four
1 mv 2 2
1 mv 2 choices given after the Statements, choose the one that
(c) .t (d) .t best describes the two Statements.
2 T2 2 T2
If two springs S1 and S2 of force constants k1 and k2 ,
13. A particle of mass 100g is thrown vertically upwards with a respectively, are stretched by the same force, it is found
speed of 5 m/s. The work done by the force of gravity that more work is done on spring S1 than on spring S2.
during the time the particle goes up is [2006] Statement 1 : If stretched by the same amount work done
(a) –0.5 J (b) –1.25 J on S1
(c) 1.25 J (d) 0.5 J Statement 2 : k1 < k2
14. The potential energy of a 1 kg particle free to move along (a) Statement 1 is false, Statement 2 is true.
æ x 4 x2 ö (b) Statement 1 is true, Statement 2 is false.
the x-axis is given by V ( x) = ç - ÷ J. (c) Statement 1 is true, Statement 2 is true, Statement 2 is
è 4 2ø the correct explanation for Statement 1
The total mechanical energy of the particle is 2 J. Then, the (d) Statement 1 is true, Statement 2 is true, Statement 2 is
maximum speed (in m/s) is [2006] not the correct explanation for Statement 1
WORK, ENERGY AND POWER P-61

Solutions & Explanations


Section-A : JEE Advanced/ IIT-JEE

C 1. (b) 2. (c) 3. (b) 4. (b) 5. (c) 6. (d)


7. (b) 8. (b) 9. (b) 10. (c) 11. (c) 12. (b)
13. (c) 14. (d)
D 1. (c) 2. (d) 3. (c, d) 4. (c) 5. (d) 6. (a, b)
E 1. Less than 2. 3 times 3. (i) 14 m/s; (ii) Same height 4. 25%
5. No 6. vC = 2 g ( y - mx) ; vF = 2 g ( y - mx) 7. 9.33 Cal.
8. 10 kg, 0.098 J 9. 4.24 m 10. 3.29 m/s 11. 4
12. 0.84; 15.02 kg
13. (a) NA = mg (3cosq – 2)

(b) For q £ cos -1 çæ ÷ö ; N B = 0, N A = mg (3cos q - 2)


2
è3ø

For q > cos-1 çæ ÷ö ; N A = 0, N B = mg (2 - 3cos q)


2
è3ø
G 1. (b) 2. (b) 3. (c) 4. (a) 5. (b)
H 1. (c)
I 1. 8J 2. 4 m/s 3. 5

Section-B : JEE Main/ AIEEE

1. (c) 2. (d) 3. (b) 4. (b) 5. (c) 6. (b) 7. (b) 8. (b)


9. (a) 10. (b) 11. (b) 12. (b) 13. (b) 14. (a) 15. (b) 16. (d)
17. (c) 18. (c) 19. (c) 20. (b)

MCQ's WITH ONE CORRECT ANSWER : dv


So, tangential acceleration, at = = kr
dt
Output Work Work is done by tangential force.
1. (b) Mechanical efficiency =
Input energy Power = Ft .v.cos 0° = (m at ) (krt )
The output work will increase because the friction
becomes less. Thus the mechanical efficiency increases. = (mkr ) (krt )

p2 = mk 2 r 2 t
2. (c) K.E. = ALTERNATE SOLUTION
2m
The centripetal acceleration
p12 p22
E1 = E2 \ = v2
m1 m2 ac = k2 r t2 Þ = k 2 rt 2
r
p12 ml p m1 1 1
\ = Þ l = = = 1 2 m 2 22
mv = k r t
p22 m2 p2 m2 4 2 Þ ... (i)
2 2
3. (b) The centripetal acceleration
m 2 22 d
2 Þ K.E. = k r t Þ (K.E.) = mk 2 r 2 t
v 2 dt
ac = k 2 r t 2 or = k 2 rt 2
r Þ Power = mk2r2t
\ v = krt
P-62 Topic-wise Solved Papers - PHYSICS
4. (b) KEY CONCEPT ALTERNATE SOLUTION
The force constant of a spring is inversely proportional The above situation can also be looked upon as the decrease
to the length of the spring. in the gravitational potential energy of spring mass system
Let the original length of spring be L and spring is equal to the gain in spring elastic potential energy.
constant is K (given)
Therefore, 1 2 2Mg
Mgx = kr , x =
2L 3 2 k
K×L= ×K' Þ K'= K 8. (b) Note : In a conservative field work done does not
3 2
5. (c) Number of particles string the blades/time µ velocity depend on the path. The gravitational field is a
of wind conservative field.
K.E. of particle µ (Velocity of wind)2 \ W1 = W2 = W3
\ Power output µ (No. of particles striking/time) × (K.E. 9. (b) We know that DU = – W for conservative forces
of particle) x x
\ Power output µ v3 DU = - ò Fdx or DU = - ò k xdx
0 0
ALTERNATE SOLUTION
æ dm ö d d kx 2
F = v ç ÷ = v (r ´ Volume) = vr (Volume) Þ U(x) – U(0) = -
è dt ø dt dt 2
= vr × (Av) = Arv2 Given U(0) = 0
Power = Force × Velocity = Arv2 × v = Arv3
Þ P µ v3 kx 2
U(x) = -
6. (d) dU(x) = – Fdx 2
x kx 2 ax 4 10. (c) When the block B is displaced towards wall 1, only
\ Ux = - ò Fdx = - spring S1 is compressed and S2 is in its natural state.
0 2 4 This happens because the other end of S2 is not attached
2k to the wall but is free. Therefore the energy stored in
U = 0 at x = 0 and at x = ; Þ we have potential
a 1
the system = k1 x2 . When the block is released, it will
energy zero twice (out of which one is at origin). 2
Also, when we put x = 0 in the given function, come back to the equilibrium position, gain momentum,
overshoot to equilibrium position and move towards
dU
we get F = 0. But F = - wall 2. As this happens, the spring S1 comes to its natural
dx length and S2 gets compressed. As there are no frictional
dU forces involved, the P.E. stored in the spring S1 gets
Þ At x = 0; = 0 i.e. the slope of the graph
dx stored as the P.E. of spring S2 when the block B reaches
should be zero. These characteristics are represented its extreme position after compressing S2 by y.
by (d). 1
1
7. (b) Let the maximum extension of the spring be x as shown \ k1 x2 = k2 y2
in the figure. Work is done by the gravitational and the 2 2
spring force. There is no change in the kinetic energy 1 1
between the initial and final position of the mass. × kx2 = 4 ky2, x2 = 4y2
2 2

y 1
\ =
x 2
11. (c) Let the radius of the circle be r. Then the two distance
travelled by the two particles before first collision is
x 2pr. Therefore v
A
2v
2v ´ t + v ´ t = 2 pr
where t is the time taken
for first collision to occur.
From Work-energy theorem; 2 pr
Wg + Ws = 0 \ t=
where Wg = work done by gravity 3v
and Ws = work done by spring \ Distance travelled by particle with velocity v is
1 2 2pr 2pr
Þ + Mgx – kx = 0 equal to v ´ = .
2 3v 3
2 Mg Therefore the collision occurs at B.
Þ x=
k
WORK, ENERGY AND POWER P-63

A
Area of D AOB F(t)
1 A
= ´ 3´ 4 = 6 N - s 4N
2v
2
120°
OA CD B 4.5 s
Also = O
B OB CB 3s C t

v 4 CD D
Þ = f
As the collision is elastic and the particles have equal 3 1.5
masses, the velocities will interchange as shown in the Þ CD = 2
figure. According to the same reasoning as above, the é1 ù
2nd collision will take place at C and the velocities will \ Area of DBCD = = - ê ´1.5 ´ 2 ú = -1.5 N-s
again interchange. ë 2 û
\ The final linear momentum = 6 – 1.5 = 4.5 N-s
A
p 2 (4.5)2
\ Kinetic energy of the block = = = 5.06 J
2m 2 ´ 2
v 14. (d) Let us consider a point on the circle
120° The equation of circle is x2 + y2 = a2
B C The force is
r é xiˆ yjˆ ù
2v F= Kê 2 2 3/ 2
+
2 2 3/ 2 ú
With the same reasoning the 3rd collision will occur at êë (x + y ) (x + y ) úû
the point A. Thus there will be two elastic collisions y
before the particles again reach at A.
12. (b) (0, a) ds
Y
P(x, y)
Ncosq
F
N yjˆ
q
x
ma Nsinq xiˆ (a,0)
P

mg r é xiˆ yjˆ ù
F = K ê 2 3/ 2 + 2 3 / 2 ú
q ëê (a ) (a ) ûú
X r K
The forces acting on the bead as seen by the observer
F = 3 éë xiˆ + yjˆùû
a
in the accelerated frame are : (a) N ; (b) mg ; (c) ma The force acts radially outwards as shown in the figure
(pseudo force). and the displacement is tangential to the circular path.
Let q is the angle which the tangent at P makes with the Therefore the angle between the force and displacement
X- axis. As the bead is in equilibrium with respect to the is 90° and W = 0
wire, therefore option (d) is correct.
N sin q = ma and N cos q = mg
a
\ tan q = … (i)
g 1 2
But y = k x2. Therefore, mv
E 2
1. (c) P = = constt \ = constt
dy t t
= 2kx = tan q … (ii)
dx
v2
From (i) & (ii) \ = constt (k )
t
a a
2kx = Þ x= ds
g 2kg \ v = kt1/2 \ = kt1/ 2
dt
13. (c) Area under F – t graph gives the impulse or the change
in the linear momentum of the body. As the initial or, ds = kt1/ 2 dt
velocity (and therefore the initial linear momentum) of By integrating, we get
the body is zero, the area under F – t graph gives the
final linear momentum of the body. 2kt 3/ 2
Þ s= + C Þ s µ t3/2
3
P-64 Topic-wise Solved Papers - PHYSICS
2. (d) The hanging part of the chain which is to be pulled up Note : The velocity changes continuously due to
can be considered as a point mass situated at the centre change in the direction. The acceleration also changes
of the hanging part. The equivalent diagram is drawn. continuously due to change in direction.
Note : The work done in bringing the mass up will be 4. (c) The expression of work done by the variable force F on
equal to the change in potential energy of the mass. the particle is given by
ur uur
W = ò F .d l
In going from (0, 0) to (a, 0), the coordinate of x varies
from 0 to 'a', while that of y remains zero. Hence, the
work done along this path is :

[Q $j . $i = 0 ]
a
ò 0 ( - Kx $j ). dx $i = 0
T
L/6 = distance of the centre W1 =
M_g from top of the table.
3
In going from (a, 0) to (a, a) the coordinate of x remains
W = Change in potential energy constant (= a) while that of y changes from 0 to 'a'.
Hence, the work done along this path is
M L MgL
´g´ =
= mgh = a a
ò 0 [(- K ( yi$ + a $j). dy $j ] = ka ò 0 dy = - Ka
3 6 18 2
W2 =
ALTERNATE SOLUTION
M Y
The mass per unit length of the chain = (a, a)
L
Let us consider length x of the chain, see fig.
To move this length upwards, a force equal to the dy
weight of chain of length x will have to be applied
upwards.
-dx O (a, 0) X
dx
x
Hence, W = W1 + W2 = – ka2
5. (d) Applying the principle of conservation of energy
æM ö (K.E.)B + (P.E.)B = (K.E.)A + (P.E.)A, v
Weight of chain of length x is ç x÷ g
è L ø we get
Small amount of work done in moving the length dx 1 2 1
upwards is mv + mgL = mu 2
2 2
ur ur æM ö
dW = F . dx = Fdx = ç x÷ g dx
è L ø Hence, v = u 2 - 2 gL ... (i)
u
The total amount of work done in moving the one third r r
length of the hanging chain on the table will be 2 2
Change in velocity = | v - u | = v + u
L/3
L/3 M M L/3 M é x2 ù 2
= 2(u - g l) [From (i)]
W= ò0 L
xgdx =
L

0
xdx = gê ú
L êë 2 úû 6. (a, b) At point A, potential energy of the ball = mghA
0
At point B, potential energy of the ball = 0
MgL At point C, potential energy of the ball = mghC
= Total energy at point A, EA = KA + mghA
18
3. (c,d) When the force is perpendicular to the velocity and Total energy at point B, EB = KB
constant in magnitude then the force acts as a centripetal Total energy at point C, EC = KC + mghC
force, and the body moves in a circular path. The force According to the law of conservation of energy.
is constant in magnitude, this show the speed is not EA = EB = EC ... (i)
changing and hence kinetic energy will remain constant. EA = EB Þ EC > KC ...(ii)
v EA = EC
KA + mghA = KB + mghC
F K - KA
v or, h A - h C = C ...(iii)
mg
Þ hA > hC ;KC > KA ...(iv)
F Option (b) is correct
F From (i),(ii) and (iv), we get hA> hC; KB > KC.Option (a) is
v correct.
v
WORK, ENERGY AND POWER P-65

SUBJECTIVE PROBLEMS : A B
1. KEY CONCEPT
By conservation of linear momentum
mv = MV ... (i) v1 v2
1 2 v
K.E. of bullet = mv
2
v2
1 2
K.E. of rifle = MV
2

1 æ mv ö
2 Again applying conservation of linear momentum for
= Mç ÷ from (i) collision at B.
2 èMø 0.02 v1 = (2.98 + 0.02) v2 = 3v2

1 m2 2 æ 1 2 ö é m ù m 0.02 v1
= v = ç mv ÷ ê ú Since, <1 Þ v2 = ... (ii)
2 M è2 ø ëM û M 3
From (i) and (ii)
\ K.E. of rifle < K.E. of bullet.
ALTERNATE SOLUTION 0.02 v1 4 v 4
0.02 v = 0.02 v1 + , v = v1 Þ =
3 3 v1 3
p2
K.E. =
2m v1 3 v 3 1
= Þ 1 - 1 = 1 - = = 0.25
v 4 v 4 4
1
For equal value of p linear momentum, K.E. µ
mass v - v1
Þ = 0.25
2. KEY CONCEPT v
For a spring, (spring constant) × (length) = Constant 1
If length is made one third, the spring constant becomes \ % loss in velocity = ´ 100 = 25%
4
three times.
5. No. An external force, the gravitational pull of earth, is acting
3. (i) u = v0, v = ? a = g, s = 10 m on the ball which is responsible for the change in momentum.
v2 – u2 = 2as 6. (a) K.E. at C = Loss in P.E. – Work done by friction.
Þ v2 – v02 = 2 × g × 10 Þ v = v02 + 20 g A

1
\ K.E. at B = m(v02 + 20 g )
2
50% of energy is lost. Therefore energy left q(
B C

1 æ v02 + 20 g ö
= mç ÷ 1 2
2 è 2 ø mvc = mg y - mmg cos q ´ AC
2
The ball rises to the same height. Therefore, 1 2 BC
\ vc = g y - mg ´ AC = gy - mgx
1 æ v02 + 20 g ö 2 AC
\ mç ÷ = mg ´ 10
2 è 2 ø \ v c = 2 g ( y - mx )
K.E. at F = loss in P.E. – Work done by friction
v02 + 20 g = 40 g
1 2
mvF = mgy - mmg cos a DG - mmg cos b GF
Þ v0 = 20g = 14 ms–1 2
(ii) Same height because if the initial velocity v0 was 1 2 GM FN
directed upwards from A then when the ball again vF = gy - mg ´ DG - mg ´ GF
2 DG GF
reaches back to A the velocity v0 is downward at A.
4. Let v be the velocity of bullet before striking A. Applying 1 2
\ vF = gy - mg (GM + FN )
conservation of linear momentum for the system of bullet 2
and plate A, we get
\ vF = 2 g ( y - mx )
0.02v = 0.02 v1 + 1 × v2
P-66 Topic-wise Solved Papers - PHYSICS
The P.E. stored in spring is given by
D
1 2 1 19.6 19.6
U= kx = ´ k ´ ´
M a(
G 2 2 k k
19.6 ´ 19.6
= = 0.098 J
b(
2 ´ 1960
9. K.E. of block = work against friction + P.E. of spring
E N F
1 2 1
Note : The result does not depend on the angles a and b. It mv = µk mg (2.14 + x) + kx2
2 2
only depends on the values of x and y.
7. Heat generated = Work done by the frictional force 1 1
× 0.5 × 32 = 0.2 × 0.5 × 9.8 (2.14 + x) + 2 × x2
= f×s 2 2
= µmg × (v × t) = 0.2 × 2 × 9.8 × 2 × 5 2.14 + x + x2 = 2.25
= 39.2 J \ x2 + x – 0.11 = 0
11 1
39.2 On solving, we get x = – or x = = 0.1 (valid answer)
= cal = 9.33cal. 10 10
4.2
Here the body stops momentarily.
8. Since the two blocks A and B are moving with constant Restoring force at Y = kx = 2 × 0.1 = 0.2 N
velocity, therefore, the net force acting on A is zero and the Frictional force at Y = µs mg = 0.22 × 0.5 × 9.8 = 1.078 N
net force acting on B will be zero. Since the spring is loaded, Since frictional force > restoring force, the body will stop here.
it will be in a deformed state. Let the extension of the spring \ The total distance travelled
be x. = AB + BD + DY = 2 + 2.14 + 0.1 = 4.24 m.
v
N
T
B
C f T A B D C
mBg
2m
2.14m
T v x
A

mg A

The forces are drawn. Y


Note : There will be no friction force between block A and C
10. When mass m is released, since M > m, the mass M will
Q f = µN. Here there is no normal reaction on A move on a dotted path with O as the centre. There will be
(because A is not pushing C) decrease in the potential energy of M which will be converted
Applying Fnet = ma on A, we get into kinetic energy of M, and increase in potential energy
mAg – T = mA × 0 of m.
\ T = mAg ... (i)
M
Applying Fnet = ma on B, we get O 1m
A
T – f = mB × 0 1m q
\ T = f = µN
= µmBg ... (ii) 1m
From (i) and (ii) 5 m
B
µmBg = mAg v
q C
mA 2 v
Þ mB = = = 10 kg
m 0.2 Decrease in P.E. of M is Mgh
Here the force acting on the spring is the tension (equal to = 2 × 9.8 × 1 = 19.6 J
restoring force) 1
K.E. of M = MV 2
T 2
\ T = kx \ x= (Let V be the velocity attained by M just before striking the
k
wall)
19.6 1 2
\ x= [Q T = 2 × 9.8 = 19.6 N from (i)] K.E. of m = mv
k 2
WORK, ENERGY AND POWER P-67
From the figure, by velocity constraint n n
æ 2 ö æ 2 ö 1
v = V cos q
O 2 çè ÷ø ´ 2 g l = g l Þ ç ÷ =
From DOAC, A 5 è 5ø 2
q
Taking log on both sides we get
2
cos q = æ 2 ö 1
5 1 n log ç = log Þ n = 3.1
5 è 5ø ÷
2
2V Therefore, number of collisions will be 4.
\ v=
5 C 12. From A to B.
(OC + CA) – OA = height attained by m u = 10 m/s (given)
é mg sin q + f ù é mg sin q + mmg cos q ù
1 + 22 + 12 - 2 = height attained by m = 5 -1 a= -ê ú = -ê úû
ë m û ë m
\ Increase in P.E. of m = mgh' = 0.5 × 9.8 ( 5 - 1 )
NOTE THIS STEP
By the principle of energy conservation M C
1 1
Mgh = MV 2 + mv 2 + mgh ' uA B
2 2
N
1 2 1 2 m
= MV + m(V cos q) + mgh ' q mg cosq
2 2 mg sinq A
f mg
q
1 1 4V 2 = – [g sin q + µg cos q ] = – g [sin q + µ cos q ]
\ 19.6 = ´ 2 ´ V 2 + 0.5 ´ + 0.5 × 9.8 ( 5 - 1 )
2 2 5 = – 10 [0.05 + 0.25 × 0.99]
On solving, we get V = 3.29 m/s = – 2.99 m/s2
11. The pendulum bob is left from position A. When it is at v=?
position C, the angular amplitude is 60°. s=6m
In DOCM v2 – u2 = 2as Þ v2 = 100 + 2 (– 2.99) × 6
Þ v = 8 m/s
Þ The velocity of mass m just before collision is 8 m/s.
The velocity of mass M just before collision is 0 m/s (given).
AFTER COLLISION
Let v1 be the velocity of mass m after collision and v2 be the
velocity of mass M after collision. Body of mass M moving
from B to C and coming to rest.
u = v2
v=0
a = – 2.99 m/s2
(same as of previous case because all other things are same
except mass. a is independent of mass)
OM l
cos 60° = Þ OM = s = 0.5
l 2
The velocity of bob at B, vB before first collision is v2 – u2 = 2as Þ (0)2 – v22 = 2 (– 2.99) × 0.5
Þ v2 = 1.73 m/s
1 2 Body of mass m moving from B to A after collision
mg l = mvB Þ vB = 2gl
2
Let after n collisions, the angular amplitude is 60° when the
bob again moves towards the wall from C, the velocity v'B B
before collision is 6 h
S=
l 1 q
mg = mv '2B Þ v'B = gl A
2 2
This means that the velocity of the bob should reduce from
h
2gl to gl due to collisions with walls. sin q =
6
The final velocity after n collisions is gl h = 6 sin q = 6 × 0.05
u = v1
\ en ( 2 g l ) = g l v = + 1m/s
where e is coefficient of restitution. (K.E. + P.E.)initial = (K.E. + P.E.)final + Wfriction
P-68 Topic-wise Solved Papers - PHYSICS

1 2 1 The graph between NA and cos q


mv1 + mgh = mv 2 + 0 + m mgs
2 2 From equation (iii) when q = 0, NA = mg.

1 2 1
v1 + 10 ´ (6 ´ 0.05) = (1) 2 + 0.25 ´ 10 ´ 6 NA NB
2 2 2mg
v1 = – 5 m/s 5mg
\ Coefficient of restitution
mg
Re lative velocity of separation
e= 2mg
Re lative velocity of approach
1 -1 +1
-5 - 1.73 cosq
= = 0.84
8-0
On applying conservation of linear momentum before and æ 2ö
When q = cos–1 ç ÷ ; NA = 0
after collision, we get è 3ø
2 × 8 + M × 0 = 2 × (– 5) + M (1.73)
The graph is a straight line as shown.
26
\ M= = 15.02 kg æ 2ö
1.73 when θ > cos -1 ç ÷ ;
è 3ø
13. The ball is moving in a circular motion. The necessary
centripetal force is provided by (mg cos q – N). Therefore, mv 2
NB – (mg cos q ) =
d
R+
2
B
C
d/2 NA
A mv 2
D Þ NB + mg cos q = ... (iv)
mgcosq V æ dö
q
mgsinq çè R + ÷ø
q 2
R
mg
Using energy conservation
1 2 éæ dö æ dö ù
mv = mg ê ç R + ÷ - ç R + ÷ cos q ú
2 ë è 2 ø è 2 ø û

mv 2
= 2mg [1 – cos q ] ... (v)
æ dö
çè R + ÷

mv 2
mg sin q – NA = ... (i)
æ dö From (iv) and (v), we get
çè R + ÷ø
2 NB + mg cos q = 2mg – 2mg cos q
According to energy conservation NB = mg (2 – 3 cos q )

1 2 æ dö 2
mv = mg ç R + ÷ (1 - cos q) ... (ii) When cos q = , NB = 0
2 è 2ø 3
From (i) and (ii) When cos q = – 1, NB = 5 mg
NA = mg (3 cos q – 2) ... (iii) Therefore the graph is as shown.
The above equation shows that as q increases NA decreases. COMPREHENSION BASED Q UESTIONS :
At a particular value of q , NA will become zero and the ball
1. (b) As the inclined plane is frictionless,
will lose contact with sphere A. This condition can be found The K. E . at B = P.E. at A
by putting NA = 0 in eq. (iii)
0 = mg (3 cos q – 2) 1 2
mv = mgh
2
æ 2ö
\ q = cos–1 ç ÷ v = 2 gh
è 3ø
WORK, ENERGY AND POWER P-69
Also since the collision is elastic, the vertical
A
component of velocity (v sin 30°) before collision
changes in direction, the magnitude remaining the same
h 30° as shown in the figure. So the rectangular components
of velocity after collision are as shown in the figure.
60° B This means that the final velocity of the block should
D
v cos 30° be horizontal making an angle 30º with BC. Therefore
30° 30° the vertical component of the final velocity of the block
v sin 30° v = 2gh is zero.
30°
E
C 4. (a)
3m 3 3m 30°
40 sin 30°
h 40m
In D ADB, tan 60° =
3
N
\h=3m \ v = 6g = 60 m / s
This is the velocity of the block just before collision. mg sin 60°
This velocity makes an angle of 30° with the vertical.
60° v
Also in right angled triangle BEC, ÐEBC= 60°.
Therefore v makes an angle of 30° with the second mg cos 60° mg
inclined plane BC. The component of v along BC is v
cos 30°.
It is given that the collision at B is perfectly inelastic mv2
therefore the impact forces act normal to the plane such N – mg cos 60° =
r
that the vertical component of velocity becomes zero.
The component of velocity along the incline BC mv2
remains unchanged and is equal to v cos 30° \ N = mg cos 60° + ...(1)
r
3 180 Loss in P.E. = mg × 40 sin 30° = 200 J
= 60 ´ = = 45 m / s
2 4 Work done in over coming friction = 150 J
BE 1 BE \ K.E. possessed by the particle = 50 J
2. (b) In D BCE, tan30° = Þ = Þ BE = 3m
CE 3 3 3 1
\ mv 2 = 50J
Applying mechanical energy conservation . 2
Mechanical energy at B = Mechanical energy at C \ mv2 = 100 J ...(2)
1
2
M ( 45) 2 + M ´ 10 ´ 3 = 12 Mvc2 From (1) and (2), N = 1 × 10 ×
1 100
+ = 5 + 2.5 = 7.5 N
2 40
45 + 60 = vc 2 \ vc = 105 m / s (a) is the correct option.
3. (c) The velocity of the block along BC just before collision 5. (b) From (2), mv2 = 100
is v cos 30°. The impact forces act perpendicular to the \ v = 10 ms–1
surface so the component of velocity along the incline (b) is the correct option.
remains unchanged.
ASSERTION & REASON TYPE QUESTIONS :

1. (c) Statement 1 : In the first case the mechanical energy is


completely converted into heat becuase of fiction. While
v sin 30° is second case, a part of mechanical energy is
converted into heat due to fiction but another part of
mechanical energy is retained in the form of potential
v
B 30° energy of the block.
v co Therefore statement 1 is correct.
s 30
°
Statement 2 : This is a wrong statement because the
30° C coefficient of friction between the block and the surface
Just after collision
does not depend on the angle of inclination.
P-70 Topic-wise Solved Papers - PHYSICS
3. 4
INTEGER VALUE CORRECT TYPE :
Let v be the speed of the block just after impulse. At B, the
1. Given m = 0.36 kg, M = 0.72 kg. block comes to rest. Therefore
The figure shows the forces on m and
M. When the system is released, let A
v
the acceleration be a. Then
T – mg = ma
Mg – T = Ma T
B
( M - m) g a m
\ a= = g /3 T
M +m
and T = 4 mg/3 mg M a Loss in K.E. of the block = Gain in P.E. of the spring
For block m : + Work done against friction
u = 0, a = g/3, t = 1, s = ? Mg 1 2 1 2
1 mv = kx + mmg .x
1 g 2 2
s = ut + at2 = 0 + ´ ´ 12 = g / 6
2 2 3
\ Work done by the string on m is k 2
\v= x + mgx .
ur r mg g 4 ´ 0.36 ´ 10 ´ 10 m
T .s = Ts = 4 ´ = = 8J
3 6 3´ 6
2
2. The velocity of B just after collision with A is \v= ´ 0.06 ´ 0.06 + 0.1 ´ 10 ´ 0.06
0.18
(mB - m A )uB 2m A u A
vB = +
mB + mA m A + mB 4
\v=
10
\N=4
m 2m m
A B C 4. 5 Here D K.E. = W = P × t
0 + 2m ´ 9 1
= = 6m/s \ mv 2 = P ´ t
m + 2m 2
The collision between B and C is completely inelastic.
2Pt 2 ´ 0.5 ´ 5
\ mB vB = (mB + mc) v \v = = = 5ms -1
m 0.2
6 ´ 2m
\ v= = 4m/s.
2m + m

1. (c) Kinetic energy of a system of particle is zero only when 4. (b) We know that F × v = Power
the speed of each particles is zero. And if speed of each \ F ´ v = c where c = constant
particle is zero, the linear momentum of the system of
particle has to be zero. dv æ mdv ö
\m ´v =c çè\ F = ma = ÷
Also the linear momentum of the system may be zero dt dt ø
even when the particles are moving. This is because v t
linear momentum is a vector quantity. In this case the 1 2
\ mò vdv = c ò dt \ mv = ct
kinetic energy of the system of particles will not be zero. 2
0 0
\ A does not imply B but B implies A.
2. (d) The elastic potential energy 2c 1 2
\ v= ´t
1 1 m
= ´ Force ´ extension = ´ 200 ´ 0.001 = 0.1 J
2 2 dx 2c 1 2 dx
\ = ´t where v =
3. (b) k = 5 ´ 10 N/m 3 dt m dt
x t
( ) 2c
1 1 1
W=
2
k x22 - x12 =
2
´ 5 ´ 103 é(0.1)2 - (0.05)2 ù
ë û
\ ò dx = m
´ ò t 2 dt
0 0
5000 3
= ´ 0.15 ´ 0.05 = 18.75 Nm 2c 2t 2 3
2 x= ´ Þ xµt 2
m 3
WORK, ENERGY AND POWER P-71
5. (c) Given : retardation µ displacement
i.e., a = - x 11. (b)

v2 x 100
vdv 30
But a = v
dv \
dx
= -x Þ ò v dv = -ò xdx 20
dx v1 0
mgH 1
mv 2 + mgh
2
(v2
2 )
- v12 = -
x2
2
Using conservation of energy,
æ1 2 ö
m (10 × 100) = m çè v + 10 ´ 20÷ø
1 æ - x2 ö
1
(
Þ m v22 - v12 = m ç
2 2 è 2 ø

2

1 2
or v = 800 or v = 1600 = 40 m/s
2
\ Loss in kinetic energy, \ DK µ x 2
ALTERNATE SOLUTION
Loss in potential energy = gain in kinetic energy
4
6. (b) Mass of over hanging chain m¢ = ´ (0.6)kg 1 1
2 m ´ g ´ 80 = mv 2 , 10 ´ 80 = v 2
Let at the surface PE = 0 2 2
2
v = 1600 or v = 40 m/s
C.M.of hanging part = 0.3 m below the table
12. (b) u = 0; v = u + aT; v = aT
4
U i = - m ¢gx = - ´ 0.6 ´ 10 ´ 0.30 Instantaneous power = F × v = m. a. at = m. a2 . t
2
DU = m ' gx = 3.6J = Workdone in putting the entire v2
\ Instantaneous power = m t
chain on the table. T2
7. (b) Workdone in displacing the particle, 1 2 1
rr 13. (b) K.E = mv = ´ 0.1 ´ 25 = 1.25 J
W = F . x = (5iˆ + 3 ˆj + 2kˆ).(2iˆ - ˆj) 2 2
= 10 – 3 = 7 joules æ1 ö
W = - mgh = - ç mv 2 ÷ = -1.25 J
8. (b) Let acceleration of body be a è2 ø

v1 é 1 2 ù
\ v1 = 0 + at1 Þ a =
t1 êëQ mgh = 2 mv by energy conservation úû
14. (a) Velocity is maximum when K.E. is maximum
v1t For minimum. P.E.,
\ v = at Þ v =
t1
dV
r r r = 0 Þ x 3 - x = 0 Þ x = ±1
Pinst = F .vr = (ma ).v dx
1 1 1
æ mv ö æ v t ö æv ö
2 Þ Min. P.E. = - =- J
= ç 1÷ ç 1 ÷ = m 1 t 4 2 4
è t1 ø è t1 ø çè t ÷ø
1 K.E.(max.) + P.E.(min.) = 2 (Given)
9. (a) Work done by such force is always zero since force is 1 9
\ K.E.(max.) = 2 + =
acting in a direction perpendicular to velocity. 4 4
\ from work-energy theorem = DK = 0 1 2
K.E.max . = mvmax .
K remains constant. 2
1 1 1 9 3
10. (b) Mv 2 = k L2 Þ 2
´ 1 ´ vmax . = Þ vmax. =
2 2 2 4 2
M 15. (b) Let the block compress the spring by x before stopping.
k kinetic energy of the block = (P.E of compressed spring)
Þv= .L
M + work done against friction.
k 1 1
Momentum = M × v = M × .L = kM . L ´ 2 ´ (4)2 = ´ 10,000 ´ x 2 + 15 ´ x
M 2 2
P-72 Topic-wise Solved Papers - PHYSICS
10,000 x2 + 30x – 32 = 0 dU ( x )
2 18. (c) At equilibrium : =0
Þ 5000 x + 15 x - 16 = 0 dx
-15 ± (15)2 - 4 ´ (5000)(-16) -12a -6b
1
\ x= = æ 2a ö 6
2 ´ 5000 Þ Þ x=ç ÷
x11 x5 è b ø
= 0.055m = 5.5cm.
16. (d) The average speed of the athelete a b b2
\ U at equilibrium = 2
- = - and U ( x=¥ ) = 0
100 1 2 æ 2a ö æ 2a ö 4a
v= = 10 m/s \ K.E. = mv ç ÷ ç ÷
10 2 è b ø è b ø
1 æ b2 ö b2
If mass is 40 kg then, K.E. = ´ 40 ´ (10) 2 = 2000 J
2 \ D = 0 -ç- ÷ =
è 4a ø 4a
1 (c) K.E. µ t or K. E. = ct
If mass is 100 kg then, K.E. = ´ 100 ´ (10) 2 = 5000 J 19.
2
17. (c) Initial kinetic energy of the system 1 2 p2
Þ mv = ct Þ = ct (Q p = mv)
1 1 1 2 2m
K.Ei = mu 2 + M (0)2 = ´ 0.5 ´ 2 ´ 2 + 0 = 1J
2 2 2 1
For collision, applying conservation of linear momentum Þ p = 2ctm Þ F = 2 cm ´
m × u = (m + M) × v 2 t

2 1
\ 0.5 ´ 2 = (0.5 + 1) ´ v Þ v = m/s Þ F µ
3 t
Final kinetic energy of the system is
1 2
1 1 2 2 1 20. (b) Q w= kx
K.E f = (m + M )v2 = (0.5 + 1) ´ ´ = J 2
2 2 3 3 3
1 1 2
æ 1ö w1 = k1x 2 ; w2 = k2 x
\ Energy loss during collision = ç1 - ÷ J = 0.67J 2 2
è 3ø Since w1 > w2 Thus (k1 > k2)
5 Momentum and Impulse

FILL IN THE BLANKS : 3. Two blocks of masses 10 kg and 4 kg are connected by a


spring of negligible mass and placed on a frictionless
1. A particle of mass 4 m which is at rest explodes into three horizontal surface. An impulse gives a velocity of 14 m/s to
fragments. Two of the fragments each of mass m are found the heavier block in the direction of the lighter block. The
to move with a speed v each in mutually perpendicular velocity of the centre of mass is (2002S)
directions. The total energy released in the process of (a) 30 m/s (b) 20 m/s
explosion is ............ (1987 - 2 Marks) (c) 10 m/s (d) 5 m/s
2. The magnitude of the force (in newtons) acting on a body 4. A ball of mass 0.2 kg rests on a vertical post of height 5 m. A
varies with time t (in micro seconds) as shown in the fig AB, bullet of mass 0.01 kg, traveling with a velocity V m/s in a
BC and CD are straight line segments. The magnitude of horizontal direction, hits the centre of the ball. After the
the total impulse of the force on the body from t = 4 ms to collision, the ball and bullet travel independently. The ball
t = 16ms is ...............Ns. (1994 - 2 Marks) hits the ground at a distance of 20 m and the bullet at a
distance of 100 m from the foot of the post. The velocity V
of the bullet is (2011)
C
800

600
Force (N)

400

200 A B
(a) 250 m/s (b) 250 2 m/s
E F D (c) 400 m/s (d) 500 m/s
0 2 4 6 8 10 12 14 16 5. A particle of mass m is projected from the ground with an
Time (m s) initial speed u0 at an angle a with the horizontal. At the
highest point of its trajectory, it makes a completely inelastic
collision with another identical particle, which was thrown
MCQ's WITH ONE CORRECT ANSWER : vertically upward from the ground with the same initial speed
u0. The angle that the composite system makes with the
1. An isolated particle of mass m is moving in horizontal plane horizontal immediately after the collision is
(x – y), along the x-axis, at a certain height above the ground, (JEE Adv. 2013)
it suddenly explodes into two fragments of masses m/4 and p p
3m/4. An instant later, the smaller fragment is at y = +15 cm. (a) (b) +a
The larger fragment at this instant is at (1997C - 1 Mark) 4 4

(a) y = -5 cm (b) y = +20 cm p p


(c) -a (d)
2 2
(c) y = +5 cm (d) y = – 20 cm
2. Two particles of masses m1 and m2 in projectile motion have
r r
velocities v1 and v2 respectively at time t = 0. They collide 1. A ball hits the floor and rebounds after an inelastic collision.
r r In this case (1986 - 2 Marks)
at time to. Their velocities become v1 ' and v2 ' at time 2to
while still moving in air. The value of (a) the momentum of the ball just after the collision is the
r r r r same as that just before the collision.
( m1v1 '+ m2v2 ') - ( m1v1 + m2v2 ) is (2001S) (b) the mechanical energy of the ball remains the same in
(a) zero (b) ( m1 + m2 ) gto the collision
(c) the total momentum of the ball and the earth is
1
(c)
2
( m1 + m2 ) gto (d) 2(m1 + m2)gto
conserved
(d) the total energy of the ball and the earth is conserved
P-74 Topic-wise Solved Papers - PHYSICS
2. A shell is fired from a cannon with a velocity v (m/sec.) at an (b) The time at which the particle passes through the
angle q with the horizontal direction. At the highest point m
in its path it explodes into two pieces of equal mass. One of equilibrium position for the first time is t = p
k
the pieces retraces its path to the cannon and the speed (in (c) The time at which the maximum compression of the
m/sec.) of the other piece immediately after the explosion is
(1986 - 2 Marks) 4p m
spring occurs is t =
(a) 3v cos q (b) 2v cos q 3 k
(d) The time at which the particle passes through the
3 3
(c) v cos q (d) v cos q
2 2 5p m
equilibrium position for the second time is t =
3. Two blocks A and B, each of mass m, are connected by a 3 k
massless spring of natural length L and spring constant K.
SUBJECTIVE PROBLEMS :
The blocks are initially resting on a smooth horizontal floor
with the spring at its natural length, as shown in fig.. A third 1. A body of mass m moving with velocity V in the X-direction
identical block C, also of mass m, moves on the floor with a collides with another body of mass M moving in Y-direction
speed v along the line joining A and B, and collides elastically with velocity v. They coalesce into one body during collision.
with A. Then (1993-2 Marks) Calculate : (1978)
v (i) the direction and magnitude of the momentum of the
L final body.
C A B (ii) the fraction of initial kinetic energy transformed into
heat during the collision in terms of the two masses.
(a) the kinetic energy of the A-B system, at maximum 2. A body of mass 1 kg, initially at rest, explodes and breaks
compression of the spring, is zero. into three fragments of masses in the ratio 1 : 1 : 3. The two
(b) the kinetic energy of the A-B system, at maximum pieces of equal mass fly off perpendicular to each other
compression of the spring, is mv2/4. with a speed of 30 m/sec each. What is the velocity of the
heavier fragment ? (1981- 3 Marks)
(c) the maximum compression of the spring is v (m / K ) 3. Three particles A, B and C of equal mass move with equal
speed V along the medians of an equilateral triangle as shown
(d) the maximum compression of the spring is v (m / 2K ) in figure. They collide at the centroid G of the triangle. After
r r r r the collision, A comes to rest, B retraces its path with the
4. The balls, having linear momenta p1 = pi and p2 = - pi , speed V. What is the velocity of C ? (1982 - 2 Marks)
undergo a collision in free space. There is no external force
r r A
acting on the balls. Let p1¢ and p¢2 be their final momenta.
The following option (s) is (are) NOT ALLOWED for any
non-zero value of p, a1, a2, b1, b2, c1 and c2. (2008)
r ˆ ˆ ˆ r ˆ
(a) pr1¢ = a1i + b1 j + c1k (b) pr1¢ = c1k G
ˆ ˆ
p¢2 = a 2 i + b 2 j p¢2 = c 2 kˆ
r r B C
(c) p1¢ = a1ˆi + b1ˆj + c1kˆ (d) p1¢ = a1ˆi + b1ˆj
r r 4. Two bodies A and B of masses m and 2 m respectively are
p¢2 = a 2 iˆ + b 2 ˆj - c1kˆ p¢2 = a 2ˆi + b1ˆj
placed on a smooth floor. They are connected by a spring. A
5. A point mass of 1 kg collides elastically with a stationary third body C of mass m moves with velocity v0 along the
point mass of 5 kg. After their collision, the 1 kg mass reverses line joining A and B and collides elastically with A as shown
its direction and moves with a speed of 2 ms–1. Which of the in Fig.
following statement(s) is (are) correct for the system of these C A B
two masses? (2010) v0
(a) Total momentum of the system is 3 kg ms–1
(b) Momentum of 5 kg mass after collision is 4 kg ms–1
(c) Kinetic energy of the centre of mass is 0.75 J At a certain instant of time t0 after collision, it is found that
(d) Total kinetic energy of the system is 4J the instantaneous velocities of A and B are the same. Further
6. A particle of mass m is attached to one end of a mass-less at this instant the compression of the spring is found to be
spring of force constant k, lying on a frictionless horizontal x0. Determine (i) the common velocity of A and B at time t0;
plane. The other end of the spring is fixed. The particle and (ii) the spring constant. (1984- 6 Marks)
starts moving horizontally from its equilibrium position at 5. A ball of mass 100 gm is projected vertically upwards from
time t = 0 with an initial velocity u0. When the speed of the the ground with a velocity of 49 m/sec. At the same time
another identical ball is dropped from a height of 98 m to
particle is 0.5 u0, it collides elastically with a rigid wall. After
fall freely along the same path as that followed by the first
this collision (JEE Adv. 2013)
ball. After some time the two balls collide and stick together
(a) The speed of the particle when it returns to its and finally fall to the ground. Find the time of flight of the
equilibrium position is u0 masses. (1985 - 8 Marks)
MOMENTUM AND IMPULSE P-75
6. A bullet of mass M is fired with a velocity 50 m/s at an angle Assuming that the dimensions of the cylinder remains
with the horizontal. At the highest point of its trajectory, it practically unchanged and that the dust sticks only to the
collides head-on with a bob of mass 3M suspended by a front face of the x-coordinate of the front of the cylinder
massless string of length 10/3 metres and gets embedded in find the x-coordinate of the front of the cylinder at t = 150s
the bob. After the collision, the string moves through an (1993-5 Marks)
angle of 120°. Find 10. A cart is moving along + x direction with a velocity of 4 m/s.
(i) the angle q ; A person on the cart throws a stone with a velocity of 6 m/s
(ii) the vertical and horizontal coordinates of the initial relative to himself. In the frame of reference of the cart the
position of the bob with respect to the point of firing of stone is thrown in y-z plane making an angle of 30° with
the bullet. Take g =10 m /s2 vertical z-axis. At the highest point of its trajectory, the stone
hits an object of equal mass hung vertically from the branch
7. An object of mass 5 kg is projected with a velocity of 20
of a tree by means of a string of length L. A completely
m/s at an angle of 60° to the horizontal. At the highest point
inelastic collision occurs, in which the stone gets embedded
of its path the projectile explodes and breaks up into two
in the object. Determine : (1997 - 5 Marks)
fragments of masses 1 kg and 4 kg. The fragments separate
horizontally after the explosion. The explosion releases (i) The speed of the combined mass immediately after the
internal energy such that the kinetic energy of the system at collision with respect to an observer on the ground,
the highest point is doubled. Calculate the separation (ii) The length L of the string such that the tension in the
between the two fragments when they reach the ground. string becomes zero when the string becomes horizontal
(1990 - 8 Mark) during the subsequent motion of the combined mass.
8. A block ‘A’ of mass 2m is placed on another block ‘B’ of 11. A car P is moving with a uniform speed of 5Ö3 m/s towards
mass 4m which in turn is placed on a fixed table. The two a carriage of mass 9 kg at rest kept on the rails at a point B as
blocks have a same length 4d and they are placed as shown shown in figure. The height AC is 120 m. Cannon balls of
in fig. The coefficient of friction (both static and kinetic) 1 kg are fired from the car with an initial velocity 100 m/s at
between the block ‘B’ and table is m . There is no friction an angle 30o with the horizontal. The first cannon ball hits the
stationary carriage after a time to and sticks to it. Determine to.
between the two blocks. A small object of mass m moving
horizontally along a line passing through the centre of mass (2001 - 10 Marks)
(cm.) of the block B and perpendicular to its face with a
speed v collides elastically with the block B at a height d
above the table. (1991 - 4 + 4 Marks) C
P

A
A 2m B

At to, the second cannon ball is fired. Assume that the resistive
force between the rails and the carriage is constant and ignore
mm the vertical motion of the carriage throughout. If the second
V B 4m 2d ball also hits and sticks to the carriage, what will be the
dd horizontal velocity of the carriage just after the second impact?
12. A particle of mass m, moving in a circular path of radius R
P with a constant speed v2 is located at point (2R, 0) at time
4d
t = 0 and a man starts moving with a velocity v1 along the
(a) What is the minimum value of v (call it v0) required to +ve y-axis from origin at time t = 0. Calculate the linear
make the block A topple ? momentum of the particle w.r.t. the man as a function of time.
(b) If v = 2v0, find the distance (from the point P in the (2003 - 2 Marks)
figure) at which the mass m falls on the table after y
collision. (Ignore the role of friction during the collision).
9. A cylindrical solid of mass 10–2 kg and cross-sessional area v2
10–4 m2 is moving parallel to its axis (the x-axis ) with a v1
uniform speed of 103 m/s in the positive direction. At t = 0,
its front face passes the plane x = 0. The region to the right R x
of this plane is filled with stationary dimension of the (0, 0)
cylinder remain practically unchanged and that the dust m
particle of uniform density 10–3 kg/m3. When dust particles
collides with the face of the cylinder, it sticks to its surface.
P-76 Topic-wise Solved Papers - PHYSICS

ASSERTION & REASON TYPE QUESTIONS : INTEGER VALUE CORRECT TYPE :


1. STATEMENT–1 : In an elastic collision between two bod-
ies, the relative speed of the bodies after collision is equal 1. A bob of mass m, suspended by a string of length l1, is given
to the relative speed before the collision. (2007) a minimum velocity required to complete a full circle in the
because vertical plane. At the highest point, it collides elastically
STATEMENT–2 with another bob of mass m suspended by a string of length
In an elastic collision, the linear momentum of the system is
conserved. l2, which is initially at rest. Both the strings are mass-less
(a) Statement–1 is True, Statement–2 is True; State- and inextensible. If the second bob, after collision acquires
ment–2 is a correct explanation for Statement–1. the minimum speed required to complete a full circle in the
(b) Statement–1 is True, Statement–2 is True; State-
ment–2 is NOT a correct explanation for Statement–1 l1
vertical plane, the ratio is (JEE Adv. 2013)
(c) Statement–1 is True, Statement–2 is False. l2
(d) Statement–1 is False, Statement–2 is True.

1. A machine gun fires a bullet of mass 40 g with a velocity 5. The figure shows the position–time (x – t) graph of one-
1200 ms–1. The man holding it can exert a maximum force of dimensional motion of a body of mass 0.4 kg. The magnitude
144 N on the gun. How many bullets can he fire per second of each impulse is [2010]
at the most? [2004]
(a) Two (b) Four
(c) One (d) Three
2. A mass ‘m’ moves with a velocity ‘v’ and collides inelastically 2

with another identical mass . After collision the lst mass x (m)
v
moves with velocity in a direction perpendicular to the
3
initial direction of motion. Find the speed of the 2 nd mass 0 2 4 6 8 10 12 14 16

after collision. [2005] t(s)

(a) 0.4 Ns (b) 0.8 Ns


m m v
3 (c) 1.6 Ns (d) 0.2 Ns
A before Aafter
collision collision 6. This question has statement I and statement II. Of the four
(a) (b) v choices given after the statements, choose the one that
3v
best describes the two statements. [IIT-JEE 2013]
v 2 Statement - I: Apoint particle of mass m moving with speed
(c) (d) v
3 3 u collides with stationary point particle of mass M. If the
3. A bomb of mass 16kg at rest explodes into two pieces of
masses 4 kg and 12 kg. The velolcity of the 12 kg mass is æ 1 2ö
maximum energy loss possible is given as f çè mv ÷ø then
4 ms–1. The kinetic energy of the other mass is 2
(a) 144 J (b) 288 J [2006]
æ m ö
(c) 192 J (d) 96 J f= ç ÷.
4. Statement -1: Two particles moving in the same direction èM + mø
do not lose all their energy in a completely inelastic collision. Statement - II: Maximum energy loss occurs when the
Statement -2 : Principle of conservation of momentum holds particles get stuck together as a result of the collision.
true for all kinds of collisions. [2010] (a) Statement - I is true, Statment - II is true, Statement - II
(a) Statement -1 is true, Statement -2 is true ; Statement -2 is the correct explanation of Statement - I.
is the correct explanation of Statement -1.
(b) Statement-I is true, Statment - II is true, Statement - II is
(b) Statement -1 is true, Statement -2 is true; Statement -2 not the correct explanation of Statement - II.
is not the correct explanation of Statement -1
(c) Statement -1 is false, Statement -2 is true. (c) Statement - I is true, Statment - II is false.
(d) Statement -1 is true, Statement -2 is false. (d) Statement - I is false, Statment - II is true.
MOMENTUM AND IMPULSE P-77

Solutions & Explanations


Section-A : JEE Advanced/ IIT-JEE
3
A 1. mv 2 2. 0.005 Ns
2
C 1. (a) 2. (d) 3. (c) 4. (d) 5. (a)
D 1. (c, d) 2. (a) 3. (b, d) 4. (a, d) 5. (a, c) 6. (a, d)

æ Mv ö mM (v 2 + V 2 )
E 1. (i) q = tan -1 ç , m V + M v ; (ii)
2 2 2 2 2. 10 2 m/s, 45°
è mV ÷ø ( m + M )( mV 2 + Mv 2 )

v0 2mv02
3. V, opposite direction to the retraced velocity of B 4. 3 , 5. 6.53 sec
3 x02
5
6. 37°, 122.6 m, 46 m 7. 44.25 m 8. (a) 6m gd (b) 6d 3m 9. 105 m
2
v2
10. 2.5 m/s, 0.318 m 11. 12 s, 15.75 m/s 12. -mv2 sin wtiˆ + m(v2 cos wt - v1 ) ˆj where w =
R
H 1. (d)
I. 1. 5

Section-B : JEE Main/ AIEEE


1. (d) 2. (d) 3. (b) 4. (a) 5. (b) 6. (d)

FILL IN THE BLANKS : ALTERNATE SOLUTION


The momentum of the two fragments of mass m are equal
1. 2mv' cos q = mv ... (i) and mutually perpendicular. Their resultant is
2mv' sin q = mv
v (mv )2 + (mv)2 = 2 mv . The momentum of the particle
before explosion is zero.
m \ the momentum after explosion should be zero. Therefore
2mv'cos q
the momentum of the third fragment is 2 mv .
q m v
2
2m p 2m2v 2 1 2
The K.E. of third fragment = = = mv
2mv'sinq
2(2m ) 4m 2
v'
1 2 1 2 1 2 3 2
1 \ Total K.E. = mv + mv + mv = mv
Þ sin q = cos q = 2 2 2 2
2 2. KEY CONCEPT Area under the F – t graph gives the
Putting this value in equation (i), we get impulse imparted to the body.
F
2mv ' v (in N)
= mv , v' = C
2 2 800

2 600
1 2 1 2 1 æ v ö
Total K.E. = mv + mv + (2m) ç
2 2 2 è 2 ÷ø 400

1 2 1 2 mv 2 3 2 200 A B
= mv + mv + = mv
2 2 2 2 E F D
0 2 4 6 8 10 12 14 16 T
P-78 Topic-wise Solved Papers - PHYSICS
The magnitude of total impulse of force on the body from Both are moving under gravitational pull.
t = 4 µs to t = 16 µs r r r
\ Applying v - u = at
= area (BCDFEB) r r r r r r
= area of BCFEB + area CDFC \ v '1 - v1 = g (2t0 ) and v '2 - v2 = g (2t0 )
r r r
1 1 \ m1v '1 - m1v1 = m1g (2t0 ) ... (i)
= (200 + 800) ´ 2 ´ 10 -6 + ´ 10 ´ 800 ´ 10-6 r r r
2 2 and m2v '2 - m2v2 = m2 g (2t0 ) ... (ii)
= 0.001 + 0.004 = 0.005 Ns Adding (i) and (ii)
r r r r r
MCQ's WITH ONE CORRECT ANSWER : [m2 v2' - m2v2 ] + [m1v1' - m1v1 ] = 2 gt0 (m1 + m2 )
r r r r
1. (a) Since the particle was initially moving in the x-direction, ( m1v1' + m2 v2 ' ) - ( m1v1 + m2v2 ) = 2 ( m1 + m2 ) gt0
the momentum in the y-direction is zero. ALTERNATE SOLUTION
y If we consider the two particles as a system then the
external force acting on the system is the gravitational
m pull (m1 + m2) g.
v
Dp
4 x Fext =
3m Dt
r r r r
v¢ 4 \ Dp = Fext Dt = ( m1v1 '+ m2v2 ') - ( m1v1 + m2 v2 )
= (m1 + m2) g × 2t0
3. (c) Just after collision
Ground m1v1 + m2 v2 10 ´ 14 + 4 ´ 0
vc = = = 10 m / s;
Therefore finally the momentum in the y-direction is m1 + m2 10 + 4
zero. Let v and v¢ be the component of velocities in the Note : Spring force is an internal force, it cannot change
y-direction at the time of explosion. Then the linear momentum of the (two mass + spring) system.
3m m Therefore vc remains the same.
\ ´ v' = v ... (i) 4. (d) For vertical motion of bullet or ball
4 4 u=0
After time t second from the explosion, fragment of s = 5m
m t=?
mass is at y = +15 cm. Let the other fragment be a= 10m/s2
4
at y¢. Both the fragments will move with constant speed
M = 0.2 kg
along y-direction. v
m y 15 m = 0.01 kg
For mass ,v= = ... (ii)
4 t t
3m y'
For mass , v' = ... (iii)
4 t 5m
From (i), (ii) and (iii),
3m y ' m 15
´ = ´ Þ y' = 5 cm
4 t 4 t
The displacement will be in – y direction.
ALTERNATE SOLUTION 20 m
Note : As the particle is moving in the x-direction, after 100 m
explosion, the y-coordinate of centre of mass of the 1 2 1
fragments is at zero. S = ut + at Þ 5 = ´ 10 ´ t 2
2 2
m1 y1 + m2 y2 Þ t = 1 sec
\ 0=
m1 + m2 For horizontal motion of ball
xball = Vball t Þ 20 = Vball × 1 = Vball
m1 m/4 For horizontal motion of bullet
Þ y2 = - y1 = - ´ 15 = -5 cm
m2 m xbullet = Vbullet × t Þ 100 = Vbullet × 1 = Vbullet
3 Applying conservation of linear momentum during
4
r collision, w e get
2. (d) At t = 0, mass m1 has velocity v1 and mV = mVbullet + MVball
r
mass m2 has velocity v2 . 0.01 V = 0.01 × 100 + 0.2 × 20
r 5
At time t = 2t0, mass m1 has velocity v1' and \V=
r = 500 m/s
mass m2 has velocity v '2 . 0.01
MOMENTUM AND IMPULSE P-79
5. (a) Activity B to M for particle thrown upwards In situation 2, which is just after the collision of C with
A, C stop and A acquires a velocity v. [head-on elastic
é u 2 sin 2 a ù collision between identical masses]
v12 - u 02 = 2(-g) ê 0 ú
êë 2g úû When A starts moving towards right, the spring suffer
a compression due to which B also starts moving
\ v12 = u 02 (1 - sin 2 a ) = u 02 cos 2 a towards right. The compression of the spring continues
\ v1 = u0 cos a ...(i) till there is relative velocity between A and B. When
this relative velocity becomes zero, both A and B move
2mv sinq 2mv with the same velocity v' and the spring is in a state of
y q maximum compression.
x 2mv cosq Applying momentum conservation in situation 1
u 2 sin 2 a and 3,
H= 0
u0 2g
u0 v
a C A B
m muo cosa m
Fig(i)
Applying conservation of linear momentum in
Y-direction
v
2mv sinq = mv1 = mu0cosa ...(ii) [from (i)] Vel=0
Applying conservation of linear momentum in C A B
X-direction Fig(ii)
2mv cosq = mu0 cosa ...(iii)
on dividing (ii) and (iii) we get
v' v'
p Vel=0
tan q = 1 \q= C A B
4
option (a) is correct Fig(iii) Max Compression

v
1. (c,d) (a) is wrong because the momentum of ball changes mv = mv' + mv' Þ v' =
2
in magnitude as well as direction.
\ K.E. of the system in situation 3 is
(b) is wrong because on collision, some mechanical
energy is converted into heat, sound energy. 1 1
mv '2 + mv '2 = mv '2
(c) is correct because for earth + ball system the impact 2 2
force is an internal force.
(d) is correct. mv 2 æ vö
2. (a) As one piece retraces its path, the speed of this piece = çèQ v ' = ÷ø
4 2
just after explosion should be v cos q This is the kinetic energy possessed by A – B system (since,
– + C is at rest).
Let x be the maximum compression of the spring.
vcosq
m Applying energy conservation
vsinq v
1 2 1 1 1
mv = mv '2 + mv '2 + Kx 2
q 2 2 2 2
vcosq
1 2 1 2 1 2
Þ mv = mv + Kx
vcosq v' 2 4 2
m m
2 2 1 2 1 2 m
Þ Kx = mv \ x= v
(At highest point just after explosion) 2 4 2 K
NOTE THIS STEP 4. (a, d) KEY CONCEPT
Applying conservation of linear momentum at the Use law of conservation of linear momentum.
highest point;
The initial linear momentum of the system is piˆ - piˆ = 0
m m Therefore the final linear momentum should also be
m (v cos q ) = ´ v '- ´ v cos q
2 2 zero.
3 v cos q = v' Option a :
3. (b, d) In situation 1, mass C is moving towards right with
p1' + p2' = (a + a )iˆ + (b + b ) ˆj + c kˆ = Final
velocity v. A and B are at rest. 1 2 1 2 1
momentum.
P-80 Topic-wise Solved Papers - PHYSICS
It is given that a1, b1, c1, a2, b2 and c2 have non-zero ALTERNATE SOLUTION
values. If a1 = x and a2 = – x. Also if b1 = y and b2 = According to law of conservation of linear momentum
– y then the î and ĵ components become zero. But 1 × u1 + 5 × 0 = 1 (–2) + 5 (v2)
Þ u1 = – 2 + 5 v 2 ...(i)
the third term having k̂ component is non-zero. This
The coefficient of restituition
gives a definite final momentum to the system which
violates conservation of linear momentum, so this is v2 - v1 v - ( -2)
e= Þ 1= 2
an incorrect option. u1 - u 2 u1 - 0
Option d:
Þ u1 = v2 + 2 ...(ii)
p1' + p2'
= (a1 + a2 )iˆ + 2b1 ˆj ¹ 0 because b1 ¹ 0 On solving (i) & (ii) we get desired results.
Following the same reasoning as above this option is 6. (a, d) The particle collides elastically with rigid wall. Here
also ruled out. V
5. (a, c) e= =1 \V = 0.5u 0
0.5u 0
For head-on elastic collision
u2 = 0 u0
u1

1 kg 5 kg
Before collision B

C A 0.5u0
(m - m2 )u1 2m2 u2
v1 = 1 +
m1 + m2 m1 + m2
Equilibrium position
(1 - 5)u1 -4
-2 = +0 Þ -2 = u1 i.e. the particle rebounds with the same speed. Therefore
1+ 5 6
the particle will return to its equilibrium position with
u1= –2m/s speed u0. option (a) is correct.
v2 The velocity of the particle becomes 0.5u0 after time t.
Then using the equation V = Vmax cos wt we get
1 kg 2 kg 0.5u0 = u0 cos wt

Þ u1 = 3m / s p 2p T
\ = ´T \t =
\ The total momentum centre of masss system 3 t 6
(before collision) = 1 × 3 + 5 × 0 = 3 kg m/s
m p m
Option (a) is correct The time period T = 2 p . Therefore t =
The velocity of centre of mass of the system before k 3 k
collision is The time taken by the particle to pass through the
m1u1 + m2 u2 1 ´ 3 + 0 1 2p m
vc = = = m/s equilibrium for the first time = 2t = . Therefore
m1 + m2 1+ 5 2 3 k
\ Kinetic energy of the center of mass option (b) is incorrect
2 The time taken for the maximum compression
1 2 1 æ 1ö = tAB + tBA + tAC
= mc vc = (1 + 5) ç ÷ = 0.75J
2 2 è 2ø
p m p m π m m é1 1 1 ù
\ Option (c) is correct = + + =π + +
Now, 3 k 3 k 3 k k êë 3 3 2 úû
(m2 - m1 )u 2 2 m1u1 (5 - 1) ´ 0 2 ´ 1 ´ 3
v2 = + = + = 1m/s 7π m
m1 + m2 m1 + m2 1+ 5 1+ 5 = . Therefore option c is incorrect.
6 k
\ Linear momentum of 5 kg particle is 5 kg m/s.
The time taken for particle to pass through the
\ Option (b) is wrong.
equilibrium position second time
Total kinetic energy of the system (before collision)
1 1 éπ mù m m æ2 ö 5 m
= ´ 1 ´ (3)2 + ´ 5 ´ (0) 2 = 4.5 J = 2ê ú+π =π çè + 1÷ø = π
2 2 ë3 kû k k 3 3 k
\ Option (d) is wrong. option (d) is correct.
MOMENTUM AND IMPULSE P-81
2. KEY CONCEPT
SUBJECTIVE PROBLEMS : The momentum of the system is zero before explosion. Since
1. Let V¢ be the velocity of the final body after collision. the momentum is zero initially, it will be zero just after
Suppose, V¢ makes an angle q with x-direction. explosion according to conservation of linear momentum
(as there is no external force acting on the system).
The resultant momentum of the fragments flying off
(m+M) V¢sinq perpendicular to each other should be equal and opposite
(m+M) V¢ to the third fragment. Resolving the momentum of the third
fragment in the lines of momentum of the other two.
mV q
6 kg m/s
(m+M) V¢cosq
6 2 kg m/s
Mv 3_ v cosq
5 q
q 6 kg m/s
(i) Applying conservation of linear momentum in X
direction 3_ v
5
(m + M) V' cos q = mV ... (i) 3_ v sin q
Applying conservation of linear momentum in Y 5
direction
Applying conservation of linear momentum in both the
(m + M) V' sin q = Mv ... (ii) directions, we get
Dividing equation (i) and (ii)
3
Mv æ Mv ö 6 = v sin q ... (i)
tan q = Þ q = tan–1 çè ÷ 5
mV mV ø
3
This gives the direction of the momentum of the final 6 = v cos q ... (ii)
body. 5
Squaring and adding (i) and (ii), we get 9 2
(m + M)2 V'2 cos2 q + (m + M)2 V'2 sin2 q Squaring and adding, we get 36 + 36 = v
25
= m2V2 + M2v2
5 5
\ v= 72 ´ = 6 2´
m 2V 2 + M 2v2 3 3
\ V' =
m+ M
= 10 2 m/s.
Thus the magnitude of the momentum of the final body
Dividing the two equations we get tan q = 1, q = 45°
= ( m + M )V ' ALTERNATE SOLUTION

= m2V 2 + M 2 v 2
p2 pR = 2p
1
K .E.i - K .E. f K .E. f (m + M )V '2
(ii) 2
=1- = 1-
K .E.i K .E.i é1 2 M 2ù Q p1 = p2 = p = mV
êë 2 mV + 2 v úû
q
m2V 2 + M 2 v 2 p1
(m + M )
DKE (m + M )2
=1-
K .E.i mV 2 + Mv 2

K .E.i - K .E. f m 2V 2 + M 2 v 2 p3
\ =1-
K .E.i (m + M ) (mV 2 + Mv 2 )
r r
Let p1 and p2 be linear momenta of two pieces
2 2 2 2 2 2 2 2 2 2 2
m V + mM v + MmV + M v - m V - M v perpendicular to each other.
=
(m + M ) (mV 2 + Mv 2 ) Note : According to the principle of conservation of linear
r
momentum, the resultant momentum pR should be equal
mM (v 2 + V 2 ) r
= and opposite to the linear momentum p3 of the third piece.
(m + M ) (mV 2 + Mv 2 )
P-82 Topic-wise Solved Papers - PHYSICS
r r
p3 = pR

(3m)V ' = 2 p = 2mV 98m


+
C –
= 2m ( 30)
78.4m
Þ V¢ = 10 2 m/s

3. mV mV mV
C
C B 1
s1 = u1t1 + a1t12
2
h = 49t – 4.9 t2 ... (ii)
A B For the ball dropped from height
mV
Initially mV Finally v22 - u22 = 2a2 s2
Initially v22 - 02 = 2 ´ 9.8 ´ (98 - h)
By symmetry, the momentum of the system is zero.
Finally \ v22 = 19.6 (98 – h) ... (iii)
The momentum of the system should be zero.
\ mV = mV' 1 2
Þ V' = V s2 = u2 t2 + a2 t2
2
The velocity of C is V and is in opposite direction to the
98 – h = 4.9 t2 ... (iv)
retraced velocity of B as shown in the figure.
4. Since the collision between C and A is elastic and their From (ii) and (iv)
masses are equal and A was initially at rest, therefore the 98 – (49t – 4.9t2) = 4.9t2 \ 98 – 49t = 0
result of collision will be that C will come to rest and A will \ t = 2 sec
initially start moving with a velocity v0. But since A is \ h = 49 × 2 – 4.9 × 22 = 78.4 m (from (ii))
connected to B with a spring, the spring will get compressed. Substituting this value of h in (i) and (ii), we get
m vo Am B 2m
C v12 – 2401 = – 19.6 × 78.4 v22 = 19.6 (98 – 78.4)

v12 = 864.36 Þ v22 = 384.16


Vel=0
Am v' B 2m
C
v'
v1 = 29.4 m/s Þ v2 = 19.6 m/s
Note : At point C where the two bodies collide, thereafter
At t = t0, the velocities of A and B become same. both bodies stick and behave as a single body.
Applying energy conservation;
Thus, we apply conservation of linear momentum, which
1 2 1 1 1 gives
mv0 = mv '2 + 2mv '2 + kx02
2 2 2 2 m1v1 – m2v2 = 2mv
where x0 is the compression in the spring at t = t0
v1 - v2 29.4 - 19.6
k \ v= = = 4.9 m/s
\ = 3v ' + x02
v02 2
... (i) 2 2
m For the combined body
Applying momentum conservation, we get u = 4.9 m/s; s1 = – 78.4; a1 = – 9.8 m/s2; t = ?
mv0 = mv' + 2mv'
1 2
v0 s = ut + at
\ v' = ... (ii) 2
3
From (i) and (ii) – 78.4 = 4.9t – 4.9t2
\ 2
t – t – 16 = 0
v2 k 2mv02
v02 - 3 ´ 0 = x02 Þ k= 1 ± 1 + 64
9 m 3x02 t= = 4.53
2
5. For the ball thrown up Total time = 4.53 + 2 = 6.53 sec.
v12 - u12 = 2a1s1 6. (i) In D AQR

v12 - (49)2 = 2( -9.8) h QR 5


sin 30° = , QR =
10 / 3 3
\ v12 - 2401 = 19.6 h ... (i) u = 50 m/s (Given)
MOMENTUM AND IMPULSE P-83
At the highest point P, the velocity of the bullet = u cos q \ The kinetic energy after collision = 2 × 250 = 500 J
1 1
R \ ´ 4 ´ v12 + ´ 1 ´ v22 = 500
2 2
30O
Y Q Þ 4v12 + v22 = 1000 ... (ii)
A
Substituting the value of v2 from (i) in (ii)
10/3
4v12 + (4v1 - 50) 2 = 1000

u cosq P Þ 4v12 + 16v12 + 2500 - 400v1 = 1000


u
( R ,H ) Þ 20v12 - 400v1 + 1500 = 0
2
Þ 2v12 - 40v1 + 150 = 0
q
O X Þ v12 - 20v1 + 75 = 0
Applying conservation of linear momentum at the highest
point Þ v12 - 15v1 - 5v1 + 75 = 0
M (u cos q ) + 3M × 0 = (M + 3M) v Þ v1 = 15 or 5 m/s Þ v2= + 10 or – 30 m/s
Mu cos q u cos q Taking the pair v1 = 15 m/s and v2 = 10 m/s.
v= = Horizontal distance covered by mass m1 and m2
4M 4
uy = 0; sy = H = 15.3; ay = 9.8 m/s2; ty = ?
Applying energy conservation principle for P and R
K.E. of the bullet-mass system at P = P.E. of the bullet-mass 1 2
s = ut + at
system at R 2
15.3 = 4.9t2 Þ t = 1.77 sec
1
(4M )v 2 = (4 M ) gh \ x1 = 15 × 1.77 = 26.55 m
2 and x2 = 10 × 1.77 = 17.7 m
Separation between the two masses
1 u 2 cos2 q æ 10 5 ö = 26.55 + 17.7 = 44.25 m.
(4M ) = 4Mg ´ ç + ÷
2 16 è 3 3ø 8. KEY CONCEPT
Since the collision is elastic in nature applying conservation
9.8 ´ 5 ´ 2 ´ 16 of linear momentum and conservation of kinetic energy
cos2 q = Þ cos q = 0.79 mv = (4m) u + mv'
50 ´ 50
where u is the velocity of mass 4m after collision and v' is
Þ q = 37° the velocity of mass 2m
R u 2 sin 2q 50 ´ 50sin 2 ´ 37o 2m
(ii) = = = 122.6 m A
2 2g 2 ´ 9.8
m v 4m
B
u 2 sin 2 q 50 ´ 50 ´ (sin 37°) 2
H= = = 46 m 4d
2g 2 ´ 9.8
Þ v' = v – 4u ... (i)
2 2
u sin q 20 ´ 20 3 1 2 1 1
7. Max Height = = ´ = 15.3m Also, mv = (4m) u 2 + mv '2
2g 2 ´ 9.8 4 2 2 2
Initial momentum of the object is horizontal direction at the Þ v2 = 4u2 + v'2 ... (ii)
highest point = 5 × 10 = 50 kg m/s From (i) to (ii)
Note : At the highest point the projectile explodes and breaks 2v
into two particles which separate horizontally. v2 = 4u2 + (v – 4u)2 Þ u =
5
20 cos 60o =10m/s Block B starts moving but the block A remains at rest as
there is no friction between A and B.
u = 20m/s For block A to topple, block B should move a distance 2d.
H Let the retardation produced in B due to friction force
60o between B and the table be a
F = mN
Applying conservation of linear momentum
Þ (4 m)a = m (6 mg)
4v1 – 1 v2 = 50 ... (i) Þ a = 1.5 µg
The kinetic energy at the highest point initially For the motion of B,
1 2v
= ´ 5 ´ 10 ´ 10 = 250 J u= , v = 0, s = 2d, a = – 1.5 µg
2 5
P-84 Topic-wise Solved Papers - PHYSICS
2 On integrating
æ 2v ö
Now, v2 – u2 = 2as Þ (0)2 – çè ÷ø = 2( -1.5µg ) 2d 150 x
5 10ò dt = ò (0.01 + 10-7 x ) dx
0 0
5
Þ v= 6µgd x2
2 \ 10 × 150 = 0.01x + 10–7
For elastic collision between two bodies 2
Þ 3000 = 0.02x + 10–7x2
(m - m2 ) u1 2m2 u2
v1 = 1 + Þ 10–6 x2 + 0.2 x – 30000 = 0
m1 + m2 m1 + m2
-0.2 ± 0.04 - 4( -30000) ´ 10 -6
Here m1 = m, m2 = 4m, u1 = 5 6µgd , u2 = 0 Þ x=
2 ´ 10-6
(m - 4m) 5 6µgd 6µgd
Þ v1 = + 0 = -3 ´ 5 = [– 0.1 ± 0.01 + 30000 ´ 10-6 ] × 106
m + 4m 5
Þ x = 105 m
= -3 6µgd 10. When the stone reaches the point Q, the component of
velocity in the + Z direction (v cos q ) becomes zero due to
3 6m gd the gravitational force in the – Z direction.
d

Note : The negative sign shows that the mass m rebounds.


It then follows a projectile motion. +Z Z L v'
+Y vx = 4m/s
Considering the vertical direction motion of this projectile. Q Y
uy = 0, sy = d, ay = g, ty = ? v
vcosq q
vsinq
1 2 1 2 P X
vx = 4m
S = ut + at Þ d = gt t=0 t=t
2 2

2d
Þ t= The stone has two velocities at Q
g
• vx in the + X direction (4 m/s)
The horizontal distance travelled by mass m during this
• v sin q in the + Y direction (6 sin 30° = 3 m/s)
time t
The resultant velocity of the stone
2d
x = 3 6µgd ´ = 6 3µd 2 = 6d 3µ v' = (v x )2 + (v sin q)2
g
9. Let the cylinder reach A' at time t = t as shown in the figure. 42 + 32 = 5m / s
=
Since the dust particles are sticking the cylinder, therefore
the mass of cylinder increases continuously with time. Since (i) Applying conservation of linear momentum at Q for
no external forces are acting, the momentum of the cylinder collision with a mass of equal magnitude
at A is the same as momentum of cylinder at A'. m × 5 = 2m × v
mv = m'v' where m' = m + Axr Note : Since, the collision is completely inelastic the two
masses will stick together. v is the velocity of the two masses
just after collision.]
\ v = 2.5 m/s
A A'
(ii) When the string is undergoing circular motion, at any
a
arbitrary position
V=103m/s V' P
x=0 t=t 2mv 2
t=0 T – 2mg cos a =
l
x
Given that, T = 0 when a = 90°
\ [0.01 ×103] = [0.01 + 10–4 × x × 10–3] × v' 2mv 2
\ 0–0= Þ v=0
dx l
10 = [0.01 + 10–7 x]
dt Þ Velocity is zero when a = 90°, i.e., in the horizontal
\ 10dt = [0.01 + 10–7 x] dx position.
MOMENTUM AND IMPULSE P-85
M For trolley after 12 seconds;
a u = 5 3 m/s
v=?
T t = 12 s
m ux
M
2mg sin a
a 2mg cosa 1 2
2mg s = 60 3 m, s = ut + at
Q 2
Applying energy conservation from Q to M, we get 1
Þ 60 3 = 5 3 × 12 + × a × 144
1 2
2mv 2 = 2mg l
2 \ a = 0 m/s2 \ v = u + at = 5 3 m/s.
To find the final velocity of the carriage after the second
v 2 (2.5) 2
Þ l= = = 0.318 m impact we again apply conservation of linear momentum in
2 g 2 ´ 9.8 the horizontal direction
11. Consider the vertical motion of the cannon ball mux + (M + m) vx= (M + 2m) vf
uy = + 100 sin 30° \ 1× 50 3 + (9 + 1) 5 3 = (9 + 2) vf
sy = – 120 m Þ vf = 15.75 m/s
ay = – 10 m/s2 r r
t = t0 12. v2 = (- v2 sin wt $i + v2 cos wt $j ); v1 = v1 $j
v2
1 2
s = ut + at \ – 120 = 50 t0 – 5 t02 t
2
\ 5t02 - 50t0 - 120 = 0
t
Þ t02 – 10 t0 – 24 = 0

(-10) ± 100 - 4(1) (-24)


\ t0 = – = 12 or – 2 [Not valid] r r r
2 v PM = v2 - v1 = - v2 sin wt i$ + (v2 cos wt - v1 ) $j
\ t0 = 12 sec. r r
p PM = mv PM = - mv2 sin wt i$ + m(v2 cos wt - v1 ) $j
+ P uy u v2
_ 30o
where w =
ux R
C r é ˆj ù
or, p PM = m êæç -v2 sin 2 tiˆö÷ + æç v2 cos 2 t - v1 ö÷
120m m = 9kg v v
è R ø è R ø ú
u =0 ë û
A B T
The horizontal velocity of the cannon ball remains the same ASSERTION & REASON TYPE QUESTIONS :
\ ux = 100 cos 30° = 50 3 ms -1 1. (d) Statement 1 : For an elastic collision, the coefficient of
\ Applying conservation of linear momentum to the restitution = 1
cannon ball-trolley system in horizontal direction. If m is the | v2 - v1 |
mass of cannon ball and M is the mass of the trolley then, e= Þ | v2 – v1| = | u1 – u2 |
| u1 - u2 |
mux + M × 0 = (m + M) vx
Þ Relative velocity after collision is equal to relative
mu x velocity before collision. But in the statement relative
\ vx = where vx is the velocity of the speed is given.
m+M
cannon ball-trolley system. Statement 2 : Linear momentum remains conserved in
an elastic collision. This statement is true.
1 ´ 50 3
\ vx= = 5 3m/s INTEGER VALUE CORRECT TYPE :
1+ 9
The second ball was projected after 12 second. Horizontal 1. 5 Velocity at the highest point of bob tied to string l1 is
distance covered by the car P acquired by the bob tied to string l2 due to elastic
head-on collision of equal masses
= 12 × 5 3 = 60 3 m
Therefore gl1 = 5gl 2
Since the second ball also struck the trolley,
Therefore, in time 12 seconds, the trolley covers a distance l
\ 1 =5
of 60 3 m. l2
P-86 Topic-wise Solved Papers - PHYSICS

1. (d) Let n be the number of bullets that the man can fire in 3. (b) Let the velocity and mass of 4 kg piece be v1 and m1
one second. and that of 12 kg piece be v2 and m2.
\ change in momentum per second = n ´ mv = F
[ m= mass of bullet, v = velocity] (Q F is the force) 16 kg
Initial momentum
F 144 ´ 1000
\n = = =3 Situation 1
=0
mv 40 ´ 1200
4 kg = m1 m2 = 12 kg Final momentum
v
= (v2 )y
v1 v2 = m2v2 – m1v1
3 Situation 2
2. (d)
Applying conservation of linear momentum
m2v2 = m1v1
u1 = v u2 = 0
m m 12 ´ 4
Þ v1 = = 12 ms -1
4
In x-direction : mv + 0 = m (0) + m(v2)x
æ v ö 1 1
In y-direction : 0 + 0 = m ç ÷ + m(v2 ) y is \ K .E.1 = m1v12 = ´ 4 ´ 144 = 288 J
è 3ø 2 2
4. (a) In completely inelastic collision, all energy is not lost
v (so, statement -1 is true) and the principle of
Þ (v2 ) y = and (v2)x = v
3 conservation of momentum holds good for all kinds of
collisions (so, statement -2 is true) . Statement -2
2
æ v ö explains statement -1 correctly because applying the
\ v2 = ç ÷ + v 2 principle of conservation of momentum, we can get the
è 3ø
common velocity and hence the kinetic energy of the
combined body.
v2 4 2v
Þ v2 = + v2 = v = 5. (b) In each impulse, x varies from 0 to 2m and again from
3 3 3
2 m to 0 during the time interval of 4s. We have ,
ALTERNATE SOLUTION Impulse = Change in momentum
In x-direction, mv = mv1 cosq ...(1)
where v1 is the velocity of second mass = m v2 - v1 = 0.4 × 2 = 0.8 Ns
mv P2 P2
In y-direction, 0= - mv1 sin q 6. (d) Maximum energy loss = -
3 2m 2(m + M)

mv é P2 1 ù
or m1v1 sin q = ...(2) êQ K.E. = = mv2 ú
3 ëê 2m 2 ûú

v/ 3 P2 é M ù 1 ì M ü
= = mv 2 í ý
v 2m êë (m + M) úû 2 îm + M þ
m v
v1 cosq Statement II is a case of perfectly inelastic collision.
q By comparing the equation given in statement I with
v1 above equation, we get
v1 sinq æ M ö æ m ö
f=ç
è m + M ÷ø instead of ç ÷
èM+mø
Squaring and adding eqns. (1) and (2)
Hence statement I is wrong and statement II is correct.
v2 2
v12 =v +2
Þ v1 = v
3 3
6 Rotational Motion

6. A rod of weight w is supported by two parallel knife edges


FILL IN THE BLANKS : A and B and is in equilibrium in a horizontal position. The
1. A uniform cube of side a and mass m rests on a rough knives are at a distance d from each other. The centre of
mass of the rod is at distance x from A . The normal reaction
horizontal table. A horizontal force F is applied normal to
on A is... and on B is..... (1997 - 2 Marks)
one of the faces at a point that is directly above the centre
of the face, at a height 3a/4 above the base. The minimum 7. A symmetric lamina of mass M consists of a square shape
with a semicircular section over of the edge of the square as
value of F for which the cube begins to tip about the edge is shown in Fig. P-10. The side of the square is 2a. The moment
.... (Assume that the cube does not slide). (1984 - 2 Marks) of inertia of the lamina about an axis through its centre of
2. A smooth uniform rod of length L and mass M has two mass and perpendicular to the plane is 1.6 Ma2. The moment
identical beads of negligible size, each of mass m, which can of inertia of the lamina about the tangent AB in the plane of
slide freely along the rod. Initially the two beads are at the the lamina is.... (1997 - 2 Marks)
centre of the rod and the system is rotating with an angular
A B
velocity w0 about an axis perpenducular to the rod and
passing through the midpoint of the rod (see figure). There
are no external forces. When the beads reach the ends of
the rod, the angular velocity of the system is .........
(1988 - 2 Marks) 2a

L L
2 2
w0

3. A cylinder of mass M and radius R is resting on a horizontal TRUE / FALSE :


platform (which is parallel to the x–y plane) with its axis
1. A triangular plate of uniform thickness and density is made
fixed along the y-axis and free to rotate about its axis. The
to rotate about an axis perpendicular to the plane of the
platform is given a motion in the x-direction given by x = A paper and (a) passing through A, (b) passing through B, by
cos ( w t). There is no slipping between the cylinder and the application of the same force, F, at C (midpoint of AB)
platform. The maximum torque acting on the cylinder during as shown in the figure. The angular acceleration in both
its motion is .................. (1988 - 2 Marks) the cases will be the same. (1985 - 3 Marks)
4. A stone of mass m, tied to the end of a string, is whirled
around in a horizontal circle. (Neglect the force due to
gravity). The length of the string is reduced gradually
keeping the angular momentum of the stone about the centre
A C B
of the circle constant. Then, the tension in the string is
given by T = Arn where A is a constant, r is the instantaneous F
radius of the circle and n = ..... (1993 - 1 Mark) 2. A thin uniform circular disc of mass M and radius R is rotating
5. A uniform disc of mass m and radius R is rolling up a rough in a horizontal plane about an axis passing through its centre
inclined plane which makes an angle of 30° with the and perpendicular to its plane with an angular velocity w .
horizontal. If the coefficients of static and kinetic friction Another disc of the same dimensions but of mass M/4 is
placed gently on the first disc coaxially. The angular velocity
are each equal to m and the only forces acting are gravitational
and frictional, then the magnitude of the frictional force acting of the system now is 2 w / 5 .
on the disc is .... and its direction is ......... (write up or down) (1986 - 3 Marks)
the inclined plane. (1997C - 1 Mark)
P-88 Topic-wise Solved Papers - PHYSICS
3. A ring of mass 0.3 kg and radius 0.1 m and a solid cylinder 5. A disc of mass M and radius R is rolling with angular speed
of mass 0.4 kg and of the same radius are given the same w on a horizontal plane as shown in Figure. The magnitude
kinetic energy and released simultaneously on a flat of angular momentum of the disc about the origin O is
horizontal surface such that they begin to roll as soon as (1999S - 2 Marks)
released towards a wall which is at the same distance from
the ring and the cylinder. The rolling friction in both cases is Y
negligible. The cylinder will reach the wall first.
w
(1989 - 2 Marks)
M
4. Two particles of mass 1 kg and 3 kg move towards each
other under their mutual force of attraction. No other force X
acts on them. When the relative velocity of approach of the O
two particles is 2 m/s, their centre of mass has a velocity of (a) (1/2) MR2w (b) MR2w
0.5 m/s. When the relative velocity of approach becomes (c) (3/2) MR w2 (d) 2 MR2w
3 m/s, the velocity of the centre of mass is 0.75 m/s. 6. A cubical block of side a is moving with velocity V on a
(1989 - 2 Marks) horizontal smooth plane as shown in Figure. It hits a ridge
at point O. The angular speed of the block after it hits O is
MCQ's WITH ONE CORRECT ANSWER : (1999S - 2 Marks)
1. A thin circular ring of mass ‘M and radius r is rotating about a
its axis with a constant angular velocity w, Two objects,
each of mass m,, are attached gently to the opposite ends of M V
a diameter of the ring. The wheel now rotates with an angular
velocity (1983 - 1 Mark)
O
wM w ( M – 2 m)
(a) (b) (a) 3V/(4a) (b) 3V/(2a)
( M + m) (M + 2m)
(c) 3V / ( 2a ) (d) zero
wM w (M + 2m) 7. A long horizontal rod has a bead which can slide along its
(c) (d)
( M + 2 m) M length and initially placed at a distance L from one end A of
2. Two point masses of 0.3 kg and 0.7 kg are fixed at the ends the rod. The rod is set in angular motion about A with
of a rod of length 1.4 m and of negligible mass. The rod is set constant angular acceleration a . If the coefficient of friction
rotating about an axis perpendicular to its length with a between the rod and the bead is m , and gravity is neglected,
uniform angular speed. The point on the rod through which then the time after which the bead starts slipping is
the axis should pass in order that the work required for (2000S )
rotation of the rod is minimum, is located at a distance of a
(1995S) A B
(a) 0.42 m from mass of 0.3 kg L
(b) 0.70 m from mass of 0.7 kg
(a) m/a (b) m / a
(c) 0.98 m from mass of 0.3 kg
(d) 0.98 m from mass of 0.7 kg 1
3. A mass m is moving with a constant velocity along a line (c) (d) infinitesimal
ma
parallel to the x-axis, away from the origin. Its angular
8. A cubical block of side L rests on a rough horizontal surface
momentum with respect to the origin (1997C - 1 Mark)
with coefficient of friction m . A horizontal force F is applied
(a) is zero (b) remains constant
on the block as shown. If the coefficient of friction is
(c) goes on increasing (d) goes on decreasing. sufficiently high so that the block does not slide before
4. A smooth sphere A is moving on a frictionless horizontal toppling, the minimum force required to topple the block is
plane with angular speed w and centre of mass velocity u. It (2000S)
collides elastically and head on with an identical sphere B at
F P
rest. Neglect friction everywhere. After the collision, their
angular speeds are wA and wB, respectively. Then M
L
(1999S - 2 Marks)
(a) wA< wB (b) wA= wB
(c) wA= w (d) wB= w (a) infinitesimal (b) mg/4
(c) mg/2 (d) mg(1 - m)
ROTATIONAL MOTION P-89
9. A thin wire of length L and uniform linear mass density r is 13. A circular platform is free to rotate in a horizontal plane
bent into a circular loop with centre at O as shown. The about a vertical axis passing through its centre. A tortoise is
moment of inertia of the loop about the axis XX' is (2000S) sitting at the edge of the platform. Now, the platform is given
X X' an angular velocity w0. When the tortoise move along a
chord of the platform with a constant velocity (with respect
90º
to the platform), the angular velocity of the platform w(t)
will vary with time t as (2002S)

w(t )
rL3 rL3
(a) (b)
8p 2 16p 2 w0
5rL3 3rL3 (a)
(c) (d)
16p 2 8p2
10. An equilateral triangle ABC formed from a uniform wire has t
two small identical beads initially located at A. The triangle
is set rotating about the vertical axis AO. Then the beads are w(t )
released from rest simultaneously and allowed to slide down,
one along AB and the other along AC as shown. Neglecting
frictional effects, the quantities that are conserved as the w0
beads slide down, are (2000S) (b)

A t

g
w(t )

B C w0
O
(a) angular velocity and total energy (kinetic and potential) (c)
(b) Total angular momentum and total energy
(c) angular velocity and moment of inertia about the axis t
of rotation
(d) total angular momentum and moment of inertia about w(t )
the axis of rotation
11. One quarter sector is cut from a
uniform circular disc of radius R. This w0
sector has mass M. It is made to (d)
rotate about a line perpendicular to
its plane and passing through the t
center of the original disc. Its
moment of inertia about the axis of 14. Consider a body, shown in figure, consisting of two identical
rotation is balls, each of mass M connected by a light rigid rod. If an
(2001S) impulse J=MV is imparted to the body at one of its ends,
what would be its angular velocity? (2003S)
1 1 L
(a) MR 2 (b) MR 2
2 4 M M
1
(c) MR 2 (d) 2 MR 2
8
12. A cylinder rolls up an inclined plane, reaches some height, J = MV
and then rolls down (without slipping throughout these
motions). The directions of the frictional force acting on the (a) V/L (b) 2V/L
cylinder are (2002S) (c) V/3L (d) V/4L
(a) up the incline while ascending and down the incline 15. A particle undergoes uniform circular motion. About which
descending point on the plane of the circle, will the angular momentum
(b) up the incline while ascending as well as descending of the particle remain conserved? (2003S)
(c) down the incline while ascending and up the incline (a) centre of the circle
while descending (b) on the circumference of the circle.
(d) down the incline while ascending as well as (c) inside the circle
descending. (d) outside the circle.
P-90 Topic-wise Solved Papers - PHYSICS
16. A horizontal circular plate is rotating about a vertical axis 21. A solid sphere of mass M and radius R having moment of
passing through its centre with an angular velocity wo. A inertia I about its diameter is recast into a solid disc of radius
man sitting at the centre having two blocks in his hands r and thickness t. The moment of inertia of the disc about an
stretches out his hands so that the moment of inertia of the axis passing the edge and perpendicular to the plane remains
system doubles. If the kinetic energy of the system is K I. Then R and r are related as (2006 - 3M, –1)
initially, its final kinetic energy will be (2004S)
(a) 2 K (b) K/ 2 (c) K (d) K/ 4 2 2
(a) r= R (b) r= R
17. A disc is rolling without slipping with angular velocity w. P 15 15
and Q are two points equidistant from the centre C. The
order of magnitude of velocity is (2004S) 2 2
(c) r= R (d) r = R
15 15
22. A small object of uniform density rolls up a curved surface
C P with an initial velocity v. It reaches up to a maximum height of
Q
3v 2
with respect to the initial position. The object is (2007)
(a) vQ > vC > vP (b) vP > vC > vQ 4g
(c) vP = vC , vQ = vC/2 (d) vP < vC > vQ
18. A block of mass m is at rest under the action of force F
against a wall as shown in figure. Which of the following
statement is incorrect? (2005S)
a v

a (a) ring (b) solid sphere


(c) hollow sphere (d) disc
F B
23. A bob of mass M is suspended
by a massless string of length
L. The horizontal velocity v at
position A is just sufficient to
(a) f = mg [where f is the friction force] make it reach the point B. The
(b) F = N [where N is the normal force] angle q at which the speed of
(c) F will not produce torque the bob is half of that at A,
(d) N will not produce torque satisfies – L
19. From a circular disc of radius R and mass 9M, a small disc of (2008)
radius R/3 is removed from the disc. The moment of inertia v
of the remaining disc about an axis perpendicular to the A
plane of the disc and passing through O is (2005S)
p p p
(a) q= (b) <q<
4 4 2
R p 3p 3p
3 (c) <q< (d) <q<p
2R/3 2 4 4
24. Look at the drawing given in the figure which has been
R drawn with ink of uniform line-thickness. The mass of ink
used to draw each of the two inner circles, and each of the
two line segments is m. The mass of the ink used to draw the
outer circle is 6 m. y
The coordinates of the centres of
the different parts are: outer circle
40 (0, 0), left inner circle (– a, a), right
(a) 4MR2 (b) MR 2 inner circle (a, a), vertical line (0,
9
0) and horizontal line (0, – a). The x
37
(c) 10 MR 2 (d) MR 2 y-coordinate of the centre of mass
9 of the ink in this drawing is
20. A particle is confined to rotate in a circular path decreasing
linear speed, then which of the following is correct?
(2009)
(2005S)
r a a
(a) L (angular momentum) is conserved about the centre (a) (b)
r 10 8
(b) only direction of angular momentum L is conserved
a a
(c) It spirals towards the centre (c) (d)
(d) its acceleration is towards the centre. 12 3
ROTATIONAL MOTION P-91
25. A small mass m is attached to a massless string whose other 2. A mass M moving with a constant velocity parallel to the
end is fixed at P as shown in the figure. The mass is X-axis. Its angular momentum with respect to the origin
undergoing circular motion in the x-y plane with centre at O (1985 - 2 Marks)
and constant angular speed w. If the angular momentum of
r (a) is zero
the system, calculated about O and P are denoted by LO
r (b) remains constant
and LP respectively, then (c) goes on increasing
z (d) goes on decreasing
3. When a bicycle is in motion, the force of friction exerted by
P the ground on the two wheels is such that it acts
(1990 - 2 Marks)
(a) in the backward direction on the front wheel and in the
O m
forward direction on the rear wheel.
w (b) in the forward direction on the front wheel and in the
backward direction on the rear wheel.
r r (c) in the backward direction on both the front and the
(a) LO and LP do not vary with time
r r rear wheels.
(b) LO varies with time while LP remains constant (d) in the forward direction on both the front the rear
r r
(c) LO remains constant while LP varies with time wheels.
r r 4. A particle of mass m is projected with a velocity v making an
(d) LO and LP both vary with time
angle of 45° with the horizontal. The magnitude of the angular
26. A thin uniform rod, pivoted at O, is rotating in the horizontal momentum of the projectile about the point of projection
plane with constant angular speed w, as shown in the figure.
At time t = 0, a small insect starts from O and moves with when the particle is at its maximum height h is
constant speed v, with respect to the rod towards the other (1990 - 2 Marks)
end. It reaches the end of the rod at t = T and stops. The
angular speed of the system remains w throughout. The (a) zero (b) mv 3
magnitude of the torque ( rt ) about O, as a function of time 4 2g
is best represented by which plot?
z mv3
(c) (d) m 2 gh3
2g
w 5. A uniform bar of length 6a and mass 8m lies on a smooth

horizontal table. Two point masses m and 2m moving in the
same horizontal plane with speed 2v and v, respectively,
O strike the bar [as shown in the fig.] and stick to the bar after
collision. Denoting angular velocity (about the centre of
mass), total energy and centre of mass velocity by w, E and
r r vc respectively, we have after collision
|t| |t |
(1991 - 2 Mark)
(a)
O
T
t
(b)
O T
t 2m

3a
r r v C
|t| |t |
(c) (d)
O t O t
T T 2a 2a
2v
mm
1. Two particles A and B initially at rest, move towards each
other under mutual force of attraction. At the instant when 3v
the speed of A is V and the speed of B is 2V, the speed of the (a) vc = 0 (b) w=
centre of mass of the system is (1982 - 3 Marks) 5a
(a) 3 V (b) V
(c) 1.5 V (d) zero v 3mv 2
(c) w= (d) E=
5a 5
P-92 Topic-wise Solved Papers - PHYSICS
6. The moment of inertia of a thin square plate ABCD, Fig., of 12. If the resultant of all the external forces acting on a system
uniform thickness about an axis passing through the centre of particles is zero, then from an inertial frame, one can surely
O and perpendicular to the plane of the plate is say that (2009)
(1992 - 2 Marks) (a) linear momentum of the system does not change in
4
time
1
A B (b) kinetic energy of the system does not change in time
(c) angular momentum of the system does not change in
O time
3 (d) potential energy of the system does not change in time
13. A sphere is rolling without slipping on a fixed horizontal
D C plane surface. In the figure, A is the point of contact, B is the
2 centre of the sphere and C is its topmost point. Then,
(a) I1 + I 2 (b) I3 + I 4 C
(c) I1 + I3 (d) I1 + I 2 + I3 + I 4
where I1, I 2 , I3 and I 4 are respectively the moments of B
intertial about axis 1, 2, 3 and 4 which are in the plane of the
plate.
7. A tube of length L is filled completely with an incomressible A
liquid of mass M and closed at both the ends. The tube is (2009)
then rotated in a horizontal plane about one of its ends with r r r r
a uniform angular velocity w . The force exerted by the liquid (a) VC - VA = 2 (VB - VC )
at the other end is (1992 - 2 Marks) r r r r
(b) VC - VB = VB - VA
M w2 L r r r r
(a) (b) M w2 L (c) VC - VA = 2 VB - VC
2
r r r
M w2 L M w 2 L2 (d) VC - VA = 4 VB
(c) (d)
4 2 14. Two solid spheres A and B of equal volumes but of different
8. A car is moving in a circular horizontal track of radius 10 m densities dA and dBare connected by a string. They are fully
with a constant speed of 10 m/s. A pendulum bob is immersed in a fluid of density dF. They get arranged into an
suspended from the roof of the car by a light rigid rod of equilibrium state as shown in the figure with a tension in the
length 1.00 m. The angle made by the rod with track is string. The arrangement is possible only if (2011)
(1992 - 2 Mark)
(a) zero (b) 30°
(c) 45° (d) 60° A
9. Let I be the moment of inertia of a uniform square plate
about an axis AB that passes through its centre and is parallel
to two of its sides. CD is a line in the plane of the plate that B
passes through thecentreof the plate and makesan angle q
with AB. The moment of inertia of the plate about the axis (a) dA < dF (b) dB > dF
CD is then equal to (1998S - 2 Marks) (c) dA > dF (d) dA + dB = 2dF
(a) I (b) I sin2 q 15. A thin ring of mass 2 kg and radius 0.5 m is rolling without
(c) I cos2 q (d) I cos2 (q/2) on a horizontal plane with velocity 1 m/s. A small ball of
10. The torque t on a body about a given point is found to be mass 0.1 kg, moving with velocity 20 m/s in the opposite
equal to A × L where A is a constant vector, and L is the direction hits the ring at a height of 0.75 m and goes vertically
angular momentum of the body about that point. From this up with velocity 10 m/s. Immediately after the collision
it follows that (1998S - 2 Marks) (2011)
dL
(a) is perpendicular to L at all instants of time.
dt
(b) the component of L in the direction of A does not
change with time.
(c) the magnitude of L does not change with time.
(d) L does not change with time
(a) the ring has pure rotation about its stationary
11. A solid cylinder is rolling down a rough inclined plane of
CM.
inclination q. Then (2006 - 5M, –1)
(b) the ring comes to a complete stop.
(a) The friction force is dissipative
(c) friction between the ring and the ground is to
(b) The friction force is necessarily changing
the left.
(c) The friction force will aid rotation but hinder translation
(d) there is no friction between the ring and the
(d) The friction force is reduced if q is reduced
ground.
ROTATIONAL MOTION P-93
16. The figure shows a system consisting of (i) a ring of outer 3. A circular plate of uniform thickness has a diameter of 56
radius 3R rolling clockwise without slipping on a horizontal cm. A circular portion of diameter 42 cm is removed from one
surfacewith angular speed w and (ii) an inner disc of radius edge of the plate as shown in figure. (1980)
2R rotating anti-clockwise with angular speed w/2. The ring
and disc are separated by frictionless ball bearings. The
point P on the inner disc is at a distance R from the origin,
where OP makes an angle of 30° with the horizontal. Then
with respect to the horizontal surface, (2012)
z
w

3Rw/2
O 2R x 42cm
56cm

Find the position of the centre of mass of the remaining


portion.
(a) the point O has linear velocity 3 Rw î 4. A block of mass M with a semicircular of radius R, rests on
11 3 a horizontal frictionless surface. A uniform cylinder of radius
(b) the point P has linear velocity Rwiˆ + Rwkˆ . r and mass m is released from rest at the top point A (see
4 4
Fig). The cylinder slips on the semicircular frictionless track.
13 3
(c) the point P has linear velocity Rwiˆ - Rwkˆ How far has the block moved when the cylinder reaches
4 4 the bottom (point B) of the track ? How fast is the block
(d) the point P has linear velocity
moving when the cylinder reaches the bottom of the track ?
æ 3ö ˆ 1 ˆ (1983 - 7 Marks)
çè 3 - ÷ø Rwi + Rwk
4 4
17. Two solid cylinders P and Q of same mass and same radius r
start rolling down a fixed inclined plane from the same height m R
at the same time. Cylinder P has most of its mass
concentrated near its surface, while Q has most of its mass B M
concentrated near the axis. Which statement(s) is(are)
correct? (2012)
(a) Both cylinders P and Q reach the ground at the same
time. 5. A particle is projected at time t= 0 from a point P on the
(b) Cylinders P has larger linear acceleration than cylinder ground with a speed v0, at an angle of 45° to the horizontal.
Q. Find the magnitude and direction of the angular momentum
(c) Both cylinders reach the ground with same translational
kinetic energy. of the particle about P at time t = v0/g (1984- 6 Marks)
(d) Cylinder Q reaches the ground with larger angular 6. A small sphere rolls down without slipping from the top of a
speed. track in a vertical plane. The track has an elevated section
and a horizontal part, The horizontal part is 1.0 metre above
SUBJECTIVE PROBLEMS :
the ground level and the top of the track is 2.4 metres above
1. A 40 kg mass, hanging at the end of a rope of length l, the ground. Find the distance on the ground with respect to
oscillates in a vertical plane with an angular amplitude q0. the point B (which is vertically below the end of the track as
What is the tension in the rope when it makes an engle q
with the vertical? If the breaking strength of the rope is shown in fig.) where the sphere lands. During its flight as a
80 kg, what is the maximum amplitude with which the mass projectile, does the sphere continue to rotate about its centre
can oscillate without the rope breaking? (1978) of mass? Explain. (1987 - 7 Marks)
2. A large mass M and a small mass m hang at two ends of a
string that passes over a smooth tube as shown in the figure.
The mass m moves around a circular path which lies in a
horizontal plane.
The length of string from the mass m
to the top of the tube is l and q is the l
‘angle’ this length makes with the
2.4 m

vertical. What should be the


m
frequency of rotation of mass m, so
that the mass M remains stationary? 1.0 m
(1978)
M B
P-94 Topic-wise Solved Papers - PHYSICS
7. A thin uniform bar lies on a frictionless horizontal surface 11. A block X of mass 0.5 kg is held by a long massless string
and is free to move in any way on the surface. Its mass is on a frictionless inclined plane of inclination 30° to the
horizontal. The string is wound on a uniform solid cylindrical
0.16 kg and length 3 meters. Two particles, each of mass drum Y of mass 2 kg and of radius 0.2 m as shown in Figure.
0.08 kg, are moving on the same surface and towards the bar The drum is given an initial angular velocity such that the
in a direction perpendicular to the bar, one with a velocity of
block X starts moving up the plane. (1994 - 6 Marks)
10 m/s, and other with 6 m/s as shown in fig. The first particle
strikes the bar at point A and the other at point B. Points A Y
and B are at a distance of 0.5m from the centre of the bar.
The particles strike the bar at the same instant of time and
stick to the bar on collision. Calculate the loss of the kinetic
X
energy of the system in the above collision process.
(1989 - 8 Marks)
30°
10 m/s A (i) Find the tension in the string during the motion.
(ii) At a certain instant of time the magnitude of the angular
velocity of Y is 10 rad s–1 calculate the distance travelled
by X from that instant of time until it comes to rest
6 m/s B
12. Two uniform thin rods A and B of length 0.6 m each and of
masses 0.01 kg and 0.02 kg respectively are rigidly joined
8. A carpet of mass M made of inextensible material is rolled end to end. The combination is pivoted at the lighter end, P
along its length in the form of a cylinder of radius R and is as shown in fig. Such that it can freely rotate about point P
kept on a rough floor. The carpet starts unrolling without in a vertical plane. A small object of mass 0.05 kg, moving
sliding on the floor when a negligibly small push is given to horizontally, hits the lower end of the combination and sticks
it. Calculate the horizontal velocity of the axis of the to it. What should be the velocity of the object so that the
system could just be raised to the horizontal position.
R
cylindrical part of the carpet when its radius reduces to . (1994 - 6 Marks)
2
(1990 - 8 Mark) P
9. A homogeneous rod AB of length L = 1.8 m and mass M is
pivoted at the centre O in such a way that it can rotate freely A
in the vertical plane (Fig). The rod is initially in the horizontal
position. An insect S of the same mass M falls vertically
with speed V on the point C, midway between the points O B
and B. Immediately after falling, the insect moves towards
the end B such that the rod rotates with a constant angular
velocity w . (1992 - 8 Marks)
13. A rectangular rigid fixed block has a long horizontal edge. A
S solid homogeneous cylinder of radius R is placed
horizontally at rest its length parallel to the edge such that
the axis of the cylinder and the edge of the block are in the
V
same vertical plane as shown in the figure below. There is
sufficient friction present at the edge so that a very small
displacement causes the cylinder to roll off the edge without
A O C B slipping. Determine: (1995 - 10 Marks)
L L L
2 4 4
(a) Determine the angular velocity w in terms of V and L.
(b) If the insect reaches the end B when the rod has turned
through an angle of 90°, determine V. R
10. A uniform thin rod of mass M and length L is standing
vertically along the y-axis on a smooth horizontal surface,
with its lower end at the origin (0, 0). A slight disturbance at
t = 0 causes the lower end to slip on the smooth surface (a) the angle qc through which the cylinder rotates before
along the positive x-axis, and the rod starts falling. it leaves contact with the edge,
(1993-1+5 Marks) (b) the speed of the centre of mass of the cylinder before
(i) What is the path followed by the centre of mass of the leaving contact with the edge, and
rod during its fall? (c) the ratio of the translational to rotational kinetic
(ii) Find the equation to the trajectory of a point on the rod energies of the cylinder when its centre of mass is in
located at a distance r from the lower end. What is the horizontal line with the edge.
shape of the path of this point?
ROTATIONAL MOTION P-95
14. A small sphere of radius R is held against the inner surface 17. A wedge of mass m and triangular cross-section (AB = BC =
of a larger sphere of radius 6R (Fig. P-3). The masses of
CA = 2R) is moving with a constant velocity - viˆ towards a
large and small spheres are 4M and M, respectively, This
sphere of radius R fixed on a smooth horizontal table as
arrangement is placed on a horizontal table. There is no
shown in Figure. The wedge makes an elastic collision with
friction between any surfaces of contact. The small sphere
the fixed sphere and returns along the same path without
is now released. Find the coordinates of the centre of the
any rotation. Neglect all friction and suppose that the wedge
larger sphere when the smaller sphere reaches the other
remains in contact with the sphere for a very short time. Dt,
extreme position. (1996 - 3 Marks)
during which the sphere exerts a constant force F on the
wedge. (1998 - 8 Marks)
A

z y R
v

x B C
(a) Find the force F and also the normal force N exerted by
15. A uniform disc of mass m and radius R is projected
the table on the wedge during the time Dt.
horizontally with velocity v0 on a rough horizontal floor so
(b) Let h denote the perpendicular distance between the
that it starts off with a purely sliding motion at t= 0. After t0
centre of mass of the wedge and the line of action of
seconds, it acquires a purely rolling motion as shown in
F. Find the magnitude of the torque due to the normal
figure (1997C - 5 Marks)
force N about the centre of the wedge, during the
interval Dt .
v0 18. A uniform circular disc has radius R and mass m. A particle
also of mass m, is fixed at a point A on the edge of the disc as
shown in Figure. The disc can rotate freely about a fixed
t=0 t = t0 horizontal chord PQ that is at a distance R/4 from the centre
C of the disc. The line AC is perpendicular to PQ.
(i) Calculate the velocity of the center of mass of the disc Initially, the disc is held vertical with the point A at its highest
at t0. position. It is then allowed to fall so that it starts rotating
(ii) Assuming the coefficient of friction to be m, calculate about PQ. Find the linear speed of the particle as it reaches
t0. Also calculate the work done by the frictional force its lowest position. (1998 - 8 Marks)
as a function of time and the total work done by it over
a time t much longer than t0 A
16. Two thin circular disks of mass 2 kg and radius 10 cm each
are joined by a rigid massless rod of length 20 cm. The axis
of the rod is along the perpendicular to the planes of the R
disk through their centres. This object is kept on a truck in
such a way that the axis of the object is horizontal and
perpendicular to the direction of the motion of the truck. Its C
R/4
friction with the floor of the truck is large enough so that the
object can roll on the truck without slipping. Take x axis as P Q
the direction of motion of the truck and z axis as the vertically
upwardsdirection. If thetruck has an acceleration of 9 m/s2.
(1997 - 5 Marks)
19. A man pushes a cylinder of mass m1with the help of a plank
of mass m2 as shown in Figure. There in no slipping at any
contact. The horizontal component of the force applied by
the man is F. (1999 - 10 Marks)
O
F m2

20cm
M1
Calculate:
(i) The force of friction on each disk,
(ii) The magnitude and the direction of the frictional torque Find
acting on each disk about the centre of mass O of the (a) the accelerations of the plank and the center of mass of
object. Express the torque in the vector form in terms the cylinder, and
(b) the magnitudes and directions of frictional forces at
of unit vectors iˆ, ˆj and k̂ in the x, y, and z directions. contact points.
P-96 Topic-wise Solved Papers - PHYSICS
20. Two heavy metallic plates are joined together at 90o to each 22. Two masses m1 and m2 connected by a light spring of natural
other. A laminar sheet of mass 30 kg is hinged at the line AB length l0 is compressed completely and tied by a string. This
joining the two heavy metallic plates. The hinges are frictionless. system while moving with a velocity v0 along +ve x-axis pass
The moment of inertia of the laminar sheet about an axis parallel through the origin at t = 0. At this position the string snaps.
to AB and passing through its center of mass is 1.2 kg-m2. Two Position of mass m1 at time t is given by the equation x1 (t) =
rubber obstacles P and Q are fixed, one on each metallic plate
v0t – A (1 – coswt). Calculate (2003 - 4 Marks)
at a distance 0.5 m from the line AB. This distance is chosen
so that the reaction due to the hinges on the laminar sheet is (a) position of the particle m2 as a function of time
zero during the impact. (2001-10 Marks) (b) l0 in terms of A.
A
23. A wooden log of mass M and length L is hinged by a
frictionless nail at O. A bullet of mass m strikes with velocity
Q v and sticks to it. Find angular velocity of the system
immediately after the collision about O. (2005 - 2 Marks)
P B
O

L
M
Initially the laminar sheet hits one of the obstacles with an m
angular velocity 1 rad/s and turns back. If the impulse on v
the sheet due to each obstacle is 6 N-s,
(a) Find the location of the center of mass of the laminar 24. A cylinder of mass m and radius R rolls down an inclined
sheet from AB. plane of inclination q. Calculate the linear acceleration of
(b) At what angular velocity does the laminar sheet come the axis of cylinder. (2005 - 4 Marks)
back after the first impact? 25. Two identical ladders, each of mass M and length L are
(c) After how many impacts, does the laminar sheet come resting on the rough horizontal surface as shown in the
to rest? figure. A block of mass m hangs from P. If the system is in
21. Three particles A, B and C, each of mass m, are connected to equilibrium, find the magnitude and the direction of frictional
each other by three massless rigid rods to form a rigid, force at A and B. (2005 - 4 Marks)
equilateral triangular body of side l . This body is placed on
a horizontal frictioness table (x-y plane) and is hinged to it at P
the point A so that it can move without friction about the
L m
vertical axis through A (see figure). The body is set into
rotational motion on the table about A with a constant angular q q
velocity w. (2002 - 5 Marks ) A B
y
26. A rectangular plate of mass M and dimension a × b is held in
A horizontal position by striking n small balls (each of mass
x
m) per unit area per second. The balls are striking in the
w shaded half region of the plate. The collision of the balls
with the plate is elastic. What is v? (2006 - 6M)

b
F
B l C
a
(a) Find the magnitude of the horizontal force exerted by
the hinge on the body.
v
(b) At time T, when the side BC is parallel to the x-axis, a
force F is applied on B along BC (as shown). Obtain the
x-component and the y-component of the force exerted (Given n = 100, M = 3 kg, m = 0.01 kg; b = 2 m; a = 1 m;g = 10
by the hinge on the body, immediately after time T. m/s2).
ROTATIONAL MOTION P-97

MATCH THE FOLLOWING :


MUTLIPLE CHOICE QUESTIONS WITH ONE CORRECT
Each question contains statements given in two columns, which have to be matched. The statements in Column-I are labelled A,
B, C and D, while the statements in Column-II are labelled p, q, r, s and t. Any given statement in Column-I can have correct
matching with ONE OR MORE statement(s) in Column-II. The appropriate bubbles corresponding to the answers to these
questions have to be darkened as illustrated in the following example :
If the correct matches are A-p, s and t; B-q and r; C-p and q; and D-s then the correct darkening of bubbles will look like the given.
p q r s t
A p q r s t
B p q r s t
C p q r s t
D p q r s t

1. Column-II shows five systems in which two objects are labelled as X and Y. Also in each case a point P is shown. Column-I gives
some statements about X and/or Y. Match these statements to the appropriate system(s) from Column II. (2009)
Column-I Column II
Y
X
(A) The force exerted by X on Y has a magnitude Mg. (p)

P
Block Y of mass M left on a fixed inclined plane X, slides on
it with a constant velocity.

(B) The gravitational potential energy of X is (q) Z

Y
X

continuously increasing. Two ring magnets Y and Z, each of mass M, are kept in
frictionless vertical plastic stand so that they repel each other.
Y rests on the base X and Z hangs in air in equilibrium. P is
the topmost point of the stand on the common axis of the
two rings. The whole system is in a lift that is going up with
a constant velocity.

(C) Mechanical energy of the system X +Y is (r)


continuously decreasing.
P
Y

A pulley Y of mass m0 is fixed to a table through a clamp X. A


block of mass M hangs from a string that goes over the
pulley and is fixed at point P of the table. The whole system
is kept in a lift that is going down with a constant velocity.
P-98 Topic-wise Solved Papers - PHYSICS

Y
(D) The torque of the weight of Y about point P is zero. (s)

X
P
A sphere Y of mass M is put in a non-viscous liquid X kept in
a container at rest. The sphere is released and it moves down
in the liquid.

Y
(t)

X
P
A sphere Y of mass M is falling with its terminal velocity in a
viscous liquid X kept in a container.

The unstretched length of each spring is L. The disk is initially at


COMPREHENSION BASED Q UESTIONS :
its equilibrium position with its centre of mass (CM) at a distance
PASSAGE - 1 L from the wall. The disk rolls without slipping with velocity
Two discs A and B are mounted coaxially on a vertical axle. The r
discs have moments of inertia I and 2 I respectively about the V0 = V0ˆi . The coefficient of friction is µ. (2008)
common axis. Disc A is imparted an initial angular velocity 2 w
using the entire potential energy of a spring compressed by a y
distance xl. Disc B is imparted an angular velocity w by a spring
having the same spring constant and compressed by a distance 2d
x2 .Both the discs rotate in the clockwise direction.
1. The ratio x1/x2 is (2007)
1 1
(a) 2 (b) (c) 2 (d)
2 2 d
2. When disc B is brought in contact with disc A, they acquire d
R
a common angular velocity in time t . The average frictional V0
torque on one disc by the other during this period is (2007) x
4. The net external force acting on the disk when its centre of
2I w 9I w 9I w 3I w
(a) (b) (c) (d) mass is at displacement x with respect to its equilibrium
3t 2t 4t 2t position is
3. The loss of kinetic energy in the above process is (2007)
(a) – kx (b) – 2kx
I w2 I w2 I w2 I w2 (c) – 2kx/3 (d) – 4kx/3
(a) (b) (c) (d)
2 3 4 6 5. The centre of mass of the disk undergoes simple harmonic
PASSAGE - 2 motion with angular frequency w equal to –
A uniform thin cylindrical disk of mass M and radius R is attached k 2k
to two identical massless springs of spring constant k which are (a) (b)
fixed to the wall as shown in the figure. The springs are attached M M
to the axle of the disk symmetrically on either side at a distance d 2k 4k
from its centre. The axle is massless and both the springs and the (c) (d)
axle are in horizontal plane. 3M 3M
ROTATIONAL MOTION P-99
6. The maximum value of V0 for which the disk will roll without 7. Which of the following statements about the instantaneous
slipping is – axis (passing through the centre of mass) is correct?
(a) It is vertical for both the cases (a) and (b)
M M (b) It is vertical for case (a); and is at 45° to the x-z plane
(a) mg (b) mg
k 2k and lies in the plane of the disc for case (b).
3M 5M (c) It is horizontal for case (a); and is at 45° to the x-z plane
(c) mg (d) mg and is normal to the plane of the disc for case (b).
k 2k
PASSAGE-3 (d) It is vertical for case (a); and is 45° to the x-z plane and
is normal to the plane of the disc for case (b).
The general motion of a rigid body can be considered to be a
combination of (i) a motion of its centre of mass about an axis, and 8. Which of the following statements regarding the angular
(ii) its motion about an instantaneous axis passing through the speed about the instantaneous axis (passing through the
centre of mass. centre of mass) is correct?
These axes need not be stationary. Consider, for example, a thin (a) It is 2w for both the cases
uniform disc welded (rigidly fixed) horizontally at its rim to a
massless, stick, as shown in the figure. When the disc-stick system (b) It is w for case (a); and w/ 2 for case (b)
is rotated about the origin on a horizontal frictionless plane with (c) It is w for case (a); and 2w for case (b)
angular speed w, the motion at any instant can be taken as a (d) It is w for both the cases.
combination of (i) a rotation of the centre of mass of the disc
about the z-axis and (ii) a rotation of the disc through an ASSERTION & REASON TYPE QUESTIONS :
instantaneous vertical axis passing through its centre of mass (as
is seen from the changed orientation of points P and Q). Both 1. STATEMENT-1 (2007)
these motions have the same angular speed w in this case If there is no external torque on a body about its center of
(2012) mass, then the velocity of the center of mass remains constant.
STATEMENT-2
The linear momentum of an isolated system remains constant.
(a) Statement-1 is True, Statement-2 is True; Statement-2
is a correct explanation for Statement-1
(b) Statement-1 is True, Statement-2 is True; Statement-2
is NOT a correct explanation for Statement-1
(c) Statement-1 is True, Statement-2 is False
(d) Statement-1 is False, Statement-2 is True
2. STATEMENT-1 : Two cylinders, one hollow (metal) and the
other solid (wood) with the same mass and identical
Now consider two similar systems as shown in the figure: Case (a) dimensions are simultaneously allowed to roll without
the disc with its face vertical and parallel to x-z plane; Case (b) the slipping down an inclined plane from the same height. The
disc with its face making an angle of 45° with x-y plane and its hollow cylinder will reach the bottom of the inclined plane
horizontal diameter parallel to x-axis. In both the cases, the disc is first.
welded at point P, and the systems are rotated with constant
and
angular speed w about the z-axis.
STATEMENT-2 : By the principle of conservation of
z Q energy, the total kinetic energies of both the cylinders are
w identical when they reach the bottom of the incline.
(2008)
(a) STATEMENT-1 is True, STATEMENT-2 is True;
STATEMENT-2 is a correct explanation for
y STATEMENT-1
P
(b) STATEMENT-1 is True, STATEMENT-2 is True;
x STATEMENT-2 is NOT a correct explanation for
Case (a) STATEMENT-1
(c) STATEMENT -1 is True, STATEMENT-2 is False
z (d) STATEMENT -1 is False, STATEMENT-2 is True
w Q
INTEGER VALUE CORRECT TYPE :
1. A binary star consists of two stars A (mass 2.2Ms) and B
45° y (mass 11Ms), where Ms is the mass of the sun. They are
P separated by distance d and are rotating about their centre
of mass, which is stationary. The ratio of the total angular
x Case (b) momentum of the binary star to the angular momentum of
star B about the centre of mass is (2010)
P-100 Topic-wise Solved Papers - PHYSICS
2. A boy is pushing a ring of mass 2 kg and radius 0.5 m with a 4. A lamina is made by removing a small disc of diameter 2R
stick as shown in the figure. The stick applies a force of 2N from a bigger disc of uniform mass density and radius 2R, as
shown in the figure. The moment of inertia of this lamina
on the ring and rolls it without slipping with an acceleration
about axes passing though O and P is IO and IP respectively.
of 0.3 m/s2. The coefficient of friction between the ground Both these axes are perpendicular to the plane of the lamina.
and the ring is large enough that rolling always occurs and The ratio IP / IO to the nearest integer is (2012)
the coefficient of friction between the stick and the ring is
(P/10). The value of P is (2011)

5. A uniform circular disc of mass 50 kg and radius 0.4 m is


rotating with an angular velocity of 10 rad s–1 about its own
axis, which is vertical. Two uniform circular rings, each of
mass 6.25 kg and radius 0.2 m, are gently placed symmetrically
3. Four solid spheres each of diameter 5 cm and mass 0.5 kg on the disc in such a manner that they are touching each
are placed with their centers at the corners of a square of other along the axis of the disc and are horizontal. Assume
side 4 cm. The moment of inertia of the system about the that the friction is large enough such that the rings are at
diagonal of the square is N ´ 10–4 kg– m2, then N is.(2011) rest relative to the disc and the system rotates about the
original axis. The new angular velocity (in rad s–1) of the
system is (JEE Adv. 2013)

1. Initial angular velocity of a circular disc of mass M is w 1. 6. Moment of inertia of a circular wire of mass M and radius R
Then two small spheres of mass m are attached gently to about its diameter is [2002]
diametrically opposite points on the edge of the disc. What (a) MR2/2 (b) MR2
is the final angular velocity of the disc? [2002] (c) 2MR2 (d) MR2/4.
æ M + mö æ M + mö 7. A particle of mass m moves along line PC with velocity v as
(a) çè ÷w (b) çè ÷w shown. What is the angular momentum of the particle
M ø 1 m ø 1
about P? [2002]
æ M ö æ M ö
(c) çè ÷w (d) çè ÷ w1.
M + 4m ø 1 M + 2mø C
2. The minimum velocity (in ms–1) with which a car driver must L
traverse a flat curve of radius 150 m and coefficient of friction
0.6 to avoid skidding is [2002] P
(a) 60 (b) 30 l
(c) 15 (d) 25 O
3. A cylinder of height 20 m is completely filled with water. The (a) mvL (b) mvl
velocity of efflux of water (in ms–1) through a small hole on (c) mvr (d) zero.
the side wall of the cylinder near its bottom is 8. A circular disc X of radius R is made from an iron plate of
(a) 10 (b) 20 [2002] thickness t, and another disc Y of radius 4R is made from an
(c) 25.5 (d) 5 t
4. Two identical particles move towards each other with iron plate of thickness . Then the relation between the
4
velocity 2v and v respectively. The velocity of centre of
mass is [2002] moment of inertia I X and IY is [2003]
(a) v (b) v/3 (a) ΙY = 32 Ι X (b) ΙY = 16 Ι X
(c) v/2 (d) zero.
5. A solid sphere, a hollow sphere and a ring are released from (c) Ι Y = Ι X (d) ΙY = 64 Ι X
top of an inclined plane (frictionless) so that they slide down 9. A particle performing uniform circular motion has angular
the plane. Then maximum acceleration down the plane is for frequency is doubled & its kinetic energy halved, then the
new angular momentum is [2003]
(no rolling) [2002]
(a) solid sphere (b) hollow sphere L L
(a) (b) 2 L (c) 4 L (d)
(c) ring (d) all same. 4 2
ROTATIONAL MOTION P-101
r 16. Consider a two particle system with particles having masses
10. Let F be the force acting on a particle having position
r r m1 and m2. If the first particle is pushed towards the centre
vector r , and T be the torque of this force about the origin. of mass through a distance d, by what distance should the
Then [2003] second particle is moved, so as to keep the centre of mass at
r r r r the same position? [2006]
(a) r .T = 0 and F .T ¹ 0
r r r r m2
d
m1
d
(b) r .T ¹ 0 and F .T = 0 (a) (b)
m1 m1 + m2
r r r r
(c) r .T ¹ 0 and F .T ¹ 0
m1
r r r r (c) d (d) d
(d) r .T = 0 and F .T = 0 m2
11. A solid sphere is rotating in free space. If the radius of the 17. Four point masses, each of value m, are placed at the corners
sphere is increased keeping mass same which one of the of a square ABCD of side l. The moment of inertia of this
following will not be affected ? [2004] system about an axis passing through A and parallel to BD
(a) Angular velocity is [2006]
(b) Angular momentum 2 2
(c) Moment of inertia (a) 2ml (b) 3ml
(d) Rotational kinetic energy (c) 3ml 2 (d) ml 2
12. One solid sphere A and another hollow sphere B are of same
mass and same outer radii. Their moment of inertia about 18. A force of –Fkˆ acts on O, the origin of the coordinate
their diameters are respectively I A and I B Such that system. The torque about the point (1, –1) is [2006]
[2004] Z
(a) I A < I B (b) I A > I B (a) F (iˆ - ˆj )
IA dA (b) - F (iˆ + ˆj )
(c) I A = I B (d) I = d
B B O
(c) F (iˆ + ˆj ) Y
where d A and d B are their densities.
13. A body A of mass M while falling vertically downwards (d) - F (iˆ - ˆj )
1 X
under gravity breaks into two parts; a body B of mass 19. A thin circular ring of mass m and radius R is rotating about
3 its axis with a constant angular velocity w. Two objects
2 each of mass M are attached gently to the opposite ends of
M and a body C of mass M. The centre of mass of
3 a diameter of the ring. The ring now rotates with an angular
bodies B and C taken together shifts compared to that of velocity w' = [2006]
body A towards [2005]
(a) does not shift w(m + 2 M ) w(m - 2 M )
(a) (b)
(b) depends on height of breaking m (m + 2 M )
(c) body B
(d) body C wm wm
(c) (d)
14. The moment of inertia of a uniform semicircular disc of mass (m + M ) (m + 2M )
M and radius r about a line perpendicular to the plane of the 20. A circular disc of radius R is removed from a bigger circular
disc through the centre is [2005] disc of radius 2R such that the circumferences of the discs
2 1 coincide. The centre of mass of the new disc is a / R form
(a) Mr 2 ` (b) Mr the centre of the bigger disc. The value of a is
5 4
(a) 1/4 (b) 1/3 [2007]
1 (c) 1/2 (d) 1/6
(c) Mr 2 (d) Mr 2
2 21. A round uniform body of radius R, mass M and moment of
15. A ‘T’ shaped object with dimensions shown in the figure, is inertia I rolls down (without slipping) an inclined plane
uur making an angle q with the horizontal. Then its acceleration
lying on a smooth floor. A force ‘ F ’ is applied at the point
is [2007]
P parallel to AB, such that the object has only the
translational motion without rotation. Find the location of P g sin q g sin q
(a) (b)
with respect to C. l [2005] 1 - MR / I2
1 + I / MR 2
3 A B g sin q g sin q
(a) l (c) (d)
2 1 + MR 2 / I 1 - I / MR 2
2 22. Angular momentum of the particle rotating with a central
(b) l P
3 2l force is constant due to [2007]
F
(c) l (a) constant torque
(b) constant force
4
(d) l (c) constant linear momentum
3 (d) zero torque
C
P-102 Topic-wise Solved Papers - PHYSICS
23. For the given uniform square lamina ABCD, whose centre 27. A mass m hangs with the help of a string wrapped around
is O, [2007] a pulley on a frictionless bearing. The pulley has mass m
D F C and radius R. Assuming pulley to be a perfect uniform
(a) I AC = 2 I EF
circular disc, the acceleration of the mass m, if the string
does not slip on the pulley, is: [2011]
(b) 2 I AC = I EF
O 2
(c) I AD = 3I EF (a) g (b) g
3
(d) I AC = I EF A B
E g 3
24. A thin rod of length ‘L’ is lying along the x-axis with its ends (c) (d) g
3 2
at x = 0 and x = L. Its linear density (mass/length) varies with
28. A thin horizontal circular disc is rotating about a vertical
æ xö
n axis passing through its centre. An insect is at rest at a
x as k ç ÷ , where n can be zero or any positive number. If point near the rim of the disc. The insect now moves along
è Lø
a diameter of the disc to reach its other end. During the
the position xCM of the centre of mass of the rod is plotted journey of the insect, the angular speed of the disc.
against ‘n’, which of the following graphs best approximates [2011]
the dependence of xCM on n? [2008]
(a) continuously decreases
xCM xCM (b) continuously increases
L (c) first increases and then decreases
L L (d) remains unchanged
(a) (b) 2
2
n n 29. A pulley of radius 2 m is rotated about its axis by a force
O O
F = (20t – 5t2) newton (where t is measured in seconds)
xCM xCM applied tangentially. If the moment of inertia of the pulley
about its axis of rotation is 10 kg-m2 the number of rotations
L L made by the pulley before its direction of motion is
L L
(c) 2
(d) 2
reversed, is: [2011]
n n (a) more than 3 but less than 6
O O
(b) more than 6 but less than 9
25. Consider a uniform square plate of side ‘a’ and mass ‘m’.
(c) more than 9
The moment of inertia of this plate about an axis
perpendicular to its plane and passing through one of its (d) less than 3
corners is [2008] 30 . A hoop of radius r and mass m rotating with an angular
velocity w0 is placed on a rough horizontal surface. The
5 2 1
(a) ma (b) ma 2 initial velocity of the centre of the hoop is zero.What will
6 12 be the velocity of the centre of the hoop when it ceases to
slip ? [JEE MAIN 2013]
7 2 2
(c) ma 2 (d) ma
12 3 rw0 rw0
(a) (b)
26. A thin uniform rod of length l and mass m is swinging freely 4 3
about a horizontal axis passing through its end. Its maximum
angular speed is w. Its centre of mass rises to a maximum rw0
(c) (d) rw0
height of : [2009] 2

1 lw 1 l 2w 2
(a) (b)
6 g 2 g

1 l 2w 2 1 l 2w2
(c) (d)
6 g 3 g
ROTATIONAL MOTION P-103

Solutions & Explanations


Section-A : JEE Advanced/ IIT-JEE
2 M w0 1
A 1. mg 2. 3. MRAw 2 4. –3
3 M + 6m 3

mg æ d - xö xW
5. , direction upward 6. çè ÷W , 7. 4.8 Ma2
6 d ø d
B 1. F 2. F 3. F 4. F
C 1. (c) 2. (c) 3. (b) 4. (c) 5. (c) 6. (a) 7. (a) 8. (c) 9. (d)
10. (b) 11. (a) 12. (b) 13. (b) 14. (a) 15. (a) 16. (b) 17. (b) 18. (d)
19. (a) 20. (b) 21. (b) 22. (d) 23. (d) 24. (a) 25. (c) 26. (b)
D 1. (d) 2. (b) 3. (a,c) 4. (b, d) 5. (a, c, d) 6. (a, b, c) 7. (a) 8. (c)
9. (a) 10. (a, b, c) 11. (c, d) 12. (a) 13. (b,c) 14. (a, b, c) 15. (c) 16. (a, b) 17. (d)
1 gM
E 1. T = mg[3cos q - 2cos q0 ] , q0 = 30° 2. n=
2p lm
m( R - r ) 2 g (R - r)
3. 9 cm from the origin towards left 4. ,m
M +m M (m + M )

mv03
5. , perpendicular to the plane of motion and is directed away from the reader.. 6. 2m, yes
2 2g

14 12v
7. 2.72 J 8. Rg 9. (a) (b) 3.5 ms–1
3 7L
2 2
é x ù æ yö
10. (i) Straight line, (ii) ê
L ú + çè r ÷ø = 1, Ellipse 11. (i) 1.63N (ii) 1.22 m 12. 6.3 m/s
ê -rú
ë2 û

4 4 gR
13. (a) q = cos -1 (b) (c) 6 14. (L + 2R, 0)
7 7

2v0 v0 mmgt0 mv02


15. (i)
3
; (ii)
3mg
,W =
2
[ 0 0] 6
3 µgt - 2v ; – 16. (i) 6 î (ii) 0.6(kˆ - ˆj ), 0.6(- ˆj - kˆ)

2 mv æ 2 mv ö 4mv
17. (a) ( 3iˆ - kˆ), ç + mg ÷ kˆ (b) ´h 18. 5gR
3Dt è 3Dt ø 3Dt
8F 4F 3FM1 FM1
19. (a) 3M + 8m , 3M + 8m (b) 3M + 8m , 3M + 8m
1 2 1 2 1 2 1 2
F 2
20. (a) 0.1 m (b) 1 radian/sec. (c) infinite 21. (a) 3mlw 2 (b) ( Fnet ) x = - , ( Fnet ) y = 3mlw
4
m1 æ m1 ö 3mv 2
22. (a) v0 t + A(1 - cos wt ) (b) ç +1 A 23. w = 24. a = g sin q
m2 è m2 ÷ø ( M + 3m) L 3
é gù
25. ê( M + m ) ú cot q, along AB. 26. 10 m/s
ë 2û
F 1. (A) ® p, t; (B) ® q, s, t; (C) ® p, r, t; (D) ® q, p
G 1. (c) 2. (a) 3. (b) 4. (d ) 5. (d) 6. (c) 7. (a) 8. (b)
H 1. (d) 2. (d)
I 1. 6 2. 4 3. 9 4. 3 5. 8
`
P-104 Topic-wise Solved Papers - PHYSICS
Section-B : JEE Main/ AIEEE

1. (c) 2. (b) 3. (b) 4. (c) 5. (d) 6. (a) 7. (d) 8. (d) 9. (a)


10. (d) 11. (b) 12. (a) 13. (a) 14. (c) 15. (d) 16. (c) 17. (c) 18. (c)
19. (d) 20. (b) 21. (b) 22. (d) 23. (d) 24. (a) 25. (d) 26. (c) 27. (b)
28. (c) 29. (a) 30. (c)

FILL IN THE BLANKS : 2f 2f


or, a = or Ra =
MR M
1. KEY CONCEPT
When the cube begins to tip about the edge the normal \ Equivalent linear acceleration ( Ra = a2 )
reaction will pass through the edge about which rotation
takes place. The torque due to N and f will be zero. 2f
a2 =
Taking moment of force about D M
\ Total linear acceleration,
3a a 2
F× = mg ´ \ F = mg amax = a1 + a2
4 2 3
f 2f 3f
N = + =
M M M
B C
F 3f
or, Aw 2 =
M
3a/4
MAw 2
A f D or, f =
mg 3
Thus, maximum torque,
2. Note : Since no external force and hence no torque is applied, tmax = f × R
the angular momentum remains constant
\ I1w1 = I2w2 MAw 2 R 1 2
= = MARw
3 3
ML2 4. Let at any instant of time t, the radius of the horizontal
Iw ´ w0 M w0
\ w2 = 1 1 = 12 = surface be r.
I2 ML2 æ Lö
2 M + 6m T = mrw2 ... (i)
+ 2m ´ ç ÷ Where m is the mass of stone and w is the angular velocity
12 è 2ø
at that instant of time t.
3. Considering the motion of the platform
x = A cos wt

dx d2x w
Þ = - Aw sin wt Þ = - Aw 2 cos wt r
dt 2
dt
The magnitude of the maximum acceleration of the platform T m
is
\ | Max acceleration | = Aw2 Also, L = Iw ... (ii)
When platform moves a torque acts on the cylinder and the From (i) and (ii)
cylinder rotates about its axis.
mrL2 mL2 æ L2 ö
Acceleration of cylinder, a1 =
f T= = ´ r , T = ç ÷ r -3
I2 (mr 2 ) 2 è mø
m
Torque t = f R
æ L2 ö
Ia = f R
= Ar -3 çè where = A is constant ÷
ø
m
fR fR
a = = Thus, n = –3
I MR 2 / 2
ROTATIONAL MOTION P-105
5. Given that the body is rolling up the inclined plane. Therefore 7. Assuming symmetric lamina to be in xy plane, we will have
the velocity of centre of mass is up the inclined plane. The
Y
component of weight (mg sin q ) is trying to move the point A B
of contact downwards. Therefore frictional force will act a
upwards.
a
N
w f X
O
O
sq
q g co
m
sinq mg Ix = Iy (Since the mass distribution is same about x-axis and
mg q = 30° y-axis)
Ix + Iy = Iz (perpendicular-axis theorem)
From Force diagram It is given that Iz = 1.6 Ma2.
mg sin q – f = mac ... (i) Hence
For rolling motion about the centre of mass of the disc O, Iz
f×R=Ia Ix = Iy = = 0.8 Ma 2
2
ac Iac Now, according to parallel-axis theorem, we get
Þ But ac = aR Þ a = , f×R= IAB = Ix + M(2a)2
R R = 0.8 Ma2 + 4Ma2
= 4.8 Ma2
fR 2 fR 2 2f
Þ ac = = = ... (ii) TRUE / FALSE :
I 1 m
mR 2
2
t
From (i) and (ii) 1. t = Ia \ a =
I
2f t = Force × perpendicular distance. Torque is same in both
mg sin q - f = m ´ the cases. But since, I will be different due to different mass
m distribution about the axis,
mg \ a will be different.
Þ 3f = mg sin q = mg sin 30° Þ f = ur
6 r dL
2. t= w
6. Let RA and RB be the reactions at the knife edges A and B, dt
respectively (as shown in the figure). For the rod to be in r M
Since, t =0
equilibrium in a horizontal position, the moment of forces ur
\ L = constant R
about the knife edges must be equal to zero.
\ I1w1 = I2w2
RA RB 1
I1 = MR 2
2
w1 = w
CM
1 1 M 2 æ 4 + 1ö 5
A x d–x B I2 = MR 2 + R =ç 2
÷ø MR = MR
2
2 2 4 è 8 8
W w2 = ?
1
Moment of forces about A I1w1 MR 2 ´ w
2 8 4
RA . 0 + xW – RBd = 0 w2 = =+ = w= w
I2 5 2´5 5
MR 2
xW 8
i.e., RB =
d 3. Total energy of the ring
Moment of forces about B = (K.E.)Rotation + (K.E.)Translational
RA . d – (d – x) W – RB0 = 0 1 2 1 2
= I w + mvc
2 2
æ d - xö
i.e., RA = ç W
è d ÷ø =
1 1
´ mr 2 w 2 + m(r w )2 (Q I = mr2, vc = rw)
2 2
ALTERNATE SOLUTION : RA + RB = W
\ RA = W – RB = mr 2 w 2
P-106 Topic-wise Solved Papers - PHYSICS
Total kinetic energy of the cylinder Dividing equation (i) by equation (ii)
= (K.E.)Rotation + (K.E.)Translational
1 K
1 ( 0.3) v12
2 1 2 2 2
= I ' w ' + Mv 'c
2 2 1 = 2
( 0.4) v22 K
1æ1 2ö 2 1 2
2 3
= çè Mr ÷ø w ' + M (r w ')
2 2 2 v1
or v2 = 1
3
= Mr 2 w '2 ... (i)
4 or, v1 = v2
Equating (i) and (ii) Thus, both will reach the wall at the same time.
4. Since no external force is acting on the two particle system
3 \ ac.m = 0
mr2w2 = Mr 2 w '2
4 Þ Vc.m = Constant.

w '2 4m 4 0.3 MCQ's WITH ONE CORRECT ANSWER :


Þ = = ´ =1
w2 3M 3 0.4 1. (c) Since the objects are placed gently, therefore no
external torque is acting on the system. Hence angular
Þ w' = w
momentum is constant.
Both will reach at the same time.
ALTERNATE SOLUTION : w1 w2
1 2
In case of ring KR = Iw
2 r
m m
1 2 2
=
2
(
mr w Q I = mr 2 )
2 2 2
I = Mr I2=Mr +mr
1 2
KT = mvc i.e., I1w1 = I2 w2
2
1 2 1 2 2
Mr2 × w1 = (Mr2 + 2mr2) w2 (Q w1 = w )
= m ( r w ) = mr w
2 2 Mw
\ w2 =
M + 2m
\ KR = KT 2. (c) The moment of inertia of the system about axis of
Total kinetic energy, K = KR + KT rotation O is
I = I1 + I2
K
or, K R = KT = = 0.3x2 + 0.7 (1.4 – x)2
2 = 0.3x2 + 0.7 (1.96 + x2 – 2.8x)
1 K = x2 + 1.372 – 1.96x
or, mv12 = ...(i)
2 2 0.3 kg

1 2 x
In case of solid cylinder, KR = Iw
2
O
1æ1 2ö 2
= çè MR ÷ø w
2 2
1.4 – x
1
= MR 2 w 2
4
0.7 kg
1
KT = MR 2 w 2 The work done in rotating the rod is converted into its
2 rotational kinetic energy.
KR 1 1 2 1 2
\ \ W= I w = [ x + 1.372 - 1.96 x] w 2
KT = 2 2 2
For work done to be minimum
2
or KT =
3
K (\ K = K R + KT ) dW
=0 Þ 2x – 1.96 = 0
dx
1 2
or Mv2 2 = K (ii) 1.96
= 0.98 m
2 3 Þ x=
2
ROTATIONAL MOTION P-107
3. (b) Angular momentum of mass m moving with a constant a
velocity about origin is r v M
a C
line parallel v 6. (a) M r w
to x-axis m q
y p
x O O
O 2
a a
O r= 2 or r2 =
2 2
L = momentum × perpendicular distance of line of action Net torque about O is zero.
of momentum from origin L = mv × y Therefore, angular momentum (L) about O will be
In the given condition mvy is a constant. Therefore conserved, or Li = Lf
angular momentum is constant. æ aö 2
ur r ur MV ç ÷ = I 0 w = ( Icm + Mr )w
è 2ø
Also, L = r ´ p
ur ìï Ma 2 æ a 2 ö üï 2 2
The direction of L is perpendicular to the plane of w= í + M ç ÷ ý w = Ma w
6 è 2 ø 3
paper directed inwards which remains constant. îï þï
4. (c) As the spheres are smooth there will be no friction (no 3v
torque) and therefore there will be no transfer of angular w=
4a
momentum. Thus A, after collision will remain with its 7. (a) Note : When we are giving an angular acceleration to
initial angular momentum. i.e., w A = w the rod, the bead is also having an instantaneous
acceleration a = La. This will happen when a force is
5. (c) KEY CONCEPT exerted on the bead by the rod. The bead has a tendency
The disc has two types of motion namely translational to move away from the centre. But due to the friction
and rotational. Therefore there are two types of angular between the bead and the rod, this does not happen to
momentum and the total angular momentum is the the extent to which frictional force is capable of holding
vector sum of these two. the bead.
The frictional force here provides the necessary
In this case both the angular momentum have the same centripetal force. If instantaneous angular velocity is
direction (perpendicular to the plane of paper and away w then
from the reader).
a
y

L
w
mLw2 = µ(ma)
R mLw2 = µ mLa
v
Þ w2 = µa
C By applying
Þ w = w0 + at,
O x We get w = at
\ a2t2 = µa
ur ur ur
L = LT + L R µ
Þ t=
LT = angular momentum due to translational motion. a
8. (c) The applied force shifts the normal reaction to one
LR = angular momentum due to rotational motion
corner as shown. At this situation, the cubical block
about C.M. starts topping about O. Taking torque about O
L = MV × R + Icmw
Icm = M.I. about centre of mass C. F L
N
1 2
= M (R w) R + MR w
2 L/2
(v = Rw in case of rolling motion and surface at rest)
O
3 2 mg
= MR w
2
L mg
F × L = mg × Þ F=
2 2
P-108 Topic-wise Solved Papers - PHYSICS
9. (d) Moment of inertia about the diameter of the circular ALTERNATE SOLUTION :The mass distribution of
this sector is same about the axis of rotation as that of
1
loop (ring) = MR 2 the complete disc about the axis. Therefore the formula
2 remains the same as that of the disc.
Using parallel axis theorem 12. (b) Imagine the cylinder to be moving on a frictionless
The moment of inertia of the loop about XX' axis is surface. In both the cases the acceleration of the centre
MR 2 3 of mass of the cylinder is g sin q. This is also the
IXX' = + MR 2 = MR 2 acceleration of the point of contact of the cylinder with
2 2 the inclined surface. Also no torque (about the centre
Where M = mass of the loop and R = radius of the loop of cylinder) is acting on the cylinder since we assumed
L the surface to be frictionless and the forces acting on
Here M = Lr and R = ; the cylinder is mg and N which pass through the centre
2p
of cylinder. Therefore the net movement of the point of
2
3 æ Lö 3L3r contact in both the cases is in the downward direction
\ IXX' =( Lr) ç ÷ = as shown. Therefore the frictional force will act in the
2 è 2p ø 8p 2
upward direction in both the cases.
10. (b) The M.I. about the axis of rotation is not constant as
the perpendicular distance of the bead with the axis of
rotation increases. F
Also since no external torque is acting. g sin q Point of
N
contact
dL g sin q
\ text = Þ L = constant Þ Iw = constant
dt
Since, I increases, w decreases. mg sin q
11. (a) Note : We cannot consider the quadrant as a single mg
mass as the distance of different particles is different q
from the axis of rotation. So, we take the help of calculus. Note : In general we find the acceleration of the point
Let us consider a segment as shown in the figure. All of contact due to translational and rotational motion
masses lying in this segment are at a distance r from and then find the net acceleration of the point of contact.
the axis and hence considered as a small differential The frictional force acts in the opposite direction to
mass dm. Let the thickness of the segment be dr. that of net acceleration of point of contact.
n w 13. (b) Since there is no external
torque, angular momentum C
remains conserved. As
moment of inertia initially
dr decreases and then B
r increases, so w will increase
initially and then decreases.
R A
Note : The M.I. of the system decreases when the
tortoise move from A to B and then increases from B
n to A.
The mass per unit area of the quadrant So the variation of w is nonlinear.
M 4M 14. (a) Change in angular momentum of the system = angular
= = impulse given to the system about the centre of mass
2 2
pR / 4 pR (Angular momentum)f – (Angular momentum)i
Area of segment
L
1 2 2 1 prdr = Mv × ... (i)
= [p(r + dr ) - pr ] = ´ 2prdr = 2
4 4 2 Let the system starts rotating with the angular
\ Mass of the segment, velocity w.
prdr 4M 2 M Moment of Inertia of the system about its axis of rotation
dm = ´ 2 = 2 rdr [centre of mass of the system]
2 pR R
2 2
æ Lö æ Lö 2ML2 ML2
æ 2M ö = M ç ÷ +Mç ÷ = =
\ M.I. of this segment about nn' = ç 2 rdr ÷ ´ r 2 è 2ø è 2ø 4 2
è R ø
From (i)
2M
= ´ r 3dr L
R 2 Iw – 0 = Mv
2
\ M.I. of the quadrant about nn'
Mv L Mv L v
R 2M R 4 MR 2
2M Þ w= ´ = 2
´ =
I 2 ML / 2 2 L
ò0
3
= ´ r dr = ´ =
R2 R2 4 2
ROTATIONAL MOTION P-109
ur 18. (d) The cubical block is in equilibrium.
15. (a) | L |= mvr
For translational equilibrium
L (a) SFx = 0 Þ F=N
(b) SFy = 0 Þ f = mg
r y
O p=mv f
ur 2a x
The direction of L (about the center) is perpendicular
to the plane containing the circular path. Both a
magnitude and direction of the angular momentum of
the particle moving in a circular path about its center O b C F
is constant. N
ALTERNATE SOLUTION :The net force acting on a
particle undergoing uniform circular motion is
centripetal force which always passes through the mg
centre of the circle. The torque
ur due to this force about
the centre is zero, therefore, L is conserved about O. For Rotational Equilibrium
16. (b) KEY CONCEPT: The angular momentum (L) is Stc = 0
conserved, since text is zero. Where tc = torque about c.m.
Here, Ii = I0 If = 2I0 Torque created by frictional force (f) about C = f × a in
wi = w0 wf = w (say) clockwise direction.
There should be another torque which should counter
w0 this torque. The normal reaction N on the block acts as
Then I0w0 = 2I0w Þ w =
2 shown. This will create a torque N × b in the
anticlockwise direction.
1
Ki = I 0 w 20 = K Such that f×a=N×b
2 Note : The normal force does not act through the centre
2
w2 of the body always. The point of application of normal
1 æw ö 1
Kf = (2 I 0 ) ´ ç 0 ÷ = ´ 2 I 0 ´ 0 force depends on all the forces acting on the body.
2 è 2 ø 2 4 19. (a) Let s be the mass per unit area.
ì1
( ü 1 K
= í I 0 w20 ý ´ =
î 2 þ 2 2
) R/ 3
ALTERNATE SOLUTION : O'
L2
KEY CONCPET : (K.E.)rotation = .
2I 2R/3
Here , L = constant
\ (K.E.)rotational × I = constant. O
When I is doubled, K.E.rotational becomes half.
17. (b) Here vc = vc
vP = vc2 + w 2 r 2
vQ = vc2 + w 2 r 2 + 2vc wr cos q
w
The total mass of the disc
= s × p R2 = 9M
The mass of the circular disc cut
2
vc P vc æ Rö p R2
wr=v'
C = s´pç ÷ = s´=M
è 3ø 9
v'=wr
Q vc Let us consider the above system as a complete disc of
O mass 9M and a negative mass M super imposed on it.
Moment of inertia (I 1 ) of the complete disc =
ALTERNATE SOLUTION 1
Note : In pure rolling, the point of contact is the 9 MR 2 about an axis passing th rough O and
instantaneous centre of rotation of all the particles of 2
the disc. On applying v = rw perpendicular to the plane of the disc.
We find w is same for all the particles then v µ r. M.I. of the cut out portion about an axis passing through
Farther the particles from O, higher is its velocity. O' and perpendicular to the plane of disc
P-110 Topic-wise Solved Papers - PHYSICS
2 For solid disc
1 æ Rö
= ´M ´ç ÷ 1 3
2 è 3ø IA'B' = IYY' + Mr2 = Mr 2 + Mr 2 = Mr 2
2 2
\ M.I. (I2) of the cut out portion about an axis passing IAB = IA'B' (given) ... (ii)
through O and perpendicular to the plane of disc
From (i) and (ii),
é1 æ Rö
2
æ 2R ö ù
2
2
2 3
= ê ´M ´ç ÷ +M ´ç ÷ ú MR 2 = Mr 2 Þ r = R
êë 2 è 3ø è 3 ø ú 5 2 15
û
[Using perpendicular axis theorem] 22. (d) By the concept of energy conservation
\ The total M.I. of the system about an axis passing æ 3v 2 ö
1 2 1 2
through O and perpendicular to the plane of the disc is mv + I w = mg ç ÷
I = I1 + I2 2 2 è 4g ø
é1 For rolling motion v = Rw
æ 2R ö ù
2 2
1 2 æ Rö
= 9 MR - ê ´ M ´ ç ÷ + M ´ ç ÷ ú
2 êë 2 è 3ø è 3 ø ú 1 2 1 v2 3 2
û \ mv + I 2 = mv
2 2 R 4
1 é 1 4ù
= 9 MR 2 - MR 2 ê + ú
2 ë18 9 û 1 v2 3 2 1 2 1 2
\ I = mv - mv = mv
2 R2 4 2 4
1 2 2 é1 + 8 ù
= 9 MR - MR ´ ê
2 ë 18 úû 1 v2 1 2
I = mv
9MR 2 9MR 2 (9 - 1)MR 2
2 R2 4
= - = = 4 MR 2
2 18 2 1
Þ I= mR 2
20. (b) Since v is changing (decreasing), L is not conserved in 2
magnitude. Since it is given that a particle is confined This is the formula of the moment of inertia of the disc.
to rotate in a circular path, it cannot have spiral path. 23. (d) This is the case of vertical motion when the body just
Since the particle has two accelerations ac and at completes the circle. Here
therefore the net acceleration is not towards the centre.
v = 5gL ..... (i)
v

at
L ac O
L–h q L v
M 2
P
h
v
Reference A
Level for PE.
ur Applying energy conservation,
The direction of L remains same even when the speed
Total energy at A = Total energy at P
decreases.
21. (b) 2
1 2 1 æ vö
mv = m ç ÷ + mgh.
A' 2 2 è 2ø
Y
M v2
B v2 = + 2gh
4
r M
3v2 3v2 3 15L
R = 2gh Þ h = = × 5gL = ...(ii)
A 4 8g 8g 8
15L
L-
B' Y' L-h 8 -7
In D OPM, cosq = = =
For solid sphere L L 8
2 3p
IAB = MR 2 = I (given) ... (i) Therefore, the value of q lies in the range <q<p
5 4
ROTATIONAL MOTION P-111
24. (a) The system is made up of five bodies (three circles and z
two straight lines) of uniform mass distribution.
Therefore we assume the system to be made up of five w
point masses where the mass of each body is
considered at its geometrical centre.
v
Y Y
O
x
(–a, a) (a, a)
(–a, a) (a, a) Let at any instant of time 't', the insect is at a distance x
m m
from O. At this instant, the moment of inertia of the
m m system is
m X 6m X
(0, 0) (0, 0) 1
(0, –a) I = ML2 + mx2 ...(ii)
3
m m where M = mass of the rod
(0, –a) L = length of the rod
m = mass of the insect
The y-coordinate of the centre of mass is From (i) & (ii)
m1 y1 + m2 y2 + m3 y3 + m4 y4 + m5 y5 d é1 ù d
ycm = t =w ML2 + mx 2 ú = w m ( x 2 )
m1 + m2 + m3 + m4 + m5 ê
dt ë 3 û dt
6m ´ 0 + m ´ 0 + m ´ a + m ´ a + m ( - a ) dx
\ ycm = = 2wmx = 2wmxv
6m + m + m + m + m dt
ma a
2
= 2wmv t [Q x = vt]
= = \ tµt (till t = T)
10 m 10 ur
25. (c) The angular momentum of the mass m about O is mr2w When the insect stops moving, L does not change
and is directed toward +z direction for all locations of and therefore t becomes constant.
m.
LP
1. (d) We know that Fext = Mac.m. ... (i)
We consider the two particles in a system. Mutual force of
P attraction is a internal force. There are no external forces
acting on the system. From (i)
ac.m. = 0
Þ vc.m. = constant.
Since, initially the vc.m. = 0
\ Finally vc.m. = 0
l
2. (b) Angular momentum
ur r ur
L=r´ p
LO L = Momentum × perpendicular distance of line of action of
v momentum w.r.t point of rotation
L = Mv × y.
r m Y
O

M v
The angular momentum of mass m about P is mvl and
is directed for the given location of m as shown in the y
figure. X
r O
The direction of LP remains changing for different
The quantities on the right side of the equation are not
locations of m. changing.
r The magnitude is constant. The direction is also constant.
r dL
26. (b) We know that |t|= 3. (a, c) When the cycle is not pedalled but the cycle is in
dt motion (due to previous effort) the wheels move in the
where L = Iw direction such that the centre of mass of the wheel move
forward. Rolling friction will act in the opposite direction to
d dI the relative motion of the centre of mass of the body with
\ t = (I w) = w ...(i)
dt dt respect to ground. Therefore the rolling friction will act in
From the situation it is clear that the moment of inertia backward direction in both the wheels. The sliding friction
for (rod + insect) system is increasing. will act in the forward direction of rear wheel during pedalling.
P-112 Topic-wise Solved Papers - PHYSICS
4. (b, d) Angular momentum = (momentum) × (perpendicular consider ABCD to be in the X-Y plane then we know that
distance of the line of action of momentum from the axis of 1
4
rotation)
A B
v cos45° v
Ö2
v
h h
45° O 3
O
Angular momentum about O
D
mv C
L= ´h ... (i) 2
2
Izz' = Ixx' + Iyy'
2
v sin q v 2 2 \ Izz' = I1 + I2
Now, h = = [Q q = 45°] ... (ii) Also, Izz' = I3 + I4
2g 4g Adding (i) and (ii),
From (i) and (ii) 2Izz' = I1 + I2 + I3 + I4
But I1 = I2 and I3 = I4 (By symmetry)
m
L= (2 gh ) h = m 2 gh3 \ 2I zz ' = I1 + I1 + I3 + I3= 2I1 + 2I3
2 Þ Izz' = I1 + I3
Also, from (i) and (ii) 7. (a) The force acting on the mass of liquid dm of length dx
at a distance x from the axis of rotation O.
mv v2 mv 3
L= ´ =
2 4g 4 2 g
F
5. (a, c, d) x
Applying conservation of linear momentum dx
dF
2m (– v) + m (2v) + 8m × 0 = (2m + m + 8m) vc
Þ vc = 0
O
+ M
2m dF = (dm) x w2 \ dF = dx ´ xw 2
- L
3a
v M
where is mass of liquid in unit length.
L
a 2a \ The force acting at the other end is for the whole liquid
2v
in tube.
m LM M 2 L
Applying conservation of angular momentum about centre
F= ò0 L
w 2 x dx =
L
w ò x dx
0
of mass L
2mv × a + m (2v) × 2a = Iw ...(i) M 2 é x2 ù M 2 é L2 ù MLw 2
= w ê ú = w ê - 0ú =
L êë 2 úû 0 L êë 2 úû 2
1
Where I = (8m) × (6a)2 + 2m × a2 + m × 4a2 8. (c) When the car is moving in a circular horizontal track of
12
radius 10 m with a constant speed, then the bob is also
I = 30ma2 ...(ii)
undergoing a circular motion. The bob is under the influence
From (i) and (ii) of two forces.
2mv ( a ) + m ( 2v ) ´ 2 a = 30ma 2 ´ w (i) T (tension in the rod)
(ii) mg (weight of the bob)
v
Þw=
5a
v
1 2
Energy after collision, E = Iw
2 Tcosq
q T
q
1 v2 3mv 2
= ´ 30 ma 2 ´ = Tsinq
mg
2 25a 2 5
6. (a, b, c) To find the moment of inertia of ABCD about an axis
passing through the centre O and perpendicular to the plane
of the plate, we use perpendicular axis theorem. If we
ROTATIONAL MOTION P-113
Resolving tension, we get 11. (c, d) As shown in the figure, the component of weight
T cos q = mg ... (i) mg sin q tends to slide the point of contact (of the cylinder
2 with inclined plane) along its direction. The sliding friction
mv acts in the opposite direction to oppose this relative motion.
And T sin q = ... (ii)
r Because of frictional force the cylinder rolls.
(Here T sin q is producing the necessary centripetal N
force for circular motion)
Dividing (ii) by (i), we get f

v 2 10 ´ 10
tan q = = = 1 Þ q = 45°
rg 10 ´ 10 q
s inq mg cosq
9. (a) A'B' ^ AB and C' D' ^ CD
mg q mg
From symmetry IAB = IA'B' and ICD = IC'D'
From theorem of perpendicular axes, Thus frictional force adds rotation but hinders translational
motion.
A' Applying Fnet = ma along the direction of inclined plane,
C' D
we get mg sin q – f = mac,
where ac = acceleration of centre of mass of the cylinder
\ f = mg sin q – mac … (i)
A q g sin q g sin q 2
B But ac = = = g sin q ... (ii)
Ic 2
mR / 2 3
1+ 1+
mR 2 mR 2
mg sin q
C D' From (i) and (ii), f =
3
B' If q is reduced, frictional force is reduced.
Izz = IAB + IA'B' = ICD + IC'D' Þ 2IAB = 2ICD r
r d psystem
\ IAB = ICD
r dL
ur
12. (a)
åFext =
dt
r r
10. (a, b, c) KEY CONCEPT t =
dt Given åFext = 0 Þ psystem = Constant
Given that Due to internal forces acting in the system, the kinetic
uur
r ur ur dL ur ur and potential energy may change with time.
t = A´ L Þ = A´ L Also zero external force may create a torque if the line
dt of action of forces are along different direction. Thus
uur the torque will change the angular momentum of the
dL
From cross-product rule, is always perpendicular to the system.
dt 13. (b, c)
ur ur
plane containing A and L. V C = 2V 0
By the dot product definition
r ur
L . L = L2
Differentiating with respect to time
uur uur uur
ur dL ur dL dL ur dL dL V B =V0
L× + L× = 2L Þ 2L × = 2L
dt dt dt dt dt
uur
dL r ur VA =0
Since, i.e. t is perpendicular to L ur
dt If V 0 is the velocity of centre of the sphere, then
uur ur
r dL dL ur ur ur ur
\ L× =0 Þ =0 V C = 2V 0 , V B = V 0 and V A = 0
dt dt ur ur ur ur ur
Þ L = constant \ V C - V B = 2V 0 - V 0 = V 0
Thus, the magnitude of L always remains constant. ur ur ur r ur
r V B -V A = V 0 - 0 = V 0
As A is a constant vector and it is always perpendicular ur ur ur ur
r \ V C -V B = V B -V A
to t,
r r (b) is the correct option.
Also, L is perpendicular to A ur ur ur ur ur
r r rr Now, | V C - V A | = | 2V 0 - 0 | = | 2V 0 |= 2 | V 0 |
Q L ^ A \ L. A = 0 ur ur ur ur
r r and | V C - V A | = 2 | V B - V C |
Thus, it can be concluded that component of L along A is
(c) is the correct option.
zero i.e., always constant.
P-114 Topic-wise Solved Papers - PHYSICS
14. (a,b,d)
Let V be the volume of spheres.
(i) 3Rw iˆ (due to translational motion)
For equilibrium of A : R
(ii) w making an angle of 30º with the vertical due
T + VdAg = Vdfg 2
\ T = Vg (df – dA) ...(1) to rotation
For T > 0, df > dA or dA < df
uur é ˆ Rw ù 3 Rw ˆ
(a) is the correct option \ vP = ê 3 Rw i - iˆ ú + k
ë 4 û 4
VdF g 11 3
Rw iˆ +
= Rw kˆ
A 4 4
VdA g
17. (d) The acceleration of the center of mass of cylinder rolling
down an inclined plane is
T
g sin q
aP =
T I
VdF g 1+ P 2
MR
B
VdB g
C

For equilibrium of B :
T + Vdfg = VdBg ac
\ T = Vg (dB – df) ...(2)
For T > 0, dB > df
(b) is the correct option
From (1) & (2) Vg (df – dA) = Vg (dB – df)
\ df – d A = d B – d f
\ 2df = dA + dB
(d) is the correct option. Here IP > I Q because in case of P the mass is
15. (c) The frictional force between the ring and the ball is concentrated away from the axis.
impulsive. The angular impulse created by this force
tends to decrease the angular speed of the ring about g sin q
aQ =
O. After the collision the angular speed decreases but IQ
the ring remains rotating in the anticlock wise direction. 1+
MR2
Therefore the friction between the ring and the ground
(at the point of contact) is towards left. \ aP < aQ
\ Q will reach the ground earlier with larger angular
òtdt speed and larger translational kinetic energy.
Jx=2N-s
SUBJECTIVE PROBLEMS :
o
Jy=1Ns mv 2
1. T – mg cos q =
l
c
16. (a, b) For rolling motion, the velocity of the point of contact mv 2
with respect to the surface should be zero. For this \ T= + mg cos q ... (i)
l
r r
3Rw( -iˆ) + v = 0
o \ v = 3Rw iˆ
o O
q0
R R
w w cos30º kˆ q
2 2

R
w sin 30º ( -iˆ ) 3R w iˆ M
T
2 P
M' P'
P
Reference mg sin q q
level for P.E. mg mg cos q

30º OM
In DOPM, cos q0 = Þ OM = l cos q0
A shown in the figure, the point P will have two l
velocities OM '
In DOP'M', cos q = Þ OM' = l cosq
l
ROTATIONAL MOTION P-115
OM' – OM = l (cos q - cos q0 ) 1 g
Loss in potential energy = Gain in kinetic energy Þ n= ... (iii)
2p l cos q
(ActivityP to P')
From (ii), T cos q = mg.
1 2 For M to remain stationary, T = Mg
Þ mg l (cos q - cos q0 ) = mv
2 \ Mg cos q = mg
Þ v2 = 2 g l (cos q - cos q 0 ) ... (ii)
m
From (i) and (ii) Þ cos q = ... (iv)
M
m
T= ´ 2 g l (cos q - cos q0 ) + mg cos q
l 1 gM
From (iii) and (iv), n =
\ T = 3mg cos q – 2 mg cos q0 2p l m
Þ T = mg (3 cos q – 2 cos q0 ) 3. Let s be the mass per unit area.
Then the mass of the whole disc = s × p R2
From equation (i) it is clear that the tension is maximum
Mass of the portion removed = s × p r2
when cos q = 1 i.e., q = 0°
R = 28 cm; r = 21 cm; OP = 7 cm
\ T = mg
Taking O as the origin
mv 2 The position of c.m.
Hence, Tmax = + mg ... (iii)
l R
From eqn. (ii)
v2 = 2 g l (1 - cos q0 ) ... (iv) r
From (iii) and (iv) O P
m
Tmax = [2 g l (1 - cos q0 )] + mg
l
\ Tmax = 3mg – 2mg cos q0 m1 x1 - m2 x2
80 = 3 × 40 – 2 × 40 cos q0 x=
m1 - m2
1
Þ 80 cos q0 = 40 Þ cos q0 = Þ q0 = 30° s ´ p R 2 (0) - s ´ p r 2 ´ 7
2 =
2. Suppose mass m moves around a circular path of radius r. s p R2 - s p r 2
Let the string makes an angle q with the vertical. Resolving
tension T, we get - (21)2 ´ 7 21 ´ 21 ´ 7
= =- = -9 cm
2
(28) - (21) 2 7 ´ 49
l
This means that the c.m. lies at a distance of 9 cm from the
q
T origin towards left.
4. C.M. of the system of two bodies in situation (i) in
T cos q
q x-coordinate
r
T sin q M ´ 0 + mx1 mx1
xC = = ... (i)
M +m M +m
mg

and, T sin q = mrw2 ... (i) +Y


T cos q = mg ... (ii) m x 1
r =(R– r)
2
rw
\ tan q = R–r
g M
r
From diagram, sin q =
l O Situation (1)
+Y
Þ r = l sin q

w2
\ tan q = l sin q R– r
g v'
v r
tan q.g g
w2 = w=
l sin q l cos q +X x
Situation (2)
2
P-116 Topic-wise Solved Papers - PHYSICS
C.M. of the system in situation (ii) in x-coordinate is For x and y
M ´ x2 + m ´ x2 In horizontal direction x = vx ´ t
x'C = = x2 ... (ii)
M +m
v0 v0 v2
Since no external force is in x-direction \ x= ´ = 0
\ xC = x'C 2 g 2g
mx1 m( R - r ) In vertical direction applying
\ x2 = =
M +m M +m 1 2
S = ut + at
Applying conservation of linear momentum, 2
Initial Momentum = Final Momentum
0 = MV – mv v0 v0 1 v02 v2 v2
y= ´ - g = 0 - 0
MV 2 g 2 g2 2g 2 g
\ v= .... (iii) Putting the values in the above equation
m
Applying the concept of conservation of energy, we get év2 æv
Loss in P.E. of mass m = Gain in K.E. of mass M and Gain in ö æ v
2
v2 ö v ù
L = m ê 0 ´ ç 0 - v0 ÷ - ç 0 - 0 ÷ 0 ú
K.E. of mass m êë 2 g è 2 ø è 2 g 2g ø 2 ú
û
1 1
Þ mg (R – r) = MV 2 + mv 2 é v03 v3 v3 v3 ù
2 2 L=m ê - 0 - 0 + 0 ú
M 2V 2 ëê 2 g 2 g 2 g 2 2 g ûú
Þ 2mg (R – r) = MV2 + m [from (iii)]
m2 mv03 é 1 1 ù
L= ê - ú
g ë2 2 2û
M 2V 2
Þ 2mg (R – r) = MV2 +
m - mv03
L=
é Mù 2 ém + M ù 2 2g
2mg (R – r) = MV2 ê1 + ú = MV ê ú
ë mû ë m û ur r ur
Now, L = r ´ p
2m 2 g ( R - r ) Note : The direction of L is perpendicular to the plane of
Þ =V2
M (m + M ) motion and is directed away from the reader.
r
2g (R - r ) p
Þ V=m
M (m + M )
5. The angular momentum is given by L = xpy – ypx 6. KEY CONCPET : Applying law of conservation of energy
= m[xvy – yvx] at point D and point A
P.E. at D = P.E. at Q + (K.E.)T + (K.E.)R where
v (K.E.)T = Translational K.E. and (K.E.)R = Rotational K.E.
(x, y) are the coordinates of the particle after time t = 0 and
g
1 2 1 2
vx, vy are the components of velocities at that time. Þ mg (2.4) = mg (1) + mv + I w ...(i)
2 2
y Since the case is of rolling without slipping
v0 v0
Ö2
D
45
O

x
v0 Q 4.43 m/s
2.4 m
Ö2
1m
For vx and vy
A B C
o v0
vx = v0 cos 45 = \ v = rw
2
(The horizontal velocity does not change with time) v
\ w= where r is the radius of the sphere
Applying v = u + at in the vertical direction to find vy r

( æv ö
è gø
)
v y = v0 sin 45o - g ç 0 ÷ Also, I=
2 2
5
mr
Putting in equation (i)
v0 v0 v0
vy = -g´ = - v0
2 g 2
ROTATIONAL MOTION P-117

1 1æ2 öv
2 é (0.16) ( 3)2 ù
mg ( 2.4 - 1) = mv 2 + ç mr 2 ÷ 2 =ê + 0.08 ´ (0.5)2 + (0.08) (0.5) 2 ú w
2 2è5 ør êë 12 úû
7v 2 = 0.08 w
or, g ´ 1.4 = \ 0.08 w = 0.16 Þ w = 2 rad/s ... (iii)
10 The rotational kinetic energy
v = 4.43 m/s
After point Q, the body takes a parabolic path. 1 2 1
= I w = ´ 0.08 ´ 2 2 = 0.16 J ... (iv)
The vertical motion parameters of parabolic motion will be 2 2
uy = 0 Sy = 1m The final kinetic energy
ay = 9.8 m/s2 ty = ? = Translational K.E. + Rotational K.E.
1 2 = 2.56 + 0.16 = 2.72 J
\ S = ut + at Þ 1 = 4.9 t 2y The change in K.E. = Initial K.E. – Final K.E.
2 = 5.44 – 2.72 = 2.72 J
1 8. Assuming the density to be uniform
ty = = 0.45sec
4.9 M
r=
Applying this time in horizontal motion of parabolic path, pR 2 h
BC = 4.43 × 0.45 = 2m Let M¢ be the mass of unrolled carpet
Note : During its flight as a projectile, the sphere continues
to rotate because of conservation of angular momentum. M'
\ r=
7. Initial Kinetic Energy æ Rö
2
pç ÷ h
1 1 1 è 2ø
= m1v12 + m2 v22 + MV 2
2 2 2
M
1 2 1 2 Þ M' =
= 0.08 ´ 10 + 0.08 ´ 6 + 0 = 5.44 J ... (i) 4
2 2
M
M
10 m/s R

m1 M'
x = 0.5m
R/2
x = 0.5m

m2
Loss in potential Energy = Gain in kinetic energy
6 m/s
R 1æMö 2 1
Mg R – M ' g = ç ÷ vc + I w 2
2 2è 4 ø 2
Applying law of conservation of linear momentum during
M R M 2 1 é 1 æ M ö æ R ö ù vc2
2
collision
m1 × v1 + m2 × v2 = (M + m1 + m2) Vc \ MgR – g = vc + ê ç ÷ ç ÷ ú
4 2 8 2 êë 2 è 4 ø è 2 ø úû [ R / 2]2
where Vc is the velocity of centre of mass of the bar and
particles sticked on it after collision æ Rw ö
0.08 × 10 + 0.08 × 6 = (0.16 + 0.08 + 0.08) Vc çèQ vc = ÷
2 ø
Þ Vc = 4 m/s
\ Translational kinetic energy after collision 14 Rg
On solving, we get vc =
1 2 3
= (M + m1 + m2 )Vc = 2.56 J ... (ii)
2 9. (a) Let us consider the system of homogeneous rod and
Applying conservation of angular momentum of the bar insect and apply conservation of angular momentum during
and two particle system about the centre of the bar. collision about the point O.
Since external torque is zero, the initial angular momentum
M
is equal to final angular momentum.
Initial angular momentum v
= m1v1 × x – m2v2x
= 0.08 × 10 × 0.5 – 0.08 × 6 × 0.5 A O B
= 0.4 – 0.24 = 0.16 kg m2s–1 (In clockwise direction) L/4
Final angular momentum = Iw L/2
é M l2 ù Angular momentum of the system before collision = angular
=ê + m1 x 2 + m2 x 2 ú w momentum of the system after collision.
ëê 12 ûú
P-118 Topic-wise Solved Papers - PHYSICS
On integration, taking limits
L
Mv × = Iw L/2 g p / 2w
4
Where I is the moment of inertia of the system just after
òL / 4 dx = 2w ò 0 cos wt dt

collision and w is the angular velocity just after collision. L


when x= , wt = 0
4
L é æ Lö ù
2
1
Þ Mv = ê M ç ÷ + ML2 ú w L/2 g p / 2w
4 êë è 4 ø 12 úû [ x] L / 4 = 2
[sin wt ] 0
2w
2 2 p
L ML é 1 1 ù ML é 3 + 4 ù L
Þ Mv ´ = + w= when x = , wt =
4 4 êë 4 3 úû 4 êë 12 úû 2 2
æL Lö g é p ù
ML2 7 12 v Þ çè - ÷ø = êësin 2 - sin 0úû
= ´ ´w Þ w= . 2 4 2w 2
4 12 7 L
(b) Note : Initially the torque due to mass OB of the rod L g 2g
Þ = Þ w=
(acting in clockwise direction) was balanced by the torque 4 2w 2 L
due to mass OA of the rod (acting in anticlockwise direction).
But after collision there is an extra mass M of the insect 12 v 12 v 2g 7
But w = Þ = Þ v= 2 gL
which creates a torque in the clockwise direction, which 7 L 7 L L 12
tends to create angular acceleration in the rod. But the same 7
is compensated by the movement of insect towards B due Þ v= 2 ´ 10 ´ 1.8 = 3.5 ms–1
12
to which moment of inertia I of the system increases. 10. (i) Initially, the rod stands vertical. A straight disturbance
Let at any instant of time t the insect be at a distance x from makes the rod to rotate. While rotating, the force acting on
the centre of the rod and the rod has turned through an the rod are its weight and normal reaction. These forces are
angle q (= wt) w.r.t its original position. vertical forces and cannot create a horizontal motion.
Therefore the centre of mass of the rod does not move
horizontally. The center of mass moves vertically
A downwards. Thus the path of the center of mass is a straight
line.
M
(ii) Trajectory of an arbitrary point of the rod
A O q=w t B Consider an arbitrary point P on the rod located at (x, y) and
N
at a distance r from the end B. Let q be the angle of
x
inclination of the rod with the horizontal at this position.
B A
Y
Mg

dL d C
Instantaneous torque, t = = ( I w) P (x,y)
dt dt y r
q
dI O N B X
=w x
dt
y
d é1 ù In D BNP, sin q = ... (i)
=w ML2 + Mx 2 ú r
dt êë12 û
x + BN x + r cos q
dx cos q = =
= 2 Mwx ... (i) L/2 L/2
dt x
This torque is balanced by the torque due to weight of Þ cos q = ... (ii)
L
insect. -r
2
t = Force × Perpendicular distance of force with axis of
rotation From (i) and (ii) sin 2 q + cos 2 q = 1
= Mg × (OM)
= Mg (x cosq) ... (ii) y2 x2
Þ + =1
From (i) and (ii) r2 æL ö
2
çè - r ÷ø
dx æ g ö 2
2M w x = Mg (x cosq) Þ dx = ç ÷ cos wt dt This is equation of an ellipse.
dt è 2w ø
ROTATIONAL MOTION P-119
11. (i) The drum is given an initial velocity such that the block
1 1
X starts moving up the plane. Þ MR 2 w 2 + mR 2 w 2 = mgS sin q
2 2
Y
1 R 2 w 2 ( M + m)
T R
w Þ =S
M 2 mg sin q
m
T 1 0.2 ´ 0.2 ´ 10 ´ 10 (2 + 0.5)
X
Þ S= ´ = 1.22 m
q 2 0.5 ´ 9.8 ´ sin 30°
ALTERNATE SOLUTION :
mg sinq mg mg cosq
q=30o 2T
a = Ra = R × [From (iii)]
MR
As the time passes, the velocity of the block decreases. The
0.2 ´ 2 ´ 1.63
linear retardation a, of the block X is given by = = 1.63rads -1
2 ´ 0.2
mg sin q - T = ma ... (i)
The linear retardation of the block and the angular
v=0
acceleration of the drum (a) are related as
a = Ra ... (ii) s=s
a=a
where R is the radius of the drum.
The retarding torque of the drum is due to tension T in the Rwo=u
string.
t=T×R
But t = Ia. Where I = M.I. of drum about its axis of rotation. Using v2 – u2 = 2as

1 2
02 – w 02 R 2 = 2 ( -a) S
\ T × R = MR a ... (iii)
2 10 ´ 10 ´ 0.2 ´ 0.2
Þ S= = 1.22 m
é 1 2ù
2 ´ 1.63
êëQ I = 2 MR úû 12. l = 0.6 m
mA = 0.01 kg
1 a 2T mB = 0.02 kg
From (ii), TR = MR 2 Þ a = m = 0.05 kg
2 R M
Substituting this value in (i) During collision, the torque of the system about P will be
zero because the only force acting on the system is through
2T æ 2m ö P (namely weight of rods/mass m/reaction at P)
mg sin q – T = m × Þ mg sin q = çè1 + ÷T
M Mø
P
(mg sin q) ´ M 0.5 ´ 9.8 ´ sin 30° ´ 2
\ T= =
M + 2m 2 + 2 ´ 0.5 mA A
= 1.63 N
(ii) The total kinetic energy of the drum and the block at 2l
the instant when the drum is having angular velocity
10 rads–1 gets converted into the potential energy of the mB B
block. v
m

S dL
h=S sinq Since, t = and t = 0
q dt
Þ L is constant.
q mg
Angular momentum before collision = mv × 2 l ... (i)
[(K.E.)Rotational]drum + {(K.E.)Translational]block = mgh Angular momentum after collision = Iw ... (ii)
Where I is the moment of inertia of the system after collision
1 2 1 2
I w + mv = mgS sin q about P and w is the angular velocity of the system.
2 2 M.I. about P : I1 = M.I. of mass m
1 2 1 I2 = M.I. of rod mA
I w + m( Rw )2 = mgS sin q [Q v = Rw ] I3 = M.I. of rod mB
2 2
I = I1 + I2 + I3
P-120 Topic-wise Solved Papers - PHYSICS
Applying energy conservation from O to O'.
é ì æ l 2 ö æ l ö 2 üï ìï æ l2 ö æ l ö 2 üï ù Loss of potential energy of cylinder
2 ï
= ê m (2 l ) + í mA ç ÷ + ç ÷ ý + í mB ç ÷ + ç + l÷ ý ú = Gain in translational K.E. + Gain in rotational K.E.
êë ïî è 12 ø è 2 ø ïþ ïî è 12 ø è 2 ø ïþ úû
1 2 1 2
mgh = mvc + I w ... (ii)
é æ l 2 l2 ö æ l 2 9l 2 ö ù 2 2
2
= ê 4 ml + m A ç + ÷ + mB ç + ÷ú Where I is the moment of inertia of the cylinder about O', its
êë è 12 4 ø è 12 4 ø úû axis of rotation, w is the angular speed, Vc is the velocity of
center of mass.
é 1 7 ù Also for rolling, vc = wR
= ê 4ml2 + m A l2 + mB l 2 ú = 0.09 kg m2
ë 3 3 û vc
Þ w= ... (iii)
From (i) and (ii) R
Iw = mv × 2 l 1
I= MR 2 ... (iv)
mv ´ 2l 0.05 ´ v ´ 2 ´ 0.6 2
Þ w= = = 0.67 v From (ii), (iii) and (iv), we get
I 0.09
Applying conservation of mechanical energy after collision. 1 2 1 1 v2
(Using the concept of mass) mgh = mvc + ´ mR 2 ´ c2
2 2 2 R
Loss of K.E. = Gain in P.E.
1 2 1 2 3 2 4 gh
1 2 æ lö æ 3l ö Þ gh = vc + vc = vc Þ vc2 =
I w = mg (2l) + m A ç ÷ g + mB g ç ÷ 2 4 4 3
2 è 2ø è 2ø
R-h
1 In DO¢MP, cos q =
Þ ´ 0.09 ´ (0.67 v) 2 R
2 Þ h = R (1 – cos q )
é 1 3ù 4g
= ê 0.05 ´ 2 + 0.01 ´ + 0.02 ´ ú ´ 9.8 ´ 0.6 \ vc2 = R (1 - cos q) ... (v)
ë 2 2û 3
Þ v = 6.3 m/s From (i) and (v), we get
13. (a) Let the original position of centre of mass of the 4 gr
cos q = (1 - cos q)
cylinder be O. While rolling down off the edge, let the cylinder 3Rg
be at such a position that its centre of mass is at a position
O'. Let ÐNPO be q. As the cylinder is rolling, the c.m. rotates 4
Þ 3 cos q = 4 – 4 cos q Þ cos q =
in a circular path. The centripetal force required for the 7
circular motion is given by the equation. (b) From (v) speed of C.M. of cylinder before leaving
contact with edge.
mvc2 4 gR æ 4 ö 4 gR
mg cos q – N =
R
vc2 = ç1 - ÷ =
3 è 7ø 7
Where N is the normal reaction and m is mass of cylinder.
4 gR
Þ vc =
N 7
O (c) Before the cylinder's c.m. reaches the horizontal line of
the edge, it leaves contact with the edge as
R h w
4
M
os
q
O'
vc q = cos -1 = 55.15°
R–h m q
gc 7
q R Therefore the rotational K.E., which the cylinder gains at
mg mgsinq
P the time of leaving contact with the edge remains the same
in its further motion. Thereafter the cylinder gains
translational K.E.
Again applying energy conservation from O to the point
where c.m. is in horizontal line with edge
The condition for the cylinder leaving the edge is N = 0 1 2 1
mgR = I w + m(v 'c ) 2
mvc2 2 2
mg cos q = 2
R 1 1 2 æ 4g ö 1 2
mgR = ´ mR ´ ç ÷ + 2 m(v 'c )
vc2 2 2 è 7 R ø
Þ cos q = ... (i)
Rg
ROTATIONAL MOTION P-121
Let after time t0, the velocity of centre of mass be v. At this
vc 4 gR / 7
Q w= = instant if the disc attains rolling motion then v = Rw
R R
v
mgR 6 mgR Þ w=
Þ mgR – = Translational K.E. = R
7 7
1 2 mgR v 2µg
Also, Rotational K.E. = I w = Also, w0 = 0, w = , t = t0 , a =
2 7 R R
Translational K .E. w = w0 + a t
\ =6
Rotational K .E. v 2µg
Þ = 0+ ´ t0 Þ v = 2µgt0 ... (i)
14. KEY CONCEPT : The concept of center of mass can be R R
applied in this problem. For translational motion
When small sphere M changes its position to other extreme
position, there is no external force in the horizontal direction. -f
u = v0, v = v, a = = – µg, t = t0
Therefore the x-coordinate of c.m. will not change. m
[xc.m.]initial = [xc.m.]final \ v = u + at
Þ v = v0 – µg t0 ... (ii)
Y
From (i) and (ii)
v 3v
v = v0 – µg × Þ = v0
2µg 2
4m 6R
2v0
m
Þ v=
O 3
L'-5 R (L,O) (L',O) R X
L From (i)
L'+5R
v v
t0 = = 0
2µg 3µg
Thin line of sphere represents initial state, dotted line of Work done by frictional force
sphere represents final state.
From (i) = Kinetic energy at time t0 – Kinetic energy at t = 0
(xc.m.)initial = (xc.m.)final 1 2 1 2 1 2
= mv + I w - mv0
M1 x1 + M 2 x2 M1 x '1 + M 2 x '2 2 2 2
Þ =
M1 + M 2 M1 + M 2
1 2µgt0
But v = v0 – µgt0, I = mR 2 , w = at0 =
4m ´ L + m ´ (5 R + L ) 4m ´ L '+ m ´ ( L '- 5 R ) 2 R
Þ =
4m + m 4m + m
2
Þ 5L + 5R = 5L'– 5R 1 1æ1 ö æ 2µgt0 ö 1
Þ 5L + 10R = 5L' Þ L + 2R = L' \ W= m[v0 - µgt0 ]2 + ç mR 2 ÷ ç ÷ - mv02
2 2è2 øè R ø 2
Since, the individual center of mass of the two spheres
has a y co-ordinate zero in its initial state and its final state mµgt0
therefore the y-coordinate of c.m. of the two sphere system = [3µgt0 - 2v0 ]
will remain zero. 2
Therefore the coordinate of c.m. of bigger sphere is (L From (iii)
+ 2R, 0).
15. KEY CONCEPT: Initially the motion is purely sliding motion. mµg æ v0 ö é v0 æ v0 ö ù mv02
W= ê ´ - 2 v0ú = -
The sliding friction acts on the body in the opposite side of 2 çè 3µg ÷ø ë 3µg çè 3µg ÷ø û 6
motion to stop the relative motion of the point of contact.
f = µN = µmg Note : When the body has attained rolling motion, the
Due to this a torque will act on the disc and the disc starts to sliding friction will cease to exist. There will only be rolling
friction which is very small.
fR µmgR 2µg The work done later on (after t0) will be
rotate t = Ia = f × R Þ a = = =
I 1 2 R W = – fr × S where fr = rolling friction.
mR
2 16. (i) The observer, let us suppose, is on the accelerated
w frame. Therefore a pseudo force ma is applied individually
v
on each disc on the centre of mass. The frictional force is
v0
acting in the + X direction which is producing an angular
f R
acceleration a.
t=0 t = t0 The torque acting on the disc is
P-122 Topic-wise Solved Papers - PHYSICS
17. (a)
Truck Z
1 ma 2 ma Y
a'
a a X

P O
R R
M N F
°
30 30°
f f
60°

+Z r Resolving the force F acting on the wedge


observe
Fx = F cos 30°; Fy = F sin 30°
+Y Note : The collision is elastic and since the sphere is fixed,
a = 9 m/s2 +X the wedge will return back with the same velocity
(in magnitude).
t = Ia = f × R
The force responsible to change the velocity of the wedge
Ia in X-direction is Fx.
Þ f= ... (i)
R Fx × Dt = mv – ( – mv)
Let a' is the acceleration of c.m. of the disc as seen by the
observer. Since the case is of pure rolling and from the é v initial ù
perspective of the observer ê ¬¾ ¾ú
a' = aR ... (ii) (Impulse) = (Change in momentum) ê ú
ê ú
Þ From (i) and (ii) ê ¾¾ ®ú
Ia ' ê v final ú
ë û
f= ... (iii)
R2
Applying Newton's law for motion in X-direction 2 mv 2 mv
\ Fx = Þ F cos 30° =
ma – f = ma' Dt Dt
æ fö 4 mv
Þ a' = ç a - ÷ ... (iv) Þ F=
è mø 3 Dt
Also moment of inertia
In vector terms
1 ur
F = Fx $i + Fy ( - k$ ) = F cos30°i$ + F sin 30°( -k$ )
2
I = mR ... (v)
2
From (iii), (iv) and (v)
3$ 1
=F× i + F ´ ( - k$ )

fö 2 2
mR ç a - ÷
1 è mø
f= Þ 2f = ma – f ur F 2mv
2 R2 Þ F = ( 3 $i - k$ ) = ( 3 $i - k$ )
2 3Dt
ma 2 ´ 9 Taking equilibrium of force in Z-direction (acting on wedge)
Þ 3f = ma Þ f = = = 6N (In + X direction)
3 3 we get
ur
f = (6 $i) N
Fy + mg = N
(ii) The position vector of point M, taking O as the origin F 2mv
uur Þ N= + mg = + mg
rm = - 0.1$j - 0.1k$ and position vector of point N 3Dt
2
uur
rN = 0.1$j - 0.1 k$ æ 2mv
N= ç
ö
+ mg ÷ k$
The torque due to friction on disc 1 about O è 3D t ø
uur uuur ur
t = r ´ f = ( -0.1$j - 0.1 k$ ) ´ (6 i$)
1 M
_

= 0.6 (k$ - $j ) N - m
The torque due to friction on disc 2 about O
uur uur ur
t = r ´ f = (+ 0.1 $j - 0.1 k$ ) ´ (6 $i )
2 N
+
(b) Taking torques on wedge about the c.m. of the wedge.
= 0.6 (- $j - k$ ) N - m F × h – Torque due to N + mg × 0 = 0
The magnitude of torque on each disc 4mv
Þ Torque due to N = F × h = ´h
| t1 | = | t 2 | = 0.6 2 N - m 3D t
ROTATIONAL MOTION P-123
18. KEY CONCEPT : During the fall, the disc-mass system gains Therefore a frictional force f will act on the top of the cylinder
rotational kinetic energy. This is at the expense of potential towards right.
energy.
A m f

a1 a
R
a
M1g F f mg N
C
R/4
P Q P f'
R/4
C' Direction of f ' : A force f is acting on the cylinder. This force
is trying to move the point of contact P towards right by an
R
acceleration
v
Reference f
level for P.E acm = acting towards right.
M1
Applying energy conservation
Total energy initially = total energy finally At the same time, the force f is trying to rotate the cylinder
about its centre of mass.
æ 2R ö æ 2R ö 1 2 f× R= I× a
mg ç 2 R + ÷ + mg èç R + ÷ = mgR + I w
è 4 ø 4 ø 2
f ´R f ´R 2f
Where I = M.I. of disc-mass system about PQ Þ a= = = in clockwise direction.
I 1 2 M 1 R
M R
mg ×
10 R
+ mg
6R 1
= mgR + I w 2 2 1
4 4 2
f 2f f
1 2 \ acm + aR = - ´R=- , i.e., towards left.
Þ 3mgR = I w M1 M1 R M1
2
Therefore, the point of contact of the cylinder with the
6mgR ground move towards left. Hence friction force acts towards
Þ w= ... (i) right on the cylinder.
I
(I)PQ = (Idisc)PQ + (Imass)PQ Note : You can assume any direction of friction at the point
of contact and solve the problem. If the value of friction
é mR 2 æ Rö ù
2
æ 5R ö
2 comes out to be positive, our assumed direction is correct
=ê + M ç ÷ ú + mç ÷ otherwise the direction of friction is opposite. The above
êë 4 è 4ø ú è 4ø
û activity is done so that if only the direction of friction is
asked, an approach may be developed.
1
[Q M.I. of disc about diameter = MR 2 ] Applying Newton's law on plank, we get
4 F – f = m2a2 ... (i)
mR 2 [4 + 1 + 25] 15mR 2
Also, a2 = 2a1 ... (ii)
= = ... (ii) Because a2 is the acceleration of topmost point of cylinder
16 8 and there is no slipping.
From (i) and (ii) Applying Newton's law on cylinder
6mgR ´ 8 16 g M1a1 = f + f ' ... (iii)
w= 2
= The torque equation for the cylinder is
15mR 5R
1 æa ö
Let v be the velocity of mass m at the lowest point of rotation f × R – f ' × R = Ia = M1 R 2 ´ ç 1 ÷
2 è Rø
æ Rö
v = wç R + ÷
è 4ø 1
[Q I = M1R 2 and Ra = a1]
2
16 g 5R
\ v= ´ = 5 gR 1 1
5R 4 \ M1Ra1 Þ f + f ' = M1a1
(f – f ') R = ... (4)
2 2
19. The man applies a force F in the horizontal direction on the
plank as shown. Therefore the point of contact of the plank Solving equation (iii) and (iv), we get
with the cylinder will try to move towards right. Therefore 3
f = M 1a1 ... (5)
the friction force F will act towards left on the plank. To 4
each and every action there is equal and opposite reaction. 1
and f ' = M1a1 ... (6)
4
P-124 Topic-wise Solved Papers - PHYSICS
From (i) and (iii) 21. (a) The mass B is moving in a circular path centred at A.
3 The centripetal force (m l w2) required for this circular motion
F – f = 2m2a1 Þ F – M 1a1 = 2m2a1 is provided by F '. Therefore a force F ' acts on A (the hinge)
4
which is equal to m l w2. The same is the case for mass C.
4F 8F
\ a1 = \ a2 = Therefore the net force on the hinge is
3M1 + 8m2 3M1 + 8m2
From (v) and (vi) Fnet = F '2 + F '2 + 2 F ' F 'cos 60°
3 4F 3FM1 1
f= M1 ´ = Fnet = 2 F '2 + 2 F '2 ´ = 3F ' = 3 mlw 2
4 3M1 + 8m2 3M 1 + 8m2 2
1 FM 1
And f ' = M1 ´ a1 = Y
4 3M1 + 8m2
A
20. Ic = 1.2 kg -m2 o X
60
A F'
F'

m Q Fnet
0.5 y c.m l l
B
P
F' F'
m = 30kg

B l C

Let y be the distance of c.m. from line AB. (b) The force F acting on B will provide a torque to the
Applying parallel axis theorem of M.I. we get system. This torque is
M.I. of laminar sheet about AB
IAB = Ic.m. + my2 l 3
F× = Ia
IAB = 12 + 30y2 ... (i) 2
The angular velocity of the laminar sheet will change after
every impact because of impulse. 3l 3 æ F ö
F× = (2ml 2 ) a Þ a = ´
Impulse = Change in linear momentum 2 4 çè m l ÷ø
6 = 30 (Vf – Vi)
The total force acting on the system along x-direction is
6 = 30 × y (wf – wi) ... (ii)
F + (Fnet)x
Also, change in angular momentum = Moment of Impulse
This force is responsible for giving an acceleration ax to the
\ IABwf – IABwi = Impulse × distance
system.
IAB (wf – wi) = 6 ×0.5 = 3
3 3
\ wf = + wi = + ( -1) ... (iii)
I AB 1.2 + 30 y 2
Note : Minus sign with wi because the direction of laminar
plate towards the obstacle is taken as – ve (assumption).
From (ii) and (iii)
l Ö3
é 3 ù c.m 2
6 = 30 × y ê - 1 + 1ú
2
ëê1.2 + 30 y ûú
é 3 ù
1 = 5y ê 2ú F
êë1.2 + 30 y úû
Therefore,
\ 1.2 + 30 y2 = 5y [+ 3] = 15y
F + (Fnet)x = 3m (ax) c.m.
\ 30y2 – 15y – 1.2 = 0
On solving, we get y = 0.1 or 0.4 F æ 3 F l Fö
\ wf = 1 rad/s if we put y = 0.1 in eq. (ii) = 3m çèQ a x = ar = 4 ml ´ = ÷
4m 3 4ø
And wf = 0.5 rad/s if we put y = 0.4 in eq. (ii)
(Not valid as per sign convention) 3F F
= \ (Fnet)x = -
Now, since the lamina sheet comes back with same angular 4 4
speed as that of incident angular speed, the sheet will swing 2
in between P and Q infinitely. (Fnet)y remains the same as before = 3 mlw .
ROTATIONAL MOTION P-125
22. (a) Note : The string snaps and the spring force comes mv ´ L
into play. The spring force being an internal force for the w=
two mass-spring system will not be able to change the é 1 2 2ù
velocity of centre of mass. êë 3 ML + mL úû
This means the location of center of mass at time t will be
V0t. mv 3mv
Þ w= =
é ML ù ( M + 3m) L
m1 x1 + m2 x2 êë 3 + mL úû
Now, xcm = = v0 t
m1 + m2
24. Applying Fnet = ma in X-direction
Þ m1 [v0t – A (1 – cos wt)] + m2x2 = v0tm1 + v0 tm2 mg sin q – f = ma ... (i)
Þ m2x2 = v0tm1 + v0tm2 – v0tm1 + m1A (1 – cos wt) The torque about O will be
Þ m2x2 = v0tm2 + m1A (1 – cos wt) t=f×R
m1 = Ia ... (ii)
Þ x2 = v0 t + A (1 – cos wt) As the case is of rolling therefore
m2 a = aR
(b) Given that x1 = v0t – A (1 – cos wt) a
dx1 Þa = ... (iii)
\ = v0 - Aw sin wt R
dt
Y
d 2 x1 2
\ = - Aw cos wt ... (i)
dt 2
N X
This is the acceleration of mass m1. When the spring comes O

q R f
d 2 x1
to its natural length instantaneously then =0
dt 2 mg sinq mg cosq
mg
and x2 - x1 = l 0 .
é m ù
\ êv0t + 1 A (1 - cos wt ) ú - éë v0t - A (1 - cos wt ) ùû = l 0
ë m2 û q

æm ö Ia
\ ç 1 + 1÷ A(1 - cos wt ) = l 0 From (ii) and (iii), f =
è m2 ø R2
Substituting this value in (i), we get
d 2 x1
Also when = 0, cos wt = 0 ( from (1)) mg sin q –
Ia
= ma
dt 2 R2
mg sin q mg sin q 2
æm ö Þ a= = = g sin q
\ l 0 = ç 1 + 1÷ A I 1 mR 2 3
è m2 ø m+ 2 m+
R 2 R2
ur
r dL é 1 ù
23. We know that t = 2
dt êëQ I = 2 mR for solid cylinder úû
r ur
Þ t ´ dt = d L 25. The various forces acting on the ladders are shown in the
r r figure.
When angular impulse ( t ´ dt ) is zero, the angular Since the system is in equilibrium, therefore
momentum is constant. In this case for the wooden SFy = 0
log-bullet system, the angular impulse about O is constant. Þ Mg + mg + Mg = N + N
Therefore,
[angular momentum of the system]initial Y
= [angular momentum of the system]final P
Þ mv × L = I0 × w ... (i) X
where I0 is the moment of inertia of the wooden log-bullet
system after collision about O mg
I0 = Iwooden log + Ibullet
1 N N
= ML2 + ML2 ... (ii) Mg Mg
3
From (i) and (ii) A f f B
P-126 Topic-wise Solved Papers - PHYSICS
(2M + m) g é dp ù
Þ N= ... (i) = x × dF = x ê n a dx ú
2 ë dt û
Considering the rotational equilibrium of one ladder as
shown in figure. Calculating torques about P x dx
Q P
O
q

mg
m2

l/2
N Mg \ Total torque,
q b a é 2 b2 ù
A f M t2 = òb/2 n(2mv) ax dx = 2mnv
2
êb - ú
êë 4 úû
Mg × PQ + f × PM = N × OP
L a 3b2
Þ Mg × cos q + f ´ L sin q = NL cos q = 2nmv ´
2 2 4
As t1 = t2
MgL
NL cos q - cos q Mg b a 3b2 a 3b
Þ f=
2 = N cot q - cot q Mg = 2nmv Þ Mg = 2nm ´ ´ v
L sin q 2 2 2 4 2 2
éæ 2N + m ö Mg ù 4Mg 4 ´ 3 ´ 10
Þ f = êç ÷ø g - cot q Þ v= =
ë è 2 2 úû 2nma ´ 3b 2 ´ 100 ´ 0.01 ´ 1 ´ 3 ´ 2
= 10 m/s
é gù
Þ f = ê( M + m) ú cot q MATCH THE FOLLOWING :
ë 2û
26. KEY CONCEPT Since the plate is held horizontal therefore 1. A ® (p,t) ; B ® (q,s,t) ; C ® (p,r,t) D ® (q, p)
net torque acting on the plate is zero.
m N
F f
3b/4 v

q
a mgsinq
mgcosq
mg
q
b/2
Mg
m' As the velocity is constant
b 3b f = mg sin q ..... (i)
Þ Mg × =F´ ... (i)
2 4 But f = mN = mmg cos q ..... (ii)
dp b From (i) and (ii)
F= n (Area) = n × (2mv) × a × ... (ii) mmg cos q = mg sin q Þ m = tan q
dt 2
From (i) and (ii) The force by X on Y is the resultant of f and N and is
b b 3b equal to
Mg × = n × (2mv) × a × ×
2 2 4 f 2 + N 2 = m 2 N 2 + N 2 = m 2 + 1N
3´ 2
Þ 3 × 10 = 100 × 2 × 0.01 × v × 1 × = ( tan 2 q + 1 ) mg cos q = sec qmg cos q = mg
4
Þ v = 10 m/s = weight of Y.
Therefore statement (a) is correct.
ALTERNATE SOLUTION Now, due to the presence of frictional force between Y
b and X, the mechanical energy of the system ( X + Y )
Torque due to weight of plate, t1 = Mg ×
2 decreases continously as Y slides down.
torque due to small element shown dotted in the figure Therefore statement (c) is correct.
ROTATIONAL MOTION P-127
(q) The force on Y by X is equal to the wt. of liquid
displaced which cannot be equal to Mg as the density
P of Y is greater than density of X (As Y is sinking)
Therefore, option (a) is in correct.
Z The gravitational potential energy of X increases
V continously because as Y moves down, the centre of
mass of X moves up.
Y Therfore option (b) is correct.
Mg
(t)
X

As the lift moves up, X also moves up and therefore


the gravitational energy of X is continously increasing. Y
option (b) is correct.
The torque of the weight of Y about P is zero as the
perpendicular distance of the line of action of force
from the point P is zero. X
Option (d) is correct. P
The force exerted by X on Y will be equal to
Sphere Y is moving with terminal velocity. Therefore,
Mg + Mg = 2mg where Mg is wt. of Y and Mg is the
the net force on Y is zero i.e.
force on Y due to Z.
Option (a) is incorrect. B Fv
(r)

P
Y
Mg
Xm g v
0
Mg = B + Fv
where B = buoyant force and Fv = viscous force.
B + Fv are exerted by X on Y.
Therefore, option (a) is correct.
In this case the force exerted by X on Y is same as the
The gravitational potential energy of X is continously
force exerted by Y on X. The force on X due to Y is
increasing because as Y moves down, the centre of
R = ( Mg ) 2 + (m0 + M ) g ]2 ¹ Mg mass of X moves up.
Option [b] is correct.
Mg The mechanical energy of the system (X + Y) is
continously decreasing to overcome the viscous
m0g forces.
Option (c) is correct.
R
Mg COMPREHENSION BASED Q UESTIONS :
Therefore, option (a) is incorrect.
The mechanical energy of the system (X + Y) is 1. (c) For disc A
continously decreasing as the system is coming down 1 2 1
and its potential energy is decreasing, the kinetic kx1 = I (2w )2
energy remaining the same. 2 2 2w
Therefore, option (c) is correct and (b) is incorrect. Þ kx12= 2I w 2
... (i)
The torque of the weight of Y about P is not zero. A
For disc B
(s)
1 2 1
kx2 = ´ 2 I w 2 w
2 2
Þ kx22 = I w 2 ... (ii) B
Y
On dividing (i) and (ii), we get
k x12 2I w 2 x1
= Þ = 2
X kx22 Iw 2 x2
P-128 Topic-wise Solved Papers - PHYSICS
2. (a) When disc B is brought in contact with disc A 5. (d) As derived in ans 4.
Let w' be the final angular velocity of both the disc
rotating together. Apply conservation of angular 4k
|Fnet| = x
momentum for the two disc system. 3
I (2w) + 2I (w) = (I + 2I) w' For S.H.M. |Fnet| = Mw2x
4 4k 4k
Þ w' = w \Mw2 = Þ w= ...(iii)
3 3 3M
Torque on disc A
6. (c) From (i) & (ii)
4
I ´ w - I ´ 2w
DLA L f - Li 3 -2 I w 2k
tA = = = = Þ –2kx + f = –2 f Þ f= ×x
t t t 3t 3
Note : The negative sign represents that the torque creates We see that the frictional force depends on x. As x
angular retardation. increases, f increases. Also, the frictional force is
3. (b) Loss in kinetic energy = (K.E.)initial – (K.E.)final maximum at x = A where A is the amplitude of S.H.M.
é1 Therefore the maximum frictional force
æ4 ö ù
2
é1 2 1 2ù
= ê I (2w ) + (2 I ) w ú - ê ( I + 2 I ) ç w ÷ ú 2k
ë2 2 û ëê 2 è3 ø ú fmax= ×A
û
3
é3 4 4 ù The force should be utmost equal to the limiting friction
= [2Iw2 + Iw2] – ê I ´ ´ w 2 ú
ë 2 3 3 û (mMg) for rolling without slipping.

8 I w2 2k
\ mMg = ×A ....(iv)
= 3I w 2 - I w 2 = 3
3 3
4. (d) When the disc is at a distance x from the mean position For S.H.M.
(equilibrium position), the forces acting on the disc are Velocity amplitude = Aw
shown in the figure \ Vo = Aw
y 3μMg
\ Vo = w from (iv)
2k
kx x
x 3μMg 4k
\ Vo = ´ from (iii)
2k 3M

C R 3M
Mean \ Vo = μg
Position kx f k
7. (a) Axis of rotation is parallel to z-axis.
\ –2kx + f = –Mac ...(1) 8. (d) Since the body is rigid, w is same for any point of the
where ac = acceleration of center of mass. Also the torque body.
acting on the disc about its center of mass C is
t = f × R = I × ac ASSERTION & REASON TYPE QUESTIONS :
1
Ia
MR 2 a 1. (d) Statement 1 : For velocity of centre of mass to remain
2 ´ c constant the net force acting on a body must be zero.
\f= =
R R R Therefore the statement 1 is false.
1 Statement 2 : The linear momentum of an isolated
[Q I= MR2 and ac = Rac for rolling without slipping] system remains constant. This statement is true.
2
2. (d) The acceleration of a body rolling down an inclined
1 plane is given by
\ f = Ma c ...(ii)
2
From (i) & (ii) g sin q
a=
I
1 1+
–2kx + Mac= – Mac MR 2
2

Þ
3
Mac= 2 kx Þ Mac =
4kx
For hollow cylinder I MR 2
2 = =1
3 MR 2 MR 2
Þ Net external force acting on the disc when its centre of
mass is at displacement x with respect to the equilibrium 1
MR 2
4kx I 2 1
position = directed towards the equilibrium. For solid cylinder = =
2 2 2
3 MR MR
ROTATIONAL MOTION P-129
Þ Acceleration of solid cylinder is more than hollow
é2 2 2ù é2 2ù
cylinder and therefore solid cylinder will reach the = 2 ê MR + Ma ú + 2 ê MR ú
bottom of the inclined plane first. ë5 û ë5 û
\ Statement -1 is false
• Statement - 2 X D
In the case of rolling there will be no heat losses.
C
Therefore total mechanical energy remains conserved.
The potential energy therefore gets converted into
kinetic energy. In both the cases since the initial
potential energy is same, the final kinetic energy will 4 cm
also be same. Therefore statement -2 is correct.
a = 4 22
INTEGER VALUE C ORRECT TYPE :
1. Let the center of mass of the binary star system be at the
origin. Then A B
Y
A B
x dx
2.2 M s c.m 11 M s 2 é8 2 2ù
origin = 4 ´ MR 2 + 2Ma 2 = M ê R + 2(a) ú
5 ë5 û
2.2M s (- x) + 11M s (d - x )
0= é æ 2 ù
2.2 M s + 11M s 8 5ö ú ´10-4
= 0.5 ê ´ ç ÷ + 2 ´ 8
ê 5 çè 2 ÷ø ú
5d ë û
Þ 0 = 2.2 Ms (– x) + 11Ms (d – x) Þ x =
6
For a binary star system, angular speed w about the centre = 0.5 [ 2 + 16] ´ 10 -4 = 9 ´ 10-4
of mass is same for both the stars. \ N=9
2 2 4. 3 Let s be the surface mass density. Then
æ 5d ö ædö
2.2 M s ç ÷ w + 11 M s ç ÷ ´ w 1
L è 6ø è 6ø IO = s[p(2 R)2 ] ´ (2R)2 -
\ Total = 2
=6 2
LB ædö
11 M s ç ÷ ´ w é1 2ù
è 6ø 2 2 2
êë 2 (s p R ) + s (p R ) ´ R úû
2. 4
Under the influence of the force of stick (2N), the point of 13
contact O of the ring with ground tends to slide. But the = psR 4
2
frictional force f2 does not allow this and creates a torque
IP = 8 p s R4 + sp(2R)2 × (2R)2
which starts rolling the ring. A friction force f1 also acts

( (2R) + R ) ùúûú
between the ring & the stick. é1 2
2 2 2 2 2
Applying Fnet = ma in the horizontal direction. We get ê s (pR )R + s(pR )
ëê 2
= 24 p s R4 – 5.5 s p R4 = 18.5 p s R4
C IP 18.5 psR4 37
2N \ = = »3
R IO 13 13
R psR 4
f1 2
f2 5. 8 Applying conservation of angular momentum
O I1w1 = I2w2
2 – f2 = 2 × 0.3 \ f2 = 1.4 N
Applying t = Ia about C we get 1
I1ω1 MR 2 ´ ω1
a \ ω2 = = 2
(f2 – f1) R = Ia = I [Q For rolling a = Ra] I2 ì1 2 2 ü
R í MR + 2[2mr ]ý
î2 þ
0.3
\ [1.4 – m × 2] × 0.5 = 2 × (0.5)2 × [Q I = MR2]
0.5 1
´ 50´ 0.4 ´ 0.4´ 10
\ m = 0.4 2
=
P 1
Given m = ´ 50´ 0.4´ 0.4 + 2[2 ´ 6.25´ 0.2´ 0.2]
\ P=4 2
10
3. 9 40
Let the four spheres be A, B, C, & D = = 8 rad / s
4 +1
IXY = I1 + IB + IC + ID = 2 IA + 2IB
P-130 Topic-wise Solved Papers - PHYSICS

1. (c) When two small spheres of mass m are attached gently, 7. (d) Angular momentum (L)
the external torque, about the axis of rotation, is zero = (linear momentum) × (perpendicular distance of
and therefore the angular momentum about the axis of the line of action of
rotation is constant. momentum from the axis of rotation)
I1 = mv × r [Here r = 0 because the line of
\ I1w1 = I 2 w 2 Þ w 2 = w1 = mv × 0 action of momentum passes
I2 =0 through the axis of rotation]
1 2 1 2 2
Here I1 = MR and I 2 = MR + 2mR mass( M )
2 2 8. (d) We know that density (d ) =
volume(V )
1
MR 2 \ M = d ´ V = d ´ ( pR 2 ´ t ) .
2 M
\ w2 = ´ w1 = w1
1 M + 4m 1
MR 2 + 2mR 2 The moment of inertia of a disc is given by I = MR 2
2 2
2. (b) For negotiating a circular curve on a levelled road, the
1 pd
maximum velocity of the car is vmax = mrg \ I= ( d ´ pR 2 ´ t ) R 2 = t ´ R4
2 2
Here m = 0.6, r = 150 m, g = 9.8
\ vmax = 0.6´150´9.8 ; 30m / s I X t X RX4 t ´ R4 1
\ = = =
3. (b) The velocity of efflux is given IY tY RY4 t 64
´ (4R )4
v = 2gh 4
Where h is the height of the free surface of liquid from 1 L
the hole 9. (a) K .E . Iw 2 , but L = Iw Þ I =
2 w
\ v = 2 ´10 ´ 20 = 20 m / s
1L 1
4. (c) The velocity of centre of mass of two particle system is \ K .E. = ´ w 2 = Lw
given by 2w 2
m v + m2 v2 m m K .E L´w K .E L´w
vc = 1 1 \ = Þ =
m1 + m2 2v v K .E ' L ' ´ w ' K .E L '´ 2w
2
m(2v ) + m(-v ) v
= = L
m+m 2 \ L' =
5. (d) This is a case of sliding (the plane being frictionless) 4
and therefore the acceleration of all the bodies is same r r r
10. (d) We know that t = r ´ F
(g sin q).
F

g sin q r
t

6. (a) M. I of a circular wire about an axis nn' passing through


the centre of the circle and perpendicular to the plane
of the circle = MR2 r r
The angle between t and r is 90° and the angle
n
Z
Y r r
between t and F is also 90°. We also know that the
dot product of two vectors which have an angle of 90°
between them is zero. Therefore (d) is the correct option.
X 11. (b) Angular momentum will remain the same since external
torque is zero.
n' 12. (a) The moment of inertia of solid sphere A about its diameter
As shown in the figure, X-axis and Y-axis lie in the 2
plane of the ring . Then by perpendicular axis theorem I A = MR 2 .
IX + IY = IZ 5
Þ 2 IX = MR2 [Q IX = IY (by symmetry) and IZ = MR2] The moment of inertia of a hollow sphere B about its
1 2
\ IX = MR
2 diameter I B = MR 2 . \ I A < IB
2 3
ROTATIONAL MOTION P-131
13. (a) Does not shift as no external force acts. The centre of ur ur uur
ˆ ˆ
18. (c) Torque t = r ´ F = (i - j ) ´ ( - Fk ) ˆ
mass of the system continues its original path. It is only
the internal forces which comes into play while breaking. = F [- iˆ ´ kˆ + ˆj ´ kˆ] = F ( ˆj + iˆ) = F ( iˆ + ˆj)
14. (c) The disc may be assumed as combination of two semi
circular parts. éSince kˆ ´ iˆ = ˆj and ˆj ´ kˆ = iˆ ù
ë û
Let I be the moment of inertia of the uniform semicircular 19. (d) Applying conservation of angular momentum I'w' = Iw
disc
(mR 2 + 2MR 2 )w ' = mR 2 w
2Mr 2 Mr 2
Þ 2I = ÞI= é m ù
2 2 Þ w' = wê
l ë m + 2 M úû
15. (d) A B 20. (b) Let the mass per unit area 2R
(0, 2 l) be s.
Then the mass of the
complete disc O R
F P (0, l) 2l = s [p(2 R) 2 ] = 4psR 2

y
The mass of the removed disc = s (pR 2 ) = psR 2
Let us consider the above situation to be a complete
(0, 0) disc of radius 2R on which a disc of radius R of negative
C mass is superimposed. Let O be the origin. Then the
uur above figure can be redrawn keeping in mind the
To have linear motion, the force F has to be applied
concept of centre of mass as :
at centre of mass.
i.e. the point ‘P’has to be at the centre of mass 2 R
4ps R
2
m y + m2 y2 m ´ 2 l + 2m ´ l 4l O –ps R
y= 1 1 = =
m1 + m2
16. (c) Initially,
3m 3
xc.m =
(4psR ) ´ 0 + ( -psR ) R
2 2

4psR 2 - psR 2
-psR 2 ´ R R 1
\ xc.m = 2
\ xc.m = -
Þa=
m ( - x ) + m2 x2 3psR 3 3
0= 1 1 Þ m1 x1 = m2 x2 ...(1)
m1 + m2 21. (b) This is a standard formula and should be memorized.
Finally,
The centre of mass is at the origin g sin q
a=
x1– d I
x2– d ¢ 1+
d d¢ MR 2
uur
m1 O m2 uur d Lc
22. (d) We know that t c =
m1 (d - x1 ) + m2 ( x2 - d ') dt
\0 = uur
m1 + m2 where tc torque about the center of mass of the body
uur
m1 and Lc = Angular momentum about the center of mass
Þ 0 = m1d - m1 x1 + m2 x2 - m2 d ' Þ d ' = m d of the body. Central forces act along the center of mass.
2
Therefore torque about center of mass is zero.
[From (1).] uur uur
17. (c) l When t c = 0 then Lc = constt.
D C
n 23. (d) By the theorem of perpendicular axes,
O Iz = Ix + Iy or, Iz = 2 Iy
2
l/
A B (Q Ix = Iy by symmetry of the figure)
Z
Y
n'
D F C
Inn' = M.I due to the point mass at B +
M.I due to the point mass at D +
M.I due to the point mass at C.
2 X
æ l ö O
I nn ' = 2 ´ m ç + m( 2l) 2
è 2 ÷ø

= ml 2 + 2ml 2 = 3ml 2 A B
E
P-132 Topic-wise Solved Papers - PHYSICS
Iz ma 2 + 3ma 2 2 2
\ I EF = ...(i) = = ma
2 6 3
Again, by the same theorem Iz = IAC + IBD = 2 IAC 26. (c) O
(\ IAC = IBD by symmetry of the figure)
I
\ I AC = z ...(ii)
2 C. M
h
From (i) and (ii), we get IEF = IAC.
24. (a) · When n = 0, x = k where k is a constant. This means C. M Reference
level for P.E.
that the linear mass density is constant. In this case
the centre of mass will be at the midelle of the rod ie at
L/2. Therefore (c) is ruled out
· n is positive and as its value increases, the rate of 1 2
increase of linear mass density with increase in The moment of inertia of the rod about O is ml .
x increases. This shows that the centre of mass will 3
The maximum angular speed of the rod is when the rod
shift towards that end of the rod where n = L as the is instantaneously vertical. The energy of the rod in
value of n increases. Therefore graph (b) is ruled out.
1
æ xö x
n this condition is I w 2 where I is the moment of inertia
· The linear mass density l = k çè ÷ø Here £1 2
L L of the rod about O. When the rod is in its extreme
With increase in the value of n, the centre of mass shift portion, its angular velocity is zero momentarily. In this
towards the end x = L such that first the shifting is at a case, the energy of the rod is mgh where h is the
higher rate with increase in the value of n and then the maximum height to which the centre of mass (C.M)
rate decreases with the value of n. rises
These characteristics are represented by graph (a). 1 2 1 æ 1 2ö 2 l 2 w2
\ mgh = I w = 2 çè 3 ml ÷ø w Þ h =
L L L n 2 6g
æ xö
ò x dm ò x (l dx) ò k çè L ÷ø .xdx 27. (b) For translational motion,
mg – T = ma .....(1) m R
xCM = 0 = 0 = 0 For rotational motion,
L L L n
æxö T
ò dm ò l dx ò k çè L ÷ø dx T.R = I a = I
a
....(2)
0 0 0 R m
Solving (1) & (2), mg
L
é x n+ 2 ù mg mg 2 mg 2 g
kê a= = = =
nú æ I ö mR 2 3m 3
ëê (n + 2) L ûú 0 L(n + 1) çè m + ÷ m+
= = R2 ø 2R 2
é k x n +1 ù
L n+2 28. (c) As insect moves along a diameter, the effective mass
ê nú and hence the M.I. first decreases then increases so
ëê (n + 1) L ûú 0 from principle of conservation of angular momentum,
angular speed, first increases then decreases.
L 29. (a) F = 20t – 5t2
For n = 0 , xCM = ; n = 1,
2 FR dw
\ a= = 4t - t 2 Þ = 4t - t 2
2L 3L I dt
xCM = ; n = 2, xCM = ;....
3 4 w t

1 ma 2 Þ ò dw = ò (4t - t ) dt
2

25. (d) Inn' = m( a 2 + a 2 ) = 0 0


12 6
n m t3
Þ w = 2t 2 - (as w = 0 at t = 0, 6s)
3
A
q 6 æ
D t3 ö
ò d q = ò ç 2t 2 - ÷ dt

0 0 è
O 36
Þ q = 36 rad Þ n = <6
2p
B r
C
30. (c) o o
n' m1
DB 2a a From conservation of angular momentum about any fix
Also, DO = = = point on the surface,
2 2 2
According to parallel axis theorem mr 2 w 0 = 2mr 2 w
æ a ö
2
ma 2 ma 2 w0 r
Imm ' = I nn ' + m ç ÷ = 6 + 2 Þ w = w0 / 2 Þ v =
2
[Q v = rw ]
è 2ø
7 Gravitation

3. If the distance between the earth and the sun were half its
FILL IN THE BLANKS : present value, the number of days in a year would have
been (1996 - 2 Marks)
1. The numerical value of the angular velocity of rotation of
the earth should be ............rad/s in order to make the effective (a) 64.5 (b) 129 (c) 182.5 (d) 730
acceleration due to gravity equal to zero. (1984 - 2 Marks) 4. An artificial satellite moving in a circular orbit around the
2. A geostationary satellite is orbiting the earth at a height of Earth has a total (kinetic + potential) energy E0. Its potential
6 R above the surface of the earth, where R is the radius of energy is (1997C - 1 Mark)
the earth. The time period of another satellite at a height of (a) –E0 (b) 1.5 E0
2.5 R from the surface of the earth is ..........hours.
(1987 - 2 Marks) (c) 2E0 (d) E0
3. The masses and radii of the Earth and the Moon are M1, R1and 5. A geo-stationary satellite orbits around the earth in a circular
M2, R2 respectively. Their centres are at a distance d apart. The orbit of radius 36,000km. Then, the time period of a spy
minimum speed with which a particle of mass m should be satellite orbiting a few hundred km above the earth's surface
projected from a point midway between the two centres so as (Rearth = 6,400km) will approximately be (2002S)
to escape to infinity is ................. (1988 - 2 Marks) (a) 1/2 hr (b) 1 hr
4. A particle is projected vertically upwards from the surface (c) 2 hr (d) 4 hr
of earth (radius Re) with a kinetic energy equal to half of the
minimum value needed for it to escape. The height to which 6. A simple pendulum is oscillating without damping. When
it rises above the surface of earth is.... (1997 - 2 Marks) the displacement of the bob is less than maximum, its
acceleration vector ar is correctly shown in : (2002S)
TRUE / FALSE :

1. It is possible to put an artificial satellite into orbit in such a


way that it will always remain directly over New Delhi.
(1984 - 2 Marks) a
(a) (b)
MCQ's WITH ONE CORRECT ANSWER : a

1. If the radius of the earth were to shrink by one percent, its


mass remaining the same, the acceleration due to gravity on
the earth’s surface would (1981 - 2 Marks)
(a) decrease (b) remain unchanged
(c) increase (d) be zero (c) (d)
a a
2. If g is the acceleration due to gravity on the earth’s surface,
the gain in the potential energy of an object of mass m
raised from the surface of the earth to a height equal to the 7. A binary star system consists of two stars A and B which
radius R of the earth, is (1983 - 1 Mark) have time period TA and TB, radius RA and RB and mass MA
and MB. Then (2006 - 3M, –1)
1
(a) mg R (b) 2 mg R (a) if TA > TB then RA > RB (b) if TA > TB then MA > MB
2
2 3
æ TA ö æ RA ö
1
(c) mg R (d) mg R (c) çè T ÷ø = çè R ÷ø (d) TA = TB
4 B B
P-134 Topic-wise Solved Papers - PHYSICS
8. A spherically symmetric gravitational system of particles 2. A solid sphere of uniform density and radius 4 units is located
with its centre at the origin O of coordinates. Two spheres
ìr0 for r £ R
has a mass density r = í of equal radii 1 unit, with their centres at A (–2, 0 ,0) and
î0 for r > R B (2, 0, 0) respectively, are taken out of the solid leaving
where r0 is a constant. A test mass can undergo circular behind spherical cavities as shown in fig (1993-2 Marks)
motion under the influence of the gravitational field of
particles. Its speed v as a function of distance r (0 < r < ¥) y
from the centre of the system is represented by – (2008)
v v
-m
A B
x
O

z
(a) (b)
Then :
R r R r (a) The gravitational force due to this object at the origin
is zero.
v v (b) the gravitational force at the point B (2, 0 ,0) is zero.
(c) the gravitational potential is the same at all points of
circle y2 + z2 = 36.
(d) the gravitational potential is the same at all points on
(c) (d) the circle y2 + z2 = 4.
3. The magnitudes of the gravitational field at distance r1 and
R r R r r2 from the centre of a uniform sphere of radius R and mass
m are F1 and F2 respectively. Then: (1994 - 2 Marks)
9. A thin uniform annular disc (see figure) of mass M has outer
radius 4R and inner radius 3R. The work required to take a F1 r1
=
unit mass from point P on its axis to infinity is (2010) (a) F2 r2 if r1 < R and r2 < R

F1 r22
=
(b) F2 r12 if r1 > R and r2 > R

F1 r1
=
(c) F2 r2 if r1 > R and r2 > R
2GM 2GM
(a) (4 2 - 5) (b) - (4 2 - 5) F1 r12
7R 7R =
(d) F2 r22 if r1 < R and r2 < R
GM 2GM
(c) (d) ( 2 - 1) 4. A satellite S is moving in an elliptical orbit around the earth.
4R 5R The mass of the satellite is very small compared to the mass
10. A satellite is moving with a constant speed ‘V’ in a circular of the earth. (1998S - 2 Marks)
orbit about the earth. An object of mass ‘m’ is ejected from (a) The acceleration of S is always directed towards the
the satellite such that it just escapes from the gravitational centre of the earth.
pull of the earth. At the time of its ejection, the kinetic energy (b) The angular momentum of S about the centre of the
of the object is (2011) earth changes in direction, but its magnitude remains
1 constant.
(a) mV 2 (b) mV2
2 (c) The total mechanical energy of S varies periodically
3 with time.
(c) mV 2 (d) 2mV 2 (d) The linear momentum of S remains constant in
2
magnitude.
5. Two spherical planets P and Q have the same uniform
density r, masses MP and MQ and surface areas A and 4A
1. Imagine a light planet revolving around a very massive star respectively. A spherical planet R also has uniform density
in a circular orbit of radius R with a period of revolution T. If r and its mass is (MP + MQ). The escape velocities from the
the gravitational force of attraction between the planet and planets P, Q and R are VP, VQ and VR, respectively. Then
the star is proportional to R–5/2 (1989 - 2 Mark) (2012)
(a) T2 is proportional to R3 (a) VQ > VR > VP (b) VR > VQ > VP
(b) T2 is proportional to R7/2
(c) T2 is proportional to R3/2 1
(c) VR/ VP = 3 (d) VP / VQ =
(d) T2 is proportional to R3/73 2
GRAVITATION P-135
6. Two bodies, each of mass M, are kept fixed with a separation (ii) If the satellite is stopped suddenly in its orbit and
2L. A particle of mass m is projected from the midpoint of the allowed to fall freely onto the earth, find the speed with
line joining their centres, perpendicular to the line. The which it hits the surface of the earth.
gravitational constant is G. The correct statement(s) is (are) 4. Distance between the centres of two stars is 10a. The masses
(JEE Adv. 2013) of these stars are M and 16M and their radii a and 2a,
(a) The minimum initial velocity of the mass m to escape respectively. A body of mass m is fired straight from the
GM surface of the larger star towards the smaller star. What
the gravitational field of the two bodies is 4 should be its minimum initial speed to reach the surface of
L the smaller star? Obtain the expression in terms of G, M
(b) The minimum initial velocity of the mass m to escape and a. (1996 - 5 Marks)
GM 5. A body is projected vertically upwards from the bottom of a
the gravitational field of the two bodies is 2
L R
crater of moon of depth where R is the radius of moon
(c) The minimum initial velocity of the mass m to escape 100
with a velocity equal to the escape velocity on the surface
2GM
the gravitational field of the two bodies is of moon. Calculate maximum height attained by the body
L from the surface of the moon. (2003 - 4 Marks)
(d) The energy of the mass m remains constant
ASSERTION & REASON TYPE QUESTIONS :
SUBJECTIVE PROBLEMS :
1. STATEMENT - 1 : An astronaut in an orbiting space station
1. Two satellites S1 and S2 revolve round a planet in coplanar above the earth experiences weightlessness.
circular orbits in the same sense. Their periods of revolution
are 1 hour and 8 hours respectively. The radius of the orbit because
of S1 is 104 km. When S2 is closest to S1, find STATEMENT - 2 : An object moving the earth under the
influence of Earth’s gravitational force is in a state of “free-
(i) the speed of S2 relative to S1,
fall”. (2008)
(ii) the angular speed of S2 as actually observed by an (a) Statement- 1 is True, Statement-2 is True, Statement-2
astronaut in S1. (1986 - 6 Marks) is a correct explanation for Statement -1
2. Three particles, each of mass m, are situated at the vertices (b) Statement -1 is True, Statement -2 is True ; Statement-
of an equilateral triangle of side length a. The only forces 2 is NOT a correct explanation for Statement - 1
acting on the particles are their mutual gravitational forces. (c) Statement - 1 is True, Statement- 2 is False
It is desired that each particle moves in a circle while (d) Statement -1 is False, Statement -2 is True
maintaining the original mutual separation a. Find the intial
INTEGER VALUE CORRECT TYPE :
velocity that should be given to each particle and also the
time period of the circular motion.
(1988 - 5 Marks) 6
1. Gravitational acceleration on the surface of a planet is g.
3. An artificial satellite is moving in a circular orbit around the 11
earth with a speed equal to half the magnitude of escape where g is the gravitational acceleration on the surface of
velocity from the earth. (1990 - 8 Mark) 2
the earth. The average mass density of the planet is
(i) Determine the height of the satellite above the earth’s 3
times that of the earth. If the escape speed on the surface of
surface.
the earth is taken to be 11 kms–1, the escape speed on the
surface of the planet in kms–1 will be (2010)

1. The kinetic energy needed to project a body of mass m from 3. Energy required to move a body of mass m from an orbit of
the earth surface (radius R) to infinity is [2002] radius 2R to 3R is [2002]
(a) mgR/2 (b) 2mgR (a) GMm/12R2 (b) GMm/3R2
(c) mgR (d) mgR/4. (c) GMm/8R (d) GMm/6R.
2. If suddenly the gravitational force of attraction between 4. The escape velocity of a body depends upon mass as
Earth and a satellite revolving around it becomes zero, then (a) m0 (b) m1 [2002]
the satellite will [2002] (c) m 2 (d) m3.
(a) continue to move in its orbit with same velocity 5. The time period of a satellite of earth is 5 hours. If the
(b) move tangentially to the original orbit in the same separation between the earth and the satellite is increased
velocity to 4 times the previous value, the new time period will become
(c) become stationary in its orbit (a) 10 hours (b) 80 hours [2003]
(d) move towards the earth (c) 40 hours (d) 20 hours
P-136 Topic-wise Solved Papers - PHYSICS
6. Two spherical bodies of mass M and 5M & radii R & 2R 15. A planet in a distant solar system is 10 times more massive
respectively are released in free space with initial separation than the earth and its radius is 10 times smaller. Given that
between their centres equal to 12 R. If they attract each other the escape velocity from the earth is 11 km s–1, the escape
due to gravitational force only, then the distance covered by velocity from the surface of the planet would be [2008]
the smaller body just before collision is [2003] (a) 1.1 km s–1 (b) 11 km s–1
(a) 2.5 R (b) 4.5 R (c) 110 km s –1 (d) 0.11 km s–1
(c) 7.5 R (d) 1.5 R 16. This question contains Statement-1 and Statement-2. Of the
7. The escape velocity for a body projected vertically upwards four choices given after the statements, choose the one
from the surface of earth is 11 km/s. If the body is projected that best describes the two statements. [2008]
at an angle of 45°with the vertical, the escape velocity will be Statement-1 :
(a) 11 2 km / s (b) 22 km/s [2003] For a mass M kept at the centre of a cube of side ‘a’, the flux
11 of gravitational field passing through its sides 4 p GM.
(c) 11 km/s (d) km / s and
2
8. A satellite of mass m revolves around the earth of radius R Statement-2:
at a height x from its surface. If g is the acceleration due to If the direction of a field due to a point source is radial and
gravity on the surface of the earth, the orbital speed of the its dependence on the distance ‘r’ from the source is given
satellite is [2004] 1
as 2 , its flux through a closed surface depends only on
gR 2 gR æ gR 2 ö 1/ 2 r
(a) (b) (c) gx (d) ç ÷ the strength of the source enclosed by the surface and not
R+x R-x è R + xø on the size or shape of the surface.
9. The time period of an earth satellite in circular orbit is
independent of [2004] (a) Statement -1 is false, Statement-2 is true
(a) both the mass and radius of the orbit (b) Statement -1 is true, Statement-2 is true; Statement -2 is
(b) radius of its orbit a correct explanation for Statement-1
(c) the mass of the satellite (c) Statement -1 is true, Statement-2 is true; Statement -2
(d) neither the mass of the satellite nor the radius of its is not a correct explanation for Statement-1
orbit. (d) Statement -1 is true, Statement-2 is false
10. If ‘g’ is the acceleration due to gravity on the earth’s surface, 17. The height at which the acceleration due to gravity becomes
the gain in the potential energy of an object of mass ‘m’ g
(where g = the acceleration due to gravity on the surface
raised from the surface of the earth to a height equal to the 9
radius ‘R' of the earth is [2004] of the earth) in terms of R, the radius of the earth, is : [2009]
1 1 R
(a) mgR (b) mgR (c) 2 mgR (d) mgR (a) (b) R / 2 (c)
4 2 2 R (d) 2 R
2
11. Suppose the gravitational force varies inversely as the nth 18. Two bodies of masses m and 4 m are placed at a distance r.
power of distance. Then the time period of a planet in circular The gravitational potential at a point on the line joining
orbit of radius ‘R’ around the sun will be proportional to them where the gravitational field is zero is: [2011]
æ n -1ö
çè ÷ 4Gm 6Gm 9Gm
(a) n (b) R 2 ø [2004] (a) - (b) - (c) - (d) zero
R r r r
æ n +1ö æ n- 2ö 19. Two particles of equal mass ‘m’ go around a circle of radius
çè ÷ çè ÷
(c) R 2 ø (d) R 2 ø R under the action of their mutual gravitational attraction.
12. The change in the value of ‘g’ at a height ‘h’ above the The speed of each particle with respect to their centre of
surface of the earth is the same as at a depth ‘d’ below the mass is : [2011RS]
surface of earth. When both ‘d’ and ‘h’ are much smaller
than the radius of earth, then which one of the following is Gm Gm Gm Gm
(a) (b) (c) (d)
correct ? [2005] 4R 3R 2R R
3h h 20. The mass of a spaceship is 1000 kg. It is to be launched from
(a) d = (b) d = the earth's surface out into free space. The value of g and R
2 2
(c) d = h (d) d =2 h (radius of earth) are 10 m/s2 and 6400 km respectively. The
13. A particle of mass 10 g is kept on the surface of a uniform required energy for this work will be :
sphere of mass 100 kg and radius 10 cm. Find the work to [2012]
be done against the gravitational force between them to (a) 6.4 × 1011Joules (b) 6.4 × 108 Joules
take the particle far away from the sphere (you may take G (c) 6.4 × 109 Joules (d) 6.4 × 1010 Joules
= 6.67× 10-11 Nm 2 / kg 2 ) [2005] 21. What is the minimum energy required to launch a satellite of
mass m from the surface of a planet of mass M and radius R
(a) 3.33 × 10 -10 J (b) 13.34 × 10 -10 J in a circular orbit at an altitude of 2R? [JEE Main 2013]
(c) 6.67 × 10 -10 J (d) 6.67 × 10 -9 J 5GmM 2GmM
14. Average density of the earth [2005] (a) (b)
(a) is a complex function of g 6R 3R
(b) does not depend on g GmM GmM
(c) is inversely proportional to g (c) (d)
(d) is directly proportional to g 2R 2R
GRAVITATION P-137

Solutions & Explanations


Section-A : JEE Advanced/ IIT-JEE
4G
A 1. 1.23 × 10–3 rad/s 2. 8.48 hr 3. v= ( M1 + M 2 ) 4. h=R
d
B 1. F
C 1. (c) 2. (a) 3. (b) 4. (c) 5. (c) 6. (c) 7. (d)
8. (c) 9. (a) 10. (b)
D 1. (b) 2. (a, c, d) 3. (a, b) 4. (a, c) 5. (b, d) 6. (b)

Gm a3
E 1. (i) -p ´ 10 4 km/hr (ii) 3 ´ 10 -4 rad/s 2. , 2p
a 3Gm

3 5GM
3. (i) 6400 km (ii) 7.92 km/s 4. 5. 99.5 R
2 a
H 1. (a)
I 1. 3

Section-B : JEE Main/ AIEEE


1. (c) 2. (b) 3. (d) 4. (a) 5. (c) 6. (c)
7. (c) 8. (d) 9. (c) 10. (b) 11. (c) 12. (d)
13. (c) 14. (d) 15. (c) 16. (b) 17. (d) 18. (c)
19. (a) 20. (d) 21. (a)

This increase in P.E. is at the expense of K.E. of mass m


FILL IN THE BLANKS :
1 2 Gm
1. We know that g' = g – Rw2 cos2 f \ mv = ( M1 + M 2 )
2 d /2
At equator, f = 0, Therefore g' = g – Rw2
where v is the velocity with which mass m is projected.
g
Here g' = 0 \ w= = 1.23 ´ 10-3 rad/s 4G
R Þ v= (M1 + M 2 )
d
2. KEY CONCEPT : According to Kepler's law T 2 µ R3
4. B Vel=0
T12 R13
= . Here R1 = R + 6 R = 7 R h v
T22 R23
and R2 = 2.5 R + R = 3.5 R A
R
24 ´ 24 7 ´ 7 ´ 7 ´ R3
Þ = Þ T2 = 8.48 hr
T22 3.5 ´ 3.5 ´ 3.5 ´ R 3
3. Increase in P.E. of system
= {(P.E.)i – (P.E.)f}
2GM
ì é GM 1M 2 GM 1m GM 2 m ù é GM 1M 2 ù ü Escape velocity, ve =
= – í ê- - - - - úý R
îë d d /2 d / 2 úû êë d ûþ
\ Kinetic energy required for escape velocity
Gm 1 2 GMm
= ( M1 + M 2 ) mve =
d /2 2 R
P-138 Topic-wise Solved Papers - PHYSICS
The kinetic energy given is half of this value
T12 R3
1 2 GMm 3. (b) According to Kepler's law = 1
\ mv = ... (i) T22 R23
2 2R
Applying energy conservation R
Here T1 = 365 days ; T2 = ?; R1 = R ; R2 =
Total energy at A = Total energy at B 2
1 2 æ GMm ö GMm æ R2 ö
3/ 2 3/ 2
mv + ç - ÷ =- ... (ii) é R /2ù
2 è R ø ( R + h) Þ T2 = T1 ç ÷
èR ø
= 365 ê ú = 129 days
1 ë R û
From (i) and (ii)
4. (c) P.E. = 2T.E.
GMm GMm GMm Þ P.E. = 2E0
- =- Note : In case of circular motion of satellite around
2R R R+h
earth / planets revolving around sun/electrons
1 1 revolving in circular orbit P.E. = 2 T.E. and |K.E.| = |T.E.|].
or, - =- Þ R + h = 2R
2R R+h 5. (c) Note : A satellite revolving near the earth's surface has
or, h = R a time period of 84.6 min.
We know that as the height increases, the time period
TRUE / FALSE : increases. Thus the time period of the spy satellite
should be slightly greater than 84.6 minutes.
1. False \ Ts = 2 hr
New Delhi is not on the equatorial plane and geostationary 6. (c) The components of acceleration are as shown
satellite is launched on the equatorial plane. r r
a = ar + at
MCQ's WITH ONE CORRECT ANSWER :
ar
GM GM a
1. (c) g = 2 and g' =
R (0.99 R) 2
at
2
g' æ R ö The resultant of transverse and radial component of
\ =ç Þ g' > g
÷
g è 0.99 R ø the acceleration is represented by ar
7. (d) Note : The gravitational force of attraction between
GMm the stars will provide the necessary centripetal forces.
2. (a) Ui = - = Initial potential energy of the system.
R In this case angular velocity of both stars is the same.
æ 2p ö
GMm Therefore time period remains the same. ç w = ÷ .
Uf = - = Final P.E. of the system. è T ø
2R
é 1 1 ù GMm
\ DU = Uf – Ui = – GMm ê - = ...(i)
ë 2R R úû 2R RA
MB MA
GM B RB
But g =
R2
R
\ GM = gR2 ... (ii)
From (i) and (ii) A 8. (c) This is a case of a sphere of uniform mass distribution.
For r ³ R
gR 2 m mgR The whole mass inside the sphere can be assumed to
DU == R
2R 2 be concentrated at the centre of the sphere. Let m be
ALTERNATE SOLUTION O the mass of test mass. Here the gravitational pull
Remember the formula between the spherically symmetrical gravitational
system and the test mass provides the necessary
mgR centripetal force required for the circular motion of the
DU =
h test mass.
1+
R æ 4 ö
G ç ro ´ pR 3 ÷ ´ m
where h = height of body of mass m from the surface of
Therefore è 3 ø mv 2
the earth and DU is the change in P.E. =
r2 r
1
Þ v2 × r = constt Þ v µ ...(i)
r
GRAVITATION P-139
For r < R The gravitational potential of the mass dm at P is
When the test mass m is inside the spherically
-G dm G 2M
symmetric gravitational system at a distance r from its dV = =- ´ xdx
centre, it will effectively get attracted towards the centre ( 4R ) 2
+ x2
2
16 R + x 2 7R2
by the mass inside the dotted sphere .
-2GM xdx
Spherically symmetrical = 2
(1)
gravitational system 7R 16 R 2 + x 2
Suppose 16R2 + x2 = t2
R Þ 2xdx = 2tdt
Þ xdx = tdt
m Also for x = 3R, t = 5R
r and for x = 4R, t = 4 2R
On integrating equation (1), taking the above limits,
we get
4 2R
2GM -2GM 4 2R
V =- ò dt = [t ]5 R
5R
7R2 7R2
æ 4 ö
G ç r0 ´ p r 3 ÷ ´ m 2
è 3
r 2
ø
=
mv
r
=
-2GM é
7R2 ë
4 2 R - 5R ùû =V =
-2GM
7R
4 2 -5( )
Þ vµr ...(ii) WP¥
equation (i) and (ii) are graphically represented by Now = V¥ - VP = -VP [Q V¥ = 0]
1
graph (c).
ALTERNATE SOLUTION
For r ³ R
\
2GM
WP¥ =
7R
4 2 -5 ( )
Force on the test mass m is F = m × | Eg | 10. (b) V is the orbital velocity. If VC is the escape velocity
Where Eg is the gravitational field intensity at the point
then Ve = 2V . The kinetic energy at the time of ejection
of observation
2 1 1
\ mv = m ´ é GM ù where M is the total mass of the KE = mVe2 = m( 2 V )2 = mV 2
r ê 2 ú 2 2
ë r û
spherical system.
1
\ vµ
r 1. (b) KEY CONCEPT : The centripetal force is provided by
For r < R Again F ' = m Eg' the gravitational force of attraction
mRw2 = GMmR–5/2
2
\ mv = m é GM ´ r ù mR ´ 4p 2 GMm
r ê 3 ú Þ = Þ T2 µ R7/2
ë R û 2
T R5 / 2
Þ vµr
2. (a,c,d) The gravitational field intensity at the point O is zero
9. (a) Let us consider a circular elemental area of radius x and
thickness dx. The area of the shaded portion = 2pxdx. (as the cavities are symmetrical with respect to O). Now the
Let dm be the mass of the shaded portion. force acting on a test mass m0 placed at O is given by
F = m0 E = m0 × 0 = 0
P Now, y2 + z2 = 36 represents the equation of a circle with
centre (0, 0, 0) and radius 6 units the plane of the circle is
4R perpendicular to x-axis.

x dx

A B
Mass M dm (-2,0,0)
O (2,0,0)
\ = = X
(
2
)
area p 4 R - p ( 3R ) 2
2pxdx 4

2M
\ dm = xdx Z
7R2
P-140 Topic-wise Solved Papers - PHYSICS
Note : Since the spherical mass distribution behaves as if
1 é GMm ù GM
the whole mass is at its centre (for a point outside on the 6. (b) mv 2 = 2 ê Þv=2
sphere) and since all the points on the circle is equidistant 2 ë L úû L
from the centre of the sphere, the circle is a gravitational The potential energy is a combined property of the three
equipotential. mass system. The kinetic energy of mass m is only its energy
The same logic holds good for option (d). which decreases as it moves.
GM (b) is the correct option.
3. (a,b) For r > R, the gravitational field is F =
r2
GM GM F1 r22
\ F1 = and F2 = Þ =
r12 r22 F2 r12
GM
For r < R, the gravitational field is F = ´r
R3
GM GM SUBJECTIVE PROBLEMS :
\ F1 = 3
´ r1 and F2 = 3
´ r2
R R 1. (i) According to Kepler's third law
F1 r1
Þ = T12 R3 T2
F2 r2 = 1 Þ R23 = R13 ´ 2
4. (a, c)Force on satellite is always towards earth, therefore, T22 R23 T12
acceleration of satellite S is always directed towards
( ) 82
3
centre of the earth. Net torque of this gravitational force \ R23 = 104 ´
2
= 64 ´ 1012
F about centre of earth is zero. Therefore, angular 1
momentum (both in magnitude and direction) of S about Þ R2 = 4 × 104 km.
centre of earth is constant throughout. Since the force Linear speed of satellite S1
F is conservative in nature, therefore mechanical
energy of satellite remains constant. Speed of S is 2pR1 2p ´ 104
maximum when it is nearest to earth and minimum when v1 = = = 2p ´ 104 km/hr
T1 1
it is farthest.
5. (b, d) Let the mass of P be m. Linear speed of satellite S2,

4 3 4 éAù
3/ 2 2pR2 (2p) (4 ´ 104 )
Then m = r´ pr = r´ p ê ú v2 = = = p ´ 104 km/hr
3 3 ë 4p û T2 8
The speed of satellite S2 w.r.t. S1
3/2
4 é 4A ù = v2 – v1 = p × 104 – 2p × 104 = – p × 104 km/hr
The mass of Q = r ´ p ê =8m
3 ë 4p úû
v2
\ The mass of R = 9 m S2
If the radius of P = r v1
Then the radius of Q = 2r
S1
é æ 4A ö
3/2
æ Aö
3/2 ù
P
êQ rQ = ç ÷ = 2 ç ÷ ú R1
ëê è 4p ø è 4p ø ûú
and radius of R = 91/3r
R2
éQ MR = MP + MQ ù
ê 3 3 3 3
ú
êë rR = r + (2r ) = 9r úû (ii) Angular speed of S2 w.r.t. S1
vr v -v
2GMP 2Gm = = 2 1
Now, vP = = Rr R2 - R1
Rp r
3.14 ´ 104 ´ 5 /18
2GMQ 2G (8m) = = 3 × 10–4 rad/s
= = 2v P 3 ´ 104 ´ 103
vQ = RQ 2r 2. The radius of the circle

2G (9 m) 2 2 a2 a
vR = = 91/3 vP r= a - =
91/3 r 3 4 3
GRAVITATION P-141
m
4. MB=16M MS=M

2a a
P
Bigger Smaller
a /Ö 3 star x star
F
FR 10a
m a m
F The force of attraction is zero at say x from the bigger star.
v Then force on mass m due to bigger star = Force on mass m
due to small star
Let v be the velocity given. The centripetal force is provided
by the resultant gravitational attraction of the two masses. GM B m GM S m 16 M M
2
= Þ =
x (10a - x) 2 x 2
(10a - x) 2
FR = F 2 + F 2 + 2 F 2 cos 60°
Þ x = 8a
m´m If we throw a mass m from bigger star giving it such a velocity
= 3 F = 3G that is sufficient to bring it to P, then later on due to greater
a2
force by the star MS it will pull it towards itself [without any
æ mv 2 ö external energy thereafter].
m2mv 2 = centripetal force÷ The energy of the system (of these masses) initially
\ 3G 2 = ç
a r è r ø = Final energy when m is at P
GM B M S GM B m GM S m 1 2
3Gmr 3 G ma Gm - - - + mv
v2 = = Þ v= 10a 2a 8a 2
a2 a2 ´ 3 a
GM B M S GM B m GM S m
Time period of circular motion =- - -
10a 8a 2a
2pr 2 p a / 3 a3 [Q M B = 16M ; M S = M ]
T= = = 2 p
v Gm 3Gm
3 5GM
a \ v=
2 a
3. (i) KEY CONCEPT : Since the satellite is revolving in a 5. Total energy at A = Total energy at B
circular orbit, the centripetal force is provided by the (K.E.)A + (P.E.)A = (P.E.)B
gravitational pull.
2GM é -GMm ìï 2 æ 99 R ö üï ù
2
v 1 GMm
mv 2 GMm m Þ m´ +ê í3R - ç ÷ ýú = -
= A 2 R 3
êë 2 R ïî è 100 ø ïþ úû R+h
( R + h) ( R + h)2
B h B
GM M R
\ v2 =
R+h E
ve R
1 1 2GM A 100
But v = ve = 99R
2 2 R 100
R
1 æ 2GM ö GM
\ ç ÷=
4è R ø R+h
Þ 2R + 2h = 4R Þ h = R = 6400 km.
(ii) KEY CONCEPT : When the satellite is stopped, its
kinetic energy is zero. When it falls freely on the Earth, its On solving we get h = 99.5 R.
potential energy decreases and converts into kinetic energy. ASSERTION & REASON TYPE QUESTIONS :
\ (P.E.)A – (P.E.)B = K.E.
1. (a) The normal force exerted by the astronaut on orbiting
-GMm æ -GMm ö 1 2 space station is zero (until the astronaut exerts some
Þ -ç = mv
2R è R ÷ø 2 muscular force). Therefore the apparent weight of
astronaut in an orbiting space station is zero. Astronaut
GM is called in a state of weightlessness. This is because
Þ v= = gR = 9.8 ´ 6.4 ´ 106
R astronaut as well as space -ship are freely falling bodies.
= 7920 m/s = 7.92 km/s Statement - 1 is true, statement - 2 is true and statement
- 2 is the correct explanation of statement - 1.
P-142 Topic-wise Solved Papers - PHYSICS

INTEGERVALUECORRECTTYPE: \
6 2 Rp
= ´
é rp 2
êQ
ù
= (given)ú
11 3 R ë r 3 û
1. We know that v = 2 gR
Rp 3 6
vp gp Rp \ = ...(iii)
\ = ´ ...(i) R 22
v g R
vp 6 3 6 3´ 6 3
gp 6 From (i), (ii) & (iii) = ´ = =
Given = ...(ii) v 11 22 11 ´ 22 11
ge 11
3 3
gp rp Rp \ vp = ´ v = ´11km/s = 3 km/s
4 11 11
Also g= p G rR \ = ´
3 g r R

1 Let t be the time taken for the two masses to collide and
1. (c) K. E = m ve2 where ve = escape velocity = 2gR x5M, xM be the distance travelled by the mass 5M and
2
M respectively.
1 For mass 5M
\ K.E = m ´ 2 gR = mgR
2 u = 0, S = x5M , t = t , a = a5M
2. (b) Due to inertia of motion it will move tangentially to the
original orbit in the same velocity. 1
3. (d) Energy required = (Potential energy of the Earth -mass S = ut + at 2
2
system when mass is at distance 3R) – (Potential energy
of the Earth -mass system when mass is at distance 2R) 1
\ x5 M = a5M t 2 ....(ii)
-GMm æ -GMm ö -GMm GMm 2
-ç +
=
3R è 2R ÷ø = 3R 2R
For mass M
u = 0, s = xM, t = t, a = aM
-2GMm + 3GMm GMm
= = 1 2
6R 6R \ s = ut + at
2
2GM
4. (a) Escape velocity, ve = 2 gR = Þ Ve µ m0 1
R Þ xM = aM t 2 … (iii)
2
Where M, R are the mass and radius of the planet
respectively. In this expression the mass of the body Dividing (ii) by (iii)
(m) is not present showing that the escape velocity is 1
independent of the mass. x5 M a5 M t 2
(c) According to Kepler’s law of planetary motion T2 µ R3 = 2 a 1
5.
xM 1 = 5M = [From (i)]
3 3 a t2 aM 5
æR ö 2
é 4R ù 2 2 M
\ T2 = T1 ç 2 ÷ =5´ ê ú = 5 ´ 23 = 40 hour
èR ø 1 ëRû \ 5 x5M = xM ....(iv)
6. (c) The gravitational force acting on both the masses is From the figure it is clear that
the same. We know that x5M+ xM= 9R ....(v)
Force = mass × acceleration. Where O is the point where the two spheres collide.
9R From (iv) and (v)
xM
xM x5M + xM = 9 R
R 2R 5
12R 45
\ 6 xM = 45R \ xM = R = 7.5 R
1 6
For same force, acceleration µ
mass 7. (c) ve = 2 gR
a5 M M 1 The escape velocity is independent of the angle at
\ = = .....(i)
aM 5M 5 which the body is projected.
GRAVITATION P-143
8. (d) Gravitational force provides the necessary centripetal 13. (c) Workdone,
force.
é -GMm ù
W = DU = U f - U i = 0 - ê
mv 2 GmM GM ë R úû
\ = also g =
( R + x) ( R + x) 2
R2
6.67 ´10 -11 ´ 100 10
W= ´ = 6.67 × 10–10 J
mv 2 2 0.1 1000
æ GM ö R
\ = mç ÷
( R + x) è R 2 ø ( R + x) 2 4
G ´ r ´ pR 3
GM Gr ´ V 3
14. (d) g = = Þg=
mv 2 R2 R2 R2 R 2
\ = mg
( R + x) ( R + x) 2 4
g= rpG. R where r ® average density
1/ 2 3
gR 2 æ gR 2 ö
\ v2 = Þ v=ç ÷ æ 3g ö
R+ x è R + xø r =ç
è 4pGR ÷ø

mv 2 GmM
Þ r is directly proportional to g.
9. (c) We have, =
R + x ( R + x )2 2GM p
x = height of satellite from earth surface (ve ) p Rp Mp Re
m = mass of satellite 15. (c) = = ´ =
(ve )e 2GM e Me Rp
2GM GM Re
Þv = or v =
( R + x) R+x

2p ( R + x ) 2 p ( R + x ) 10M e Re
T= = = ´ = 10
v Me R e /10
GM
R+x \ (ve ) p = 10 ´ (ve )e = 10 ´ 11 = 110 km / s
which is independent of mass of satellite 16. (b) Gravitational flux through a closed surface is given by
uuur r
GmM
10. (b) At earth surface, P.E. of system is – ò E g dS = -4pGM
R
where, M = mass enclosed in the closed surface
At a distance R from the earth's surface, P.E .of system
1
GmM This relationship is valid when | E g | µ 2 .
is - r
2R
-GmM GmM GmM g' R2
\ DU = + ; DU = 17. (d) We know that g =
2R R 2R ( R + h) 2

GM GM 1 2
Now = g; \ = gR \ DU = mgR g /9 é R ù
\ =ê \ h = 2R
R2 R 2 g ë R + h úû
11. (c) F = KR–n = MRw2 Þ w 2 = KR - ( n +1) 18. (c) Let the gravitational field at P, distant x from mass m,
be zero.
- ( n +1)
Gm 4Gm 1 2
or w = KR 2
\ = Þ =
x2 ( r - x )2 x r-x
- (n +1) + ( n +1)
2p \ r – x = 2x
µR 2
\T µ R 2
T r
x=
3
é 2h ù
12. (d) Variation of g with altitude is, gh = g ê1 - ú ; m P 4m
ë Rû
x
é dù r
variation of g with depth is, gd = g ê1 - ú
ë Rû Gm 4Gm 9Gm
Gravitational potential at P, V = - r - 2r = -
r
Equating g h and gd , we get d = 2h 3 3
P-144 Topic-wise Solved Papers - PHYSICS
19. (a) Here, centripetal force will be given by the gravitational
GMm GM
force between the two particles. Þ mg = Þ g= … (ii)
2
R R2
Gm2 The required energy for this work is given by
= mw 2 R
( 2R) 2
R = U - W = mgR 2
Gm m m = 1000 × 10 × 6400 × 103
Þ = w2
4R 3 = 6.4 × 1010 Joules
21. (a) As we know,
Gm
Þ w= -GMm
4 R3 Gravitational potential energy =
r
If the velocity of the two particles with respect to the
centre of gravity is v then v = wR and orbital velocity, v0 = GM / R + h

Gm Gm 1 GMm 1 GM GMm
v= ´R= Ef = mv02 - = m -
4R 3
4R 2 3R 2 3R 3R
20. (d) The work done to launch the space ship GMm æ 1 ö - GMm
= ç - 1÷ =
¥ ur uur ¥ 3R è 2 ø 6R
GMm
W = - ò F.dr = - ò 2 dr
R R r -GMm
Ei = +K
R
GMm
W =+ … (i) Ei = E f
R
The force of attraction of the earth on the space ship, 5GMm
when it was on the earth's surface Therefore minimum required energy, K =
6R
GMm
F=
R2
Mechanical Properties
8 of Solids & Fluids

FILL IN THE BLANKS : 3. Water in a closed tube (see Fig) is heated with one arm
vertically placed above a lamp. Water will begin to circulate
1. A wire of length L and cross sectional area A is made of a
along the tube in counter-clockwise direction.
material of Young’s modulus Y. If the wire is stretched by an
amount x, the work done is ................. (1987 - 2 Marks) (1983 - 2 Marks)
2. A solid sphere of radius R made of a material of bulk modulus
K is surrounded by a liquid in a cylindrical container. A
massless pistion of area A floats on the surface of the liquid.
When a mass M is placed on the piston to compress
the liquid the fractional change in the radius of the sphere,
d R/R, is ........... (1988 - 2 Mark) A B
3. A piece of metal floats on mercury. The coefficients of
volume expansion of the metal and mercury are g1 and g 2
respectively. If the temperatures of both mercury and the
metal are increased by an amount DT , the fraction of the
volume of the metal submerged in mercury changes by the
factor ................... (1991 - 2 Mark) 4. A block of ice with a lead shot embedded in it is floating on
4. A horizontal pipeline carries water in a streamline flow. water contained in a vessel. The temperature of the system
At a point along the pipe, where the cross- sectional is maintained at 0°C as the ice melts. When the ice melts
area is 10 cm 2, the water velocity is 1 ms–1 and the completely the level of water in the vessel rises .
pressure is 2000 Pa. The pressure of water at another (1986 - 3 Marks)
point where the cross-sectional area is 5 cm 2, is... Pa.
(Density of water = 103 kg.m –3) (1994 - 2 Marks) MCQ's WITH ONE CORRECT ANSWER :

TRUE / FALSE : 1. A vessel containing water is given a constant acceleration


‘a’ towards the right along a straight horizontal path. Which
1. A man is sitting in a boat which is floating in a pond. If the
of the following diagrams in Fig. represents the surface of
man drinks some water from the pond, the level of the water
the liquid ? (1981- 2 Marks)
in the pond decreases. (1980)
2. A barometer made of a very narrow tube (see Fig) is placed
at normal temperature and pressure. The coefficient of a a a
volume expansion of mercury is 0.00018 per C° and that of
the tube is negligible. The temperature of mercury in the
barometer is now raised by 1°C, but the temperature of the (a) (b) (c)
atmosphere does not change. Then the mercury height in 2. The following four wires are made of the same material.
the tube remains unchanged. (1983 - 2 Marks) Which of these will have the largest extension when the
vacuum same tension is applied ? (1981- 2 Marks)
(a) length = 50 cm , diameter = 0.5 mm
Hg (b) length = 100 cm, diameter = 1 mm
(c) length = 200 cm, diameter = 2 mm
(d) length = 300 cm, diameter = 3 mm.
P-146 Topic-wise Solved Papers - PHYSICS
3. A U-tube of uniform cross section (see Fig) is partially filled
with a liquid I. Another liquid II which does not mix with
liquid I is poured into one side. It is found that the liquid
levels of the two sides of the tube are the same, while the
level of liquid I has risen by 2 cm. If the specific gravity of
liquid I is 1.1, the specific gravity of liquid II must be h
(1983 - 1 Mark)
r

II I 2R
(a) Mg (b) Mg – Vrg
(c) Mg + pR2hrg (d) rg(V + pR2h)
7. A wooden block, with a coin placed on its top, floats in
water as shown in figure. The distance l and h are shown
(a) 1.12 (b) 1.1 here. After some time the coin falls into the water. Then
(c) 1.05 (d) 1.0 (2002S )
4. A homogeneous solid cylinder of length L (L < H/2), cross- coin
sectional area A/5 is immersed such that it floats with its axis
vertical at the liquid-liquid interface with length L/4 in the l
denser liquid as shown in the figure. The lower density
h
liquid is open to atmosphere having pressure P0. Then
density D of solid is given by (1995S)
(a) l decreases and h increases
(b) l increases and h decreases
(c) both l and h increase (d) both l and h decrease
8. The adjacent graph shows the estension (Dl) of a wire of
3L/4 length 1 m suspended from the top of a roof at one end and
d
H with a load W connected to the other end. If the cross-
L/4 sectional area of the wire is 10–6 m2, calculate the Young’s
2d modulus of the material of the wire. (2003S)

Dl (x10 -4 m)

5 4 4
(a) d (b) d
4 5 3

d 2
(c) 4d (d) 1
5
W (N)
5. A large open tank has two holes in the wall. One is a square 20 40 60 80
hole of side L at a depth y from the top and the other is a
(a) 2 × 1011 N/m (b) 2 × 10–11 N/m
circular hole of radius R at a depth 4 y from the top. When (c) 3 × 10–12 N/m (d) 2 × 10–13 N/m
the tank is completely filled with water, the quantities of 9. Water is filled in a container upto height 3m. A small hole
water flowing out per second from both holes are the same. of area ‘a’ is punched in the wall of the container at a
Then, R is equal to (2000S) height 52.5 cm from the bottom. The cross sectional area
of the container is A. If a/A = 0.1 then v2 is (where v is the
L L
(a) (b) 2 pL (c) L (d) velocity of water coming out of the hole) (2005S)
2p 2p (a) 50 (b) 51
6. A hemispherical portion of radius R is removed from the (c) 48 (d) 51.5
bottom of a cylinder of radius R. The volume of the remaining 10. When temperature of a gas is 20ºC and pressure is changed
cylinder is V and its mass M. It is suspended by a string in from p1 = 1.01 × 105 Pa to p2 =1.165 × 105 Pa then the
a liquid of density r where it stays vertical. The upper surface volume changed by 10%. The bulk modulus is (2005S)
of the cylinder is at a depth h below the liquid surface. The (a) 1.55 × 105 Pa (b) 0.115 × 105 Pa
(c) 1.4 × 10 Pa5 (d) 1.01 × 105 Pa
force on the bottom of the cylinder by the liquid is (2001S)
MECHANICAL PROPERTIES OF SOLIDS & FLUIDS P-147
11. A glass tube of uniform internal radius (r) has a valve
separating the two identical ends. Initially, the valve is in a
tightly closed position. End 1 has a hemispherical soap A
bubble of radius r. End 2 has sub-hemispherical soap bubble
as shown in figure. Just after opening the valve, (2008)
m
B

(a) the balance A will read more than 2 kg


1 (b) the balance B will read more than 5 kg
2
(c) the balance A will read less than 2 kg and B will read
(a) air from end 1 flows towards end 2. No change in the more than 5 kg
volume of the soap bubbles (d) the balance A and B will read 2 kg and 5 kg respectively
(b) air from end 1 flows towards end 2. Volume of the soap 3. A vessel contains oil (density = 0.8 gm/cm3) over mercury
bubble at end 1 decreases (density = 13.6 gm cm3). A homogeneous sphere floats with
(c) no changes occurs half its volume immersed in mercury and the other half in oil.
(d) air from end 2 flows towards end 1. volume of the soap The density of the material of the sphere in gm/cm3 is
bubble at end 1 increases (1988 - 2 Mark)
12. A thin uniform cylindrical shell, closed at both ends, is (a) 3.3 (b) 6.4
partially filled with water. It is floating vertically in water in (c) 7.2 (d) 12.8
half-submerged state. If rc is the relative density of the 4. Two rods of different materials having coefficients of thermal
material of the shell with respect to water, then the correct
expansion a1, a 2 and Young’s modulii Y1, Y2 respectively
statement is that the shell is (2012- II)
(a) more than half-filled if rc is less than 0.5. are fixed between two rigid massive walls. The rods are
(b) more than half-filled if rc is more than 1.0. heated such that they undergo the same increase in
(c) half-filled if rc is more than 0.5. temperature. There is no bending of the rods. If a1 : a 2 =
(d) less than half-filled if rc is less than 0.5.
2 : 3, the thermal stresses developed in the two rods are
13. One end of a horizontal thick copper wire of length 2L and
radius 2R is welded to an end of another horizontal thin equal provided Y1 : Y2 is equal to (1989 - 2 Mark)
copper wire of length L and radius R. When the arrangement (a) 2 : 3 (b) 1 : 1
is stretched by applying forces at two ends, the ratio of the (c) 3 : 2 (d) 4 : 9
elongation in the thin wire to that in the thick wire is
5. Water from a tap emerges vertically downwards with an
(JEE Adv. 2013)
initial spped of 1.0 m s–1. The cross-sectional area of the tap
(a) 0.25 (b) 0.50
is 10–4 m2. Assume that the pressure is constant throughout
(c) 2.00 (d) 4.00
the stream of water, and that the flow is steady. The cross-
sectional area of the stream 0.15 m below the tap is
(1998S - 2 Marks)
1. A body floats in a liquid contained in a beaker. The whole
system as shown in Figure falls freely under gravity. The (a) 5.0 × 10–4 m2 (b) 1.0 × 10–5 m2
upthrust on the body is (1982 - 3 Marks) (c) 5.0 × 10–5 m2 (d) 2.0 × 10–5 m2
6. A solid sphere of radius R and density r is attached to one
end of a mass-less spring of force constant k. The other end
of the spring is connected to another solid sphere of radius
R and density 3r. The complete arrangement is placed in a
liquid of density 2r and is allowed to reach equilibrium. The
(a) zero
correct statement(s) is (are) (JEE Adv. 2013)
(b) equal to the weight of the liquid displaced
(c) equal to the weight of the body in air 4pR3rg
(d) equal to the weight of the immersed portion of the body (a) The net elongation of the spring is
3k
2. The spring balance A reads 2 kg with a block m suspended
from it. A balance B reads 5 kg when a beaker with liquid is 8pR3rg
put on the pan of the balance. The two balances are now so (b) The net elongation of the spring is
3k
arranged that the hanging mass is inside the liquid in the
(c) The light sphere is partially submerged
beaker as shown in the figure. In this situation:
(d) The light sphere is completely submerged
(1985 - 2 Marks)
P-148 Topic-wise Solved Papers - PHYSICS
9. A container of large uniform cross-sectional area A resting
SUBJECTIVE PROBLEMS : on a horizontal surface, holds two immiscible, non-viscous
1. A column of mercury of 10 cm length is contained in the and incompressible liquids of densities d and 2d, each of
middle of a narrow horizontal 1 m long tube which is closed height H/2 as shown in the figure. The lower density liquid
at both the ends. Both the halves of the tube contain air at is open to the atmosphere having pressure P0 .
a pressure of 76 cm of mercury. By what distance will the (1995 - 5 + 5 Marks)
column of mercury be displaced if the tube is held vertically?
H/2 d
(1978)
2. A point mass m is suspended at the end of a massless wire
of length l and cross section A. If Y is the Young’s modulus H/2 2d h
for the wire, obtain the frequency of oscillation for the simple
harmonic motion along the vertical line. (1978) x
3. A cube of wood supporting 200 gm mass just floats in water. (a) A homogeneous solid cylinder of length L(L < H/2),
When the mass is removed, the cube ruses by 2cm. What is cross-sectional area A/5 is immersed such that it floats
the size of the cube? (1978) with its axis vertical at the liquid-liquid interface with
4. A beaker containing water is placed on the pan of balance length L/4 in the denser liquid. Determine:
which shows a reading of M gms. A lump of sugar of mass m (i) the density D of the solid and
gms and volume Vcc. is now suspended by a thread in such
(ii) the total pressure at the bottom of the container.
a way that it is completely immersed in water without
touching the beaker and without any overflow of water. (b) The cylinder is removed and the original arrangement
What will be the reading of the balance just when the lump is restored. A tiny hole of area s(s << A) is punched on
of sugar is immersed? How will the reading change as the the vertical side of the container at a height h(h < H/2).
time passes on? (1978) Determine :
5. A boat floating in a water tank is carrying a number of large (i) the initial speed of efflux of the liquid at the hole,
stones. If the stones are unloaded into water, what will (ii) the horizontal distance x travelled by the liquid
happen to the water level? (1979) initially, and
6. Two identical cylindrical vessels with their bases at the same (iii) the height hm at which the hole should be punched
level each contain a liquid of density r (rho). The height of so that the liquid travels the maximum distance xm
the liquid in one vessel is h1 and in other is h2. The area of initially. Also calculate xm.
either base is A. What is the work done by gravity in (Neglect the air resistance in these calculations.)
equalizing the levels when the two vessels are connected ? 10. A large open top container of negligible mass and uniform
(1981- 4 Marks) cross-sectional area A has a small holes of cross-sectional
7. A wooden plank of length 1 m and uniform cross-section is area A/100 in its side wall near the bottom. The container is
hinged at one end to the bottom of a tank as shown in fig kept on a smooth horizontal floor and contains a liquid of
The tank is filled with water upto a height 0.5 m. The specific density r and mass m0. Assuming that the liquid starts
flowing out horizontally through the hole at t = 0, Calculate
gravity of the plank is 0.5. Find the angle q that the plank
makes with the vertical in the equilibrium position. (Exclude (1997C - 5 Marks)
the case q = 0°) (1984- 8 Marks) (i) the acceleration of the container, and
(ii) its velocity when 75% of the liquid has drained out.
11. A non-viscous liquid of constant density 1000 kg/m3 flows
q in a streamline motion along a tube of variable cross section.
The tube is kept inclined in the vertical plane as shown in
8. A ball of density d is dropped on to a horizontal solid surface. Figure. The area of cross section of the tube two points P
It bounces elastically from the surface and returns to its and Q at heights of 2 metres and 5 metres are respectively
original position in a time t1. Next, the ball is released and it 4×10–3 m2 and 8×10–3 m2. The velocity of the liquid at point
falls through the same height before striking the surface of P is 1 m/s. Find the work done per unit volume by the
a liquid of density of dL (1992 - 8 Marks) pressure and the gravity forces as the fluid flows from point
P to Q. (1997 - 5 Marks)
(a) If d < dL, obtain an expression (in terms of d, t1 and dL)
for the time t2 the ball takes to come back to the position
from which it was released. Q
(b) Is the motion of the ball simple harmonic?
(c) If d = dL, how does the speed of the ball depend on its
depth inside the liquid? Neglect all frictional and other P 5m
dissipative forces. Assume the depth of the liquid to 2m
be large.
MECHANICAL PROPERTIES OF SOLIDS & FLUIDS P-149
12. A wooden stick of length L, radius R and density r has a 16. A tube has two area of cross-sections as shown in figure.
small metal piece of mass m (of negligible volume) attached to The diameters of the tube are 8 mm and 2 mm. Find range of
its one end. Find the minimum value for the mass m (in terms water falling on horizontal surface, if piston is moving with
of given parameters) that would make the stick float vertically a constant velocity of 0.25 m/s, h = 1.25 m (g = 10 m/s2)
in equilibrium in liquid of density s (> r).(1999 - 10 Marks) (2004 - 2 Marks)
13. A uniform solid cylinder of density 0.8 g/cm3 floats in
equilibrium in a combination of two non-mixing liquids A
and B with its axis vertical. The densities of the liquids A
and B are 0.7 g/cm3 and 1.2 g/cm3, respectively. The height h
of liquid A is hA = 1.2 cm. The length of the part of the
cylinder immersed in liquid B is hB = 0.8 cm.
(2002 - 5 Marks ) 17. A uniform wire having mass per unit length l is placed over
Air h a liquid surface. The wire causes the liquid to depress by
y(y << a) as shown in figure. Find surface tension of liquid.
Neglect end effect. (2004 - 2 Marks)
hA
A y

hB
B

a a
(a) Find the total force exerted by liquid A on the cylinder.
18. A U tube is rotated about one of it’s limbs with an angular
(b) Find h, the length of the part of the cylinder in air.
velocity w. Find the difference in height H of the liquid
(c) The cylinder is depressed in such a way that its top (density r) level, where diameter of the tube d << L.
surface is just below the upper surface of liquid A and is
(2005 - 2 Marks)
then released. Find the acceleration of the cylinder
immediately after it is released. w
14. A bubble having surface tension T and radius R is formed
on a ring of radius b (b << R). Air is blown inside the tube
with velocity v as shown. The air molecule collides
perpendicularly with the wall of the bubble and stops. H
Calculate the radius at which the bubble separates from the
ring. (2003 - 4 Marks)
b R

v
L

COMPREHENSION BASED Q UESTIONS :


15. Shown in the figure is a container whose top and bottom
diameters are D and d respectively. At the bottom of the PASSAGE-I
container, there is a capillary tube of outer radius b and A cylindrical tank has a hole of diameter 2r in its bottom. The hole
inner radius a. is covered wooden cylindrical block of diameter 4r, height h and
D density r/3.
P

h1 r/3
4r
h r
h2 h

d 2r
The volume flow rate in the capillary is Q. If the capillary is
removed the liquid comes out with a velocity of v0. The Situation I : Initially, the tank is filled with water of density r to a
density of the liquid is given as r. Calculate the coefficient height such that the height of water above the top of the block is
of viscosity h. (2003 - 4 Marks) h1 (measured from the top of the block).
P-150 Topic-wise Solved Papers - PHYSICS
Situation II : The water is removed from the tank to a height h2 6. After the drop detaches, its surface energy is (2010)
(measured from the bottom of the block), as shown in the figure.
(a) 1.4 ´ 10–6 J (b) 2.7 ´ 10 –6 J
The height h2 is smaller than h (height of the block) and thus the
block is exposed to the atmosphere. (c) (d)
5.4 ´ 10 –6 J 8.1 ´ 10 –6 J
1. Find the minimum value of height h1 (in situation 1), for
which the block just starts to move up? (2006 - 5M, –2) ASSERTION & REASON TYPE QUESTIONS :
2h 5h
(a) (b) 1. STATEMENT-1 : The stream of water flowing at high speed
3 4
from a garden hose pipe tends to spread like a fountain
5h 5h
(c) (d) when held vertically up, but tends to narrow down when
3 2
held vertically down.
2. Find the height of the water level h2 (in situation 2), for
which the block remains in its original position without the and
application of any external force (2006 - 5M, –2) STATEMENT-2 : In any steady flow of an incompressible
h 4h fluid, the volume flow rate of the fluid remains constant.
(a) (b) (2008)
3 9
2h (a) STATEMENT-1 is True, STATEMENT-2 is True;
(c) (d) h STATEMENT-2 is a correct explanation for STATE-
3
MENT-1
3. In situation 2, if h2 is further decreased, then
(2006 - 5M, –2) (b) STATEMENT-1 is True, STATEMENT-2 is True;
(a) cylinder will not move up and remains at its original STATEMENT-2 is NOT a correct explanation for
position STATEMENT-1
h (c) STATEMENT -1 is True, STATEMENT-2 is False
(b) for h2 = , cylinder again starts moving up (d) STATEMENT -1 is False, STATEMENT-2 is True
3
h INTEGER VALUE CORRECT TYPE :
(c) for h2 = , cylinder again starts moving up
4
1. Two soap bubbles A and B are kept in a closed chamber
h where the air is maintained at pressure 8 N/m2. The radii of
(d) for h2 = , cylinder again starts moving up
5 bubbles A and B are 2 cm and 4 cm, respectively. Surface
PASSAGE-II tension of the soap-water used to make bubbles is 0.04 N/m.
When liquid medicine of density r is to put in the eye, it is done Find the ratio nB/nA, where n A and nB are the number of
with the help of a dropper. As the bulb on the top of the dropper moles of air in bubbles A and B, respectively. [Neglect the
is pressed, a drop forms at the opening of the dropper. We wish to effect of gravity.] (2009)
estimate the size of the drop. We first assume that the drop formed 2. A cylindrical vessel of height 500 mm has an orifice (small
at the opening is spherical because that requires a minimum hole) at its bottom. The orifice is initially closed and water is
increase in its surface energy. To determine the size, we calculate
filled in it up to height H. Now the top is completely sealed
the net vertical force due to the surface tension T when the radius
of the drop is R. When this force becomes smaller than the weight with a cap and the orifice at the bottom is opened. Some
of the drop, the drop gets detached from the dropper. (2010) water comes out from the orifice and the water level in the
4. If the radius of the opening of the dropper is r, the vertical vessel becomes steady with height of water column being
force due to the surface tension on the drop of radius R 200 mm. Find the fall in height (in mm) of water level due to
(assuming r << R) is opening of the orifice.
(a) 2p rT (b) 2p RT [Take atmospheric pressure = 1.0 × 105 N/m2, density of
water = 1000 kg/m3\ and g = 10 m/s2. Neglect any effect of
2pr 2T 2pR 2T surface tension.] (2009)
(c) (d)
R r 3. A 0.1 kg mass is suspended from a wire of negligible mass.
The length of the wire is 1m and its crosssectional area is
5. If r = 5 ´ 10–4 m, r = 103 kgm –3 , g = 10 ms –2 , T = 0.11Nm –1 ,
4.9 × 10–7 m2. If the mass is pulled a little in the vertically
the radius of the drop when it detaches from the dropper is
downward direction and released, it performs simple
approximately (2010)
harmonic motion of angular frequency 140 rad s–1. If the
(a) 1.4 ´ 10 –3 m (b) 3.3 ´ 10 –3 m Young’s modulus of the material of the wire is n × 109 Nm–2,
(c) 2.0 ´ 10 –3 m (d) 4.1 ´ 10–3 m the value of n is (2010)
MECHANICAL PROPERTIES OF SOLIDS & FLUIDS P-151

1. A spring of force constant 800 N/m has an extension of 5


cm. The work done in extending it from 5 cm to 15 cm is Vg (r1 – r2 ) Vgr1
(a) (b)
(a) 16 J (b) 8 J [2002] k k
(c) 32 J (d) 24 J
2. A wire fixed at the upper end stretches by length l by Vg r1 Vg (r1 – r2 )
(c) (d)
applying a force F. The work done in stretching is [2004] k k
(a) 2Fl (b) Fl 10. A jar is filled with two non-mixing liquids 1 and 2 having
F Fl densities r1 and, r2 respectively. A solid ball, made of a
(c) (d)
2l 2 material of density r3 , is dropped in the jar. It comes to
3. Spherical balls of radius ‘R’ are falling in a viscous fluid of equilibrium in the position shown in the figure.Which of
viscosity ‘h’ with a velocity ‘v’. The retarding viscous force the following is true for r1, r1and r3? [2008]
acting on the spherical ball is [2004]
(a) inversely proportional to both radius ‘R’ and velocity r1

‘v’
(b) directly proportional to both radius ‘R’ and velocity ‘v’ r3
(c) directly proportional to ‘R’ but inversely proportional
to ‘v’
(d) inversely proportional to ‘R’ but directly proportional
to velocity ‘v’
4. If two soap bubbles of different radii are connected by a (a) r3 < r1 < r2 (b) r1 > r3 > r2
tube. [2004] (c) r1 < r2 < r3 (d) r1 < r3 < r2
(a) air flows from the smaller bubble to the bigger 11. A capillary tube (A) is dipped in water. Another identical
(b) air flows from bigger bubble to the smaller bubble till tube (B) is dipped in a soap-water solution. Which of the
the sizes are interchanged following shows the relative nature of the liquid columns in
(c) air flows from the bigger bubble to the smaller bubble the two tubes? [2008]
till the sizes become equal
(d) there is no flow of air. A B
5. If ‘S’ is stress and ‘Y’ is young’s modulus of material of a
wire, the energy stored in the wire per unit volume is
2
S
(a) (b) 2S 2Y [2005] (a)
2Y
S 2Y
(c) (d)
2Y S2 A B
6. A 20 cm long capillary tube is dipped in water. The water
rises up to 8 cm. If the entire arrangement is put in a freely
falling elevator the length of water column in the capillary
tube will be [2005] (b)
(a) 10 cm (b) 8 cm
(c) 20 cm (d) 4 cm
7. A wire elongates by l mm when a load W is hanged from it. If
the wire goes over a pulley and two weights W each are A B
hung at the two ends, the elongation of the wire will be (in
mm) [2006]
(a) l (b) 2l
(c) zero (d) l/2 (c)
8. If the terminal speed of a sphere of gold (density = 19.5 kg/m3)
is 0.2 m/s in a viscous liquid (density = 1.5 kg/m3), find the
terminal speed of a sphere of silver (density = 10.5 kg/m3) of
the same size in the same liquid [2006]
(a) 0.4 m/s (b) 0.133 m/s A B
(c) 0.1 m/s (d) 0.2 m/s
9. A spherical solid ball of volume V is made of a material of
density r1. It is falling through a liquid of density r1 (r2< r1). (d)
Assume that the liquid applies a viscous force on the ball
that is proportional to the square of its speed v, i.e., Fviscous
= –kv2 (k > 0). The terminal speed of the ball is [2008]
P-152 Topic-wise Solved Papers - PHYSICS
12. Two wires are made of the same material and have the same 18. Two mercury drops (each of radius ‘r’) merge to form bigger
volume. However wire 1 has cross-sectional area A and wire drop. The surface energy of the bigger drop, if T is the
2 has cross-sectional area 3A. If the length of wire 1 increases surface tension, is : [2011RS]
by Dx on applying force F, how much force is needed to
stretch wire 2 by the same amount? [2009] (a) 4 pr 2T (b) 2pr 2T
(a) 4 F (b) 6 F
(c) 28 / 3 pr 2T (d) 25/ 3 pr 2T
(c) 9 F (d) F
19. If a ball of steel (density r = 7.8 g cm–3) attains a terminal
13. A ball is made of a material of density r where
velocity of 10 cm s–1 when falling in water (Coefficient of
roil < r < rwater with roil and rwater representing the
viscosity hwater = 8.5 × 10–4 Pa.s), then, its terminal velocity
densities of oil and water, respectively. The oil and water are
immiscible. If the above ball is in equilibrium in a mixture of in glycerine (r = 1.2 g cm–3, h = 13.2 Pa.s) would be, nearly
this oil and water, which of the following pictures represents [2011RS]
its equilibrium position ? [2010] (a) 6.25 × 10–4 cm s–1 (b) 6.45 × 10–4 cm s–1
(c) 1.5 × 10–5 cm s–1 (d) 1.6 × 10–5 cm s–1
20. If a spring of stiffness ‘k’ is cut into parts ‘A’ and ‘B’ of
length l A : l B = 2 : 3, then the stiffness of spring ‘A’ is
water oil given by [2011RS]
3k 2k
(a) (b)
(a) (b) 5 5
oil 5
water (c) k (d)
2k
21. A thin liquid film formed between a U-shaped wire and a light
slider supports a weight of 1.5 × 10–2 N (see figure). The
length of the slider is 30 cm and its weight negligible. The
surface tension of the liquid film is
water oil [2012]

(c) (d)
oil
water FILM

14. Two identical charged spheres are suspended by strings of


equal lengths. The strings make an angle of 30° with each
other. When suspended in a liquid of density 0.8g cm–3, the
angle remains the same. If density of the material of the W
sphere is 1.6 g cm–3 , the dielectric constant of the liquid is
(a) 0.0125 Nm–1 (b) 0.1 Nm–1
[2010]
(c) 0.05 Nm–1 (d) 0.025 Nm–1
(a) 4 (b) 3
22. A uniform cylinder of length L and mass M having cross-
(c) 2 (d) 1
15. Work done in increasing the size of a soap bubble from a sectional area A is suspended, with its length vertical, from
radius of 3 cm to 5 cm is nearly (Surface tension of soap a fixed point by a massless spring such that it is half
solution = 0.03 Nm–1) [2011] submerged in a liquid of density s at equilibrium position.
(a) 0.2 p mJ (b) 2p mJ The extension x0 of the spring when it is in equilibrium is:
(c) 0.4p mJ (d) 4p mJ [ JEE Main 2013 ]
16. Water is flowing continuously from a tap having an internal Mg Mg æ LAs ö
diameter 8 × 10–3 m. The water velocity as it leaves the tap is (a) (b) ç1 – ÷
k k è M ø
0.4 ms–1. The diameter of the water stream at a distance
2 × 10–1 m below the tap is close to: [2011] Mg æ LAs ö Mg æ LAs ö
(a) 7.5 × 10–3 m (b) 9.6 × 10–3 m (c) ç1 – ÷ (d) ç1 + ÷
(c) 3.6 × 10–3 m (d) 5.0 × 10–3 m k è 2M ø k è M ø
17. A metal rod of Young’s modulus Y and coefficient of thermal 23. Assume that a drop of liquid evaporates by decrease in its
expansion a is held at its two ends such that its length surface energy, so that its temperature remains
remains invariant. If its temperature is raised by t°C, the unchanged.What should be the minimum radius of the drop
linear stress developed in it is [2011RS] for this to be possible? The surface tension is T, density of
Y liquid is r and L is its latent heat of vaporization.
(a) (b) Y at [JEE Main 2013 ]
at
1 at (a) rL/T (b) T / rL
(c) (d)
(Y at ) Y (c) T/rL (d) 2T/rL
MECHANICAL PROPERTIES OF SOLIDS & FLUIDS P-153

Solutions & Explanations


Section-A : JEE Advanced/ IIT-JEE

YAx 2 Mg
A 1. 2. 3. ( g 2 - g 1 ) DT 4. 500 Pa.
2L 3 Ak
B 1. F 2. F 3. F 4. F
C 1. (c) 2. (a) 3. (b) 4. (a) 5. (a) 6. (d)
7. (d) 8. (a) 9. (a) 10. (a) 11. (b) 12. (a) 13. (c)
D 1. (a) 2. (b, c) 3. (c) 4. (c) 5. (c) 6. (a, d)

1 YA
E 1. 2.95 cm 2. 3. l = 10 cm 4. Mg + Vdwg; Increasing 5. Fall
2p mL

rAg d L t1
6. (h1 - h2 )2 7. 45° 8. (a) d - d (b) no (c) remains same
4 L

5d æ 3H L ö 3H - 4h 3H 3H
9. (a) (i) (ii) P0 + ç + ÷ dg (b) (i) g (ii) (3H - 4h)h (iii) ,
4 è 2 4ø 2 8 4

g gm0
10. (i) (ii) 11. 29.025 × 103 J/m3 ; 29.4 × 103 J/m3 12. pR 2 L( rs - r)
50 2 Ar

4T p 1 2 é d4 ù 4
13. (a) zero (b) 0.25 cm (c) g/6, ­ 14. 15. 8Ql ´ 2 rv0 ê1 - 4 ú ´ a 16. 2m
rv 2 ëê D ûú

lag w 2 L2
17. 18. H =
2y 2g
G 1. (c) 2. (b) 3. (a) 4. (c) 5. (a) 6. (b)
H 1. (a)
I 1. 6 2. 6 3. 4

Section-B : JEE Main/ AIEEE

1. (b) 2. (d) 3. (b) 4. (a) 5. (a) 6. (c) 7. (a) 8. (c)


9. (a) 10. (d) 11. (c) 12. (c) 13. (b) 14. (c) 15. (c) 16. (c)
17. (b) 18. (c) 19. (a) 20. (d) 21. (d) 22. (c) 23. (d)

(V f - Vi ) Mg Vi - V f Mg
FILL IN THE BLANKS : Þ - = Þ =
Vi AK Vi AK
1 2
1. W = ´ Y ´ (strain) ´ Yd 4 3 4
2 pR - p( R - dR )3
3 3 Mg
1 x2 YAx 2 Þ =
= ´ Y ´ 2 ´ AL = 4 3 AK
pR
2 L 2L 3
-DP R3 - [ R3 - 3R 2dR] Mg
2. K= Þ =
DV / V
R3 AK
Mg DV Mg
where DP= \ - = dR Mg
A V AK Þ =
R 3 AK
P-154 Topic-wise Solved Papers - PHYSICS
3. KEY CONCEPT
Using the relation for floatation, vdHgg = Vdmg TRUE / FALSE :
Fraction of volume of metal submerged in mercury 1. When the man drinks some water from the pond, his weight
v dm increases and therefore the boat will sink further. The further
= = = K1 (say) sinking of the boat will displace the same volume of water in
V d Hg
pond as drunk by man. Therefore, there will no change in
the level of water in the pond.
vdHgg v'd'Hgg
2. Pressure P1= P2 = 1 atm = hrg
V V' Note :
v v' On changing the temperature, g will not change and
atmospheric pressure will not change.
\ h × r = constant.
Vdmg V'd'mg When temperature is increased, the density of Hg decreases
and hence, h increases.
T T +DT
In second case, when temperature is increased by DT.
v'd'Hgg = V'd'm g
h
v' d'
Þ = m = Fraction of volume of metal submerged P1 P2
V ' d 'Hg
in mercury = K2 (say)
K 2 d 'm ´ d Hg d 'm ´ d 'Hg (1 + g 2 DT ) (1 + g 2 DT ) 3. When water is heated at end x, the density decreases and
\ = = =
K1 d 'Hg ´ d m d 'Hg ´ d 'm (1 + g 1DT ) (1 + g 1DT ) the water moves up. This is compensated by the movement
of water from Y to X i.e., in clockwise direction.
= (1 + g2 DT) (1 + g1 DT)–1
= (1 + g2 DT) (1 – g1 DT)
= 1 + ( g – g ) DT
2 1
Note :
If g2 – g1 then k2 > k1
i.e., metal block will get immersed deeper
If g2 < g1 then k2 < k1
i.e. metal block will rise a bit as compared to its previous
position.
K2 K 2 - K1
- 1 = ( g 2 - g 1 ) DT Þ = ( g 2 - g 1 ) DT X Y
K1 K1
4. KEY CONCEPT
Applying equation of continuity at cross section 1 and 2 4. When the block of ice melts, the lead shot will ultimately
A1v1 = A2v2 Þ 10 × 1 = 5 × v2 sink in the water. When lead shot sinks, it will displace water
Þ v2 = 2m/s equal to its own volume. But when lead shot was embedded
in ice, it displaced more volume of water than its own volume
P1=2000Pa because dlead > dwater. Therefore, level of water will fall.
P2
MCQ's WITH ONE CORRECT ANSWER :
–1
v1=1ms v2
A2=5cm2 1. (c)
A1 =10cm2
A (dx)
D
Applying Bernoulli's theorem
1 1
P1 + rv12 = P2 + rv22
2 2
1 1
Þ 2000 +´ 1000 ´ 12 = P2 + ´ 1000 ´ 22 B C
2 2
Þ P2 = 500 Pa Let us consider a small dotted segment of thickness dx
for observation.
MECHANICAL PROPERTIES OF SOLIDS & FLUIDS P-155
Since, this segment is accelerated towards right, a net 7. (d) l decreases as the block moves up. h will also
force is acting in this segment towards right from the decreases because when the coin is in water it will
liquid towards the left of ABCD. According to Newton's displace a volume of water, equal to its own volume,
third law, the segment ABCD will also apply a force on whereas when it is on the block it displaces more volume
the previous section creating a pressure on it which than to own volume (because density of coin is greater
makes the liquid rise. than density of water).
F Dl F l
T/A T ´l T l 8. (a) Y = = .
2. (a) Y = Þ Dl = = ´ A l A Dl
Dl / l A´Y Y A
20 ´ 1
T l = = 2 ´ 1011 N / m 2 .
Here, is constant. Therefore, Dl µ . 10 -6 ´ 10 -4
Y A
9. (a) The square of the velocity of efflux
l 2 gh
is largest in the first case. v2 =
A 2
3. (b) Pressure in limb I at B = Pressure in limb II at A æ aö
1- ç ÷
è Aø
2 ´ 10 ´ 2.475
or, v2 = = 50 m2/s2
h P1 II h P2 I 1 - (0.1) 2
A B
A
2.475 = h

3m
hr1g = hr2g Þ r1 = r2 v
AD
4. (a) Weight of cylinder = Upthrust due to upper liquid + 0.525 m
Upthrust due to lower liquid.
æA ö æ Aö æ 3 ö æ Aö æ L ö h = 3 - 0.525 = 2.475 m
D ç ´ L ´ g ÷ = d ç ÷ ç L ÷ g + 2d ç ÷ ç ÷ ´ g
è5 ø è 5ø è 4 ø è 5 ø è 4ø
Dp (1.165 ´ 105 - 1.01 ´ 105 )
10. (a) B = =
5d DV / V 0.1
\ D= = 1.55 × 105 Pa
4
5. (a) Equating the rate of flow, we have 11. (b) We know that excess pressure in a soap bubble is
inversely proportional to its radius. The soap bubble
(2 gy ) ´ L2 = (2 g ´ 4 y ) pR 2 at end 1 has small radius as compared to the soap bubble
at end 2 (given). Therefore excess pressure at 1 is more.
[Flow = (area) × (velocity), velocity = 2gx ]
where x = height from top
L
Þ L2 = 2pR2 Þ R = 2 1
2p
6. (d) KEY CONCEPT : As the value is opened, air flows from end 1 to end 2
According to Archimedes principle and the volume of soap bubble at end 1 decreases.
Upthrust = Wt. of fluid displaced 12. (a) Let V be the volume of the material of which the cylinder
is made. The cylinder is half immersed in water.
Therefore the volume of water displaced because of
V
h the material of the cylinder is . Let
P1 2
h be the total height of the cylinder. As the cylinder is
V half submerged therefore buoyant force
A

Fbottom – Ftop = Vrg h


V rw g hArw g
\ Fbottom = Ftop + Vrg B= + h ' h

= P1 × A + Vrg
2 2 2
= (hrg) × (pR2) + Vrg
= rg [pR2 h + V] where A is the area of cross-section of the cylinder
P-156 Topic-wise Solved Papers - PHYSICS
The weight of the cylinder W = Vrc g dHg + doil 13.6 + 0.8
The weight of the water inside the cylinder Þ d= = = 7.2g / cm3
= h'Arw g 2 2
For equilibrium, Dl
4. (c) D l = l a DT Þ = a DT
V rw g Ahrw g l
+ = V rc g + h ' Arw g Stress = Y × strain
2 2
Here rw = 1 \ Stress = Ya DT
For first rod stress = Y1a1D T
V h For second rod stress = Y2a2D T
\ + A = V rc + h ' A
2 2 Since, stresses are equal
\ V + hA = 2Vrc + 2h'A \ Y1a1D T = Y2a2D T
\ V [1 – 2rc] + hA = 2h'A Y1 a 2 3
or, = =
h V Y2 a1 2
\ h' = + [1 - 2rc ]
2 2A 5. (c) KEY CONCEPT :
h The equation of continuity is : v1A1 = v2A2
If rc < 0.5 then h ' > where v and A represent the speed of water stream and
2
its area of cross section, respectively. We are given
h that
and if rc > 0.5 then h ' < v1 = 1.0 ms–1
2
A1 = 10–4 m2
h v2 = velocity of water stream at 0.15 m below the tap
if rc = 0, h ' =
2 A2 = ?
For calculating v2
F / p (2R) 2 F / p R 2 u = 1 m/s; s = 1.5 m, a = 10 m/s2 and v = ?
13. (c) y= =
Dl1 / 2L Dl 2 / L v2 – u2 = 2as
v2 – 1 = 2 × 10 × 0.15 Þ v = 2 m/s
Dl 2
\ =2 v1 A1 1 ´ 10 -4
Dl1 Hence, A2 = = = 5 ´ 10 -5 m2
v2 2
option (c) is correct
6. (a, d)
Consider the equilibrium of the system of both spheres
and the spring.
1. (a) The whole system falls freely under gravity
Upthrust = weight of fluid displaced
= (mass of fluid displaced) × g r
For a freely falling body, g = 0
\ Upthrust = 0.
2. (b,c) When the block of mass m is arranged as shown in the 2r
figure, an upthrust FT will act on the mass which will
decrease the reading on A.
Note :
According to Newton's third law, to each and every
action, there is equal and opposite reaction. 3r
So FT will act on the liquid of the beaker which will
increase the reading in B.
4 3 4
The weight of system = pR (3r )g + pR 3r g
3 3
A
é4 ù
= 4 ê p R 3 rg ú
ë3 û
This is to be balanced by the buoyant force.
FT FT This can be possible only when the light sphere is
completely submerged. In this way the buoyant force
B
Fs B
3. (c) Weight of sphere
= Upthrust due to Hg + Upthrust due to oil
V V
Vdg = d Hg g + doil ´ g
2 2 W
MECHANICAL PROPERTIES OF SOLIDS & FLUIDS P-157
éæ 4 ö ù é4 ù From fig. (b), due to equilibrium
B = êç p R 3 ÷ ´ 2ú ´ (2r) ´ g = 4 ê p R 3 rg ú T = mg ... (i)
ë è 3 ø û ë 3 û
Now considering the equilibrium of the heavy sphere T/A
But Y =
Fs + B = W l/ L
\ Fs = W – B YAl
4 3 4 3 Þ T= ... (ii)
\ Kx = pR (3r )g - pR (2r )g L
3 3 From (i) and (ii)
4 R 3 rg YAl
\x = p
3 K mg = ... (iii)
L
(a) and (d) are correct options. From fig. (c)
SUBJECTIVE PROBLEMS : Restoring force
1. M is the mid-point of tube AB. é YA(l + x ) YA l ù
= – [T' – mg] = – ê - [from (iii)]
At equilibrium ë L L úû
p1× A + mg = p2 × A
p1 × A + 10 × A × dHgg = p2 × A -YAx
Þ p1 + 10dHg × g = p2 ... (i) =
L
A A On comparing this equation with F = – mw2x, we get
YA
45 cm
mw2 =
L
YA 2p YA
p1 P Þ w= Þ =
x 10 cm mL T mL
P'
M
M 1 1 YA
10 cm
Frequency f = =
T 2p mL
p2 Q' Q
3. Let the edge of cube be l. When mass is on the cube of
mg wood
45 cm
200 g + l3 d wood g = l3d water g
B B Þ l3 d wood = l3 d water - 200 ... (i)
For air present in column AP When the mass is removed
p × 45 × A = p1 × (45 + x) × A
45 l3 d wood = (l - 2) l2 d water ... (ii)
Þ p1 = ´ 76d Hg ´ g ... (ii) From (i) and (ii)
45 + x
For air present in column QB l3 d water - 200 = (l - 2) l 2 d water
p × 45 × A = p2 × (45 – x) × A
45 But d water = 1
Þ p2 = ´ 76dHg ´ g ... (iii)
45 - x \ l3 - 200 = l 2 (l - 2)
From (i), (ii) and (iii)
45 ´ 76 ´ d Hg g Þ l = 10 cm
45
+ 10 d Hg ´ g = ´ 76 ´ d Hg ´ g 4. When the lump of sugar is just immersed
45 + x 45 - x
T = Mg – B
45 ´ 76 45 ´ 76
Þ + 10 =
45 + x 45 - x
x = 2.95 cm.
2.
T
L

Mg
T B
l B
x T'
mg
mg
Fig. (a) Fig. (b) Fig. (c)
(For equilibrium of lump of sugar)
P-158 Topic-wise Solved Papers - PHYSICS
The reading on the pan balance Therefore,
= Mg + Vdw g
where V = Volume of lump of sugar é h 2 h2 æ h + h ö 2 ù
dw = density of water DU = rAg ê 1 + 2 - ç 1 2 ÷ ú
êë 2 2 è 2 ø ú
When the lump is half dissolved, û
V V
The reading on the pan balance = Mg + ds g + dw g rAg
2 2 = (h1 - h2 )2
When ds= density of sugar. 4
Since ds > dw This change in P.E. is the work done by gravity.
\ the reading will increase. 7. (a) For equilibrium Fnet = 0 and tnet = 0
Thus, we can conclude that as the time passes the reading
will keep increasing.
5. KEY CONCEPT :
When the stones were in the boat, the weight of stones x
were balanced by the buoyant force.
F1 q
Vsds = Vl dl

1m
0.5m
Vl , Vs = volume of liquid and stone respectively mg
q
dl , ds = density of liquid and stone respectively
Since, ds > dl \ Vs < Vl O
Therefore when stones are put in water, the level of water
falls. Taking moment about O
6. P.E. of liquid in cylinder 1
l æ l - xö
mg ´ sin q = FT ç sin q ... (i)
h h1 rAgh12 2 è 2 ÷ø
U1 = (m) g 1 = (r × A × h1) g =
2 2 2
Note : Also FT = wt. of fluid displaced = [ (l - x) A] ´ rw g... (ii)
[The total mass can be supposed to be concentrated at the
And m = (l A) 0.5rw ... (iii)
center of the filled part which will be at height h1/2]
Where A is the area of cross section of the rod.
rAgh22
Similarly P.E. of liquid in cylinder 2 U2 = From (i), (ii) and (iii)
2
rAg 2 l æ l - xö
\ Total P.E. initially U = U1 + U2 = (h1 + h22 ) (l A) 0.5rw g ´ sin q = [(l - x ) A]rw g ´ ç sin q
2 2 è 2 ÷ø
After the equalising of levels.
Here, l = 1 m
h rAg 2
P.E. of liquid in cylinder 1 U1' = mg = h \ (1 – x)2 = 0.5 Þ x = 0.293 m
2 2
From the diagram
1 2 1 2
0.5 0.5
cos q = = Þ q = 45°
1 - x 0.707

h1
8. (a) Let the ball be dropped from a height h. During fall
h t1 2v
h2 V = ut + at = 0 + g Þ t1 =
2 g
h rAg 2 In the second case the ball is made to fall through the
P.E. of liquid in cylinder 2 U2' = mg = h same height and then the ball strikes the surface of
2 2
\ Total P.E. finally U' = U1' + U2' = rAgh2 liquid of density dL. When the ball reaches inside the
The change in P.E. liquid, it is under the influence of two force (i) Vdg, the
é h2 h2 ù weight of ball in downward direction (ii) VdLg, the
DU = U – U' = rAg ê 1 + 2 - h2 ú upthrust in upward direction.
ëê 2 2 ûú Note :
Total volume remains the same.
The viscous forces are absent. (given)
Ah1 + Ah2 = Ah + Ah
Since, dL > d
h +h
Þ h= 1 2 the upward force is greater and the ball starts retarding.
2
MECHANICAL PROPERTIES OF SOLIDS & FLUIDS P-159
A
A A L A 3L
h ´ L ´ D ´ g = ´ ´ 2d ´ g + ´ ´d´g
5 5 4 5 4
2d 3d 5d
Þ D= + =
4 4 4
B
u=V (ii) Total pressure at the bottom of the cylinder =
VdLg Atmospheric pressure + Pressure due to liquid of
density d + Pressure due to liquid of density 2d +
a=?
Pressure due to cylinder [Weight/Area]
t=t
Vdg dL A
´L´D´g
H H
v=0 C P = P0 + dg + ´ 2d ´ g + 5
2 2 A
For motion B to C
u = V , v = 0, t = t, a = – a æ 3H L ö é 5d ù
Þ P = P0 + ç + ÷ dg êQ D =
v = u + at Þ 0 = v + (- a ) t è 2 4 ø ë 4 úû
v (b) KEY CONCEPT :
Þ t= Applying Bernoulli's theorem
a
Fnet Vd L g - Vdg (d L - d ) g éH æH ö ù
Now, a = = = P0 + ê ´ d ´ g + ç - h÷ 2d ´ g ú
m Vd d ë 2 è 2 ø û
vd 1
Þ t= ... (iii) = P0 + (2d )v 2
(d L - d ) g 2
Therefore,
(3H - 4h)
2 dv Þ v= g
t2 = t1 + 2t = t1 + 2
(d L - d ) g
Horizontal Distance x
2d t1 g é d ù ux = v; t = t; x = vt .... (i)
= t1 + = t1 ê1 + ú For vertical motion of liquid falling from hole
(d L - d ) g 2 ë dL - d û uy = 0, Sy = h, ay = g, ty = t
d t 1 2
Þ t2 = L 1 S= ut + at
dL - d 2
(b) Since the retardation is not proportional to
displacement, the motion of the ball is not simple 1 2 2h
harmonic. Þ h= gt Þ t = ... (ii)
2 g
(c) If d = dL then the retardation a = 0. Since the ball strikes
the water surface with some velocity, it will continue Po
with the same velocity in downward direction (until it
is interrupted by some other force).
9. (a) (i) H
KEY CONCEPT : 2
Since the cylinder is in equilibrium in the liquid therefore H -h
2
Weight of cylinder = upthrust v Po
mg = FT1 + FT2 where Reference h
level for P.E
x
FT1 and FT2 = upthrust due to lower and upper liquid
respectively From (i) and (ii)

2h g 2h
x = vy × = (3H - 4h) ´
g 2 g
d A/5
F H/2
D
3L = (3H - 4h) h ... (iii)
4
mg For finding the value of h for which x is maximum, we
L
4 H/2
differentiate equation (iii) w.r.t. t
2d
A dx 1
= [3H – 4h)h]–1/2 {3H – 8h}
dt 2
P-160 Topic-wise Solved Papers - PHYSICS
dx m0
Putting = 0 for finding values h for maxima/minima m0 = Ah × r Þ h =
dt Ar
1
[(3H – 4h)] –1/2 [3H – 8h] = 0 m0 gm0
2 \ v' = 2 g ´ 0.25 ´ =
Ar 2 Ar
3H
Þ h= 11. Given that
8
r = 1000 kg/m3, h1 = 2m, h2 = 5 m
é æ 3H ö ù 3H A1 = 4 × 10–3m2, A2 = 8 × 10–3 m2, v1 = 1 m/s
\ xm = ê3H - 4 çè 8 ÷ø ú 8 [From (iii)] Equation of continuity
ë û
A1v1
12H 3H 6H 3H A1 v1 = A2 v2 \ v2 = = 0.5 m/s
= ´ = = A2
8 8 8 4
According to Bernoulli's theorem,
10. (i) Let at any instant of time during the flow, the height of
liquid in the container is x. 1
(p1 – p2) = rg (h2 – h1) – r (v22 - v12 )
The velocity of flow of liquid through small hole in the 2
orifice by Toricelli's theorem is Where (p1 – p2) = work done/vol. [by the pressure]
v= 2gx ... (i) rg (h2 – h1) = work done/vol. [by gravity forces]
The mass of liquid flowing per second through the Now, work done/vol. by gravity forces
orifice = rg (h2 – h1) = 103 × 9.8 × 3 = 29.4 × 103 J/m3.
= r × volume of liquid flowing per second 1 1 é1 ù 3
And r (v22 - v12 ) = ´ 103 ê - 1ú = - ´ 103 J/m3
dm A 2 2 ë4 û 8
= r ´ 2 gx ´ ... (ii)
dt 100 = – 0.375 × 103 J/m3
Therefore, the rate of change of momentum of the \ Work done / vol. by pressure
system in forward direction = 29.4 × 103 – 0.375 × 103 J/m3 = 29.025 × 103 J/m3.
dm 2 gx ´ A ´ r 12. Note :
= ´v = (from (i) and (ii))
dt 100 For the wooden stick-mass system to be in stable equilibrium
(Alternatively you may use F = rav2) the center of gravity of stick-mass system should be lower
The rate of change of momentum of the system in the than the center of buoyancy. Also in equilibrium the centre
backward direction of gravity (N) and the centre of buoyancy (B) lie on the
= Force on backward direction = m × a same vertical axis.
Where m is mass of liquid in the container at the instant t The above condition 1 will be satisfied if the mass is towards
the lower side of the stick as shown in the figure.
m = volume × density = A × x × r
\ The rate of change of momentum of the system in the The two forces will create a torque which will bring the
backward direction stick-mass system in the vertical position of the stable
equilibrium.
= Axr × a
By conservation of linear momentum Let l be the length of the stick immersed in the liquid.

2 gxAr g
Axr × a = Þ a= FB = p R2 hs g
100 50

C
x P q
l/2 B
N
q L/2 (p R2 Lr) g
v
O
(ii) By Toricelli's theorem
mg
v' = 2 g ´ (0.25h)
Where h is the initial height of the liquid in the l
container. m0, the initial mass is Then OB = .
2
MECHANICAL PROPERTIES OF SOLIDS & FLUIDS P-161
For vertical equilibrium But according to equation of continuity
FG = FB Þ (M + m) g = FB A2 v0
Þ p R2 L r g + mg = p R2 l s g v1 =
A1
pR2 Lr + m
l= ... (1)
1 é 2 æ A2 ö ù
2
pR2 s r ê v - v ú
\ P + rgH = 0 ç 0÷
Let the distance of the center of mass N of the (rod + mass) 2 ê è A1 ø ú
system from the origin O be ON = y. Then ë û
My1 + my2
1 2 é æ A2 ö ù
2
y=
M +m P + rgH = r v0
ê1 - ç ÷ ú
Since mass m is at O, the origin, therefore y2 = 0 2 ê è A1 ø ú
ë û
M ( L / 2) + m ´ 0 ML Here, P + rgH = DP
\ y= =
M +m 2 ( M + m) According to Poisseuille's equation

( p R 2 L r) L p( DP)a 4 p( DP)a 4
= ... (2) Q= Þ h=
2 (p R 2 L r + m) 8hl 8Ql
Therefore for stable equilibrium é æ A ö2ù
p ( P + rgH ) a 4 p 1
l \ h= = ´ rv02 ê1 - ç 2 ÷ ú ´ a4
>y 8Ql 8Ql 2 ê è A1 ø ú
2 ë û
p R2 L r + m ( p R 2 L r) L A2 d 2
\ > Where =
2 (p R 2 L s) 2 (p R 2 L r + m) A1 D 2

Þ m ³ p R 2 L ( rs - r) p 1 2 é d4 ù 4
h= ´ rv0 ê1 - 4 ú ´ a
\ Minimum value of m is p R 2 L ( rs - r) 8Ql 2 êë D úû
13. (a) As the pressure exerted by liquid A on the cylinder is 16. From law of continuity
radial and symmetric, the force due to this pressure A1 v1 = A2 v2
cancels out and the net value is zero. Given A1 = p × (4 × 10–3 m)2, A2 = p × (1× 10–3 m)2
(b) For equilibrium, Buoyant force = weight of the body
Þ hArAAg + hBrB Ag = (hA + h + hB) A rCg v1 A2
A1 v2
(where rC = density of cylinder)
æ hAr A + hB rB ö
h= ç
è rC ÷ø - (hA + hB ) = 0.25cm h

FBuoyant - Mg
(c) a = x
M
v1 = 0.25 m/s
é hAr A + rB (h + hB ) - (h + hA + hB ) rC ù
=ê ú g p ´ (4 ´ 10 -3 ) 2 ´ 0.25
ë rC (h + hA + hC ) û \ v2 = 2 = 4 m/s
p ´ (1 ´ 10-3 )
g
= upwards
6 1 2 2h
14. KEY CONCEPT : Also, h = gt Þ t =
2 g
When the force due to excess pressure in the bubble equals
the force of air striking at the bubble, the bubble will detach 2h 2 ´ 1.25
from the ring. Horizontal range x = v2 × t = v2 = 4´ = 2m
g 10
4T 4T
\ rAv2 = ´A Þ R= 17. The free body diagram of wire is given below. If l is the
R rv 2 length of wire, then for equilibrium 2F sin q = W.
15. KEY CONCEPT :
When the tube is not there, using Bernoulli's theorem F F
q
y
1 1
P + P0 + rv12 + rgH = rv02 + P0
2 2 q q
1
Þ P + rgH = r(v02 - v12 ) W
2
P-162 Topic-wise Solved Papers - PHYSICS
F=S× l COMPREHENSION BASED Q UESTIONS :
or, 2S × l × sin q = l × l × g
1. (c) Consider the equilibrium of wooden block.
lg Forces acting in the downward direction are
or, S=
2 sin q
P1 ´ p(2r )2
lg alg é yù
\ S=
2y / a
=
2y êëQ sin q = a úû

d
18. w

P1 P0 W P2
H
(a) Weight of wooden cylinder
2 r hr
dx = p (2r ) ´ h ´ ´ g = p ´ 4r 2 g
x 3 3
(b) Force due to pressure (P1) created by liquid of height
A dm B h1 above the wooden block is
L
= P1 × p (2r)2 = [P0 + h1rg] × p (2r)2
Weight of liquid of height H Force acting on the upward direction due to pressure
P 2 exerted from below the wooden block and
pd 2 atmospheric pressure is
= ´ H ´ r´ g ... (i)
4 = P2 ´ p é(2r )2 - r 2 ù + P0 ´ p (r ) 2
ë û
Let us consider a mass dm situated at a distance x from A as
shown in the figure. The centripetal force required for the = [ P0 + ( h1 + h )rg ] ´ p ´ 3r 2 + P0 pr 2
mass to rotate = (dm) xw2 At the verge of rising
\ The total centripetal force required for the mass of
length L to rotate [ P0 + (h1 + h)rg ] ´ ( p ´ 3r 2 ) + pr 2 P0
L pd 2 p ´ 4r 2 hrg
= ò0 ( dm) xw 2 where dm = r ´ ´ dx = [ P0 + h1rg ] ´ 4pr 2 +
4 3
\ Total centripetal force 5h
or, h1 =
3
Læ ö
= ç r ´
pd 2
ò 0 è 4 ´ dx÷ø ´ xw
2
( ) 2. (b) KEY CONCEPT :
Considering equilibrium of wooden block.
Total downward force = Total force upwards
Wt. of block + force due to atmospheric pressure =
pd 2 L
= r´ ´ w 2 ò x dx Force due to pressure of liquid + Force due to
4 0 atmospheric pressure
r
pd 2 L2 p (16r 2 ) ´ g + P0 p ´ 16 r 2
= r´ ´ w2 ´ ... (ii) 3
4 2
= [h2rg + P0] p [(16 – 4)r2] + P0 × 4r2
This centripetal force is provided by the weight of liquid of
4
height H. Þ h = h2
9
From (i) and (ii)
3. (a) When the height h2 of water level is further decreased,
2 2 2 2 then the upward force acting on the wooden block
pd pd w ´L decreases. The total force downward remains the same.
´ H ´r´ g = r´ ´
4 4 2 This difference will be compensated by the normal
reaction by the tank wall on the wooden block. Thus
w 2 L2 the block does not moves up and remains at its original
H= position.
2g
MECHANICAL PROPERTIES OF SOLIDS & FLUIDS P-163
4. (c) The vertical force due to surface tension For bubble B :
2 pr 2 T
If PB is the pressure inside the bubble then
ærö
= (Tcosq) × 2pr = T çè ÷ø ´ 2pr = 4T 4 ´ 0.04 2
R R PB - 8 = = = 4 Þ PB = 12 N / m
RB 0.04
According to ideal gas equation
q 3 4
r PBVB = nB RTB Þ 12 ´ p (0.04) = n B RTB … (ii)
q 3
R Dividing (ii) by (i) we get
4
12 ´ p(0.04)3
n
3 = B [Q TA = TB ]
4 3 nA
16 ´ p(0.02)
3
5. (a) When the drop is about to detach from the dropper
nB
Weight = vertical force due to surface tension \ =6
nA
4 3 2pr 2T
pR rg = 2. Initially, the pressure of air column above water is P1 = 105
3 R
Nm–2 and volume V1 = (500 - H ) A , where A is the area of
æ 3 r 2T ö 3 (5 ´ 10 -4 ) 2 ´ 0.11 cross-section of the vessel.
\R4 = ç ÷= ´ = 4.12 ´ 10 -12
è 2 rg ø 2 1000 ´ 10 Finally, the volume of air column above water is
\ R = 1.42 × 10–3 m 300 A. If P2 is the pressure of air then
U P2 + rgh = 105
6. (b) We know that, T =
a 200
\ U = T × a = T × 4pR2 \ P2 + 103 ´ 10 ´ = 105
1000
22
= 0.11 ´ 4 ´ ´ (1.42 ´ 10 -3 ) 2 = 2.7 ´ 10 -6 J P1 = 105 N / m2
7 P2
V1 = (500 - H ) A

500 mm
ASSERTION & REASON TYPE QUESTIONS : V2 = 300 A
1. (a) We know that volume flow rate (V) of an incompressible
fluid in steady flow remains constant.
H
200 mm
V=a×v
where a = area of cross-section and v = velocity
Þ If v decreases a increases and vice - versa.
When stream of water moves up, its speed (v) decreases
Initially Finally
and therefore 'a' increases i.e. the water spreads out as
a fountain. When stream of water from hose pipe moves \ P2 = 9.8 ´ 104 N / m 2
down, its speed increases and therefore area of cross- As the temperature remains constant, according to Boyle’s
section decreases.
Therefore statement-1 is true and statement-2 is the
law 1 1 = P2V2
PV
correct explanation of statement-1. \ 105 ´ (500 - H ) A = (9.8 ´ 10 4 ) ´ 300 A
INTEGER VALUE CORRECT TYPE : Þ H = 206 mm
\ The fall of height of water level due to the opening of
1. For bubble A : orifice = 206 – 200 = 6 mm
If PA is the pressure inside the bubble then
K
3. We know that w = ...(i)
m
B
A FL æ YA ö
Here Y = ÞF =ç ÷l
Al è Lø
2 cm 4 cm Comparing the above equation with F = kl we get
–2
8 Nm æ YA ö
K =ç ÷ ...(ii)
è Lø
4T 4 ´ 0.04 2
PA - 8 = = = 8 Þ PA = 16 N / m YA
RA 0.02 From (i) & (ii), w =
ml
According to ideal gas equation,
4 n ´ 109 ´ 4.9 ´ 10-7
PAVA = n A RTA Þ 16 ´ p(0.02)3 = n A RTA … (i) \ 140 = \ n=4
3 0.1 ´ 1
P-164 Topic-wise Solved Papers - PHYSICS

1. (b) Small amount of work done in extending the spring by Case (ii) At equilibrium T = W
dx is
W/A W/A
dW = k x dx \Y = ÞY =
l/2 l/L
0.15 L/2
800 é
\W= k ò x dx =
2 ë
(0.15)2 - (0.05)2 ù = 8 J
û Þ Elongation is the same.
0.05
2 r 2 ( d1 - d 2 ) g
2. (d) Work done by constant force in displacing the object 8. (c) Terminal velocity, vT =
by a distance l. 9h
vT (10.5 - 1.5) 9
1 2
= Þ vT = 0.2 ´ = 0.1 m/s
= ´ stress × strain ×volume 0.2 (19.5 - 1.5) 2 18
2
9. (a) The condition for terminal speed (vt) is
1 F l Fl
= ´ ´ ´ A´ L = Weight = Buoyant force + Viscous force
2 A L 2
3. (b) From Stoke's law, Vg (r1 - r2 )
\ V r1 g = V r2 g + kvt2 \ vt =
viscous force F = 6phrv k
10. (d) From the figure it is clear that liquid 1 floats on liquid 2.
hence F is directly proportional to radius & velocity.
The lighter liquid floats over heavier liquid. Therefore
4. (a) Let pressure outside be P0.
we can conclude that r1 < r2
2T
\ P1 ( in smaller bubble ) = P0 + Also r3 < r2 otherwise the ball would have sink to the
r bottom of the jar.
2T Also r3 > r1 otherwise the ball would have floated in
P2 ( in bigger bubble ) = P0 + ( R > r) liquid 1. From the above discussion we conclude that
R
r1 < r3 < r2.
\ P1 > P2 11. (c) In case of water, the meniscus shape is concave
hence air moves from smaller bubble to bigger bubble. upwards. Also according to ascent formula
5. (a) Energy stored per unit volume, 2T cosq
h=
1 rrg
E= ´ stress ´ strain
2 The surface tension (T) of soap solution is less than
We know that, water. Therefore rise of soap solution in the capillary
tube is less as compared to water. As in the case of
1 stress 1 S2 water, the meniscus shape of soap solution is also
E= ´ stress ´ = . concave upwards.
2 Y 2 Y
6. (c) Water fills the tube entirely in gravity less condition 12. (c) l
i.e., 20 cm.
7. (a) Case (i) A Y 3A Y

Wire (1)
l/3
Wire (2)
T
As shown in the figure, the wires will have the same
Young’s modulus (same material) and the length of the
T T wire of area of cross-section 3A will be l/3 (same volume
W as wire 1).
For wire 1,
W W F/A
Y= ...(i)
D x/l
At equilibrium, T = W
For wire 2 ,
W/A F '/ 3 A
Y= .....(1) Y=
l/L Dx /( l / 3)
...(ii)
MECHANICAL PROPERTIES OF SOLIDS & FLUIDS P-165
F l F' l From (1) and (2)
´
From (i) and (ii) , = ´ Þ F ' = 9F
A Dx 3 A 3Dx Fe 2 Fe'
13. (b) Oil will float on water so, (2) or (4) is the correct option. = (2)
mg mg
But density of ball is more than that of oil,, hence it will
sink in oil.
Þ Fe = 2Fe'
14. (c) If K be the dielectric constant, then
30° Fe
Fe' =
T K
T cos 15°
2 Fe
\ Fe = ÞK=2
K
Fe
T sin 15° 15. (c) W = 2T DV
W = 2T4p[(52) – (3)2] × 10–4
mg = 2 × 0.03 × 4p [25 – 9] × 10–4 J = 0.4p × 10–3 J
= 0.4p mJ
Fe = T sin15° 16. (c) From Bernoulli's theorem,
mg = T cos15° 1 1
P0 + rv12rgh = P0 + rv22 + 0
2 2
Fe
Þ tan 15° = ...(1)
mg v2 = v12 + 2 gh = 0.16 + 2 ´ 10 ´ 0.2 = 2.03 m/s
In liquid , From equation of continuity
Fe ' = T 'sin15° ...(A) A2v2 = A1v1

mg = FB + T 'cos15° D22 D2
p ´ v2 = p 1 v1
4 4

Þ v1 3.55 × 10–3 m
30° D1D2 =
v2
FB T¢
17. (b) For linear expansion
T¢ cos 15°
Dl
Dl = la.DT \ = a.DT
F¢e
l
T¢ sin 15°
F/A F/A
and Y = =
Dl / l a.DT
mg

But FB = Buoyant force

= V (d - r) g = V (1.6 - 0.8) g = 0.8 Vg


So, F = AY at (Q DT = t )
m 0.8 mg mg
= 0.8 g= = F
d 1.6 2 Thermal stress = = Y at.
A
mg 18. (c) Sum of volumes of 2 smaller drops
\ mg = + T 'cos 15°
2 = Volume of the bigger drop
mg 4 4
Þ = T 'cos 15° ...(B) 2. pr 3 = pR 3 Þ R = 21/ 3 r
2 3 3
2
2F ' Surface energy = T .4pR
From (A) and (B), tan 15° = e
mg = T 4p 22 / 3 r 2 = T .2
8/3
pr 2 .
P-166 Topic-wise Solved Papers - PHYSICS
19. (a) V rg = 6phrv + V rl g At equilibrium, F = W
2Tl = mg
Þ Vg ( r - rl ) = 6phrv
mg 1.5 ´ 10-2 1.5
T= = = = 0.025 N/m = 0.025Nm
('
)
Also Vg r- ri = 6ph ' rv ' 2l 2 ´ 30 ´ 10-2 60
22. (c) From figure, kx 0 + FB = Mg
(r - rl' )
\ v 'h' = ´ vh L KX0
(r - rl ) kx 0 + s Ag = Mg
2 FB
(r - rl' )vh [Q mass = density × volume]
Þ v' = ´ Mg
(r - rl ) h ' L
Þ kx 0 = Mg - s Ag
2
(7.8 - 1.2) 10 ´ 8.5 ´ 10-4
= ´ sLAg
(7.8 - 1) 13.2 Mg -
Þ x0 = 2 = Mg æ1 - LAs ö
ç ÷
\ v ' = 6.25 ´ 10-4 cm/s k k è 2M ø
Hence, extension of the spring when it is in equilibrium
2l æ 3l ö
20. (d) Here, l A = , lB = ç ÷
5 è 5ø Mg æ LAs ö
is, x 0 = ç1 - ÷
k è 2M ø
k l = k Al A = k B l B k l = k A æç 2l ö÷ 23. (d) When radius is decrease by DR,
è 5ø
4pR 2 DRrL = 4pT[R 2 - (R - DR) 2 ]
5k 5k
kA = Þ kB = .
2 3 Þ rR 2 DRL = T[R 2 - R 2 + 2RDR - DR 2 ]
21. (d) The surface tension of the liquid film is given as
Þ rR 2 DRL = T2RDR [ DR is very small]
F
T = , where F is the force, and l = length of the
2l 2T
ÞR=
slider. rL
F = 2 lT
Heat & Thermodynamics
9 and Gases
FILL IN THE BLANKS : 9. A container of volume 1m3 is divided into two equal parts
by a partition. One part has an ideal gas at 300K and the
1. One mole of a mono-atomic ideal gas is mixed with one mole other part is vacuum. The whole system is thermally isolated
of a diatomic ideal gas. The molar specific heat of the mixture
from the surroundings. When the partition is removed, the
at constant volume is ....... (1984- 2 Marks)
gas expands to occupy the whole volume. Its temperature
2. The variation of temperature of a material as heat is given to
it at a constant rate is shown in the figure. The material is in will now be....... (1993-1 Mark)
solid state at the point O. The state of the material at the 10. An ideal gas with pressure P, volume V and temperature T is
point P is ........ (1985 - 2 Marks) expanded isothermally to a volume 2V and a final pressure
Pi. If the same gas is expanded adiabatically to a volume 2V,
the final pressure is Pa. The ratio of the specific heats of the
C
D Pa
T gas is 1.67 . The ratio P is ..... (1994 - 2 Marks)
A P
1
B
11. Two metal cubes A and B of same size are arranged as shown
O in Figure. The extreme ends of the combination are maintained
HEAT ADDED

3. During an experiment, an ideal gas is found to obey an at the indicated temperatures. The arrangement is thermally
additional law VP2 =constant, The gas is initially at a insulated. The coefficients of thermal conductivity of A and
temperature T, and volume V. When it expands to a volume B are 300 W/m °C and 200 W/m°C, respectively. After steady
2V, the temperature becomes.......... (1987 - 2 Marks) state is reached the temperature t of the interface will be .....
4. 300 grams of water at 25° C is added to 100 grams of ice at (1996 - 2 Marks)
0°C. The final temperature of the mixture is .....°C.
(1989 - 2 Marks)
5. The earth receives at its surface radiation from the sun at
the rate of 1400 W m–2. The distance of the centre of the sun
from the surface of the earth is 1.5 × 1011 m and the radius of
the sun is 7 × 108 m. Treating the sun as a black body, it
follows from the above data that its surface temperature
is...........K. (1989 - 2 Marks)
6. r
A solid copper sphere (density and specific heat c) of 12. A ring shaped tube contains two ideal gases with equal
radius r at an initial temperature 200 K is suspended inside masses and relative molar masses M1= 32 and M2 = 28. The
a chamber whose walls are at almost 0K. The time required gases are separated by one fixed partition and another
for the temperature of the sphere to drop to 100K is movable stopper S which can move freely without friction
.................... (1991 - 2 Marks) inside the ring. The angle a as shown in the figure is ...........
7. A point source of heat of power P is placed at the centre of degrees. (1997 - 2 Marks)
a spherical shell of mean radius R. The material of the shell M1 M2
has thermal conductivity K. If the temperature difference
between the outer and inner surface of the shell in not to
exceed T, the thickness of the shell should not be less than
............ (1991 - 1 Mark)
a
8. A substance of mass M kg requires a power input of P watts
to remain in the molten state at its melting point. When the
power source is turned off, the sample completely solidifies
in time t seconds. The latent heat of fusion of the substance S
is .................... (1992 - 1 Mark)
P-168 Topic-wise Solved Papers - PHYSICS
13. A gas thermometer is used as a standard thermometer for 8. Two spheres of the same meterial have radii 1 m and 4 m and
measurement of temperature. When the gas container of temperatures 4000K and 2000K respectively. The energy
the thermometer is immersed in water at its triple point 273.16 radiated per second by the first sphere is greater than that
K, the pressure in the gas thermometer reads 3.0 × 104 N/m2. by the second. (1988 - 2 Marks)
When the gas container of the same thermometer is immersed
MCQ's WITH ONE CORRECT ANSWER :
in another system, the gas pressure reads 3.5 × 104 N/m2.
The temperature of this system is therefore ....... °C 1. A constant volume gas thermometer works on (1980)
(1997C - 1 Mark) (a) The Principle of Archimedes
14. Earth receives 1400 W/m 2 of solar power. If all the solar (b) Boyle’s Law
energy falling on a lens of area 0.2 m2 is focused on to a (c) Pascal’s Law
block of ice of mass 280 grams, the time taken to melt the ice (d) Charle’s Law
will be... minutes. (Latent heat of fusion of ice = 3.3 × 10 5 J/ 2. A metal ball immersed in alcohol weighs W1 at 0°C and W2
kg.) (1997 - 2 Marks) at 50°C. The coefficient of expansion of cubical the metal is
less than that of the alcohal. Assuming that the density of
TRUE / FALSE : the metal is large compared to that of alcohol, it can be
1. The root-mean square speeds of the molecules of different shown that (1980)
ideal gases, maintained at the same temperature are the same. (a) W1 > W2 (b) W1 = W2
(1981- 2 Marks) (c) W1 < W2
2. The volume V versus temperature T graphs for a certain 3. A wall has two layers A and B, each made of different material.
amount of a perfect gas at two pressure p1 and p2 are as Both the layers have the same thickness. The thermal
shown in Fig. It follows from the graphs that p1 is greater conductivity of the meterial of A is twice that of B. Under
than p2. (1982 - 2 Marks) thermal equilibrium, the temperature difference across the
V p1 wall is 36°C. The temperature difference across the layer A
is ( 1980)
(a) 6°C (b) 12°C
p2 (c) 18°C (d) 24°C
4. An ideal monatomic gas is taken round the cycle ABCDA as
shown in the P – V diagram (see Fig.). The work done
T during the cycle is (1983 - 1 Mark)
3. Two different gases at the same temperature have equal P
root mean square velocities. (1982 - 2 Marks) 2P, V 2P, 2V
7 B C
4. The ratio of the velocity of sound in Hydrogen gas ( g = )
5
5 A D
to that in Helium gas ( g = ) at the same temperature is P, V P,2V
3 V
21 (a) PV (b) 2 PV
. (1983 - 2 Marks)
5 1
(c) PV (d) zero
5. The curves A and B in the figure shown P-V graphs for an 2
isothermal and an adiabatic process for an ideal gas. The
isothermal process is represented by the curve A. æ 5ö
5. If one mole of a monatomic gas ç g = ÷ is mixed with one
(1985 - 3 Marks) è 3ø

æ 7ö
mole of a diatomic gas ç g = ÷ , the value of g for mixture
è 5ø
P A
is (1988 - 1 Mark)
B (a) 1.40 (b) 1.50
(c) 1.53 (d) 3.07
V 6. From the following statements concerning ideal gas at any
6. At a given temperature, the specific heat of a gas at constant given temperature T, select the correct one(s) (1995S)
pressure is always greater than its specific heat at constant (a) The coefficient of volume expansion at constant
volume. (1987 - 2 Marks) pressure is the same for all ideal gases
7. The root mean square (rms) speed of oxygen molecules (O2) (b) The average translational kinetic energy per molecule
at a certain temperature T (degree absolute) is V. If the of oxygen gas is 3kT, k being Boltzmann constant
(c) The mean-free path of molecules increases with
temperature is doubled and oxygen gas dissociates into
increases in the pressure
atomic oxygen, the rms speed remains unchanged.
(d) In a gaseous mixture, the average translational kinetic
(1987 - 2 Marks)
energy of the molecules of each component is different
HEAT & THERMODYNAMICS AND GASES P-169
7. Three rods of identical cross-sectional area and made from 15. A gas mixture consists of 2 moles of oxygen and 4 moles of
thesame metal from thesides of an isosceles traingle ABC, argon at temperature T. Neglecting all vibrational modes,
right-angled at B. The points A and B are maintained at the total internal energy of the system is(1999S - 2 Marks)
temperatures T and ( 2 ) T respectively. In the steady state, (a) 4 RT (b) 15 RT
(c) 9 RT (d) 11 RT
the temperature of the point C is Tc. Assuming that only
16. The ratio of the speed of sound in nitrogen gas to that in
heat conduction takes place, Tc / T is (1995S)
helium gas, at 300 K is (1999S - 2 Marks)
1 3
(a) (b) (a) (2 / 7 (b) (1 / 7)
2( 2 - 1) 2 +1
1 (c) ( 3) / 5 (d) ( 6 ) / 5
1
(c) (d) 17. A monatomic ideal gas, initially at temperature T1, is enclosed
3 ( 2 - 1) 2 +1
in a cylinder fitted with a frictionless piston. The gas is
8. Two metallic spheres S1 and S2 are made of the same material allowed to expand adiabatically to a temperature T2 by
and have got identical surface finish. The mass of S1 is
releasing the piston suddenly. If L1 and L2 are the length of
thrice that of S2. Both the spheres are heated to the same
high temperature and placed in the same room having lower the gas column before and after expansion respectively, then
temperature but are thermally insulated from each other. The T1
ratio of the initial rate of cooling of S1 to that of S2 is (1995S) T2 is given by (2000S )
1 1
(a) (b) æ L1 ö
2/3
L1
3 3
(a) çè L ÷ø (b) L2
1 2
3 æ 1 ö3
(c) (d) ç ÷ 2/3
1 è 3ø L2 æ L2 ö
(c) L1 (d) çè L ÷ø
9. The intensity of radiation emitted by the sun has its maximum 1
value at a wavelength of 510 nm and that emitted by the
18. A block of ice at –10°C is slowly heated and converted to
North Star has the maximum value at 350 nm. If these stars
steam at 100°C. Which of the following curves represents
behave like blackbodies, then the ratio of the surface
temperatures of the Sun and the North Star is the phenomenon qualitatively ? (2000S)
(a) 1.46 (b) 0.69 (1997C - 1 Mark)
(c) 1.21 (d) 0.83
T T
10. The average translational energy and the rms speed of
(a) (b)
molecules in a sample of oxygen gas at 300 K are
6.21 × 10–21 J and 484 m/s respectively. The corresponding Heat supplied Heat supplied
values at 600 K are nearly (assuming ideal gas behavior)
(1997C - 1 Mark)
(a) 12.42 × 10–21 J, 968 m/s (b) 8.78 × 10–21 J, 684 m/s T T
(c) 6.21 × 10–21 J, 968 m/s (d) 12.42 × 10–21 J, 684 m/s (c) (d)
11. The average translational kinetic energy of O2 (relative molar Heat supplied Heat supplied
mass 32) molecules at a particular temperature is 0.048 eV.
The translational kinetic energy of N2 (relative molar mass 19. An ideal gas is initially at temperature T and volume V. Its
28) molecules in eV at the same temperature is volume is increased by DV due to an increase in temperature
(a) 0.0015 (b) 0.003 (1997 - 1 Mark) DV
(c) 0.048 (d) 0.768 DT, pressure remaining constant. The quantity d =
V DT
12. A vessel contains 1 mole of O2 gas (relative molar mass 32)
at a temperature T. The pressure of the gas is P. An identical varies with temperature as (2000S)
vessel containing one mole of He gas (relative molar mass Y
Y
4) at a temperature 2T has a pressure of (1997 - 1 Mark)
(a) P/8 (b) P (c) 2P (d) 8P
13. A spherical black body with a radius of 12 cm radiates 450 (a) d (b) d
W power at 500 K. if the radius were halved and the o X o X
T T + DT T T + DT
temperature doubled, the power radiated in watt would be Temperature K Temperature K
(a) 225 (b) 450 (1997 - 1 Mark)
(c) 900 (d) 1800 Y
14. A closed compartment containing gas is moving with some Y
acceleration in horizontal direction. Neglect effect of gravity.
Then the pressure in the compartment is (1999S - 2 Marks) d
(a) same everywhere (b) lower in the front side (c) o X (d) d
T T + DT o X
(c) lower in the rear side (d) lower in the upper side T T + DT
Temperature K
Temperature K
P-170 Topic-wise Solved Papers - PHYSICS
20. Starting with the same initial conditions, an ideal gas expands
1 + 60g Fe 1 - 60 g Fe
from volume V1 to V2 in three different ways. The work done (a) (b)
by the gas is W1 if the process is purely isothermal, W2 if 1 + 60g hg 1 + 60 g Hg
purely isobaric and W3 if purely adiabatic. Then (2000S)
y 1 + 60 g Fe 1 + 60 g Hg
(c) (d)
Isobaric 1 - 60g Hg 1 + 60 g Fe
W2
Isothermal 26. An ideal gas is taken through the cycle A ® B ® C ® A ,
P W1
Adiabatic as shown in the figure. If the net heat supplied to the gas in
W3
the cycle is 5J, the work done by the gas in the process
x
o V1 V2 C ® A is (2002S )
(a) W2 > W1 > W3 (b) W2 > W3 > W1
C B
(c) W1 > W2 > W3 (d) W1 > W3 > W2 2
3
21. The plots of intensity versus wavelength for three black V(m )
bodies at temperature T1, T2 and T3 respectively are as 1
shown. Their temperatures are such that (2000S ) A

Y
T3 P(N/m2) 10
T2 (a) –5 J (b) –10 J
T1 (c) –15 J (d) –20 J
I 27. Which of the following graphs correctly represents the
dV / dP
o x variation of b = - with P for an ideal gas at constant
V
l
temperature ? (2002S )
(a) T1 > T2 > T3 (b) T1 > T3 > T2
(c) T2 > T3 > T1 (d) T3 > T2 > T1
22. Three rods made of same material and having the same
cross-section have been joined as shown in the figure. Each (a) (b)
rod is of the same length. The left and right ends are kept at
0oC and 90oC respectively. The temperature of the junction
of the three rods will be (2001S) P P
90 oC
0 oC
(c) (d)
90 oC
(a) 45oC (b) 60oC
(c) 30oC (d) 20oC P P
23. In a given process on an ideal gas, dW = 0 and dQ < 0. Then 28. An ideal Black-body at room temperature is thrown into a
for the gas (2001S) furnace. It is observed that (2002S )
(a) the temperature will decrease (a) initially it is the darkest body and at later times the
(b) the volume will increase brightest
(c) the pressure will remain constant (b) it is the darkest body at all times
(d) the temperature will increase (c) it cannot be distinguished at all times
24. P-V plots for two gases during adiabatic processes are (d) initially it is the darkest body and at later times it cannot
shown in the figure. Plots 1 and 2 should correspond be distinguished
respectively to (2001S) 29. The graph, shown in the adjacent diagram, represents the
P variation of temperature (T) of two bodies, x and y having
same surface area, with time (t) due to the emission of
radiation. Find the correct relation between the emissivity
and absorptivity power of the two bodies (2003S)
1
2
V T y
(a) He and O2 (b) O2 and He x
(c) He and Ar (d) O2 and N2
25. When a block of iron floats in mercury at 0oC, fraction k1 of
its volume is submerged, while at the temperature 60 oC, a
fraction k2 is seen to be submerged. If the coefficient of t
volume expansion of iron is gFe and that of mercury is gHg, (a) Ex > Ey & ax < ay (b) Ex < Ey & ax > ay
then the ratio k1/k2 can be expressed as (2001S) (c) Ex > Ey & ax > ay (d) Ex < Ey & ax < ay
HEAT & THERMODYNAMICS AND GASES P-171
30. Two rods, one of aluminum and the other made of steel, 35. Two identical rods are connected between two containers
having initial length l1 and l2 are connected together to one of them is at 100ºC and another is at 0ºC. If rods are
form a single rod of length l1 + l2. The coefficients of linear connected in parallel then the rate of melting of ice is q1 gm/
expansion for aluminum and steel are aa and as and sec. If they are connected in series then the rate is q2. The
respectively. If the length of each rod increases by the same ratio q2/ q1 is (2004S)
amount when their temperature are raised by t0C, then find (a) 2 (b) 4
the ratiol1/(l1 + l2) (2003S) (c) 1/2 (d) 1/4
(a) a s/ a a (b) a a/ a s 36. An ideal gas is initially at P1, V1 is expanded to P2, V2 and
(c) a s/( a a + a s ) (d) a a/( a a + a s ) then compressed adiabatically to the same volume V1 and
31. The PT diagram for an ideal gas is shown in the figure,
pressure P3. If W is the net work done by the gas in complete
where AC is an adiabatic process, find the corresponding
PV diagram. (2003S) process which of the following is true (2004S)
(a) W > 0 ; P3 > P1 (b) W < 0 ; P3 > P1
(c) W > 0 ; P3 < P1 (d) W < 0 ; P3 < P1
A 37. Variation of radiant energy emitted by sun, filament of
P tungsten lamp and welding arc as a function of its
wavelength is shown in figure. Which of the following option
is the correct match ? (2005S)
B C
El
T
A
A
P
P
T3
(a) C (b)
B C B

V V T2

A A T1
P P

(c) (d) B l
C
B C
(a) Sun-T3, tungsten filament - T1, welding arc - T2
V V
32. 2 kg of ice at –200C is mixed with 5kg of water at 200C in an (b) Sun-T2, tungsten filament - T1, welding arc - T3
insulating vessel having a negligible heat capacity. Calculate (c) Sun-T3, tungsten filament - T2, welding arc - T1
the final mass of water remaining in the container. It is given (d) Sun-T1, tungsten filament - T2, welding arc - T3
that the specific heats of water & ice are 1kcal/kg/0C & 0.5 38. In which of the following process, convection does not
kcal/kg/0C while the latent heat of fusion of ice is 80 kcal/kg take place primarily (2005S)
(a) 7 kg (b) 6 kg (2003S) (a) sea and land breeze
(c) 4 kg (d) 2 kg (b) boiling of water
33. Three discs A, B and C having radii 2, 4, and 6 cm respectively (c) heating air around a furnace
are coated with carbon black. Wavelength for maximum (d) warming of glass of bulb due to filament
intensity for the three discs are 300, 400 and 500 nm 39. A spherical body of area A and emissivity e = 0.6 is kept
respectively. If QA, QB and QC are power emitted by A, B and inside a perfectly black body. Total heat radiated by the
D respectively, then (2004S) body at temperature T (2005S)
(a) QA will be maximum (b) QB will be maximum (a) 0.4 AT4 (b) 0.8 AT4
(c) QC will be maximum (d) QA = QB = QC (c) 0.6 AT4 (d) 1.0 AT4
34. If liquefied oxygen at 1 atmospheric pressure is heated from 40. Calorie is defined as the amount of heat required to raise
50 k to 300 k by supplying heat at constant rate. The graph temperature of 1 g of water by 1oC and it is defined under
of temperature vs time will be (2004S) which of the following conditions? (2005S)
(a) From 14.5 oC to 15.5 oC at 760 mm of Hg
(b) From 98.5 oC to 99.5 oC at 760 mm of Hg
T (c) From 13.5 oC to 14.5 oC at 76 mm of Hg
(a) (b) T (d) From 3.5 oC to 4.5oC at 76 mm of Hg
41. Water of volume 2 litre in a container is heated with a coil of
t t
1 kW at 27 oC. The lid of the container is open and energy
dissipates at rate of 160 J/s. In how much time temperature
will rise from 27oC to 77oC [Given specific heat of water is
4.2 kJ/kg] (2005S)
(c) T (d) T (a) 7 min (b) 6 min 2 s
(c) 8 min 20 s (d) 14 min
t t
P-172 Topic-wise Solved Papers - PHYSICS
42. Water is filled up to a height h in a beaker of radius R as The time to transport the same amount of heat in the
shown in the figure. The density of water is r, the surface configuration-II is (JEE Adv. 2013)
tension of water is T and the atmospheric pressure is P0.
Consider a vertical section ABCD of the water column Configuration-I Configuration-II
through a diameter of the beaker. The force on water on one
side of this section by water on the other side of this section
has magnitude (2007) 2k

2R k 2k k
x
(a) 2.0 s (c) 4.5 s
B
(b) 3.0 s (d) 6.0 s
49. Two non-reactive monoatomic ideal gases have their atomic
A
masses in the ratio 2 : 3. The ratio of their partial pressures,
C
h when enclosed in a vessel kept at a constant temperature, is
4 : 3. The ratio of their densities is (JEE Adv. 2013)
(a) 1 : 4 (b) 1 : 2
D (c) 6 : 9 (d) 8 : 9
(a) | 2P0Rh + pR2rgh – 2RT |
(b) | 2P0Rh + R rgh2 – 2RT |
(c) | P0pR2 + Rrgh2 – 2RT | 1. At room temperature, the rms speed of the molecules of a
(d) | P0pR2 + Rrgh2 + 2RT | certain diatomic gas is found to be 1930 m/s. The gas is
43. An ideal gas is expanding such that PT2 = constant. The (1984- 2 Marks)
coefficient of volume expansion of the gas is – (2008) (a) H2 (b) F2
(a) 1/T (b) 2/T (c) O2 (d) Cl2
(c) 3/T (d) 4/T 2. 70 calories of heat required to raise the temperature of
44. A real gas behaves like an ideal gas if its (2010) 2 moles of an ideal gas at constant pressure from 30°C to
(a) pressure and temperature are both high 35°C. The amount of heat required (in calories) to raise the
(b) pressure and temperature are both low temperature of the same gas through the same range (30°C
(c) pressure is high and temperature is low to 35°C) at constant volume is : (1985 - 2 Marks)
(d) pressure is low and temperature is high
(a) 30 (b) 50
45. 5.6 liter of helium gas at STP is adiabatically compressed to
(c) 70 (d) 90
0.7 liter. Taking the initial temperature to be T1, the work
3. Steam at 100°C is passed into 1.1 kg of water contained in a
done in the process is (2011)
calorimeter of water equivalent 0.02 kg at 15°C till the
9 3 temperature of the calorimeter and its contents rises to 80°C.
(a) RT (b) RT
8 1 2 1 The mass of the steam condensed in kilogram is
15 9 (1986 - 2 Marks)
(c) RT1 (d) RT
8 2 1 (a) 0.130 (b) 0.065
46. A mixture of 2 moles of helium gas (atomic mass = 4 amu) (c) 0.260 (d) 0.135
and 1 mole of argon gas (atomic mass = 40 amu) is kept at 4. A cylinder of radius R made of a material of thermal
300 K in a container. The ratio of the rms speeds conductivity K1 is surrounded by a cylindrical shell of inner
radius R and outer radius 2R made of a material of thermal
æ vrms (helium) ö
çè v (argon) ÷ø is (2012) conductivity K2. The two ends of the combined system are
rms maintained at two different temperatures. There is no loss
(a) 0.32 (b) 0.45 of heat across the cylindrical surface and the system is in
(c) 2.24 (d) 3.16 steady state. The effective thermal conductivity of the
47. Two moles of ideal helium gas are in a rubber balloon at
system is (1988 - 2 Marks)
30°C. The balloon is fully expandable and can be assumed
(a) K1 + K2 (b) K1K2 / (K1 + K2)
to require no energy in its expansion. The temperature of
the gas in the balloon is slowly changed to 35°C. The amount (c) (K1 + 3K2) / 4 (d) (3K1 + 3K2) / 4
of heat required in raising the temperature is nearly (take R 5. For an ideal gas : (1989 - 2 Marks)
= 8.31 J/mol.K) (2012) (a) the change in internal energy in a constant pressure
(a) 62 J (b) 104 J process from temperature T1 to T2 is equal to nCv
(c) 124 J (d) 208 J (T2 – T1), where Cv is the molar specific heat at constant
48. Two rectangular blocks, having identical dimensions, can volume and n the number of moles of the gas.
be arranged either in configuration-I or in configuration-II (b) the change in internal energy of the gas and the work
as shown in the figure. One of the blocks has thermal done by the gas are equal in magnitude in an adiabatic
conductivity k and the other 2k. The temperature difference process.
between the ends along the x-axis is the same in both the (c) the internal energy does not change in an isothermal
configurations. It takes 9 s to transport a certain amount of process.
heat from the hot end to the cold end in the configuration-I. (d) no heat is added or removed in an adiabatic process.
HEAT & THERMODYNAMICS AND GASES P-173
6. When an ideal diatomic gas is heated at constant pressure, 12. Two cylinders A and B fitted with pistons contain equal
the fraction of the heat energy supplied which increases the amounts of an ideal diatomic gas at 300 K. The piston of A is
internal energy of the gas is (1990 - 2 Marks) free to move, while that B is held fixed. The same amount of
2 3 heat is given to the gas in each cylinder. If the rise in
(a) (b) temperature of the gas in A is 30 K, then the rise in
5 5 temperature of the gas in B is (1998S - 2 Marks)
3 5 (a) 30 K (b) 18 K
(c) (d) (c) 50 K (d) 42 K
7 7
7. Three closed vessels A, B and C are at the same temperature 13. During the melting of a slab of ice at 273 K at atmospheric
T and contain gases which obey the Maxwellian distribution pressure, (1998S - 2 Marks)
of velocities. Vessel A contain only O2, B only N2 and C a (a) positive work is done by the ice-water system on the
mixture of equal quantities of O2 and N2. If the average atmosphere.
speed of the O2 molecules in vessel A is v1 that of the N2 (b) positive work is done on the ice- water system by the
molecules in vessel B is v2, the average speed of the O2 atmosphere.
molecules in vessel C is (1992 - 2 Marks) (c) the internal energy of the ice-water system increases.
(d) the internal energy of the ice-water system decreases.
v1 + v2 14. A blackbody is at a temperature of 2880 K. The energy of
(a) (b) v1
2 radiation emitted by this object with wavelength between
1 499 nm and 500 nm is U1, between 999 nm and 1000 nm is U2
3kT and between 1499 nm and 1500nm is U3. The Wien constant
(c) (v1.v2 ) 2 (d)
M b = 2.88×106nm K. Then (1998S - 2 Marks)
where M is the mass of an oxygen molecule. (a) U1 = 0 (b) U3 = 0
8. An ideal gas is taken from the state A (pressure P, volume V) (c) U1 > U2 (d) U2 > U1
to the state B (pressure P/2, volume 2V) along a straight line 15. A bimetallic strip is formed out of two identical strips one of
path in the P-V diagram. Select the correct statement (s) copper and the other of brass. The coefficients of linear
from the following: (1993-2 Marks) expansion of the two metals are ac and aB. On heating, the
(a) The work done by the gas in the process A to B exceeds temperature of the strip goes up by DT and the strip bends
the work that would be done by it if the system were to form an arc of radius of curvature R. Then R is.
taken from A to B along the isotherm. (a) proportional to DT (1999S - 3 Marks)
(b) In the T-V diagram, the path AB becomes a part of a (b) inversely proportional to DT
parabola (c) proportional to |aB – aC|
(c) In the P-T diagram, the path AB becomes a part of a (d) inversely proportional to |aB – aC|
hyperbola 16. Two identical containers A and B with frictionless pistons
(d) In going from A to B, the temperature T of the gas first contain the same ideal gas at the same temperature and the
increases to a maximum value and then decreases. same volume V. The mass of the gas in A is mA, and that in B
9. Two bodies A and B ahave thermal emissivities of 0.01 and is mB. The gas in each cylinder is now allowed to expand
0.81 respectively. The outer surface areas of the two bodies isothermally to the same final volume 2V. The changes in
are the same. The two bodies emit total radiant power of the the pressure in A and B are found to be DP and 1.5 DP
same rate. The wavelength lB corresponding to maximum respectively. Then (1998S - 2 Marks)
spectral radiancy in the radiation from B shifted from the (a) 4mA = 9mB (b) 2mA = 3mB
wavelenth corresponding to maximum spectral radiancy in
(c) 3mA = 2mB (d) 9mA = 4mB
the radiation from A, by 1.00 mm. If the temperature of A is
5802 K : (1994 - 2 Marks) 17. Let v, v rms and vp respectively denote the mean speed. root
(a) the temperature of B is 1934 K mean square speed, and most probable speed of the
(b) l B = 1.5 mm molecules in an ideal monatomic gas at absolute temperature
(c) the temperature of B is 11604 K T. The mass of a molecule is m. Then (1998S - 2 Marks)
(d) the temperature of B is 2901 K (a) no molecule can have a speed greater than 2 v rms
10. The temperature of an ideal gas is increased from 120 K to
480 K. If at 120 K the root-mean-square velocity of the gas (b) no molecule can have a speed less than v p / 2
molecules is v, at 480 K it becomes (1996 - 2 Marks) v p < v < v rms
(c)
(a) 4v (b) 2v
(c) v/2 (d) v/4 3
(d) the average kinetic energy of a molecule is mv 2p .
11. A given quantity of a ideal gas is at pressure P and absolute 4
temperature T. The isothermal bulk modulus of the gas is 18. A vessel contains a mixture of one mole of oxygen and two
(1998S - 2 Marks) moles of nitrogen at 300 K. The ratio of the average rotational
2 kinetic energy per O2 molecule to that per N2 molecule is
(a) P (b) P (a) 1 : 1 (1998S - 2 Marks)
3
(b) 1 : 2
3 (c) 2 : 1
(c) P (d) 2P
2 (d) depends on the moments of inertia of the two molecules
P-174 Topic-wise Solved Papers - PHYSICS
19. A black body of temperature T is inside chamber of T0 23. A composite block is made of slabs A, B, C, D and E of
temperature initially. Sun rays are allowed to fall from a hole different thermal conductivities (given in terms of a constant
in the top of chamber. If the temperature of black body (T) K and sizes (given in terms of length, L) as shown in the
and chamber (T0) remains constant, then (2006 - 5M, –1) figure. All slabs are of same width. Heat ‘Q’ flows only from
left to right through the blocks. Then in steady state
(2011)
T0
T

(a) Black body will absorb more radiation


(b) Black body will absorb less radiation
(c) Black body emit more energy
(d) Black body emit energy equal to energy absorbed by it
20. Cv and Cp denote the molar specific heat capacities of a gas
at constant volume and constant pressure, respectively.
Then (2009)
(a)heat flow through A and E slabs are same.
(a) Cp – Cv is larger for a diatomic ideal gas than for a
(b)heat flow through slab E is maximum.
monoatomic ideal gas
(b) Cp + Cv is larger for a diatomic ideal gas than for a (c)temperature difference across slab E is smallest.
monatomic ideal gas (d)heat flow through C = heat flow through B + heat flow
(c) Cp / Cv is larger for a diatomic ideal gas than for a through D.
monoatomic ideal gas 24. The figure below shows the variation of specific heat
(d) Cp . Cv is larger for a diatomic ideal gas than for a capacity (C) of a solid as a function of temperature (T). The
monoatomic ideal gas temperature is increased continuously from 0 to 500 K at a
21. The figure shows the P-V plot of an ideal gas taken through constant rate. Ignoring any volume change, the following
a cycle ABCDA. The part ABC is a semi-circle and CDA is statement(s) is (are) correct to a reasonable approximation.
half of an ellipse. Then, (2009) (JEE Adv. 2013)
P
A
3

2 B
D
C
1 C

0 1 2 3 V
(a) the process during the path A ® B is isothermal
(b) heat flows out of the gas during the path B ® C ® D 100 200 300 400 500
(c) work done during the path A ® B ® C is zero (a) The rate at which heat is absorbed in the range 0 -100 K
(d) positive work is done by the gas in the cycle ABCDA varies linearly with temperature T.
22. One mole of an ideal gas in initial state A undergoes a cyclic
(b) Heat absorbed in increasing the temperature from
process ABCA, as shown in the figure. Its pressure at A is
P0. Choose the correct option(s) from the following (2010) 0-100 K is less than the heat required for increasing the
V temperature from 400-500 K.
4V0 B (c) There is no change in the rate of heat absorption in the
range 400-500 K.
(d) The rate of heat absorption increases in the range
V0 A 200-300 K.
C
T0 T SUBJECTIVE PROBLEMS :
(a) Internal energies at A and B are the same 1. A sinker of weight w0 has an apparent weight w1 when
(b) Work done by the gas in process AB is P0V0 ln 4 weighed in a liquid at a temperature t1 and w2 when weight
P0 in the same liquid at temperature t2. The coefficient of cubical
(c) Pressure at C is
4 expansion of the material of sinker is b. What is the
T0 coefficient of volume expansion of the liquid.
(d) Temperature at C is
4 (1978)
HEAT & THERMODYNAMICS AND GASES P-175
2. Three rods of material X and three rods of material Y are B
connected as shown in the figure. All the rods are of identical
length and cross-sectional area. If the end A is maintained at
2
60°C and the junction E at 10°C. Calculate the temperature
of the junctions B, C and D. The thermal conductivity of X
is 0.92 cal/sec-cm-°C and that of Y is 0.46 cal/sec-cm-°C.
(1978) 1
X
C
X 1 2
A B (In the figure, CA is parallel to the V-axis and BC is parallel to
X E
Y the T-axis)
Y Y 9. A lead bullet just melts when stopped by an obstacle.
D Assuming that 25 per cent of the heat is absorbed by the
3. A room is maintained at 20°C by a heater of resistance of 20 obstacle, find the velocity of the bullet if its initial temperature
ohms connected to 200 volts mains. The temperature is is 27°C.
uniform throughout the room and the heat is transmitted (Melting point of lead = 327°C, specific heat of lead = 0.03
through a glass window of area 1 m2 and thinkness 0.2 cm. calories /gm/°C, latent heat of fusion of lead = 6 calories /
Calculate the temperature outside. Thermal conductivity of gm, J = 4.2 joules /calorie). (1981- 3 Marks)
glass is 0.2 cal/m-sec-°C and mechanical equivalent of heat 10. Calculate the work done when one mole of a perfect gas is
is 4.2 joules/cal. (1978) compressed adiabatically. The initial pressure and volume
4. Given samples of 1 c.c. of hydrogen and 1c.c. of oxygen, of the gas are 105 N/m2 and 6 litres respectively. The final
both at N.T.P. which sample has a larger number of molecules? volume of the gas is 2litre. Molar specific heat of the gas at
(1979) constant volume is 3R/2. (1982 - 8 Marks)
5. A composite rod is made by joining a copper rod, end to 11. A solid sphere of copper of radius R and a hollow sphere of
end, with a second rod of different material but of the same the same material of inner radius r and outer radius A are
cross-section. At 25°C, the composite rod is 1 metre in length, heated to the same temperature and allowed to cool in the
of which the length of the copper rod is 30 cm. At 125°C the same environment. Which of them starts cooling faster ?
length of the composite rod increases by 1.91 mm. (1982 - 2 Marks)
When the composite rod is not allowed to expand by holding 12. One gram mole of oxygen at 27° and one atmospheric
it between two rigid walls, it is found that the length of the pressure is enclosed in a vessel. (1983 - 8 Marks)
(i) Assuming the molecules to be moving with Vrms, Find
two constituents do not change with the rise of temperature.
the number of collisions per second which the
Find the Young’s modulus and the coefficient of linear
molecules make with one square metre area of the
expansion of the second rod.
vessel wall.
[Coefficient of linear expansions of copper = 1.7 × 10–5/°C
(ii) The vessel is next thermally insulated and moved with
Young's modulus of Copper = 1.3 × 1011 N/m2] (1979) a constant speed Vo. It is then suddenly stopped. The
6. A Solid material is supplied with heat at a constant rate. process results in a rise of the temperature of the gas
The temperature of the material is changing with the heat by 1°C. Calculate the speed Vo.
input as shown in the graph in figure. Study the graph 13. The rectangular box shown in Fig has a partition which can
carefully and answer the following questions : (1980) slide without friction along the length of the box. Initially
each of the two chambers of the box has one mole of a
E mono-atomic ideal gas (g = 5/3) at a pressure P0, volume V0
TEMPERATURE

and temperature T0. The chamber on the left is slowly heated


C D by an electric heater. The walls of the box and the partition
A B
are thermally insulated. Heat loss through the lead wires of
the heater is negligible. The gas in the left chamber expands
pushing the partition until the final pressure in both
O HEAT INPUT chambers becomes 243 P0 /32. Determine (i) the final
(i) What do the horizontal regions AB and CD represent? temperature of the gas in each chamber and (ii) the work
(ii) If CD is equal to 2AB, what do you infer? done by the gas in the right chamber. (1984- 8 Marks)
(iii) What does the slope of DE represent?
(iv) The slope of OA > the slope of BC. What does this
indicate?
7. A jar contains a gas and a few drops of water at T°K. The
pressure in the jar is 830 mm of Hg. The temperature of the
14. Two glass bulbs of equal volume are connected by a narrow
jar is reduced by 1%. The saturated vapour pressures of
tube and are filled with a gas at 0°C and a pressure of 76 cm
water at the two temperatures are 30 and 25 mm of Hg.(1980)
of mercury. One of the bulbs is then placed in melting ice
Calculate the new pressure in the jar.
and the other is placed in a water bath maintained at 62°C.
8. A cyclic process ABCA shown in the V-T diagram (fig) is
What is the new value of the pressure inside the bulbs? The
performed with a constant mass of an ideal gas. Show the volume of the connecting tube is negligible.
same process on a P-V diagram (1981- 4 Marks) (1985 - 6 Marks)
P-176 Topic-wise Solved Papers - PHYSICS
15. An electric heater is used in a room of total wall area 137 m2 (i) How many degrees of freedom do the gas molecules
to maintain a temperature of + 20°C inside it, when the outside have?
temperature is – 10°C. The walls have three different layers (ii) Obtain the work done by the gas during the expansion
materials. The innermost layer is of wood of thickness as a function of the initial pressure P and volume V.
2.5 cm, the middle layer is of cement of thickness 1.0 cm and
the outermost layer is of brick of thickness 25.0 cm. Find the 21. Three moles of an ideal gas æç C p = 7 Rö÷ at pressure, PA
è 2 ø
power of the electric-heater. Assume that there is no heat
loss through the floor and the ceiling. The thermal and temperature TA is isothermally expanded to twice its
conductivities of wood, cement and brick are 0.125, 1.5 and initial volume. It is then compressed at constant pressure to
1.0 watt/m/°C respectively. (1986 - 8 Marks) its original volume. Finally gas is compressed at constant
16. A thin tube of uniform cross-section is sealed at both ends. volume to its original pressure PA. (1991 - 4 + 4 Marks)
It lies horizontally, the middle 5 cm containing mercury and (a) Sketch P - V and P - T diagrams for the complete process.
the two equal end containing air at the same pressure P. (b) Calculate the net work done by the gas, and net heat
When the tube is held at an angle of 60° with the vertical supplied to the gas during the complete process.
direction, the length of the air column above and below the 22. Two moles of helium gas undergo a cyclic process as shown
mercury column are 46cm and 44.5 cm respectively. Calculate in Fig. Assuming the gas to be ideal, calculate the following
the pressure P in centimeters of mercury. (The temperature quantities in this process (1992 - 8 Marks)
of the system is kept at 30°C). (1986 - 6 Marks) A
2 atm B
17. An ideal gas has a specific heat at constant pressure
5R
CP = . The gas is kept in a closed vessel of volume 0.0083
2 1 atm C
m3, at a temperature of 300 K and a pressure of 1.6 × 10 6 N/ D
m2. An amount of 2.49 × 104 Joules of heat energy is supplied
300 K 400 K
to the gas. Calculate the final temperature and pressure of (a) The net change in the heat energy
the gas. (1987 - 7 Marks) (b) The net work done
18. Two moles of helium gas (g = 5/3) are initially at temperature (c) The net change in internal energy
27°C and occupy a volume of 20 litres. The gas is first 23. A cylindrical block of length 0.4 m and area of cross section
expanded at constant pressure until the volume is doubled. 0.04 m2 is placed coaxially on a thin metal disc of mass
Then it undergoes an adiabatic change until the temperature 0.4 Kg and the same cross section. The upper face of the
returns to its initial value. (1988 - 6 Marks) cylinder is maintained at a constant temperature of 400 K
(i) Sketch the process on a p-V diagram. and the initial temperature of the disc is 300 K. If the thermal
(ii) What are the final volume and pressure of the gas ? conductivity of the materical of the cylinder is 10 watt/m-K
(iii) What is the work done by the gas ? and the specific heat of the material of the disc is 600 J/kg.K,
19. An ideal monatomic gas is confined in a cylinder by a spring- how long will it take for the temperature of the disc to
loaded piston of cross-section 8.0 × 10–3 m2. Initially the increase to 350 K ? Assume, for purpose of calculation, the
gas is at 300 K and occupies a volume of 2.4 × 10–3 m3 and thermal conductivity of the disc to be very high and the
the spring is in its relaxed (unstretched, uncompressed) system to be thermally insulated except for the upper face
state, fig. The gas is heated by a small electric heater until of the cylinder. (1992 - 8 Marks)
the piston moves out slowly by 0.1 m. Calculate the final 24. One mole of a mono atomic ideal gas is taken through the
temperature of the gas and the heat supplied (in joules) by cycle shown in Fig: (1993 - 4+4+2 Marks)
the heater. The force constant of the spring is 8000 N/m, A ® B : adiabatic expansion
atmospheric pressure is 1.0 × 105 Nm–2. The cylinder and
B ® C : cooling at constant volume
the piston are thermally insulated. The piston is massless
and there is no friction between the piston and the cylinder. C ® D : adiabatic compression
Neglect heat loss through lead wires of the heater. The heat D ® A : heating at constant volume.
capacity of the heater coil is negligible. Assume the spring P
to be massless. (1989 - 8 Mark) A
B
Open atmosphere

Rigid D C
Heater support
V
The pressure and temperature at A, B, etc. are denoted by
PA , TA , PB , TB etc., respectively. Given that TA = 1000 K,
PB= (2/3)PA and PC = (1/ 3) PA , calculate the following
20. An ideal gas having initial pressure P, volume V and
temperature T is allowed to expand adiabatically until its quantities :
T (i) The work done by the gas in the process A ® B
volume becomes 5.66 V while its temperature falls to . (ii) The heat lost by the gas in the process B ® C.
2
(1990 - 7 Mark) (iii) The temperature TD. [Given : (2 / 3)2/5 = 0.85]
HEAT & THERMODYNAMICS AND GASES P-177
25. An ideal gas is taken through a cyclic thermodynamic (i) What is the temperature of Helium in each of the states
process through four steps. The amounts of heat involved A, B, C and D?
in these steps are Q1 = 5960 J, Q2 = –5585 J, Q3 = –2980J and (ii) Is there any way of telling afterwards which sample of
Q4 = 3645J, respectively. The correspending quantities of Helium went through the process ABC and which went
work involved are W1 = 2200 J, W2 = – 825 J,W3 = – 1100 J through the process ADC? Write Yes or No.
and W4 respectively. (1994 - 6 Marks) (iii) How much is the heat involved in each of the process
1. Find the value of W4. ABC and ADC?
2. What is the efficiency of the cycle 32. One mole of a diatomic ideal gas (g = 1.4) is taken through a
26. A closed container of volume 0.02 m 3 contains a mixture of cyclic process starting from point A. The process A ® B
neon and argon gases, at a temperature of 27°C and pressure
of 1 × 105 Nm–2. The total mass of the mixture is 28 g. If the is an adiabatic compression, B ® C is isobaric expansion,
molar masses of neon and argon are 20 and 40 g mol–1 C ® D is an adiabatic expansion, and D ® A is isochoric.
respectively, find the masses of the individual gases in the The volume ratios are VA / VB = 16 and VC / VB = 2 and the
container assuming them to be ideal (Universal gas constant temperature at A is TA = 300 K.Calculate the temperature of
R = 8.314 J/mol – K). (1994 - 6 Marks) the gas at the points B and D and find the efficiency of the
27. A gaseous mixture enclosed in a vessel of volume V consists cycle. (1997 - 5 Marks)
of one mole of a gas A with g (=Cp / Cv) = 5 / 3 and another 33. The apparatus shown in the figure consists of four glass
gas B with g = 7/5 at a certain temperature T. The relative columns connected by horizontal sections. The height of
molar masses of the gases A and B are 4 and 32, respectively. two central columns B and C are 49 cm each. The two outer
The gases A and B do not react with each other and are columns A and D are open to the atmosphere. A and C are
assumed to be ideal. The gaseous mixture follows the maintained at a temperature of 95° C while the columns B
equation PV19/13 = constant, in adiabatic processes. and D are maintained at 5°C. The height of the liquid in A
(1995 - 10 Marks) and D measured from the base the are 52.8 cm and 51cm
(a) Find the number of moles of the gas B in the gaseous respectively. Determine the coefficient of thermal expansion
mixture. of the liquid. (1997 - 5 Marks)
(b) Compute the speed of sound in the gaseous mixture at
T = 300 K.
(c) If T is raised by 1K from 300 K, find the % change in
the speed of sound in the gaseous mixture.
(d) The mixture is compressed adiabatically to 1/5 of its
initial volume V. Find the change in its adiabatic A B C D
compressibility in terms of the given quantities. 95°C 5°C 95°C 5°C
28. At 27°C two moles of an ideal monoatomic gas occupy a
volume V. The gas expands adiabaticallly to a volume 2V.
Calculate (i) the final temperature of the gas, (ii) change in 34. A thin rod of negligible mass and area of cross-section
its internal energy, and (iii) the work done by the gas during 4 × 10–6 m2, suspended vertically from one end, has a length
this process. (1996 - 5 Marks) of 0.5 m at 100°C. The rod is cooled to 0°C, but prevented
29. The temperature of 100g of water is to be raised from 24°C to from contracting by attaching a mass at the lower end. Find
90°C by adding steam to it. Calculate the mass of the steam (i) this mass, and (ii) the energy stored in the rod. Given for
required for this purpose. (1996 - 2 Marks) the rod, Young’s modulus = 1011 N/m2, Coefficient of linear
30. A double-pane window used for insulating a room thermally expansion 10–5 K–1 and g = 10 m/s2. (1997C - 5 Marks)
from outside consists of two glass sheets each of area 1m 2 35. One mole of an ideal monatomic gas is taken round the
and thickness 0.01 m separated by a 0.05 m thick stagnant cyclic process ABCA as shown in Figure. Calculate
air space. In the steady state, the room glass inter-face and (1998 - 8 Marks)
the glass -outdoor interface are at constant temperatures of
P
27°C and 0°C respectively. Calculate the rate of heat flow
through the window pane. Also find the temperatures of B
3P0
other interfaces. Given thermal conductivities of glass and
air as 0.8 and 0.08 W m–1 K–1 respectively. (1997C - 5 Marks)
31. A sample of 2 kg of monatomic Helium (assumed ideal) is
taken through the process ABC and another sample of 2 kg
of the same gas is taken through the process ADC (see fig) P0
Given molecular mass of Helium = 4) (1997C - 5 Marks) A C
P
4 2 B C V0 2V0 V
(× 10 N/m )10
(a) the work done by the gas.
5 (b) the heat rejected by the gas in the path CA and the
A D heat absorbed by the gas in the path AB;
(c) the net heat absorbed by the gas in the path BC;
(d) the maximum temperature attained by the gas during
10 20 V (m3) the cycle.
P-178 Topic-wise Solved Papers - PHYSICS
36. A solid body X of heat capacity C is kept in an atmosphere 41. A monoatomic ideal gas of two moles is taken through a
whose temperature is TA = 300 K. At time t = 0 the temperature cyclic process starting from A as shown in figure. The volume
of X is T0 = 400 K. It cools according to Newton’s law of
VB V
cooling. At time t1, its temperature is found to be 350 K. ratiosare = 2 and D = 4. If the temperature TA at A is
(1998 - 8 Marks) VA VA
At this time (t1), the body X is connected to a large box Y at 27oC. (2001-10 Marks)
atmospheric temperature TA, through a conducting rod of
length L, cross-sectional area A and thermal conductivity K. V VD D C
The heat capacity of Y is so large that any variation in its
temperature may be neglected. The cross-sectional area A
of the connecting rod is small compared to the surface area VB B
of X. Find the temperature of X at time t = 3t1.
VA A
37. Two moles of an ideal monatomic gas, initially at pressure
p1 and volume V1, undergo an adiabatic compression until O
TA TB
its volume is V2. Then the gas is given heat Q at constant
Calculate, T
volume V2. (1999 - 10 Marks)
(a) the temperature of the gas at point B,
(a) Sketch the complete process on a p – V diagram. (b) heat absorbed or released by the gas in each process,
(b) Find the total work done by the gas, the total change in (c) the total work done by the gas during the complete
its internal energy and the final temperature of the gas. cycle.
[Give your answer in terms of p1, V1, V2, Q and R] Express your answer in terms of the gas constant R.
38. Two moles of an ideal monatomic gas is taken through a 42. A cubical box of side 1 meter contains helium gas (atomic
cycle ABCA as shown in the P-T diagram. During the weight 4) at a pressure of 100 N/m2. During an observation
time of 1 second, an atom travelling with the root-mean-
process AB, pressure and temperature of the gas vary such
square speed parallel to one of the edges of the cube, was
that PT = Constant. If T1 = 300 K, calculate
found to make 500 hits with a particular wall, without any
(2000 - 10 Marks)
25
collision with other atoms. Take R = J/mol-K and
3
k = 1.38 × 10-23 J/K (2002 - 5 Marks)
(a) Evaluate the temperature of the gas.
(b) Evaluate the average kinetic energy per atom.
(c) Evaluate the total mass of helium gas in the box.
43. An insulated container containing monoatomic gas of molar
mass m is moving with a velocity v0. If the container is
suddenly stopped, find the change in temperature.
(2003 - 2 Marks)
(a) the work done on the gas in the process AB and 44. Hot oil is circulated through an insulated container with a
(b) the heat absorbed or released by the gas in each of the wooden lid at the top whose conductivity
processes. K = 0.149 J/(m-ºC-sec), thickness t = 5 mm, emissivity = 0.6.
Temperature of the top of the lid is maintained at Tl = 127ºC.
Give answer in terms of the gas constant R.
If the ambient temperature Ta = 27ºC. (2003 - 4 Marks)
39. An ice cube of mass 0.1 kg at 0oC is placed in an isolated
container which is at 227oC. The specific heat S of the
container varies with temperature T according to the
empirical relation S = A + BT, where A = 100 cal/kg-K and B =
2 ´ 10-2 cal/kg-K2. If the final temperature of the container is
27oC, determine the mass of the container. (Latent heat of
fusion of water = 8 ´ 104 cal/kg, Specific heat of water = 103 Ta = 27ºC
cal/kg-K). (2001-5 Marks) Hot Oil
40. A 5 m long cylindrical steel wire with radius 2 ´ 10-3 m is
suspended vertically from a rigid support and carries a Calculate :
bob of mass 100 kg at the other end. If the bob gets snapped, (a) rate of heat loss per unit area due to radiation from the
calculate the change in temperature of the wire ignoring lid.
(b) temperature of the oil.
radiation losses. (For the steel wire: Young’s modulus
= 2.1 ´ 1011 Pa; Density = 7860 kg/m3; Specific heat = 420 J/kg-K). (Given s = 17 ´ 10 -8 Wm -2 K -4 )
(2001-5 Marks) 3
HEAT & THERMODYNAMICS AND GASES P-179
45. A diatomic gas is enclosed in a vessel fitted with massless Ts
movable piston. Area of cross section of vessel is 1 m 2.
Initial height of the piston is 1 m (see the figure). The initial
T1 T2 Insulation
temperature of the gas is 300 K. The temperature of the gas
is increased to 400 K, keeping pressure constant, calculate
the new height of the piston. The piston is brought to its
Furnace
initial position with no heat exchange. Calculate the final
48. A cubical block of co-efficient of linear expansion as is
temperature of the gas. You can leave answer in fraction.
submerged partially inside a liquid of co-efficient of volume
(2004 - 2 Marks)
expansion gl. On increasing the temperature of the system
by DT, the height of the cube inside the liquid remains
1m2 unchanged. Find the relation between as and gl.
(2004 - 4 Marks)
49. A cylinder of mass 1 kg is given heat of 20,000J at
atmospheric pressure. If initially the temperature of cylinder
1m
is 20°C, find (2005 - 6 Marks)
(a) final temperature of the cylinder.
46. A small spherical body of radius r is falling under gravity in (b) work done by the cylinder.
a viscous medium. Due to friction the medium gets heated. (c) change in internal energy of the cylinder
How does the rate of heating depends on radius of body (Given that specific heat of cylinder = 400 J kg–1 °C–1,
when it attains terminal velocity? (2004 - 2 Marks) coefficient of volume expansion = 9 × 10–5 °C–1, Atmospheric
47. A cylindrical rod of length l, thermal conductivity K and pressure = 105 N/m2 and Density of cylinder = 9000 kg/m3)
area of cross section A has one end in the furnace at 50. 0.05 kg steam at 373 K and 0.45 kg of ice at 253K are mixed in
temperature T1 and the other end in surrounding at an insulated vessel. Find the equilibrium temperature of the
temperature T 2 . Surface of the rod exposed to the mixture. Given, Lfusion = 80 cal/g = 336 J/g, Lvaporization = 540
surrounding has emissivity e. Also T2 = Ts + DT and Ts >> DT. cal/g = 2268 J/g, Sice = 2100 J/Kg K = 0.5 cal/gK and Swater =
If T1 – Ts µ DT, find the proportionality constant. 4200 J/Kg K = 1 cal/gK (2006 - 6M)
(2004 - 4 Marks)

MATCH THE FOLLOWING :


MUTLIPLE CHOICE QUESTIONS WITH ONE CORRECT
Direction (Q. No. 1 to Q.No. 3) : Each question contains statements given in two columns, which have to be matched. The
statements in Column-I are labelled A, B, C and D, while the statements in Column-II are labelled p, q, r, s and t. Any given
statement in Column-I can have correct matching with ONE OR MORE statement(s) in Column-II. The appropriate bubbles
corresponding to the answers to these questions have to be darkened as illustrated in the following example :
If the correct matches are A-p, s and t; B-q and r; C-p and q; and D-s then the correct darkening of bubbles will look like the given.
p q r s t
A p q r s t
B p q r s t
C p q r s t
D p q r s t

1. Heat given to process is positive, match the following option of Column I with the corresponding option of column II :
Column I Column II (2006 - 6M)
(A) JK (p) DW > 0
P(atm)
(B) KL (q) DQ < 0
J
(C) LM (r) DW < 0 30
M
(D) MJ (s) DQ > 0 20

10 K L
3
V(m )
10 20
P-180 Topic-wise Solved Papers - PHYSICS
2. Column I contains a list of processes involving expansion of an ideal gas. Match this with Column II describing the thermodynamic
change during this process. Indicate your answer by darkening the appropriate bubbles of the 4 × 4 matrix given in the ORS.
Column I Column II (2008)
(A) An insulated container has two chambers (p) The temperature of the gas decreases
separated by a valve. Chamber I contains an ideal gas and the
Chamber II has vacuum. The valve is opened.

I II

Ideal gas vacuum

(B) An ideal monoatomic gas expands to twice its remains (q) The temperature of the gas increases or
original volume such that its pressure P µ 1/V2 where V constant
is the volume of the gas
(C) An ideal monoatomic gas expands to twice its original volume (r) The gas loses heat
such that its pressure P µ 1/V4/3 where V is its volume
(D) An ideal monoatomic gas expands such that its (s) The gas gains heat
pressure P and volume V follows the behaviour
shown in the graph
P

V
V1 2V1

3. One mole of a monatomic gas is taken through a cycle


P
ABCDA as shown in the P-V diagram. Column II give the F
characteristics involved in the cycle. Match them with each 32 P0
of the processes given in Column I. (2011)

P0 G
E H
V
V0
Column I Column II
Match the paths in List I with the magnitudes of the work
(A) Process A ® B (p) Internal energy decreases done in List II and select the correct answer using the codes
(B) Process B ® C (q) Internal energy increases given below the lists.
(C) Process C ® D (r) Heat is lost List I List II
(D) Process D ® A (s) Heat is gained P. G ® E 1. 160 P0V0 ln2
(t) Work is done on the gas Q. G ® H 2. 36 P0V0
R. F ® H 3. 24 P0V0
DIRECTIONS Q. No. 4 : Following question has matching lists.
S. F ® G 4. 31 P0V0
The codes for the list have choices (a), (b), (c) and (d) out of
Codes:
which ONLY ONE is correct.
P Q R S
4. One mole of a monatomic ideal gas is taken along two cyclic (a) 4 3 2 1
processes E ® F ® G ® E and E ® F ® H ® E as shown in (b) 4 3 1 2
the PV diagram. The processes involved are purely (c) 3 1 2 4
isochoric, isobaric, isothermal or adiabatic.(JEE Adv. 2013) (d) 1 3 2 4
HEAT & THERMODYNAMICS AND GASES P-181
PASSAGE – 2
COMPREHENSION BASED Q UESTIONS :
A small spherical monoatomic ideal gas bubble (g = 5/3) is trapped
PASSAGE – 1 inside a liquid of density r (see figure). Assume that the bubble
A fixed thermally conducting cylinder has a radius R and height does not exchange any heat with the liquid. The bubble contains
L0. The cylinder is open at its bottom and has a small hole at its n moles of gas. The temperature of the gas when the bubble is at
top. A piston of mass M is held at a distance L from the top the bottom is T0, the height of the liquid is H and the atmospheric
surface, as shown in the figure. The atmospheric pressure is P 0. pressure is P0 (Neglect surface tension). (2008)
2R P0
L
Liquid

L0 H
y

4. As the bubble moves upwards, besides the buoyancy force


Piston the following forces are acting on it
(a) Only the force of gravity
1. The piston is now pulled out slowly and held at a distance
2L from the top. The pressure in the cylinder between its (b) The force due to gravity and the force due to the
top and the piston will then be (2007) pressure of the liquid
(c) The force due to gravity, the force due to the pressure
P0 of the liquid and the force due to viscosity of the liquid
(a) P0 (b)
2 (d) The force due to gravity and the force due to viscosity
P0 Mg P0 Mg of the liquid
(c) ∗ (d) , 5. When the gas bubble is at a height y from the bottom, its
2 οR 2 2 οR 2
Therefore the pressure inside the cylinder is P0 throughout temperature is –
the slow pulling process. 2/5
2. While the piston is at a distance 2L from the top, the hole at æ P + rl gH ö
(a) T0 ç 0
the top is sealed. The piston is then released, to a position è P0 + rl gy ÷ø
where it can stay in equilibrium. In this condition, the
distance of the piston from the top is (2007) 2/5
æ P + rl g (H - y) ö
æ 2P οR 2 ö÷ æ P οR 2 , Mg ÷ö (b) T0 ç 0
(a) çç 0 ÷÷ (2L) (b) ççç 0 ÷÷ (2L) è P0 + rl gH ÷ø
çç 2 ÷ çè οR 2 P ÷÷
è οR P0 ∗ Mg ÷ø 0 ø
3/5
æ P + rl gH ö
æ P οR 2 ∗ Mg ÷ö æ P οR 2 ÷ö (c) T0 ç 0
çç 0 ÷÷ (2L) (d) ççç 0 ÷÷ (2L) è P + r gy ÷ø
l
(c) çç ÷ ÷
0
è οR P0 ÷ø èç οR P , Mg ÷ø
2 2
0
3/5
3. The piston is taken completely out of the cylinder. The hole æ P + rl g (H - y) ö
(d) T0 ç 0
at the top is sealed. A water tank is brought below the cylinder è P + r gH ÷ø
0 l
and put in a position so that the water surface in the tank is
at the same level as the top of the cylinder as shown in the 6. The buoyancy force acting on the gas bubble is (Assume R
figure. The density of the water is r. In equilibrium, the height is the universal gas constant)
H of the water column in the cylinder satisfies (2007)
(P0 + rl gH) 2 / 5
(a) rl nRgT0
(P0 + rl gy)7 / 5

rl nRgT0
L0 (b) 2/5
(P0 + rl gH) [P0 + rl g (H - y)]3/ 5
H
(P0 + rl gH)3 / 5
(c) rl nRgT0
(a) rg(L0 – H)2 + P0(L0 – H) + L0P0 = 0 (P0 + rl gy)8 / 5
(b) rg(L0 – H)2 – P0(L0 – H) – L0P0 = 0
rl nRgT0
(c) rg(L0 – H)2 + P0(L0 – H) – L0P0 = 0 (d)
3/ 5
(P0 + rl gH) [P0 + rlg (H - y)]2 / 5
(d) rg(L0 – H)2 – P0(L0 – H) + L0P0 = 0
P-182 Topic-wise Solved Papers - PHYSICS
2. A piece of ice (heat capacity = 2100 J kg–1 °C–1 and latent heat
ASSERTION & REASON TYPE QUESTIONS : = 3.36 × 105 J kg–1) of mass m grams is at –5°C at atmospheric
1. Statement-1 : The total translational kinetic energy of all pressure. It is given 420 J of heat so that the ice starts melting.
the molecules of a given mass of an ideal gas is 1.5 times Finally when the ice-water mixture is in equilibrium, it is found
the product of its pressure and its volume. (2007) that 1 gm of ice has melted. Assuming there is no other heat
because exchange in the process, the value of m is (2010)
Statement-2 : The molecules of a gas collide with each other 3. Two spherical bodies A (radius 6 cm ) and B(radius 18 cm )
and the velocities of the molecules change due to the are at temperature T1 and T2, respectively. The maximum
collision. intensity in the emission spectrum of A is at 500 nm and in
that of B is at 1500 nm. Considering them to be black bodies,
(a) Statement-1 is True, Statement-2 is True; Statement-2
what will be the ratio of the rate of total energy radiated by
is a correct explanation for Statement-1
A to that of B? (2010)
(b) Statement-1 is True, Statement-2 is True; Statement-2
is NOT a correct explanation for Statement-1 1
4. A diatomic ideal gas is compressed adiabatically to of
(c) Statement-1 is True, Statement-2 is False 32
(d) Statement-2 is False, Statement-2 is True its initial volume. If the initial temperature of the gas is Ti (in

INTEGERVALUECORRECTTYPE: Kelvin) and the final temperature is a Ti , the value of a is


(2010)
1. A metal rod AB of length 10x has its one end A in ice at 0.°C, 5. Steel wire of lenght ‘L’ at 40°C is suspended from the ceiling
and the other end B in water at 100 °C. If a point P onthe rod and then a mass ‘m’ is hung from its free end. The wire is
is maintained at 400 °C, then it is found that equal amounts cooled down from 40°C to 30°C to regain its original length
of water and ice evaporate and melt per unit time. The latent ‘L’. The coefficient of linear thermal expansion of the steel is
heat of evaporation of water is 540 cal/g and latent heat of 10–5 /° C, Young’s modulus of steel is 1011 N/m2 and radius
melting of ice is 80 cal/g. If the point P is at a distance of of the wire is 1 mm. Assume that L >>diameter of the wire.
l x from the ice end A, find the value l. Then the value of ‘m’ in kg is nearly (2011)
[Neglect any heat loss to the surrounding.] (2009)

1. Which statement is incorrect? [2002] 7. At what temperature is the r.m.s velocity of a hydrogen
(a) all reversible cycles have same efficiency molecule equal to that of an oxygen molecule at 47°C?
(b) reversible cycle has more efficiency than an irreversible (a) 80 K (b) –73 K [2002]
one (c) 3 K (d) 20 K.
(c) Carnot cycle is a reversible one 8. Even Carnot engine cannot give 100% efficiency because
(d) Carnot cycle has the maximum efficiency in all cycles. we cannot [2002]
2. Heat given to a body which raises its temperature by 1°C is (a) prevent radiation
(a) water equivalent (b) thermal capacity [2002] (b) find ideal sources
(c) specific heat (d) temperature gradient (c) reach absolute zero temperature
3. Infrared radiation is detected by [2002] (d) eliminate friction.
(a) spectrometer (b) pyrometer 9. 1 mole of a gas with g = 7/5 is mixed with 1 mole of a gas
(c) nanometer (d) photometer
with g = 5/3, then the value of g for the resulting mixture is
4. Which of the following is more close to a black body?
(a) 7/5 (b) 2/5 [2002]
(a) black board paint (b) green leaves [2002]
(c) black holes (d) red roses (c) 24/16 (d) 12/7.
10. Two spheres of the same material have radii 1 m and 4 m and
5. Cooking gas containers are kept in a lorry moving with
temperatures 4000 K and 2000 K respectively. The ratio of
uniform speed. The temperature of the gas molecules inside
will [2002] the energy radiated per second by the first sphere to that by
the second is [2002]
(a) increase
(a) 1 : 1 (b) 16 : 1
(b) decrease
(c) remain same (c) 4 : 1 (d) 1 : 9.
11. “Heat cannot by itself flow from a body at lower temperature
(d) decrease for some, while increase for others
to a body at higher temperature” is a statement or
6. If mass-energy equivalence is taken into account, when
water is cooled to form ice, the mass of water should consequence of [2003]
(a) second law of thermodynamics
(a) increase [2002]
(b) conservation of momentum
(b) remain unchanged
(c) decrease (c) conservation of mass
(d) first law of thermodynamics
(d) first increase then decrease
HEAT & THERMODYNAMICS AND GASES P-183
12. During an adiabatic process, the pressure of a gas is found 21. The temperature of the two outer surfaces of a composite
to be proportional to the cube of its absolute temperature. slab, consisting of two materials having coefficients of
The ratio CP / CV for the gas is [2003] thermal conductivity K and 2K and thickness x and 4x,
respectively, are T2 and T1 (T2 > T1 ) . The rate of heat transfer
4
(a) (b) 2 æ A(T2 - T1 ) K ö
3
through the slab, in a steady state is ç
è ÷ø f ,
x
5 3
(c) (d) with f equal to [2004]
3 2
x 4x
13. Which of the following parameters does not characterize
the thermodynamic state of matter? [2003]
(a) Temperature (b) Pressure
(c) Work (d) Volume
14. A Carnot engine takes 3´ 10 6 cal. of heat from a reservoir T2 2K T1
K
at 627 o C , and gives it to a sink at 27 o C . The work done
by the engine is [2003]
(a) 4.2 ´ 10 6 J (b) 8.4 ´ 10 6 J
2 1
(c) 16.8 ´ 10 6 J (d) zero (a) (b)
15. The earth radiates in the infra-red region of the spectrum. 3 2
The spectrum is correctly given by [2003] 1
(a) Rayleigh Jeans law (c) 1 (d)
3
(b) Planck’s law of radiation 22. Which of the following is incorrect regarding the first law
(c) Stefan’s law of radiation of thermodynamics? [2005]
(d) Wien’s law (a) It is a restatement of the principle of conservation of
16. According to Newton’s law of cooling, the rate of cooling energy
of a body is proportional to ( Dq)n , where Dq is the (b) It is not applicable to any cyclic process
(c) It introduces the concept of the entropy
difference of the temperature of the body and the (d) It introduces the concept of the internal energy
surroundings, and n is equal to [2003] 23. The figure shows a system of two concentric spheres of
(a) two (b) three (c) four (d) one radii r1 and r2 are kept at temperatures T1 and T2, respectively.
17. One mole of ideal monatomic gas ( g = 5 / 3) is mixed with The radial rate of flow of heat in a substance between the
two concentric spheres is proportional to [2005]
one mole of diatomic gas ( g = 7 / 5) . What is g for the
mixture? g Denotes the ratio of specific heat at constant
pressure, to that at constant volume [2004] r1
(a) 35/23 (b) 23/15 T1
(c) 3/2 (d) 4/3
18. If the temperature of the sun were to increase from T to 2T r2 T2
and its radius from R to 2R, then the ratio of the radiant
energy received on earth to what it was previously will be ær ö ( r2 - r1 )
(a) In ç 2 ÷ (b)
(a) 32 (b) 16 [2004] è r1 ø ( r1 r2 )
(c) 4 (d) 64 r1 r2
19. Which of the following statements is correct for any (c) ( r2 - r1 ) (d)
( r2 - r1 )
thermodynamic system ? [2004]
(a) The change in entropy can never be zero 24. A system goes from A to B via two processes I and II as
(b) Internal energy and entropy and state functions shown in figure. If DU1 and DU2 are the changes in internal
(c) The internal energy changes in all processes energies in the processes I and II respectively, then [2005]
(d) The work done in an adiabatic process is always zero. p II
20. Two thermally insulated vessels 1 and 2 are filled with air at
temperatures (T1 , T2 ), volume (V1, V2 ) and pressure A B
I
( P1, P2 ) respectively. If the valve joining the two vessels is
opened, the temperature inside the vessel at equilibrium will v
be [2004]
(a) relation between DU1 and DU 2 can not be determined
(a) T1T2 ( PV 1 1 + P V
2 2 ) /( PV T
1 1 2 + P V T
2 2 1 )
(b) (T1 + T2 ) / 2 (b) DU1 = DU 2
(c) T1 + T2 (c) DU 2 < DU1
(d) T1T2 ( PV 1 1 + P2V2 ) /( PV
1 1T1 + P2V2T2 ) (d) DU 2 > DU1
P-184 Topic-wise Solved Papers - PHYSICS
25. The temperature-entropy diagram of a reversible engine 31. A Carnot engine, having an efficiency of h = 1/10 as heat
cycle is given in the figure. Its efficiency is [2005] engine, is used as a refrigerator. If the work done on the
T system is 10 J, the amount of energy absorbed from the
reservoir at lower temperature is [2007]
2T0 (a) 100 J (b) 99 J
(c) 90 J (d) 1 J
T0 32. One end of a thermally insulated rod is kept at a temperature
T1 and the other at l2. The rod is composed of two sections
S of length l1 and l2 and thermal conductivities K1 and K2
2S0 S0 respectively. The temperature at the interface of the two
1 1 section is [2007]
(a) (b) T1 l1 l2 T2
4 2
2 1
(c) (d)
3 3
K1 K2
26. A gaseous mixture consists of 16 g of helium and 16 g of
Cp ( K1l1T1 + K 2 l2T2 ) ( K 2l2T1 + K1l1T2 )
oxygen. The ratio of the mixture is [2005] (a) (b)
Cv ( K1l1 + K 2 l2 ) ( K1l1 + K 2 l2 )
(a) 1.62 (b) 1.59
( K 2l1T1 + K1l2T2 ) ( K1l2T1 + K 2l1T2 )
(c) 1.54 (d) 1.4 (c) (d)
27. Assuming the Sun to be a spherical body of radius R at a ( K 2 l1 + K1l2 ) ( K1l2 + K 2 l1 )
temperature of TK, evaluate the total radiant powerd incident 33. If CP and CV denote the specific heats of nitrogen per unit
of Earth at a distance r from the Sun [2006] mass at constant pressure and constant volume respectively,
T4 T4 then [2007]
(a) 4pr02 R 2 s (b) pr02 R 2 s (a) CP – CV = 28R (b) CP – CV = R/28
r2 r2 (c) CP – CV = R/14 (d) CP – CV = R
T4 T4 34. The speed of sound in oxygen (O2) at a certain temperature
(c) r02 R 2 s (d) R2 s is 460 ms–1. The speed of sound in helium (He) at the same
2 2
4 pr r
where r0 is the radius of the Earth and s is Stefan's constant. temperature will be (assume both gases to be ideal) [2008]
28. Two rigid boxes containing different ideal gases are placed (a) 1421 ms–1 (b) 500 ms–1
(c) 650 ms –1 (d) 330 ms–1
on a table. Box A contains one mole of nitrogen at temperature
T0, while Box contains one mole of helium at temperature 35. An insulated container of gas has two chambers separated
by an insulating partition. One of the chambers has volume
æ 7ö V1 and contains ideal gas at pressure P1 and temperature
çè ÷ø T0 . The boxes are then put into thermal contact with
3 T1. The other chamber has volume V2 and contains ideal
each other, and heat flows between them until the gases gas at pressure P2 and temperature T2. If the partition is
reach a common final temperature (ignore the heat capacity removed without doing any work on the gas, the final
of boxes). Then, the final temperature of the gases, Tf in equilibrium temperature of the gas in the container will be
terms of T0 is [2006]
3 7 1 1 + P2V2 )
T1T2 ( PV 1 1T1 + P2V2T2
PV
(a) T f = T0 (b) T f = T0 (a) (b) [2008]
7 3 1 1T2 + P2V2T1
PV 1 1 + P2V2
PV
3 5
(c) T f = T0 (d) T f = T0 PV1 1T2 + P2V2T1 1 1 + P2V2 )
T1T2 ( PV
2 2 (c) + (d)
29. The work of 146 kJ is performed in order to compress one
PV
1 1 P V
2 2 1 1 1 + P2V2T2
PV T
kilo mole of gas adiabatically and in this process the 36. A long metallic bar is carrying heat from one of its ends to
temperature of the gas increases by 7°C. The gas is the other end under steady–state. The variation of
(R = 8.3 J mol–1 K–1) [2006] temperature q along the length x of the bar from its hot end
(a) diatomic is best described by which of the following figures? [2009]
(b) triatomic q q
(c) a mixture of monoatomic and diatomic
(d) monoatomic
30. When a system is taken from state i to state f along the path (a) (b)
iaf, it is found that Q =50 cal and W = 20 cal. Along the path
ibf Q = 36 cal. W along the path ibf is [2007] x x
a f q q

i b (c) (d)
(a) 14 cal (b) 6 cal x x
(c) 16 cal (d) 66 cal
HEAT & THERMODYNAMICS AND GASES P-185
DIRECTIONS for questions 37 to 39 : Questions are based on 44. Three perfect gases at absolute temperatures T1, T2 and T3
the following paragraph. are mixed. The masses of molecules are m1, m2 and m3 and
Two moles of helium gas are taken over the cycle ABCDA, as the number of molecules are n1, n2 and n3 respectively.
shown in the P-T diagram. Assuming no loss of energy, the final temperature of the
mixture is : [2011]
5 A B n1T1 + n2T2 + n3T3
2 × 10 (a) n1 + n2 + n3

P (Pa) n1T12 + n2T22 + n3T32


(b)
n1T1 + n2T2 + n3T3
5
1 × 10 D C
T n12T12 + n22T22 + n32T32
300K 500K (c)
n1T1 + n2T2 + n3T3
37. Assuming the gas to be ideal the work done on the gas in
taking it from A to B is : (T1 + T2 + T3 )
(d)
(a) 300 R (b) 400 R 3
(c) 500 R (d) 200 R 45. A Carnot engine operating between temperatures T1 and T2
38. The work done on the gas in taking it from D to A is : 1
(a) + 414 R (b) – 690 R has efficiency . When T2 is lowered by 62 K its efficiency
6
(c) + 690 R (d) – 414 R
39. The net work done on the gas in the cycle ABCDA is : 1
increases to . Then T1 and T2 are, respectively: [2011]
(a) 276 R (b) 1076 R 3
(c) 1904 R (d) zero (a) 372 K and 330 K (b) 330 K and 268 K
40. One kg of a diatomic gas is at a pressure of 8 × 104N/m2. The (c) 310 K and 248 K (d) 372 K and 310 K
density of the gas is 4kg/m3. What is the energy of the gas 46. 100g of water is heated from 30°C to 50°C. Ignoring the
due to its thermal motion? [2009] slight expansion of the water, the change in its internal energy
(a) 5 × 104 J (b) 6 × 104 J is (specific heat of water is 4184 J/kg/K): [2011]
(c) 7 × 104 J (d) 3 × 104 J (a) 8.4 kJ (b) 84 kJ
41. Statement 1 : The temperature dependence of resistance is (c) 2.1 kJ (d) 4.2 kJ
usually given as R = Ro (1 + a Dt). The resistance of a wire 47. The specific heat capacity of a metal at low temperature (T)
changes from 100W to 150W when its temperature is is given as
increased from 27°C to 227°C. This implies th at 3
æ T ö
a = 2.5 × 10–3/°C. C p (kJK -1kg -1 ) = 32 ç
Statement 2 : R = Ro (1+ a Dt) is valid only when the change è 400 ÷ø
in the temperature DT is small and A 100 gram vessel of this metal is to be cooled from 20ºK
DR = (R – R0) << Ro. [2009] to 4ºK by a special refrigerator operating at room
(a) Statement-1 is true, Statement-2 is true; Statement-2 is temperature (27°C). The amount of work required to cool
the correct explanation of Statement-1. the vessel is [2011RS]
(b) Statement-1 is true, Statement-2 is true; Statement-2 is (a) greater than 0.148 kJ
not the correct explanation of Statement-1. (b) between 0.148 kJ and 0.028 kJ
(c) Statement-1 is false, Statement-2 is true. (c) less than 0.028 kJ
(d) Statement-1 is true, Statement-2 is false. (d) equal to 0.002 kJ
48. A container with insulating walls is divided into equal parts
42. A diatomic ideal gas is used in a Carnot engine as the working by a partition fitted with a valve. One part is filled with an
substance. If during the adiabatic expansion part of the cycle ideal gas at a pressure P and temperature T, whereas the
the volume of the gas increases from V to 32 V, the efficiency other part is completly evacuated. If the valve is suddenly
of the engine is [2010] opened, the pressure and temperature of the gas will be :
(a) 0.5 (b) 0.75 P T
(c) 0.99 (d) 0.25 (a) , (b) P, T [2011RS]
2 2
43. A thermally insulated vessel contains an ideal gas of
molecular mass M and ratio of specific heats g. It is moving T P
with speed v and it's suddenly brought to rest. Assuming (c) P, (d) ,T
2 2
no heat is lost to the surroundings, its temperature increases 49. An aluminium sphere of 20 cm diameter is heated from 0°C
by: [2011] to 100°C. Its volume changes by (given that coefficient of
( g - 1) Mv2 K gM 2v linear expansion for aluminium a Al = 23 ´ 10-6/ ° C )
(a) 2 gR
(b) K
2R [2011RS]
( g - 1) ( g - 1) 2 (a) 2.89 cc (b) 9.28 cc
(c) Mv 2 K (d) 2( g + 1) R Mv K (c) 49.8 cc (d) 28.9 cc
2R
P-186 Topic-wise Solved Papers - PHYSICS
50. A wooden wheel of radius R is made of two semicircular part
(see figure). The two parts are held together by a ring made
of a metal strip of cross sectional area S and length L. L is

loge (q – q0)

loge (q – q0)
slightly less than 2pR. To fit the ring on the wheel, it is
heated so that its temperature rises by DT and it just steps
over the wheel. As it cools down to surrounding temperature, (c) (d)
it presses the semicircular parts together. If the coefficient
of linear expansion of the metal is a, and its Young's modulus 0 0
t
t
is Y, the force that one part of the wheel applies on the other
part is : 53. A Carnot engine, whose efficiency is 40%, takes in heat
[2012] from a source maintained at a temperature of 500K. It is
desired to have an engine of efficiency 60%. Then, the intake
temperature for the same exhaust (sink) temperature must
be : [2012]
R (a) efficiency of Carnot engine cannot be made larger than
50%
(b) 1200 K
(c) 750 K
(d) 600 K

(a) (b) 54. p


2pSY aDT SY aDT
2p0 A B
(c) pSY aDT (d) 2SY aDT
51. Helium gas goes through a cycle ABCDA (consisting of p0
two isochoric and isobaric lines) as shown in figure D C
Efficiency of this cycle is nearly : (Assume the gas to be
close to ideal gas) [2012] v0 2v0 v

The above p-v diagramrepresents the thermodynamic cycle


of an engine, operatingwith an ideal monatomic gas. The
B C amount of heat, extracted from the source in a single cycle is
2P0
[JEE Main 2013]

P0 D æ 13 ö
A (a) p 0v 0 (b) ç ÷ p0 v0
è2ø

æ 11 ö
V0 2V0 (c) ç ÷ p0 v0 (d) 4p0v0
è2ø
55. If a piece of metal is heated to temperature q and then allowed
(a) 15.4 % (b) 9.1 %
to cool in a room which is at temperature q0, the graph between
(c) 10.5% (d) 12.5 % the temperature T of the metal and time t will be closest to
52. A liquid in a beaker has temperature q(t) at time t and q0 is [JEE Main 2013]
temperature of surroundings, then according to Newton's
law of cooling the correct graph between log e (q - q0 ) and T
T
t is : [2012] q0
(a) (b)

O t O t
loge (q – q0)
loge (q – q0)

T T
(a) (b) q0 q0
(c) (d)
0 0
t t O t O t
HEAT & THERMODYNAMICS AND GASES P-187

Solutions & Explanations


Section-A : JEE Advanced/ IIT-JEE
A 1. 4 cal. 2. Partly solid and partly liquid. 3. 2T 4. 0°C 5. 5803K

4pR 2 KT P´t
6. 1.71rrc 7. t= 8. Lfusion = 9. 300 K
P m
10. 0.628 11. 60°C 12. 192º 13. 45.6°C 14. 5.5 min.
B 1. F 2. F 3. F 4. F 5. T 6. T
7. F 8. F
C 1. (d) 2. (c) 3. (b) 4. (a) 5. (b) 6. (a) 7. (b) 8. (d)
9. (b) 10. (d) 11. (c) 12. (c) 13. (d) 14. (b) 15. (d) 16. (c)
17. (d) 18. (a) 19. (c) 20. (a) 21. (b) 22. (b) 23. (a) 24. (b)
25. (a) 26. (a) 27. (a) 28. (a) 29. (c) 30. (c) 31. (b) 32. (b)
33. (b) 34. (c) 35. (d) 36. (b) 37. (a) 38. (d) 39. (d) 40. (a)
41. (c) 42. (b) 43. (c) 44. (d) 45. (a) 46. (d) 47. (d) 48. (a)
49. (d)
D 1. (a) 2. (b) 3. (a) 4. (c) 5. (a, b, c, d) 6. (d) 7. (b)
8. (a, b, d) 9. (a, b) 10. (b) 11. (b) 12. (d) 13. (b, c) 14. (d)
15. (b, d) 16. (c) 17. (c, d) 18. (a) 19. (a, c, d) 20. (b, d) 21. (b, d)
22. (a, b) 23. (a, c, d) 24. (a, b, c, d)
w2 - w1 b( w0 - w1 )
E 1. + 2. TB = 30°C, TC = TD = 20°C 3. 15.24°C
( w0 - w2 )(t2 - t1 ) ( w0 - w2 )
4. Same 5. 1.1 × 1011 N/m2; 2 × 10–5 °C–1 7. 817 mmHg 9. 12.96 m/s 10. –973.1 J
11. hollow sphere 27
12. 1.97 × 10 , 35.6 m/s 13. 12.9 T0, 2.25 T0, –15.58T0
14. 83.75 cm Hg 15. 9000 W 16. 75.4 cm 17. 675 K, 3.6 × 106 N/m2
18. (ii) 113 l, 0.44 × 105 N/m2 (iii) 12450 J 19. 800 K, 720 J 20. (i) 5 (ii) 1.25 PV
21. (b) 0.58 RTA, 0.58 RTA 22. (a) 1153 J (b) 1153J (c) Zero 23. 166.38 secs
24. (i) 1870 J (ii) –5298 J (iii) 500 K 25. (i) 765 J (ii) 10.82%
-5
26. Mass of Neon = 4gm, mass of Argon = 24gm 27. (a) 2 mole (b) 400.03 m/s (c) 1/6 (d) -8.27 ´10 V
28. (i) 189 K (ii) –2767 J (iii) 2767 J 29. 0.0122 Kg 30. 41.54 J/s, 26.48°C, 0.52°C
31. (i) TA = 120.3 K, TB = 240.6 K, TC = 481.2 K, TD = 240.6 K (ii) No (iii) QABC = 3.25 × 106 J, QADC = 2.75 × 106 J
32. TB = 909 K, TD = 791 K, 61.4% 33. 6.67 × 10–5 per °C 34. (i) 40 Kg (ii) 0.1 J
é -2 KAt1 ù
5 P0V0 25 P0V0
35. (a) P0V0 (b) - P0V0 , 3P0V0 (c) (d) 36. ëê 300 + 12.5e CL ûú Kelvin
2 2 8R

3 é æ V ö2/3ù 3 é æ V ö 2/3ù 5 / 3 -2 / 3
PV ê1 - 1 ú , Q - 1 1 ê1 - ç
PV 1 ú PV1 1 V2 Q
37. (b) 2 1 1 ê èç V ø÷ ú 2 ê è V2 ø ÷ ú , + 38. (a) 1200R (b) –2100R, 831.6 R
ë 2 û ë û 2 R 3 R
39. 0.495 Kg 40. 0.00457°C 41. (a) 600 K (b) 1500R, 831.8R, –900R, –831.8R (c) 600R
42. (a) 160K (b) 3.312 × 10–21 J (c) 0.3012 gm

mv02 4 2/5
m, 400 æç ö÷ K
4
43. 44. (a) 595 W/m2 (b) 419.83 K 45. 46. Rate of heat produced µ r 5
3R 3 è3ø

é 4eslTs3 ù
47. ê1 + ú 48. g = 2a 49. 69.99°C, 0.0499J, 19999.95 J 50. 273K or 0°C
ëê K ûú
P-188 Topic-wise Solved Papers - PHYSICS
F 1. (A)-(q); (B)-(p, s); (C)-(s); (D)-(q, r) 2. (A)-(q); (B)-(p, r); (C)-(p, s); (D)-(q, s)
3. (A)-(p, r, t); (B)-(p, r); (C)-(q, s); (D)-(r, t) 4. (a)
G 1. (a) 2. (d) 3. (c) 4. (d) 5. (b) 6. (b)
H. 1. (b)
I. 1. 9 2. 8 3. 9 4. 4 5. 3
Section-B : JEE Main/ AIEEE
1. (a) 2. (b) 3. (b) 4. (a) 5. (a) 6. (c) 7. (d)
8. (c) 9. (c) 10. (a) 11. (a) 12. (d) 13. (c) 14. (b)
15. (d) 16. (d) 17. (c) 18. (d) 19. (b) 20. (a) 21. (d)
22. (b, c) 23. (d) 24. (b) 25. (d) 26. (a) 27. (b) 28. (c)
29. (a) 30. (b) 31. (c) 32. (d) 33. (b) 34. (a) 35. (a)
36. (a) 37. (b) 38. (a) 39. (a) 40. (a) 41. (c) 42. (b)
43. (c) 44. (a) 45. (d) 46. (a) 47. (d) 48. (d) 49. (d)
50. (d) 51. (a) 52. (a) 53. (c) 54. (b) 55. (c)

FILL IN THE BLANKS : \ The total energy released per second per unit surface
area of the Sun
1. KEY CONCEPT :
1400 ´ 4p ´ (1.5 ´ 1011 ) 2
3 =
Cv for monoatomic gas = RT 4 p ´ (7 ´ 108 ) 2
2
This energy E is also equal to E = sT4
5
Cv for diatomic gas = RT 1
2 é 1400 ´ 4 p ´ (1.5 ´ 1011 ) 2 ù 4
\ T= ê 8 2 -8
ú » 5803 K
3 5 ëê 4 p ´ (7 ´ 10 ) ´ 5.67 ´ 10 ûú
RT + RT
2 2 6. The energy emitted per second when the temperature of the
Therefore average = = 2R = 4 cal.
2 copper sphere is T and the surrounding temperature T0
2. AB represent a process when physical state changes from
= s (T4 – T04 ) × A = s .T4 A [Q T0 = 0] ...(i)
solid to liquid and the temperature remains unchanged.
Since P is a point between A and B, therefore the material is We know that
partly solid and partly liquid. dQ dT
dQ = mcdT Þ = mc ... (ii)
3. PV = RT (Ideal gas equation) dt dt
RT From (i) and (ii)
Þ P= ... (i) dT
V sT4A = mc
Given that VP2 = const ... (ii) dt
From (i) and (ii) 4
r ´ p r 3cdT
mcdT 3 é 4 3ù
T2 Þ dt = 4
= 4 2 êëQ m = r ´ 3 pr úû
\ = const. sT A s T ´ 4p r
V
rrc dT
T12 T22 V2 2V Þ dt =
\ = Þ T2 = T1 =T = 2T 3s T 4
V1 V2 V1 V Integrating both sides
4. The heat required for 100 g of ice at 0° C to change into 100
t rrc 100 dT rrc é 1 ù
water at 0°C = mL = 100 × 80 ×4.2 = 33,600 J ... (i) ò0 dt =
3s ò 200 T 4 3s êë 3T 3 úû 200
= -
The heat released by 300g of water at 25°C to change its
temperature to 0°C = mcDT = 300 × 4.2 × 25 = 31,500 J ... (ii)
Since the energy in eq. (ii) is less than of eq. (i) therefore the rrc é 1 1 ù
t= - ê - ú
final temperature will be 0°C. 9s ëê (100) 3
(200)3 ûú
5. The energy received per second per unit area from Sun at a
distance of 1.5 × 1011 m is 1400 J/sm2. The total energy 7rrc 7rrc
t= » = 1.71rrc
released by Sun/per second. 6
(72 ´ 10 ) s 72 ´ 10 (5.67 ´ 10 -8 )
6
= 1400 × 4p × (1.5 × 1011)2.
HEAT & THERMODYNAMICS AND GASES P-189
7. KEY CONCEPT : The heat transferred through B per second
When the spherical shell is thin, t << R. In this case, Q2= K2 A (t – 0)
At steady state K1 A (100 – t) = K2A (t – 0)
Þ 300 (100 – t) = 200 (t – 0) Þ 300 – 3t = 2t Þ t = 60° C
12. The movable stopper will adjust to a position with equal
R pressure on either sides. Applying ideal gas equation to the
t two gases, we get
m m
P PV1 = n1RT = RT , PV2 = n2RT = RT
M1 M2
K
V2 M1 32 8
Hence, = = =
V1 M 2 28 7
The rate of flow of heat from the sphere to the surroundings
360°
K (4pR 2 )T a= ´ 8 = 192°
P= (8 + 7)
t 13. In case of gas thermometer, the volume of the gas container
where T is the temperature difference and t is the thickness remains constant. Therefore
of steel then P1 P2 3 ´ 10 4 3.5 ´ 10 4
= Þ =
4pR 2 KT T1 T2 273.16 T2
t=
P Þ T2 = 318.6 K = (318.6 – 273) °C = 45.6°C
8. Since P joules per second of heat is supplied to keep the 14. Solar power received by earth = 1400 W/m 2
substance in molten state, it means that the substance in Solar power received by 0.2 m2 area
the molten state at its melting point releases P Joule of heat = (1400 W/m2) (0.2 m2) = 280 W
in one second. Mass of ice = 280 g = 0.280 kg
Heat required to melt ice
= (0.280) (3.3 × 105) = 9.24 × 104 J
If t is the time taken for the ice to melt, we will have
P Joule/sec.
é Eù
(280)t = 9.24 × 104 J êQ P = t ú
P Joule/second ë û
M
9.24 ´ 104
t= s = 330s º 5.5min
280
TRUE / FALSE :
1. KEY CONCEPT :
3RT
The power is turned off then the heat input becomes zero. Þ c =
M
But heat output continues. It takes t seconds for the
Þ At the same temperature
substance to solidify (given). Therefore total heat released
in t seconds = P × t = mLfusion 1

M
P ´t
Lfusion = i.e., dependent on molar mass and hence rms speed c will be
m different for different ideal gases.
9. In this expansion, no work is done because the gas expands 1
in vacuum. Therefore DW = 0 2. For a particular temperature T, V µ
P
As the process is a adiabatic, Q = 0. From first law of
Volume is greater for pressure P1
thermodynamics, DU = 0 i.e. temperature remains constant.
\ p1 < p2
10. For isothermal expansion
1
P 3. For a particular termperature Crms µ
P × V = Pi × 2V Þ Pi = M
2
i.e., Crms will have different values for different gases.
For adiabatic expansion
g 1 RT
P P
PVg = Pa × (2V)g Þ Pa = = (C H 2 )1 M1 g1 M 2
2g 21.67 4. = = ´
(C H e ) 2 g 2 RT g 2 M1
Pa P 2 2
\ = ´ = = 0.628 M2
Pi 2 1.67 P 2 1.67

11. The heat transferred through A per second 7/5 4 7 3 42


= ´ = ´ ´2 =
Q1= K1 A (100 – t) 5/3 2 5 5 25
P-190 Topic-wise Solved Papers - PHYSICS
5. The slope of P-V curve is more for adiabatic process than \ Average Cv will be
for isothermal process. From the graph it is clear that slope 1 é3 5 ù
for B is greater than the slope for A. Cv = ê R + R ú = 2 R
2 ë2 2 û
6. Cp – Cv = R
7. We know that \ C p = R + 2R = 3R [Q Cp – Cv = R]
3RT 3R (2T ) Cp 3R
v= then v' = \ g= = = 1.5
M M /2 Cv 2 R
\ v' = 2v ALTERNATE SOLUTION
8. Energy radiated per second by the first sphere n1 + n 2 n1 n
E1 = esT 4A = es (4000)4 × 4p × 11 × 1 = + 2
= 1024 × p × 1012 × es g -1 g1 - 1 g 2 - 1
Energy radiated per second by the second sphere 2 5
E2 = es × (2000)4 × 4p × 4 × 4 Here, n1 = n2 = 1, g1 = , g2 =
5 3
= 1024 p × 1012 × es 6. (a) For an ideal gas PV = nRT
E1 = E2 Þ Coefficient of volume expansion
æ DV ö nR
MCQ's WITH ONE CORRECT ANSWER : ç ÷ = = Constant
è DT øp P
1. (d) Note : At constant volume, Charle's law is used.
2. (c) W1 = mg – Vda g 3
Note : Average translation K.E. for O2 is kT
2
da g (Three degrees of freedom for translational motion).
W2 = mg – V'd'a g = mg – V (1 + 50 gb)
(1 + 50 g a ) Now decrease in pressure increases the volume.
Þ It increases mean free path of the molecules. Also
é 1 + 50 g b ù average K.E. does not depend on the gas, so molecules
= mg – V dag ê ú
ë1 + 50 g a û of each component of mixture of gases have same
average translational energy.
V = Volume of ball at 0° C 7. (b) Heat flow from B to A, A to C and C to B (for steady
da = density of alcohol at 0° C
state condition, DQ Dt is same)
V' = Volume of ball at 50° C
da' = density of alcohol at 0° C A
T
Given gb < ga
2a
1 + 50 g b a
\ 1 + 50 gb < 1 + 50 ga \ <1
1 + 50 g a TC
B a C
\ W2 > W1 or W1 < W2 2T
3. (b) q A - q B = 36°C (Given) DQ k A DT
Where =
KA = 2KB (Given) Dt l
KA KB æ DT ö
q + q æ DT ö
l A l B For sides AC and CB ç =
qC = è 2a ÷ø AC çè a ÷ø CB
KA KB
+
l l T - Tc Tc - 2 T
Þ = Þ T - Tc = 2 Tc - 2T
qA qC qB 2a a
Tc 3
Þ 3T = Tc ( 2 + 1) Þ =
KA KB T 2 +1
8. (d) According to Stefan's law
l l DQ = esAT4 Dt
2q A + q B 2q A + q A - 36 3(q A - 12) also, DQ = mc DT
\ qC = = = or, mc DT = esAT4 Dt
3 3 3
DT esAT 4 esT 4 é æ 3m ö
2/3ù
\ q A - qC = 12 or, = = êp ç ú
4. (a) The work done during the cycle = area enclosed in the Dt mc mc ê è 4 prø÷ úû
ë
curve 1/ 3
æ 1ö
3 = kç ÷
5. (b) For monoatomic gas, Cv = R è mø
2 1/ 3 1/ 3
DT1 / Dt1 æ m2 ö æ 1ö
5 \ = =ç ÷
For diatomic gas, Cv = R DT2 / Dt2 èç m1 ø÷ è 3ø
2
HEAT & THERMODYNAMICS AND GASES P-191
9. (b) According to Weins displacement law, 15. (d) The internal energy of n moles of a gas is
lmT = constant
1
(lm )S × TS = (lm )NS × TNS S = Sun; NS = North Star U= nFRT
2
TS (l ) 350
Þ = m NS = = 0.69 where F = number of degrees of freedom.
TNS (l m )S 510 Internal energy of 2 moles of oxygen at temperature T is
10. (d) The formula for average kinetic energy is 1
U1 = ´ 2 ´ 5RT = 5RT [F = 5 for oxygen molecule]
3 (K.E.)600K 600 2
K.E. = kT \ =
2 (K.E.)300K 300 Internal energy of 4 moles of argon at temperature T is
1
Þ (K.E.)600K = 2 × 6.21 ×10–21 J = 12.42 × 10–21 J U2 = ´ 4 ´ 3RT = 6RT
2
Also the formula for r.m.s. velocity is Total internal energy = 11 RT
3KT (C ) 600
Crms = \ rms 600K = VN g N 2 M He 7/5´ 4
m (Crms )300K 300 16. (c) = =
VHe g He M N 2 5 / 3 ´ 28
Þ (Crms)600K = 2 ´ 484 = 684 m / s
11. (c) Average translational kinetic energy of an ideal gas 7 3´ 4 3
= ´ =
3 5 5 ´ 28 5
molecule is kT which depends on temperature only.. 17. (d) Here TVg –1 = constant
2
Therefore, if temperature is same, translational kinetic 5
energy of O2 and N2 both will be equal. As g = , hence TV2/3 = constant
3
nRT
12. (c) PV = nRT or P = or P µ T Now T1 L12 / 3 = T2 L22 / 3 (Q V µ L);
V
(Q V and n are same.) 2/3
T1 æ L2 ö
Therefore, if T is doubled, pressure also becomes two Hence, =
times, i.e.., 2P. T2 çè L1 ÷ø
13. (d) The energy radiated per second by a black body is
given by Stefan's Law 18. (a)
E
= sT 4 ´ A , where A is the surface area.
t Temp.
E
= sT 4 ´ 4pr 2 ( Q For a sphere, A = 4 p r2) 100o 100o
t Water Steam
E 0o 0o
Case (i) : = 450, T = 500 K, r = 0.12 m Ice Water
t
\ 450 = 4ps (500)4 (0.12)2 ... (i)
–10o Heat supplied
E Ice
Case (ii) : = ?, T = 1000 K, r = 0.06 m
t 1. The temp. of ice changes from –10°C to 0°C.
E 2. Ice at 0°C melts into water at 0°C.
\ = 4 p s (1000)4 (0.06) 2 ... (ii) 3. Water at 0°C changes into water at 100°C.
t
Dividing (ii) and (i), we get 4. Water at 100°C changes into steam at 100°C.
19. (c) We know that V/T = constant
E / t (1000) 4 (0.06) 2 2 4 V + DV V
= = =4 \ =
450 (500)4 (0.12)2 22 T + DT T
E or VT + TDV = VT + VDT
Þ = 450 × 4 = 1800 W or TDV = VDT
t
14. (b) When a enclosed gas is accelerated in the positive DV 1
or =
x-direction then the pressure of the gas decreases along V DT T
the positive x-axis and follows the equation 20. (a) Work done is equal to area under the curve on PV
DP = – r a dx diagram.
where r is the density and a the acceleration of the 21. (b) According to Wien's law, lT = constant
container. From graph l1 < l3 < l2
The result will be more pressure on the rear side and \ T1 > T3 > T2.
less pressure on the front side.
P-192 Topic-wise Solved Papers - PHYSICS
22. (b) Let q°C be the temperature at B. Let Q is the heat 25. (a) For equilibrium in case 1 at 0° C
flowing per second from A to B on account of Upthrust = Wt. of body
temperature difference. \ K1Vd2g = Vd1g

KA (90 - q) d1
\ Q= ... (i) Þ K1 = ... (i)
l d2
where K = Thermal conductivity of the rod K1Vd2g
A = Area of cross-section of the rod
d1
l = Length of the rod V

A
90oC d2 Vd1g

Q At 0oC
D B
qoC For equilibrium in case 2 at 60° C
0oC 2Q 0oC Note : When the temperature is increased the density
Q will decrease.
C
\ d1' = d1 (1 + gFe × 60)
90oC and d2' = d2 (1 + gHg × 60)
Again upthrust = Wt. of body
By symmetry, the same will be the case for heat flow \ K2V'd2'g = V'd1'g
from C to B. K2Vd
' '2g
\ The heat flowing per second from B to D will be
d'1
KA (q - 0) V'
2Q = ... (ii)
l
Dividing eq. (ii) by eq. (i) d'2 V'd'1g
q At 60oC
2=
90 - q
é d2 ù d1
Þ q = 60° \ K2 ê ú=
23. (a) From the first law of thermodynamics êë1 + g Hg ´ 60 úû 1 + g Fe ´ 60
dQ = dU + dW
Here dW = 0 (given) é 1 + g Fe ´ 60 ù d
\ dQ = dU \ K2 ê ú= 1
Now since dQ < 0 (given) êë 1 + g Hg ´ 60 úû d2
\ dQ is negative
Þ dU = – ve Þ dU decreases. K1 1 + g Fe ´ 60
Þ =
Þ Temperature decreases. K 2 1 + g Hg ´ 60
24. (b) For adiabatic process PV g = constant
26. (a) For cyclic process;
Cp Qcyclic = WAB + WBC + WCA = 10 J + 0 + WCA = 5 J
Also for mono atomic gas g = = 1.67
CV Þ WCA = – 5 J
27. (a) PV = constant. Differentiating,
Also for diatomic gas g = 1.4
PdV æ 1 ö æ dV ö æ 1 ö
= -V ; b = - ç ÷ ç = Þ b×P=1
dP è V ø è dP ÷ø çè P ÷ø
\ Graph between b and P will be a rectangular
hyperbola.
P 28. (a) Note :
1 According to Kirchoff's law, good absorbers are good
2 emitters as well.
At high temperature (in the furnace), since it absorbs
more energy, it emits more radiations as well and hence
V is the brightest.
Since, gdiatomic < gmono atomic 29. (c) The graph shows that for the same temperature
\ Pdiatomic > Pmono atomic difference (T2 – T1), less time is taken for x. This means
Þ Graph 1 is for diatomic and graph 2 is for mono the emissivity is more for x. According to Kirchoff's
atomic. law, a good emitter is a good absorber as well.
HEAT & THERMODYNAMICS AND GASES P-193
T 34. (c) Q = mc DT
Þ Q = mc (T – t0) ... (i)
\ From 50 K to boiling temperature, T increases
T2 linearly.
During boiling, equation is
y Q = mL
x Temperature remains constant till boiling is complete
T1 After that, again eqn. (i) is followed and temperature
increases linearly.
35. (d)

o o
t 100 C K A 0C
30. (c) The lengths of each rod increases by the same amount l
K A
\ Dl a = Dl s
Þ l1a a t = l 2 a s t
l 2 aa l a
Þ = Þ 2 +1 = a +1 o
100 C K A
o
l1 a s l1 as 0C
l l
l 2 + l1 a a + a s l1 as
Þ = Þ =
l1 as l1 + l 2 a a + a s
K 2 A (100)
31. (b) If we study the P – T graph we find AB to be a q1 =
isothermal process, AC is adiabatic process given. Also l
for an expansion process, the slope of adiabatic curve A (100) KA (100)
is more (or we can say that the area under the P – V q2 = =
graph for isothermal process is more than adiabatic l l 2
+
process for same increase in volume). K K
Only graph (b) fits the above criteria. q2 KA (100) l 1
32. (b) Heat required to convert 5 kg of water at 20°C to 5 kg of \ = ´ =
q1 2l K 2 A(100) 4
water at 0°C
= mCw DT = 5 × 1 × 20 = 100 kcal 36. (b) In the first process W is + ve as DV is positive, in the
Heat released by 2 kg. Ice at – 20°C to convert into 2 kg second process W is – ve as DV is – ve and area under
of ice at 0°C the curve of second process is more
= mCice DT = 2 × 0.5 × 20 = 20 k cal. \ Net Work < 0 and also P3 > P1.
How much ice at 0°C will convert into water at 0°C for
giving another 80 kcal of heat
Q = mL Þ 80 = m × 80 P3
Þ m = 1 kg
Therefore the amount of water at 0°C P1
= 5 kg + 1kg = 6 kg
Thus, at equilibrium, we have, [6 kg water at 0°C + 1kg P2
ice at 0°C].
33. (b) We know that
lmT = Constant
lA< lB < lC V1 V2
So, TA > TB > TC
37. (a) According to Wein's displacement law
ì C C C ü
íQ TA = ,T =
-7 B
,T =
-7 C ý lm × T = constant
î 3 ´ 10 4 ´ 10 5 ´ 10 -7 þ
Here, l m3 < l m2 < l m1
Q = esAT4
e = 1 black body Þ T3 > T2 > T1
\ Q = sAT4
C4
\ QA = s.p (2 × 10–2)2 ×
27 ´ 10 -28
3
C2 2
QB = s.p (4 × 10–2)2 × 1
64 ´ 10 -28 T1 T2
T3

C2
and QC = s.p (6 × 10–2)2 ×
625 ´ 10 -28 The temperature of Sun is higher than that of welding
From comparison QB is maximum. arc which in turn is greater than tungsten filament.
P-194 Topic-wise Solved Papers - PHYSICS
38. (d) Heat transfer of glass bulb from filament is through We know that change in volume due to thermal
radiation. A medium is required for convection process. expansion is given by dV = V g dT ...(ii)
As a bulb is almost evacuated, heat from the filament is
transmitted through radiation. where g = coefficient of volume expansion.
39. (d) In this question the given options are wrong as all the From (i) and (ii)
four options contain e in place of s. æ 3ö 3
When a spherical body is kept inside a perfectly block V g dT = ç ÷ VdT Þ g =
è Tø T
body then the total heat radiated by the body is equal 44. (d) A real gas behaves as an ideal gas when the average
to that of the black body. distance between the gas molecules is large enough
40. (a) 1 Calorie is the amount of heat required to raise so that (i) the force of attraction between the gas
temperature of 1 gm of water from 14.5°C to 15.5°C at molecules becomes almost zero (ii) the actual volume
760 mm of Hg. of the gas molecules is negligible as compared to the
41. (c) As shown in the figure, the net heat absorbed by the occupied volume of the gas.
water to raise its temperature The above conditions are true for low pressure and
= (1000 – 160) = 840 J/s
high temperature.
Now, the heat required to raise the temperature of water
45. (a) Initially
from 27° C to 77°C is
V1 = 5.6l, T1 = 273K, P1 = 1 atm,
Q = mc Dt = 2 × 4200 × 50 J
5
160 J/s g = (For monoatomic gas)
3
The number of moles of gas is
2l 5.6l 1
n= =
22.4l 4
Finally (after adiabatic compression)
V2 = 0.7l
1000 J/s For adiabatic compression
Therefore the time required T1V1g-1 = T2V2 g-1
Q 2 ´ 4200 ´ 50 5
t= = = 500 sec = 8 min 20 sec æ V1 ö
g -1 -1
840 840 æ 5.6 ö 3
\ T2 = T1 ç ÷ = T1 ç ÷ = T1 (8) 2/ 3 = 4T1
é hrg ö ù è V2 ø è 0.7 ø
42. (b) The force is ê æç P0 + ÷ø ´ (2 R ´ h) ú - 2 RT
è 2 We know that work done in adiabatic process is
ë û
nRDT 9
P0 W= = RT1
g -1 8
2R
FT vrms (helium) Margon 40
46. (d) = = = 10 » 3.16
h/2 vrms (argon) Mhelium 4
h
P0 + rg
2 47. (d) The heat is supplied at constant pressure.
Therefore,
Q = n Cp Dt
5
= 2 éê R ùú ´ Dt
Note : In the first part the force is created due to ë2 û
pressure and in the second part the force is due to
surface tension T. æ 5 ö
\ Force = 2P0Rh + Rrgh2 – 2RT çQ Cp = R for mono-atomic gas ÷
è 2 ø
43. (c) PT2 = constant (given)
5
PV =2× × 8.31 × 5 = 208 J
Also for an ideal gas = constt 2
T
From the above two equations, after eliminating P. 48. (a) Given HI = HII
V ADT (A + A) DT
= constt Þ V = k T3 where k = constant k1 ´ t1 = k 2 ´ t2
T3 l+l l
dV dT k1 t1
Þ =3 \t2 = ´ ...(i)
V T k2 4
æ 3ö Where k1 and k2 are the equivalent conductivities in
Þ dv = ç ÷ VdT ...(i)
è Tø configuration I and II respectively.
HEAT & THERMODYNAMICS AND GASES P-195
For configuration I : KADT K1 A1DT K 2 A2 DT
l+l l l 2 3 = +
= + \ = l l l
k1 k 2k k1 2k or, Kp (2R)2 = K1p R2 + K2p [(2R)2 – R2]
4k K + 3K 2
\ k1 = ...(ii) or, K= 1
3 4
For configuration II : 5. (a, b, c, d)
k2(A+A) = kA + 2kA (a) For all thermal processes.
\ k2 =
3k
...(iii) DU = nCv DT where DT = (T2 - T1 )
2 (b) According to first law of thermodynamics.
4k DQ = DU + DW
9
From (i) , (ii) and (iii) t 2 = 3 ´ = 2sec In an adiabatic process DQ = 0.
3k 4 or, 0 = DU + DW
2
option (a) is correct or, DU = DW
49. (d) P1M1 = d1RT (c) In the isothermal process, DT = 0.
P2M2 = d2RT Q DU = 0
P1 M1 d1 (d) In the adiabatic process, DQ = 0.
\ ´ =
P2 M 2 d 2 DU nCv DT Cv 1 1 5
6. (d) = = = = =
4 2 d1 Q p nC p DT C p g 7 / 5 7
´ =
3 3 d2 7. (b) Note : All three vessels are at same temperature.
According to Maxwell's distribution of speed, average
d 8
\ 1 = option (d) is correct. speed of molecules of a gas v µ T .
d2 9
\ The velocity of oxygen molecules will be same in
A as well as C.
8. (a, b, d)
3RT 3 ´ 8.314 ´ 298 The work done by the gas in the process A to B exceeds the
1. (a) M = 2 = ´ 1000 = 2gm
work that would be done by it if the system were taken from
Crms 1930 ´ 1930
\ The gas is H2 A to B along the isothermal line. This is because the work
2. (b) Applying done is the area under the P-V indicator diagram. As shown
Q = nCp DT Þ 70 = 2 × Cp × 5 by the diagram the area under the graph in first diagram will
\ Cp = 7 cal/mol K. be more than in second diagram. When we extrapolate the
Cp – Cv = R graph shown in figure (i). Let P0 be the intercept on P-axis
Cv = 5 cal/mol K and V0 be the intercept on V-axis. The equation of the line
\ Q = nCv DT = 2 × 5 × 5 = 50 cal. AB can be written as
ALTERNATE SOLUTION P0
Q2 nCv DT 1 Q P= - V + P0 ... (i) [Q y = mx + c ]
= = Þ Q2 = 1 V0
Q1 nC p DT g g
RT
70 But V =
Þ Q2 = = 50 cal P
1.4
3. (a) Heat lost by steam = Heat gained by P
(water + calorimeter) Po
P
mL + m × c × (100 – 80) = 1.12 × c × (80 – 15) isothermal
P A
m [540 + 1 × 20] = 1.12 × 1 × 65 process
m = 0.13 kg P
4. (c) Total transfer of heat per second through the composite
= Heat transfer per second from material with thermal p B p
conductivity K1 + Heat transfer per second from 2 2
material with thermal conductivity K2.
V 2V Vo V V 2V V
K2 Fig (i) Fig (ii)
R
K1
P0 RT
2R \ P= - ´ + P0
V0 P
l Þ P2V0 – PP0V0 = – P0RT ... (ii)
P-196 Topic-wise Solved Papers - PHYSICS
Relation between P and T is the equation of a parabola.
Also, (Vrms )1 T V 120
10. (b) = 1 Þ =
RT (Vrms )2 T2 (Vrms )2 480
P=
V
From (i) and (ii) V 1
Þ = Þ (Vrms)2 = 2V
RT P P0 2 (Vrms )2 2
= - 0 V + P0 Þ RT = - V + P0V 11. (b) For an isothermal process
V V0 V0
PV = constant
Note :The above equation is of a parabola
On differentiating, we get
(between T and V)
PdV + VdP = 0
P0 2 P0
Equation T = - V + V dP
V0 R R Þ P= =K (Bulk Modulus)
dV / V
Differentiating the above equation w.r.t.V, we get 12. (d) A is free to move, therefore heat will be supplied at
dT P P constant pressure
= - 0 ´ 2V + 0
dV V0 R R \ DQA = nCpDTA ... (i)
B is held fixed, therefore heat will be supplied at constant
dT volume.
When = 0 then
dV \ DQB = nCvDTB ... (ii)
P0 P V0 But DQA = DQB (given)
´ 2V = 0 Þ V =
V0 R R 2 æ Cp ö
Also,
\ nCpDTA = nCvDTB \ DTB = ç ÷ DTA
è Cv ø
d 2T -2P0 = g (DTA) [g = 1.4 (diatomic)]
= = – ve = (1.4) (30 K)
dV 2 V0 R
\ DTB = 42 K
V0 13. (b, c) There is a decrease in volume during melting of an ice
Þ V= is the value for maxima of temperature slab at 273 K. Therefore, negative work is done by ice-
2
Also, water system on the atmosphere or positive work is
PA VA = PB VB done on the ice-water system by the atmosphere.
Þ TA= TB (From Boyle's law) Hence, option (b) is correct.
Þ In going from A to B, the temperature of the gas first NOTE : Secondly heat is absorbed during melting (i.e. dQ is
V0 positive) and as we have seen, work done by ice-water
increase to a maximum (at V = ) and the decreases and system is negative (dW is negative.) Therefore, from first
2 law of thermodynamics
reaches back to the same value.
dU = dQ – dW
9. (a, b) change in internal energy of ice-water system, dU will be
positive or internal energy will increase.
Energy emitted per second by body A = eAs TA4 A 14. (d) According to Wien's displacement law,
lmT = 2.88 × 106 nmK
Energy emitted per second by body B = eBs TB4 A The wavelength at the peak of the spectrum becomes
Given that power radiated are equal
2.88 ´ 106 nmK
eAs T A4 A = eBs TB4 A lm = = 103 nm
2880 K
1/ 4
æ eA ö
Þ TB = ç ÷ ´ TA = 1934 K U2
è eB ø
According to Wein's displacement law El
1
lm µ U1
T U3
Since temperature of A is more therefore (lm)A is less
\ (lm)B – (lm)A = 1 × 10–6 m (given) ... (i)
lm
Also according to Wein's displacement law
(lm)A TA = (lm)B TB l

(l m ) A TB 5802 NOTE : Thus, the maximum energy is radiated for 103 nm


Þ = = ... (ii) wavelength. It follows that the energy radiated between 499 nm
(l m ) B TA 1934
to 500 nm will be less than that emitted between 999 nm to
On solving (i) and (ii), we get 1000 nm, i.e., U1 < U2 or U2 > U1.
lB = 1.5 × 10–6 m.
HEAT & THERMODYNAMICS AND GASES P-197
15. (b, d) 17. (c, d)We know that
Brass 8 RT 3 RT 2RT
v= ; vrms = and vp =
Copper pM M M
d From these expressions, we can conclude that
vp < v < vrms
Also the average kinetic energy of gaseous molecules
R is

q 1 2 1 æ3 ö 3
E = mvrms = m ç v 2p ÷ = mv 2p
2 2 è2 ø 4
18. (a) NOTE : The law of equipartition of energy states that
Co-efficient of linear expansion of brass is greater than 'For a dynamical system in thermal equilibrium, the
energy of a system is equally distributed among its
that of copper i.e., a B > a C . Now,,
various degrees of freedom and the energy associated
LB = L0 (1 + a B DT ) 1
with each degree of freedom per molecule is k.T. In
or, ( R + d ) q = L0 (1 + a B DT ) 2
this case, O2 and N2 both have two degrees of rotational
Again, LC = L0 (1 + a C DT ) kinetic energy and since the temperature is also same,
the ratio of the average rotational kinetic energy is 1 : 1.
or, Rq = L0 (1 + a C DT )
19. (a, c, d)
(R + d )q 1 + a B DT Since sun rays fall on the black body, it will absorb
\ =
Rq 1 + aC DT more radiation and since, its temperature is constant it
will emit more radiation. The temperature will remain
R+d same only when energy emitted is equal to energy
or, = (1 + a B DT ) (1 - aC DT ) , by binomial
R absorbed.
theorem.
20. (b, d) C p - Cv = R for all gases
d
or, 1 + = 1 + ( a B - aC ) DT - a B aC (DT )2 For monoatomic gas :
R
3 5 5
d d Cv = R ; Cp = R ; g =
or, = ( a B - a C ) DT or R = 2 2 3
R ( a B - a C ) DT
15
1 1 C p .Cv = ; C p + Cv = 4
\ Rµ and R µ . 4
DT a B aC
-
For diatomic gas :
16. (c) Container A Container B
5 7 7
mA mB Cv = R ; C p = R ; g = and
PAV = RT PBV = RT 2 2 5
M M 35
mA mB C p .Cv = ; C p + Cv = 6
RT RT 4
P'A (2V) = P'B (2V) =
M M 21. (b, d) In case of an isothermal process we get a rectangular
m A RT m A RT hyperbola in a P-V diagram. Therefore option (a) is
Þ PA – P'A = - wrong.
MV M (2V )
TD < TB. Therefore in process B ® C ® D, DU is
m A RT negative. PV decreases and volume also decreases,
Þ DP = ... (i) therefore W is negative. From first law of
2 MV
thermodynamic, Q is negative i.e., there is a heat loss
mB RT mB RT
and PB – P'B = - option (b) is correct.
MV M (2V ) WAB > WBC.
mB RT Therefore work done during path A ® B ® C is
1.5 DP = ... (ii) positive, option (c) is wrong.
2 MV
Dividing (i) and (ii) Work done is clockwise cycle in a PV diagram is
positive. Option (d) is correct.
1.5DP mB
= 22. (a, b) Process A to B
DP MA As the temperature remains the same, this process is
3 mB isothermal. Therefore there is no change in the internal
Þ = Þ 3mA = 2mB
2 mA energy. Option (a) is correct.
P-198 Topic-wise Solved Papers - PHYSICS
V Also
DTC DTC
4V0 B QC = = = 2 KZ ( DTC )
RC 1/ 2 KZ
DTB DTC 3KZ ( DTC )
QB = = = [Q DTB = DTC]
RB 4 / 3KZ 4
V0 A DTD DTC 5KZ ( DTc )
C QD = = = [Q DTD = DTC ]
RD 4 / 5KZ 4
T0 T 3 KZ ( DTC ) 5 KZ ( DTC )
QB + QD = +
P0 4 4
Also P0 V0 = PB × 4 V0 Þ PB = 8 KZ ( DTC )
4 = = 2 KZ ( DTC ) = QC
Work done 4
4V0 \ (d) is the correct option.
W = nRT0 loge 24. (a, b, c, d) We know that dQ = m C dT in the range 0 to 100K
V0 From the graph, C increases linearly with temperature
therefore the rate at which heat is absorbed varies
= P0V0 log e 4 [Q P0V0 = nRT0 ]
linearly with temperature. Option (a) is correct
Option (b) is correct As the value of C is greater in the temperature range
The process BC is not clear. Therefore no judgement 400-500K, the heat absorbed in increasing the
can be made for point C. temperature from 0 - 100K is less than the heat required
23. (a, c, d) for increasing the temperature from 400 - 500K option
It is given that heat Q flows only from left to right (b) is correct.
through the blocks. Therefore heat flow through A and From the graph it is clear that the value of C does not
E slabs are the same. change in the temperature range 400-500K, therefore
\ [a] is correct option there is no change in the rate of heat absorption in
this range. Option (c) is correct.
As the value of C increases from 200-300K, the rate of
A B E
3K QB RB heat absorption increases in the range 200-300K. Option
2K 6K (d) is also correct.
C RC SUBJECTIVE PROBLEMS :

4K QC 1. W0 – W1 = V ´ d l ´ g ... (i)
QA QE
W0 – W2 = V '´ d 'l ´ g ... (ii)
Also, V' = V (1 + b DT) ... (iii)
RA D RE and dl = dl ' (1 + g l DT ) ... (iv)
5K QD RD
From (ii), (iii) and (iv)
QA = QB + QC + QD = QE V (1 + b DT ) ´ d l
W0 – W2 = ´ g ... (v)
Since heat flow through slabs A and E is same, 1 + g l DT
[b] is not correct. Dividing (i) and (v), we get
W0 - W1 Vd l g (1 + g l D T )
l =
We know that resistance to heat flow is R = W0 - W2 V (1 + bDT ) d l g
KA
Let the width of slabs be Z. Then W0 - W1 1 + g l DT W0 - W1 1 + g l (t2 - t1 )
Þ = Þ =
L 1 4L 4 W0 - W2 1 + bDT W0 - W2 1 + b (t2 - t1 )
RA = = ,R = =
2 K (4L) Z 8KZ B 3K ( LZ ) 3KZ Þ (W0 – W1) [1 + b (t2 – t1) = (W0 – W2) [1 + g l (t2 - t1 )]

4L 1 4L 4 W2 - W1 b (W0 - W1 )
RC = = , RD = = Þ gl = +
4 K (2 LZ ) 2 KZ 5K ( LZ ) 5 KZ (W0 - W2 ) (t2 - t1 ) (W0 - W2 )
2. KX = 0.92 cal/sec-cm-°C
L 1 KY = 0.46 cal/sec-cm-°C
RE = =
6 K (4 LZ ) 24 KZ C [T ]
C
Now, DT = QR X X
As R E is least, DTE is also smallest ie since the X
B
resistance to heat flow is least for slab E, the A E
[60oC] Y [TB] [10oC]
temperature difference across is smallest.
\ Option (c) is the correct answer. Y Y
[TD]
D
HEAT & THERMODYNAMICS AND GASES P-199
NOTE THIS STEP : The heat flow through AB is divided NOTE : The slope DE indicates that the temperature of the
into two path BC and BD. Symmetry shows that no heat will solid begins to rise.
flow through CD. Therefore (iv) The reciprocal of heat capacity in solid state is greater
KY A (60 - TB ) K X A (TB - 10) KY A (TB - 10) than the reciprocal of heat capacity in liquid state
= +
l 2l 2l æ 1 ö æ 1 ö
>ç ÷ Þ (mc)liquid > (mc)solid
On solving the above equation, we get çè ÷ø
mc solid è mc ø liquid
TB = 30°C
As C is a point at the middle of BE therefore temperature at Þ cliquid > csolid
C is 20°C. 7. P1 = 830 – 30 = 800 mm Hg
Similarly temperature at D is also 20°C. P2 ?
3. Heat entering per second to the room, V1 = V V2 = V
2 T1 = T T2 = T – 0.01 T = 0.99 T
V 200 ´ 200
= = 2000 J ... (i) P1V1 P2 V2
R 20 =
Heat transmitted through the glass window T1 T2
DH KA (DT ) P1T2 800 ´ 0.099T
= \ P2 = = = 792 mmHg
t l T1 T
0.2 ´ 4.2 ´ 1 ´ (20 - T )
... (ii) \ Total pressure in the jar = 792 + 25 = 817 mm Hg
0.2 ´ 10 -2 8. A ® B A straight line between A and B in V-T graph indicates
0.2 ´ 4.2 ´ 1 ´ ( 20 - T ) V µ T Þ Pressure is constant.
2000 =
0.2 ´ 10 -2 P
A B
2000 ´ 0.2 ´ 10-2
Þ 20 – T = = 4.76
0.2 ´ 4.2
Þ T = 15.24°C
4. PV = nRT
When P, T are same n µ V C
As volumes are same, both samples will have equal number
of molecules V
B ® C Volume is constant. Since the temperature is
5. Cu m decreasing, the pressure should also decrease.
C ® A The temperature is constant but volume decreases.
30cm 70cm
The process is isothermal.
Increase in length of copper rod = DlCu aCu × DT 9. Lead bullet just melts when stopped by an obstacle. Given
= 30 × 1.7 × 10–5 × 100 = 0.051 cm that 25% of the heat is absorbed by the obstacle. Therefore
Increase in length of second rod = l a DT 75% heat is used in melting of lead. Initial temp. = 27°C
M.P. = 300°C
= 70 × a × 100 = 700 a (0.75) K.E. = Heat utilised in increasing the temperature and
Total increase in length = 0.051 + 7000 a = 0.191cm heat utilised to melt lead at 300°C
Þ a = 2 ´ 10-5 per °C 1
(0.75) × Mv 2 = Mc DT + ML
The force required to maintain the copper rod at its initial 2
length when the rod is heated,
1 2
F = YA aCuT (0.75) × v = (0.03 × 300 + 6) × 4.2
2
8
= 1.3 ´ 1011 A ´ 1.7 ´ 10 -5 ´ 100 = 2.21 ´ 10 A dyne [4.2 to convert into S.I. system]
v = 12.96 m/s
(A = the area of cross-section)
10. Work don in an adiabatic process is
\ Force required to maintain the second rod = YAa T
1
\ Y ´ A ´ 2 ´ 10 -5 ´ 100 = 2.21 ´ 108 A W= [ P V - PV ]
2
1- g 2 2 1 1
\ Y = 1.1 ´ 1011 newton/metre
Here, P1 = 105 N/m2, V1 = 6 l = 6 × 10–3 m3
6. (i) Region AB : Heat is absorbed by the material at a
constant temperature called the melting point. g
æV ö
The phase changes from solid to liquid. P2 = P1 ç 1 ÷ , V2 = 2 l = 2 × 10–3
Region CD : Heat is absorbed by the material at a èV ø 2
constant temperature called the boiling point. The
3
phase changes from liquid to gas. Given that Cv = R
(ii) Latent heat of vaporisation = 2 (latent heat of fusion) 2
(iii) Q = mcg DT. 5
\ Cp = R [Q C p - Cv = R]
DT 1 2
The slope DE = =
Q mcg
P-200 Topic-wise Solved Papers - PHYSICS
Cp 2R 2 ´ 8.314
\ g= = 1.67 \ v0 = = = 35.6 ms–1
Cv M( g - 1) 32
´ (1.41 - 1)
1.67 100
é6ù [ Q g = 1.41 for O2 (diatomic gas)]
\ P2 = 105 ê ú = 105 × (3)1.67 = 6.26 × 105 N/m2
ë 2û 13. For the left chamber
1 P0V0 P0 ´ 243
\ W= [6.26 × 105 × 2 × 10–3 – 105 × 6 × 10–3] = ´ V1
1 - 1.67 T0 32 ´ T1
1 652 243 V1T0
W= [1252 – 600] = - = – 973.1 J Þ T1 = ´ ... (i)
-0.67 0.67 32 V0
Work done is negative because the gas is compressed. For the right chamber for adiabatic compression
11. NOTE : Since the temperature and surface area is same,
243
therefore the energy emitted per second by both spheres is We get, P0 V0g = P0 ´ ´ V2g
same. 32
We know that Q = mcDT V2 æ 32 ö
3/ 5
8
Since Q is same and c is also same (both copper). Þ =ç ÷ Þ V2 = V0
V0 è 243 ø 27
1 But V1 + V2 = 2V0
\ mµ
DT 8 46
Mass of hollow sphere is less; \ V1 = 2V0 – V2 = 2V0 – V0 = V0 ... (ii)
27 27
\ Temperature change will be more.
From (i) and (ii)
\ Hollow sphere will cool faster.
12. (i) F = P × A = 105 × 1 = 105 N 243 46 ´ V0
T1 = ´ ´ T0
Dp 32 V0 ´ 27
F= Þ Dp = F ´ Dt = 105 ´ 1 = 105 .. (i)
Dt 207
Now, momentum change per second or, T1 = T0 = 12.9T0 (approx.)
16
(Dp) = n × 2mv ...(ii)
To find the temperature in the second chamber (right), we
Where n is the number of collisions per second per
square metre area apply
From (i) and (ii) g 1-g
æ T1 ö æ P2 ö
105 çè T ÷ø = çè P ÷ø
n × 2mv = 105 \ n= 2 1
2mv
Root mean square velocity 5/3 1- 5/ 3
æ T0 ö æ 243 P0 ö
Þ çè T ÷ø =ç Þ T2 = 2.25 T0
3RT
=
3 ´ 8.314 ´ 300 2 è 32 P ÷ø 0
v= = 483.4 m/s
M 32 /1000 Work done in right chamber (adiabatic process)
According to mole concept 6.023 × 1023 molecules will
have mass 32 g 1
W= ( P2V2 - P0V0 )
32 1- g
\ 1 molecule will have mass g
6.023 ´ 1023 3 é 243 8 ù
=- ê P ´ V - P0V0 ú
105 ´ 6.023 ´ 1023 2 ë 32 0 27 0 û
\ n= = 1.97 ´ 1027
2 ´ 32 ´ 483.4
3æ9 ö 15
(ii) The kinetic energy of motion of molecules will be = - ç - 1÷ P0V0 = - ´ RT0 = –15.8 T0
converted into heat energy. è
2 4 ø 8
1 2 14. Let x moles shift from high temperature side to low
K.E. of 1 gm mole of oxygen = Mv 0 ... (i) temperature side.
2
where v0 is the velocity with which the vessel was for left bulb PV = nRT
moving. 76 × V = nR × 273 Initially
The heat gained by 1 gm mole of molecules at constant P' × V = (n + x) R × 273 Finally
volume for 1°C rise in temperature Dividing, we get
= nCv DT = 1 × Cv × 1 = Cv ... (ii)
P' n + x
From (i) and (ii) = ... (i)
1 R 76 n
Mv02 = Cv But, Cv = V V
2 g -1
1 R n n n+x n-x
Mv02 = Initially
2 g -1 0OC 62OC
Finally
HEAT & THERMODYNAMICS AND GASES P-201
For right bulb Substituting this value in (iii)
76 × V = nR × 273 Initially P1 × x1 = [P1 + 2.5] × x2
P' × V = (n – x) R × 335 Finally P1 × 46 = [P1 + 2.5] × 44.5
On dividing,
44.5 ´ 2.5
P ' n - x 335 \ P1 =
= ´ ... (ii) 1.5
76 x 273 Substituting this value in (ii)
From (i) and (ii)
44.5 ´ 2.5
n + x n - x 335 P×x = ´ 46
= ´ 1.5
n n 273
é 46 + 44.5 ù 44.5 ´ 2.5
Þ n=
608
x. ... (iii)
Þ P× ê
ë 2 úû = 1.5
´ 46
62
Substituting the value of (iii) in (i), we get x1 + x 2
[Q x= ] Þ P = 75.4 cm
P' 62 2
= 1+
76 608 17. We know that PV = nRT
670 PV 1.6 ´ 106 ´ 0.0083 16
Þ P' = ´ 76 = 83.75cm Hg \ n= = = = 5.33 moles
608 RT 8.3 ´ 300 3
15. Heat flow per second 5R 3R
Cp = Þ Cv =
K1 K2 K3 2 2
Wood Cement Brick qB = -10°C When 2.49 × 104 J of heat energy is supplied at constant
QA = 20°C
volume then we can use the following relationship to find
d1 d2 d3 change in temperature.
Q = nCv DT
(q A - q B ) A
H= Q 2.49 ´ 10 4
d1 d2 d3 =
+ + \ DT = = 375 K
k1 k 2 k3 nCv 3
5.33 ´ ´ 8.3
2
[20 - ( -10)] ´ 137
= Therefore, the final temperature
é 2.5 1 25 ù -2 = 300 + 375 = 675 K
êë 0.125 + 1.5 + 1 úû ´ 10 Applying Gay Lussac's Law, to find pressure.
= 9000 W P1 P2
This rate of heat has to be supplied by the heater to keep =
T1 T2
the temperature constant.
16. Let A be the area of cross-section of the tube. PT 1.6 ´ 106 ´ 675
Since temperature is the same, applying Boyle's law on the Þ P2 = 1 2 = = 3.6 × 106 Nm–2
T1 300
side AB
P × (x × A) = P2 × (x2 × A) ... (i) 18. (i) P – V diagram is drawn below.
Applying Boyle's law in section CD
P
P × (x × A) = P1 × (x1 × A) ... (ii) (P1,V1)
From (i) and (ii) (P2,V2)
P1=P2
P1 × (x1 × A) = P2 × (x2 × A) 1 2
Þ P1x1 = P2x2
where P2 = P1 + Pressure due to mercury column (P3,V3)
P3
x2 3
m
5c P1 20l 40l V
x x x1 D
C (ii) P1V1 = nRT1
P P P2 \ P1 × 20 × 10–3 = 2 × 8.3 × 300
B
A B C D 30O P1 = 2.49 × 105 Nm–2
5cm A
PV PV
Pressure due to mercury column Applying 1 1 = 2 2
T1 T2
F mg sin 30° Vdg sin 30°
P= = = For 1 ® 2
A A A
20 40
(A ´ 5) ´ dg sin 30° = Þ T2 = 600 K
= = 5 sin 30° cm of Hg 300 T2
A 2 ® 3 is adiabatic expansion.
P2 = P1 + 5 sin 30° = P1 + 2.5
P-202 Topic-wise Solved Papers - PHYSICS
T2V2g -1 = T3V3g -1 20. (i) Let pressure = P, Volume = V and Temperature = T be
the initial quantities and Pressure = P', Volume = 5.66 V
1 1 Temperature = T/2 be the final quantities.
é T ù g -1 é 600 ù 5 -1 For adiabatic process
\ V3 = V2 ê 2 ú = 40 ê ú = 113l
T
ë 3û ë 300 û 3 T
TV g -1 = (5.66 V ) g -1
5 2
[Q g = for mono atomic gas] Þ 2 = (5.66)g –1
3
Now, P3V3 = nRT3 Taking log on both sides, log 2 = (g – 1) log 5.66
Þ g = 1.4
nRT3 2 ´ 8.3 ´ 300
Þ P3 = = = 0.44 × 105 N/m2 2 2
V3 113 ´ 10-3 But g = 1 + Þ 1.4 = 1 +
f f
(NOTE : T3 = T1 given)
(iii) W = W12 + W23 2
Þ f= =5
nR 0.4
= P1 (V2 – V1) + (T - T )
g -1 2 3 Thus degrees of freedom of gas molecules = 5
W12 = work done at constant pressure (ii) For adiabatic process the pressure-volume relationship
W23 = work done in adiabatic condition is
2 ´ 8.3 g g
= 2.49 × 105 (40 – 20) 10–3 + (600 - 300) 1 1 = P2V2
PV
5
-1
3 P P
Þ P2 = =
= 4980 + 7470 = 12450 J (5.66) 1.4 11.32
19. KEY CONCEPT : The final pressure on the gas
Work done for adiabatic process
= atm pressure + pressure due to compression of spring
kx æ P ö
PV - ç (5.66 V )
P2 = Patm +
A PV - P V è 11.32 ÷ø
W= 1 1 2 2 =
8000 ´ 0.1 g -1 1.4 - 1
Þ P2 = 105 + = 2 × 105 N/m2
8 ´ 10 -3 Þ W = 1.25 PV

21. (a)
T1=300K Atm pressure = 105N/m2 P
Ideal
Heater Monoatomic A
(PA,VA,TA)
Gas
k = 8000N/m

,T )
The final volume, (PB,VA C (PB,2VA,TA)
V2 = V1 + xA C B
= 2.4 × 10–3 + 0.1 × 8 × 10–3 = 3.2 × 10–3 m3
PV PV P2V2T1
Applying 1 1
= 2 2 Þ T2 = T
T1 T2 PV
1 1

2 ´ 105 ´ 3.2 ´ 10-3 ´ 300 Process A to B (isothermal expansion)


Þ T2 = = 800 K. PAVA = PBVB
105 ´ 2.4 ´ 10 -3
PA
NOTE : Heat supplied by the heater is used for expansion Þ PAVA = PB× 2VA Þ PB =
of the gas, increasing its temperature and storing potential 2
energy in the spring. Process B to C (isobaric compression)
\ Heat supplied VB VC 2V A VA T
1 2 = Þ = Þ TC = A
= P DV + nCv D T + k x TB TC TA TC 2
2
Process C to A [volume is constant]
PV 1
= 105 [0.8 × 10–3] + 1 1
Cv DT + kx 2 PC PA P P
RT1 2 = Þ B = A
TC TA TC TA
105 ´ 2.4 ´ 10 -3 3 1
= 80 + ´ ´ 2 ´ 500 + ´ 8000 × 0.1 PA / 2 PA T
2 ´ 300 2 2 Þ = Þ TC = A
= 720 J TC TA 2
HEAT & THERMODYNAMICS AND GASES P-203
Work done from C to D (Isobaric process)
WCD = nR (TD – TC) = nR (300 – 400)
P
= – 100nR = – 200 × 8.32 = – 1664 J
PA A (P ,V ,T ) Work done from D to A (Isothermal process)
A A A
PD 1
WDA = 2.303nRT log10 = 2.303nR × 300 log10
PA 2
= – 207.9nR
B = – 415.8 × 8.32 = – 3459.5 J
C (PB,2VA,TA) The total work done = WAB + WBC + WCD + WDA
= 1153 J
(a) DU = Q – W
O VA 2VA For complete cycle DU = 0
V \ Q = W = 1153 J
Let the system initially be at point A at pressure PA and temp (b) W = 1153 J
TA and volume VA. (c) DU = 0. Since, the process is cyclic.
Process A to B 23. Initially the temperature at the open end is 400K and the
The system is isothermally expanded and reaches a new temperature at the metal disc-cylinder interface is 300K.
state B (PB, 2VA, TA) as shown in the figure. q
Process B to C
The system is the compressed at constant pressure to its
original volume to reach at state C (PB, VA, TC)
H
Process C to A
Finally at constant volume, the pressure is increased to its Open
End
original pressure to reach the state A again. Metal disc Cylinder
(b) The total work done Insulation
W = WA® B + WB® C + WC® A NOTE : As heat passes through the cylinder and reaches
the metal disc, the temperature of metal disc rises. Since the
VB conductivity (thermal) of metal disc is very high, soon the
= nRTA loge + nR (TC – TA) + 0
VA temperature of the whole disc will rise and along with that
the temperature of the other end of the cylinder (metal disc-
2VA æT ö cylinder interface) also rises simultaneously.
= 2.303 × 3 × R × TA log10 + 3 R ç A - TA ÷
VA è 2 ø Let at any instant of time, the temperature of the metal disc-
cylinder interface is q. At this instant the rate of heat crossing
3 the cylinder.
= 2.08 RTA – RTA = 0.58RTA
2 dQ KA(400 - q)
= ... (i)
NOTE : The total work done is equal to the heat exchanged dt l
as the process is cyclic. The same amount of heat is received by the metal disc.
22. Let us find out the work done in the cycle Therefore
P dQ dq
A B = mc ... (ii)
2atm dt dt
m = mass of disc
c = specific heat of metal disc
1atm C From (i) and (ii)
D
d q KA(400 - q)
mc =
dt l
æ d q ö mcl
çè ÷´ = dt
300K 400K T 400 - q ø KA
Work done from A to B (Isobaric process) On integrating
WAB = nR (TB – TA) At t = 0, q = 300
= nR × 100 = 2 × 200 × 8.32 = 1664 J At t = t, q = 350
Work done from B to C (Isothermal process) t mcl 350 d q
PB ò 0 dt = KA ò 300 (400 - q)
WBC = 2.303nRT log10
PC
-mcl
2
t= [ log(400 - q) ] 300
350
KA
= 2.303nR × 400 log10 = 277.2 nR
1 -0.4 ´ 600 ´ 0.4 ´ 2.303 400 - 350
= 554.4 × 8.32 = 4612.6 = log10 = 166.38 sec.
10 ´ 0.04 400 - 300
P-204 Topic-wise Solved Papers - PHYSICS
24. Given TA = 1000 K
PD PA
2 1 D to A is isochoric process =
PB = PA PC = PA TD TA
3 3
PA TA
P Þ = ... (iv)
PD TD
A
Multiplying (i) and (iii)
B
g -1 g
æ PC PA ö æT T ö
çè P P ÷ø = ç C ´ A ÷ .... (v)
D D B è D
T TBø

C Multiplying (ii) and (iv)


æ PA PC ö æ TC TA ö
V çè P P ÷ø = çè T T ÷ø ... (vi)
B D B D
(i) WAB (adiabatic expansion)
From (v) and (vi)
nR[TA - TB ]
WAB = g -1 g
g -1 æ TC TA ö æT T ö T T
çè T T ÷ø = ç C A ÷ Þ A C =1
Here, n = 1, R = 8.31 J mol–1 k–1, TA = 1000 K B D è TBTD ø TB TD
5
g= (For mono atomic gas) TATC 1000 ´ 425
3 Þ TD = = = 500K
TB 850
To find TB, we use
g -1 g
25. (i) The process is cyclic, therefore DU = 0
g 1-g g 1-g æP ö æT ö Now, DQ = DU + DW
TA PA = TB PB Þ ç A÷ = ç A ÷ ... (i)
èP øB èT ø B Þ DQ = DW
Þ Q1 + Q2 + Q3 + Q4 = W1 + W2 + W3 + W4
1-g 1-5 / 3 Þ 5960 – 5585 – 2980 + 3645 = 2200 – 825 – 1100 + W4
éP ù g é 3ù 5/3 Þ W4 = 765 J
Þ TB = TA ê A ú = 1000 ê ú = 850 K
ë PB û ë 2û (ii) KEY CONCEPT :

1 ´ 8.31[1000 - 850] Work done W + W2 + W3 + W4


h= = 1
\ WAB = = 1870 J Heat supplied Q1 + Q4
5/ 3-1
(ii) Heat Lost B ® C
1040
Q = nCv DT = nCv (TB – TC) Þ = = 10.82%
9605
3
Here, n = 1, Cv = R (For mono atomic gas), ALTERNATE SOLUTION
2
TB = 850 K æ Q2 ö (5585 + 2980)
h = ç1 - ÷ ´ 100 = 1 - = 10.82%
PB PC è Q1 ø (5960 + 3645)
To find TC, we use = (volume constant)
TB TC 26. The total pressure exerted by the mixture P = 105 Nm–2
Temperature T = 300 K ; Volume = 0.02 m3
PC TC Let there be x gram of Ne. Then mass of Ar will be 28 – x.
Þ = ..(ii)
PB TB
x
Number of moles of Neon = ;
é 1 ù 20
PC 1 ê PC 3 PA 1 ú
Þ TC = ´ TB = ´ 850 = 425 K êQ = = ú 28 - x
PB 2 ê PA 2 PA 2 ú Number of moles of Argon =
40
ë 3 û
Partial pressure due to Neon;
3 ( x / 20) RT
\ Q = 1× × 8.31 [425 – 850] = – 5298 J p1 =
2 V
(iii) Temperature TD : C to D is adiabatic compression Partial pressure due to Argon
g -1 g
æ PC ö æT ö [(28 - x ) / 40] RT
çè P ÷ø =ç C ÷ ...(iii) p2 =
D è TD ø V
HEAT & THERMODYNAMICS AND GASES P-205
But according to Dalton's law of partial pressure Dv 1 DT
P = p1 + p2 Percentage change ´ 100 = ´ ´ 100
v 2 T
x RT (28 - x) RT
105 = + Dv 1 1 1
20V 40V ´ 100 = ´ 100 = %
v 2 300 6
105 ´ 40V (d) PVg = Const.
Þ = 2x + 28 – x Differentiating the above equation
RT Vg (dP) – P (gVg –1 dV) = 0
Þ Vg dP = gPVg–1dV
105 ´ 40 ´ 0.02
Þ = x + 28 Þ x = 4g
8.314 ´ 300 dP gPV g -1 gP
Þ = g
= gPV g -1-g =
Þ Mass of Neon = 4g dV V V
\ Mass of Argon = 24g
-dP
5 + 7nB 19 Þ = -g P
27. (a) = Þ nB = 2 mol. dV / V
3 + 5nB 13 \ Bulk Modulus B = g P
We know that 1 1
\ Compressibility K = =
nA + nB n n B gP
= A + B
g m -1 g A - 1 g B - 1 1 1
\ K1 = and K2 =
where gm = Ratio of specific heats of mixture gP1 gP2
Here, nA = 1, g A = 5 3, g B = 7 5
1 1 1æ 1 1ö
According to the relationship DK = K2 – K1 = - = ç - ÷
gP2 gP1 g è P2 P1 ø
19
19 g g
PV 13 = constant, we get gm = Since the process is adiabatic, P2V2 = PV
13 1 1
(b) On substituting the values we get nB = 2 mol. g g
We know that velocity of sound in air is given by the æ V1 ö æ V1 ö
\ P2 = P1 ç ÷ = P1 ç = P1 5g
relationship è V2 ø è V1 / 5 ÷ø
gP m
v= where d = density = 1æ 1 1ö 1 æ 1 ö
d V \ DK = ç g
- ÷= ç g
- 1÷
g è P1 5 P1 ø gP1 è 5 ø
(n A + nB )
Also, PV = (nA + nB) RT Þ PV = RT (n A + nB ) RT (1 + 2) ´ 8.31 ´ T 24.93T
V P1 = = =
V V V
g (n A + nB ) RT g (n A + nB ) RT 1 æ 1 ö
\ v= = Þ DK = -1
m m 19 T çè 519 /13 ÷ø
V´ ´ 24.93 ´
V 13 V
Mass of the gas, m = nAMA + nBMB = 1 × 4 + 2 × 32 = – 8.27 × 10–5 V [Q T = 300 K]
= 68 g/mol = 0.068 kg/mol 5
28. (i) T1 = 27 + 273 = 300 K; g = (for monoatomic gas)
19(1 + 2) ´ 8.314 ´ 300 3
\ v= = 400.03 ms–1 V1 = V
13 ´ 0.068 V2 = 2V
(c) Velocity of sound, T2 = ?
Since the gas expands adiabatically.
g RT g R(T + DT )
v= and v + Dv = g -1 g -1
M M T1V1 = T2V2
1/ 2 g -1 5/ 3-1
v + Dv T + DT æ DT ö æV ö éV ù
Þ = = ç1 + ÷ Þ T2 = T1 ç 1 ÷ = 300 ê ú = 189 K
v T è T ø èV ø 2 ë 2V û

DT -nR (T2 - T1 ) -2 ´ 8.31(189 - 300)


When DT << T then << 1 (ii) W = =
T g -1 5/ 3 -1
Dv 1 DT + 8.31 ´ 111 ´ 3
\ 1+ = 1+ = = + 2767J
v 2 T 2
P-206 Topic-wise Solved Papers - PHYSICS
Change in internal Energy 2000
According to first law of thermodynamics 31. No of moles of Helium n = = 500 mol
DQ = DU + DW But D Q = 0 4
(as the process is adiabatic) Process ABC
\ DU = – DW = – 2767 J From A to B (isovolumic process)
(iii) W = 2767 J From the figure
29. Heat lost by steam = Heat gained by water PA= 5 × 104 N/m2, VA = 10 m3
msLfus = mwc DT n = 500 mol, R = 8.31 mol–1 K–1
TA = ?
mwc DT 0.1 ´ 4200 ´ 66 PAVA = nRTA
Þ ms = = = 0.0122 kg
L fus 540 ´ 103 ´ 4.2 PV 5 ´ 10 4 ´ 10
Þ TA = = = 120.3 K
nR 500 ´ 8.31
q1 q2
Applying combined gas equation for A to B
30. H H
qA I II III q PAV A PBVB
K1 K2 K3 B =
TA TB
d1 d2 d3
q A > qB PT 10 ´ 104 ´ 120.3
Þ TB = B A = = 240.6 K
At steady state
PA 5 ´ 104
From B to C
K1
First material : H= A(q A - q1 ) ... (i) PBVB PCVC
d1 =
TB TC
K2
Second material : H= A(q1 - q 2 ) ... (ii) V T 20 ´ 240.6
d2 Þ TC = C B = = 481.2 K
VB 10
K3 Heat involved in the process ABC
Third material : H= A(q2 - q B ) ... (iii)
d3 QABC = QAB + QBC
= nCv DT + nCp DT
H d1 = 500 Cv × (240.6 – 120.3) + 500 Cp × (481.2 – 240.6)
From (i) q A - q1 = ... (iv)
A K1 é3 5 ù
= 500 ê ´ 8.31 ´ 120.3 + ´ 8.31 ´ 240.6ú
H d2 ë 2 2 û
From (ii) q1 - q 2 = ... (v)
A K2 QABC = 3.25 × 106 J
For process ADC
H d3 PAVA PDVD
From (iii) q 2 - q B = ... (vi) =
A K3 A to D :
TA TD
Þ Adding the above three equations, we get
VDTA 20
Hæd d d ö Þ TD = = ´ 120.3 = 240.6 K
q A - qB = ç 1 + 2 + 3 ÷ VA 10
A è K1 K 2 K 3 ø
TD = 240.6 K
(q A - q B ) A NOTE : Process A to C via B and A to C via D have same
Þ H= change in internal energy (state function)
d1 d 2 d 3
+ + Applying first law of thermodynamics for process A to C
K1 K 2 K 3
via B
Substituting the values QABC = DE + DW
(27 - 0)1 Workdone DW can be found by the area under the graph
H= = 41.54 J / s ABC
0.01 0.05 0.01
+ + DW = 10 × 10 × 104 = 106 J
0.8 0.8 0.8 \ DE = QABC – DW = 3.25 × 106 – 106
From (iv) = 2.25 × 106 J
41.54 0.01 Now applying first law of thermodynamics in process A to C
27 - q1 = ´ Þ q1 = 26.48° C via D
1 0.8
QADC = DE + DW'
From (vi) Also, DW ' = 10 × 5 × 104 = 0.5 × 106 J
41.54 0.01 \ QADC = 2.25 × 106 + 0.5 × 106 = 2.75 × 106 J
q2 - 0 = ´ = 0.52° C Þ QADC = 2.75 × 106 J
1 0.8
HEAT & THERMODYNAMICS AND GASES P-207
ALTERNATE SOLUTION For D ® A process : Heat released
QADC = QAD + QDC = nCpDT + nCvDT
R
(ii) NO. Q2 = nCv (TD – TA) = n (T - TA )
32. n = 1, for diatomic gas, g -1 D
2 7 R 5R
g = 1+ = = 1.4 = 1. (791 - 300) = ´ 491
5 5 (2 / 5) 2
A ® B, adiabatic compression (Q No heat is exchanged in adiabatic processes).
B ® C, isobaric expansion
C ® D, adiabatic expansion nR
Now, WAB = - (T - TA )
D ® A, isochoric process g -1 B

P
R 5R
=- (900 - 300) = - ´ 609
(2 / 5) 2
B C
WBC = – nR (TC – TB) = 1× R (1818 – 909) = 909 R
nR R
WCD = - (TC - TD ) = + (1818 - 791)
g -1 (2 / 5)

D 5R
= ´ 1027
2
A 5R 5R
V
Wnet = 909 R + (1027 – 609) = 909 R + × 418
2 2
= 909 R + 1045 R = 1954 R
VA V \ Efficiency
Given = 16, C = 2
VB VB 1954 R
TA = 300 K, TB = ?, TD = ?, h = ? = 100 × (Wnet/Q1) = 100 × = 61.4%
3182 R
For adiabatic compression process A ® B
33. Let the pressure at point O be P0. Since the liquid is at
g -1 g -1
TAVA = TBVB or equilibrium at M
PA + h1r95°g = P0 + h r5° g
g-1 Þ P0= PA + h1r95° g – h r5° g ... (i)
æV ö
TB = ç A ÷ TA = (16)2 / 5 ´ 300 = 909 K Since the liquid is at equilibrium at N
è VB ø Þ PA + h2r5° g = P0 + h r95° g
\ For isobaric process B ® C : According to Charles' law Þ P0 = PA + h2r5° g – h r95° g ... (ii)
VB VC
As =
TB TC
PA PA
æV ö h1
or TC = TB ç C ÷ = 909 [2] = 1818 K h2
èV ø B
O
For adiabatic expansion process C ® D : o
A
95C oB C oD
VA V V 5C h o
95C 5C
As = 16 and C = 2; hence A = 8
VB VB VC
M N
According to Poisson's law,
From (i) and (ii)
g -1
TCVC = TDVDg -1 PA + h1r95° g – h r5° g = PA + h2r5° g – h r95° g
r
5o
g -1 2/5
éV ù é1ù 1818 Þ = 1.018 ... (i)
\ TD = TC ê C ú = 1818 ê ú = = 791K r
ë VD û ë8û (64)1/ 5 95o

For B ® C process : Heat absorbed We know that


Q1 = nCp (TC – TB) r0 = r1 (1 + g DT)
Applying the above formula, we get
gR (7 / 5) R r0 = r95° (1 + g × 95)
=n (TC - TB ) = 1 (1818 - 909)
g -1 (2 / 5) r0 = r5° (1 + g × 5)
7R r5° 1 + 95 g
= ´ 909 @ 3182 R \ = ... (ii)
2 r95° 1 + 5 g
P-208 Topic-wise Solved Papers - PHYSICS
From (i) and (ii) Heat absorbed by the gas on the path AB during
isochoric process
1 + 95 g DQAB = nCvDT = 1 × (3R/2) (TB – TA)
Þ g = 2.002 × 10–4
1 + 5 g = 1.018
3R é 3P0V0 P0V0 ù
But g = 3a = - = 3P0V0
2 êë 1 ´ R 1 ´ R úû
g 2.002 ´ 10-4 (c) As DU = 0 in cyclic process, hence,
Þ a= = = 6.67 × 10–5 °C –1
3 3 DQ = DW
34. (i) NOTE : When the rod is cooled, its length should DQAB + DQCA + DQBC = DW
decrease. P0V0 P0V0
DQBC = P0V0 – =
The change in length 2 2
Dl = l a DT = 0.5 × 10–5 × 100 = 0.5 × 10–3 m NOTE : As net heat is absorbed by the gas during
But to prevent the rod from contracting a mass is path BC, temp. will reach maximum between B and C.
attached to the lower end which extends the rod by é 2P ù
0.5 × 10–3 m. Then (d) Equation for Line BC is P = – ê 0 ú V + 5P0,
ë V0 û
T/A YDl A
Y= Þ =T RT
Dl/l l P= [For one mole]
V
1011 ´ 0.5 ´ 10-3 ´ 4 ´ 10 -6 2 P0 2
Þ =T \ RT = – V + 5P0V ... (i)
0.5 V0
Þ T = 400 N.
Since an equilibrium is established at the lower end of dT
For maximum; = 0,
the rod dV
\ T = mg
2P0
Þ 400 = m × 10 Þ m = 40 kg - × 2V + 5P0 = 0;
V0
1
(ii) Energy stored = stress × strain × volume 5V0
2 \ V= ... (ii)
4
-3
1 400 0.5 ´ 10 Hence from equation (i) and (ii)
= ´ ´ ´ 0.5 ´ 4 ´ 10-6 = 0.1 J
2 4 ´ 10 -6 0.5 2
-2 P0 æ 5V0 ö æ 5V ö
ALTERNATE SOLUTION : RTmax = ´ç + 5 P0 ç 0 ÷
V0 è 4 ÷ø è 4 ø
1
Energy stored = × Force × Extension 25 25 P0V0 25
2 = – 2P0V0 × + = P0V0
16 4 8
1
= × 400 × 0.5 × 10–3 = 0.1 J 25 P0V0
2 \ Tmax =
8 R
5R 3R 36. Case (i)
35. n = 1, For monoatomic gas : Cp = , Cv =
2 2 According to Newton's law of cooling
Cyclic process dT dT
A ® B Þ Isochoric process = – K' (T – TA) Þ = – K'dt
dt T - TA
C ® A Þ Isobaric compression
On integrating, we get
(a) Work done = Area of closed curve ABCA during cyclic
process. i.e. DABC 350 dT t1
ò400 T - TA = K ò0 dt
1 1
DW = × base × height = V0 × 2P0 = P0V0 t1
2 2 350
– [loge (T – TA) ] 400 = K' [t ] 0
(b) Heat rejected by the gas in the path CA during isobaric
compression process 350 - 300
Þ - log e = K ' t1
DQCA = nCp DT = 1 × (5R/2) (TA – TC) 400 - 300
2 P0V0 PV 100
TC = , TA = 0 0 Þ loge = K ' t1 or K't1 = loge 2 ... (i)
I ´R I´R 50
Case (ii)
5R é P0V0 2 P0V0 ù 5 NOTE : When the body X is connected to a large box Y. In
DQCA = - = - P0V0
2 êë R R úû 2 this case cooling occurs by Newton's law of cooling as well
as by conduction
HEAT & THERMODYNAMICS AND GASES P-209

dT KA(T - TA ) g -1
\ - = K '(T - TA ) + 1 é V1 æ V1 ö ù
dt CL = ê PV - P V
. . . ú
V2 èç V2 ø÷ ú
1 1 1 2
g -1 ê
dT é KA ù ë û
Þ - = ê K '+ (T - TA ) (for t > t1]
dt ë CL úû For Monoatomic gas,
Where K = coefficient of thermal conductivity of the rod. 2 5
g=1+ = ,
3 3
- dT é KA ù
Þ = ê K '+ dt
T - TA ë CL úû 3é æ V1 ö
2/3ù
W = ê PV
1 1 - PV
1 1ç
ú
On integrating, we get 2ê è V2 ø÷ ú
ë û
T dT 3t1 æ KA ö
-ò = ò ç K '+ ÷ø dt
350 T - T t1 è CL é æ V ö 2/3ù
A 3
1 1 ê1 - ç ú
1
= PV
æ KA ö 3t1 2 ê è V2 ÷ø ú
- [ log e (T - TA )] 350 = ç K '+ ÷ [t ]
T ë û
Þ
è CL ø t1 (ii) DU = DUAB + DUBC = Q – W
350 - 300 æ KA ö é æ V ö 2/3ù
Þ loge = ç K '+ ÷ 2t = 2 K ' t1 3
1 1 ê1 - ç ú
è 1
T - 300 CL ø = Q – PV
2 ê è V2 ÷ø ú
ë û
50 2KA
Þ loge = 2(log e 2) + t1 [according to first law of thermodynamics]
T - 300 CL
2 KA é B ® C Q = DU BC + 0 ù
50 {log e 4 + t1} ê A ® B Q = DU + W ú
=e CL ë AB û
T - 300
(iii) For process BC : DUBC = nCv DT = Q
-2 KAt1 [Q WBC = 0 ]
Þ T – 300 = 50 e -[log e 4] ´ e CL
R 3
For monoatomic gas Cv = = R,
-2 KAt1 g -1 2
Þ T = [300 + 12.5 e CL ] Kelvin
37. n = no. of moles of = 2, 3R
\ DUBC = Q = 2 × .DT
(A) The complete process is shown on P-V diagram in the 2
figure. Q
Hence DT = .
P 3R
3 C
According to Poission's Law :
Heating at constant
Volume V2 g -1 g -1
For the process AB, TAVB = TBVB
P2 B
2
Adiabatic g -1 g -1
compression æV ö 1 1 æ V1 ö
PV
or TB = TA ç 1 ÷ =
P1 A è V2 ø nR çè V2 ÷ø
1
5 / 3 -2 / 3
P1 g 1-g PV V2
\ TB = .V1 .V2 = 1 1
V 2R 2R
V2 V1
5 / 3 -2 / 3
PV
1 1 V2 Q
(B) (i) Total work done Hence, TC = TB + DT = +
2R 3R
1 1 - P2V2 )
( PV
W = WAB + WBC = +0 38. P
( g - 1)
[ Q WBC = PDV = P × 0 = 0] B C
2P1
g
æV ö
According to Poisson's law, P2 = P1 ç 1 ÷
è V2 ø
P1 A
g
1 é æ V1 ö ù
\ W= ê PV
1 1 - P1 ç V ú
è V2 ø÷
2
g -1 ê ú T
ë û T1 2T2
P-210 Topic-wise Solved Papers - PHYSICS
(A) Number of moles, n = 2, T1 = 300 K 39. Here the equilibrium temperature is 273 + 27 = 300 K
During the process A ® B Also according to the principle of calorimetry
PT = constant Heat lost by container = Heat gained by ice.
or P2V = constant = K (say) Heat lost by container :
K NOTE : Since specific heat is variable, we need to take the
Therefore, P = help of calculus to find the heat lost by the container.
V
Let dQ be the heat lost when the temperature decreases by
Therefore,
dT at any instant when the temperature of the container
VB VB K is T.
WA® B = òVA P.dV = òVA V
dV
\ dQ = mc dT
where m is the mass of the container and C = A + BT is
= 2 K [ VB - VA ] specific heat at that temperature
= 2[ KVB - KV A ] \ dQ = m (A + BT) dT
On integrating, we get
2 2
= 2[ ( PB VB )VB - ( PA VA )V A ] (K = P2V) 300 é BT 2 ù
300

= 2 [PBVB – PAVA ] Q = ò500 m ( A + BT ) dT = m ê AT + ú


êë 2 úû
= 2 [nRTB – nRTA] 500
= 2nR (T1 – 2T1] = (2) (2) (R) [300 – 600] = –1200 R = – 21600 m calorie (heat lost)
Therefore work done on the gas in the process AB is Heat gained by ice
1200 R. This heat is to be divided into two parts
(B) Heat absorbed/released in different processes. (i) 0° ice ® 0° water
Since the gas is monoatomic, therefore, (ii) 0° water ® 27° water
3 5 5 Q1 = mL Q2 = mcDT
Cv = R and Cp = R and g =
2 2 3 = 0.1 × 80,000 = 0.1 × 103 × 27
Process A – B = 8000 cal = 2700 cal
æ3 ö \ Q1 + Q2 = 8000 + 2700 = 10,700 cal ... (i)
DU = nCv DT = (2) ç R ÷ (TB – TA) Heat lost = heat gained
è2 ø
21600 m = 10,700
æ3 ö Þ m = 0.495 kg
= (2) ç R ÷ (300 – 600) = – 900 R
è2 ø 40. NOTE : When the mass of 100 kg is attached, the string is
QA ® B = WA ® B + DU = ( – 1200 R) – (900 R) under tension and hence in the deformed state. Therefore it
QA ® B = – 2100 R (Heat released) has potential energy (U) which is given by the formula.
ALTERNATE SOLUTION
1
For PVx = Constt., Molar heat capacity U = × stress × strain × volume
2
R R R R
C= + = +
g -1 1 - x 5 1 1 (stress) 2
-1 1 - = ´ ´ pr 2 l
3 2 2 Y
Here P V2 = constant
1 ( Mg / pr 2 ) 2 1 M 2 g 2l
or, PV 1 / 2 = constant = ´ pr 2 l = ... (i)
2 Y 2 p r 2Y
1
\ x= This energy is released in the form of heat, thereby raising
2
Þ C = 3.5 R the temperature of the wire
QA ® B = nC DT = 2 (3.5 R) (300 – 600) = – 2100 R Q = mcDT ... (ii)
Process B – C : Process is isobaric therefore From (i) and (ii), since U = Q, therefore
æ5 ö 1 M 2 g 2l 1 M 2 g 2l
QB ® C = nCp DT = (2) ç R ÷ (TC – TB) \ mcDT = \ DT =
è2 ø 2 p r 2Y 2 p r 2Ymc
æ 5 ö Here, m = mass of string = density × volume of string
= 2 ç R ÷ (2T1 – T1) = (5R) (600 – 300) = 1500 R
è2 ø = r × pr2 l
Heat is absorbed
Process C – A : Process is isothermal 1 M 2 g2
\ DT =
æP ö 2 (p r 2 )2 Yc r
DU = 0 and QC ® A = WC ® A = nRTC ln ç C ÷
è PA ø 1 (100 ´ 10) 2
æ 2P ö = ´
= nR (2T1) ln ç 1 ÷ = (2) (R) (600) ln (2) = 1200 R ´ 0.6932 2 (3.14 ´ 2 ´ 10-3 ) 2 ´ 2.1 ´ 1011 ´ 420 ´ 7860
è P1 ø = 0.00457°C
QC ® A = 831.6 R (absorbed)
HEAT & THERMODYNAMICS AND GASES P-211
41. (a) Since AB is a straight line in V-T graph
distance 2
V Crms = = = 1000 m/s
\ = Constant (Isobaric process) time 1/ 500
T
VA VB 3RT
\ = (a) But Crms =
TA TB M

VB é VB ù 3 ´ 25 / 3 ´ T
TB = ´ TA = 2 × 300 = 600 K êQ = 2ú Þ 1000 = Þ T = 160 K
VA ë V A û 4 ´ 10-3
(b) (i) A to B is a isobaric process (b) Average kinetic energy of an atom of a monoatomic
5 3
\ Q = nCpDT = 2 × R ´ 300 gas = kT
2 2
é 5 ù
êQ C p = 2 R for monoatomic gas ú
= 1500 R
ë û \ Eav =
3
2
3
( )
kT = ´ 1.38 ´ 10-23 ´ 160
2
NOTE : Heat is absorbed as Q is positive.
= 3.312 × 10–21 Joules
(ii) B to C is an isothermal process.
Since the temperature is not changing m
(c) From gas equation PV = RT
\ Internal energy change dU = 0 M
\ From first law of thermodynamics dQ = dW
PVM 100 ´ 1 ´ 4
Vf m= = Þ m = 0.3012 gm
\ Q = 2.303 × nRT log10 RT 25 / 3 ´ 160
Vi 43. When container is stopped, velocity decreases by v0.
= 2.30. × 2 × R × 600 × log10 2
1
= 2763.6 × log10 2 × R = 831.8 R Therefore change in kinetic energy = (nm)v02 ... (i)
NOTE : Heat is absorbed since temperature is same 2
but volume increases. Here n = number of moles of gas present in the container.
(iii) C To D is a isochoric process The kinetic energy at a given temperature for a monatomic
\ dW = 0 3
gas is = ´ nRT
\ Q = nCv DT = n æç R ö÷ ( TA - TB )
3 2
è2 ø
3
3 \ Change in kinetic energy = ´ nR( DT ) .... (ii)
= 2 × R × (– 300) = – 900 R 2
2
Volume is constant as heat is released. where DT = Change in temperature
(iv) D to A is an isothermal process From (i) and (ii)

Vf 3 1 mv02
nR (DT ) = (nm)v02 \ DT =
\ Q = 2.303 × nRT log10 2 2 3R
Vi
44. (a) The rate of heat loss per unit area per second due to
æVf ö radiation is given by Stefan's-Boltzmann law
= 2.303 × 2 R × 300 × log ç ÷ = – 831.8 R
è Vi ø E = e s (T4 – T04 )
Heat is released as Q is positive. 17
(c) Total work done = 0.6 × ´ 10-8[(400)4 - (300)4 ]
3
= QA ® B + QB ® C + QC ® D + QD ® A
= (1500 R + 831.8 R) – (900 R + 831.8 R) = 600 R = 595 Watt/m2
(b) Let Toil be the temperature of the oil.
1 Then rate of heat flow through conduction = Rate of
42. The distance travelled by an atom of helium in sec
500 heat flow through radiation
(time between two successive collision) is 2m. Therefore KA(Toil - T )
root mean square speed = 595 × A
l
where A is the area of the top of lid
595 ´ l 595 ´ 5 ´ 10 -3
ÞToil = +T = + 400 = 419.83 K
k 0.149
45. At constant pressure,
1m T1 T2
we have =
V1 V2
P-212 Topic-wise Solved Papers - PHYSICS
also, V = A × h 48. Initially, at temperature T
T1 T FB = mg
\ = 2 Axrl g = ALrb g
Ah1 Ah2
T h 400 4 FB At T
Þ h2 = 2 1 = ´1 = m
T1 300 3
when the gas is compressed without heat exchange, the ro
process is adiabatic x
g -1 2/5
æV ö æ 4ö
\ T '1 = T2 ç 2 ÷ = 400 ç ÷ K mg
è V1 ø è 3ø
rl
46. Rate of heat produced = F × v
= (6 p h r v) v [Q Viscous F = 6 p h r v]
Þ xr l = Lrb ... (i)
2
é 2 (s - r)r 2 g ù At temperature T + DT
= (6 p h r) ê ú
ëê 9 h ûú F'B
é 2 (s - r)r 2 g ù
êQ Terminal velocity = ú
êë 9 h úû x
Þ Rate of heat produced µ r 5
47. From the figure it is clear that emission takes place from the mg
surface at temperature T2 (circular cross section). Heat
conduction and radiation through lateral surface is zero.
Heat conducted through rod is FB ' = mg
KA(T1 - T2 ) Dt A ' xr 'l g = ALrb g [mg remains the same as above]
Q=
l Now, A' = A (1 + 2aDT)
Energy emitted by surface of rod in same time Dt, is
r 'l = rl (1 - gDT )
4 4
E = e s A (T2 - Ts ) Dt \ A(1 + 2a DT) x rl (1 - gDT ) g = ALrbg
Since rod is at thermal equilibrium therefore E = Q
Þ xrl (1 + 2a DT )(1 - gDT ) = Lrb
KA(T1 - T2 )Dt
hence, = e s A (T24 - Ts4 ) Dt Þ xrl (1 + 2a DT ) (1 - gDT ) = xrl [From (i)]
l
Þ 1 + 2a DT – g DT = 1
e s (T24 - Ts4 ) l Þ g = 2a
Þ T1 – T2 = 49. (a) Heat supplied to the cylinder = Energy used to raise
K
the temperature of cylinder + Energy used for work
Here T2 - Ts = DT and Ts >> DT done by the cylinder.
Energy used to raise the temperature = mcDT
esl é
T1 - ( DT + Ts ) = ( DT + Ts )4 - Ts 4 ùû = 1 × 400 × (T – 20) ... (i)
K ë where T°C is the final temperature of the cylinder.
éæ DT 4 ù Energy used for work done
esl ö = Patm (V2– V1) = 105 (V2 – V1)
T1 - ( DT + Ts ) = ´ Ts 4 êç 1 + ÷ - 1ú
... (ii)
K ëêè Ts ø úû The final volume V2 = V1 [1 + g (T – 20)]
Þ V2 – V1 = V1g (T – 20) ... (iii)
esl é 4D T ù From (ii) and (iii),
T1 - ( DT + Ts ) = ´ Ts 4 ê1 + - 1ú
K Ts Energy used for work done = 105 V1g (T – 20)
ë û
1 é m 1 ù
4es l 3 = 105 × ´ 9 ´ 10-5 (T - 20) êQV1 = =
or T1 – (Ts + DT) = Ts DT 9000 ë d 9000 úû
K
= 0.001 (T – 20) ... (iv)
æ 4 e s l Ts3 ö \ Heat supplied to the cylinder
or T1 – Ts = çç + 1÷ DT = 400 (T – 20) + 0.001 (T – 20)
K ÷
è ø 20,000 = 400.001 (T – 20)
Þ T = 69.99°C » 70°C
æ 4 esl Ts3 ö
\ The proportionality constant = çç 1 + ÷ (b) Work done = 0.001 (69.99 – 20) = 0.0499 J
è K ÷ø (c) Change in internal energy = 20,000 – 0.0499 = 19999.95 J.
HEAT & THERMODYNAMICS AND GASES P-213
50. (1) Heat lost by steam at 100°C to change to 100°C water Here PV2 = Constant. Therefore n = 2
mLvap = 0.05 × 2268 × 1000 = 1,13,400 J R
(2) Heat lost by 100°C water to change to 0°C water \ C = Cv + = Cv – R
1- 2
= 0.05 × 4200 × 100 = 21,000 J
(3) Heat required by 0.45 kg of ice to change its temperature 3
For mono atomic gas Cv = R
from 253 K to 273 K 2
= m × Sice × DT = 0.45 × 2100 × 20 = 18,900 J
(4) Heat required by 0.45 kg ice at 273 K to convert into 3 R
\C= R – R =
0.45 kg water at 273 K 2 2
= mLfusion = 0.45 × 336 × 1000 = 151,200 J Substituting this value in (1) we get
From the above data it is clear that the amount of heat R
required by 0.45 kg of ice at 253 K to convert into 0.45 kg of Q=n × × DT..
2
water at 273 K (1,70,100 J) cannot be provided by heat lost
In this case the temperature decreases i.e. DT is negative.
by 0.05 kg of steam at 373 K to convert into water at 273 K.
Therefore Q is negative. This in turn means that heat is lost
Therefore the final temperature will be 273 K or 0°C. by the gas during the process. (r) is the correct option.
MATCH THE FOLLOWING : (C)-(p, s) : Proceeding in the same way we get in this case
V1/3× T = constant
1. (A) ® (q) : JK is a isovolumic process. Therefore work done Þ As th e gas expands and volume increases, the
is zero. But there is decrease in pressure. Now DQ = DU + temperature decreases. Therefore (p) is the correct option
DW. Therefore DQ = DU. In this case DU = nCvDT and P µT. 4
Since pressure has decreased means temperature has In this process, x = .
3
decreased. Therefore DU is negative and so is DQ.
(B) ® (p, s) : KL is a isobaric process. Pressure is constant. R 3 3R 3 -3R
The volume is increasing therefore DW > 0. Also there is an \C = Cv + = R+ = R–3R =
4 2 -1 2 2
increase in temperature. For both the case heat is absorbed. 1-
3
Therefore DQ > 0.
(C) ® (s) : LM is a isovolumic process. Therefore work æ -3R ö
done is zero. The process is accompanied by increases in
\Q=nç ÷ Δt
è 2 ø
pressure. In this case, the temperature has increased and As DT is negative, Q is positive. This in turn means that
therefore DU > 0. Therefore DQ > 0. heat is gained by the gas during the process (s) is the correct
(D) ® (q, r) : The process MJ is accompained with decrease option.
in volume. Therefore DW < 0. Also from the graph we can D ( PV)
conclude that the temperature in the process decreases. (D)-(q, s) : Also DT =
Therefore DU is also negative nR
Þ DQ < 0. Here D(PV) is positive \ DT is positive
2. (A)-(q) : As the ideal gas expands in vacuum, no work is \ temperature increase s (q) is the correct option
done (W = 0). Also the container is insulated therefore no From the graph it is clear that during the process the pressure
of the gas increases which shows that the internal energy
heat is lost or gained (Q = 0). According to first law of
of the gas has increased. Also the volume increases which
thermodynamics
means work is done by the system which needs energy.
DU = Q + W
From these two interpretation we can comfortably conclude
\ DU = 0
that the gas gains heat during the process.
Þ There is no change in the temparature of the gas
(s) is the correct option.
(B)-(p, r) : Given PV2 = constant ....(i) 3. A-p,r,t; B-p,r; C-q,s; D-r,t
PV (A) Process A ® B
Also for an ideal gas = constant This is an isobaric process in which the volume of the gas
T
From (i) & (ii) V × T = constant decreases. Therefore work is done on the gas.
As the gas expands its volume increases and temperature W = P (3V – V) = 2PV
decreases Also temerature at B is less than temperature at A
\ Heat is lost & internal energy decreases.
\ (p) is the correct option
(p, r, t) are correct matching
To find whether heat is released or absorbed let us find a
(B) Process B ® C
relationship between Q and change in temperature DT.
This is an isovolumic/isochoric process in which the pressure
We know that Q = n C D T ...(i)
decreases
where C = molar specific heat
Here temperature at B is less than temperature at C.
Also for a polytropic process we have \ Heat is lost and internal energy decreases.
R (p, r) are correct matching.
C =Cv + and PVn = constant (C) Process C ® D
1- n
P-214 Topic-wise Solved Papers - PHYSICS
This is isobaric expansion where temperature at D is greater 3. (c) At equilibrium, p = P
than temperature at C. Therefore internal energy increases Þ p = P0 + (L0 – H) rg ... (i)
and heat is gained. Also P0 × (pR2 L0) = p [pR2 (L0 – H)]
(q, s) are correct matching
(D) D ® A
This is a process in which volume decreases. Therefore
P0
work is done on the gas.
Applying PV = nRT
p
9 PV
for D P (9V) = 1 RTD \ TD = L0
R
P H
9 PV P
for A 3P (3V) = 1RTA \ TA =
R
Þ TA = TD
L0 P0
\DU = 0 Þ p= ... (ii)
Now, DQ = DU + W L0 - H
\ DQ = W. From (i) and (ii)
The energy obtained by the gas by work done on it is lost to
the surroundings as D U = 0. L0 P0
= P0 + ( L0 - H ) r g
\ (r, t) are correct matching. L0 - H
4. (a) WGE = P0 (V0 – 32 V0) = – 31 P0V0 Þ L0P0 = P0 (L0 – H) + (L0 – H)2 r g
WGH = P0 (8V0 – 32V0) = – 24 P0V0 Þ rg (L0 – H)2 + P0 (L0 – H) – L0P0 = 0
P0 (8V0 ) - 32P0 (V0 ) 4. (d) The forces acting besides buoyancy force are
(WFH)adiabatic = = 36P0 V0 (a) Force of gravity (vertically downwards)
5
1- (b) Viscous force (vertically downwards)
3
Force due to pressure of the liquid is the buoyant force.
32V0 5. (b) It is given that the bubble does not exchange any heat
(WFG) isothermal = 1(32 P0V0) loge
V0 with the liquid. This means that while the bubble moves
= 32 P0V0 loge 25 up and expand, the process is adiabatic.
= 160 P0V0 loge2 For adiabatic expansion the pressure -temperature
(a) is the correct option relationship is
COMPREHENSION BASED Q UESTIONS : T g P1-g = constt
1. (a) NOTE : When the piston is pulled out slowly, the 1-g
pressure drop produced inside the cylinder is almost \ T g P1-g = T g P 1-g Þ Tg = Tg æ P1 ö
1 2 2 2 1ç ÷
instantaneously neutralised by the air entering from è P2 ø
outside into the cylinder (through the small hole at the
top). 1-g
Therefore, the pressure inside the cylinder is P0 éP ù g
Þ T2 = T1 ê 1 ú
throughout the slow pulling process. ë P2 û
2. (d) KEY CONCEPT : The condition for equilibrium of the
piston is Seal Here T1 = T0, P1 = P0 + H rl g ,
Mg = (P0 – p) pR 2
y 2L 5
p P2 = P0 + ( H - y )rl g , g =
- Mg 3
Þ p= + P0
pR 2 P0 5
1- 5/3
NOTE : Since the cylinder is thermally conducting, the é P0 + H rl g ù 3
temperature remains the same. \ T2 = T0 ê ú
Therefore ë P0 + ( H - y )rl g û
P0 × (2L × pR2) = py × pR2
-2 3
´
P é P0 + H rl g ù 3 5
y = 0 ´ 2L = T0 ê ú
p ë P0 + ( H - y )rl g û

P0 é P ´ pR 2 ù 2
= ´ 2L = ê 0 2 ú ´ 2L é P + ( H - y )rl g ù 5
é Mg ù êë P0 pR - Mg úû T2 = T0 ê 0
P
ê 0 - ú ú
ë pR 2 û ë P0 + H rl g û
HEAT & THERMODYNAMICS AND GASES P-215
6. (b) Buoyancy force = weight of fluid displaced 2. Heat supplied = Heat used in converting m grams of ice
= (mass of fluid displaced) g from –5°C to 0°C + Heat used in converting 1 gram of ice at
= Vrl g ...(ii) 0°C to water at 0°C
where V = Volume of fluid displaced 2100 1 ´ 3.36 ´ 105
= Volume of the bubble. Þ 420 = m ´ ´5+
1000 1000
Now, PV = nRT
Þ 420 = m ´ 10.5 + 336
nRT nRT 84
ÞV = = \m= = 8 grams
P P0 + ( H - y )rl g 10.5 T2
Where P is pressure of the bubble at an arbitrary
location distant y from the bottom. T1
Substituting the value of tempertaure from equtaion (i)
we get
r1= 6cm r2= 18cm
2
3.
nR T [ P + ( H - y )rl g ]5
V= ´ 0 0
[ P0 + ( H - y )rl g ] 2
[ P0 + H rl g ]5 A
B
nRT0 lm = 500 nm lm =1500 nm
= ...(iii) 1 2
3 2
Rate of total energy radiated by A
[ P0 + ( H - y )rl g ]5 [ P0 + H rl g ] 5
Rate of total energy radiated by B
From (ii) and (iii)
4 2
nRT0rl g sT14 (4pr12 )æT ö æ r ö
Buoyancy force = 2 = = ç 1÷ ´ç 1÷
3 sT2 (4pr2 ) è T2 ø è r2 ø
4 2

[ P0 + ( H - y )rl g] 5 [ P0 + H rl g ] 5
4
æ l m ö æ r ö 2 é T1 l m2 ù
÷ ç ÷ êQ =
1
ASSERTION & REASON TYPE QUESTIONS : =ç
2
by Wein 's law ú
è l m1 ø è r2 ø êë T2 l m1 ûú
1. (b) Statement 1 : The total kinetic energy of n moles of gas
4 2
3 æ 1500 ö æ 6 ö
is K = nRT =ç
è 500 ÷ø çè 18 ÷ø
=9
2
But PV = nRT 4. For an adiabatic process, the temperature-volume
\ K = 1.5 PV relationship is
Statement one is true. g -1
Statement 2 : The molecules of a gas collide with each æV ö
other and the velocities of the molecules change due T1V1g -1 = T2V2 g -1 Þ T1 = T2 ç 2 ÷
è V1 ø
to collision.
But statement 2 is not a correct explanation of V1
statement 1. Here g = 1.4 (for diatomic gas). V2 = , T1 = Ti , T2 = aTi
32
INTEGERVALUECORRECTTYPE: é1ù
1.4 -1
\ Ti = aTi ê ú
A 400°C B ë 32 û
0.4
1. 0°C lx 100°C é1ù aTi
\ Ti = aTi ê 5 ú =
ë2 û 4
P \a=4
For heat flow from P to 0 5. (3) We know that
dm1 KA 400 mg / A mg l
Lf = ..... (i) Y= = ....(1)
dt lx Dl / l ADl
For heat flow from P to B Also Dl = l a DT ...(2)
dm2 KA 300 From (1) and (2)
Lvap = ..... (ii)
dt 10 x - lx mg l mg
Y= =
é dm1 dm2 ù Al a DT A a DT
êë Given dt = dt úû
YA a DT 1011 ´ p(10-3 )2 ´ 10-5 ´10
On solving (i) and (ii), we get l = 9 \ m= = =p»3
g 10
P-216 Topic-wise Solved Papers - PHYSICS

1. (a) All reversible engines working for the same temperature 10. (a) The energy radiated per second is given by E = esT 4 A
of source and sink have same efficiencies. If the
temperatures are different, the efficiency is different. For same material e is same. s is stefan's constant
2. (b) Heat required for raising the temperature of the whole
E1 T14 A1 T14 4pr12 (4000)4 ´12 1
body by 1ºC is called the thermal capacity of the body. \ = = = =
4 4 2
3. (b) Pyrometer is used to detect infra-red radiation. E2 T2 A2 T2 4pr2 (2000)4 ´ 42 1
4. (a) Black board paint is quite approximately equal to black 11. (a) This is a statement of second law of thermodynamics
bodies.
5. (a) The temperature depends on the kinetic energy of the 12. (d) P µ T 3 Þ PT -3 = constant ....(i)
gas molecules .
But for an adiabatic process, the pressure temperature
3 relationship is given by
Average kinetic energy per mole = kT
2 g
Higher the velocity of gas molecules, higher is the 1-g g 1-g ....(ii)
average kinetic energy per mole, and higher is the P T = constant Þ PT = constt.
temperature of the gas.
g 3
6. (c) When water is cooled to form ice, energy is released From (i) and (ii) = -3 Þ g = -3 + 3g Þ g =
from water in the form of heat. As energy is equivalent 1- g 2
to mass therefore when water is cooled to ice, its mass 13. (c) Work is a path function. The remaining three parameters
decreases. are state function.
8RT T2 (273 + 27)
7. (d) vrms = h = 1- = 1- 300 1 2
pM 14. (b) = 1- = 1- =
T1 (273 + 627) 900 3 3
TH 2 TO2
For vrms to be equal M =
H2 M O2 W
But h =
Q
Here M H 2 = 2; M O2 = 32 ;
TO 2 = 47 + 273 = 320 K W 2 2 2
\ = Þ W = ´ Q = ´ 3 ´ 106
Q 3 3 3
TH 2 320
\ = Þ TH 2 = 20 K 6 6
2 32 = 2 ´ 106 cal = 2 ´ 10 ´ 4.2 J = 8.4 ´ 10 J
8. (c) In carnot's cycle we assume frictionless piston, absolute 15. (d) Wein’s law correctly explains the spectrum
insulation and ideal source and sink (reservoirs). The 16. (d) According to Newton’s law of cooling
efficiency of carnot's cycle is given by
dQ
T2 - µ (Dq)
h = 1- dt
T1
For h = 1 or 100 %, T2 = 0 K. 5 7
17. (c) g1 = g2 =
The temperature of 0 K (absolute zero) can not be 3 5
obtained . n1 = 1, n2 = 1
9. (c) If n1 moles of adiabatic exponent g1 is mixed with n2
moles of adiabatic exponent g 2 then the adiabatic n1 + n2 n n
= 1 + 2
component of the resulting mixture is given by g -1 g1 - 1 g 2 - 1
1 +1 1 1 3 5
n1 + n2 n n Þ = + = + =4
= 1 + 2 g - 1 5 - 1 7 -1 2 2
g -1 g1 - 1 g 2 - 1
3 5
1 +1 1 1 2 3
= + \ =4 Þ g=
g -1 7 5 g -1 2
-1 -1
5 3
2 2 4
2 5 3
18. (d) E = s AT 4 ; A µ R \ E µ R T
\ = + =4
g -1 2 2
E2 R22 T24
\ =
6 3 E1 R12 T14
\ 2 = 4g - 4 Þ g = =
4 2
HEAT & THERMODYNAMICS AND GASES P-217
put R2 = 2R, R1 = R To measure the radial rate of heat flow, integration
T2 = 2T, T1 = T technique is used, since the area of the surface through
E2 (2R)2 (2T )4 which heat will flow is not constant.
Þ = = 64
E1 R 2T 4 æ dQ ö
r2
1
T2

19. (b) Internal energy and entropy are state function, they Then, ç
è dt ÷ø ò r2 dr = -4 pK ò dT
do not depend upon path taken. r1 T1
20. (a) Here Q = 0 and W = 0. Therefore from first law of
dQ é 1 1 ù
ê - ú = -4pK [T2 - T 1 ]
thermodynamics DU = Q + W = 0
\ Internal energy of the system with partition = dt ë r1 r2 û
Internal energy of the system without partition.
dQ -4pKr1r2 (T2 - T1 )
n1Cv T1 + n2 Cv T2 = (n1 + n2 )Cv T or =
dt (r2 - r1 )
n1T1 + n2T2
\T = dQ r r
n1 + n2 \ µ 1 2
dt ( r2 - r1 )
PV
1 1 PV 24. (b) Change in internal energy do not depend upon the
But n1 = and n2 = 2 2
RT1 RT2 path followed by the process. It only depends on initial
and final states i.e.,
PV
1 1 ´ T + P2V2 ´ T DU1 = DU2
1 2
RT1 RT2 T T ( PV + P V ) 1
\T= = 1 2 11 2 2 25. (d) Q1 = T0 S 0 + T0 S0
PV P V
1 1+ 2 2 1 1T2 + P2V2T1
PV 2
RT1 RT2 T
3
=T0 S 0
21. (d) The thermal resistance 2
x 4x 3x Q2 = T0(2S0 – S0) 2T
+ = 0 Q1
KA 2 KA KA = T0S0
Q3
dQ DT (T2 - T1 ) KA 1 ì A(T2 - T1 ) K ü and Q3 = 0 T0
\ = = = í ý Q2
dt 3x 3x 3î x þ W Q1 - Q2 S
KA h= = S0 2S 0
Q1 Q1
1
\f = Q2 TS
3 1
= 1- = 1- 0 0 =
22. (b, c)First law is applicable to a cyclic process. Concept of Q1 3 3
T S
entropy is introduced by the second law. 2 0 0
n1Cv1 + n2 Cv2
23. (d) T - dT 26. (a) For mixture of gas, Cv =
n1 + n2
dr
3 1 5 5
·
T1
4 ´ R + ´ R 6R + R 29 R ´ 23 29 R
r 2 2 2 = 4
r1 = = =
æ 1 ö 9 9´4 18
4 +
èç 2ø
÷ 2
T2
r2 5R 1 7 R
n1C p1 + n2C p2 4´ + ´
and C p = = 2 2 2
Consider a shell of thickness (dr) and of radii (r) and (n1 + n2 ) æ 1ö
the temperature of inner and outer surfaces of this shell çè 4 + ÷ø
2
be T, (T – dT)
dQ 7
10 R + R
dt
= rate of flow of heat through it
= 4 = 47 R
9 18
KA[(T - dT ) - T ] - KAdT 2
= =
dr dr
Cp 47 R 18
dT 2 2
Þ
\ = ´ = 1.62
= -4pKr (Q A = 4pr ) Cv 18 29 R
dr
P-218 Topic-wise Solved Papers - PHYSICS

27. (b) Total power radiated by Sun = sT 4 ´ 4pR 2 32. (d) T1 l1 l2 T2


A
sT 4 ´ 4pR2
The intensity of power at earth's surface =
4pr 2 K1 K2
Let T be the temperature of the interface. As the two
sT 4 R 2 sections are in series, the rate of flow of heat in them
Total power received by Earth = (pr02 )
r2 will be equal.
28. (c) Heat lost by He = Heat gained by N2 K1 A(T1 - T ) K2 A(T - T2 ) ,
\ =
n1Cv1 DT1 = n2Cv2 DT2 l1 l2
K1T1l 2 + K 2T2 l1 K l T + K 2 l1T2
3 é7 ù 5 é \ T= = 1 2 1 .
R T - Tf ù K 2 l1 + K1l 2 K1l 2 + K 2 l1
2 êë 3 0 úû = 2 R ëT f - T0 û
33. (b) According to Mayer's relationship CP – CV = R
7T0 - 3T f = 5T f - 5T0
CP CV R
\ - =
12 3 M M M
Þ 12T0 = 8T f Þ T f = T0 Þ T f = T0 Here M = 28.
8 2
gRT
nRDT 1000 ´ 8.3 ´ 7 34. (a) The speed of sound in a gas is given by v =
29. (a) W = Þ -146000 = M
1- g 1- g
58.1 58.1 vO2 g O2 M He 1.4 4
or 1 - g = - Þ g = 1+ = 1.4 \ = ´ = ´ = 0.3237
146 146 vHe M O2 g He 32 1.67
Hence the gas is diatomic.
30. (b) For path iaf, Q = 50 cal, W = 20 cal vO 2460
\ vHe = == 1421 m / s
a f 0.3237 0.3237
35. (a) Here Q = 0 and W = 0. Therefore from first law of
thermodynamics DU = Q + W = 0
i b \ Internal energy of the system with partition =
Internal energy of the system without partition.
By first law of thermodynamics,
n1Cv T1 + n2 Cv T2 = (n1 + n2 )Cv T
DU = Q - W = 50 – 20 = 30 cal.
n1T1 + n2 T2
For path ibf \T =
n1 + n2
Q = 36 cal ; W = ?
or, W = Q – DU PV PV
1 1
(Since, the change in internal energy does not depend But n1 = and n2 = 2 2
RT1 RT2
on the path, therefore DU = 30 cal)
\ W = Q – DU = 36 – 30 = 6 cal. PV
1 1 PV
´ T1 + 2 2 ´ T2
31. (c) The efficiency (h) of a Carnot engine and the coefficient RT1 RT2 T T ( PV + P V )
of performance (b) of a refrigerator are related as \T= = 1 2 11 2 2
PV P V
1 1+ 2 2 1 1T2 + P2V2T1
PV
1- h 1 RT1 RT2
b= Here, h =
h 10
36. (a) The heat flow rate is given by
1
1- dQ kA(q1 - q)
10 =
\ b= = 9. dt x
æ 1ö
èç 10 ø÷ x dQ x dQ
Þ q -q = Þ q = q1 -
1 kA dt
kA dt
Q2
Also, Coefficient of performance (b) is given by b =,
W where q1 is the temperature of hot end and q is
where Q2 is the energy absorbed from the reservoir. temperature at a distance x from hot end.
The above equation can be graphically represented by
or, 9 = Q2 \ Q2 = 90 J. option (a) .
10
HEAT & THERMODYNAMICS AND GASES P-219
37. (b) A to B is an isobaric process. The work done 2
W = nR (T2 - T1 ) = 2R (500 - 300) = 400 R \ T = (32) 5 .T Þ T1 = 4T2
1 2
38. (a) Work done by the system in the isothermal process
T2 T2 1 3
PD Now, efficiency = 1 - = 1- = 1 - = = 0.75.
DA is W = 2.303 nRT log10 T1 4T2 4 4
PA
43. (c) Here, work done is zero.
So, loss in kinetic energy = change in internal
1 ´ 105
= 2.303 ´ 2 R ´ 300 log10 = – 414 R energy of gas
2 ´ 105
1 2 R
Therefore work done on the gas is + 414 R. mv = nCv DT = n DT
2 g -1
39. (a) The net work in the cycle ABCDA is
W = W AB + WBC + WCD + WDA 1 2 m R Mv 2 ( g - 1)
mv = DT \ DT = K
2 M g -1 2R
P
= 400 R + 2.303nRT log B + (-400 R) - 414R n1
PC 44. (a) Number of moles of first gas = N
A

2 ´ 105 n2
= 2.303 ´ 2R ´ 500log 5
- 414 R Number of moles of second gas = N
1 ´ 10 A
= 693.2 R – 414 R = 279.2 R
n3
mass 1 Number of moles of third gas = N
40. (a) Volume = = m3 A
density 4 If there is no loss of energy then
5 5 1 P1V1 + P2V2 + P3V3 = PV
K.E = PV = ´ 8 ´ 104 ´ = 5 ´ 10 4 J
2 2 4 n1 n n
RT1 + 2 RT2 + 3 RT3
Alternatively: NA NA NA

5 5m 5 m PM n1 + n2 + n3 n T + n T + n3T3
K.E = nRT = RT = ´ [Q PM = dRT ] = RTmix T = 1 1 2 2
2 2M 2M d NA mix n1 + n2 + n3

5 mP 5 1 ´ 8 ´ 104 45. (d) Efficiency of engine


= = ´ = 5 ´ 104 J
2 d 2 4 T2 T2 5
h1 = 1 - =
41. (c) (The relation R = R0 (1 + a Dt ) is valid for small values T1 Þ T1 6 ....(i)

of Dt and R0 is resistance at 0°C and also (R – R0) T2 - 62 1


should be much smaller than R0. So, statement (1) is Again, h2 = 1 - = ....(ii)
T1 3
wrong but statement (2) is correct.
Solving (i) and (ii), we get,
42. (b) 5
P T1 T1 = 372 K and T2 = × 372 = 310 K
(V, T1) 6
46. (a) DU = DQ = mcDT = 100 × 10–3 × 4184 (50 – 30) » 8.4 kJ
(32 V, T2) 47. (d) Required work = energy released

T2 Here, Q = mc dTò
V 4
æ T3 ö
We have, TV g-1 = constant
= ò 0.1 ´ 32 ´ ç ÷ dT » 0.002 kJ.
è 4003 ø
20

Þ T1V g-1 = T2 (32V )g-1 Þ T1 = (32) g-1.T2 Therefore, required work = 0.002 kJ

7 48. (d)
For diatomic gas, g = P, T
5 Vacuum

2
\ g -1 =
5 It is the free expansion
P-220 Topic-wise Solved Papers - PHYSICS
\ So, T remains constant 52. (a) Newton's law of cooling
Þ PV
1 1 = P2V2 dq
= - k(q - q0 ) Þ dq = - kdt
V dt (q - q0 )
æ Pö
Þ P = P2 (V ) , P2 = ç ÷
2 è 2ø Integrating
Þ log(q - q0 ) = -kt + c
49. (d) DV = V0 g . DT , but g = 3a.
Which represents an equation of straight line.
DV = V0 (3a )DT Thus the option (a) is correct.
53. (c) The efficiency of the engine is given as
4
= p (10) ´ 3 ´ 23 ´ 10 -6 ´ 100
3
æ T ö
3 h = ç 1 - 2 ÷ ´ 100
è T1 ø
DV = 28. 9 cc
For first case
50. (d) The Young modulus is given as
T1 = 500 K; h = 40
F/S
Y= æ T2 ö 40 T2
DL / L 40 = ç 1 - ÷ ´ 100 Þ = 1-
è 500 ø 100 500
Here it is given as
T2 60
F Þ = Þ T2 = 300 K
Y= ´ 2 pR {L = 2pR} 500 100
S2 pDR
For second case :
FR
or Y= …(i) æ 300 ö
S.DR 60
= ç1- T ÷
The coefficient of linear expansion is given as 100 è 2 ø

DR 300 40
a=
RD T T2 =
100
DR R 1
Þ = a.DT Þ = … (ii) 100 ´ 300
R DR aDT Þ T2 = Þ T2 = 750 K
40
From equation (i) and (ii)
54. (b) Heat is extracted from the source in path DA and AB is
F
Y= Þ F = Y.S.aDT 3 æ P0V0 ö 5 æ 2 P0V0 ö
S.aDT DQ = R + R
2 çè R ÷ø 2 çè R ÷ø
\ The ring is pressing the wheel from both sides,
Thus æ 13ö
3 5
Fnet = 2F = 2YSaDT Þ P0V0 + 2 P0V0 = ç ÷ P0V0
2 2 è 2ø
output work
51. (a) The efficiency h = 55. (c) According to Newton’s law of cooling, the temperature
input work goes on decreasing with time non-linearly.
Input work = Work done in going A to B + workdone in
going B to C and the work done in going C to D.
n n
Wi = (P0V0 ) + (2P0V0 ) + 2P0V0
2 2
where n = degree of freedom
which is 3 for mono-atomic gases like He
æ3 3 ö æ 3 + 10 ö 13
= ç + .2 + 2 ÷ P0V0 = ç ÷ P0V0 = P0V0
è2 2 ø è 2 ø 2
and W0 = P0V0
P0V0 2
h= =
13 13
P0V0
2
2 200
Efficiency in % h = ´ 100 = ; 15.4%
13 13
Simple Harmonic Motion
10 (Oscillations)
5. For a particle executing SHM the displacement x is given by
FILL IN THE BLANKS : x = A coswt. Identify the graph which represents the variation
1. An object of mass 0.2 kg executes simple harmonic oscillation of potential energy (PE) as a function of time t and
along the x-axis with a frequency of (25/p) Hz. At the position displacement x (2003S)
x = 0.04, the object has Kinetic energy of 0.5 J and potential I II PE
energy 0.4 J. The amplitude of oscillations is .......m. PE III
(1994 - 2marks)
t IV

MCQ's WITH ONE CORRECT ANSWER : x


1. Two bodies M and N of equal masses are suspended from (a) 1, III (b) II, IV (c) II, III (d) I, IV
6. A simple pendulum has time period T1. The point of
two separate massless springs of spring constants k1 and suspension is now moved upward according to the relation
k2 respectively. If the two bodies oscillate vertically such y = Kt2, (K = 1 m/s2) where y is the vertical displacement.
that their maximum velocities are equal, the ratio of the T12
amplitude of vibration of M to that of N is (1988 - 1mark) The time period now becomes T2. The ratio of is
T22
k1 (g = 10 m/s2) (2005S)
(a) k2 (b) k1 / k 2 (a) 5/6 (b) 6/5 (c) 1 (d) 4/5
7. The x-t graph of a particle undergoing simple harmonic
k2 motion is shown below. The acceleration of the particle at
(c) k (d) k2 / k1 t = 4 / 3 s is (2009)
1
2. A particle free to move along the x-axis has potential energy 1
given by U(x) = k [1–exp(–x2)] for -¥ £ x £ +¥, where k is
x (cm)

a positive constant of appropriate dimensions. Then t(s)


0 4 8 12
(1999S - 2marks)
(a) at points away from the origin, the particle is in unstable
–1
equilibrium
(b) for any finite nonzero value of x, there is a force directed
away from the origin 3 2 –p2
(a) p cm/s 2 (b) cm/s 2
(c) if its total mechanical energy is k/2, it has its minimum 32 32
kinetic energy at the origin.
p2 3 2
(d) for small displacements from x = 0, the motion is simple (c) cm/s 2 (d) – p cm/s2
harmonic 32 32
3. The period of oscillation of a simple pendulum of length L 8. A uniform rod of length L and mass M is pivoted at the
suspended from the roof of a vehicle which moves without centre. Its two ends are attached to two springs of equal
friction down an inclined plane of inclination a , is given by spring constants k. The springs are fixed to rigid supports
as shown in the figure, and the rod is free to oscillate in the
L L horizontal plane.
(a) 2p (b) 2p (2000S) The rod is gently pushed through a small
g cos a g sin a
angle q in one direction and released.
L L The frequency of oscillation is
(c) 2p (d) 2p (2009)
g g tan a
4. A particle executes simple harmonic motion between x = -A
and x = +A. The time taken for it to go from 0 to A/2 is T1 and 1 2k 1 k
(a) (b)
to go from A/2 to A is T2. Then (2001S) 2p M 2p M
(a) T1 < T2 (b) T1 > T2 1 6k 1 24k
(c) T1 = T2 (d) T1 = 2T2 (c) (d)
2p M 2p M
P-222 Topic-wise Solved Papers - PHYSICS
9. The mass M shown in the figure oscillates in simple harmonic 3. A uniform cylinder of length L and mass M having cross
motion with amplitude A. The amplitude of the point P is sectional area A is suspended, with its length vertical, from
a fixed point by a massless spring, such that it is
half- submerged in a liquid of density r at equilibrium
k1 k2 (2009)
M
position. When the cylinder is given a small downward push
P and released it starts oscillating vertically with small
amplitude. If the force constant of the spring is k, the
frequency of oscillation of the cylinder is (1990 - 2mark)
k1A k2 A
(a) (b) 1/ 2 1/ 2
k2 k1 1 æ k - Arg ö 1 æ k + Arg ö
(a) ç ÷ (b) ç ÷
k1 A k2 A 2p è M ø 2p è M ø
(c) k1 + k2 (d) k1 + k2 1/ 2
1/ 2
1 æ k + rgL ö 1 æ k + Arg ö
10. A point mass is subjected to two simultaneous sinusoidal (c) çè ÷ø (d)
displacements in x-direction, x1(t) = A sin wt and x2(t) = 2p M 2 p èç Arg ø÷
4. A highly rigid cubical block A of small mass M and side L is
æ 2p ö
A sin ç wt + ÷ . Adding a third sinusoidal displacement fixed rigidly on to another cubical block B of the same
è 3 ø dimensions and of low modulus of rigidity h such that the
x3(t) = B sin (wt + f) brings the mass to a complete rest. The
values of B and f are (2011) lower face of A completely covers the upper face of B. The
lower face of B is rigidly held on a horizontal surface. A
3p 4p small force F is applied perpendicular to one of the sides
(a) 2 A, (b) A,
4 3 faces of A. After the force is withdrawn, block A executes
5p p small oscillations the time period of which is given by
(c) 3 A, (d) A, (1992 - 2mark)
6 3
11. A small block is connected to one end of a massless spring of
Mh
un-stretched length 4.9 m. The other end of the spring (see (a) 2p M hL (b ) 2p
the figure) is fixed. The system lies on a horizontal frictionless L
surface. The block is stretched by 0.2 m and released from ML M
(c) 2p (d) 2p
rest at t = 0. It then executes simple harmonic motion with h hL
angular frequency w = p/3 rad/s. Simultaneously at t = 0, a
5. One end of a long metallic wire of length L is tied to the
small pebble is projected with speed v form point P at an
ceiling. The other end is tied to a massless spring of spring
angle of 45° as shown in the figure. Point P is at a horizontal
constant K.A mass m hangs freely from the free end of the
distance of 10 m from O. If the pebble hits the block at t = 1 s,
spring. The area of cross-section and the Young’s modulus
the value of v is (take g = 10 m/s2) (2012) of the wire are A and Y respectively. If the mass is slightly
z pulled down and released, it will oscillate with a time period
T equal to: (1993-2marks)

v m(YA + KL)
(a) 2p(m / K )1/ 2 (b) 2p
° YAK
45
O P
x (c) 2p[(mYA / KL)1/ 2 (d) 2p[(mL / YA)1/ 2
10 m 6. A particle of mass m is executing oscillations about the origin
on the x axis. Its potential energy is V(x) = k | x |3 where k is
(a) 50 m/s (b) 51 m/s a positive constant. If the amplitude of oscillation is a, then
its time period T is (1998S - 2marks)
(c) 52 m/s (d) 53 m/s
(a) proportional to 1/ a (b) independent of a
(c) proportional to a (d) proportional to a 3 2
1. A particle executes simple harmonic motion with a frequency. 7. Three simple harmonic motions in the same direction having
f. The frequency with which its kinetic energy oscillates is the same amplitude a and same period are superposed. If
(1987 - 2marks) each differs in phase from the next by 45°, then.
(a) f/2 (b) f (1999S - 3marks)
(c) 2f (d) 4f (a) the resultant amplitude is (1 + 2)a
2. A linear harmonic oscillator of force constant 2 × 106 N/m
(b) the phase of the resultant motion relative to the first is
and amplitude 0.01 m has a total mechanical energy of 160 J.
90°
Its (1989 - 2 Mark)
(c) the energy associated with the resulting motion is
(a) maximum potential energy is 100 J
(b) maximum kinetic energy is 100 J (3 + 2 2 ) times the energy associated with any single
(c) maximum potential energy is 160 J motion
(d) maximum potential energy is zero (d) the resulting motion is not simple harmonic.
SIMPLE HARMONIC MOTION (Oscillations) P-223
2 2 5. Two non-viscous, incompressible and
8. The function x = A sin wt + B cos wt + C sin wt cos wt
represent SHM for which of the option(s) immiscible liquids of densities r and
1.5 r are poured into the two limbs of
(a) for all value of A, B and C (C ¹ 0) (2006 - 5M, –1)
(b) A = B, C = 2B a circular tube of radius R and small
(c) A = –B, C = 2B cross section kept fixed in a vertical R
(d) A = B, C = 0 plane as shown in fig. Each liquid O
9. A metal rod of length ‘L’ and mass ‘m’ is pivoted at one end. occupies one fourth the circumference q
A thin disc of mass ‘M’ and radius ‘R’ (<L) is attached at its of the tube. (1991 - 4 + 4 marks)
center to the free end of the rod. Consider two ways the disc
is attached: (case A). The disc is not free to rotate about its
centre and (case B) the disc is free to rotate about its centre. (a) Find the angle q that the radius to the interface makes
The rod disc system performs SHM in with the vertical in equilibrium position.
vertical plane after being released from
the same displaced position. Which of (b) If the whole is given a small displacement from its
the following statement(s) is (are) true? equilibrium position, show that the resulting
(2011) oscillations are simple harmonic. Find the time period
of these oscillations.
(a) Restoring torque in case A = Restoring torque in
case B 6. Two identical balls A and B each of mass 0.1 kg, are attached
(b) restoring torque in case A < Restoring torque in case B to two identical massless springs. The spring-mass system
(c) Angular frequency for case A > angular frequency for is constrained to move inside a rigid smooth pipe bent in
case B. the form of a circle as shown in Fig. The pipe is fixed in a
(d) Angular frequency for case A < Angular frequency for
case B. horizontal plane.
The centres of the balls can move
SUBJECTIVE PROBLEMS : in a circle of radius 0.06 p meter.
1. A mass M attached to a spring, oscillates with a period of Each spring has a natural length
2sec. If the mass is increased by 2 kg the period increases of 0.06p meter and spring constant
by one sec. Find the initial mass M assuming that Hook’s 0.1 N/m. Initially, both the balls are
Law is obeyed. (1979)
displaced by an angle q = p / 6 A B
2. Two masses m 1 and m 2 are
suspended together by a massless radian with respect to the diameter 6 m
p/6 0.0 p/6
spring of spring constant k. When PQ of the circle (as shown in Fig.) P Q
the masses are in equilibrium, m1 and released from rest.
is removed without disturbing the (1993 - 6 marks)
system. Fin d the angular
frequency and amplitude of
oscillation of m2. (1981 - 3 marks) (i) Calculate the frequency of oscillation of ball B.
m1 (ii) Find the speed of ball A when A and B are at the two
m2 ends of the diameter PQ.
(iii) What is the total energy of the system
3. An ideal gas is enclosed in a vertical cylindrical container
and supports a freely moving piston of mass M. The piston 7. A thin rod of length L and area of cross-section S is pivoted
and the cylinder have equal cross-sectional area A. at its lowest point P inside a stationary, homogeneous and
Atmospheric pressure is p0, and when the piston is in non-viscous liquid . The rod is free to rotate in a vertical
equilibrium, the volume of the gas is V0. The piston is now plane about a horizontal axis passing through P. The density
displaced slightly from its equilibrium position. Assuming d1 of the material of the rod is smaller than the density d2 of
that the system is completely isolated from its surroundings, the liquid. The rod is displaced by a small angle q from its
show that the piston executes simple harmonic motion and equilibrium position and then released. Show that the motion
find the frequency of oscillation. (1981- 6 marks) of the rod is simple harmonic and determine its angular
4. Two light springs of force constants k1 and k2 and a block
of mass m are in one line AB on a smooth horizontal table frequency in terms of the given parameters.
such that one end of each spring is fixed on rigid supports (1996 - 5 Marks)
and the other end is free as shown in the figure. The distance
CD between the free ends of the springs is 60 cms. If the
block moves along AB with a velocity 120 cm/sec in between
the springs, calculate the period of oscillation of the block
(k1 = 1.8 N/m, k2 = 3.2 N/m, m = 200 gm) (1985 - 6 Marks)
60 cm
k1 k2
v
A C m D B
P-224 Topic-wise Solved Papers - PHYSICS
8. A sphere of radius R is half submerged in liquid of density during its subsequent motion after detachment.
r. If the sphere is slightly pushed down and released, find (aw2 > g) (2005 - 4 Marks)
the frequency of oscillation. (2004 - Marks)
9. A small body attached to one end of a vertically hanging
spring is performing SHM about its mean position with
angular frequency w and amplitude a. If at a height y* from
the mean position, the body gets detached from the spring, y
calculate the value of y* so that the height H attained by the yo
mass is maximum. The body does not interact with the spring
m

MATCH THE FOLLOWING :


MUTLIPLE CHOICE QUESTIONS WITH ONE CORRECT
Each question contains statements given in two columns, which have to be matched. The statements in Column-I are labelled A,
B, C and D, while the statements in Column-II are labelled p, q, r, s and t. Any given statement in Column-I can have correct
matching with ONE OR MORE statement(s) in Column-II. The appropriate bubbles corresponding to the answers to these
questions have to be darkened as illustrated in the following example :
If the correct matches are A-p, s and t; B-q and r; C-p and q; and D-s then the correct darkening of bubbles will look like the given.
p q r s t
A p q r s t
B p q r s t
C p q r s t
D p q r s t

1. Column I describes some situations in which a small object moves. Column II describes some characteristics of these motions.
Match the situations in Column I with the characteristics in Column II and indicate your answer by darkening appropriate
bubbles in 4 × 4 matrix given in the ORS. (2007)
Column I Column II
(A) The object moves on the x -axis under a (p) The object executes a simple harmonic motion.
conservative force in such a way that its

''speed " and position'' satisfy v = c c2 - x 2


1
where c1 and c2 are positive constants.
(B) The object moves on the x- axis in such a way that (q) The object does not change its direction.
its velocity and its displacement from the origin
satisfy v = – kx,where k is a positive constant.
(C) The object is attached to one end of a mass-less (r) The kinetic energy of the object keeps on
spring of a given spring constant. The other decreasing.
end of the spring is attached to the ceiling of an
elevator. Initially everything is at rest. The elevator
starts going upwards with a constant acceleration a.
The motion of the object is observed from the elevator
during the period it maintains this acceleration.
(D) The object is projected from the earth's surface (s) The object can change its direction only once.

vertically upwards with a speed 2 GM e / R e ,


where, Me is the mass of the earth and Re is the
radius of the earth, Neglect forces from objects other
than the earth.
SIMPLE HARMONIC MOTION (Oscillations) P-225
2. Column I gives a list of possible set of parameters measured in some experiments. The variations of the parameters in the form of
graphs are shown in Column II. Match the set of parameters given in Column I with the graph given in Column II. Indicate your
answer by darkening the appropriate bubbles of the 4 × 4 matrix given in the ORS. (2008)
Column I Column II
(A) Potential energy of a simple pendulum (y axis) as (p) y
a function of displacement (x axis)

y x

(B) Displacement (y axis) as a function of time (x axis) (q)


for a one dimensional motion at zero or constant
acceleration when the body is moving along the positive
x-direction.
x
O
y
(C) Range of a projectile (y axis) as a function of its (r)
velocity (x axis) when projected at a fixed angle.

x
O
y

(D) The square of the time period (y axis) of a simple (s)


pendulum as a function of its length (x axis)
x
O

COMPREHENSION BASED Q UESTIONS : 2. For periodic motion of small amplitude A, the time period T
of this particle is proportional to
PASSAGE - 1
When a particle of mass m moves on the x-axis in a potential of the m 1 m
form V(x) = kx 2 it performs simple harmonic motion. The (a) A (b)
a A a
m
corresponding time period is proportional to ,as can be seen a 1 a
k (c) A (d)
easily using dimensional analysis. However, the motion of a particle m A m
can be periodic even when its potential energy increases on both 3. The acceleration of this particle for |x| > X0 is
sides of x = 0 in a way different from kx2 and its total energy is
such that the particle does not escape to infinity. Consider a particle V0
of mass m moving on the x-axis. Its potential energy is V(x) = ax4 (a) proportional to V0 (b) proportional to mX
0
(a > 0) for |x| near the origin and becomes a constant equal to V0
for |x| > X0 (see figure). (2010) V0
(c) proportional to (d) zero
mX 0
V(x)
PASSAGE - 2 (2011)
Phase space diagrams are useful tools in analyzing all kinds of
V0
dynamical problems. They are especially useful in studying the
changes in motion as initial position and momenum are changed.
x Here we consider some simple dynamical systems in one
X0
dimension. For such systems, phase space is a plane in which
1. If the total energy of the particle is E, it will perform position is plotted along horizontal axis and momentum is plotted
periodic motion only if
along vertical axis. The phase space diagram is x(t) vs. p(t) curve
(a) E < 0 (b) E > 0
(c) V0 > E > 0 (d) E > V0 in this plane. The arrow on the curve indicates the time flow. For
P-226 Topic-wise Solved Papers - PHYSICS
example, the phase space diagram for a particle moving with
constant velocity is a straight line as shown in the figure. We use
the sign convention in which positon or momentum upwards (or
to right) is positive and downwards (or to left) is negative.
Momentum

4. The phase space diagram for a ball thrown vertically up (a) E1 = 2 E2 (b) E1 = 2E2
from ground is (c) E1 = 4E2 (d) E1 = 16E2
6. Consider the spring-mass system, with the mass submerged
in water, as shown in the figure. The phase space diagram
for one cycle of this system is

(a)

(a)
(b)

(b)
(c)

(c)

(d)

5. The phase space diagram for simple harmonic motion is a


circle centered at the origin. In the figure, the two circles
represent the same oscillator but for different initial (d)
conditions, and E1 and E2 are the total mechanical energies
respectively. Then
SIMPLE HARMONIC MOTION (Oscillations) P-227

1. In a simple harmonic oscillator, at the mean position [2002] 8. A body executes simple harmonic motion. The potential
(a) kinetic energy is minimum, potential energy is maximum energy (P.E), the kinetic energy (K.E) and total energy (T.E)
are measured as a function of displacement x. Which of the
(b) both kinetic and potential energies are maximum following statements is true ? [2003]
(c) kinetic energy is maximum, potential energy is minimum (a) K.E. is maximum when x = 0
(d) both kinetic and potential energies are minimum. (b) T.E is zero when x = 0
2. If a spring has time period T, and is cut into n equal parts, (c) K.E is maximum when x is maximum
then the time period of each part will be [2002] (d) P.E is maximum when x = 0
9. The bob of a simple pendulum executes simple harmonic
(a) T n (b) T / n motion in water with a period t, while the period of oscillation
(c) nT (d) T of the bob is t0 in air. Neglecting frictional force of water and
3. A child swinging on a swing in sitting position, stands up,
given that the density of the bob is (4 / 3) ´ 1000 kg/m 3 .
then the time period of the swing will [2002]
(a) increase What relationship between t and t0 is true [2004]
(b) decrease (a) t = 2t 0 (b) t = t0 / 2
(c) remains same
(c) t = t0 (d) t = 4t 0
(d) increases of the child is long and decreases if the child 10. A particle at the end of a spring executes S.H.M with a
is short period t1. while the corresponding period for another spring
4. A mass M is suspended from a spring of negligible mass. is t2. If the period of oscillation with the two springs in
The spring is pulled a little and then released so that the series is T then [2004]
mass executes SHM of time period T. If the mass is increased (a) T -1 = t1-1 + t 2-1 (b) T 2 = t12 + t 22
5T m
by m, the time period becomes . Then the ratio of is (c) T = t1 + t2 (d) T -2 = t1-2 + t 2-2
3 M
11. The total energy of a particle, executing simple harmonic
[2003] motion is [2004]
3 25 16 5 (a) independent of x (b) µ x2
(a) (b) (c) (d)
5 9 9 3 1/ 2
(c) µ x (d) µ x
5. Two particles A and B of equal masses are suspended from where x is the displacement from the mean position, hence
two massless springs of spring of spring constant k1 and total energy is independent of x.
k2, respectively. If the maximum velocities, during oscillation, 12. A particle of mass m is attached to a spring (of spring constant k)
are equal, the ratio of amplitude of A and B is [2003] and has a natural angular frequency w0. An external force
F(t) proportional to cos wt (w ¹ w 0 ) is applied to the
k1 k2 k2 k1 oscillator. The time displacement of the oscillator will be
(a) (b) k (c) (d) k
k2 1 k1 2 proportional to [2004]
1 1
6. The length of a simple pendulum executing simple harmonic (a) 2 2 (b) 2 2
m (w 0 + w ) m (w 0 - w )
motion is increased by 21%. The percentage increase in the
time period of the pendulum of increased length is [2003] m m
(c) 2 2 (d) 2 2
(a) 11% (b) 21% (c) 42% (d) 10% w0-w (w 0+w )
7. The displacement of a particle varies according to the 13. In forced oscillation of a particle the amplitude is maximum
for a frequency w1 of the force while the energy is maximum
relation x = 4(cos pt + sin pt ). The amplitude of the particle for a frequencyw2 of the force; then [2004]
is [2003] (a) w1 < w2 when damping is small and w1 > w2 when
damping is large
(a) – 4 (b) 4
(b) w1 > w2
(c) 4 2 (d) 8 (c) w1 = w2
(d) w1 < w2
P-228 Topic-wise Solved Papers - PHYSICS
14. Two simple harmonic motions are represented by the 20. Two springs, of force constants k1 and k2 are connected to
p a mass m as shown. The frequency of oscillation of the
equations y1 = 0.1 sin æç100pt + ö÷ and y 2 = 0.1 cos pt . mass is f. If both k1 and k2 are made four times their original
è 3ø
values, the frequency of oscillation becomes
The phase difference of the velocity of particle 1 with respect
m
to the velocity of particle 2 is [2005] k1 k2
p -p p -p
(a) (b) (c) (d)
3 6 6 3 (a) 2 f (b) f /2 [2007]
(c) f /4 (d) 4 f
15. The function sin 2 (wt ) represents [2005]
21. A particle of mass m executes simple harmonic motion with
(a) a periodic, but not simple harmonic motion with a period amplitude a and frequency n. The average kinetic energy
p during its motion from the position of equilibrium to the end
w is [2007]
(b) a periodic, but not simple harmonic motion with a period p 2 ma 2 n2
(a) 2p 2 ma 2 n2 (b)
2p
1 2 2
w (c) ma n (d) 4p 2 ma 2 n2
4
p 22. The displacement of an object attached to a spring and
(c) a simple harmonic motion with a period
w executing simple harmonic motion is given by x = 2 × 10–2
cos pt metre.The time at which the maximum speed first
2p
(d) a simple harmonic motion with a period occurs is [2007]
w (a) 0.25 s (b) 0.5 s
16. The bob of a simple pendulum is a spherical hollow ball (c) 0.75 s (d) 0.125 s
filled with water. A plugged hole near the bottom of the 23. A point mass oscillates along the x-axis according to the
oscillating bob gets suddenly unplugged. During observa-
tion, till water is coming out, the time period of oscillation law x = x0 cos(wt - p / 4) . If the acceleration of the particle
would [2005] is written as a = A cos(wt + d ) ,then [2007]
(a) first decrease and then increase to the original value
(b) first increase and then decrease to the original value (a) A = x0 w 2 , d = 3p / 4
(c) increase towards a saturation value
(d) remain unchanged (b) A = x0, d = -p / 4

d x
2 (c) A = x0 w 2 , d = p / 4
17. If a simple harmonic motion is represented by
2
+ ax = 0 ,
dt 2
(d) A = x0 w , d = -p / 4
its time period is [2005]
24. If x, v and a denote the displacement, the velocity and the
2p 2p acceleration of a particle executing simple harmonic motion
(a) (b)
a a of time period T, then, which of the following does not
change with time? [2009]
(c) 2p a (d) 2pa (a) aT/x (b) aT + 2pv
18. The maximum velocity of a particle, executing simple (c) aT/v (d) a2T2 + 4p2v2
harmonic motion with an amplitude 7 mm, is 4.4 m/s. The
25. Two particles are executing simple harmonic motion of the
period of oscillation is [2006]
same amplitude A and frequency w along the x-axis. Their
(a) 0.01 s (b) 10 s (c) 0.1 s (d) 100 s
mean position is separated by distance X0(X0 > A). If the
19. Starting from the origin a body oscillates simple harmonically maximum separation between them is (X0 + A), the phase
with a period of 2 s. After what time will its kinetic energy be difference between their motion is: [2011]
75% of the total energy? [2006]
p p
1 1 (a) (b)
(a) s (b) s 3 4
6 4
p p
1 1 (c) (d)
6 2
(c) s (d) s
3 12
SIMPLE HARMONIC MOTION (Oscillations) P-229
26. A mass M, attached to a horizontal spring, executes S.H.M. 29. The amplitude of a damped oscillator decreases to 0.9 times
with amplitude A1. When the mass M passes through its its original magnitude in 5s. In another 10s it will decrease to
mean position then a smaller mass m is placed over it and
a times its original magnitude, where a equals
both of them move together with amplitude A2. The ratio of
(a) 0.7 (b) 0.81 [JEE Main 2013]
æ A1 ö
çè A ÷ø is: [2011] (c) 0.729 (d) 0.6
2
30. An ideal gas enclosed in a vertical cylindrical container
1
M +m supports a freely moving piston of mass M. The piston and
(b) æç ö2
M
(a)
M è M + m ÷ø the cylinder have equal cross sectional area A. When the
piston is in equilibrium, the volume of the gas is V0 and its
1
æ M + mö 2 M pressure is P0. The piston is slightly displaced from the
(c) çè ÷ (d)
M ø M +m equilibrium position and released. Assuming that the system
is completely isolated from its surrounding, the piston executes
27. A wooden cube (density of wood ‘d’) of side ‘ l ’ floats in
a liquid of density ‘r’ with its upper and lower surfaces a simple harmonic motion with frequency [JEE Main 2013]
horizontal. If the cube is pushed slightly down and released,
it performs simple harmonic motion of period ‘T’ 1 AgP0 1 V0 MP0
(a) (b)
[2011RS] 2p V0 M 2p A 2 g

ld lr
(a) 2p (b) 2p
1 A 2 gP0 1 MV0
rg dg (c) (d)
2p MV0 2p AgP0
ld lr
(c) 2p (d) 2p
(r - d ) g (r - d ) g
28. If a simple pendulum has significant amplitude (up to a factor
of 1/e of original) only in the period between t = 0s to t = t s,
then t may be called the average life of the pendulum.
When the spherical bob of the pendulum suffers a
retardation (due to viscous drag) proportional to its velocity
with b as the constant of proportionality, the average life
time of the pendulum is (assuming damping is small) in
seconds : [2012]
0.693
(a) (b) b
b
1 2
(c) (d)
b b
P-230 Topic-wise Solved Papers - PHYSICS

Solutions & Explanations


Section-A : JEE Advanced/ IIT-JEE
A 1. 0.06
C 1. (d) 2. (d) 3. (a) 4. (a) 5. (a) 6. (b) 7. (d) 8. (c) 9. (d)
10. (b) 11. (a)
D 1. (c) 2. (b, c) 3. (b) 4. (d) 5. (b) 6. (a) 7. (a, c) 8. (a, b, c) 9. (a, d)

k m1 g A p0 g 1 2p 1.803R
E 1. 1.6 kg 2. , 3. 4. 2.83 sec. 5. (a) q = tan -1 (b)
m2 k 2p V0 M 5 g

1 -1 3(d 2 - d1 ) g
6. (i) s (ii) 0.02p m/s (iii) 3.95 × 10–4 J 7.
p 2d1 L

1 3g g
8. 9. y* =
2p 2 R w2

F 1. (A) ® p; (B) ® q, r; (C) ® p; (D) ® q, r 2. (A) ® p; (B) ® q, r, s; (C) ® s; (D) ® q


G 1. (c) 2. (b) 3. (d) 4. (d) 5. (c) 6. (b)

Section-B : JEE Main/ AIEEE


1. (c) 2. (b) 3. (b) 4. (c) 5. (c) 6. (d) 7. (c) 8. (a) 9. (a) 10. (b) 11. (a) 12. (b)
13. (c) 14. (b) 15. (c) 16. (b) 17. (a) 18. (a) 19. (a) 20. (a) 21. (b) 22. (b) 23. (a) 24. (a)
25. (a) 26. (c) 27. (a) 28. (d) 29. (c) 30. (c)

Given that v1 = v2
FILL IN THE BLANKS :
2p 2p
1. x = 0.04 m, K.E. = 0.5 J and P.E. = 0.4 J \ a1 × = a2 ´
T1 T2
T.E. = (0.5 + 0.4) J = 0.9 J
1 1 m
Now, T.E. = mw 2 a 2 = m ´ 4 p 2 v 2 a 2 2p
2 2 a1 T1 k1 k2
Þ = = =
1 25 25 a2 T2 m k1
Þ 0.9 = ´ 0.2 ´ 4p 2 ´ ´ ´ a2 2p
2 p p k2
3 2. (d) Let us plot the graph of the mathematical equation
Þ a= = 0.06m
50
dU 2
U (x) = K éê1 - e - x ùú , F = -
2
= 2 kxe - x
MCQ's WITH ONE CORRECT ANSWER : ë û dx

1. (d) Both the bodies oscillate in simple harmonic motion, U(x)


for which the maximum velocities will be
2p
v1 = a1w1 = a1 × ... (i) 1
T1 C B

2p A
v2 = a2w2 = a2 × ... (ii) x=0 x
T2
SIMPLE HARMONIC MOTION (Oscillations) P-231
From the graph it is clear that the potential energy is 7. (d) From the graph it is clear that the amplitude is 1 cm and
minimum at x = 0. Therefore, x = 0 is the state of stable the time period is 8 second. Therefore the equation for
equilibrium. Now if we displace the particle from x = 0 the S.H.M. is
then for displacements the particle tends to regain the æ 2p ö æ 2p ö
x = a sin ç ÷ ´ t = 1sin ç ÷ t
2kx èT ø è 8ø
position x = 0 with a force F = . Therefore for small
2 p
ex x = sin t
values of x we have F µ x. 4
3. (a) As shown in the figure, gsina is the pseudo acceleration The velocity (v) of the particle at any instant of time ‘t’
applied by the observer in the accelerated frame is
Y dx d é æ p ö ù p æ pö
v= = sin ç ÷ t = cos ç ÷ t
dt dt êë è 4 ø úû 4 è 4ø
The acceleration of the particle is
g sin a sin a g sin a X
2
a d2x æ pö æ pö
g cos a sin a = - ç ÷ sin ç ÷ t
dt 2 è 4ø è 4ø
g
4
At t = s we get
a
3
2
d2x æ pö p 4 -p2 p
ax = gsinacosa = - ç ÷ sin ´ = sin
ay = g – gsin2a = g(1 – sin2 a) = gcos2a dt 2 è 4ø 4 3 16 3

ax2 + a 2y - 3p 2
a= = cm / s 2
32
8. (c) lq
= g 2 sin 2 a cos 2 a + g 2 cos 4 a
2
= g cosa sin 2 a + cos2 a = g cos a
q
L l/2
\ T = 2p M
g cos a
NOTE : Whenever point of suspension is accelerating
L lq
use T = 2p
geff 2
Figure shows the rod at an angle q with respect to its
4. (a) The velocity of a body executing S.H.M. is maximum at equilibrium position. Both the springs are stretched
its centre and decreases as the body proceeds to the
lq
extremes. Therefore if the time taken for the body to go by length .
from O to A/2 is T1 and to go from A/2 to A is T2 then 2
obviously The restoring torque due to the springs
t = – 2 (Restoring force) × perpendicular distance
T1 < T2
5. (a) NOTE : In S.H.M., at extreme position, P.E. is maximum æ lq ö l l2
when t = -2k ç ÷ ´ = - k q ... (i)
è 2 ø 2 2
t = 0, x = A. If I is the moment of inertia of the rod about M then
i.e., at time t = 0, the particle executing S.H.M. is at its
extreme position. d 2q
t = Ia = I … (ii)
Therefore P.E. is max. The graph I and III represent the dt 2
above characteristics. From (i) & (ii) we get
6. (b) y = kt2 d 2q l 2q
I = -k d 2q k l2 -k l2
2
d y 2 Þ = - q = q
dy dt 2 dt 2 I 2 M l 2 /12 2
\ = 2 kt or = 2k
dt dt 2 d 2q 6k
Þ =- q
or ag = 2m / s 2 (Q k = 1 m/s2 given) dt 2 M
Comparing it with the standard equation of rotational
l SHM we get
We know that T = 2p
g
d 2q
= -w2q Þ w 2 = 6k Þ w = 6k
T12 g2 T12 12 6 dt 2 M M
\ = Þ = =
T22 g1 T22 10 5 6k 1 6k
Þ 2 pv = Þ v=
[Q g1 = 10 m/s2 and g2 = g + 2 = 12 m/s2] M 2p M
P-232 Topic-wise Solved Papers - PHYSICS
9. (d) 2. (b, c) The total energy of the oscillator
k1 k2 1 2
= kA = Max. K.E.
M 2
P
1 6 2
= ´ 2 ´ 10 ´ (0.01) = 100 J
Case (i) 2
As total mechanical energy = 160 J
x1 The P.E. at equilibrium position is not zero.
P.E. at mean position = (160 – 100) J = 60 J
A \ Max P.E. = (100 + 60 ) J = 160 J.
M Extreme Mean
P
position position
Case (ii)
K.E. = 0 K.E. = 100J
In case (ii), the springs are shown in the maximum P. E. = 160J P. E. = 60J
3. (b) If x is the displacement then,
compressed position. If the spring of spring constant
k1 is compressed by x1 and that of spring constant k2 \ M w 2 x = [ rAg + k ] x
is compressed by x2 then 1/2 1/2
é rAg + k ù 1 é rAg + k ù
x1 + x2 = A Þ w= ê Þ v=
… (i)
ë M û
ú 2p êë M úû
k1 x1 4. (d) NOTE : When a force is applied on cubical block A in
and k1 x1 = k2 x2 Þ x2 = …(ii) the horizontal direction then the lower block B will get
k2
distorted as shown by dotted lines and A will attain a
From (i) & (ii) new position (without distortion as A is a rigid body)
as shown by dotted lines.
k x k A
x1 + 1 1 = A Þ x1 = 2
k2 k2 + k1
A
10. (b) Two sinusoidal displacements have amplitude A each, F
L
p L
with a phase difference of 2 . It is given that
3 DL
L
sinusoidal displacement x3(t) brings the mass to a
complete rest. This is possible when the amplitude of B Fixed

p For cubical block B


third is A and is having a phase difference of 4 with
3 F/A F L F L F
h= = ´ = 2´ =
respect to x1 (t) as shown in the figure. DL / L A DL L D L L ´ DL
Þ F = hLDL
hL is a constant
2p /3 Þ Force F µ DL and directed towards the mean position,
oscillation will be simple harmonic in nature.
A A Here, Mw2 = hL
2p /3 hL 2p hL M
Þ w= Þ = Þ T = 2p
4p /3 M T M hL
A 5. (b) Let us consider the wire also as a spring. Then the case
becomes that of two spring attached in series. The
2V sin q equivalent spring constant is
11. (a) T =
g 1 1 1
= +
keq k k '
2V sin 45º where k' is the spring constant of the wire
\ 1=
g F/A F L
Now, Y = = ´
DL / L A D L
\ v = 50 ms -1 F YA
or = or, k ' = YA
DL L L
We know that time period of the system
m é1 1 ù
T = 2p = 2p m ê + ú
1. (c) NOTE : During one complete oscillation, the kinetic keq ë k k 'û
energy will become maximum twice.
Therefore the frequency of kinetic energy will be 2f. é1 L ù m (YA + kL )
Þ T = 0 2p m ê + ú = 2p
ë k YA û kYA
SIMPLE HARMONIC MOTION (Oscillations) P-233
6. (a) U (x) = k | x |3 9. (a,d)
Applying t = I a
[U ] ML2T -2
\ [k] = = = ML-1T -2 æl ö
[ x3 ] L3 For case A : mg ç sin q÷ + Mg (l sin q) = I A a A
è2 ø
Now time period may depend on T µ (mass) x
(amplitude)y (k)z
\ [M0L0T] = [M]x [L]y [ML–1 T–2]z
= [Mx + z Ly – z T – 2z]
Equating the powers, we get l/2 q
– 2z = 1 or z = – 1/2
y–z=0
or y = z = – 1/2 l/2 sin q
Hence T µ (amplitude) –1/2 µ a–1/2 mg
7. (a, c) From superposition principle l/2
y = y1 + y2 + y3
= a sin wt + a sin (wt + 45°) + a sin (wt + 90°) lsin q
= a[sin wt + sin (wt + 90°] + a sin (wt + 45°)
= 2a sin (wt + 45°) cos 45° + a sin (wt + 45°)
= ( 2 + 1 ) a sin (wt + 45°) Mg
= A sin (wt + 45°)
Therefore resultant motion is simple harmonic of
æl ö
amplitude A = ( 2 + 1 ) a For case B : mg çè sin q÷ø + Mg (l sin q) = I B a B
2
and which differ in phase by 45° relative to the first. The restoring torque in both the cases is same.
Energy in SHM µ (amplitude)2 é Q E = mA2 w 2 ù
1 Also I A > I B \ a A < a B
ê 2 ú
ë û \ wA < wB
2 (a) and (d) are correct options.
Eresultant æ A ö
\ = ç ÷ = ( 2 + 1) 2 = (3 + 2 2)
Esingle è aø
SUBJECTIVE PROBLEMS :
\ Eresultant = (3 + 2 2) Esingle
1.
8. (a, b, c) The given equation is
x = A sin2 wt + B cos2 wt + C sin wt cos wt
The time period T of the spring is T = 2p M
NOTE THIS STEP k
Rearranging the equation in a meaningful form (for
interpretation of SHM) or, 2 = 2 p M .... (i)
k
A 2 B 2 C
x = (2sin wt ) + (2 cos wt ) + (2sin wt cos wt ) In the second case
2 2 2
A B C M +2
= [1 - cos 2wt ] + [1 + cos 2wt ] + [sin 2wt ] 3 = 2p ... (ii)
2 2 2 k
From (i) and (ii)
C
(a) For A = 0 and B = 0, x = sin (2wt )
2 2 M 4 M
= Þ = Þ M = 1.6 kg.
C 3 M +2 9 M +2
The above equation is that of SHM with amplitude 2.
2
and angular frequency 2w. Thus option (a) is correct.
(b) If A = B and C = 2B then x = B + B sin 2wt
This is equation of SHM. The mean position of the
particle executing SHM is not at the origin.
Option (b) is correct.
(c) A = – B, C = 2B; Therefore m 1

x = B cos 2wt + B sin 2wt m 2


m 2

Let B = X cos f = X sin f then


x = X sin 2wt cos f + X cos 2wt sin f NOTE
This represents equation of SHM. When mass m1 is removed then the equilibrium will get
(d) A = B, C = 0 and x = A. This equation does not represents disturbed. There will be a restoring force in the upward
SHM. direction. The body will undergo S.H.M. now.
P-234 Topic-wise Solved Papers - PHYSICS
Let x1 be the extension of the spring when ( m1 + m2 ) are F æ Mg ö gAx æ Mg ö gA2 x
Þ = ç p0 + ÷ Þ F = ç p0 + ÷
suspended and x2 be the extension of spring when m1 is A è A ø V0 è A ø V0
removed.
æ Mg ö gA2 x Mg ö gA2 x
( m1 + m2 ) g Þ Ma = ç p0 +
æ
Þ a = ç p0 +
\ kx1 = ( m1 + m2 ) g or x1 = è ÷
A ø V0 è ÷
A ø V0 M
k
Comparing it with a = w x, we get
2
m2 g
and, kx2 = m2 g or x2 = Mg ö gA2
k æ
w= çè p0 + ÷
Amplitude of oscillation = x1 - x2 A ø V0 M

( m1 + m2 ) g - m2 g m1 g If
Mg
is small as compared to p0 then
or A = = A
k k
Let at any instant the mass m2 be having a displacement x p0 gA2 A p0 g
from the mean position. Restoring force will be w= = 2pf \ f=
V0 M 2p V0 M
k
F = – k x or m2a = – k x Þ a = – x 4. The mass will strike the right spring, compress it. The K.E.
m2 of the mass will convert into P.E. of the spring. Again the
Comparing this with a = – w2x , spring will return to its natural size thereby converting its
P.E. to K.E. of the block. The time taken for this process will
k k
we get w2 = Þ w= T m
m2 m2 be , where T = 2p .
2 k
3. Let the piston be displaced by a distance x
60 cm
æ Mg ö g æ Mg ö k1 k2
Then ç p0 + ÷ø V0 = çè p0 + + p÷ (V0 - Ax ) g v
è A A ø
A C m D B
Mg
[Q Initial pressure on the gas P1 = p0 + T m 0.2
A \ t1 = =p =p = 0.785 sec
2 k2 3.2
Mg The block will move from A to B without any acceleration.
Final pressure on the gas P2 = p0 + + p]
A The time taken will be
60
t2 = = 0.5
120
A Now the block will compress the left spring and then the
spring again attains its natural length. The time taken will be
x
m 0.2
t3 = p =p = 1.05 sec.
V0 k1 1.8
Again the block moves from B to A, completing one
oscillation. The time taken for doing so
where p is the extra pressure due to which the compression
x takes place. 60
t4 = = 0.5
Final volume of the gas V2 = V0 – Ax 120
The above equation can be rearranged as \ The complete time of oscillation will be
= t1 + t2 + t3 + t4
æ Mg ö é ù
çè p0 + + p ÷ (V0 - Ax) g ê ú é Ax ù
g = 0.785 + 0.5 + 1.05 + 0.5
A ø p = 2.83 (app.)
1= =ê1 + úê 1 - ú
æ
p +
Mg ö g
V ê p0 + Mg ú ë V0 û 5.
çè 0 ÷ 0 ë A û
A ø

æ ö
p gAx ç p ÷ æ gAx ö
Þ 1 = 1+ - +
Mg V0 ç Mg ÷ çè V0 ÷ø O
R
p0 + ç p0 ÷ M o
A è A ø q
45
Q
°-q
The last term is negligible as p and x are small 45
K
p gAx æ Mg ö gAx P N m2g
\ = \ p = ç p0 + ÷
Mg V0 è A ø V0 m1g
p0 +
A At equilibrium, taking torque of liquids about O
SIMPLE HARMONIC MOTION (Oscillations) P-235

æ Torque due to ö æ Torque due to ö \ Force exerted by each spring on each ball = 2 kx
çè liquid of density r ÷ø = çè liquid of density1.5 r ÷ø \ Total force on each ball due to two springs = 4 kx
\ Restoring torque about origin O = - ( 4kx ) R
m2g × QM = m1g × PN
\ m2g R sin (45° + q ) = m1gR sin (45° – q ) \ t = -4k ( Rq) R, where q = Angular displacement
VrgR sin (45° + q ) = 1.5VrgR sin (45° – q ) ... (i)
or t = -4 kR 2 q
sin (45 + q) Since torque (t) is proportional to q, each ball executes
Þ = 1.5
sin (45 - q) angular SHM about the centre O.
sin 45° cos q + cos 45° sin q 3 Again, t = -4kR 2 q
Þ =
sin 45° cos q - cos 45° sin q 2 or I a = -4 kR 2q where a = angular acceleration
1
Þ tan q =
5 or ( mR ) a = -4kR q or a = - æçè 4mk ö÷ø q
2 2

NOTE THIS STEP : Let us now displace the liquids in


anticlockwise direction along the circumference of tube 1 a
through an angle a. \ Frequency f =
The net torque 2p q
t = m2gR sin (45° + q + a) – m1gR sin (45° – q – a) 1 4k
= VrgR sin (45° + q + a) – 1.5VrgR (45° – q – a) \ Frequency of each ball =
2p m
= VrgR sin ( q + 45°) cos a + VrgR cos (45° + q )
sin a – 1.5 VrgR sin (45° – q) cos a 1 4 ´ 0.1 1
+ 1.5 VrgR cos(45° – q) sin a = = sec -1 ..(ii)
2p 0.1 p
Using eq. (i) we get
(ii) Let velocity at the mean position be vmax.
t = V rgR éëcos ( 45° + q ) sin a + 1.5cos ( 45° - q ) sin aùû Loss in elastic potential energy = Gain in kinetic energy
t = VrgR [cos (45° + q ) + 1.5 cos (45° – q )] sin a é1 æ pö ù
2
when a is small (given) é1 2 ù
2 ê K ç 2 R ÷ ú = 2 ´ ê mvmax ú
\ sin a » a êë 2 è 6 ø úû ë2 û
t = VrgR [cos (45° + q ) + 1.5 cos (45° – q )]a
Since, t and a are proportional and directed towards mean K Rp
position. \ vmax = ´ = 0.02 p m/s
m 3
\ The motion is simple harmonic.
Moment of inertia about O is é1 2 ù
I = VrR2 + 1.5 VrR2 (iii) Total energy = 2 ê mvmax úû
ë2
I
T = 2p é1 2ù
C = 2 ê ´ 0.1(0.02p) ú
ë 2 û
(V r ´ 2.5R 2 ) = 3.95 × 10–4 J
= 2p 7. Let q be the angle made by the rod at any instant t.
[cos(45 + q) + 1.5cos(45 - q)]V rgR
The volume of the rod = LS
1.803R 1 Weight of the rod = LS d1g
= 2p (using the value tan q = ) Upthrust acting on rod = LS d2g
g 5
6. (i) As both the balls are displaced by an angle q = p / 6
radian with respect to the diameter PQ of the circle and
released from rest. It results into compression of spring LSd 2g
in upper segment and an equal elongation of spring in
lower segment. Let it be x. PB and QA denote x in the
figure. M
q
q
B A L/2
LSd 1g
p = Rq P N
q q d2
Q
P R
Since, d2 > d1 (given). Therefore net force acting at the
centre of mass of the rod at tilted position is
(LS d2g – LSd1g)
Taking moment of force about P
Compression = Rq = elongation = x
P-236 Topic-wise Solved Papers - PHYSICS
t = (LSd2g – LSd1g) × PN After detaching from spring, net downward acceleration of
where PN = perpendicular distance of line of action of force the block = g.
from P \ Height attained by the block = h

\ t = LSg (d2 – d1) ×


L
sin q
\ h= y+
v2
or h = y +
w 2
(a 2
- y2 )
2 2g 2g
when q is small, sin q » q
dh
For h to be maximum, = 0, y = y *.
L2 Sg dy
\ t= (d 2 - d1 )q. Since, t µ q
2
w2 2
( -2 y *) or 0 = 1 - 2w y *
\ Motion is simple harmonic. dh
\ = 1+
On comparing it with t = C q dy 2g 2g

L2 Sg w2 y * g
We get C= (d 2 - d1 ) or = 1 or y * =
2 g w2
L2 Sg Since aw 2 > g ( given )
Þ Iw2 = (d 2 - d1 ) ... (i)
2
g
The moment of inertia I of the rod about P \ a> \ a > y *. y * from mean position < a.
2 w2
1 æ Lö
= ML2 + M ç ÷ g
12 è 2ø Hence y* = .
w2
1 1
I= ML2 = LSd1L2 ... (ii) MATCH THE FOLLOWING :
3 3
From (i) and (ii) 1. A®p
3 2
L L Sg 2 2
Reason : For a simple harmonic motion v = a w - x . On
w2 ´ Sd1 = (d2 - d1 )
3 2
comparing it with v = c1 c2 - x 2 we find the two
3Sg (d 2 - d1 ) 3(d2 - d1 ) g
Þ w= Þ w= comparable.
2 LSd1 2d1 L
B ® q, r
8. At equilibrium net force is zero. Reason : v = – k x
\ Fmg = Fbouyancy when x is positive; v is – ve, and as x decreases, v decreases.
4 3 2 3 r Therefore kinetic energy will decreases. When x = 0, v = 0.
or r s ´ pr ´ g = r ´ pr ´ g Þ r s =
3 3 2 Therefore the object does not change its direction.
Let the sphere is slightly displaced downward by x. When x is negative, v is positive. But as x decreases in
\ Fres = – pR2 xrg magnitude, v also decreases. Therefore kinetic energy
[NOTE : Q Volume of submerged portion of sphere increases decreases. When x = 0, v = 0. Therefore the object does not
by pR2 x, hence bouyont force increases by pR2 xrg] change its direction.
C®p
-pR 2 rg 3g
\ a= =- x Reason : When a = 0, let the spring have an extension x.
4 3 2R Then k x = mg.
pR rs
3 When the elevator starts going upwards with a constant
F 4 3 acceleration, as seen by the observer in the elevator, the
[Q acceleration a = and m = pR rs ] object is at rest.
m 3
\ ma + mg = k x'
Q a µ - x \ motion is simple harmonic.
Þ ma = k (x' – x) (Since a is constant)
3g D ® q, r
\ w= [comparing with a = – w2x]
2R The speed is 2 times the escape speed. Therefore the
Thus frequency of oscillation object will leave the earth. It will therefore not change the
1 3g direction and its kinetic energy will keep on decreasing.
or n= 2. A® p
2p 2 R
When simple pendulum is displaced from the mean position
9. If a small mass is attached to one end of a vertically hanging
spring then it performs SHM. towards any of the extreme position, its potential energy
Angular frequency = w, Amplitude = a increases. In case of a S.H.M. we get a parabola for potential
energy versus displacement
Under SHM, velocity v = w a 2 - y 2 B ® q, r, s
SIMPLE HARMONIC MOTION (Oscillations) P-237
S = ut for zero acceleration. Therefore we get a straight line
-dV ( x)
passing through the origin. Therefore option (q) is correct. 3. (d) F=
dx
1 As V (x) = constant for x > X0
S = ut + at2 for constant positive acceleration. In this
2 \F=0 for x > X0
case we get a part of parabola as a graph line between s Since F = 0, a = 0
versus t as shown by graph (s). 4. (d) When the ball is thrown upwards, at the point of throw
(p) is ruled out because if a is –ve and v is positive. (O) the linear momentum is in upwards direction (and
R has a maximum value ) and the position is zero. As the
time passes, the ball moves upwards and its momentum
C®s goes on decreasing and the position becomes positive.
u The momentum becomes zero at the topmost point (A).
u 2 sin 2q Momentum
R= Þ R ¥ u2 (for consant q and g)
g
l
D ® q T = 2π \T2¥ l 0 A
g Position
T 2

l
As the time increases, the ball starts moving down
with an increasing linear momentum in the downward
COMPREHENSION BASED Q UESTIONS : direction (negative) and reaches back to its original
position.
1. (c) If the energy is zero, the particle will not perform
These characteristics are represented by graph (d).
oscillations. Therefore E should be greater than zero.
5. (c) We know that the
Further if E = V0, the potential energy will become
constant as depicted in the graph given. In this case total mechanical energy µ (Amplitude)2
also the particle will not oscillate. \ E1 µ (2a)2
Therefore E should be greater than zero but less & E2 µ a2
than V0. E1
2. (b) We can get the answer of this question with the help of \ =4
E2
dimensional analysis.
Given potential energy = ax4 6. (b) As the mass is osicillating in water its amplitude will go
on decreasing and the amplitude will decrease with
Potential energy ML2T -2 time. Options (c) and (d) cannot be true.
a= = = [ ML-2T -2 ]
x 4
L4 When the position of the mass is at one extreme end in
the positive side (the topmost point), the momentum is
1 m 1 M zero. As the mass moves towards the mean position
Now = =T
A a L ML-2T -2 the momentum increases in the negative direction.
Therefore option (b) is correct. These characteristics are depicted in option (b).

1. (c) KEY CONCEPT : The kinetic energy (K. E.) and When the spring is cut into n equal parts, the spring
potential energy (U) of a simple harmonic oscillator is constant of one part becomes nk. Therefore the new
given by, time period,
1 1
K.E = k ( A2 - x 2 ) ; U = kx 2 m T
2 2 T ' = 2p =
Where A = amplitude and k = mw2 nk n
x = displacement from the mean position l
At the mean position x = 0 3. (b) KEY CONCEPT : The time period T = 2p where
1 2 g
\ K.E. = kA = Maximum and U = 0 l = distance between the point of suspension and the
2
2. (b) Let the spring constant of the original spring be k. centre of mass of the child.
m As shown in the figure, l ¢ < l
Then its time period T = 2p where m is the mass of
k
oscillating body. \ T ¢ < T i.e., the period decreases.
P-238 Topic-wise Solved Papers - PHYSICS
point of suspension l l
9. (a) t = 2p ; t 0 = 2p
g eff g

l' Buoyant
l force 1000 Vg

4
´1000 Vg
3
Case (ii) child standing Case (i) child sitting Weight
M æ4 ö 1000
4. (c) T = 2p Net force = ç - 1÷ ´1000 Vg = Vg
k è3 ø 3
1000 Vg g
M + m 5T g eff = = l
T ' = 2p = 4 4 \ t = 2p
k 3 3 ´ ´ 1000 V g/4
3
M +m 5 M 25 t = 2t0
\ 2p = ´ 2p M +m = ´M
k 3 k 9 m
10. (b) For first spring, t1 = 2p ,
m 25 m 25 16 k1
1+ = Þ = -1 =
M 9 M 9 9
5. (c) Maximum velocity during SHM = Aw m
For second spring, t 2 = 2 p
But k = mw2 k2
k k1k 2
\ w=
m when springs are in series then, keff =
kl + k 2
k
\ Maximum velocity = A m ( kl + k 2 )
m \ T = 2p
Here the maximum velocity is same and m is also same k1k 2

A1 k2 m m t2 t
2 2
\ A1 k1 = A2 k 2 \ =
A2 k1 \ T = 2p + = 2p + 1
k 2 k1 2 2
(2p) (2p)
l 1.21l 2
Þ T = t1 + t 2
2 2
6. (d) T = 2p and T ' = 2p
g g where x is the displacement from the mean position
(Q l ' = l + 21% of l) 11. (a) At any instant the total energy is
1 2
T '- T kA = constant, where A0 = amplitude
% increase = ´ 100 2 0
T hence total energy is independent of x.
1.21l - l 12. (b) Equation of displacement is given by
=
l
´ 100 = ( )
1.21 - 1 ´ 100
x = A sin(wt + f)
= (1.1 - 1) ´ 100 = 10% where A =
F0
=
F0
2 2
7. (c) x = 4(cos pt + sin pt ) 2
m (w 0 - w )
2 2 m (w 0 - w )
æ sin pt cos pt ö here damping effect is considered to be zero
= 2 ´ 4ç + ÷ 1
è 2 2 ø \x µ 2 2
m (w 0 - w )
= 4 2(sin p t cos 45° + cos p t sin 45°)
13. (c) Since energy µ ( Amplitude)2, the maximum for both
x = 4 2 sin(p t + 45°) of them occurs at the same frequency
on comparing it with x = A sin(wt + f) \ w1 = w2
we get A = 4 2 dy1 æ pö
14. (b) v1 = = 0.1 ´ 100p cos ç100pt + ÷
1 dt è 3ø
8. (a) K.E. = mw 2 (a 2 - x 2 )
2 dy 2 æ pö
v2 = = - 0.1p sin pt = 0.1p cos ç pt + ÷
1 è 2ø
When x = 0, K.E is maximum and is equal to mw 2 a 2 . dt
2
p p 2 p - 3p p
\ Phase diff. = f1 - f 2 = - = =–
3 2 6 6
SIMPLE HARMONIC MOTION (Oscillations) P-239
15. (c) Clearly sin2 wt is a periodic function as sin wt is
1 K1 + K 2
p or, f= ....(i)
periodic with period 2p m
w When both k1 and k2 are made four times their original
values, the new frequency is given by
1 4 K1 + 4 K 2
f '=
0 p/w 2p/w 3p/w 2p m
2 æ 1 K1 + K 2 ö
d y 1 4( K1 + 4K 2 )
µ -y = = 2ç
m ÷ø
For SHM 2
dt 2p m è 2p
dy =2 f from eqn. (i)
= 2w sin wt cos wt = w sin 2wt 21. (b) KEY CONCEPT : The kinetic energy of a particle
dt
executing S.H.M. is given by
2
d y 2 1 2 2 2
= 2w cos 2wt which is not proportional to –y. K= ma w sin wt
2
dt 2
Hence it is not in SHM. where, m = mass of particle
16. (b) Centre of mass of combination of liquid and hollow a = amplitude
portion (at position l ), first goes down ( to l + D l) and w = angular frequency
when total water is drained out, centre of mass regain t = time
its original position (to l ), Now,
1
l average K.E. = < K > = < mw2 a2 sin2 wt >
T = 2p 2
g 1
\ ‘T’ first increases and then decreases to original = mw2a2 <sin2 wt >
2
value.
1 æ 1ö æ 2 1ö
= mw2a2 çè ÷ø çè Q < sin q > = ÷ø
2 2 2
1 2 2 1
= mw a = ma2 (2pn)2 (Q w = 2pn )
4 4
c
2 2 2
or, < K > = p ma n
2 22. (b) Here, x = 2 × 10–2 cos p t
d x 2 Speed is given by
17. (a) = -ax = -w x
dt 2 dx
v= = 2 × 10–2 p sin p t
2p 2p dt
Þ w = a or T = = For the first time, the speed to be maximum,
w a
p
18. (a) Maximum velocity, sin p t = 1 or, sin p t = sin
2
2p
vmax = a w , vmax = a ´ p 1
T Þ pt = or,, t = = 0.5 sec.
-3
2 2
2pa 2 ´ 3.14 ´ 7 ´ 10 23. (a) Here,
ÞT = = » 0.01 s
vmax 4.4 x = x0 cos (wt – p / 4 )
19. (a) K.E. of a body undergoing SHM is given by, dx æ pö
\ Velocity, v = = - x0 w sin ç wt - ÷
1 2 2 2 1 2 2 dt è 4ø
K .E. = ma w cos wt , T .E. = ma w Acceleration,
2 2
Given K.E. = 0.75 T.E. dv 2 æ pö
a= = - x0 w cos ç wt - ÷
2 p dt è 4ø
Þ 0.75 = cos wt Þ wt =
6 é pö ù
æ
p p´2 1 = x0 w 2 cos ê p + ç wt - ÷ ú
Þt= Þt= Þt= s ë è 4øû
6´w 6 ´ 2p 6
20. (a) The two springs are in parallel. æ 3p ö
= x0 w 2 cos ç wt + ÷ ...(1)
\ Effective spring constant, è 4ø
K = K1 + K2 Acceleration, a = A cos (wt + d) ...(2)
Now, frequency of oscillation is given by Comparing the two equations, we get
1 K 3p
A = x0w2 and d = .
f = 4
2p m
P-240 Topic-wise Solved Papers - PHYSICS
2
24. (a) For an SHM, the acceleration a = -w2 x where w is d2 x b dx k
a Þ 2
+ + x=0 … (1)
a constant. Therefore is a constant. The time period dt m dt m
x
aT Let x = e lt is the solution of the equation (1)
T is also constant. Therefore is a constant.
x
(a) Let, x1 = A sin w t and x = A sin (w t + f)
2 dx d2 x
25. = le l t Þ = l 2 e lt
æ fö f dt dt 2
x2 – x1 = 2A cos çè wt + 2 ÷ø sin 2 Substituting inthe equation (1)
As the maximum separation between the particles is A, b k
l 2 e lt + le lt + e lt = 0
f p m m
\ 2 A sin =A Þ f=
2 3 b k
26. (c) The net force becomes zero at the mean point. l2 + l+ =0
Therefore, linear momentum must be conserved. m m

\ Mv1 = (M + m)v2 MA1


k
= ( M + m) A2
k b b2 k
- ± -4
M m+M m m 2 m -b ± b 2 - 4km
l= =
æ k ö 2 2m
\ çV = A M ÷ Q A1 M = A2 M + m
è ø -b
t
Solving the equation (1) for x, we have x = 2
e m
A1 m+M
\ =
A2 M k l = +b
w = w0 2 - l 2 where w0 = ,
27. (a) m 2
1 2
The average life = =
d l b
l
bt
-
2m
29. (c) Q A = A0 e (where, A0 = maximum amplitude)
l0 r According to the questions, after 5 second,
b(5)
-
0.9A 0 = A 0 e 2m … (i)
At equilibrium After 10 more second,
Fb = mg b(15)
-
rAl 0g = dAlg ....(i) A = A0 e 2m …(ii)
From eqns (i) and (ii)
A = 0.729 A0 \ a = 0.729
d Mg
30. (c) = P0 P0V0 g = PV g
A
Mg = P0A … (1) P0 Ax0 g = PA( x0 - x) g
l0+ x
g
P0 x0
P=
Restoring force, ( x0 - x )g Piston
F = mg – Fb' Let piston is displaced by distance x
F = mg - rA ( l 0 + x ) g æ P xg ö
Mg - ç
0 0 x
rg ÷ A = Frestoring
çè ( x - x) g ÷ø
dAla = dAlg - rAl 0 g - rgAx a = - x 0 x0 Cylinder
dl containing
Therefore, wooden cube performs S.H.M. æ x0g ö ideal gas
rg ld P0 A ç1 - ÷ = Frestoring [ x0 - x » x0 ]
\ w= Þ T = 2p çè ( x - x )g ÷ø
0
dl rg
28. (d) The equation of motion for the pendulum, suffering gP0 Ax
retardation F=-
x0
F = – kx - bv
\ Frequency with which piston executes SHM.
d2 x dk d2 x k b dx
Þ m 2 + kx + b =0 Þ + x+ =0 1 gP0 A 1 gP0 A2
dt dt dt 2 m m dt f = =
2p x0 M 2p MV0
11 Waves

8. A plane progressive wave of frequency 25 Hz, amplitude


FILL IN THE BLANKS : 2.5 × 10–5 m and initial phase zero propagates along the
negative x-direction with a velocity of 300 m/s. At any instant,
1. A travelling wave has the frequency n and the particle the phase difference between the oscillations at two points
displacement amplitude A. For the wave the particle velocity 6 m ap ssart along the line of propagation is ...., and the
corresponding amplitude difference is ....... m.
amplitude is __________and the particle acceleration (1997C - 1 Mark)
amplitude is________. (1983 - 2 Marks)
TRUE / FALSE :
2. Sound waves of frequency 660 Hz fall normally on a perfectly
reflecting wall. The shortest distance from the wall at which 1. A man stands on the ground at a fixed distance from a siren
which emits sound of fixed amplitude. The man hears the
the air particles have maximum amplitude of vibration is ..... sound to be louder on a clear night than on a clear day.
metres. (1984- 2 Marks) (1980)
3. Two simple harmonic motions are represented by the 2. A plane wave of sound travelling in air is incident upon a
plane water surface. The angle of incidence is 60°. Assuming
equations snell’s law to be valid for sound waves, it follows that the
sound wave will be refracted into water away from the normal.
y1 = 10sin (3pt + p / 4) and y2 = 5(sin 3pt + 3 cos3pt) (1984- 2 Marks)
3. A source of sound with frequency 256 Hz is moving with a
Their amplitudes are in the ratio of .......... (1986 - 2 Marks) velocity V towards a wall and an observer is stationary
4. In a sonometer wire, the tension is maintained by suspending between the source and the wall. When the observer is
between the source and the wall he will hear beats
a 50.7 kg mass from the free end of the wire. The suspended (1985 - 3 Marks)
mass has a volume of 0.0075 m 3 . The fundamental frequency MCQ's WITH ONE CORRECT ANSWER :
of vibration of the wire is 260 Hz. If the suspended mass is 1. A cylindrical tube open at both ends, has a fundamental
completely submerged in water, the fundamental frequency frequency ‘f’ in air. The tube is dipped vertically in air. The
will become .............Hz. (1987 - 2 Marks) tube is dipped vertically in water so that half of it is in water.
The fundamental frequency of the air column in now
5. The amplitude of a wave disturbance propagating in the (1981- 2 Marks)
1 f 3f
positive x-direction is given by y = at time t = 0 and (a) (b)
(1 + x ) 2 2 4
(c) f (d) 2f
1 2. A wave represented by the equation y = a cos (kx - wt ) is
by y = at t = 2 seconds, where x and y are superposed with another wave to form a stationary wave
[1 + ( x - 1) 2 ] such that point x = 0 is a node. The equation for the other
wave is (1988 - 1 Mark)
in metres. The shape of the wave disturbance does not
(a) a sin (kx + wt ) (b) – a cos (kx - wt )
change during the propagation. The velocity of the wave is
.............. m/s. (1990 - 2 Marks) (c) – a cos (kx + wt ) (d) – a sin (kx - wt )
6. A cylinder resonance tube open at both ends has 3. An object of specific gravity r is hung from a thin steel wire.
The fundamental frequency for transverse standing waves
fundamental frequency F in air. Half of the length of the in the wire is 300 Hz. The object is immersed in water so that
tube is dipped vertically in water. The fundamental frequency one half of its volume is submerged. The new fundamental
to the air column now is .................... (1992 - 1 Mark) frequency in Hz is (1995S)
1/ 2 1/ 2
7. A bus is moving towards a huge wall with a velocity of æ 2r - 1 ö æ 2r ö
5 ms–1. The driver sounds a horn of frequency 200 Hz. The (a) 300çç ÷÷ (b) 300çç ÷÷
è 2r ø è 2r - 1 ø
frequency of the beats heard by a passenger of the bus will
be..... Hz (Speed of sound in air = 342 ms–1) æ 2r ö æ 2r - 1 ö
(c) 300çç ÷÷ (d) 300çç ÷÷
(1994 - 2 Marks) è 2r - 1 ø è 2r ø
P-242 Topic-wise Solved Papers - PHYSICS
4. A wave disturbance in a medium is described by 12. Two pulses in a stretched string whose centers are initially
æ pö 8 cm apart are moving towards each other as shown in the
y ( x, t ) = 0.02 cos ç 50pt + ÷ cos(10 px) where x and y are figure. The speed of each pulse is 2 cm/s. After 2 seconds,
è 2ø
the total energy of the pulses will be (2001S)
in metre and t is in second (1995S)
(a) A node occurs at x = 0.15 m
(b) An antinode occurs at x = 0.3 m
(c) The speed wave is 5 ms–1
(d) The wave length is 0.3 m
5. The extension in a string, obeying Hooke’s law, is x. The 8 cm
speed of sound in the stretched string is v. If the extension
in the string is increased to 1.5x, the speed of sound will be (a) zero
(1996 - 2 Marks) (b) purely kinetic
(a) 1.22v (b) 0.61v (c) purely potential
(c) 1.50v (d) 0.75v (d) partly kinetic and partly potential
6. An open pipe is suddenly closed at one end with the result 13. The ends of a stretched wire of length L are fixed at x = 0 and
that the frequency of third harmonic of the closed pipe is
x = L. In one experiment, the displacement of the wire is
found to be higher by 100Hz than the fundamental frequency
y1 = A sin (px/L) sin wt and energy is E1 and in another
of the open pipe. The fundamental frequency of the open
pipe is (1996 - 2 Marks) experiment its displacement is y2 = A sin (2px/L) sin 2wt and
(a) 200 Hz (b) 300 Hz energy is E2. Then (2001S)
(c) 240 Hz (d) 480 Hz (a) E2 = E1 (b) E2 = 2E1
7. A whistle giving out 450 Hz approaches a stationary (c) E2 = 4E1 (d) E2 = 16 E1
observer at a speed of 33 m/s. The frequency heard by the 14. A siren placed at a railway platform is emitting sound of
observer in Hz is (1997C - 1 Mark)
frequency 5 kHz. A passenger sitting in a moving train A
(a) 409 (b) 429
records a frequency of 5.5 kHz while the train approaches
(c) 517 (d) 500
the siren. During his return journey in a different train B he
8. A travelling wave in a stretched string is described by the
equation y = A sin (kx –wt) The maximum particle velocity is records a frequency of 6.0 kHz while approaching the same
(1997 - 1 Mark) siren. The ratio of the velocity of train B to that train A is
(a) Aw (b) w / k (a) 242/252 (b) 2 (2002S )
(c) dw / dk (d) x / t (c) 5/6 (d) 11/6
9. A train moves towards a stationary observer with speed 34
m/s. The train sounds a whistle and its frequency registered 15. A sonometer wire resonates with a given tuning fork forming
by the observer is f1. If the train's speed is reduced to 17 standing waves with five antinodes between the two bridges
m/s, the frequency registered is f2. If the speed of sound is when a mass of 9 kg is suspended from the wire. When this
340 m/s, then the ratio f1/f2 is (2000S ) mass is replaced by a mass M, the wire resonates with the
(a) 18/19 (b) 1/2 same tuning fork forming three antinodes for the same
(c) 2 (d) 19/18 positions of the bridges. The value of M is (2002S )
10. Two vibrating strings of the same material but lengths L and
(a) 25 kg (b) 5 kg
2L have radii 2r and r respectively. They are stretched under
the same tension. Both the strings vibrate in their (c) 12.5 kg (d) 1/25 kg
fundamental nodes, the one of length L with frequency v1 16. A police car moving at 22 m/s, chases a motorcyclist. The
and the other with frequency v2. The raio v1/v2 is given police man sounds his horn at 176 Hz, while both of them
by (2000S ) move towards a stationary siren of frequency 165 Hz.
(a) 2 (b) 4 Calculate the speed of the motorcycle, if it is given that he
(c) 8 (d) 1
does not observes any beats. (2003S)
11. Two monatomic ideal gases 1 and 2 of molecular masses m1
and m2 respectively are enclosed in separate containers
Police Car Motorcycle
kept at the same temperature. The ratio of the speed of sound
in gas 1 to that in gas 2 is given by (2000S )
Stationary
22 m/s
m1 m2 V Siren
176 Hz (165 Hz)
(a) m2 (b) m1
(a) 33m/s (b) 22m/s
m1 m2
(c) m2 (d) m1 (c) zero (d) 11m/s
WAVES P-243
17. In the experiment for the determination of the speed of sound 22. In the experiment to determine the speed of sound using a
in air using the resonance column method, the length of the resonance column, (2007)
air column that resonates in the fundamental mode, with a
tuning fork is 0.1 m. When this length is changed to 0.35 m, (a) prongs of the tuning fork are kept in a vertical plane
the same tuning fork resonates with the first overtone. (b) prongs of the tuning fork are kept in a horizontal
Calculate the end correction. (2003S) plane
(a) 0.012 m (b) 0.025 m
(c) in one of the two resonances observed, the length of
(c) 0.05 m (d) 0.024 m the resonating air column is close to the wavelength
18. A pipe of length l1, closed at one end is kept in a chamber of sound in air
of gas of density r1. A second pipe open at both ends is
placed in a second chamber of gas of density r2. The (d) in one of the two resonances observed, the length of
compressibility of both the gases is equal. Calculate the the resonating air column is close to half of the
length of the second pipe if frequency of first overtone in wavelength of sound in air
both the cases is equal (2004S) 23. A transverse sinusoidal wave moves along a string in the
4 r 4 r
positive x-direction at a speed of 10 cm/s. The wavelength of
(a) l1 2 (b) l1 1 the wave is 0.5 m and its amplitude is 10 cm. At a particular time
3 r1 3 r2
t, the snap–shot of the wave is shown in figure. The velocity of
r2 r1
point P when its displacement is 5 cm is – (2008)
(c) l1 (d) l1 y
r1 r2
19. In a resonance tube with tuning fork of frequency 512 Hz,
first resonance occurs at water level equal to 30.3 cm and P
second resonance occurs at 63.7 cm. The maximum possible
error in the speed of sound is (2005S) x
(a) 51.2 cm/s (b) 102.4 cm/s
(c) 204.8 cm/s (d) 153.6 cm/s
20. An open pipe is in resonance in 2nd harmonic with
frequency f 1 . Now one end of the tube is closed and 3p ˆ 3p
frequency is increased to f2 such that the resonance again (a) jm/s (b) - ĵ m / s
occurs in nth harmonic. Choose the correct option 50 50
(2005S)
3p 3p
(c) î m / s (d) - î m / s
3 5 50 50
(a) n = 3, f 2 = f1 (b) n = 3, f 2 = f1
4 4 24. A vibrating string of certain length l under a tension T
resonates with a mode corresponding to the first overtone
3 5
(c) n = 5, f 2 = f1 (d) n = 5, f 2 = f1 (third harmonic) of an air column of length 75 cm inside a
4 4 tube closed at one end. The string also generates 4 beats
21. A massless rod of length L is suspended by two identical per second when excited along with a tuning fork of
strings AB and CD of equal length. A block of mass m is frequency n. Now when the tension of the string is slightly
suspended from point O such that BO is equal to ‘x’. Further increased the number of beats reduces 2 per second.
it is observed that the frequency of 1st harmonic in AB is Assuming the velocity of sound in air to be 340 m/s, the
equal to 2nd harmonic frequency in CD. ‘x’ is frequency n of the tuning fork in Hz is (2008)
(2006 - 3M, –1) (a) 344 (b) 336
(c) 117.3 (d) 109.3
A C
25. A hollow pipe of length 0.8 m is closed at one end. At its
open end a 0.5 m long uniform string is vibrating in its
second harmonic and it resonates with the fundamental
frequency of the pipe. If the tension in the wire is 50 N and
O the speed of sound is 320 ms–1, the mass of the string is
B D
x L (a) 5 grams (b) 10 grams (2010)
(c) 20 grams (d) 40 grams
m
26. A police car with a siren of frequency 8 kHz is moving with
uniform velocity 36 km/hr towards a tall building which reflects
L 4L the sound waves. The speed of sound in air is 320 m/s. The
(a) (b)
5 5 frequency of the siren heard by the car driver is (2011)
3L L (a) 8.50 kHz (b) 8.25 kHz
(c) (d) (c) 7.75 kHz (d) 7.50 kHZ
4 4
P-244 Topic-wise Solved Papers - PHYSICS
27. A student is performing the experiment of resonance column. 8. A wave is represented by the equation
The diameter of the column tube is 4 cm. The frequency of
the tuning fork is 512 Hz. The air temperature is 38°C in p
y = A sin (10 p x + 15 p t +)
which the speed of sound is 336 m/s. The zero of the meter 3
scale coincides with the top end of the resonance column where x is in meters and t is in seconds. The expression
tube. When the first resonance occurs, the reading of the represents : (1990 - 2 Marks)
water level in the column is (2012) (a) a wave travelling in the positive x-direction with a
(a) 14.0 cm (b) 15.2 cm velocity 1.5 m/s.
(c) 16.4 cm (d) 17.6 cm
(b) a wave traveling in the negative x-direction with a
velocity 1.5 m/s.
(c) a wave travelling in the negative x-direction having a
1. A wave equation which gives the displacement along the
wavelength 0.2 m.
y-direction is given by y = 10–4 sin (60t + 2x) where x and y are
in metres and t is time in seconds. This represents a wave (d) a wave travelling in the positive x-direction having a
(1982 - 3 Marks) wavelength 0.2 m.
(a) travelling with a velocity of 30 m/s in the negative x 9. Two identical straight wires are stretched so as to produce
direction 6 beats per second when vibrating simultaneously. On
(b) of wavelength pm changing the tension slightly in one of them, the beat
(c) of frequency 30/p hertz frequency remains unchanged. Denoting by T1 , T2 the
(d) of amplitude 10–4 m traveling along the negative
x-direction higher and the lower initial tension in the strings, then it
2. A tr ansverse wave is descr ibed by the equation could be said that while making the above changes in
tension, (1991 - 2 Marks)
æ xö
y = y0 sin 2p ç ft - ÷ . The maximum particle velocity is (a) T2 was decreased (b) T2 was increased
è lø
equal to four times the wave velocity if (1984- 2 Marks) (c) T1 was decreased (d) T1 was increased
y0 y0 10. The displacement y of a particle executing periodic motion
(a) l = p (b) l = p
4 2 æ1 ö
(c) l = py0 (d) l = 2py0 is given by y = 4cos 2 ç t ÷ sin(1000 t )
è2 ø
3. An air column in a pipe, which is closed at one end, will be
in resonance with a vibrating tuning fork of frequency 264 This expression may be considereed to be a result of the
Hz if the length of the column in cm is : (1985 - 2 Marks) superposition of (1992 - 2 Marks)
(a) 31.25 (b) 62.50 (a) two (b) three
(c) 93.75 (d) 125 (c) four (d) five
4. A tube, closed at one end and containing air, produces, 11. A sound wave of frequency f travels horizontally to the
when excited, the fundamental note of frequency 512 Hz. If right. It is reflected from a large vertical plane surface moving
the tube is open at both ends the fundamental frequency
that can be excited is (in Hz) (1986 - 2 Marks) to left with a speed v. The speed of sound in medium is C
(a) 1024 (b) 512 (1995S)
(c) 256 (d) 128 (a) The number of wave striking the surface per second is
5. The displacement of particles in a string stretched in the (c + v )
x-direction is represented by y. Among the following f
expressions for y, those describing wave motion are : c
(1987 - 2 Marks)
c( c - v )
(a) cos kx sin wt (b) k x - w 2 t 2
2 2 (b) The wavelength of reflected wave is
f (c + v )
(c) cos 2 (kx + wt ) (d) cos (k2x2 – w2t2)
(c + v)
6. An organ pipe P1 closed at one end vibrating in its first (c) The frequency of the reflected wave is f
(c - v)
harmonic and another pipe P2 open at ends vibrating in its (d) The number of beats heard by a stationary listener to
third harmonic are in resonance with a given tuning fork.
vf
The ratio of the length of P1 to that of P2 is the left of the reflecting surface is
(1988 - 2 Marks) c-v
12. –2
A string of length 0.4 m and mass 10 kg is tightly clamped
(a) 8/3 (b) 3/8
(c) 1/6 (d) 1/3 at its ends. The tension in the string is 1.6 N. Identical wave
7. Velocity of sound in air is 320 m/s. A pipe closed at one end pulses are produced at one end at equal intervals of time,
has a length of 1 m. Neglecting end corrections, the air Dt. The minimum value of Dt which allows constructive
column in the pipe can resonate for sound of frequency : interference between successive pulses is(1998 - 2 Marks)
(1989 - 2 Marks) (a) 0.05 s (b) 0.10 s
(a) 80 Hz (b) 240 Hz
(c) 320 Hz (d) 400 Hz (c) 0.20 s (d) 0.40 s
WAVES P-245
13. The (x, y) co-ordinates of the corners of a square plate are (c) the amplitude of vibration of the ends of the prongs is
(0, 0), (L, 0), (L, L) and (0, L). The edges of the plate are typically around 1 cm
clamped and transverse standing waves are set up in it. If (d) the length of the air-column at the first resonance was
u(x, y) denotes the displacement of the plate at the point somewhat shorter than 1/4th of the wavelength of the
(x, y) at some instant of time, the possible expression(s) for sound in air
u is (are) (a = positive constant) (1998 - 2 Marks) 20. A person blows into open-end of a long pipe. As a result, a
(a) a cos (px/2L) cos (py/2L) high pressure pulse of air travels down the pipe.
(b) a sin (px/L) sin (py/L) When this pulse reaches the other end of the pipe,
(c) a sin (px/L) sin (2py/L) (2012)
(d) a cos (2px/L) sin (py/L) (a) a high-pressure pulse starts travelling up the pipe, if
the other end of the pipe is open.
14. A transverse sinusoidal wave of amplitude a, wavelength l
(b) a low-pressure pulse starts travelling up the pipe, if the
and frequency f is travelling on a stretched string. The
other end of the pipe is open.
maximum speed of any point on the string is v/10, where v is
(c) a low-pressure pulse starts travelling up the pipe, if the
the speed of propagation of the wave. If a = 10–3m and
other end of the pipe is closed.
v = 10 m s–1, then l and f are given by (1998 - 2 Marks)
(d) a high-pressure pulse starts travelling up the pipe, if
(a) l = 2 p ´ 10 - 2 m (b) l = 10 - 3 m the other end of the pipe is closed.
21. A horizontal stretched string, fixed at two ends, is vibrating
(c) f = 103 Hz /(2p) (d) f = 104 Hz in its fifth harmonic according to the equation, y(x, t) = (0.01
15. y(x, t) = 0.8/[4x+5t)2+5] represents a moving pulse, where x m) sin [(62.8 m–1)x] cos[(628 s–1)t]. Assuming p = 3.14, the
and y are in meter and t in second. Then correct statement(s) is (are) (JEE Adv. 2013)
(1999 - 3 Marks) (a) The number of nodes is 5
(a) pulse is moving in + x direction (b) The length of the string is 0.25 m
(b) in 2 s it will travel a distance of 2.5 m (c) The maximum displacement of the midpoint of the string,
from its equilibrium position is 0.01 m
(c) its maximum displacement is 0.16 m
(d) The fundamental frequency is 100 Hz
(d) it is a sysmmetric pulse
22. Two vehicles, each moving with speed u on the same
16. In a wave motion y = a sin (kx - wt), y can represent
horizontal straight road, are approaching each other. Wind
(1999 - 3 Marks)
blows along the road with velocity w. One of these vehicles
(a) electric field (b) magnetic field blows a whistle of frequency f1. An observer in the other
(c) displacement (d) pressure vehicle hears the frequency of the whistle to be f2. The
17. Standing waves can be produced (1999 - 3 Marks) speed of sound in still air is V. The correct statement(s) is
(a) on a string clamped at both the ends. (are) (JEE Adv. 2013)
(b) on a string clamped at one end free at the other (a) If the wind blows from the observer to the source,
(c) when incident wave gets reflected from a wall f2 > f1
(d) when two identical waves with a phase difference of p (b) If the wind blows from the source to the observer,
are moving in the same direction f2 > f1
18. As a wave propagates, (1999 - 3 Marks) (c) If the wind blows from observer to the source,
(a) the wave intensity remains constant for a plane wave f2 < f1
(b) the wave intensity decreases as the inverse of the (d) If the wind blows from the source to the observer,
distance from the source for a spherical wave f2 < f1
(c) the wave intensity decreases as the inverse square of
the distance from the source for a spherical wave SUBJECTIVE PROBLEMS :
(d) total intensity of the spherical wave over the spherical 1. AB is a cylinder of length 1m fitted with a thin flexible
surface centred at the source remains constant at all diaphragm C at the middle and other thin flexible diaphragms
times. A and B at the ends. The portions AC and BC contain
19. A student performed the experiment to measure the speed hydrogen and oxygen gases respectively. The diaphragms
of sound in air using resonance air-column method. Two A and B are set into vibrations of same frequency. What is
resonances in the air-column were obtained by lowering the the minimum frequency of these vibrations for which
water level. The resonance with the shorter air-column is diaphragm C is a node? (Under the conditions of experiment
the first resonance and that with the longer air-column is vH 2 = 1100 m/s, vO2 = 300 m/s). (1978)
the second resonance. Then, (2009)
(a) the intensity of the sound heard at the first resonance A C B
was more than that at the second resonance
(b) the prongs of the tuning fork were kept in a horizontal H2 O2
plane above the resonance tube
P-246 Topic-wise Solved Papers - PHYSICS
2. A copper wire is held at the two ends by rigid supports. At
Given for steel Y = 2 ´ 1011 N/m2
30°C, the wire is just taut, with negligible tension. Find the
speed of transverse waves in this wire at 10°C.
a = 1.21 ´ 10-5 per o C (1984 - 6 Marks)
Given : Young modulus of copper = 1.3 ´ 1011 N/m2 .
10. The vibrations of a string of length 60 cm fixed at both ends
Coefficient of linear expansion of copper = 1.7 ´ 10-5 oC -1. are represented by the equation—
Density of copper = 9 ´ 103 kg/m3 . (1979) æ px ö
y = 4 sin ç ÷ cos (96 pt) (1985 - 6 Marks)
3. A tube of a certain diameter and of length 48 cm is open at è 15 ø
both ends. Its fundamental frequency of resonance is found
to be 320 Hz. The velocity of sound in air is 320 m/sec. Where x and y are in cm and t in seconds.
Estimate the diameter of the tube. (1980) (i) What is the maximum displacement of a point at x = 5
One end of the tube is now closed. Calculate the lowest cm?
frequency of resonance for the tube. (ii) Where are the nodes located along the string?
4. A metal wire of diameter 1mm is held on two knife edges (iii) What is the velocity of the particle at x = 7.5 cm at
separated by adistance of 50 cm. The tension in the wire is t = 0.25 sec.?
100 N. The wire, vibrating with its fundamental frequency, (iv) Write down the equations of the component waves
and a vibrating tuning fork together produce 5 beats/sec. whose superposition gives the above wave
The tension in the wire is then reduced to 81 N. When the 11. Two tuning forks with natural frequencies of 340 Hz each
two are excited, beats are created again at the same rate. move relative to a stationary observer. One fork moves away
Calculate from the observer, while the other moves towards him at the
(i) The frequency of the fork. same speed. The observer hears beats of frequency 3 Hz.
Find the speed of the tuning fork. (1986 - 8 Marks)
(ii) The density of the material of the wire. (1980)
12. The following equations represent transverse waves :
5. A source of sound of frequency 256 Hz is moving rapidly
towards wall with a velocity of 5 m/sec. How many beats (1987 - 7 Marks)
per second will be heard if sound travels at a speed of z1 = A cos (kx - wt );
330 m/sec? (1981 - 4 Marks)
6. A string 25 cm long and having a mass of 2.5 gm is under z2 = A cos (kx + wt ); z3 = A cos (ky - wt )
tension. A pipe closed at one end is 40 cm long. When the Identify the combination (s) of the waves which will produce
string is set vibrating in its first overtone and the air in the (i) standing wave (s), (ii) a wave travelling in the directon
pipe in its fundamental frequency, 8 beats per second are making an angle of 45° degrees with the positive x and
heard. It is observed that decreasing the tension in the string positive y axes. In each case, find the positions at which the
decreases beat frequency. If the speed of sound in air is resultant intensity is always zero.
320 m/s, find the tension in the string. (1982 - 7 Marks)
13. A train approaching a hill at a speed of 40 km/hr sounds a
7. A sonometer wire under tension of 64 Newtons vibrating in whistle of frequency 580 Hz when it is at a distance of 1 km
its fundamental mode is in resonance with a vibrating tuning from a hill. A wind with a speed of 40 km/hr is blowing in the
fork. The vibrating portion of the sonometer wire has a length direction of motion of the train Find (1988 - 5 Marks)
of 10 cm and a mass of 1 gm. The vibrating tuning fork is
(i) the Frequency of the whistle as heard by an observer
now moved away from the vibrating wire with a constant
on the hill,
speed and an observer standing near the sonometer hears
one beat per second. Calculate the speed with which the (ii) the distance from the hill at which the echo from the hill
tuning fork is moved if the speed of sound in air is 300 m/s. is heard by the driver and its frequency.
(1983 - 6 Marks) (Velocity of sound in air =1,200 km/hr)
8. A uniform rope of length 12 m and mass 6 kg hangs vertically 14. A source of sound is moving along a circular orbit of radius
from a rigid support. A block of mass 2 kg is attached to the 3 metres with an angular velocity of 10 rad/s. A sound
free end of the rope. A transverse pulse of wavelength 0.06 detector located far away from the source is executing linear
m is produced at the lower end of the rope. What is the simple harmonic motion along the line BD with an amplitude
wavelength of the pulse when it reaches the top of the rope? BC = CD = 6 metres. The frequency of oscillation of the
(1984 - 6 Marks) 5
detector is per second. The source is at the point A when
9. A steel wire of length 1 m, mass 0.1 kg and uniform p
cross-sectional area 10–6 m2 is rigidly fixed at both ends. the detector is at the point B. If the source emits a continous
The temperature of the wire is lowered by 20° C. If transverse sound wave of frequency 340 Hz, find the maximum and the
waves are set up by plucking the string in the middle, minimum frequencies recorded by the detector.
calculate the frequency of the fundamental mode of (1990 - 7 Mark)
vibration.
WAVES P-247
20. A band playing music at a frequency f is moving towards a
6m 6m wall at a speed nb. A motorist is following the band with a
3m · · · speed vm. If v is the speed of sound, obtain an expression
A B C D
for the beat frequency heard by the motorist.
A1 (1997 - 5 Marks)
A2 · · · 21. The air column in a pipe closed at one end is made to vibrate in
A B C D its second overtone by a tuning fork of frequency 440 Hz. The
A3 speed of sound in air is 330 m s–1. End corrections may be
15. The displacement of the medium in a sound wave is given neglected. Let P0 denote the mean pressure at any point in the
pipe, and DP0 the maximum amplitude of pressure variation.
by the equation y1 = A cos (ax + bt) where A, a and b are
(a) Find the length L of the air column. (1998 - 8 Marks)
positive constants. The wave is reflected by an obstacle
situated at x = 0. The intensity of the reflected wave is 0.64 (b) What is the amplitude of pressure variation at the middle
times that of the incident wave. (1991 - 4 × 2 Marks) of the column?
(a) What are the wavelength and frequency of incident (c) What are the maximum and minimum pressures at the
wave? open end of the pipe?
(b) Write the equation for the reflected wave. (d) What are the maximum and minimum pressures at the
(c) In the resultant wave formed after reflection, find the closed end of the pipe?
maximum and minimum values of the particle speeds in 22. A long wire PQR is made by joining two wires PQ and QR of
the medium. equal radii PQ has length 4.8 m and mass 0.06 kg. QR has
(d) Express the resultant wave as a superposition of a length 2.56 m and mass 0.2 kg. The wire PQR is under a
standing wave and a travelling wave. What are the tension of 80 N. A sinusoidal wave-pulse of amplitude 3.5
positions of the antinodes of the standing wave ? cm is sent along the wire PQ from the end P. No power is
What is the direction of propagation of travelling wave? dissipated during the propagation of the wave-pulse.
16. Two radio stations broadcast their programmes at the same Calculate. (1999 - 10 Marks)
amplitude A and at slightly different frequencies w1 and (a) the time taken by the wave-pulse to reach the other
end R of the wire, and
w2 respectively, where w1 - w 2 = 103 Hz A detector (b) the amplitude of the reflected and transmitted
receives the signals from the two stations simultaneously. wave-pulses after the incident wave-pulse crosses the
joint Q.
It can only detect signals of intensity ³ 2A2 .
23. A 3.6 m long vertical pipe resonates with a source of
(1993 - 4 Marks)
frequency 212.5 Hz when water level is at certain height in
(i) Find the time interval between successive maxima of
the pipe. Find the height of water level (from the bottom of
the intensity of the signal received by the detector. the pipe) at which resonance occurs. Neglect end correction.
(ii) Find the time for which the detector remains idle in Now, the pipe is filled to a height H ( » 3.6 m). A small hole is
each cycle of the intensity of the signal. drilled very close to its bottom and water is allowed to leak.
17. A metallic rod of length 1m is rigidly clamped at its mid Obtain an expression for the rate of fall of water level in the
point. longitudinal stationary waves are set up in the rod in pipe as a function of H. If the radii of the pipe and the hole
such a way that there are two nodes on either side of the are 2 × 10-2 m and 1 × 10-3 m respectively, calculate the time
mid-point. The amplitude of an antinode is 2 × 10–6 m. interval between the occurance of first two resonances.
Write the equation of motion at a point 2 cm from the mid- Speed of sound in air is 340 m/s and g = 10 m/s2.
point and those of the constituent waves in the rod. (2000 - 10 Marks)
(Young’s Modulus of the material of the 24. A boat is traveling in a river with a speed 10 m/s along the
rod = 2 ´1011 Nm -2 ; density = 8000 kgm -3 ) stream flowing with a speed 2 m/s. From this boat, a sound
transmitter is lowered into the river through a rigid support.
(1994 - 6 Marks) The wavelength of the sound emitted from the transmitter
18. A whistle emitting a sound of frequency 440 Hz is tied to a inside the water is 14.45 mm. Assume that attenuation of
string of 1.5m length and rotated with an angular velocity of sound in water and air is negligible.
20 rad s–1 in the horizontal plane. Calculate the range of (a) What will be the frequency detected by a receiver kept
frequencies heard by an observer stationed at a large inside the river downstream?
distance from the whistle. (1996 - 3 Marks) (b) The transmitter and the receiver are now pulled up into
19. The first overtone of an open organ pipe beats with the first air. The air is blowing with a speed 5 m/s in the direction
overtone of a closed organ pipe with a beat frequency of 2.2 opposite the river stream. Determine the frequency of
the sound detected by the receiver.
Hz. The fundamental frequency of the closed organ pipe is
(Temperature of the air and water = 20oC; Density of river
110 Hz. Find the lengths of the pipes. (1997C - 5 Marks)
water = 103 kg/m3;
P-248 Topic-wise Solved Papers - PHYSICS
Bulk modulus of the water = 2.088 ´ 109 Pa; Gas constant 26. A tuning fork of frequency 480 Hz resonates with a tube
R = 8.31 J/mol-K; closed at one end of length 16 cm and diameter 5 cm in
Mean molecular mass of air = 28.8 ´ 10-3 kg/mol; CP/CV for fundamental mode. Calculate velocity of sound in air.
air = 1.4) (2001 - 10 Marks) (2003 - 2 Marks)
25. Two narrow cylindrical pipes A and B have the same length. 27. A string tied between x = 0 and x = l vibrates in fundamental
Pipe A is open at both ends and is filled with a monoatomic mode. The amplitude A, tension T and mass per unit length
gas of molar mass MA. Pipe B is open at one end and closed m is given. Find the total energy of the string.
at the other end, and is filled with a diatomic gas of molar (2003 - 4 Marks)
mass MB. Both gases are at the same temperature.
(2002 - 5 Marks ) x=0 x=l
(a) If the frequency of the second harmonic of the 28. A whistling train approaches a junction. An observer
fundamental mode in pipe A is equal to the frequency standing at junction observes the frequency to be 2.2 KHz
of the third harmonic of the fundamental mode in pipe and 1.8 KHz of the approaching and the receding
B, determine the value of MA/MB. train respectively. Find the speed of the train (speed of sound
(b) Now the open end of pipe B is also closed (so that the = 300 m/s) (2005 - 2 Marks)
pipe is closed at both ends). Find the ratio of the 29. A transverse harmonic disturbance is produced in a string.
fundamental frequency in pipe A to that in pipe B. The maximum transverse velocity is 3 m/s and maximum
transverse acceleration is 90 m/s2. If the wave velocity is 20
m/s then find the waveform. (2005 - 4 Marks)

MATCH THE FOLLOWING :


MUTLIPLE CHOICE QUESTIONS WITH ONE CORRECT
Each question contains statements given in two columns, which have to be matched. The statements in Column-I are labelled A,
B, C and D, while the statements in Column-II are labelled p, q, r, s and t. Any given statement in Column-I can have correct
matching with ONE OR MORE statement(s) in Column-II. The appropriate bubbles corresponding to the answers to these
questions have to be darkened as illustrated in the following example :
If the correct matches are A-p, s and t; B-q and r; C-p and q; and D-s then the correct darkening of bubbles will look like the given.
p q r s t
A p q r s t
B p q r s t
C p q r s t
D p q r s t

1. Each of the properties of sound listed in the column A primarily depends on one of the quantities in column B. Write down the
matching pairs from the two columns. (1980)
Column A Column B
A. pitch p. Waveform
B. quality q. frequency
C. loudness r. intensity
2. Column I shows four systems, each of the same length L, for producing standing waves. The lowest possible natural frequency
of a system is called its fundamental frequency, whose wavelength is denoted as lf. Match each system with statements given
in Column II describing the nature and wavelength of the standing waves. (2011)
Column I Column II
(A) Pipe closed at one end (p) Longitudinal waves

(B) Pipe open at both ends (q) Transverse waves


WAVES P-249
(C) Stretched wire clamped at both ends (r) lf = L

(D) Stretched wire clamped at both ends (s) lf = 2L


and at mid-point

(t) lf = 4L

COMPREHENSION BASED Q UESTIONS :

Intensity
PASSAGE - 1 (b)
Waves y1 = A cos(0.5px - 100pt ) and y2 = A cos(0.46px - 92pt )
f1 f 2 Frequency
are travelling along x-axis. (Here x is in m and t is in second)

1. Find the number of times intensity is maximum in time interval

Intensity
of 1 sec. (2006 – 5M, –2)
(a) 4 (b) 6 (c)
(c) 8 (d) 10
2. The wave velocity of louder sound is (2006 – 5M, –2) f1 f 2 Frequency
(a) 100 m/s (b) 192 m/s
(c) 200 m/s (d) 96 m/s
Intensity

3. The number of times y1 + y2 = 0 at x = 0 in 1 sec is


(a) 100 (b) 46 (2006 – 5M, –2) (d)
(c) 192 (d) 96
PASSAGE - 2 f1 f 2 Frequency
Two trains A and B moving with speeds 20 m/s and 30 m/s
respectively in the same direction on the same straight track, with 6. The spread of frequency as observed by the passengers in
B ahead of A. The engines are at the front ends. The engine of train B is (2007)
train A blows a long whistle.
(a) 310 Hz (b) 330 Hz
Assume that the sound of the whistle is composed of components
varying in frequency from f1 = 800 Hz to f2 = 1120 Hz, as shown in (c) 350 Hz (d) 290 Hz
the figure. The spread in the frequency (highest frequency –
INTEGER VALUE CORRECT TYPE :
lowest frequency) is thus 320 Hz. The speed of sound in still air is
340 m/s. 1. A 20 cm long string, having a mass of 1.0 g, is fixed at both
4. The speed of sound of the whistle is (2007) the ends. The tension in the string is 0.5 N. The string is set
(a) 340 m/s for passengers in A and 310 m/s for passengers into vibrations using an external vibrator of frequency 100
in B Hz. Find the separation (in cm) between the successive
(b) 360 m/s for passengers in A and 310 m/s for passengers nodes on the string. (2009)
in B 2. A stationary source is emitting sound at a fixed frequency
(c) 310 m/s for passengers in A and 360 m/s for passengers
f0, which is reflected by two cars approaching the source.
in B
The difference between the frequencies of sound reflected
(d) 340 m/s for passengers in both the trains
from the cars is 1.2% of f0. What is the difference in the
5. The distribution of the sound intensity of the whistle as
speeds of the cars (in km per hour) to the nearest integer ?
observed by the passengers in train A is best represented
by (2007) The cars are moving at constant speeds much smaller than
the speed of sound which is 330 ms–1. (2010)
3. When two progressive waves y 1 = 4 sin (2x – 6t) and
Intensity

y2 = 3sin æç 2 x - 6t - p ö÷ are superimposed, the amplitude of


(a)
è 2ø
f1 f 2 Frequency
the resultant wave is (2010)
P-250 Topic-wise Solved Papers - PHYSICS

1. Length of a string tied to two rigid supports is 40 cm. 8. A tuning fork of known frequency 256 Hz makes 5 beats per
Maximum length (wavelength in cm) of a stationary wave second with the vibrating string of a piano. The beat
produced on it is [2002] frequency decreases to 2 beats per second when the tension
(a) 20 (b) 80 in the piano string is slightly increased. The frequency of
the piano string before increasing the tension was
(c) 40 (d) 120.
[2003]
2. Tube A has both ends open while tube B has one end closed,
(a) 256 + 2 Hz (b) 256 – 2 Hz
otherwise they are identical. The ratio of fundamental
frequency of tube A and B is [2002] (c) 256 – 5 Hz (d) 256 + 5 Hz
(a) 1 : 2 (b) 1 : 4 9. The displacement y of a particle in a medium can be
expressed as,
(c) 2 : 1 (d) 4 : 1.
3. A tuning fork arrangement (pair) produces 4 beats/sec with æ pö
y = 10-6 sin ç100t + 20 x + ÷ m where t is in second and x
one fork of frequency 288 cps. A little wax is placed on the è 4ø
unknown fork and it then produces 2 beats/sec. The in meter. The speed of the wave is [2004]
frequency of the unknown fork is [2002] (a) 20 m/s (b) 5 m/s
(a) 286 cps (b) 292 cps (c) 2000 m/s (d) 5p m/s
(c) 294 cps (d) 288 cps. 10. When two tuning forks (fork 1 and fork 2) are sounded
4. A wave y = a sin(wt–kx) on a string meets with another simultaneously, 4 beats per second are heard. Now, some
wave producing a node at x = 0. Then the equation of the tape is attached on the prong of the fork 2. When the
unknown wave is [2002] tuning forks are sounded again, 6 beats per second are
(a) y = a sin( w t + kx) (b) y = –a sin( w t + kx) heard. If the frequency of fork 1 is 200 Hz, then what was
the original frequency of fork 2? [2005]
(c) y = a sin( w t – kx) (d) y = –a sin( w t – kx)
(a) 202 Hz (b) 200 Hz
5. When temperature increases, the frequency of a tuning fork (c) 204 Hz (d) 196 Hz
(a) increases [2002] 11. An observer moves towards a stationary source of
(b) decreases sound, with a velocity one-fifth of the velocity of sound.
(c) remains same What is the percentage increase in the apparent
frequency ? [2005]
(d) increases or decreases depending on the material
(a) 0.5% (b) zero
6. The displacement y of a wave travelling in the x -direction is
given by (c) 20 % (d) 5 %
12. A whistle producing sound waves of frequencies 9500 HZ
æ pö and above is approaching a stationary person with speed v
y = 10 - 4 sin ç 600 t - 2 x + ÷ metres
è 3ø ms–1. The velocity of sound in air is 300 ms–1. If the person
can hear frequencies upto a maximum of 10,000 HZ, the
where x is expressed in metres and t in seconds. The speed maximum value of v upto which he can hear whistle is
of the wave - motion, in ms-1 , is [2003]
15
(a) 300 (b) 600 (a) 15 2 ms -1 (b) ms -1 [2006]
2
(c) 1200 (d) 200
7. A metal wire of linear mass density of 9.8 g/m is stretched (c) 15 ms -1 (d) 30 ms-1
with a tension of 10 kg-wt between two rigid supports 1
metre apart. The wire passes at its middle point between the 13. A string is stretched between fixed points separated by 75.0
poles of a permanent magnet, and it vibrates in resonance cm. It is observed to have resonant frequencies of 420 Hz
when carrying an alternating current of frequency n. The and 315 Hz. There are no other resonant frequencies between
frequency n of the alternating source is [2003] these two. Then, the lowest resonant frequency for this
string is [2006]
(a) 50 Hz (b) 100 Hz
(a) 105 Hz (b) 1.05 Hz
(c) 200Hz (d) 25Hz
(c) 1050 Hz (d) 10.5 Hz
WAVES P-251
14. A sound absorber attenuates the sound level by 20 dB. The 20. The transverse displacement y (x, t) of a wave on a string is
intensity decreases by a factor of [2007]
given by y ( x, t ) = e
(
- ax 2 + bt 2 + 2 ab ) xt ) . This represents a:
(a) 100 (b) 1000
(c) 10000 (d) 10 [2011]
15. While measuring the speed of sound by performing a b
resonance column experiment, a student gets the first (a) wave moving in – x direction with speed
a
resonance condition at a column length of 18 cm during
winter. Repeating the same experiment during summer, she (b) standing wave of frequency b
measures the column length to be x cm for the second
1
resonance. Then [2008] (c) standing wave of frequency
b
(a) 18 > x (b) x > 54
(c) 54 > x > 36 (d) 36 > x > 18 a
(d) wave moving in + x direction speed
16. A wave travelling along the x-axis is described by the b
equation y(x, t) = 0.005 cos (a x – bt). If the wavelength and 21. A travelling wave represented by y = A sin (wt – kx) is
the time period of the wave are 0.08 m and 2.0s, respectively, superimposed on another wave represented by
then a and b in appropriate units are [2008]
y = A sin (wt + kx). The resultant is [2011RS]
(a) a = 25.00 p , b = p (a) A wave travelling along + x direction
0.08 2.0 (b) A wave travelling along – x direction
(b) a= ,b =
p p (c) A standing wave having nodes at
nl
0.04 1.0 x= , n = 0,1, 2....
(c) a= ,b = 2
p p
(d) A standing wave having nodes at
p
(d) a = 12.50p, b =
2.0
æ 1ö l
17. Three sound waves of equal amplitudes have frequencies x = ç n + ÷ ; n = 0,1, 2....
è 2ø 2
(n –1), n, (n + 1). They superpose to give beats. The number
of beats produced per second will be : [2009] 22. Statement - 1 : Two longitudinal waves given by
equations : y1 ( x, t ) = 2a sin(wt - kx ) and y2 ( x, t ) = a
(a) 3 (b) 2 sin (2wt - 2kx) will have equal intensity..
(c) 1 (d) 4 Statement - 2 : Intensity of waves of given frequency in
same medium is proportional to square of amplitude only.
18. A motor cycle starts from rest and accelerates along a straight
(a) Statement-1 is true, statement-2 is false. [2011RS]
path at 2m/s2. At the starting point of the motor cycle there
(b) Statement-1 is true, statement-2 is true, statement-2
is a stationary electric siren. How far has the motor cycle
is the correct explanation of statement-1
gone when the driver hears the frequency of the siren at
(c) Statement-1 is true, statement-2 is true, statement-2
94% of its value when the motor cycle was at rest? (Speed
is not the correct explanation of statement-1
of sound = 330 ms–1) [2009]
(d) Statement-1 is false, statement-2 is true.
(a) 98 m (b) 147 m 23. A cylindrical tube, open at both ends, has a fundamental
(c) 196 m (d) 49 m frequency, f, in air. The tube is dipped vertically in water so
that half of it is in water. The fundamental frequency of the
19. The equation of a wave on a string of linear mass density air-column is now : [2012]
0.04 kg m–1 is given by (a) f (b) f/2
(c) 3f/4 (d) 2f
é æ t x öù 24. A sonometer wire of length 1.5 m is made of steel. The tension
y =0.02(m) sin ê2p ç - ÷ú . in it produces an elastic strain of 1%. What is the fundamental
ë è 0.04(s ) 0.50(m) ø û
frequency of steel if density and elasticity of steel are
The tension in the string is [2010] 7.7 × 103 kg/m3 and 2.2 × 1011 N/m2 respectively ?
[JEE-Main 2013]
(a) 4.0 N (b) 12.5 N (a) 188.5 Hz (b) 178.2 Hz
(c) 0.5 N (d) 6.25 N (c) 200.5 Hz (d) 770 Hz
P-252 Topic-wise Solved Papers - PHYSICS

Solutions & Explanations


Section-A : JEE Advanced/ IIT-JEE
A 1. A (2pv), A (2pv)2 2. 0.125 m 3. 1: 1 4. 240 Hz 5. 0.5 ms–1
6. f 7. 6 Hz 8. p, 0
B 1. F 2. T 3. F
C 1. (c) 2. (c) 3. (a) 4. (c) 5. (a) 6. (a)
7. (d) 8. (a) 9. (d) 10. (d) 11. (b) 12. (b)
13. (c) 14. (b) 15. (a) 16. (b) 17. (b) 18. (b)
19. (c) 20. (d) 21. (a) 22. (a) 23. (a) 24. (a)
25. (b) 26. (a) 27. (b)
D 1. (a,b,c,d) 2. (b) 3. (a, c) 4. (a) 5. (a, c) 6. (c)
7. (a,b,d) 8. (b, c) 9. (b, c) 10. (b) 11. (a, b, c) 12. (b)
13. (b, c) 14. (a, c) 15. (b, c, d) 16. (a,b,c,d) 17. (a, b, c) 18. (a,c,d)
19. (a,d) 20. (b, d) 21. (b, c) 22. (a, b)
E 1. 1650 Hz 2. 70 m/s 3. 3.33 cm; 163 Hz 4. (i) 95 Hz; (ii) 12738.85 Kg/m3
5. 8 6. 27.04 N 7. 0.75 m/s 8. 0.12 m 9. 11 Hz
æ px ö æ px ö
10. (i) 3.46 cm (ii) 0, 15, 30 (iii) zero (iv) 2sin ç 96pt + ÷ and -2sin ç 96pt - ÷
è 15 ø è 15 ø
(2 n + 1) p (2 n + 1) p
11. 1.5 m/s 12. (i) z1 and z2; where n = 0, 1, 2, ... (ii) z1 and z3;
2K K
13. (i) 599 Hz (ii) 0.935 km, 621 Hz 14. 438.7 Hz, 257.3 Hz
2p b
15. (a) , (b) y = –0.8Acos(ax – bt) (c) 1.8 Ab, 0
a 2p
é ( -1)n ù p 2p
ê n + ú p
(d) y = -1.6 A sin ax sin bt + 0.2 A cos(ax + bt ) sec . (ii) ´ 10 -3 sec .
êë 2 úû a , –X direction 16. (i)
103 2

é 2p ù 2p
y = 2 x10-6 cos(0.1p) sin (25000 pt ) , y1 = 10-6 sin ê (5000t - x)ú , y2 = 10-6 sin é ù
17.
ë 0.4 û êë 0.4 (5000t + x)úû

(v + vm ) ´ 2vb f
18. 403.3 Hz to 484 Hz 19. 0.9933 m, 1.006 m 20.
v 2 - vb2

15 DP0
21. (a) m (b) (c) equal to mean pressure (d) P0 + DP0 , P0 - DP0 22. (a) 0.14s (b) 2.0 cm, 1.5 cm
16 2
-dH 400 3
23. = (1.11 ´ 10 -2 ) H , 43 sec. 24. (a) 1.007 × 105 Hz, (b) 1.03 × 105 Hz 25. (a) (b)
dt 189 4

p 2Ta 2 é 3 ù
26. 336 m/s 27. 28. 30 m/s 29. y = 0.1 sin ê30t ± x ± f ú
4l ë 2 û
F 1. A ® q; B ® p; C ® r 2. A ® p, t; B ® p, s; C ® q, s; D ® q, r
G 1. (a) 2. (c) 3. (d) 4. (b) 5. (a) 6. (a)
I. 1. 5 2. 7 3. 5

Section-B : JEE Main/ AIEEE


1. (b) 2. (c) 3. (b) 4. (b) 5. (b) 6. (a)
7. (a) 8. (c) 9. (b) 10. (d) 11. (c) 12. (c)
13. (a) 14. (a) 15. (b) 16. (a) 17. (b) 18. (a)
19. (d) 20. (a) 21. (d) 22. (a) 23. (a) 24. (b)
WAVES P-253

Also at t = 2 and x = 1, again y = 1


FILL IN THE BLANKS : The wave pulse has travelled a distance of 1m in 2 sec.
1. Since y = A sin (wt – k x) 1
Displacement amplitude = A (Max displacement) \ v= = 0.5ms -1
2
dy 6. In figure (i)
Particle velocity, v = = A w cos (wt – k x)
dt c c
l
\ Velocity amplitude = Aw =2pnA = l Þ l = 2 l and f = =
2 l 2l
Particle acceleration
In figure (ii)
dv
Acc = = – A w2 sin (wt – kx) l' l c c
dt Þ = Þ l ' = 2l and f ' = = =f
\ Acceleration (Max acc) amplitude = Aw2 4 2 l ' 2l
= 4p2n2A
c 330
2. c = vl \ l= = = 0.5m
v 660 l/2
The rarefaction will be at a distance of
l 0.5 l
= = 0.125 m
4 4
3. y1 = 10 sin (3pt + p/4) ... (i)
y2 = 5 sin 3pt + 5 3 cos 3pt ... (ii)
é1 3 ù Fig (i) Fig (ii)
\ y2 = 5 × 2 ê sin 3pt + cos3pt ú = 10 sin (3pt +p/3)
ë2 2 û
7. The first frequency that driver of bus hears is the original
The ratio of amplitudes is 10 : 10 = 1 : 1 frequency of 200 Hz. The second frequency that driver hears
T T is the frequency of sound reflected from the wall. The two
4. KEY CONCEPT : v = nl and v = Þ nl = frequencies of sound heard by driver is
m m
(a) Original frequency (200 Hz.)
where T = tension in the string, m = mass per unit length. (b) Frequency of sound reflected from the wall (n')
When 50.7 kg mass is suspended
(The driver behaves
Fundamental mode as an observer)
vo =5ms-1 vs=5ms–1
50.7 ´ g
n1× 2 l = ... (i) [Q l = 2 l ]
m (The image of the bus
formed by the
When mass is submerged in water wall behaves as source)
New tension T2 = weight – upthrust
= 50.7 g – 0.0075 × 1000 × g = 43.2 g The frequency of sound reflected from the wall
é v + v0 ù
43.2g n' = n ê ú
\ n2 × 2 l = ... (ii) ë v - vs û
m
On dividing (ii) and (i), we get é 342 + 5 ù
Þ v' = 200 ê » 206 Hz.
ë 342 - 5 úû
n2 43.2
= \ Frequency of beats = n' – n = 6 Hz.
n1 50.7 8. v = 300 m/s and n = 25

43.2 43.2 v 300


Þ n2 = n1 = 260 = 240 Hz. \ l= = = 12 m
50.7 50.7 n 25
For a path difference of l, the phase difference is 2p.
1 For a path difference of 6m, the phase difference is
5. As y =
(1 + x )2 2p ´ 6
= p rad
At t = 0 and x = 0, we get y = 1. l
P-254 Topic-wise Solved Papers - PHYSICS
The equation of the plane progressive wave is 3. (a) KEY CONCEPT
2p é 2p ù 1 T
y = A sin (vt + x) = 2.5 ´ 10-5 sin ê (300t + x) ú f= ;
l ë 12 û 2l m
é 2p ù In air : T = mg = rVg
Now, y1 = 2.5 × 10–5 sin ê (300t + x) ú
ë 12 û 1 rVg
\ f= ... (i)
2l m
é 2p ù
and y2 = 2.5 × 10–5 sin ê 12 (300t + x + 6)ú In water : T = mg – upthrust
ë û
Þ y1 – y2 = 2.5 × 10–5 V Vg
= Vrg – rw g = (2r - rw )
2 2
é ì 2p ü ì 2p üù
êsin í 12 (300t + x )ý - sin í 12 (300t + x + 6)ý ú
ë î þ î þû Vg
(2r - rw )
1 2 1 Vgr (2r - rw )
é p p ù \ f '= =
2l m 2l 2r
= 2.5 × 10–5 êë 2 cos 12 (600t + 2 x + 6)sin 12 (-6) úû = 0 m
1/ 2
f' 2r - rw æ 2r - rw ö
TRUE / FALSE : = f '= f ç
f 2r è 2r ø÷
1. The intensity of sound at a given point is the energy per
second received by a unit area perpendicular to the direction 1/ 2
é 2r - 1 ù
of propagation. = 300 ê ú Hz
1 ë 2r û
I = r V w 2 A2
2 4. (c) Comparing it with
Also intensity varies as distance from the point source as y (x, t) = A cos (wt + p/2) cos k x
1 If k x = p/2, a node occurs;
Iµ 2 \ 10p x = p/2 Þ x = 0.05 m
r
NOTE : None of the parameters are changing in case of a If k x = p, an antinode occurs
clear night or a clear day. Þ 10px = p Þ x = 0.1 m
Therefore the intensity will remain the same. Also speed of wave
2. Speed of sound waves in water is greater than in air. w 50p
3. NOTE : If the sound reaches the observer after being = = = 5m/s and
k 10p
reflected from a stationary surface and the medium is also
l = 2p/k = 2p/10p = 0.2 m
stationary, the image of the source will become the source
5. (a) KEY CONCEPT : According to Hooke's law FR µ x
of reflected sound.
[Restoring Force FR = T, tension of spring]
Thus in both the cases, one sound coming directly from the
source and the other coming after reflection will have the T
same apparent frequency (Since velocity of source w.r.t. Velocity of sound by a stretched string v =
m
observer is same in both the cases). Therefore no beats will
where m is the mass per unit length
be heard.
\ vµ T
MCQ's WITH ONE CORRECT ANSWER :
v T
1. (c) Case (i) Hence v µ T or, =
v' T'
l
Here =l \ l = 2l
2 T' 1.5x
or v' = v =v = 1.22 v
v v T x
\ f= = ... (i) 6. (a) For both end open
l 2l
Case (ii) l

l' l v v
Here = Þ l' = 2l \ f ' = = = f
4 2 l ' 2l
2. (c) NOTE : Stationary wave is produced when two waves 2l1
travel in opposite direction. = l Þ l1 = 2l
4
Now, y = a cos (k x – wt) – a cos (k x + wt)
\ y = 2a sin kx sin wt is equation of stationary wave c c
n1 = = ... (i)
which gives a node at x = 0. l1 2l
WAVES P-255
For one end closed 13. (c) E µ A2n2 where A = amplitude and n = frequency.
For third harmonic Also w = 2pn Þ w µ n
In case 1 : Amplitude = A and n1 = n
3l 2 4l In case 2 : Amplitude = A and n2 = 2n
= l Þ l2 =
4 3
E2 A2 n22
c 3c \ = =4 Þ E2 = 4E1
n2 = = ... (ii) E1 A2 n12
l 2 4l
Given n2 – n1 = 100 æ v + vö
14. (b) Using the formula n ' = n ç A
From (i) and (ii) è v ÷ø
n2 3/ 4 3 v A + v 5.5 V +V 6 v
= = = and B = Þ B =2
n1 1/ 2 2 v 5 V 5 vA
On solving, we get 5 9g 3 Mg
n1 = 200 Hz. 15. (a) f0 = =
7. (d) vs = 0 33 m/s 2l µ 2l µ
Þ M = 25 kg
· ¾¾
® · NOTE : Using the formula of a vibrating string,
0
Vs (Observer)
(Source) p T
f = where p = number of loops.
2l m
é v ù é 330 ù In each case, the wire vibrates, in resonance with the
n' = n ê ú = 450 ê = 500 Hz
ë v - v sû ë 330 - 33 úû same tuning fork. Frequency of wire remains same while
p and T change.
dy
8. (a) v = = - Aw cos (kx - wt ) p1 T1 p T2
dt \ = 2 or p1 T1 = p2 T2
2l m 2l m
\ vmax = Aw
340 10 T2 p
9. (d) n1 = n0 = n0 ; or = 1
340 - 34 9 T1 p2

M ´g 5
340 20 n1 10 19 19
= ´ = = or M = 5 ´ 5 ´ 9 or M = 25 kg.
n2 = n0 = n0 ; n2 9 20 18 9´g 3 3´3
340 - 17 19

1 æ T ö 1 æ T ö 16. (b) f1= frequency of the police car heard by motorcyclist,


10. (d) n1 = ç ÷ and n2 = 4l ç 2 ÷ f2 = frequency of the siren heard by motorcyclist.
2l è 4pr rø
2 è pr rø 330 - v 330 + v
f1 = ´ 176; f2 = ´ 165;
330 - 22 330
v 1 T l Q f1 – f2 = 0 Þ v = 22 m/s
n= = [where = length of string]
l l m 2 l
17. (b) l1 + x = or, l = 4 (l1 + x )
n1 1 é mass r ´ A ´ length ù 4
\ = 2´ =1 êQ m = length = length
= rA ú
3l 4
n2 2 ë û (l 2 + x ) = or l = (l 2 + x)
4 3
æ gRT ö 1 v v v 3v
11. (b) KEY CONCEPT : Crms = ç Here Crms µ ; \ n1 = = \ n2 = =
è M ÷ø m l1 4(l1 + x ) l 2 4 (l 2 + x )
Given n1 = n2
Crms1 æm ö
\ = ç 2÷
Crms2 è m1 ø
0.1m=l1 l1 +x 0.35m=l 2 l2 +x
12. (b)

After two seconds pulses will overlap each other.


NOTE : According to superposition principle the string
will not have any distortion and will be straight.
Hence there will be no P.E. The total energy will be
only kinetic.
P-256 Topic-wise Solved Papers - PHYSICS
Here n is a odd number. From (i) and (ii)
v 3v
or, = or, x = 0.025 m
4(l1 + x ) 4 (l 2 + x) n
f1
f2 =
18. (b) Frequency of first overtone in closed pipe, 4
5
3v P For first resonance, n = 5, f2 = f1
n= ... (i) 4
4 l1 r1
Frequency of first overtone in open pipe, 1 TAB
21. (a) Frequency of Ist harmonic of AB =
2l m
1 P
n' = ... (ii)
l2 r2 1 TCD
Frequency of 2nd harmonic of CD =
From equation (i) and (ii) l m
Given that the two frequencies are equal.
4 r
Þ l 2 = l1 1 TAB
3 r2 1 TAB 1 TCD
\ = Þ = TCD
2l m l m 4
Þ TAB = 4TCD ... (i)
19. (c)
l1 +e
l2 +e A B

l AB l CD = l
=l
2
TAB TCD

O
B x L–x D
For first resonance For second resonance
l 3l m
l1 + e = l2 + e = L
4 4
But v = nl
For rotational equilibrium of massless rod, taking torque
4 3v
\ v = n (l 2 + e ) Þ l2 + e = ...(i) about point O.
3 4v TAB × x = TCD (L – x) ... (ii)
v For translational equilibrium,
\ v = n 4( l1 + e) Þ l1 + e = ... (ii) TAB + TCD = mg ... (iii)
4n
On solving, (i) and (iii), we get
Subtracting (i) and (ii),
mg 4mg
v = 2n (l 2 - l1 ) \ Dv = 2n ( Dl 2 + Dl1 ) TCD = \ TAB =
5 5
= 2 × 512 × (0.1 + 0.1) cm/s = 204.8 cm/s
20. (d) Substituting these values in (ii), we get
4mg mg L
´x= ( L - x) Þ x =
nth harmonic 5 5 5
22. (a) As shown in the figure, the fringes of the tuning force
l l are kept in a vertical plane.
l=l
v v
\ f1 = = ... (i)
l l
4l
l=
n
v nv
\ f2 = = ... (ii)
l 4l
WAVES P-257
23. (a) Since the wave is sinusoidal moving in positive x-axis
é 1 Tù v
the point will move parallel to y-axis therefore options \ 2´ê ú=
(c) and (d) are ruled out. As the wave moves forward in 2
ë 1 l m û 4l2
positive X-direction, the point should move upwards
i.e. in the positive Y-direction. Therefore correct option 1 50 320
\ = \ m = 0.02 kg m–1
is a. 0.5 m 4 ´ 0.8
ALTERNATE SOLUTION - 1 : The mass of the string = m l1
= 0.02 × 0.5 kg = 10g
dy dy
Velocity of point P = = – (velocity of wave ) × 26. (a) The frequency f of the siren = 8 kHz.
dt dx The frequency of sound incident on the tall building
dy é v ù
Here is negative . Therefore velocity at point P is f1 = f ê ú ...(i)
dx ë v - vs û
positive and is along y-axis only. Where v = velocity of sound in air and
ALTERNATE SOLUTION - 2 : vs = velocity of police car (source)
Equation of a wave moving in positive x-axis is given The frequency of sound reflected by the tall building
as y = A sin (wt– f) or vP =Aw cos (wt–f) will be same as the incident frequency (f1).
Here y = 5 cm, A = 10 cm, The frequency of the reflected sound as perceived by
\5 = 10 sin (wt – f) Þ wt– f = 30º the driver of the police car moving towards the tall
Substituting this value in the equation of velocity we building is
get vP = 0.10 × w cos 30º é v + vo ù
f2 = f1 ê ú
n 0.10 ë v û
Now v = n l \ v= = = 0.2
l 0.5 where vo = velocity of police car (observer)
\ w = 2pv = 2p × 0.2 = 0.4 π é v ù é v + vo ù
f2 = f ê úê ú from (i)
3 3 ë v - vs û ë v û
Þ vP= 0.1× 0.4p × = π.
2 50 é v + vo ù
It has to be in positive y direction. = f êv-v ú
24. (a) The frequency (v) produced by the air column is ë s û

given by Here v = 320 m/s (given)


5
l×v=vÞ v=
v vo = vs = 36 ´ = 10 m/s
18
l
é 320 + 10 ù 33
3l f2 = 8 ê ú = 8 ´ » 8.5 kHz
Also, = l = 75cm = 0.75 m ë 320 - 10 û 31
4
27. (b) Considering the end correction [e = 0.3 D], we get
4 ´ 0.75 340 ´ 3 l l
\l= Þ v= = 340 Hz L+e= ÞL= –e
3 4 ´ 0.75 4 4
\ The frequency of vibrating string = 340. Since this
336 ´ 100 é vù
string produces 4 beats/sec with a tuning fork of =
512 ´ 4
- 0.3 ´ 4 êëQ l = u úû
frequency n therefore n = 340 + 4 or n = 340 – 4. With
increase in tension, the frequency produced by string = 15.2 cm
increases. As the beats/sec decreases therefore n =
340 + 4 = 344 Hz.
25. (b) Frequency of 2nd harmonic of string = Fundamental 1. (a,b,c,d) y = 10–4 sin (60t + 2x)
frequency produced in the pipe Comparing the given equation with the standard wave
equation travelling in negative x-direction
y = a sin (wt + k x)
we get amplitude a = 10–4m
30
0.5 m Also, w = 60 rad/s \ 2pf = 60 Þ f= Hz
p
2p
Also, k = 2 Þ =2 Þ l=pm
l
0.8 m 30
We know that v = f l = ´ p = 30 m/s
p
P-258 Topic-wise Solved Papers - PHYSICS

é xù v
2. (b) y = y0 sin 2p ê f t - ú 6. (c) n1 = for first harmonic
ë lû 4l1
dy é æ x öù l1
\ = ê y0 cos 2 p ç f t - ÷ ú ´ 2pf
dt ë è l øû
3v
n2 = for third harmonic
é dy ù 2l 2
or, ê ú = y0 ´ 2pf
ë dt û max l 2

Given that the maximum particle velocity is equal to four


times the wave velocity (c = f × l) v 3v l1 1
Q n1 = n2 \ = Þ =
4l1 2l 2 l2 6
p y0
\ y0 × 2pf = 4(f × l) \ l=
2 7. (a,b,d)
3. (a,c) The wavelengths possible in an air column in a pipe
which has one closed end is
4l 1m
l= So, c = nl
(2n + 1) In general, we can write for a closed end pipe that
4l (2n - 1) c
300 = 264 × v= where n = 1, 2, 3,....
2n + 1 4l
n = 264 Hz as it is in resonance with a vibrating turning fork l 4l
of frequency 264 Hz. (2n – 1) =l Þ l =
4 (2n - 1)
330 ´ (2 n + 1)
l= c 3c 5c
264 ´ 4 \ n= , , ,.... = 80, 240, 400....
For n = 1, l = 0.3125 m = 31.25 cm 4l 4l 4l
For n = 2, l = 0.9375 m = 93.75 cm 8. (b,c) y = A sin (10 px + 15 pt + p/3)
The standard equation of a wave travelling in – X direction
é 2p ù
4. (a) is y = A sin ê (vt + x) + (f)ú
ël û
l l
é 2p v 2p ù
Þ y = A sin ê t+ x + fú
l ë l l û
= l (Fundamental mode) \ l = 4l
4 Comparing it with the given equation, we find
l' 2p v 2p
= l (Fundamental mode) \ l ' = 2l
= 15p and = 10p
2 l l
c c
\ n= = = 512 Hz (given) 1 15p 1
l 4l Þ l= = 0.2 m and v = ´ = 1.5 m/s
5 2p 5
c c æ cö
and n' = = = 2 ç ÷ = 2 × 512 = 1024 Hz. 1 T
l ' 2l è 4l ø 9. (b,c) As, f = \ fµ T
2p m
5. (a,c) For wave motion, the differential equation is
æ Given that T1 > T2 \ f1 > f 2
¶2 y w2 ö ¶ 2 y
= ç constant 2 ÷ 2
¶t 2 è k ø ¶x Initially beat frequency ( f1 - f2 ) = 6.
The beat frequency remains unchanged which is possible
¶2 y 2 ¶2 y when f2 increases and f1 decreases. Thus T2 increases and
or =v ....(i)
¶t 2 ¶x 2 T1 decreases.
NOTE : The wave motion is characterized by the two ætö æ t ö
conditions 10. (b) y = 4 cos2 çè ÷ø sin (1000 t) = 2 ç 2cos2 sin1000t ÷
2 è 2 ø
f ( x, t ) = f ( x , t + T ) ....(ii) = 2 [cos t + 1] sin 1000t
f ( x, t ) = f ( x + l , t ) ....(iii) = 2 cos t sin 1000t + 2 sin 1000t
= sin 1000t + sin 999t + 2 sin 1000t
WAVES P-259
11. (a, b, c)
For such a case, we may consider that the observer is 14. (a,c)
standing along the stationary surface. Then NOTE : For a transverse sinusodial wave travelling on a
string, the maximum velocity is aw.
æ c + vö v 10
f '= f ç
è c ÷ø But maximum velocity is = = 1m/s
10 10
\ aw = 1
æ c + vö
Frequency of reflected wave is f '' = f ç Þ 10–3 × 2pn = 1
è c - v ÷ø
1 103
2v Þ n= = Hz
Þ Beat freq. = f '' – f =
c -1
. 2p ´ 10-3 2p
v 10
c c (c - v) \ l= = = 2p ´ 10-2 m
Wavelength of reflected wave = = n 103 / 2p
f '' f (c + v)
12. (b) KEY CONCEPT : The time required for constructive 0.8 0.8
15. (b,c,d) y = 2
= 2 ... (1)
interference equal to the time period of a wave pulse. (4x + 5t) + 5 é 5 ù
16 ê x + t ú + 5
ë 4 û
For a string frequency f = 1 F
2l m We know that equation of moving pulse is
y = f (x + vt) ... (2)
\ Time period, T = 2l m On comparing (1) and (2), we get
F 5 -1 2.5 -1
v= ms = ms
10-2 4 2
mass
F = 1.6 N, m = = = 2.5 ´ 10-2 So, the wave will travel a distance of 2.5 m in 2 sec.
length 0.4
0.8
-2 Q y=
2.5 ´ 10 ( 4 x + 5t )2 + 5
\ T = 2 ´ 0.4 = 0.1 sec.
1.6
0.8
13. (b,c) Due to the clamping of At x = 0, t = 0, y = = 0.16 m
the square plate at the edges, 5
its displacements along the x y \ maximum displacement is 0.16 m
and y axes will individually be y
zero at the edges. Only (0, L) R Q (L, L)
th e choices (b) and (c)
predict these displacements
correctly. This is because
sin 0 = 0. P x
Option (a) : (0, 0) (L, 0)

u ( x, y ) =0 at x = L, y = L –x O x
The graph for the given equation is drawn. This is symmetric
u ( x, y ) ¹ 0 at x =0, y =0 about y-axis.
Option (b) : 16. (a,b,c,d) In the wave motion y = a ( kx - wt ) , y can represent
u ( x, y ) = 0 at x = 0, y = 0 [Q sin 0 = 0] electric and magnetic fields in electromagnetic waves and
displacement and pressure in sound waves.
u ( x, y ) = 0 at x = L, y = L [Q sin p = 0] 17. (a,b,c) Standing waves are produced by two similar waves
Option (c) : superposing while travelling in opposite direction.
This can happen in case (a), (b) and (c).
u ( x, y ) = 0 at x = 0, y = 0 [Q sin 0 = 0] 18. (a,c,d) For a plane wave, intensity (energy crossing per unit
area per unit time) is constant at all points.
u ( x, y ) = 0 at x = L, y = L [Q sin p = 0, sin 2p = 0] But for a spherical wave, intensity at a distance r from a
Option (d) : P
point source of power (P), is given by I =
u ( x, y ) = 0 at y = 0, y = L [Q sin 0 = 0, sin p = 0] 4p r 2
u ( x, y ) ¹ 0 at x = 0, x = L [Q cos0 = 1, cos 2p = 1] 1
Þ Iµ 2
r
P-260 Topic-wise Solved Papers - PHYSICS
But the total intensity of the spherical wave over the
spherical surface centered at the source remains constant Wind blowsfromsource toobserver
Case2 :
at all times. é (V + w) + u ù
f2 = f1 ê \f2 > f1
ú
NOTE : For line source I µ
1 ë (V + w) - u û
r
(a) and (b) are correct options
19. (a, d)
At second resonance the length of air column is more as SUBJECTIVE PROBLEMS :
compared to first resonance. Now, longer the length of air
column, more is the absorption of energy and lesser is the 1. It is given that C acts as a node. This implies that at A and B
intensity of sound heard. antinodes are formed. Again it is given that the frequencies
are same.
e

p1 harmonic p2 harmonic
H2 O2
Antinode Node Antinode
A C B
0.5m 0.5m

v1 v p1 v1 3
As shown in the figure, the length of the air column at the
Þ ´ p1 = 2 ´ p2 or = =
4l 4l p2 v2 11
1 or, 11p1 = 3p2
first resonance is somewhat shorter than th of the
4 This means that the third harmonic in AC is equal to 11th
wavelength of the sound in air due to end correction. harmonic in CB.
l l Now, the fundamental frequency in AC
l+e = \ l = -e
4 4 v1 1100
20. (b, d) = = = 550 Hz
When sound pulse is reflected through a rigid boundary 4l 4 ´ 0.5
(closed end of a pipe), no phase change occurs between the and the fundamental frequency in CB
incident and reflected pulse i.e., a high pressure pulse is v2 300
reflected as a high pressure pulse. = = = 550 Hz
When a sound pulse is reflected from open end of a pipe, a 4l 4 ´ 0.5
phase change of a radian occurs between the incident and \ Frequency in AC = 3 × 550 = 1650 Hz
the reflected pulse. A high pressure pulse is reflected as a and frequency in CB = 11 × 150 = 1650 Hz
low pressure pulse. 2. (a) Using the formula of the coefficient of linear expansion
21. (b, c) of wire, Dl = laDq we get
y = [0.01 sin (62.8x)] cos (628 t).
F = YAaDq
M Speed of transverse wave is given by

l 1 F é Alr ù
Length of string = 5 ´ = 5 ´ = 0.25 m v= ê where m = mass per unit length = l = Arú
2 20 m ë û
é 2p ù
êëQ l = 62.8úû =
YAaDq
=
Y aDq
Ar r
The midpoint M is an antinode and has the maximum
displacement = 0.01 m
v ω/k 1.3 ´ 1011 ´ 1.7 ´ 10 -5 ´ 20
The fundamental frequency = = = 20Hz = = 70 m/s
2l 2l 9 ´ 103
22. (a,b) Source Observer 3. Tube open at both ends :
S O
u u v 320
f1 f2
(a) v = \ 320 =
2(l + 0.6 D) 2(0.48 + 0.6 ´ D)
Wind blowsfromobserver tosource
Case1: \ f2 > f1 0.48 + 0.6 D = 0.5 Þ 0.6 D = 0.02
é (V - w) + u ù
f2 = f1 ê ú 0.02
ë (V - w) - u û Þ D= ´ 100 cm = 3.33 cm
60
WAVES P-261
Tube closed at one end :
v 320
v= = 40cm=l
4(l + 0.3D) 4 (0.48 + 0.3 ´ 0.033) 25cm l/4
» 163 Hz 6. l

1 T 1 100 10
4. n= = = ... (i)
2l m 2 ´ 0.5 m m
The frequency of the tuning fork is either n + 5 or n – 5.
NOTE : On decreasing the tension, the frequency will
decrease. First Overtone
Therefore the frequency of tuning fork should be n – 5.
2.5 ´ 10-3
1 T' 1 81 9 Mass of string per unit length = = 0.01 kg/m
Now, n' = = = 0.25
2l m 2 ´ 0.5 m m
This n' should be n – 10. 1 T 1 T
\ Frequency, ns = = ... (i)
l m 0.25 0.01
10 9 10 - 9
\ - 10 = Þ = 10 Fundamental frequency
m m m
l
1 \ = 0.4 Þ l = 1.6 m
m= ... (ii) 4
100
c 320
d2 1 \ nT = = = 200 Hz ... (ii)
r´ p = ( m = density × volume) lT 1.6
4 100 Given that 8 beats/second are heard. The beat frequency
4 decreases with the decreasing tension. This means that beat
Þ r= = 12738.85 kg/m3 frequency decreases with decreasing ns. So beat frequency
100 ´ p´ (10-3 )2
is given by the expression.
10 n = ns – nT
From (i) and (ii) n = = 100Hz
1/100 1 T
\ 8= - 200 Þ T = 27.04 N
\ Frequency of the fork is 95 Hz. 0.25 0.01
7. Mass per unit length of sonometer wire
5.
m 0.001
= = = 0.01 kg/m
v0 = 5 m/s v0 = 5 m/s l 0.1

S I T 64
v= = = 8 ´ 10
(Observer) (Source) m (0.01)

l
Also, = 0.1 Þ l = 0.2
2
v 8 ´ 10
\ f= = = 400 Hz
n = 256 Hz l 0.2
c = 330 m/s Since tuning fork is in resonance therefore frequency of
NOTE : If the sound reaches the observer after being tuning fork is 400 Hz. The observer is hearing one beat per
reflected from a stationary surface and the medium is also second when the tuning fork is moved away with a constant
stationary, the image of the source in the reflecting surface speed v.
will become the source of the reflected sound.
é c - v0 ù
n' = n ê ú
ë c - vs û 10cm
v0, vs are + ve if they are directed from source to the observer v
and – ve if they are directed from observer to source.
é 330 - (-5) ù
n' = 256 ê = 264 Hz
ë 330 - 5 úû
\Beat frequency = 264 – 256 = 8
P-262 Topic-wise Solved Papers - PHYSICS
The frequency of tuning fork as heard by the observer æ p xö
standing stationary near sonometer wire can be found with (iii) y = 4sin ç ÷ cos (96p t )
è 15 ø
the help of Doppler effect.
é c - v0 ù
dy æ p xö
c v= = 4sin ç [– 96p sin (96 pt)]
n' = n ê ú= [Q v0 = 0 m/s] dt è 15 ÷ø
ë c + vs û c + vs
At x = 7.5 cm, t = 0.25 cm
400 ´ 300 æ p ´ 7.5 ö
\ n' = v = 4sin ç [– 96p sin (96 p × 0.25)]
300 + v s è 15 ÷ø
Since the beat frequency is 1 and as the tuning fork is going
away from the observer, its apparent frequency is (normal æ pö
= 4sin ç ÷ [- 96 p sin (24p)] = 0
frequency – 1) = 400 – 1 = 399 è 2ø

400 ´ 300 æ p xö
\ 399 = (iv) y = 4sin ç ÷ cos[96 p t ]
300 + v s è 15 ø

or, v s = 0.75 m/s é æ p xö ù


8. KEY CONCEPT : The velocity of wave on the string is
= 2 ê 2 sin ç
è ÷ø cos (96 p t ) ú
ë 15 û
given by the formula
é æ p xö æ p xöù
= 2 êsin ç 96 p t + ÷ø - sin çè 96 p t - ÷
v=
T
ë è 15 15 ø úû
m
where T is the tension and m is the mass per unit length. æ p xö æ p xö
= 2sin ç 96 p t + ÷ - 2sin çè 96 p t - ÷
Since the tension in the string will increase as we move up è 15 ø 15 ø
the string (as the string has mass), therefore the velocity of = y1 + y2
wave will also increase. (m is the same as the rope is uniform)
p xö
where y1 = 2 sin æç 96 p t + ÷
v1 T 2 ´ 9.8 1 è 15 ø
\ = 1 = = \ v2 = 2v1
v2 T2 8 ´ 9.8 2
æ p xö
Since frequency remains the same and y2 = – 2 sin ç 96 p t - ÷
è 15 ø
\ l2 = 2l1 = 2 × 0.06 = 0.12 m
9. KEY CONCEPT : Using the formula of the coefficient of 11. The apparent frequency from tuning fork T1 as heard by the
linear expansion, observer will be
Dl = la ´ Dq
v v

stress T A T A
Also, Y= = = \ T = YA
A a Dq
strain Dl l aAq
T2 T1
The frequency of the fundamental mode of vibration. Observer

1 T 1 YA a Dq c
n= = n1 = ´n ... (i)
2l m 2l m c-v
where c = velocity of sound
1 2 ´ 1011 ´ 10 -6 ´ 1.21 ´ 10 -5 ´ 20 v = velocity of turning fork
= The apparent frequency from tuning fork T2 as heard by the
2 ´1 0.1
observer will be
= 11 Hz
c
æ p xö n2 = ´n ... (ii)
10. (i) Here amplitude, A = 4 sin ç ÷ c+v
è 15 ø Given n1 – n2 = 3
At x = 5 m é 1 1 ù c ´ n ´ 2v
\ c×n ê - ú = 3 or, 3 = 2
æ p ´ 5ö ë c - n c + n û c - v2
A = 4sin ç = 4 × 0.866 = 3.46 cm
è 15 ÷ø
c ´ n ´ 2v
(ii) Nodes are the position where A = 0 Since, v < < c \ 3=
c2
æ px ö
\ sin ç ÷ = 0 = sin np \ x = 15 n 3 ´ 340 ´ 340
è 15 ø \ v= = 1.5 m / s
340 ´ 340 ´ 2
where n = 0, 1, 2 x = 15 cm, 30 cm, 60 cm, ....
WAVES P-263
12. (i) KEY CONCEPT : When two progressive waves having
1 x
same amplitude and period, but travelling in opposite t2 = + ... (ii)
direction with same velocity superimpose, we get standing (1200 + 40) (1200 - 40)
waves. where tAH = time taken by sound from A to H with velocity
The following two equations qualify the above criteria and (1200 + 40)
hence produce standing wave tHB = time taken by sound from H to B with velocity 1200 – 40
z1 = A cos (k x – wt) From (i) and (ii)
z2 = A cos (k x + wt) 1- x 1 x
The resultant wave is given by z = z1 + z2 t1 = t2 Þ = +
40 1200 + 40 1200 - 40
Þ z = A cos (k x – wt) + A cos (k x + wt)
= 2A cos k x cos wt Þ x = 0.935 km
The resultant intensity will be zero when The frequency of echo as heard by the driver can be
2A cos k x = 0 calculated by considering that the source is the acoustic
image.
(2n + 1)
Þ cos k x = cos p é ( v - vm ) + v s ù
2 n'' = n ê ú
ë ( v - vm ) - v0 û
2n + 1 (2n + 1) p
Þ kx= p Þ x=
2 2k é (1200 - 40) + 40 ù
where n = 0, 1, 2, ... = 580 ê ú = 621 Hz
ë (1200 - 40) - 40 û
(ii) The transverse waves
z1 = A cos (k x – wt) 14. The angular frequency of the detector = 2pn
z3 = A cos (k y – wt) 5
Combine to produce a wave travelling in the direction making = 2p ´ = 10 rad / s
p
an angle of 45° with the positive x and positive y axes. The angular frequency of the detector matches with that of
The resultant wave is given by z = z1 + z3 the source.
z = A cos (k x – wt) + A cos (ky – wt)
( x - y) é k ( x + y ) - 2wt ù A1 w=10rad/s
Þ z = 2A cos cos ê úû
2 ë 2
The resultant intensity will be zero when
A2 6m 6m
k ( x - y) k ( x - y) A B C D
2 A cos =0 Þ cos =0
2 2
k ( x - y) 2n + 1 (2 n + 1)
Þ = p Þ (x – y) = p A3
2 2 k
13. (i) The frequency of the whistle as heard by observer on the Þ When the detector is at C moving towards D, the source
hill is at A1 moving leftwards. It is in this situation that the
frequency heard is minimum
é v + vm ù
n' = n ê ú
ë v + vm - vs û é v - v0 ù (340 - 60)
n' = n ê ú = 340 ´ = 257.3Hz
ë v + vs û (340 + 30)
x Again when the detector is at C moving towards B, the
source is at A3 moving rightward. It is in this situation that
A B
the frequency heard is maximum.
1km
Hill é v + v0 ù (340 + 60)
n'' = n ê ú = 340 ´ = 438.7 Hz
é 1200 + 40 ù ë v - vs û (340 - 30)
= 580 ê = 599 Hz
ë1200 + 40 - 40 úû 15. (a) KEY CONCEPT : Use the equation of a plane
(ii) Let echo from the hill is heard by the driver at B which progressive wave which is as follows.
is at a distance x from the hill.
æ 2p ö
The time taken by the driver to reach from A to B y = A cos ç x + 2pn t÷
è l ø
1- x
t1 = ... (i) The given equation is
40 y1 = A cos (ax + bt)
The time taken by the echo to reach from hill
2p 2p
t2 = tAH + tHB On comparing, we get =a Þ l=
l a
P-264 Topic-wise Solved Papers - PHYSICS
Also, 2pn = b The equation of resultant wave according to superposition
principle
b
Þ n= y = y1 + y2 = A sin 2pn1t + A sin 2pn2t
2p
(b) Since the wave is reflected by an obstacle, it will suffer = A [sin 2 pn1t + sin 2pn2t]
a phase difference of p. The intensity of the reflected wave (2pn1 + 2pn 2 )t (2pn1 + 2pn 2 )t
is 0.64 times of the incident wave. = A × 2 sin cos
2 2
Intensity of original wave I µ A2
Intensity of reflected wave I ' = 0.64 I = 2A sin p (n1 + n2) t cos p (n1 – n2) t
Þ I ' µ A'2 Þ 0.64 I µ A'2 where the amplitude A' = 2A cos p (n1 – n2) t
Þ 0.64 A2 µ A'2 Þ A' µ 0.8A Now, intensity µ (Amplitude)2
So the equation of resultant wave becomes Þ I µ A'2
y2 = 0.8A cos (ax – bt + p) = – 0.8 A cos (ax – bt) I µ 4A2 cos2p (n1 – n2) t
(c) KEY CONCEPT : The resultant wave equation can be
The intensity will be maximum when
found by superposition principle
y = y1 + y2 cos2p (n1 – n2) t = 1
= A cos (ax + bt) + [– 0.8 A cos (ax – bt)] or, cos p (v1 – v2) t = 1
The particle velocity can be found by differentiating the or, p (n1 – n2) t = np
above equation
(w1 - w 2 ) 2np
dy Þ t = n p or,, t =
v= = – Ab sin (ax + bt) – 0.8 Ab sin (ax – bt) 2 w1 - w 2
dt
= – Ab [sin (ax + bt) + 0.8 sin (ax – bt)] \ Time interval between two maxima
= – Ab [sin axcos bt + cos ax sin bt
2 np 2(n - 1)p 2p 2p
+ 0.8 sin ax cos bt – 0.8 cos ax sin bt] or, - or, = sec
v = – Ab [1.8 sin ax cos bt + 0.2 cos ax sin bt] w1 - w 2 w1 - w 2 w1 - w 2 103
The maximum velocity will occur when sin ax = 1 and Time interval between two successive maximas is
cos bt = 1 under these condition cos ax = 0 and sin bt = 0 2p × 10–3 sec
\ | vmax | = 1.8 Ab
(ii) For the detector to sense the radio waves, the resultant
Also, | vmin | = 0
intensity > 2A2
(d) y = [A cos (ax + bt)] – [0.8 A cos (ax – bt)]
= [0.8 A cos (ax + bt) + 0.2 A cos (ax + bt)] \ Resultant amplitude > 2A
– [0.8 A cos (ax – bt)]
= [0.8 A cos (ax + bt) – 0.8 A cos (ax – bt)] or, 2 A cos p ( n1 - n2 ) t ³ 2 A
+ 0.2 A cos (ax + bt)]
1 é ( w - w 2 )t ù 1
é ì ( ax + bt ) + (ax - bt ) ü or, cos p ( n1 - n2 ) t ³ or, cos ê 1 ³
= 0.8 A ê -2 sin í ý 2 2 ú
ë î 2 þ ë û 2

ì ( ax + bt ) - (ax - bt ) ü ù é (w - w 2 )t ù
sin í ý ú 0.2 A cos (ax + bt)] The detector lies idle when the values of cos ê 1 ú
î 2 þû ë 2 û
Þ y = – 1.6 A sin ax sin bt + 0.2 A cos (ax + bt) 1
where (– 1.6 A sin ax sin bt) is the equation of a standing is between 0 and
2
wave and 0.2 A cos (ax + bt) is the equation of travelling
wave. p
(w1 - w 2 )t p
The wave is travelling in –x direction. \ is between and
2 2 4
NOTE : Antinodes of the standing waves are the positions
where the amplitude is maximum,
p p
é pù \ t1 = and t2 =
i.e., sin ax = 1 = sin ê np + (-1)n ú w1 - w 2 2(w1 - w 2 )
ë 2û
\ The time gap = t1 – t2
é ( -1)n ù p p p p
Þ x = ên + ú = – =
êë 2 úû a w1 - w 2 2(w1 - w 2 ) 2(w1 - w 2 )
16. Let the two radio waves be represented by the equations
p
y1 = A sin 2pn1t = ´ 10-3 sec
y2 = A sin 2pn2t 2
WAVES P-265
17. The placements of the nodes and antinodes on the rod are We know that
as shown in the figure. v = rw = 1.5 × 20 = 30 ms–1
When the source is instantaneously at the position A, then
1m
the frequency heard by the observer will be
A N N N N N A
é v ù é 330 ù
Mid Point n' = n ê ú = 440 êë 330 - 30 úû = 484 Hz
l ë v - vs û
Clamped
4 When the source is instantaneously at the position B, then
the frequency heard by the observer will be
l
\ l+ = 0.5 Þ l = 0.4 m
4 é v ù é 330 ù
Also, the velocity of waves produced in the rod,
n'' = n ê ú = 440 ê 330 + 30 ú = 403.3Hz
ë v + vs û ë û

Y 2 ´ 1011 Hence the range of frequencies heard by the observer is


v= = = 5000 m/s 403.3 Hz to 484 Hz.
r 8 ´ 10 3
19.
Since, amplitudeof antinode= 2 × 10–6 m
\ 2a = 2 × 10–6 m Þ a = 10–6 m
The equation of the wave moving in the positive X-direction lo
will be First overtone frequency
2p l0 = l '
y1 = a sin (vt - x)
l
v v 330
Þ ( n1 )0 = = =
-6 2p l ' l0 l0
Þ y1 = 10 sin (5000t - x )
0.4
Fundamental frequency
The equation of wave after reflection and moving in X-axis
is Closed pipe

é 2p ù
y2 = 10-6 sin ê (5000t + x)ú
ë 0.4 û
The equation of the stationary wave is lc

2p 2p l1
y = 2a cos x sin vt lc = Þ l1 = 4l c
l l 4
æ 2p ö æ 2p ö v
\ y = 2 × 10–6 cos ç x sin ç
è 0.4 ÷ø è 0.4
´ 5000t ÷
ø Þ ( n1 )c = = 110 Hz (given)
4l c
Equation of wave at x = 2 cm But beat frequency is 2.2
æ 2p ö æ 2p ö Case 1 : (n1)0 > (n1)c
y = 2 × 10–6 cos ç ´ 0.02÷ sin ç ´ 5000t ÷ (n1)0 – (n1)c= 2.2
è 0.4 ø è 0.4 ø
y = 2× 10–6 cos (0.1 p) sin (25000 pt) 330
Þ - 330 = 2.2 Þ l 0 = 0.9933 m
18. The whistle which is emitting sound is being rotated in a l0
circle.

vs=rw =30m/s
A
lc
r= 1.5m
Case 2 : (n1)0 < (n1)c
(n1)c – (n1)0 = 2.2
O
vo=0 330
w=20rad-1 330 - = 2.2 Þ l = 1.006 m
l0 0

20. KEY CONCEPT : Motorist will listen two sound waves.


B v =rw
s One directly from the sound source and other reflected from
=30m/s
the fixed wall. Let the apparent frequencies of these two
r = 1.5 m (given); w = 20 rads–1 (given) waves as received by motorist are f 'and f '' respectively.
P-266 Topic-wise Solved Papers - PHYSICS
For Direct Sound : Vm will be positive as it moves towards
the source and tries to increase the apparent frequency. Vb é 2p 15 ù DP
Amplitude = DP0 cos ê ´ ú= 0
will be taken positive as it move away from the observer and ë (330 / 440) 32 û 2
hence tries to decrease the apparent frequency value. (c) At open end of pipe, pressure is always same i.e. equal
v + vm to mean pressure Q DP = 0, Pmax = Pmin = P0
f'= f ... (1)
v + vb (d) At the closed end :
Maximum Pressure = P0 + DP0
f1 Minimum Pressure = P0 – DP0
Vm Reflected 22. (a) Mass per unit length of PQ
wave Fixed
Vb Wall 0.06
m1 = kg / m
4.8
O S
0.06 kg 0.2 kg
For reflected sound :
P 4.8 m Q 2.56 m R
For sound waves moving towards stationary observer (i.e.
wall), frequency of sound as heard by wall 0.2
Mass per unit length of QR, m2 = kg / m
v 2.56
f1 = f
v - vb Velocity of wave in PQ is
After reflection of sound waves having frequency f1 fixed
T 80
wall acts as a stationary source of frequency f1 for the moving v1 = = = 80 ms -1 [Q T = 80 N given]
observer i.e. motorist. As direction of motion of motorist is m1 0.06 / 4.8
opposite to direction of sound waves, hence frequency f '' Velocity of wave in QR is
of reflected sound waves as received by the motorist is
v + vm v + vm T 80
f1 = f v2 = = = 32 m/s
f '' = ... (2) m2 0.2 / 2.56
v v - vb
Hence, beat frequency as heard by the motorist \ Time taken for the wave to reach from P to R
= tPQ + tQR
æ v + vm v + vm ö
Df = f '' – f ' = ç - f
è v - vb v + vb ÷ø =
4.8 2.56
+ = 0.14s
80 32
2vb ( v + vm ) f (b) When the wave which initiates from P reaches Q
or, Df = (a denser medium) then it is partly reflected and partly
v 2 - vb 2
transmitted.
21. (a) For second overtone as shown,
In this case the amplitude of reflected wave
5l 4l
= l \ l= æ v2 - v1 ö
4 5 Ar = ç Ai ... (i)
Also, v = nl è v2 + v1 ÷ø
4l 15 where Ai = amplitude of incident wave.
Þ 330 = 440 × Þ l = m. Also amplitude of transmitted wave is
5 16
l æ 2v2 ö
At = ç Ai ... (ii)
(Pressure (Pressure antinode) è v1 + v2 ÷ø
node) or
displacement displacement node
antinode From (i), (ii)
x=L MID POINT x=O Therefore, At = 2 cm and Ar = – 1.5 cm.
Pressure variation x=x Pressure variation
is zero is max or min 23. Speed of sound, v = 340 m/s.
(b) KEY CONCEPT : At any position x, the pressure is Let l 0 be the length of air column corresponding to the
given by fundamental frequency. Then
DP = DP0 cos kx cos wt
v
2p = 212.5
Here amplitude A = DP0 cos kx = DP0 cos x 4l 0
l
15 15 v 340
For x = = m (mid point) or l0 = = = 0.4 m.
2 ´ 16 32 4(212.5) 4(212.5)
WAVES P-267
NOTE : In closed pipe only odd harmonics are obtained.
2.4 dH t
Now, let l1, l 2 , l 3 , l 4 , etc. be the lengths corresponding Þ ò3.2 = -(1.11 ´ 10-2 )ò dt
H 0
to the 3rd harmonic, 5th harmonic, 7th harmonic etc. Then
Þ 2[ 2.4 - 3.2] = -(1.1 ´ 10 -2 ).t
æ v ö
3ç = 212.5 Þ l1 = 1.2 m; Þ t » 43second
è 4l1 ÷ø
24. KEY CONCEPT : The question is based on Doppler's effect
æ v ö where the medium through which the sound is travelling is
5ç = 212.5 Þ l 2 = 2.0 m also in motion.
è 4l 2 ÷ø By Doppler's formula
æ v ö é c + v m ± v0 ù
7ç = 212.5 Þ l 3 = 2.8 m; n' = n ê ú ... (1)
è 4l 3 ÷ø ë c + vm ± vs û
NOTE : Sign convention for Vm is as follows :
æ v ö If medium is moving from s to O then + ve and vice versa.
9ç = 212.5 Þ l 4 = 3.6 m
è 4l 4 ÷ø Similarly v0 and vs are positive if these are directed from S to
O and vice versa.
(a) Situation 1.
10m/s
0.4m

2m/s
1.2m

S
2.0m

O
2.8m

(Transmitter)
(Receiver)
3.2m

3.4m

1.6m

B 2.088 ´ 109
0.8m

Velocity of sound in water c = =


r 103
or heights of water level are (3.6 – 0.4) m, (3.6 – 1.2) m, c = 1445 m/s; vm = + 2 m/s; v0 = 0; vs = 10 m/s
(3.6 – 2.0) m and (3.6 – 2.8) m. é 1445 + 2 - 0 ù
\ n' = n ê = n[1.007]
Therefore heights of water level are 3.2 m, 2.4 m, 1.6 m and ë1445 + 2 - 10 úû
0.8 m.
Let A and a be the area of cross-sections of the pipe and c 1445
Now, n = = = 105 Hz
hole respectively. Then l 14.45 ´ 10-3
A = p (2 × 10–2)2 = 1.26 × 10–3 m–2 \ n' = 1.007 × 105 Hz
and a = p (10–3)2 = 3.14 × 10–6 m2 (b) Situation 2.
Velocity of efflux, v = 2 gH g RT
Continuity equation at 1 and 2 gives, In air c = = 344 m/s
M
æ - dH ö Applying formula (1)
a 2 gH = A ç
è dt ÷ø é 344 - 5 - 0 ù
n' = n ê = 1.03 × 105 Hz
Therefore, rate of fall of water level in the pipe, ë 344 - 5 - 10 úû
æ -dH ö a
çè ÷= 2 gH vm = 5ms-1
dt ø A
S
Substituting the values, we get O
-6
-dH 3.14 ´ 10 10m/s (Receiver)
= 2 ´ 10 ´ H
dt 1.26 ´ 10 -3
- dH 25. (a) Second harmonic in pipe A is
Þ = (1.11 × 10–2) H
dt
é v ù 1 g A RT
Between first two resonances, the water level falls from 2 ( n0 ) A = 2 ê ú =
3.2 m to 2.4 m. ë 2l û l M A
Third harmonic in pipe B is
dH
\ = -1.11 ´ 10-2 dt
évù 3 g B RT
H 3 ( n0 ) B = 3 ê ú =
ë 4l û 4l MB
P-268 Topic-wise Solved Papers - PHYSICS
Given n A = nB While the train is receding

1 g A RT 3 g B RT vS
= n'' = n
l MA 4l MB v S + vT

2 Here, v' = 1.8 KHz = 1800 Hz (given)


M A g A æ 4ö 5 / 3 16 400
or, = ´ç ÷ = ´ =
M B g B è 3ø 7/5 9 189 300
\ 1800 = n ... (ii)
A B
300 + vT
Gas (Monoatomic) Gas (Diatomic) Dividing (i) and (ii)
MA MB
l l 2200 300 300 + vT
= ´
1800 300 - v T 300
(v0 ) A g A MB 3
Now, = ´ =
(v0 ) B g A MB 4 Þ vT = 30 m/s
26. KEY CONCEPT : In the fundamental mode 29. KEY CONCEPT : The wave form of a transverse harmonic
disturbance is
l v
(l + 0.6 r ) = = Þ v = 4f (l + 0.6 r ) = 336 m/s. y = a sin (wt ± kx ± f)
4 4f
Given vmax = aw = 3 m/s ... (i)
l 2p 2p p
27. Here l = or l = 2l Since, k = = = Amax = aw2 = 90 m/s2 ... (ii)
2 l 2l l
Velocity of wave v = 20 m/s ... (iii)
The amplitude of vibration at a distance x from x = 0 is given
Dividing (ii) by (i)
by A = a sin k x
Mechanical energy at x of length dx is aw 2 90
= Þ w = 30 rad/s ... (iv)
1 1 aw 3
dE = (dm) A2 w 2 = (µdx)(a sin k x )2 (2pn) 2
2 2 Substituting the value of w in (i), we get
= 2p2µn2a2 sin2 kx dx
3
But v = nl a= = 0.1m ... (v)
30
v v2 T / µ 2p 2p w 30 3
\ n= Þ n2 = 2 = 2 éQ v = T / m ù = = = =
l l 4l ë û Now, k = ... (vi)
l v / n v 20 2

T / µ 2 2 ìæ p ö ü From (iv), (v) and (vi) the wave form is


\ dE = 2p2µ a sin íç ÷ x ý dx
4l 2 îè l ø þ 3
y = 0.1 sin [30t ± x ± f]
\ Total energy of the string 2

l T /µ æ p xö MATCH THE FOLLOWING :


E = ò dE = ò 2p µ
2
a 2 sin 2 ç ÷ dx
0 4l2 è l ø
1. (A) Pitch q. frequency
(B) quality p. waveform
p 2Ta 2
= (C) loudness r. intensity
4l
2. A-p,t; B-p,s; C-q,s; D-q,r
28. Let the speed of the train be vT (A) Pipe closed at one end
While the train is approaching Waves produced are longitudinal (sound waves)
Let n be the actual frequency of the whistle. Then L
vS
n' = n
v S - vT
lf
where vS = Speed of sound = 300 m/s (given) 4
n' = 2.2 K Hz. = 220 Hz (given)
lf
300 = L \ lf = 4 L
\ 2200 = n ... (i) 4
300 - vT
WAVES P-269
(p, t) are correct matching NOTE : Wave velocity is same because it depends on the
(B) Pipe open at both ends medium in which the wave is travelling.
waves produced are longitudinal (sound waves) Now, at x = 0,
y1 + y2 = (A cos 10 pt) + (A cos 92 pt) = 0
L Þ cos 100 pt = – cos 92 pt = cos (– 92 pt)
= cos [(2n + 1)p – 92 pt
lf 2n + 1
Þ t=
2 192

lf 1 191
= L \ lf = 2L when t = 0, n = - and when t = 1, n = = 95.2
2 2
2
Þ net amplitude is zero for n = 96 times (the nearest
(p, s) are correct matching.
answer).
(c) Stretched wire clamped at both ends 4. (b) The speed of sound depends on the frame of reference
Waves produced are transverse in nature. (waves on string) of the observer.
5. (a) Since all the passengers in train A are moving with a
velocity of 20 m/s therefore the distribution of sound
lf intensity of the whistle by the passengers in train A is
uniform.
2
é v - v0 ù é 340 - 30 ù 31
lf 6. (a) n ' = n1 ê ú = 800 ê 340 - 20 ú = 800 ´ 32
= L \ lf = 2L ë v - vs û ë û
2
(q, s) are correct matching. é v - v0 ù 31
(D) Stretched wave clamped at both ends & mid point n '' = n2 ê ú = 1120 ´ 32
ë v - vs û
Waves produced are transverse in nature (waves on string)
31 31
\ n ''- n ' = (1120 – 800) × = 320 ´ = 310 Hz.
32 32
lf INTEGERVALUECORRECTTYPE:
lf = L
(q, r) are correct matching. T 0.5
1. We know that, n = = -3
= 10m/s
m 10 / 0.2
COMPREHENSION BASED Q UESTIONS :
The wavelength of the wave established
1. (a) 2. (c) 3. (d)
The equations are y 1 = A cos (0.5 px – 100 pt) and v 10
y2 = A cos (0.46 px – 92 pt) represents two progressive wave l= = = 0.1m = 10cm
f 100
travelling in the same direction with slight difference in the
frequency. This will give the phenomenon of beats. \ The distance between two successive nodes
Comparing it with the equation l 10
y = A cos (kx – wt), we get = = = 5cm
2 2
w1 = 100 p Þ 2pf1 = 100 p Þ f1 = 50 Hz and
2. Let v be the speed of sound and vc and f0 the speed and
2p frequency of car.
K1 = 0.5 p Þ = 0.5p Þ l1 = 4 m
l1 The frequency of sound reflected by the car is

w é v + vc ù
Wave velocity = l1f1 = 200 m/s [Alternatively use v = ] \ f1¢ = f 0 ê ú
K ë v - vc û
w2 = 92 p Þ 2pf2 = 92 p Þ f2 = 46 Hz Differentiating the above equation w.r.t. vc ,we get
Therefore beat frequency = f1 – f2 = 4 Hz and
2p 200 é d d ù
ê (v - vc ) (v + vc ) - (v + vc ) (v - vc ) ú
K2 = 0.46 p Þ = 0.46p Þ l2= d f1¢ dvc dvc
l2 46 = f0 ê ú
2
dvc ê (v - vc ) ú
200 êë úû
Wave velocity = ×46 = 200 m/s
46
P-270 Topic-wise Solved Papers - PHYSICS

df1¢ é 2v ù 2v 3.
2 2
Resultant amplitude, A = A1 + A2 + 2 A1 A2 cos f
\ = f0 ê 2ú
= f 0 2 (Q vc << v)
dvc êë (v - vc ) úû v
p
= 42 + 32 + 2 ´ 4 ´ 3 ´ cos = 16 + 9 + 0 = 5
df1¢ 2 2
\ ´ 100 = ´ dvc
f0 v

2 ´ dvc
\ 0.012 = \ dvc = 0.198 m/s » 7 km/h
330

1. (b) This will happen for fundamental mode of vibration as As temperature increases, l increases and therefore f
shown in the figure. decreases.
l æ pö
Here = 40 \ l = 80 cm 6. (a) y = 10-4 sin ç 600t - 2x + ÷
2 è 3ø
But y = A sin ( wt - kx + f)
S1 S2
On comparing we get w = 600; k=2
40 cm w 600
S1 and S2 are rigid support v= = = 300 ms -1
k 2
2. (c) KEY CONCEPT : The fundamental frequency for 7. (a) KEY CONCEPT : For a string vibrating between two
rigid support, the fundamental frequency is given by
v
closed organ pipe is given by uc =
4l n=
v
=
1 T
2l 2l m
l
l=
l = l /4 2

l
The fundamental frequency for open organ pipe is l=
2
v u0 v 4l 2
given by u0 = \ u = 2l ´ v = 1 1 10 ´ 9.8
2l c = = 50 Hz
2 ´ 1 9.8 ´ 10-3
3. (b) A tuning fork produces 4 beats/sec with another tuning
fork of frequency 288 cps. From this information we As the string is vibrating in resonance to a.c of
can conclude that the frequency of unknown fork is frequency n, therefore both the frequencies are same.
288 + 4 cps or 288 – 4 cps i.e. 292 cps or 284 cps. When 8. (c) A tuning fork of frequency 256 Hz makes 5 beats/
a little wax is placed on the unknown fork, it produces second with the vibrating string of a piano. Therefore
2 beats/sec. When a little wax is placed on the unknown the frequency of the vibrating string of piano is (256 ±
fork, its frequency decreases and simultaneously the 5) Hz ie either 261Hz or 251 Hz. When the tension in
beat frequency decreases confirming that the frequency the piano string increases, its frequency will increases.
Now since the beat frequency decreases, we can
of the unknown fork is 292 cps.
conclude that the frequency of piano string is 251Hz
4. (b) To form a node there should be superposition of this
9. (b) From equation given,
wave with the reflected wave. The reflected wave
should travel in opposite direction with a phase change w 100
w = 100 and k = 20, v = = = 5m / s
of p. The equation of the reflected wave will be k 20
y = a sin (wt + kx + p) 10. (d) No. of beats heard when fork 2 is sounded with fork 1
Þ y = – a sin (wt + kx) = Dn = 4
5. (b) KEY CONCEPT : The frequency of a tuning fork is Now we know that if on loading (attaching tape) an
given by the expression unknown fork, the beat frequency increases (from 4 to
6 in this case) then the frequency of the unknown fork
m2 k Y
f= 2 is given by,
4 3 pl 2 r n = n 0 - Dn = 200 – 4 = 196 Hz
WAVES P-271

é vù
êv + 5 ú 2p 2p
é v + v0 ù é6ù But k = and w =
11. (c) n' = nê ú = n ëê v ûú = n êë 5 úû l T
ë v û
2p 2p
n' 6 n' - n 6 - 5 Þ = a and =b
= ; = ´ 100 = 20% l T
n 5 n 5 Given that l = 0.08 m and T = 2.0s
é v ù é 300 ù 2p 2p
12. (c) n' = n ê ú Þ 10000 = 9500ê ú \ a= = 25p and b = =p
ë v - vs û ë 300 - v û 0.08 2
17. (b) Maximum number of beats = ( n + 1) – ( n – 1) = 2
Þ 300 - v = 300 ´ 0.95 Þ v = 300 - 285 = 15 ms -1
2
v u=0 a = 2m/s vm
13. (a) Given nv = 315 and ( n + 1) = 420 18. (a)
2l 2l Electric s Motor
n + 1 420 siren cycle
Þ = Þn =3
n 315
v 2m - u 2 = 2as Þ v2m = 2 ´ 2 ´ s
v v
Hence 3 ´ = 315 Þ = 105 Hz
2l 2l \ vm = 2 s
Lowest resonant frequency is when n = 1 According to Doppler’s effect
Therefore lowest resonant frequency = 105 Hz.
é v - vm ù
æI ö æ I2 ö v' = vê ú
14. (a) We have, L1 = 10 log ç 1 ÷ ; L2 = 10 log ç ÷ ë v û
èI ø 0
è I0 ø
é 330 - 2 s ù
æI ö æI ö 0.94v = v ê ú Þ s = 98.01 m
\ L1 – L2 = 10 log ç 1 ÷ - 10log ç 2 ÷
è I0 ø è I0 ø ë 330 û

æI I ö æI ö é æ t ö x ù
or, DL = 10 log ç 1 ´ 0 ÷ or, DL = 10log ç 1 ÷ 19. (d) y = 0.02(m)sin ê 2p ç ÷ - 0.50(m) ú
è I2 ø ë è 0.04( s ) ø û
è I0 I 2 ø
Comparing this equation with the standard wave
æI ö æI ö
or, 20 = 10 log ç 1 ÷ or, 2 = log ç 1 ÷ equation
è 2ø
I è I2 ø
y = a sin(wt - kx )
I I we get
or, 1 = 102 or, I2 = 1 .
I2 100
2p 1
w= Þ n= = 25 Hz
Þ Intensity decreases by a factor 100. 0.04 0.04
v v 2p
15. (b) For first resonant length n = = (in winter) k= Þ l = 0.5 m
4l1 4 ´ 18 0.50
For second resonant length \ velocity, v = nl = 25 × 0.5 m/s = 12.5 m/s
3v ' 3v ' v 3v' Velocity on a string is given by
n' = = (in summer) \ =
4l 2 4x 4×18 4× x T
v= 2 = (12.5)2 × 0.04 = 6.25 N
v' v' m \ T = v ´m
\ x = 3 ´ 18 ´ \ x = 54 ´ cm
v v
20. (a) Given wave equation is y(x,t) = e(- ax +bt 2 + 2 ab xt )
2

v' > v because velocity of light is greater in summer as


compared to winter (v µ T ) -[( ax ) 2 + ( b t ) 2 + 2 a x . b t ] -( ax + bt ) 2
=e =e
\ x > 54cm æ ö
2
b ÷
- çè x + tø
16 (a) y(x, t) = 0.005 cos (ax - bt) (Given) = e a

Comparing it with the standard equation of wave It is a function of type y = f (x + vt)


y(x, t) = a cos (kx - wt) we get b
Þ Speed of wave =
k = a and w = b a
P-272 Topic-wise Solved Papers - PHYSICS
21. (d) y = A sin (wt – kx) + A sin (wt + kx)
v v
y = 2A sin wt cos kx Thus n c = Þ nc … (iii)
l0 /2 2l
For standing wave nodes
cos kx = 0 From equations (i) and (iii)
n 0 = nc
2p p
.x = (2n + 1)
l 2 Thus, nc = f ( Q n0 = f is given)
24. (b) Fundamental frequency,
\ x=
( 2n + 1) l , n = 0,1, 2,3,...........
4
v 1 T 1 T é T mù
22. (a) Since, I µ A2w 2 f = = = êQ v = and m = ú
2l 2l m 2l Ar ë m lû
I1 µ (2a)2 w 2 , I 2 µ a 2 (2w ) 2

I1 = I 2 Tl T Y Dl 1 gDl
Also, Y = Þ = Þ f = ....(i)
ADl A l 2l lr
Intensity depends on frequency also.
23. (a) The fundamental frequency of open tube Dl
l = 1.5 m, = 0.01, r = 7.7 × 103 kg/m3 (given)
v l
n0 = … (i)
2l0 g = 2.2 × 1011 N/m2 (given)
where l is the length of the tube Dl
Putting the value of l, , r and g in eqn. (i) we get,
v = speed of sound l
That of closed pipe
2 103
v f = ´
nc = 7 3
4lc … (ii)
or, f » 178.2 Hz
l0
According to the problem lc =
2
12 Electrostatics

4. Two parallel plate capacitors of capacitances C and 2C are


FILL IN THE BLANKS : connected in parallel and charged to a potential difference
1. Five identical capacitor plates, each of area A, are arranged V. The battery is then disconnected and the region between
such that adjacent plates are at a distance d apart, the plates the plates of the capacitor C is completely filled with a
are connected to a source of emf V as shown in the figure material of dielectric constant K. The potential differences
(1984- 2 Marks) across the capacitors now becomes............. (1988 - 2 Marks)
– 5. A point charge q moves from point P to point S along the
+ path PQRS (fig.) in a uniform electric field E pointing parallel
1 2 34 5 to the positive direction of the X-axis. The cooridnates of
the points P, Q, R and S are (a, b, O), (2a, O, O) (a, – b, O) and
(O, O,O) respectively. The work done by the field in the
above process is given by the expression ................
The charge on plate 1 is ..... and on plate 4 is ..... (1989 - 2 Marks)
2. Figure shows line of constant potential in a region in which Y
an electric field is present. The values of the potential are
written in brackets. Of the points A, B and C, the magnitude
of the electric field is greatest at the point .......
(1984- 2 Marks) P
S Q X
R E

6. The electric potential V at any point x, y, z (all in metres) in


space is given by V = 4x2 volts. The electric field at the point
A (1m, 0, 2 m) is .................... V/m. (1992 - 1 Mark)
7. Five point charges, each of value + q coul, are placed on
B
five vertices of a regular hexagon of side L metres. The
magnitude of the force on the point charge of value – q
coul. placed at the centre of the hexagen is ....................
newton. (1992 - 1 Mark)
V
50

C
V

q q
40

V
30
10 0V
V
2

3. Two small balls having equal positive charges Q (coulomb) q q


on each are suspended by two insulating strings of equal
length L (metre) from a hook fixed to a stand. The whole set
TRUE / FALSE :
up is taken in a satellite into space where there is no gravity
(state of weightlessness). The angle between the two strings 1. The work done in carrying a point charge from one point to
is ........... and the tension in each string is ........ newtons. another in an electrostatic field depends on the path along
(1986 - 2 Marks) which the point charge is carried. (1981- 2 Marks)
P-274 Topic-wise Solved Papers - PHYSICS
2. Two identical metallic spheres of exactly equal masses are 3. A parallel plate capacitor of capacitance C is connected to a
taken. One is given a positive charge Q coulombs and the battery and is charged to a potential difference V. Another
other an equal negative charge. Their masses after charging capacitor of capacitance 2C is similarly charged to a potential
are different. (1983 - 2 Marks) difference 2V. The charging battery is now disconnected
3. A small metal ball is suspended in a uniform electric field and the capacitors are connected in parallel to each other in
with the help of an insulated thread. If high energy X-ray such a way that the positive terminal of one is connected to
beam falls on the ball, the ball will be deflected in the direction the negative terminal of the other. The final energy of the
of the field. (1983 - 2 Marks) configuration is (1995S)
4. Two protons A and B are placed in between the two plates 3
of a parallel plate capacitor charged to a potential difference (a) zero (b) CV 2
2
V as shown in the figure. The forces on the two protons are
25 9
identical. (1986 - 3 Marks) (c) CV 2 (d) CV 2
6 2
+ –
4. An electron of mass me, initially at rest, moves through a
certain distance in a uniform electric field in time t1. A proton
of mass mp, also, intially at rest, takes time t2 to move through
an equal distance in this uniform electric field. Neglecting
the effect of gravity, the ratio t2 / t1 is nearly equal to
(1997C - 1 Mark)
(a) 1 (b) (m p / me )1/ 2
°
B (c) (me / m p )1/ 2 (d) 1836
°
A 5. A non-conducting ring of radius 0.5 m carries a total charge
of 1.11× 10 –10 C distributed non-uniformly on its
circumference producing an electric field E everywhere in
l= 0
space. The value of the integral ò - E.dl
l =¥
( l = 0 being center of the ring) in volts is (1997C - 1 Mark)
+ – (a) +2 (b) –1
V
5. A ring of radius R carries a uniformly distributed charge + Q. (c) –2 (d) zero
A point charge – q is placed on the axis of the ring at a 6. Two identical metal plates are given positive charges Q1
distance 2R from the centre of the ring and released from and Q2 (<Q1) respectively. If they are now brought close
rest. The particle executes a simple harmonic motion along together to form a parallel plate capacitor with capacitance
the axis of the ring. (1988 - 2 Marks) C, the potential difference between them is
6. An electric line of forces in the x – y plane is given by the (1999 - 2 Marks)
equation x2 + y2 = 1. A particle with unit positive charge, (a) (Q1+Q2)/(2C) (b) (Q1+Q2) / C
initially at rest at the point x = 1, y = 0 in the x – y plane, will (c) (Q1– Q2) /C (d) (Q1– Q2) /(2C)
move along the circular line of force. (1988 - 2 Marks) 7. For the circuit shown in Figure, which of the following
statements is true? (1999 - 2 Marks)
MCQ's WITH ONE CORRECT ANSWER :
1. A hollow metal sphere of radius 5 cms is charged such that
the potential on its surface is 10 volts. The potential at the
centre of the sphere is (1983 - 1 Mark)
(a) zero
(b) 10 volts
(c) same as at a point 5 cms away from the surface
(d) same as at a point 25 cms away from the surface (a) With S1 closed V1=15 V, V2 = 20 V
2. Two point charges +q and –q are held fixed at (–d, o) and (b) With S3 closed, V1= V2 = 25 V
(d, o) respectively of a x-y coordinate system. Then (1995S) (c) With S1 and S2 closed, V1 = V2 = 0
(a) The electric field E at all points on the x-axis has the
same direction (d) With S1 and S3 closed, V1 = 30 V , V2 = 20 V
(b) Electric field at all points on y-axis is along x-axis 8. Three charges Q, +q and +q are placed at the vertices of a
(c) Work has to be done in bringing a test charge from ¥ right-angled isosceles triangle as shown. The net
to the origin electrostatic energy of the configuration is zero if Q is equal
(d) The dipole moment is 2qd along the x-axis to (2000S)
ELECTROSTATICS P-275
Q
12. A uniform electric field pointing in positive x-direction exists
in a region. Let A be the origin, B be the point on the x-axis
at x = +1 cm and C be the point on the y-axis at y = +1 cm.
Then the potentials at the points A, B and C satisfy:
+q +q (2001S)
a
(a) VA < VB (b) VA > VB
-q -2q (c) VA < VC (d) VA > VC
(a) (b) 13. Two equal point charges are fixed at x = – a and x = + a on
1+ 2 2+ 2 the x-axis. Another point charge Q is placed at the origin.
(c) –2q (d) +q The change in the electrical potential energy of Q, when it is
9. A parallel plate capacitor of area A, plate separation d and displaced by a small distance x along the x-axis, is
capacitance C is filled with three different dielectric materials approximately proportional to (2002S)
having dielectric constants k1, k2 and k3 as shown. If a (a) x (b) x 2
single dielectric material is to be used to have the same (c) x 3 (d) 1/x
capacitance C in this capacitor, then its dielectric constant 14. Two identical capacitors, have the same capacitance C. One
k is given by (2000S ) of them is charged to potential V1 and the other V2. The
negative ends of the capacitors are connected together.
A/2 A/2 When the positive ends are also connected, the decrease in
energy of the combined system is (2002S)
K1 K2 d/2
d (a)
1
4
(
C V12 - V22 ) (b)
1
4
(
C V12 + V22 )
K3
C (V ) C (V )
1 2 1 2
(c) 1 - V2 (d) 1 + V2
A 4 4
A = Area of plates 15. A metallic shell has a point charge ‘q’ kept inside its cavity.
Which one of the following diagrams correctly represents
1 1 1 1 1 1 1 the electric lines of forces? (2003S)
(a) = + + (b) = +
K K1 K 2 2 K3 K K1 + K 2 2 K3

K1 K 2
(c) K= + 2K3 (d) K = K1 + K2 + 2K3
K1 + K 2 (a) (b)
10. Three positive charges of equal value q are placed at the
vertices of an equilateral triangle. The resulting lines of force
should be sketched as in (2001S)

(c) (d)
(a) (b)
16. Six charges of equal magnitude, 3 positive and 3 negative
are to be placed on PQRSTU corners of a regular hexagon,
such that field at the centre is double that of what it would
have been if only one +ve charge is placed at R. Which of
the following arrangement of charge is possible for P, Q, R,
(c) (d) S, T and U respectively. (2004S)
P Q
11. Consider the situation shown in the figure. The capacitor A
has a charge q on it whereas B is uncharged. The charge U O R
appearing on the capacitor B a long time after the switch is T S
closed is (2001S) (a) + , +, +, –, –, – (b) –, + , +, +, –, –
q (c) –, + , +, –, +, – (d) + , –, +, –, +, –
+ 17. A Gaussian surface in the figure is shown by dotted line.
+ The electric field on the surface will be (2004S)
+
+ S
+ q1 q2
+
–q1
A B
(a) zero (b) q/2 (a) due to q1 and q2 only (b) due to q2 only
(c) q (d) 2q (c) zero (d) due to all
P-276 Topic-wise Solved Papers - PHYSICS
18. Three infinitely long charge sheets are placed as shown in 22. Positive and negative point charges of equal magnitude are
figure. The electric field at point P is (2005S)
æ aö æ -a ö
Z kept at çè 0, 0, ÷ø and ç 0, 0, ÷ respectively. The work
2 è 2ø
s done by the electric field when another positive point charge
Z = 3a is moved from (–a, 0, 0) to (0, a, 0) is (2007)
P (a) positive
-2s (b) negative
Z=a (c) zero
(d) depends on the path connecting the initial and final
X positions
-s 23. Consider a system of three charges q/3, q/3 and –2q/3 placed
Z = -a
at points A, B and C, respectively, as shown in the figure.
2s 4s Take O to be the centre of the circle of radius R and angle
(a) k̂ (b) k̂ CAB = 60° (2008)
e0 e0 y
2s ˆ 4s ˆ
(c) - k (d) - k
e0 e0
B
19. A long, hollow conducting cylinder is kept coaxially inside
another long, hollow conducting cylinder of larger radius.
C
Both the cylinders are initially electrically neutral. (2007) x
(a) A potential difference appears between the two 60° O
cylinders when a charge density is given to the inner
cylinder. A
(b) A potential difference appears between the two
cylinders when a charge density is given to the outer q
cylinder. (a) The electric field at point O is directed along
8pe 0 R 2
(c) No potential difference appears between the two
cylinders when a uniform line charge is kept along the the negative x-axis
axis of the cylinders (b) The potential energy of the system is zero
(d) No potential difference appears between the two (c) The magnitude of the force between the charges at C
cylinders when same charge density is given to both q2
the cylinders. and B is
54pe 0 R 2
20. Consider a neutral conducting sphere. A positive point
charge is placed outside the sphere. The net charge on the q
sphere is then (2007) (d) The potential at point O is
12pe0 R
(a) negative and distributed uniformly over the surface of
24. A parallel plate capacitor C with plates of unit area and
the sphere
separation d is filled with a liquid of dielectric constant
(b) negative and appears only at the point on the sphere K = 2. The level of liquid is d/3 initially. Suppose the liquid
closest to the point charge level decreases at a constant speed v, the time constant as a
(c) negative and distributed non-uniformly over the entire function of time t is – (2008)
surface of the sphere
(d) zero
21. A spherical portion has been removed from a solid sphere
having a charge distributed uniformly in its volume as shown
in the figure. The electric field inside the emptied space is C
(2007) d d/3 R

6e 0 R (15d + 9vt ) e 0 R
(a) (b)
5d + 3vt 2d 2 - 3dvt - 9v 2t 2
(a) zero everywhere (b) non-zero and uniform 6 e0 R (15d - 9vt ) e 0 R
(c) non-uniform (d) zero only at its center (c) (d)
5d - 3vt 2d 2 - 3dvt - 9v 2t 2
ELECTROSTATICS P-277
25. Three concentric metallic spherical shells of radii R, 2R, 3R, r
29. Consider an electric field E = E0 xˆ where E0 is a constant.
are given charges Q1, Q2, Q3, respectively. It is found that
the surface charge densities on the outer surfaces of the The flux through the shaded area (as shown in the figure)
shells are equal. Then, the ratio of the charges given to the due to this field is (2011)
shells, Q1 : Q2 : Q3, is (2009)
z
(a) 1 : 2 : 3 (b) 1 : 3 : 5
(c) 1 : 4 : 9 (d) 1 : 8 : 18 (a,0,a) (a,a,a)
26. A disc of radius a / 4 having a uniformly distributed charge
6C is placed in the x - y plane with its centre at (–a / 2, 0, 0).
A rod of length a carrying a uniformly distributed charge 8C
is placed on the x - axis from x = a /4 to x = 5a / 4. Two point
charges – 7 C and 3 C are placed at (a / 4, – a /4, 0) and y
(0,0,0) (0,a,0)
(– 3a /4, 3a / 4, 0), respectively. Consider a cubical surface x
formed by six surfaces x = ± a / 2, y = ± a / 2, z = ± a / 2. The
electric flux through this cubical surface is (a) 2E0a2 (b) 2 E0 a 2
y
E0 a 2
(c) E0 a2 (d)
2
30. A 2 mF capacitor is charged as shown in the figure. The
x percentage of its stored energy dissipated after the switch
S is turned to position 2 is (2011)

(2009)

–2C 2C
(a) (b) e0
e0 (a) 0% (b) 20%
10C 12C (c) 75% (d) 80%
(c) (d)
e0 e0 31. Which of the field patterns given below is valid for electric
field as well as for magnetic field? (2011)
27. A uniformly charged thin spherical shell of radius R carries
uniform surface charge density of s per unit area. It is made
of two hemispherical shells, held together by pressing them
with force F (see figure). F is proportional to (2010) (a) (b)

F F

1 2 2 1 2 (c) (d)
(a) s R (b) s R
e0 e0

1 s2 1 s2
(c) (d)
e0 R e0 R 2
32. A wooden block performs SHM on a frictionless surface
28. A tiny spherical oil drop carrying a net charge q is balanced with frequency, n0. The block carries a charge +Q on its
in still air with a vertical uniform electric field of strength r
81p surface. If now a uniform electric field E is switched-on as
´ 105 Vm –1 . When the field is switched off, the drop shown, then the SHM of the block will be (2011)
7
is observed to fall with terminal velocity 2 ´ 10 –3 ms –1. Given
g = 9.8 m s –2 , viscosity of the air = 1.8 ´ 10–5 Ns m–2 and
the density of oil = 900 kg m –3 , the magnitude of q is (2010)
(a) of the same frequency and with shifted mean position.
(a) 1.6 ´ 10–19 C (b) 3.2 ´ 10 –19 C
(b) of the same frequency and with the same mean position
(c) 4.8 ´ 10–19 C (d) 8.0 ´ 10 –19 C (c) of changed frequency and with shifted mean position.
(d) of changed frequency and with the same mean position.
P-278 Topic-wise Solved Papers - PHYSICS
33. Two large vertical and parallel metal plates having a
separation of 1 cm are connected to a DC voltage source of
potential difference X. A proton is released at rest midway
between the two plates. It is found to move at 45° to the 1. Two equal negative charges –q are fixed at points (0, – a)
vertical JUST after release. Then X is nearly and (0, a) on y – axis. A positive charge Q is released from
(a) 1 ´ 10–5 V (b) 1 ´ 10–7 V (2012) rest at the point (2a, 0) on the x - axis. The charge Q will
(c) 1 ´ 10 V–9 (d) 1 ´ 10–10 V (1984- 2 Marks)
34. Consider a thin spherical shell of radius R with centre at the (a) execute simple harmonic motion about the origin
origin, carrying uniform positive surface charge (b) move to the origin remain at rest
density. The variation of the magnitude of the electric field
r (c) move to infinity
E(r ) and the electric potential V(r) with the distance r from (d) execute oscillatory but not simple harmonic motion
the centre, is best represented by which graph? 2. A parallel plate air capacitor is connected to a battery. The
(2012) quantities charge, voltage, electric field and energy
|()|
®
Er Vr () associated with this capacitor are given by Q0, V0, E0 and
U0 respectively. A dielectric slab is now introduced to fill
the space between the plates with battery still in connection.
The corresponding quantities now given by Q, V, E and U
(a) are related to the previous one as (1985 - 2 Marks)

O r (a) Q > Q0 (b) V > V0


R
® (c) E > E0 (d) U > U 0
|E(r)| V(r) 3. A charge q is placed at the centre of the line joining two
equal charges Q. The system of the three charges will be in
equilibrium if q is equal to : (1987 - 2 Marks)
(b)
Q Q
(a) - (b) -
O r 2 4

|()|
®
Er Vr() (c) +
Q
4
(d) +
Q
2
4. A parallel plate capacitor is charged and the charging battery
is then disconnected. If the plates of the capacitor are moved
(c)
farther apart by means of insulating handles :
O r (1987 - 2 Marks)
R
(a) the charge on the capacitor increases.
|()|
®
Er Vr () (b) the voltage across the plates increases.
(c) the capacitance increases.
(d) the electrostatic energy stored in the capacitor increases
(d) 5. A solid conducting sphere having a charge Q is surrounded
by an uncharged concentric conducting hollow spherical
O R r shell. Let the potential difference between the surface of the
35. In the given circuit, a charge of +80 mC is given to the upper solid sphere and that of the outer surface of the hollow shell
plate of the 4 mF capacitor. Then in the steady state, the charge be V. If the shell is now given a charge of – 3Q, the new
on the upper plate of the 3 mF capacitor is (2012) potential difference between the same two surfaces is :
(1989 - 2 Marks)
(a) V (b) 2V
(c) 4V (d) – 2V
6. Seven capacitors each of capacitance 2mF are to be
connected in a configuration to obtain an effective
æ 10 ö
capacitance of ç ÷ mF . Which of the combination (s) shown
è 11ø
(a) + 32 mC (b) + 40 mC in figure will achieve the desired result ? (1990 - 2 Marks)
(c) + 48 mC (d) + 80 mC
ELECTROSTATICS P-279
11. A dielectric slab of thickness d is inserted in a parallel plate
capacitor whose negative plate is at x = 0 and positive plate
(a) (b) is at x = 3d. The slab is equidistant from the plates. The
capacitor is given some charge. As one goes from 0 to 3d,
(a) the magnitude of the electric field remains the same.
(b) the direction of the electric field remains the same.
(c) the electric potential increases continuously.
(c) (d) (d) the electric potential increases at first, then decreases
and again increases. (1998S - 2 Marks)
7. A parallel plate capacitor of plate area A and plate separation 12. A charge +q is fixed at each of the points x = x0, x = 3x0,
d is charged to potential difference V and then the battery is x = 5x0,..... x = ¥ on the x axis, and a charge –q is fixed at
disconnected. A slab of dielectric constant K is then inserted each of the points x = 2x0, x = 4x0, x = 6x0,.... x = ¥. Here x0
between the plates of the capacitor so as to fill the space is a positive constant. Take the electric potential at a point
between the plates. If Q, E and W denote respectively, the due to a charge Q at a distance r from it to be Q/(4pe0r).
magnitude of charge on each plate, the electric field between Then, the potential at the origin due to the above system of
the plates (after the slab is inserted), and work done on the charges is (1998S - 2 Marks)
system, in question, in the process of inserting the slab, q
then (1991 - 2 Marks) (a) 0 (b) 8pe 0 x0 ln 2
e 0 AV e 0 KAV
(a) Q = (b) Q = q ln2
d d (c) ¥ (d) 4pe0 x0
V e AV 2 é 1ù 13. A positively charged thin metal ring of radius R is fixed in
(c) E = (d) W = 0 ê1- ú
Kd 2d ë K û the xy plane with its centre at the origin O. A negatively
8. Two identical thin rings, each of radius R metres, are coaxially charged particle P is released from rest at the point (0, 0, z0)
where z0 > 0. Then the motion of P is (1998S - 2 Marks)
placed a distance R metres apart. If Q1 coulomb, and Q2
(a) periodic, for all values of z0 satisfying 0 < z0 < ¥
coulomb, are respectively the charges uniformly spread on (b) simple harmonic, for all values of z0 satisfying 0 < z0 £ R
the two rings, the work done in moving a charge q from the (c) approximately simple harmonic, provided z0 < < R
centre of one ring to that of the other is (1992 - 2 Marks) (d) such that P crosses O and continues to move along
q(Q1 - Q2 ) ( 2 -1) the negative z axis towards z = - ¥
(a) zero (b) 14. A non-conducting solid sphere of radius R is uniformly
(4 2pe 0 R )
charged. The magnitude of the electric field due to the sphere
at a distance r from its centre (1998S - 2 Marks)
q 2(Q1 + Q2 ) q (Q1 + Q2 ) ( 2 + 1)
(c) (d) (a) increases as r increases, for r < R.
(4pe 0 R ) (4 2pe 0 R ) (b) decreases as r increases, for 0 < r < ¥.
ur (c) decreases as r increases, for R < r < ¥.
9. The magnitude of electric field E in the annular region of (d) is discontinuous at r = R.
a charged cylindrical capacitor. (1996 - 2 Marks) 15. An ellipsoidal cavity is carved within a perfect conductor. A
(a) is same throughout positive charge q is placed at the centre of the cavity. The
(b) is higher near the outer cylinder than near the inner points A and B are on the cavity surface as shown in the
cylinder figure. Then (1999S - 3 Marks)
(c) varies as 1/r, where r is the distance from axis
(d) varies as 1/r2 where r is the distance from the axis.
10. A metallic solid sphere is placed in a uniform electric fied. A
The lines of force follow the path(s) shown in Figure as
(1996 - 2 Marks) B
q

(a) electric field near A in the cavity = electric field near B


in the cavity
(b) charge density at A = charge density at B
(c) potential at A = potential at B
(a) 1 (b) 2 (d) total electric field flux through the surface of the cavity
(c) 3 (d) 4 is q/e0
P-280 Topic-wise Solved Papers - PHYSICS
16. A spherical symmetric charge system is centered at origin. (c) If the electric field between two point charges is zero
Given, Electric potential (2006S - 5 Marks) somewhere, then the sign of the two charges is the
Q Q same.
V= (r £ R0 ), V = (r > R0 ) (d) The work done by the external force in moving a
4pe0 R0 4pe0 r
unit positive charge from point A at potential VA
V to point B at potential VB is (VB - VA).
21. A cubical region of side a has its centre at the origin.
It encloses three fixed point charges, -q at (0, -a/4, 0), +3q
at (0, 0, 0) and -q at (0, +a/4, 0). Choose the correct options(s)
(2012)
z
a
r
R0
–q
(a) Within r = 2R0 total enclosed net charge is Q
(b) Electric field is discontinued at r = R0
q 3
(c) Charge is only present at r = R0 x
(d) Electrostatic energy is zero for r < R0
17. Under the influence of the Coulomb field of charge + Q, a (a) The net electric flux crossing the plane x = +a/2
charge –q is moving around it in an elliptical orbit. Find out is equal to the net electric flux crossing the plane
the correct statement(s). (2009) x = -a/2
(a) The angular momentum of the charge – q is constant (b) The net electric flux crossing the plane y = +a/2 is more
(b) The linear momentum of the charge – q is constant than the net electric flux crossing the plane
(c) The angular velocity of the charge –q is constant y = -a/2.
(d) The linear speed of the charge –q is constant q
18. A few electric field lines for a system of two charges Q1 and (c) The net electric flux crossing the entire region is e
0
Q2 fixed at two different points on the x-axis are shown in (d) The net electric flux crossing the plane z = +a/2 is equal
the figure. These lines suggest that (2010) to the net electric flux crossing the plane
x = +a/2.
22. Six point charges are kept at the vertices of a regular hexagon
of side L and centre O, as shown in the figure. Given that
Q Q
1 q
K= , which of the following statement(s) is (are)
4pe 0 L2
(a) |Q1| > |Q2| correct?
(b) |Q1| < |Q2| L E–q
(c) at a finite distance to the left of Q1 the electric field is +q F
zero
P
(d) at a finite distance to the right of Q2 the electric field is
zero
19. A spherical metal shell A of radius RA and a solid metal
sphere B of radius RB(<RA) are kept far apart and each is A S T D
given charge ‘+Q’. Now they are connected by a thin metal +2q O
–2q
wire. Then (2011)
(a) E Ainside = 0 (b) QA > QB
R
s A RB on surface on surface
(c) = (d) EA < EB B+ q
s B RA – qC
(2012)
20. Which of the following statement(s) is/are correct? (2011)
(a) If the electric field due to a point charge varies as r –2.5 (a) The electric field at O is 6K along OD
instead of r –2, then the Gauss law will still be valid. (b) The potential at O is zero
(b) The Gauss law can be used to calculate the field (c) The potential at all points on the line PR is same
distribution around an electric dipole. (d) The potential at all points on the line ST is same
ELECTROSTATICS P-281
23. Two non-conducting solid spheres of radii R and 2R, having (i) If Q = – q, will the charges at the corners move towards
uniform volume charge densities r1 and r2 respectively, touch centre or fly away from it.
each other. The net electric field at a distance 2R from the (ii) For what value of Q will the charges remain stationary?
centre of the smaller sphere, along the line joining the centres In this situation how much work is done in removing
r1 the charges to infinity?
of the spheres, is zero. The ratio can be 2. A rigid insulated wire frame, in the form of right triangle
r2
(JEE Adv. 2013) ABC is set in a vertical plane. Two beads of equal masses m
each carrying charges q1 and q2 are connected by a chord
32 of length l and can slide without friction on the wires.
(a) – 4 (b) -
25 Considering the case when the beads are stationary,
32 determine : (1978)
(c) (d) 4
25 A
24. In the circuit shown in the figure, there are two parallel plate
capacitors each of capacitance C. The switch S1 is pressed a
first to fully charge the capacitor C1 and then released. The
switch S2 is then pressed to charge the capacitor C2. After
some time, S2 is released and then S3 is pressed. After some 30° 60°
time (JEE Adv. 2013) B C

(i) the angle a,


S1 S2 S3
(ii) the tension in the chord, and
(iii) the normal reactions on the beads.
If the chord is now cut, what are the values of the charges
C1 C2 C3 for which the beads continue to remain stationary?
2V0 3. A pendulum bob of mass 80 mg and carrying a charge of
2×10–8 coulombs is at rest in a horizontal uniform electric
field of 20,000 volt/metre. Find the tension in the thread of
the pendulum and the angle it makes with the vertical.
(1979)
(a) The charge on the upper plate of C1 is 2CV0 4. A charged particle is free to move in an electric field. Will it
(b) The charge on the upper plate of C1 is CV0 always move along an electric line of force? (1979)
(c) The charge on the upper plate of C2 is 0 5. A charge ‘Q’ is distributed over two concentric hollow
(d) The charge on the upper plate of C2 is –CV0 spheres of radii ‘r’ and ‘R’ (>r) such that the surface densities
25. Two non-conducting spheres of radii R1 and R2 and carrying are equal. Find the potential at the common centre.
uniform volume charge densities +r and –r, respectively, (1981- 3 Marks)
are placed such that they partially overlap, as shown in the 6. A thin fixed ring of radius 1 metre has a positive charge 1 × 10–5
figure. At all points in the overlapping region coulomb uniformly distributed over it. A particle of mass
0. 9 gm and having a negative charge of 1 × 10–6 coulomb is
placed on the axis at a distance of 1 cm from the centre of the
r r ring. show that the motion of the negatively charged particle
is approaximately simple harmonic. Calculate the time period
of oscillations. (1982 - 5marks)
R1 R2 7. The figure shows two identical parallel plate capacitors
connected to a battery with the switch S closed . The switch
is now opened and the free space between the plates of the
capacitors is filled with a dielectric of dielectric constant (or
(JEE Adv. 2013) relative permittivity) 3. Find the ratio of the total electrostatic
(a) The electrostatic field is zero
energy stored in both capacitors before and after the
(b) The electrostatic potential is constant
introduction of the dielectric. (1983 - 6 Marks)
(c) The electrostatic field is constant in magnitude
(d) The electrostatic field has same direction S

SUBJECTIVE PROBLEMS :
1. Three charges each of value q, are placed at the corners of V A C B C
an equilateral triangle. A fourth charge Q is placed at the
centre of the triangle. (1978)
P-282 Topic-wise Solved Papers - PHYSICS
8. Two fixed, equal, positive charges, each of magnitude (c) If a particle of charge + q starts form rest at the centre
5 × 10–5 coul are located at points A and B separated by a of the circle, show by a short quantative argument that
distance of 6 m. An equal and opposite charge moves the particle eventually crosses the circle. Find its speed
towards them along the line COD, the perpendicular when it does so.
bisector of the line AB. (1985 - 6 Marks) 14. (a) A charge of Q coulomb is uniformly distributed over a
spherical volume of radius R metres. Obtain an
A +q expression for the energy of the system.
(b) What will be the corresponding expression for the
O –q energy needed to completely disassemble the planet
D C earth against the gravitational pull amongst its
constituent particles ?
B +q Assume the earth to be a sphere of uniform
mass density. Calculate this energy, given the product
The moving charge, when it reaches the point C at a of the mass and the radius of the earth to be
distance of 4 m from O, has a kinetic energy of 4 joules. 2.5 × 1031 kg. m.
Calculate the distance of the farthest point D which the (c) If the same charge of Q coulomb as in part (a) above is
negative charge will reach before returning towards C.
given to a spherical conductor of the same radius R,
9. Three point charges q, 2q and 8q are to be placed on a 9 cm
what will be energy of the system ?(1992 - 10 Marks)
long straight line. Find the positions where the charges
15. Two parallel plate capacitors A and B have the same
should be placed such that the potential energy of this
separation d = 8.85 × 10–4 m between the plates. The plate
system is minimum. In this situation, what is the electric
field at the position of the charge q due to the other two area of A and B are 0.04 m2 and 0.02m2 respectively. A slab of
charges? (1987 - 7 Marks) dielectric constant (relative permittivity) K = 9 has
10. Three particles, each of mass 1 gm and carrying a charge q, dimensions such that it can exactly fill the space between
are suspended from a common point by insulated massless the plates of capacitor B. (1993 - 2 + 3 + 2 Marks)
strings, each 100 cm long. If the particles are in equilibrium
and are located at the corners of an equilateral triangle of A B A B
side length 3 cm, calculate the charge q on each particle.
(Take g = 10 m/s2). (1988 - 5 Marks)
11. A point particle of mass M is attached to one end of a
massless rigid nonconducting rod of length L. Another point 110V
particle of the same mass is attached to the other end of the (a) (b) (c)
rod. The two particles carry charges +q and – q respectively.
This arrangement is held in a region of a uniform electric (i) The dielectric slab is placed inside A as shown in figure
field E such that the rod makes a small angle q (say of (a). A is then charged to a potential difference of 110V.
Calculate the capacitance of A and the energy stored in
about 5 degree) with the field direction, fig. Find an expression
it.
for the minimum time needed for the rod to become parallel
(ii) The battery is disconnected and then the dielectric
to the field after it is set free. (1989 - 8mark)
slab is moved from A. Find the work done by the external
agency in removing the slab from A.
(iii) The same dielectric slab is now placed inside B, filling
Q+q it completely. The two capacitors A and B are then
q
E
Q-q connected as shown in figure (c). Calculate the energy
stored in the system.
16. A circular ring of radius R with uniform positive charge
12. Three concentric spherical metallic shells A, B and C of radii
density l per unit length is located in the y-z plane with its
a, b and c (a < b <c) have surface charge densities s , – s
centre at the origin O. A particle of mass m and positive
and s respectively.. (1990 -7 Marks)
(i) Find the potential of the three shells A, B and C. charge q is projected from the point P ( R 3, 0, 0) on the
(ii) If the shells A and C are at the same potential, obtain positive x-axis directly towards O, with an initial speed v.
the relation between the radii a, b and c. Find the smallest (non-zero) value of the speed v such that
13. Two fixed charges – 2Q and Q are located at the points with the particle does not return to P. (1993-4 Marks)
coordinates (–3a, 0) and (+ 3a, 0) respectively in the x-y
17. Two square metal plates of side 1 m are kept 0.01 m apart like
plane. (1991 - 4 + 2 + 2 Marks)
a parallel plate capacitor in air in such a way that one of their
(a) Show that all points in the x-y plane where the electric
edges is perpendicular to an oil surface in a tank filled with
potential due to the two charges is zero, lie on a circle.
an insulating oil. The plates are connected to a battery of
Find its radius and the location of its centre.
emf 500 V. The plates are then lowered vertically into the oil
(b) Give the expression V (x) at a general point on the
x - axis and sketch the function V (x) on the whole at a speed of 0.001 ms–1. Calculate the current drawn from
x-axis. the battery during the process. (Dielectric constant of
oil = 11, e0 = 8.85 × 10–12C2N–1m–1) (1994 - 6 Marks)
ELECTROSTATICS P-283
18. The capacitance of a parallel plate capacitor with plate area dropped, along the axis of the disc, from a height H with
A and separation d is C. The space between the plates is zero initial velocity. The particle has q m = 4 Î0 g / s
filled with two wedges of dielectric constants K1 and K2,
respectively. Find the capacitance of the resulting capacitor. (1999 - 10 Marks)
(1996 - 2 Marks) (a) Find the value of H if the particle just reaches the disc.
(b) Sketch the potential energy of the particle as a function
of its height and find its equilibrium position.
23. Four point charges +8mC, –1mC, –1mC, and +8mC are fixed
27 3 3 27
19. Two isolated metallic solid spheres of radii R and 2R are at the points - m,- m, + m and + m
charged such that both of these have same charge density 2 2 2 2
s. The spheres are located far away from each other, and respectively on the y-axis. A particle of mass 6 × 10–4 kg and
connected by a thin conducting wire. Find the new charge charge +0.1 mC moves along the -x direction. Its speed at
density on the bigger sphere. (1996 - 3 Marks) x = + ¥ is V0. Find the least value of V0 for which the particle
20. Two capacitors A and B with capacities 3 mF and 2 mF are will cross the origin. Find also the kinetic energy of the
charged to a potential difference of 100 V and 180 V particle at the origin. Assume that space is gravity free.
respectively. The plates of the capacitors are connected as 1
shown in the figure with one wire from each capacitor free. Given = 9 ´ 109 Nm 2 / C 2 . (2000 - 10 Marks)
4pe 0
The upper plate of A is positive and that of B is negative. An
uncharged 2 mF capacitor C with lead wires falls on the free 24. Charges +q and –q are located at the corners of a cube of
ends to complete the circuit. Calculate (1997 - 5 Marks) side as show in the figure. Find the work done to separate
the charges to infinite distance. (2003 - 2 Marks)
–q +q
–q
+q

+q –q
–q +q
(i) the final charge on the three capacitors. and 25. A charge +Q is fixed at the origin of the co-ordinate system
uur
(ii) the amount of electrostatic energy stored in the system while a small electric dipole of dipole moment p pointing
before and after the completion of the circuit. away from the charge along the x-axis is set free from a point
21. A conducting sphere S1 of radius r is attached to an insulating far away from the origin. (2003 - 4 Marks)
handle. Another conducting sphere S2 of radius R is
(a) Calculate the K.E. of the dipole when it reaches to a
mounted on an insulating stand. S2 is initially uncharged.
point (d, 0).
S1 is given a charge Q, brought into contact with S2, and
removed. S1 is recharged such that the charge on it is again (b) Calculate the force on the charge +Q at this moment.
Q; and it is again brought into contact with S2 and removed. 26. Two uniformly charged large plane sheets S1 and S2 having
This procedure is repeated n times. (1998 - 8 Marks) charge densities s1 and s2 (s1 > s2) are placed at a distance
(a) Find the electrostatic energy of S2 after n such contacts d parallel to each other. A charge q0 is moved along a line of
with S1. length a(a < d) at an angle 45º with the normal to S1. Calculate
the work done by the electric field (2004)
(b) What is the limiting value of this energy as n ® ¥ ?
27. A conducting liquid bubble of radius a and thickness
22. A non-conducting disc of radius a and uniform positive t (t << a) is charged to potential V. If the bubble collapses to
surface charge density s is placed on the ground, with its a droplet, find the potential on the droplet.
axis vertical. A particle of mass m and positive charge q is (2005 - 2 Marks)
MATCH THE FOLLOWING :
MUTLIPLE CHOICE QUESTIONS WITH ONE CORRECT
Each question contains statements given in two columns, which have to be matched. The statements in Column-I are labelled A,
B, C and D, while the statements in Column-II are labelled p, q, r, s and t. Any given statement in Column-I can have correct
matching with ONE OR MORE statement(s) in Column-II. The appropriate bubbles corresponding to the answers to these
questions have to be darkened as illustrated in the following example :
If the correct matches are A-p, s and t; B-q and r; C-p and q; and D-s then the correct darkening of bubbles will look like the given.
p q r s t
A p q r s t
B p q r s t
C p q r s t
D p q r s t
P-284 Topic-wise Solved Papers - PHYSICS
1. Six point charges, each of the same magnitude q, are arranged in different manners as shown in Column-II. In each case, a point
M and line PQ passing through M are shown. Let E be the electric field and V be the electric potential at M (potential at infinity
is zero) due to the given charge distribution when it is at rest. Now, the whole system is set into rotation with a constant angular
velocity about the line PQ. Let B be the magnetic field at M and m be the magnetic moment of the system in this condition.
Assume each rotating charge to the equivalent to a steady current. (2009)

Column-I Column-II
+ – Q
Charges are at the corners of a regular hexagon. M is at the centre
(A) E = 0 (p)
– of the hexagon. PQ is perpendicular to the plane of the hexagon.
+
(B) V¹0 M

P+ –
(C) B=0
P
Charges are on a line perpendicular to PQ at equal intervals. M is
(D) m ¹ 0
– + – the mid-point between the two innermost charges.
(q) + – +
M
Q
Charges are placed on two coplanar insulating rings at equal
Q intervals. M is the common centre of the rings. PQ is perpendicular
+ – + to the plane of the rings.
(r) – M –

P +
Charges are placed at the corners of a rectangle of sides a and 2a
– + – and at the mid points of the longer sides. M is at the centre of the
rectangle. PQ is parallel to the longer sides.
M
(s) P Q
– + –

P
+ –
Charges are placed on two coplanar, identical insulating rings at
equal intervals. M is the mid-point between the centres of the
) + + M– –
rings. PQ is perpendicular to the line joining the centres and
Q coplanar to the rings.

1. The electric field at r = R is


COMPREHENSION BASED Q UESTIONS :
(a) independent of a
The nuclear charge (Ze) is non-uniformly distributed within a (b) directly proportional to a
nucleus of radius R. The charge density r (r) [charge per unit (c) directly proportional to a2
volume] is dependent only on the radial distance r from the centre (d) inversely proportional to a
of the nucleus as shown in figure The electric field is only along 2. For a = 0, the value of d (maximum value of r as shown in the
the radial direction. (2008) figure) is –
3Ze 3Ze
(a) 3 (b)
4pR pR 3
4Ze Ze
d (c) 3 (d)
3pR 3pR 3
3. The electric field within the nucleus is generally observed
to be linearly dependent on r. This implies.
r (a) a = 0 (b) a = R/2
O a R (c) a = R (d) a = 2R/3
ELECTROSTATICS P-285

ASSERTION & REASON TYPE QUESTIONS : 2. Four point charges, each of +q, are rigidly fixed at the four
corners of a square planar soap film of side ‘a’. The surface
1. STATEMENT-1 : For practical purposes, the earth is used tension of the soap film is g. The system of charges and
as a reference at zero potential in electrical circuits.
1/ N
and é q2 ù
STATEMENT-2 : The electrical potential of a sphere of planar film are in equilibrium, and a = k ê ú , where ‘k’
ëê g ûú
radius R with charge Q uniformly distributed on the surface
is a constant. Then N is (2011)
Q
is given by 4pe R . (2008) 3. An infinitely long solid cylinder of radius R has a uniform
0
volume charge density r. It has a spherical cavity of radius
(a) STATEMENT-1 is True, STATEMENT-2 is True; R/2 with its centre on the axis of the cylinder, as shown in
STATEMENT-2 is a correct explanation for the figure. The magnitude of the electric field at the point P,
STATEMENT-1 which is at a distance 2R from the axis of the cylinder, is
(b) STATEMENT-1 is True, STATEMENT-2 is True;
STATEMENT-2 is NOT a correct explanation for 23rR
given by the expression . The value of k is
STATEMENT-1 16K e 0
(c) STATEMENT -1 is True, STATEMENT-2 is False
(2012)
(d) STATEMENT -1 is False, STATEMENT-2 is True

INTEGERVALUECORRECTTYPE:
1. A solid sphere of radius R has a charge Q distributed in its
volume with a charge density r = kra, where k and a are
constants and r is the distance from its centre.
R 1
If the electric field at r = is times that at r = R, find the
2 8
value of a. (2009)

1. On moving a charge of 20 coulomb by 2 cm, 2 J of work is (a) q /4 p Î0 L (b) zero


done, then the potential difference between the points is
(a) 0.1 V (b) 8 V [2002] (c) q/2 p Î0 L (d) q/3 p Î0 L
(c) 2 V (d) 0.5 V. 4. If a charge q is placed at the centre of the line joining two
2. If there are n capacitors in parallel connected to V volt source, equal charges Q such that the system is in equilibrium then
then the energy stored is equal to [2002] the value of q is [2002]
1 (a) Q/2 (b) -Q/2
(a) CV (b) nCV2 (c) Q/4 (d) -Q/4
2 5. Capacitance (in F) of a spherical conductor with radius 1 m
1 is [2002]
(c) CV2 (d) CV2
2n (a) 1.1´ 10 -10 (b) 10 -6
3. A charged particle q is placed at the centre O of cube of
length L (A B C D E F G H). Another same charge q is placed (c) 9 ´ 10 -9 (d) 10 -3
at a distance L from O. Then the electric flux through ABCD 6. If the electric flux entering and leaving an enclosed surface
is [2002] respectively is f1 and f 2 , the electric charge inside the surface
H will be [2003]
E D
(a) (f2 - f1 )e o (b) (f1 + f2 ) / e o
A
O
q q (c) (f2 - f1) / e o (d) (f1 + f2 )e o
G C 7. A sheet of aluminium foil of negligible thickness is
F B introduced between the plates of a capacitor. The
L capacitance of the capacitor [2003]
(a) decreases (b) remains unchanged
(c) becomes infinite (d) increases
P-286 Topic-wise Solved Papers - PHYSICS
8. A thin spherical conducting shell of radius R has a charge q. 14. Four charges equal to -Q are placed at the four corners of a
Another charge Q is placed at the centre of the shell. The square and a charge q is at its centre. If the system is in
equilibrium the value of q is [2004]
R
electrostatic potential at a point P a distance from the Q Q
2 (a) - (1 + 2 2) (b) (1 + 2 2)
centre of the shell is [2003] 2 4
2Q 2Q 2q Q Q
(a) (b) 4pe R - 4pe R (c) - (1 + 2 2) (d) (1 + 2 2)
4pe o R o o
4 2
15. A charged oil drop is suspended in a uniform field of 3×104
2Q q (q + Q)2 v/m so that it neither falls nor rises. The charge on the drop
(c) + (d) 4pe R
4pe o R 4pe o R o
will be (Take the mass of the charge = 9.9×10–15 kg and
g = 10 m/s2) [2004]
-18 (a) 1.6×10–18 C (b) 3.2×10–18 C
9. The work done in placing a charge of 8 ´ 10 coulomb on
a condenser of capacity 100 micro-farad is [2003] (c) 3.3×10–18 C (d) 4.8×10–18 C
16. Two point charges + 8q and – 2q are located at x = 0 and
(a) 16 ´ 10 -32 joule (b) 3.1´10 -26 joule x = L respectively. The location of a point on the x axis at
which the net electric field due to these two point charges is
(c) 4 ´10-10 joule (d) 32 ´ 10-32 joule
zero is
10. Three charges –q1 , +q2 and –q3 are place as shown in the
figure. The x - component of the force on –q1 is proportional L
(a) (b) 2 L [2005]
to [2003] 4
(c) 4 L (d) 8 L
Y 17. Two thin wire rings each having a radius R are placed at a
-q3 distance d apart with their axes coinciding. The charges on
the two rings are +q and -q. The potential difference
between the centres of the two rings is [2005]
a é1 ù
b q 1
(a) ê - ú
2p Î0 ëê R R 2 + d 2 úû
-q1 +q2 X
qR
q2 q3 q2 q3 (b)
(a) - 2
cos q (b) + sin q 4p Î0 d 2
b2 a b2 a 2

q é1 1 ù
q2 q3 q2 q3 (c) ê - ú
(c) + cos q (d) - sin q 4p Î0
b2 a2 b2 a2 ëê R R 2 + d 2 úû
(d) zero
11. The length of a given cylindrical wire is increased by 100%. 18. A parallel plate capacitor is made by stacking n equally
Due to the consequent decrease in diameter the change in spaced plates connected alternatively. If the capacitance
the resistance of the wire will be [2003] between any two adjacent plates is ‘C’ then the resultant
(a) 200% (b) 100% capacitance is [2005]
(c) 50% (d) 300% (a) (n + 1) C (b) (n – 1) C
12. Two spherical conductors B and C having equal radii and (c) nC (d) C
carrying equal charges on them repel each other with a force 19. A charged ball B hangs from a silk thread S, which makes an
F when kept apart at some distance. A third spherical angle q with a large charged conducting sheet P, as shown
conductor having same radius as that B but uncharged is
in the figure. The surface charge density s of the sheet is
brought in contact with B, then brought in contact with C
proportional to [2005]
and finally removed away from both. The new force of
repulsion between B and C is [2004]
(a) F/8 (b) 3 F/4
(c) F/4 (d) 3 F/8 P
q
13. A charge particle ‘q’ is shot towards another charged particle S
‘Q’ which is fixed, with a speed ‘v’. It approaches ‘Q’ upto
a closest distance r and then returns. If q were given a
speed of ‘2v’ the closest distances of approach would be
[2004] B
(a) r/2 (b) 2 r (a) cot q (b) cos q
(c) r (d) r/4 (c) tan q (d) sin q
ELECTROSTATICS P-287
20. A fully charged capacitor has a capacitance ‘C’. It is 25. Charges are placed on the vertices of a square as shown.
discharged through a small coil of resistance wire embed- ur
Let E be the electric field and V the potential at the centre.
ded in a thermally insulated block of specific heat capacity
If the charges on A and B are interchanged with those on D
‘s’ and mass ‘m’. If the temperature of the block is raised by
and C respectively, then [2007]
‘ DT ’, the potential difference ‘V’ across the capacitance is
q q
mC DT 2mC DT
(a) (b) [2005] A B
s s

2ms DT ms DT
(c) (d)
C C
D C
21. An electric dipole is placed at an angle of 30° to a non- -q -q
uniform electric field. The dipole will experience
[2006] ur
(a) E changes, V remains unchanged
(a) a translational force only in the direction of the field ur
(b) a translational force only in a direction normal to the (b) E remains unchanged, V changes
ur
direction of the field (c) both E and V change
(c) a torque as well as a translational force ur
(d) a torque only (d) E and V remain unchanged
22. Two insulating plates are both uniformly charged in such a 26. The potential at a point x (measured in m m) due to some
way that the potential difference between them is charges situated on the x-axis is given by
V2 – V1 = 20 V. (i.e., plate 2 is at a higher potential). The V(x) = 20/(x2 – 4) volt
plates are separated by d = 0.1 m and can be treated as The electric field E at x = 4 m m is given by [2007]
infinitely large. An electron is released from rest on the inner
(a) (10/9) volt/ m m and in the +ve x direction
surface of plate 1. What is its speed when it hits plate 2?
(e = 1.6 × 10–19 C, me = 9.11 × 10–31 kg) [2006] (b) (5/3) volt/ m m and in the –ve x direction
(c) (5/3) volt/ m m and in the +ve x direction
(d) (10/9) volt/ m m and in the –ve x direction
Y
27. A parallel plate condenser with a dielectric of dielectric
0.1 m constant K between the plates has a capacity C and is
charged to a potential V volt. The dielectric slab is slowly
X removed from between the plates and then reinserted. The
net work done by the system in this process is [2007]
1
1 2 (a) zero (b) ( K - 1) CV 2
2
(a) 2.65 × 106 m/s (b) 7.02 × 1012 m/s
(c) CV 2 ( K - 1) (d) ( K - 1) CV 2
6
(c) 1.87 × 10 m/s (d) 32 × 10–19 m/s
K
23. Two spherical conductors A and B of radii 1 mm and 2 mm
are separated by a distance of 5 cm and are uniformly 28. If gE and gM are the accelerations due to gravity on the
charged. If the spheres are connected by a conducting wire surfaces of the earth and the moon respectively and if
then in equilibrium condition, the ratio of the magnitude of Millikan’s oil drop experiment could be performed on the
the electric fields at the surfaces of spheres A and B is two surfaces, one will find the ratio [2007]
(a) 4 : 1 (b) 1 : 2 [2006] electronic charge on the moon
(c) 2 : 1 (d) 1 : 4 to be
electronic charge on the earth
24. An electric charge 10 –3 C is placed at the origin (0, 0) of
X – Y co-ordinate system. Two points A and B are situated at (a) gM / g E (b) 1

2, 2 and (2, 0) respectively. The potential difference (c) 0 (d) gE / gM

between the points A and B will be [2007] 29. A parallel plate capacitor with air between the plates has
capacitance of 9 pF. The separation between its plates is
(a) 4.5 volts (b) 9 volts
‘d’. The space between the plates is now filled with two
(c) Zero (d) 2 volt
dielectrics. One of the dielectrics has dielectric constant k1
P-288 Topic-wise Solved Papers - PHYSICS

d Q
= 3 and thickness while the other one has dielectric 34. Let P (r ) = r be the charge density distribution for a
3 pR 4
2d solid sphere of radius R and total charge Q. For a point ‘p’
constant k2 = 6 and thickness . Capacitance of the inside the sphere at distance r1 from the centre of the sphere,
3
the magnitude of electric field is : [2009]
capacitor is now [2008]
(a) 1.8 pF (b) 45 pF Q Qr12
(a) (b)
(c) 40.5 pF (d) 20.25 pF 4p Î0 r12 4p Î0 R 4
30. A thin spherical shell of radus R has charge Q spread
uniformly over its surface. Which of the following graphs
Qr12
most closely represents the electric field E(r) produced by (c) (d) 0
the shell in the range 0 £ r < ¥, where r is the distance from 3p Î0 R 4
the centre of the shell? [2008] 35. A thin semi-circular ring of radius r has a positive charge q
ur
E(r) distributed uniformly over it. The net field E at the centre O
E(r)
is [2010]
j
(a) r (b) r
O R O R

E(r) E(r) i
O
q ˆj q ˆj
(a) 2 2 (b) -
(c) r (d) r 4p e 0 r 4p e 0 r 2
2
O R O R
q ˆj q ˆj
(c) - (d)
31. Two points P and Q are maintained at the potentials of 2p e 0 r 2
2
2p e 0 r 2
2

10 V and – 4 V, respectively. The work done in moving 36. Let there be a spherically symmetric charge distribution with
100 electrons from P to Q is : [2009]
(a) 9.60 × 10–17J (b) –2.24 × 10–16 J æ5 rö
charge density varying as r(r ) = r0 çè - ÷ø upto r = R ,
(c) 2.24 × 10–16 J (d) –9.60× 10–17 J 4 R
32. A charge Q is placed at each of the opposite corners of a and r(r ) = 0 for r > R , where r is the distance from the
square. A charge q is placed at each of the other two corners.
origin. The electric field at a distance r(r < R) from the origin
If the net electrical force on Q is zero, then Q/q equals:
is given by [2010]
[2009]
(a) –1 (b) 1 r0 r æ 5 r ö 4pr0 r æ 5 r ö
(a) ç - ÷ (b) ç - ÷
1 4e 0 è 3 R ø 3e 0 è 3 R ø
(c) - (d) -2 2
2
4r0 r æ 5 r ö r0 r æ 5 r ö
33. This question contains Statement-1 and Statement-2. Of (c) ç - ÷ (d) 3ε çè 4 - R ÷ø
4ε 0 è 4 R ø 0
the four choices given after the statements, choose the one
that best describes the two statements. [2009] 37. Two identical charged spheres suspended from a common
Statement-1 : For a charged particle moving from point P to point by two massless strings of length l are initially a
point Q, the net work done by an electrostatic field on the distance d(d << l) apart because of their mutual repulsion.
particle is independent of the path connecting point P to The charge begins to leak from both the spheres at a constant
point Q. rate. As a result charges approach each other with a velocity
Statement-2 : The net work done by a conservative force v. Then as a function of distance x between them, [2011]
on an object moving along a closed loop is zero. (a) v µ x–1 (b) v µ x½
(a) Statement-1 is true, Statement-2 is true; Statement-2 is (c) v µ x (d) v µ x–½
the correct explanation of Statement-1. 38. The electrostatic potential inside a charged spherical ball is
(b) Statement-1 is true, Statement-2 is true; Statement-2 is given by f = ar2 + b where r is the distance from the centre
not the correct explanation of Statement-1. and a, b are constants. Then the charge density inside the
(c) Statement-1 is false, Statement-2 is true. ball is : [2011]
(d) Statement-1 is true, Statement-2 is false. (a) –6ae 0 r (b) –24pae 0
(c) –6ae 0 (d) –24pae 0 r
ELECTROSTATICS P-289
39. Two positive charges of magnitude ‘q’ are placed, at the An insulating solid sphere of radius R has a uniformly
ends of a side (side 1) of a square of side ‘2a’. Two negative positive charge density r. As a result of this uniform charge
charges of the same magnitude are kept at the other corners. distribution there is a finite value of electric potential at the
Starting from rest, if a charge Q moves from the middle of centre of the sphere, at the surface of the sphere and also at
side 1 to the centre of square, its kinetic energy at the a point out side the sphere. The electric potential at infinite
centre of square is [2011RS] is zero.
Statement -1 : When a charge q is take from the centre of
1 2 qQ æ 1 ö
(a) zero (b) 1+ the surface of the sphere its potential energy changes by
4 pe 0 a çè ÷

qr
1 2 qQ æ 2 ö 1 2 qQ æ 1 ö 3e0 .
(c) 1- (d) 1-
4 pe 0 a çè ÷
5ø 4 pe 0 a çè ÷
5ø Statement -2 : The electric field at a distance r (r <R) from
40. In a uniformly charged sphere of total charge Q and radius rr
R, the electric field E is plotted as function of distance from the centre of the sphere is .
3e 0
the centre, The graph which would correspond to the above
will be: [2012] (a) Statement 1 is true, Statement 2 is true; Statement 2 is
not the correct explanation of statement 1.
(b) Statement 1 is true Statement 2 is false.
(c) Statement 1 is false Statement 2 is true.
E E (d) Statement 1 is true, Statement 2 is true, Statement 2 is
the correct explanation of Statement 1
(a) (b) 43. Two capacitors C1 and C2 are charged to 120 V and 200 V
respectively. It is found that connecting them together the
Rr Rr potential on each one can be made zero. Then
[JEE Main 2013]
(a) 5C1 = 3C2 (b) 3C1 = 5C2
E E (c) 3C1 + 5C2 = 0 (d) 9C1 = 4C2
44. Two charges, each equal to q, are kept at x = – a and x = a on
(c) (d)
q
the x-axis. A particle of mass m and charge q0 = is placed
Rr Rr 2
at the origin. If charge q0 is given a small displacement
41. This equation has statement 1 and statement 2 of the four (y <<a) along the y-axis, the net force acting on the particle
choices given the statements, choose the one that describes is proportional to [JEE Main 2013]
the two statements. (a) y (b) –y
Statement 1: Davisson-Germer experiment established the
wave nature of electrons. 1 1
(c) (d) –
Statement 2 : If electrons have wave nature, they can y y
interfere and show diffraction. 45. A charge Q is uniformly distributed over a long rod AB of
(a) Statement 1 is false, Statement 2 is true. length L as shown in the figure. The electric potential at the
(b) Statement 1 is true, Statement 2 is false point O lying at distance L from the end A is
(c) Statement 1 is true, Statement 2 is true, Statement 2 is [JEE Main 2013]
the correct explanation for statement 1
O A B
(d) Statement 1 is true, Statement 2 is true, Statement 2 is
L L
not the correct explanation of Statement 1
42. This questions has statement-1 and statement-2. Of the four Q 3Q
choices given after the statements, choose the one that (a) 8pe 0 L (b) 4pe L
0
best describe the two statements. [2012]
Q Q ln 2
(c) 4pe0 L ln 2 (d) 4pe L
0
P-290 Topic-wise Solved Papers - PHYSICS

Solutions & Explanations


Section-A : JEE Advanced/ IIT-JEE

e0 A 2e 0 A 1 Q2 3V
A 1. ´V ; ´V 2. B 3. 180°, ´ 2 4.
d d 4pe 0 4 L k+2

1 q´q
5. –qEa 6. –8 7. 4pe 0 L2
B 1. F 2. T 3. T 4. T 5. F 6. F
C 1. (b) 2. (b) 3. (b) 4. (b) 5. (a) 6. (d) 7. (d)
8. (b) 9. (b) 10. (c) 11. (a) 12. (b) 13. (b) 14. (c)
15. (c) 16. (c) 17. (d) 18. (c) 19. (a) 20. (d) 21. (b)
22. (c) 23. (c) 24. (a) 25. (b) 26. (a) 27. (a) 28. (d)
29. (c) 30. (d) 31. (c) 32. (a) 33. (c) 34. (d) 35. (c)
D 1. (d) 2. (a, d) 3. (b) 4. (b, d) 5. (a) 6. (a) 7. (a, c, d)
8. (b) 9. (c) 10. (d) 11. (b, c) 12. (d) 13. (a, c) 14. (a, c)
15. (c, d) 16. (a,b,d) 17. (a) 18. (a, d) 19. (a, b, c, d) 20. (c, d) 21. (a, c, d)
22. (a, b, c) 23. (d) 24. (b, d) 25. (c, d)

4 3q q2
E 1. (i) Move towards centre; (ii) Q = , 3(2 + 3) K 2
9 a

qq
2. (i) 60°; (ii) mg = ± k 12 2 ; (iii) N1 = 3 mg ; N2 = mg 3. 9 × 10–4 N, 27°2' 4. No
l

KQ( R + r ) 3
5. 6. 0.628 sec. 7. 8. 8.48 m 9. 0 10. 3.16 × 10–9 C
R2 + r 2 5

p ML s s æ a2 ö s æ a 2 - b 2 + c2 ö
11. 12. (i) Î (a - b + c) , Î ç b - b + c÷ , Î ç
c
÷ (ii) c = a + b
2 2qE 0 0è ø 0 è ø

é 1 2 ù 1 æ Qq ö
13. (a) 4a, (5a, 0) (b) KQ ê - (c) ç ÷
ë 3a - x 3a + x úû 4p Î0 è 2 ma ø

3Q 2 3GM 2 Q2
14. (a) (b) , 1.5 ´ 1032 J (c)
20p Î0 R 5R 8p Î0 R

lq
15. (i) 2 × 10–9 F, 1.21 × 10–5 J (ii) 4.84 × 10–5 J (iii) 1.1 × 10–5 J 16. 2 Î0 m

K1 K2 A Î0 K 5s
17. 4.425 × 10–9 Amp. 18. log 1 19.
d ( K1 - K 2 ) K2 6
20. (i) 90 × 10–6 C, 210 × 10–6 C, 150 × 10–6 C (ii) 4.74 × 10–2 J, 1.8 × 10–2 J
2
1 1 ìï QR é æ R ö n ù üï a
4a
21. (a) ´ ´í ê1 - ç ÷ ú ý (b) 22. (a) (b)
2 4p Î0 R ï r ê è R + r ø ú ï 3 3
î ë ûþ
ELECTROSTATICS P-291

1 q2 4
23. 3 m/s, 3 × 10–4 J 24. . (3 3 - 3 6 - 2)
4p Î0 a 6

1 pQ 1 2 pQ ˆ q0 (s1 - s 2 ) a æ aö
1/ 3
25. (a) 4p Î 2 (b) 4p Î i 26. 27. V ç ÷
3 2 2 Î0
0 d 0 d è 3t ø
28. a= 2
F 1. (A)-(p, r, s): (B)-(r, s); (C)-(p, q, t); (D)-(r, s)
G 1. (a) 2. (b) 3. (c)
H 1. (a)
I 1. 2 2. 3 3. 6
Section-B : JEE Main/ AIEEE
1. (a) 2. (b) 3. (b) 4. (d) 5. (a) 6. (a) 7. (b) 8. (c) 9. (d)
10. (b) 11. (d) 12. (d) 13. (d) 14. (b) 15. (c) 16. (b) 17. (a) 18. (b)
19. (c) 20. (c) 21. (c) 22. (a) 23. (c) 24. (c) 25. (a) 26. (a) 27. (a)
28. (b) 29. (c) 30. (a) 31. (c) 32. (d) 33. (a) 34. (b) 35. (c) 36. (a)
37. (d) 38. (c) 39. (d) 40. (c) 41. (c) 42. (c) 43. (b) 44. (a) 45. (d)

4. Initially charge on capacitance C = q1 = CV


FILL IN THE BLANKS : Charge on capacitance C = q2 = 2CV
1. On the plate 1 there is +ve charge
KC
e0 A
q = CV = ´V V'
d
-2 e 0 A V'
On the plate 4 the charge is –2q = ´V 2C
d
because plate 4 form two capacitors, one with 3 and the Finally charge on capacitance C = q1' = KCV '
other with 5.
Charge on capacitance 2C = q2' = 2CV '
Total charge will remain conserved
3V
\ CV + 2C V = KCV ' + 2CV ' or, V ' =
K+2
5. NOTE : Since electric field is conservative in nature, the
work done by the field along PQRS will be same as along
PMS
ur uuuur
Work done from P to M = F . PM
= F (PM) cos 90° = 0
2. It is greatest at point B because at B the equipotential
surfaces are closest.
3. There is no gravitational force acting. Only electrostatic Y
force of repulsion is acting which will take the two balls as E
far as possible. The angle between the two strings will be P (a,b,o)
180°. The tension in the string will be equal to the
electrostatic force of repulsion ) Q
0,0
O (0, S a M X
Q 180 Q
T T
R
1 Q´Q 1 Q2
T= ´ = ´
4 p e 0 (2 L)2 4 p e 0 4L2
P-292 Topic-wise Solved Papers - PHYSICS
ur uuur Mathematically for path A ® P ® B
Work done from M to S = F . MS
WAPB
= F (MS) cos 180° [Q F = qE] = VB – VA
= – qEa q
6. Electric potential V = 4x2 volts or, WAPB = q (VB – VA) ... (i)
The electric potential changes only along x-axis, For path A ® Q ® B
We know that
W AQB
-dV d (4 x 2 ) or,
q
= VB – VA
Ex = Þ Ex = - = - 8x
dx dx
or, WAQB = q (VB – VA) ... (ii)
The electric field at point (1, 0, 2) will be (here x = 1)
Ex = – 8 volt/m. Since the R.H.S. of eq. (i) and (ii) same
7. KEY CONCEPT : If we place a charge q at the sixth vertex of \ WAPB = WAQB
the regular hexagon then the net force on the charge – q ALTERNATE SOLUTION
placed at the centre of hexagon will be zero (due to symmetry). Electrostatic force is conservative in nature, therefore work
done is path independent.
2. The metallic sphere which gets negatively charged gains
electrons and hence its mass increases.
The metallic sphere which gets positively charged loses
electrons and hence its mass decreases.
3. When high energy X-ray beam falls, it will knock out
electrons from the small metal ball making it positively
charged. Therefore the ball will be deflected in the direction
of electric field.
The force on charge – q due to the charge q placed on the
sixth vertex balances the net force on charge – q due to the
other five charges placed at the five vertices.
The force on charge – q due to charge q placed at sixth
vertex will be
1 q´q 1 q2
F= =
4pe 0 L2 4p Î0 L2
when L is the distance of the centre of hexagon from any 4. The electric field produced between the parallel plate
vertex (directed form O to C). capacitor is uniform. The force acting on charged particle
The magnitude of force on the point charge of value placed in an electric field is given by F = qE.
1 q2 In the case of two protons, q and E are equal and hence
–q coulomb placed at the centre of the hexagon is force will be equal.
4pe 0 L2
5. KEY CONCEPT : Force on charge (– q) due to small charge
directed from O to C.
dq situated at length d l is given by
TRUE / FALSE : qdq
dF = k
1. Let us consider two points A and B in an electric field. Let 5 R2
the potential at A and B be VA and VB respectively.
NOTE : The potential difference between two point B and A Resolving this force into two parts dFcos q and dFsin q as
is the amount of work done in carrying unit positive charge shown in figure.
from A to B between the two point.

If we take another diametrically opposite length dl, the


charge on it being dq. Then the force on charge (– q) by this
small charge dq will be
qdq
dF = k
5 R2
ELECTROSTATICS P-293
Again resolving this force, we find dFsin q components of 1 2 1
U= CeqVnet = (3C )V 2
the two forces cancel out and dFcos q component adds up. 2 2
\ The total force
3 é qnet 3CV ù
2 pR 2 pR kqdq 2R =CV 2 êQ Vnet = = =Vú
F= ò0 dF cos q = ò
0 5R 2
´
5R
2 ë Cnet 3C û
ALTERNATE SOLUTION
Charge on length 2pR = Q C and 2C are in parallel to each other.
Q dl \ Resultant capacity = (2C + C)
\ Charge on length d l = = dq CR = 3C
2pR
Net potential = 2V – V
2 pR 2kq Q dl VR = V
\ F= ò0 5 5R 2
´
2 pR 1 1 3
\ Final energy = C R (VR )2 = (3C )(V )2 = CV 2
2 2 2
2kQq 2kQq
= 3
´ 2pR = 4. (b) Let the distance to be travelled be x. Let the strength of
5 5 ´ 2 pR 5 5 R2 uniform electric field be E. For electron
This is not an equation of simple harmonic motion. eE 1 eE 2
6. For a particle to move in circular motion, we need a centripetal u = 0, s = x, a = , t = t1, S = ut + ´ t1
me 2 me
force which is not available.
The statement is false. 1 eE 2
Þ x= ´ t1 ... (i)
MCQ's WITH ONE CORRECT ANSWER : 2 me
For proton
1. (b) The potential at the surface of a hollow or conducting
sphere is same as the potential at the centre of the eE 1 2
u = 0, s = x, a = , t = t2, S = ut + at
sphere and any point inside the sphere. mp 2
2. (b) The two charges form an electric dipole. If we take a
point M on the X-axis as shown in the figure, then the 1 eE 2
Þ x= ´ t2 ... (ii)
net electric field is in –X-direction. 2 mp
\ Option (a) is incorrect.
From (i) and (ii)
Y 1/ 2
t22 mp t2 é m p ù
= Þ =ê ú
N E t12 me t1 ë me û
Kq
+q M E –q E Vo = R V¥ = 0
X
(–d, 0) (d, 0) M l = 0 ur uur
p 5. (a) ò - E . d l = V0 - V¥
l=¥ q+
+ +

+ R
+
+
kq +
= -0 +
r + O
+
+
+
+ +
If we take a point N on Y-axis, we find net electric field 9 ´ 109 ´ 1.11 ´ 10 -10 + +
=
++
along +X direction. The same will be true for any point +
0.5
+
non uniform charge
on Y-axis. (b) is a correct option. » 2V distribution on the ring
NOTE : For any point on the equatorial line of a dipole,
the electric field is opposite to the direction of dipole 6. (d)
moment.
(b) W¥ – 0 = q (V¥–V0) = q (0 – 0) = 0
\ (c) is incorrect. The direction of dipole moment is
from –ve to + ve. Therefore (d) is incorrect.
3. (b) Energy stored,
C
_
+
V
Within the capacitor,
_ 2V Q1 Q
+
2C E1 = ; E2 = 2 ;
2 e0 A 2e 0 A
P-294 Topic-wise Solved Papers - PHYSICS
where A = area of each plate
Ceq and C3 are in series
d = separation between two plate
1 1 d d
E = E1 – E2 = (Q1 - Q2 ) \ = +
2e 0 A C Ae 0 ( K1 + K 2 ) 2 Ae 0 K3
Hence, V = Ed KAe 0
But C = for single equivalent capacitor
1 d Q - Q2 d
= (Q1 - Q2 ) = 1
2 e0 A 2C d d d
\ = +
7. (d) With the closing of switch S3 and S1 the negative KAe0 Ae 0 ( K1 + K 2 ) 2 Ae 0 K 3
charge on C2 will attract the positive charge on C2
thereby maintaining the negative charge on C1. The 1 1 1
or = + .
negative charge on C1 will attract the positive charge K K1 + K 2 2 K3
on C1. No transfer of charge will take place. Therefore
10. (c) Electric field lines do not form closed loops. Therefore
p.d across C1 and C2 will be 30 V and 20 V.
options (a), (b) and (d) are wrong.
Qq q 2 Qq Option (c) is correct. There is repulsion between similar
8. (b) Here we have + + =0 charges.
a a a 2
11. (a) When S is closed, there will be no shifting of negative
or Q 2 +q 2 +Q = 0 charge from plate A to B as the charge – q is held by the
charge + q. Neither there will be any shifting of charge
or Q ( 2 + 1) = - q 2 from B to A.
12. (b) NOTE : As we move along the direction of electric
q 2 2q field the potential decreases.
\ Q= - =- \ VA > VB
2 +1 2+ 2
y
9. (b)

(0, 1) C
E

(0, 0) (1, 0)
x
A B

13. (b) Initial energy


2Qq q Q q
Ui =
4pe 0 a x = –a 0 x=+a
Initially

q Q q
Let C1 = Capacity of capacitor with K1 Final energy x
C2 = Capacity of capacitor with K2
C3 = Capacity of capacitor with K3 a+x a–x
Finally
æ A ö e ´ 2 Ae 0 K1
\ C1 = K1 ç ÷ 0 = 2Qqa
è 2ø d d Qq é 1 1 ù =

æ A ö e ´ 2 Ae 0 K 2
Uf = +
4pe 0 ë a + x a - x úû
ê (
4 pe0 a 2 - x 2 )
\ C2 = K 2 ç ÷ 0 =
è 2ø d d é ù
2Qq ê 1 a ú
U i - Uf = -
\ C3 = K3 ( A)
e0 ´ 2 2 Ae0 K 3
= ëê (
4pe 0 ê a a 2 - x 2 ú
úû )
d d
C1 and C2 are in parallel é ù
2Qq ê a 2 - x 2 - a 2 ú -2Qqx2
= =
\ Ceq =
Ae 0
d
( K1 + K2 ) êë (
4pe 0 ê a a 2 - x 2 ú )
úû 4pe 0 a3
ELECTROSTATICS P-295
2 15. (c) Electric field everywhere inside the metallic portion of
when x << a then x is neglected in denominator..
shell is zero.
æ -Qq ö 2 Hence options (a) and (d) are incorrect.
=ç 3÷
x Electric field lines are always normal to a surface. Hence
è 2pe 0 a ø option (b) is incorrect. Only option (c) represents the

(
or U i - U f ) is proportional to x2 correct answer.
ur Kq
14. (c) Initially 16. (c) | E |= 2
r
C
Electric field due to P on O is cancelled by electric field
q1 =CV1 due to S on O.
V1 Similarly Electric field due to Q on O is cancelled by
electric field due to T and O.
The electric field due to R on O is in the same direction
C q2 =CV2 ur
as that of U on O. Therefore the net electric field is 2E .
V2

1 P- +Q
Initial energy = C (V12 + V22 ) (i)
2 E
q1' + q2' = CV1 + CV2 (ii) E
(charge conservation) - r +
U R
q'1 E O E

E
E
-
+T S

17. (d) The flux through the Gaussian surface is due to the
charges inside the Gaussian surface. But the electric
field on the Gaussian surface will be due to the charges
present in side the Gaussian surface and outside it. It
q'2
will be due to all the charges.
Potential difference across both the capacitors will be 18. (c) Figure shows the electric fields due to the sheets 1, 2
the same. and 3 at point P. The direction of electric fields are
according to the charge on the sheets (away from
q1 ' q2 ' positively charge sheet and towards the negatively
= Þ q1' = q2'
C C charged sheet and perpendicular).
From equation (ii)
C (V1 + V2 ) 1
q1 ' = Z = 3a P -s
2 E1
\ Final energy E2
E3 2
é C 2 (V1 + V2 )2 ù æ Q2 ö Z=a -2s
=ê ú´2 ç Using U = ÷
4 ´ 2C è 2C ø
êë úû

C (V1 + V2 ) 2 3
Final energy = Z = –a -s
4
\ Change in energy = Initial energy – Final energy The total electric field
1 C ur ur ur ur
= C (V12 + V22 ) - (V12 + V22 + 2V1V2 ) E = E1 + E 2 + E 3
2 4
C = E1 ( -k$ ) + E2 ( -k$ ) + E3 ( - k$ )
2 2 2 2
= [2V1 + 2V2 - V1 - V2 - 2V1V2 ]
4 é s 2s s ù $ 2s $
C =ê + + ú (-k ) =- k
2 2
ë 0 e 2 e 2 e0 û e0
= (V1 - V2 ) 0
4
P-296 Topic-wise Solved Papers - PHYSICS
19. (a) When a charge density is given to the inner cylinder, 23. (c) The electric field due to A and B at O are equal and
the potential developed at its surface is different from opposite producing a resultant which is zero. The
that on the outer cylinder. This is because the potential electric field at O due to C is
decreases with distance for a charged conducting
cylinder when the point of consideration is outside the 1 2q / 3 q
cylinder. E= = .
4π Îo R 2 6 π Îo R 2
But when a charge density is given to the outer cylinder,
it will change its potential by the same amount as that \ Option [A] is not correct. The electric potential at O is
of the inner cylinder. Therefore no potential difference
will be produced between the cylinders in this case.
20. (d) When a positive point charge is placed outside a q
conducting sphere, a rearrangement of charge takes B 3
30 º
place on the surface. But the total charge on the sphere - 2q C R
is zero as no charge has left or entered the sphere. O
3 60º
net charge = 0 R
q A
- +
- + 3
- +
+q - +
- +
é +q / 3 ù é +q / 3 ù é -2q / 3 ù
VO= K ê ú +K ê ú +K ê =0
ë R úû
21. (b) Let us consider a uniformly charged solid sphere
ë R û ë R û
without any cavity. Let the charge per unit volume be s
and O be the centre of the sphere. Let us consider a \ Option [D] is wrong
uniformly charged sphere of negative charged density AC AB
s having its centre at O'. Also let OO' be equal to a. In D ABC = sin 30º ÞAC= =R
Let us consider an arbitrary point P in the small sphere. AB 2
The electric field due to charge on big sphere
BC 3AB
ur s uuur Also =sin 60º Þ BC = = 3R
E1 = OP AB 2
3e 0 Potential energy of the system
é( q / 3)( 2/ 3) ù é ( q/3)( -2q / 3) ù é( q/3)( -2q / 3) ù
O Kê ú+Kê ú +Kê ú
ë 2R û ë R û ë 3R û
a
P O¢ kq 2 é1 2 ù
=
9R ê2 - 2 - ú ¹0
ë 3û
\ Option [B] is wrong
Also the electric field due to small sphere
Magnitude of force between B and C is
ur s uuur
E2 = PO ' \ The total electric field
3e 0 1 ( 2q / 3)( q / 3) q2
F= =
ur ur ur
E = E1 + E 2 =
s uuur uuur
[OP + PO] =
s uuur
OO '
4π Îo
( 3R )2 54π Îo R 2
3e 0 3e 0 24. (a) Let the level of liquid at an instant of time 't' be x. Then
Thus electric field will have a finite value which will be
uniform. dx
v=– Þ dx = –vdt
22. (c) The charges make an electric dipole. A and B points lie dt
on the equatorial plane of the dipole.
Therefore, potential at A = potential at B = 0 x t
W = q (VA – VB) = q × 0 = 0 Þ ò dx = – v ò dt
d /3 0
®

d
d
Þx – =– vt x
®
®
®

3
d
Þ x= – vt
3
Also the capacitance can be considered as an equivalent
of two capacitances in series such that
ELECTROSTATICS P-297

1 1 1 \ 2
Q3 = 9[4pR x] - Q1 - Q2 = 9[4pR x ] - [4pR 2 x ]
2
= +
Ceq C1 C 2
-3[4pR 2 x] = 5[4pR 2 x ]
1 1 1 d- x x
Þ = + = + Þ Q1 : Q2 : Q3 = 1: 3 : 5
Ceq Îo A Îo AK Îo A Îo AK
d –x x 26. (a) From the figure it is clear that the charge enclosed in
the cubical surface is 3C + 2C – 7C = –2C. Therefore
1 1 éd - x x ù 1 é Kd - Kx + x ù the electric flux through the cube is
\ = + ú =
Ceq Îo A êë 1 K û Îo A êë K ú
û qin -2C
f= =
e0 e0
Kd + x (1 - K ) Î0 AK
= \ Ceq =
Îo AK Kd + x(1 - K) Y
Z

d
But A = 1, K = 2 and x = – vt
3
Î0 ´1 ´ 2
\ Ceq =
éd ù
2d + ê - vt ú (1 - 2 ) X
ë 3 û 2C
3C
–7C
2 Î0 6 Î0
= =
d
2d - + vt
5d + 3vt
3 27. (a) The electrostatic pressure at a point on the surface of
6R Î0
\ Time constant t = RCeq = s2
5d + 3vt a uniformly charged sphere =
2 Î0
25. (b) The charges on the surfaces of the metallic spheres are
shown in the diagram. It is given that the surface charge
densities on the outer surfaces of the shells are equal. s2
\ The force on a hemispherical shell = ´ pR 2
Therefore 2 Î0
+ +
+ – –(Q1+ Q2) – 28. (d) When the electric field is on
– (Q1+ Q2) + +
+ – Q 1+ Q2+ Q3
Force due to electric field = weight
– + –
– + + – + qE = mg

+ – –Q 1

– + 4 3 4pR 3rg
+ – + qE = p R rg \q = ...(i)
3 3E
+ – + R Q1 – +
– + When the electric field is switched off

– + 2R +– Weight = viscous drag force
+ –
– + +
– mg = 6phRvt
+ 3R –
+ – – +
+ 4 3 9hvt
+ pR rg = 6phRvt \ R = ...(ii)
Q1 Q1 + Q2 Q1 + Q2 + Q3 3 2rg
= = = x(say)
4pR 2 4p (2 R ) 2 4 p(3R ) 2
3
\ Q1 = 4pR x 2 4 é 9hvt ù 2 rg
From (i) & (ii) q = p ê ´
3 ë 2rg úû E
Q1 + Q2 = 4p(2R )2 k = 4[4pR 2 x ]
3
Þ Q2 = 4[4pR 2 x ] - Q1 4 é 9 ´ 1.8 ´ 10-5 ´ 2 ´ 10-3 ù 2 900 ´ 9.8 ´ 7
= ´pê ú ´
= 4[4pR 2 x ] - 4pR 2 x = 3[4pR2x] 3 ë 2 ´ 900 ´ 9.8 û 81p ´ 105

Also Q1 + Q2 + Q3 = 4p(3R)2 x = 9[4pR 2 x] = 7.8 × 10–19 C


P-298 Topic-wise Solved Papers - PHYSICS
r 32. (a) The frequency of SHM performed by wooden block is
29. (c) Given E = Eo xˆ
This shows that the electric field acts along + x direction 1 k
and is a constant. The area vector makes an angle n0 =
2p m
of 45° with the electric field. Therefore the electric
flux through the shaded portion whose area is
r r
a´ 2a = 2 a 2 is f = E. A = EA cos q = E0 ( 2 a2 ) cos
E
2 1
45° = E0 ( 2 a ) ´ = E0a2 +Q
2
z

when electric field is switched on, the value of k and m


is not affected and therefore the frequency of SHM
(a,0,a) (a,a,a) q = 45° remains the same. But as an external force QE starts
2a acting on the block towards right, the mean position of
E
q A QE
y SHM shifts towards right by
(0,0,0) (0,a,0) k
correct option is (a).
Note : In SHM if a constant additional force is applied
then it only shift the quilibrium position and does not
x change the frequency fo SHM.
30. (d) When S and 1 are connected
The 2mF capacitor gets charged. The potential 33. (c) The two forces acting on the proton just after the release
difference across its plates will be V. are shown in the figure. In this situation
The potential energy stored in 2 mF capacitor
1 1 qE
U i = CV 2 = ´ 2 ´ V 2 = V 2 +
2 2 q
When S and 2 are connected
The 8mF capacitor also gets charged. During this v
charging process current flows in the wire and some mg
amount of energy is dissipated as heat. The energy
loss is
qE = mg [\ q = 45º]
1 C1 C2 2
DU = 2 C + C (V1 - V2 ) æV ö
q ç ÷ = mg
1 2 \
èdø
Here, C1 = 2mF, C2 8 mF, V1= V, V2 = 0
mgd 1.67 ´ 10 -27 ´ 10 ´ 10 -2
1 2 ´8 4 \ V= = = 10 -9 V
\ DU = ´ (V - 0)2 = V 2 q 1.6 ´ 10 -19
2 2+8 5
34. (d) For a thin uniformly positive charged spherical shell
DU (i) Inside the shell at any point
The percentage of the energy dissipated = ´ 100
Ui
1 q
E = O and V = = constt.
4 2 4 p Î0 R
V
= 5 ´ 100 = 80% where q = charge on sphere
V2 R = Radius of sphere
31. (c) The pattern of field lines shown in option (c) is correct (ii) Outside the shell at any point at any distance r
because 1 1
(a) a current carrying toroid produces magnetic field from the centre E µ and V µ
2 r
lines of such pattern r
(b) a changing magnetic field with respect to time in 35. (c) The total charge on plate A will be – 80 µC. If qB and qC
a region perpendicular to the paper produces be the charges on plate B and C then
induced electric field lines of such pattern. qB + qC = 80 µC ...(1)
ELECTROSTATICS P-299

2. (a, d)

+80 mC

4mF
–80 mC A vo vo
(i) P.d. = V0 P.d. = V0
+qB B +qC C
Capacitance = C Capacitance = KC
3mF (ii) Q0 = CV0 [K is the dielectric constant of
2mF –qC slab K > 1]
–qB
New charge = KCV0
(iii) Potential Energy New potential energy
1 1
= CV02 = KCV02
2 2
Also 2µF and 3µF capacitors are in parallel. Therefore,
V0 V0
qB qC (iv) E = E=
= d d
2 3 3. (b) q has to be negative for equilibrium.
80 - qC qC
\ = 1 F12 2 F23 3
2 3
F13 +Q F21 –q +Q
240
\ 240 – 3qC = 2qC \ qC = = 48µC l l
5
Considering equilibrium of 1
This charge will obviously be positive. F13 = F12
KQ ´ Q KQ ( - q ) Q
2
= \q= -
1. (d) Let us consider the positive charge Q at any instant of (2l) l2 4
time t at a distance x from the origin. It is under the 4. (b, d)
r r Charge on plate is q
influence of two forces F1 (= F) and F2 (= F). On
resolving these two forces we find that F sin q cancels e0 A
C=
out. The resultant force is d
q
q = CV Þ V = ,
C
1
q ´V
U=
2
Charge on plate is q
e0 A
C' = Þ C' < C,
d'
q
V' = Þ V' > V
C'
1
qV' Þ U' = U
U' =
2
kQq x 5. (a) The potential inside the shell will be the same
FR = 2F cos q = 2 × ´ everywhere as on its surface. As we add – 3Q charge
( x2 + a 2 ) x2 + a 2 on the surface, the potential on the surface changes
by the same amount as that inside. Therefore the
2k Q q x potential difference remains the same.
= 2
( x + a 2 )3 / 2 6. (a) The equivalent capacitance
Since FR is not proportional to x, the motion is NOT simple 1 1 1 1 11
harmonic. The charge Q will accelerate till the origin and gain = + + =
Ceq 2 2 2 ´ 5 10
velocity. At the origin the net force is zero but due to
momentum it will cross the origin and more towards left. As it 10
comes on negative x-axis, the force is again towards the origin. Þ Ceq = µF
11
P-300 Topic-wise Solved Papers - PHYSICS

éæ æ ö ù
Q2 ö Q1
(T.P.E.)B = ê ç q + ç ÷ qú
ê è 4pe 0 R ø÷ çè 4pe R 2 + R 2 ÷ø ú
ë 0 û

q é Q ù
= ê Q2 + 1 ú
4 pe 0 R ë 2û

q é Q Q ù
\ W= ê Q1 + 2 - Q2 - 1 ú
4pe 0 R ë 2 2û
7. (a, c, d)
e0 K e0 A q (Q1 - Q2 ) æ 2 - 1ö
As C = A C' = =
d d 4pe 0 R çè 2 ÷ø
9. (c) Let l be the charge per unit length. Let us consider a
Gaussian surface (dotted cylinder).
V Applying Gauss's law
K V '=
K - -
ur uur l l
f = Ñò E.ds =
+ +
e0 - -
+ +
Q = CV For the flat portions of -
l + + -
e0 A e 0 AV Gaussian surface, the
= ´V Q= = C 'V ' angle between electric - + r + -
d d
Q will remain same as no charge is leaving or entering the field and surface is 90°. + +
plates during the process of slab insertion Hence flux through flat - -

Now, Q = C ' V ' = C ' E 'd portions is zero.


NOTE : By symmetry, the electric field on the curved surface
e 0 AV is same throughout.
ur uur
Q 1 V
E'= = d ´ = The angle between E and ds is 0° (for curved surface)
C ' d K e 0 A d Kd ll ll
d Þ E ò ds = Þ E × 2 pr l =
e0 e0
Work done is the change in energy stored
l 1
1 2 1 2
Þ E= Þ Eµ
W = CV - C 'V ' 2 pe 0r r
2 2 10. (d) The electric lines of force cannot enter the metallic
2 2 sphere as electric field inside the solid metallic sphere
1 e 0 AV 1 K e0 A æ V ö é Vù
=
2 d
-
2 d
´ç ÷
èKø êëQ V ' = E ' d = K úû is zero. Also, the origination and termination of the
electric lines of force from the metallic surface is
normally (directed towards the centre).
1 e0 A 2 é 1 ù r
W= V ê1 - ú 11. (b, c) In region I and III, there will be electric field E0 directed
2 d ë Kû
from positive to negative. In region II, due to orientation
8. (b) The work done in moving a charge from A to B ur
of dipoles, there is an electric field E k present in
ur ur
opposite direction of E 0 . But since E 0 is also present,
r r ur
the net electric field is E0 - Ek in the direction of E 0
as shown in the diagram. (Q E0 > Ek)

Ek
W = (T.P.E.)A – (T.P.E.)B where T.P.E. = Total Potential Energy Eo Enet= Eo – Ek Eo
_ +
éæ æ ö ù
Q1 ö Q2 DIELECTRIC
(T.P.E.)A = ê ç ´ q + ç ÷ qú SLAB
ê è 4pe 0 R ø÷ çè 4pe R 2 + R 2 ÷ø ú
ë 0 û

q é Q ù
= Q1 + 2 ú
4pe 0 R êë 2û x=0 d d d x = 3d
ELECTROSTATICS P-301
14. (a, c)
E KEY CONCEPT : The expressions of the electric field inside
d 2d 3d 1 Qq
0 x the sphere (r < R) E = . r ; outside the sphere
4pe 0 R 3
E0 1 Q
(R < r < ¥) E = .
4pe 0 r 2
V C
B
A

x
O d 2d 3d

NOTE : When one moves opposite to the direction of electric


field, the potential always increases. The stronger the electric Hence, E increases for r < R and decreases for R < r < ¥.
field, the more is the potential increase. Since in region II, 15. (c, d)
the electric field is less as compared to I and III therefore the When two points are connected with a conducting path in
increase in potential will be less but there has to be increase electrostatic condition, then the potential of the two points
in potential in all theregionsfrom x = 0 to x = 3d. Also where is equal. Thus potential at A = Potential at B
(c) is the correct option.
dV Option (d) is a result of Gauss's law
E is uniform, = constt.
dx
q
12. (d) Potential at origin will be given by Total electric flux through cavity =
Î0
q é1 1 1 1 ù Option (a) and (b) are dependent on the curvature which is
V= ê - + - + ....ú
different at points A and B.
4pe 0 ë x0 2 x0 3x0 4 x0 û
16. (a,b,d)
(a) The whole charge V
q 1 é 1 1 1 ù
= .
4pe0 x0 ê1 - 2 + 3 - 4 + ....ú Q will be enclosed in a
ë û sphere of diameter 2R0.
(b) Electric field E =
q
V= ln (2) zero inside the sphere. R0
4pe 0 x0 r
Hence electric field is + +
+ +
13. (a, c) Let Q be the charge on the ring, the negative charge – discontinued at r = R0. ++ ++
+
q is released from point P (0, 0, Z0). The electric field at P (c) Changes in V and ++
++++++

due to the charged ring will be along positive z-axis and its E are continuously R0
+ +++++
presen t for
magnitude will be
r > R0 . Option (c) is
1 QZ 0 incorrect.
++

E= . (d) For r < R 0 , the


+
4pe 0 ( R + Z 02 )3 / 2
2 + ++
+++++
potential V is constant
Therefore, force on charge P will be towards centre as and the electric
E
shown, and its magnitude is intensity is zero.
Obviously, the
1 Qq electrostatic energy is
Fe = qE = . .Z ... (1)
4pe 0 ( R + Z 02 )3 / 2 0
2 zero for r < R0.
17. (a) R0 r
Similarly, when it crosses the origin, the force is again towards v
centre O.
Thus the motion of the particle is periodic for all values of –q
Z0 lying between 0 and ¥.
Secondly if Z0 << R, (R2 + Z 02 )3/2 » R3 +Q

1 Qq
Fe = . . Z0 [From equation 1]
4pe 0 R3 The situation is shown in the figure which is similar to
i.e. the restoring force Fe µ – Z0. Hence the motion of the a planet revolving around sun. The distance of – q
particle will be simple harmonic. (Here negative sign implies from + Q is changing, therefore, force between the
that the force is towards its mean position). charges will change.
P-302 Topic-wise Solved Papers - PHYSICS
The speed of the charge – q will be greater when the 21. (a, c, d)
charge is nearer to + Q as compared to when it is far.
a a
Therefore, the angular velocity of charge – q is also The electric flux passing through x = + , x = - , z =
variable. The direction of the velocity changes 2 2
continously, therefore, linear momentum is also variable. a
The angular momentum of (– q) about Q is constant + is same due to symmetry..
2
because the torque about + Q is zero.
The net electric flux through the cubical region is
18. (a,d) The electric field lines are orginating from Q1 and
terminating on Q2. Therefore Q1 is positive and Q2 is -q + 3q - q q .
=
negative. e0 e0
As the number of lines associated with Q1 is greater
than that associated with Q2, therefore |Q1| > |Q2|. 22. (a, b, c)
Option (a) is correct.
At a finite distance on the left of Q1 , the electric field
intensity cannot be zero because the electric field
+ F E q –
created by Q1 will be greater than Q2. This is because
the magnitude of Q1 is greater and the distance smaller +2 –2q
é Qù A D
êë E µ r 2 úû
At a finite distance to the right of Q2, the electric field
is zero. Here, the electric field created by Q2 at a + –q
B C
particular point will cancel out the electric field created
by Q1.
19. (a, b, c, d)
EB + EE
Electric field inside a spherical metallic shell with charge
on the surface is always zero. Therefore option [a] is
correct.
When the shells are connected with a thin metal wire
then electric potentials will be equal, say V. 60º
1 QA 1 QB O 60º EA ED P
\ 4p Î R = 4 p Î R = V
0 A 0 B
As RA > RB therefore QA > AB. option [b] is also correct.
EF + EC
QA
2 2 2
s A 4 pR A RB QA RB 4 pÎ0 R AV
As = = ´ = ´ uuur uuur
s B QB 2 QB R 2 4 p Î0 RBV
RA |EA | uur uuur |ED | uuur uuur
2
4 pRB
A Here =|EB |=|EC | = =| EE |=|EF | = K
2 2
\ EO = EA + ED + (EF + EC) cos 60º + (EB + EC) cos 60º
s A RB
\ s = 1 1
B RA = 2K + 2K + (K + K) × + (K + K) × = 6K
2 2
Option (c) is also correct
The electric potential at O is
s s
Also E A = A & EB = B 1
Î0 Î0 VO = [2q + q + q – q – q – 2q] = 0
4p e 0 L
E A s A RB PR is perpendicular bisector (the equatorial line) for the
= = <1 \ E <E
EB sB RA A B electric dipoles AB, FE and BC. Therefore the electric
Option (d) is also correct potential will be zero at any point on PR.
20. (c, d) At any point ST, the electric field will be directed from S to T.
(a) is not correct because it is valid only when E µ r–2 The potential decreases along the electric field line.
(b) is not correct 23. (b, d)
(c) is correct as between two point charges we will get a Electric field E1 due to smaller sphere at P is
point where the electric field due to the two point
4
charges cancel out each other. r1 ´ p R 3
1 3
(d) is correct when the work done is without accelerating E1 =
the charge. 4pe 0 (2R)2
ELECTROSTATICS P-303

+ + + + + SUBJECTIVE PROBLEMS :
+ + + + + + +
+ + ® ®
1. (i) The force on charge q kept at A due to charges kept at
+ + + + E2 P E1 + + + + B and C
+ + + + + + + +
+ + + 2R R + + + + q2 3
+ + + + + + + + F1 = 2F cos 30° = 2 × 9 × 109 × ´
r1 2 2
a
+ + + + +

r2 æ q2 ö
F1 = 3 ´ ç 9 ´ 109 2 ÷
1 r pR rR è a ø
E1 = ´ 1 = 1
4p Î0 3 4 Î0 ´3 The force on q due to charge (– q) kept at D
Electric field E2 due to bigger sphere at P is
r R q2 9 æ q2 ö
E2 = 2 F2 = 9 × 109 = ´ ç 9 ´ 109 ´ 2 ÷
3Î 0 (2a / 3)2 4 è a ø
rR r R
As E1 = E 2 \ 1 = 2 Clearly the two forces are not equal. Also as F2 > F1 the
4 Î0 ´3 3 Î0 charges will move towards the centre.
r1
=4
r2
Option (d) is correct.
24. (b, d)
Step 1 : When S1 is pressed. The capacitor C1 gets charged
such that its upper plate acquires a positive charge + 2 CV0
and lower plate – 2 CV0.
Step 2 : When S2 is pressed (S1 open). As C1 = C2 the
charge gets distributed equal. The upper plates of
C1 and C2 now take charge + CV0 each and lower plate
– CV0 each.
(b) and (d) are correct option.
25. (c, d)
Let us consider a point P on the overlapping region. The
electric field intensity at P due to positively charged sphere (ii) For charges to remain stationary
r
rr1
= q2 3 9 q2Q 4 3q
3 Î0 2× K ´ = ´K´ 2 Þ =Q
a2 2 4 a 9
The electric field intensity at P due to negatively charged The charge Q should be negative.
r The potential energy of the system is
rr 2
sphere = . The total electric field, é q2 q2 ù é 4 3 q´q ù
3 Î0 = 3 êK 2 + K 2 ú + 3 êK ´ ú
ur ur êë a a úû êë 9 (2a / 3) 2 úû
ur rr1 rr r r r
E= + 2 = é r1 + r 2 ù
3 Î0 3 Î0 3 Î0 ë û
q2 q2 q2
ur r r = 6K × +3 3K = 3(2 + 3) K
E= r a2 a2 a2
3 Î0
Therefore the electric field is same in magnitude and This is the amount of work needed to move the charges to
direction option (c) and (d) are correct. infinity.
2.

P
r1 r2
+r –r
r
Because of equilibrium of charge q1
N1 = mg sin 60° + (T – F) sin a ... (i)
and (T – F) cos a = mg cos 60° ... (ii)
P-304 Topic-wise Solved Papers - PHYSICS
N1
(T–F) cos a
3.
a
T–F
60° mg sin 60°
mg cos 60°
[(T–F) sin a ]
mg
30°

Because of equilibrium of charge q2


(T – F) sin a = mg cos 30° ... (iii)
and N2 = (T – F) cos a + mg sin 30° ... (iv)

m = 80 × 10–3 g = 80 × 10–6 kg,


q = 2 × 10–8 C;
E = 20,000 V/m
T sin q = qE ... (i)
T cos q = mg ... (ii)

qE 2 ´ 10-8 ´ 2 ´ 104
\ tan q = = = 0.5
From (i) and (iii) mg 80 ´ 10-6 ´ 9.8
N1 = mg sin 60° + mg cos 30°
\ q = 27° 2'
æ 3 3ö Substituting the value of q in (i), we get
= mg ç + = 3 mg
è 2 2 ÷ø T sin (27° 2') = 4 × 10–4 \ T = 9 × 10–4 N.
4. NOTE : When a positively charged particle starts from rest
From (ii) and (iv) in a uniform electric field, it will move along the direction of
æ 1 1ö electric field.
N2 = mg cos 60° + mg sin 30° = mg ç + ÷ = mg Thus charged particle moves along the line of force only if
è 2 2ø
it is straight. But if the electric field is variable, the charged
qq particle will move under the influence of instantaneous
Also, F = k 1 22 velocity and the force of electric field.
l 5. Let q be the charge on the inner sphere and (Q – q) be the
Now from eqn. (ii) and (iii), we get charge on outer sphere.
(T – F)2 cos2 a + (T – F)2 sin2 a
= m2g2 cos2 60° + m2g2 cos2 30°

é1 3ù
Þ (T – F)2 = m2g2 ê + ú = m2g2
ë4 4û
Þ T – F = ± mg ... (v)
Þ T = mg + F
qq
= mg + k 1 22 ... (vi)
l
[Taking positive sign]
From (ii) and (v)
mg cos a = mg cos 60° Þ cos a = cos 60°
Given that surface charge densities are equal.
\ a = 60°
when the string is cut, T = 0 q Q-q æ qö
\ = çè Surface charge density,s = ÷ø
\ From (vi) 4pr 2 4pR 2 A
or, qR2 = (Q – q) r2 or, qR2 = Qr2 – qr2
q1q2 mg l2
mg = ± k 2 Þ q1q2 = ±
l k Qr 2
\ q=
Now the charges should be unlike for equilibrium. R2 + r 2
ELECTROSTATICS P-305
Potential at O due to inner sphere Comparing the above equation with A = -w x we get 2

q K æ Qr 2 ö
Vi = K = ç 2 2÷ kQq kQq
r r èR +r ø \ w2 = or w =
mr 3
mr 3
Qr
Vi = K
R2 + r 2 2p kQq
\ =
Potential at O due to outer sphere T mr 3
Ké Qr 2 ù
V0 = K (Q - q ) = R êQ - 2 2 ú mr 3
R êë R + r ûú T = 2p
kQq
é 2 2 2ù
K ëQR + Qr - Qr û K ( QR )
2
= 1/ 2
=
R (R2 + r 2 ) R R2 + r 2 ( ) é 0.9 ´ 10-3 ´ 13
T = 2 × 3.14 ê 9 -5 -6
ù
ú
êë 9 ´ 10 ´ 10 ´ 10 úû
KQR
= 6.28 [0.01]1/2 = 6.28 [0.1]
(
= R2 + r 2
) T = 0.628 sec
The total potential at the common centre 7. The potential difference across each capacitor is V.
V = Vi + V0 (a) Total Energy = Energy in A + Energy in B

KQ( R + r ) 1 1
KQr KQR =CV 2 + CV 2 = CV2
= + = 2 2
R 2 + r 2 R2 + r 2 R2 + r 2
When the switch opened and a dielectric is inserted
6. KEY CONCEPT : The electric field due a uniformly charged
between the plates of capacitors, the new capacitance
ring of radius r at a point distant x from its center on its axis
is 3C.
is given by
1 3
Energy in A = (3C )V 2 = CV 2 (V is the same)
2 2

1 q2 1 (CV ) 2
Energy in B = = ´
2 KC 2 3C

CV 2
= (charge on capacitor B remains same when
6
switch is opened)
Total Energy = Energy in A + Energy in B
3 1 5
Qx \ Total Energy = CV 2 + CV 2 = CV 2 ... (ii)
E= k 2 6 3
( r 2 + x 2 )3 / 2
Total Energy initially CV 2 3
r = 1m = =
Total energy finally 5 5
Q = 10–5 C CV 2
3
mass of particle m = 0.9 × 10–3 kg
charge on particle q = – 10–6 C 8. Total energy of the system of three charges when the charge
\ Force on the negative charge q will be F = qE –q is at C
= P.E. + K.E.
- kQq -k Q q
\ F= 2 2 3/ 2
´x or, mA = ´x é Kq ´ q K (q)( - q) Kq (- q) ù
(r + x ) ( r + x 2 )3 / 2
2
=ê + + +4 ... (i)
ë 6 5 5 úû
Qq
or, A=–k ´x
2
m ( r + x 2 )3 / 2

kQq
For x << r A = - ´x
r3
Þ The motion is simple harmonic in nature.
P-306 Topic-wise Solved Papers - PHYSICS
Final energy of the system of three charges when – q is at D 10. Each mass will be in equilibrium under the act of three force
and momentarily at rest namely tension of string, weight, resultant electrostatic force
= P.E. + K.E. of the two other charges out of these three forces F and mg
é Kq ´ q Kq (- q ) are perpendicular.
Kq ( - q ) ù
=ê + + ú +0
ëê 6 x 2 + 32 x 2 + 32 úû
Kq ´ q 2Kq(- q)
= + ... (ii)
6 x 2 + 32
By the principle of conservation of energy from (i) and (ii),
we get
kq ´ q 2kq (- q) kq ´ q 2kq(- q)
+ +4= +
6 5 6 x 2 + 32
é1 1 ù Let T make an angle q with the vertical
2 = kq2 ê - ú
ëê 5 x 2 + 32 úû 2
OC = (0.03)2 - (0.015)2 = 0.0173 m
é1 ù 3
1
2 = 9 ´ 109 ´ 5 ´ 10 -5 ´ 10 -5 ´ 5 ê 5 - ú \ OM = 0.9997
êë x 2 + 32 úû NOTE THIS STEP : Resolving T in the direction of mg and
é1 ù F and applying the condition of equilibrium, we get
45 1
2= ê - ú T cos q = mg; T sin q = F
2 ëê 5 x 2 + 32 ûú
F
1 1 4 9-4 5 1 \ tan q = ... (i)
= - = = = mg
x 2 + 32 5 45 45 45 9
\ x2 + 9 = 81 \ x = 8.48 m F= 2
FCA 2
+ FCB + 2 FCA FCB cos a
9. For the potential energy to be minimum the bigger charges
should be farthest. Therefore the combination would be as 2 2 2 1
shown in the diagram. \ F= FCA + FCA + 2 FCA ´
2
(0.09–x) x
kq 2
F= 3FCA = 3 ´ ... (ii)
(CA) 2
2q q 8q [where FCB = Force on C due to B
The potential energy of the system is FCA = Force on C due to A
ur ur
é 8q ´ 2 q 8 q ´ q q(2q) ù | F CB | = | F CA | and a = 60° ]
U= K ê + +
ë 0.09 x (0.09 - x) úû OC 0.0173
For potential energy to be minimum using Also, tan q = = .... (iii)
OM 0.9997
dU é -8q ´ q q ´ 2q( -1) ù From (i), (ii) and (iii)
=0 \ Kê - ú=0
2
dx
ëê x (0.09 - x)2 ûú 0.0173 3 ´ 9 ´ 109 ´ q 2
4 1 =
Þ = 0.9997 (0.03) 2 ´ 10 -3 ´ 9.8
2
x (0.09 - x)2
On solving, we get q = 3.16 × 10–9 C.
Þ 2 (0.09 – x) = x Þ 2 × 0.09 – 2x = x
11. Torque acting on the dipole
2 ´ 0.09
Þ x= = 2 × 0.03 = 0.06 m
3
Electric field at q due to the two charges
ur ur ur k ´ 8q k (2q )
E = E1 + E 2 Þ E = 2
-
x (0.09 - x ) 2
9 ´ 109 ´ 8q 9 ´ 109 ´ 2 q
= -
(0.06)2 (0.03) 2

9 ´ 8 ´ 1013 q 9 ´ 2 ´ 1013 q
= - =0
6´6 3´3
ELECTROSTATICS P-307
When q is small then sin q » q 13. (a) Let P be a point in the X-Y plane with coordinates (x, y)
at which the potential due to charges – 2Q and + Q
\ t = (qEL) q
placed at A and B respectively be zero.
\ Iw2 = qEL (Q t = I a ) ... (i)

ML2 ML2 ML2


Now, I= + = ... (ii)
4 4 2
From (i) and (ii)
ML2 2 2qE
w = qEL Þ w =
2 ML

2p 2qE ML
Þ = Þ T = 2p
T ML 2qE
This is the time period of complete oscillation.
\ Time for the dipole to align along the direction of electric
field will be
T 2p ML p ML
t= = = .
4 4 2qE 2 2qE

12.

K (2Q ) K (+Q)
\ =
(3a + x) 2 + y 2 (3a - x) 2 + y 2

Þ 2 (3a - x )2 + y 2 = (3a + x) 2 + y 2
Þ 4 [(3a – x)2 + y2] = [(3a + x)2 + y2]
Charge on Shell A = qA = s (4pa2) Þ 4 [6a2 + x2 – 12ax + y2] = [6a2 + x2 + 12ax + y2]
Charge on Shell B = qB = s (4pb2) Þ 3x2 + 3y2 – 3ax + 27a2 = 0
Charge on Shell C = qC = s (4pc2) Þ x2 + y2 – 10ax + 9a2 = 0
The potential of shell A Þ (x – 5a)2 + (y – 0)2 = (4a)2
kq A kqB kqC This is the equation of a circle with centre at (5a, 0) and
VA = + + radius 4a. Thus C (5a, 0) is the centre of the circle.
a b c
(b) For x > 3a
k s (4 pa 2 ) k ( -s ) (4 pb ) k s (4 pc 2 )
2 To find V (x) at any point on X-axis, let us consider a point
= + + (arbitrary) M at a distance x from the origin.
a b c
1 4 pa 2 1 (4pb 2 ) 1 (4 pc 2 )
= ´s´ - s + ´s
4pe0 a 4pe 0 b 4pe 0 c

s kq A kqB kqC The potential at M will be


= [a - b + c] Similarly,, VB = + +
e0 b b c
K ( -2Q) K (+Q) 1
V (x) = + where k =
kq A kqB kqC x + 3a ( x - 3a) 4pe0
and VC = + +
c c c
é 1 2 ù
\ V (x) = KQ ê - for | x | > 3a
s é a2 ù s é a2 - b2 + c 2 ù ë x - 3a x + 3a úû
VB = ê - b + c ú and VC = ê ú
e 0 êë b úû e 0 êë c úû Similarly, for 0 < | x | < 3a

Given that VA = VC é 1 2 ù
V (x) = KQ ê - ú
ë 3a - x 3a + x û
s s é a 2 - b2 + c 2 ù Since circle of zero potential cuts the x-axis at (a,0) and
(a - b + c ) = ê ú
e0 e 0 ëê c ûú (9a, 0)
or ac – bc + c2 = a2 – b2 + c2 or c = a + b Hence, V (x) = 0 at x = a, at x = 9a
P-308 Topic-wise Solved Papers - PHYSICS
• From the above expressions
V (x) ® ¥ at x ® 3a and V (x) ® – ¥ at x ® – 3a rx 2
\ Potential at the surface of the sphere of radius x =
• V (x) ® 0 as x ® ± ¥ 3e 0
1 Work done in bringing the charge dq on the sphere of radius x
• V (x) varies in general.
x
rx 2 rx 2
dW = × dq Þ dW = × 4p x2 r dx
V 3e 0 3e 0
Therefore the work done in accumulating the charge Q over
a spherical volume of radius R meters
R
–3a R 4pr2 4pr2 é x5 ù 4pr2 R5
x
a 3a 9a W= ò0 3e 0
x 4 dx =
3e 0
ê ú =
3e 0 5
ëê 5 ûú 0
2
4 p æ Q ö R5 3Q 2
= ç ÷ =
3e 0 è 4 / 3pR3 ø 5 20 pe0 R
(c) Applying Energy Conservation
This is also the energy stored in the system.
(K.E. + P.E.)centre = (K.E. + P.E.)circumference
(b) The above energy calculated is
é Qq 2Qq ù 1 2 é Qq 2Qq ù
0+K ê - ú = mv + K ê - 3Q 2 3KQ 2
ë 2a 8a û 2 ë 6a 12a úû E= = where K =
1
5 ´ (4pe 0 ) R 5R 4pe 0
1 2 KQq KQq 1 æ Qq ö NOTE : In case of earth and gravitational pull, K may be
mv = , v= = ç ÷
2 4a 2 ma 4 pe 0 è 2 ma ø replaced by G. Therefore the energy required to disassemble
the planet earth against the gravitational pull amongst its
14. (a) Let us consider a shell of the thickness dx at a distance
constituent particle is the work required to make earth from
x from the centre of a sphere
its constituent particles.

3GM 2
\ E= [ Q Q is replaced by M]
5R

GM GM 2
But g = Þ gMR =
R2 R
Kq1q2 G m1m2
F= 2
;F=
r r2
4 é 4 ù 3 3
The vol. of the shell = p ê( x + dx )3 - p x 3 ú \ E= gMR = ´ 10 ´ 2.5 ´ 1031 = 1.5 ´ 1032 J
3 ë 3 û 5 5
(c) During the charging process, let at any instant the
4 é
= p ( x + dx)3 - x 3 ù spherical conductor has a charge q on its surface.
3 ë û
1 q
The potential at the surface = ´
4 3 é æ dx ö ù
3
4pe 0 R
= p x ê ç1 + ÷ - 1ú
3 êë è xø úû
dq
4 3 é 3dx ù q
= p x ê1 + - 1ú
3 ë x û
4 3 3dx
px ´
= = 4px2dx R
3 x
Let r be the charge per unit volume of the sphere
\ Charge of the shell = dq = 4px2rdx ... (i)
Potential at the surface of the sphere of radius x Small amount of work done in increasing charge dq more on
the surface will be
4
r ´ px 3 é qù 1 q
1
êëQ V = k r úû
= ´ 3 dW = ´ ´ dq
4pe 0 x 4pe 0 R
ELECTROSTATICS P-309
\ Total amount of work done in bringing charge Q on the
surface of spherical conductor. 2.2 ´ 2.2 ´ 10-14 é 2 - 0.4 ù
= êë 2 ´ 0.4 úû
Q 2 ´ 10-9
1 Q 1 é q2 ù Q2
W= ò
4pe 0 R 0
qdq = ê ú
4pe 0 R êë 2 úû
=
(8pe 0 R) = 1.21×
1.6
´ 10 -5 = 4.84 × 10–5 J
0
0.4
ALTERNATE SOLUTION
Solution of part (c) e 0 e r AB
(iii) The capacitance of B =
If we consider the charged spherical capacitor as an isolated d
capacitor then the energy stored in the capacitor is given
by 8.85 ´ 10-12 ´ 9 ´ 0.02
=
8.85 ´ 10-4
1 Q2
E= where C = capacitance of the capacitor CB = 1.8 × 10–9 F
2 C
The charge on A, qA = 2.2 × 10–7 C gets distributed into two
For an isolated spherical capacitor C = 4pe0R
parts.
Q2 \ q1 + q2 = 2.2 × 10–7 C
\ E= also the potential difference across A = p.d. across B
8p e0R
15. (i) NOTE : The capacitor A with dielectric slab can be q1 q
= 2
considered as two capacitors in parallel, one having dielectric C A CB
slab and one not having dielectric slab. Each capacitor has
A CA 0.4 ´ 10-9
an area of . Þ q1 = q2 = q = 0.22 q2
2 CB 1.8 ´ 10-9 2
The combined capacitance is \ 0.22 q2 + q2 = 2.2 × 10–7
+ 2.2
110V + + + + Þ q2 = ´ 10-7 = 1.8 × 10–7 C
1.22
A - - - + Þ q1 = 0.4 × 10–7 C
A/2 A/2
- q12 q2
Total energy stored = + 2
C = C1 + C2 2C A 2C B
( A / 2) e0 ( A / 2) e 0 er A e 0
= + = [1 + e r ]
d d 2 d 0.4 ´ 0.4 ´ 10-14 1.8 ´ 1.8 ´ 10-14
= +
2 ´ 0.4 ´ 10-9 2 ´ 1.8 ´ 10-8
0.4 ´ 8.85 ´ 10-12
= [1 + 9] = 2 ×10–9 F = 0.2 × 10–5 + 0.9 × 10–5 = 1.1 × 10–5 J
2 ´ 8.85 ´ 10-4 16. Let the particle be at a distance x from the origin at a point P
1 1 at any instant. The electric field at P due to charge on the
\ Energy stored = CV 2 = × 2 × 10–9 × (110)2 circular ring is
2 2
= 1.21 × 10–5 J
(ii) Work done in removing the dielectric slab = (Energy
stored in capacitor without dielectric) – (Energy stored in
capacitor with dielectric).
NOTE : While taking out the dielectric, the charge on the
capacitor plate remains the same.

q2 q2
\ W= - Here, C = 2 × 10–9 F,
2C ' 2C

Ae0 0.04 ´ 8.85 ´ 10 -14


C'= = = 0.4 × 10–9 F
d 8.85 ´ 10-4
q = CV = 2 × 10–9 × 110 = 2.2 × 10–7 C 1 2 pR l x
E= from P to X
-7 2 4 pe 0 ( R + x 2 )3 / 2
2
(2.2 ´ 10 ) é 1 1 ù
\ W= ê -9
- -9 ú \ Force acting on q at P
2 ë 0.4 ´ 10 2 ´ 10 û
F = qE
P-310 Topic-wise Solved Papers - PHYSICS
Small amount of work done in moving the particle toward O
1 Q
for an infinitesimally small distance dx V ( x) = . , at A.
dW = Fdx
4pe 0 R2 + x 2

1 2 pRl xq 1 2 pR l
= .dx VO = . , at O. or V = l
4pe 0 ( R 2 + x 2 )3 / 2 4pe 0 R O
2e 0
The total work done
1 2 pR l l
VP = =
2pR lq 0 é ù 4 pe0 4e 0
W=
4pe 0 ò ê 2
x dx
2
3 R (R + x ) 3/ 2 ú ... (1) R2 + ( 3R ) 2

ëê ûú
Potential difference between points O and P = V
0 é x dx ù \ V = VO - VP
To calculate, ò ê 2 2
3 R (R + x ) 3/ 2 ú
ëê ûú l l l
or V= - or V =
Let R2 + x2 = t \ 2xdx = dt 2e 0 4e 0 4e 0
dt The kinetic energy of the charged particle is converted into
Þ xdx = its potential energy at O.
2
\ Potential energy of charge (q) = qV
0 0
0
dt 1é 2 ù 1 é 1 ù
1 2
\ ò 2t 3 / 2
= -
2 êë t úû
=- = ê-
t êë
ú
úû
Kinetic energy of charged particle = mv
3R 3R R + x2
2
3R 2
For minimum speed of particle so that it does not return to P,
1 1 1 1 1 1 2 2qV 2q ´ l
=- + =- + =- mv = qV or v 2 = =
R 2
R + 3R 2 R 2 R 2 R 2 m m ´ 4e 0
Substituting in (i), we get
ql
2pR lq 1 lq or v =
W= ´ = 2 e0 m
4pe 0 2 R 4e 0
17. The adjacent figure is a case of parallel plate capacitor. The
This work done needs to be supplied by the kinetic energy combined capacitance will be
given to the charge initially.
1 2 lq
\ mv =
2 4e 0

lq
Þ v=
2e 0 m
This is the minimum velocity through which the charge q
must be projected to reach the origin.
ALTERNATE SOLUTION : Potential energy can be found
at the initial point A and final point O. The difference in C = C1 + C2
potential energy has to be provided by the K.E. of the charge k e 0 ( x ´ 1) e0 [(1 - x ) ´ 1)
at A. = +
d d
e0
C= [kx + 1 - x] ... (i)
d
Differentiating the above quation w.r.t. time
dC e 0 dx e
= (k - 1) = 0 (k - 1)v
dt d dt d
dx
where v=
dt
dq dC
We know that q = CV, =V
dt dt
ELECTROSTATICS P-311
From (iii) and (iv)
1 1 é K 2 ld - K 2 xd + K1 xd ù
e0 Þ = ê ú
I= V (k - 1)v dC Ae 0 ( dx ) ë K1K 2 û
d

500 ´ 8.85 ´ 1012 1 1 1


Þ = ´ [ K 2 ld + d ( K1 - K 2 ) x ]
I=
0.01
(11 – 1) × 0.001 dC Ae 0 ( dx ) K1K 2
= 4.425 × 10–9 Amp.
K1K 2 A e 0dx
18. Case (i) When no dielectric : Þ = dC
K 2 ld + d ( K1 - K 2 ) x
e0 A
Given C= To find the capacitance of the whole capacitor, we integrate
d
Case (ii) When dielectric is filled : A small dotted element the above equation.
of thickness dx is considered as shown in the figure. l K1 K 2 Ae 0 dx
C= ò0 K2 ld + d ( K1 - K2 ) x
l dx
= K1K2Ae0 ò0 K 2 ld + d ( K1 - K 2 ) x
l
= K1K2Ae0 éê log[ K 2 ld + d ( K1 - K 2 ) x] ùú
ë d ( K1 - K 2 ) û0

K1K 2 Ae 0
= [log{K 2 ld + d ( K1 - K 2 )l} - log K 2 ld ]
The small capacitance of the dotted portion d ( K1 - K 2 )
1 1 1
= + where dC1 = capacitance of capacitor K1 K 2 Ae0
dC dC1 dC2 = [log{K 2 ld + dK1l - dK 2 l} - log K 2 ld ]
d ( K1 - K2 )
with dielectric K1
dC2 = capacitance of capacitor with dielectric K2. K1 K 2 Ae0 K
Let l,b the length and breadth of the capacitor plate. C= log 1
d ( K1 - K 2 ) K2
Therefore l ´ b = A .
K1 (bdx ) e 0
dC1 =
d' 19.
d é xù
d' = d – x = d ê1 - ú
l ë lû
K1b ( dx) e 0 K1bl ( dx) e0
\ dC1 = -
é xù d (l - x )
d ê1 - ú
ë lû
For sphere of R
K A e ( dx)
= 1 0 q1
d (l - x ) s= \ q1 = s × 4 p R2
4pR 2
K 2 e 0(bdx ) K e bdx
Similarly, dC2 = = 2 0 For sphere of radius 2R
d -d' xd
d -d + q2
l s= Þ q2 = s × 16 p R2
4 p (2 R ) 2
K 2 e 0b.l.dx K 2 e 0Adx
= Total charge
xd xd
1 d (l - x ) xd q = q1 + q2 = 20spR 2
\ = +
dC K1 Ae 0 ( dx ) K 2 Ae 0 ( dx ) note : When the two spheres are connected then the potential
on the two spheres will be same. There will be a
1 é ld - xd xd ù rearrangement of charge for this to happen.
= ê + ú
Ae 0 ( dx ) ë K1 K2 û Let q1' and q2' be the new charges on the two spheres.
P-312 Topic-wise Solved Papers - PHYSICS
Since V1 = V2 Applying Kirchoff's law in the closed loop, we get
1 q1' 1 q2 ' q1 q2 q3
= -6
- -6
+ =0
4pe 0 R 4pe 0 2 R 3 ´ 10 2 ´ 10 2 ´ 10-6
Þ 2q1 – 3q2 + 3q3 = 0 ....(iii)
q2' On solving (i), (ii) and (iii), we get
Þ q1' = ... (ii)
2 q1 = 90 × 10–6 C, q2 = 210 × 10–6 C,
Total charge on the system of two spheres remains constant and q3 = 150 × 10–6 C,
before and after connection. (ii) Amount of electrostatic energy in the system initially
\ q1 ' + q2 ' = q1 + q2 1 2 1
C A (VA ) + C B (VB )
2
Ui = U A + U B =
2 2
q2' 3q2'
+ q2' = s ´ 20pR 2 Þ = s ´ 20pR 2 1 1
2 2 =× 3 × 10–6 (100)2 + × 2 × 10–6 (180)2
2 2
q2' s 5 = 4.74 × 10–2J
Þ = ´
2 3 2 Amount of electrostatic energy stored finally
4p (2 R )

q2' 5s 1 q12 1 q22 1 q32


Þ New charge density on bigger sphere = = Uf = + +
4p (2 R ) 2 6 2 C A 2 CB 2 CC

20. (i) KEY CONCEPT : Use charge conservation to solve 1 (90 ´ 10 -6 )2 1 (210 ´ 10 -6 ) 2 1 (150 ´ 10-6 ) 2
this problem. = + +
2 3 ´ 10-6 2 2 ´ 10-6 2 2 ´ 10 -6
INITIALLY :
1 (150 ´ 10-6 )2
+ = 1.8 × 10–2 J
2 2 ´ 10-6
A B
+ + 21. KEY CONCEPT : When the spheres S1 and S2 come in
+ + contact, there is transference of charges till the potential of
+ + the two spheres becomes equal.
3mF 2mF During first contact
q1 q2
V1 = V2 [q1 charge shifts from S1 to S2]
100V 180V
Charge on capacitor A K (Q - q1 ) Kq1 æ R ö
Þ = Þ q1 = Q ç
qA = 3 × 10–6 × 100 = 3 × 10–4C r R è R + r ÷ø
Charge on capacitor B During second contact again
qB = 2 × 10–6 × 180 = 3.6 × 10–4C V1 = V2
FINALLY :
Kq2
q2 C(2mF) K [Q – (q2 – q1)] =
R
+
+ [(q2 – q1) charges shifts from S1 to S2]
+
é R æ R ö ù
2
Þ q2 = Q ê +ç ÷ ú
êë R + r è R + r ø úû
A B
+ + On third contact again
+ + V1 = V2
+ +
3mF 2mF K [Q - (q3 - q2 )] Kq3
q q3 =
1 r R
Let the charge on capacitor A, C and B be q1, q2 and q3 [(q3 – q2) charge shifts from S1 to S2]
respectively.
By charge conservation. é R æ R ö
2
æ R ö ù
3
Þ q3 = Q ê +ç ÷ +ç ÷ ú
The sum of charge on +ve plate of capacitor A and C should êë R + r è R + r ø è R + rø ú
û
be equal to qA
\ q1 + q2 = 3 × 10–4C ... (i) On nth contact by symmetry
Similarly the sum of charge on –ve plates of capacitor C and V1 = V2
B will be equal to qB K [Q - (qn - qn -1 )] Kqn
\ – q2 – q3 = – 3.6 × 10–4C =
r R
Þ q2 + q3 = 3.6 × 10–4C ... (ii) [(qn – qn – 1) charge shifts from S1 to S2]
ELECTROSTATICS P-313
NOTE : According to law of conservation of energy,
é R æ R ö
2
æ R ö ù
n
Þ qn = Q ê +ç + .... + ç ú loss of gravitational potential energy = gain in electric
÷ ÷
êë R + r è R + r ø è R + rø ú
û potential energy
mgH = qDV
æ R ö é {R /( R + r ) } ù
n = q [V(0) – V (H)]
=Q ç ÷ ê1 - ú
è R + r ø ê 1 - R /( R + r ) ú s
ë û mgH = q [a – { (a 2 + H 2 ) - H }] ... (1)
2e 0
QR é æ R ö

= ê1 - ç ÷ú sq
r êë è R + r øúû From the given relation : = 2mg
2e 0
The electrostatic energy of S2 after n contacts is
Putting this in equation (1), we get,
2
1 ìï QR é æ R ö ù üï
n
1 qn2
1 mgH = 2mg [a - { (a 2 + H 2 ) - H }]
Un = = ´ í ê1 - ç ÷ úý
2 C 2 4pe 0 R ï r ê è R + r ø ú ï
î ë ûþ
or H = 2[a + H - (a 2 + H 2 )]
(\ C = 4pe 0 R )
(b) The limiting value or H = 2a + 2H – 2 (a2 + H 2 )
2
é ì é n ù üù
2 ( a 2 + H 2 ) = H + 2a
lim U n = lim ê 1 ´ 1 ïí QR ê1 - æç R ö÷ ú ïý ú
or
n®¥ x ®¥ ê 2 4pe 0 R ï r ê è R + r ø ú ïú or 4a2 + 4H2 = H2 + 4a2 + 4aH
ë î ë û þû
or 3H2 = 4aH
Q2 R 4a
= or H= [Q H = O is not valid]
2(4pe 0 )r 2 3
ALTERNATE SOLUTION (b) Total potential energy of the particle at height H
Limiting value of energy as n ® ¥. U (x) = mgx + qV (x)
Let us calculate qn when n tends to ¥. qs
= mgx + ( a 2 + x 2 - x)
a 2 e0
For G.P., S¥ = where r1 = common ratio
1 - r1
= mgx + 2mg [ (a 2 + x 2 ) - x] ....(2)
é ù
QR ê 1 ú QR
\ q¥ = ê or q¥ = U( A) = mgH + 2mg é a 2 + H 2 - H ù
R+r ê R úú r êë úû
1-
ë R+r û
= mg é 2 a2 + H 2 - H 2 ù ....(3)
2 2 ëê ûú
æ QR ö
q¥ 1
\ U¥ = =ç ÷ ´
2C è r ø 2 ´ ( 4pe 0 ) ´ ( R ) dU
For equilibrium : =0
dH
Q2 R 2 Q2 R
or U¥ = or U ¥ = a
r 2 ´ 2 ´ 4pe0 R 2 ( 4pe0 ) r 2 This gives : H = \ U min = 3 mga
3
22. (a) KEY CONCEPT : The K.E. of the particle, when it
reaches the disc is zero. From equation (2), graph between U(x) and x is as shown
Given that a = radius of disc, s = surface charge density, above.
q/m = 4e0g/s
Potential due to a charged disc at any axial point situated at
a distance x from O is,
s
V (x) = [ a 2 + x2 - x]
2 e0

s
Hence, V (H) = [ a2 + H 2 - H ]
2e 0

sa
and V(O) =
2e 0
P-314 Topic-wise Solved Papers - PHYSICS
23. Let the particle at some instant be at a point P distant x from
the origin. As shown in the figure, there are two forces of 2 ´ 9 ´ 109 ´ 8 ´ 10 -6 2 ´ 9 ´ 109 ´ 10 -6
repulsion acting due to two charges of + 8 mC. The net force V= -
5 27 5 3
is 2F cos a towards right. + +
2 2 2 2
Similarly there are two forces of attraction due to two charges
of – 1 mC. The net force due to these force is 2F' cos b é8 1ù 4
towards left. = 2 × 9 × 109 × 10–6 ê - ú = 2.7 ´ 10 V
ë 4 2û
Kinetic energy is required to overcome the force of repulsion
5
from µ to x = .
2
The work done in this process is W = q (V)
5
where V = p.d between ¥ and x = .
2
\ W = 0.1 × 10–6 × 2.7 × 104 = 2.7 × 10–3 J
1
By energy conservation mV02 = 2.7 × 10–3
2

The net force on charge 0.1 µC is zero when 1


Þ ´ 6 ´ 10-4 V02 = 2.7 ´ 10-3
2F cos a = 2F' cos b 2
Þ V0 = 3 m/s
K ´ 8 ´ 10 -6 ´ 0.1 ´ 10-6 x K.E. at the origin
´ Potential at origin
2 27
æ 2 27 ö x2 +
ç x + 2 ÷ 2 2 ´ 9 ´ 109 ´ 8 ´ 10 -6 2 ´ 9 ´ 109 ´ 10-6
è ø Vx = 0 = -
27 3
2 2
K ´ 1 ´ 10-6 ´ 0.1 ´ 10-6 x
= ´ = 2.4 × 104
2 3
æ 2 3ö x2 + Again by energy conservation
ç x + 2÷ 2
è ø
K.E. = q éVx= 5 - Vx = 0 ù
êë 2
úû
8 1 \ K.E. = 0.1 × 10–6 [2.7 × 104 – 2.4 × 104]
Þ =
é 2 27 ù
3/ 2
é 2 3ù
3/ 2 = 0.1 × 10–6 × 0.3 × 104
êë x + 2 úû êë x + 2 úû = 3 × 10–4 J
1 q 2 é -3 3 1 ù
2/3 2/3 24. Wexternal = DPE = ê + - ú´4
ìï 3 é 2 3 ù3 / 2 üï ìï é 27 ù üï
3/ 2 4pe 0 a ë 1 2 3û
Þ í2 ê x + ú ý = íêx2 + ú ý
ïî ë 2û ï ïî ë 2û ï
þ þ 1 q2 4
= . [3 3 - 3 6 - 2]
4pe 0 a 6
æ 3ö 27 27 12
Þ 4 ç x2 + ÷ = x2 + Þ 3x2 = - ALTERNATE SOLUTION
è 2ø 2 2 2
KEY CONCEPT : There are eight charge in total and for
potential energy consideration we take charges in pairs.
5
Þ x=± The total number of charged pairs will be 8C2 = 28. Out of
2 these
This means that we need to move the charge from – ¥ to (a) 12 pairs of unlike charges are at a separation a.

5 (b) 12 pairs of like charges are at a separation 2a .


. Thereafter the attractive forces will make the charge
2 (c) 4 pairs unlike charges are at a separation 3a .
move to origin.
12(q) ( - q) 12q 2
For (a) above, P.E. = =-
5 4pe 0 a 4pe 0 a
The electric potential of the four charges at x = is
2
ELECTROSTATICS P-315
Work done by electric field
12(q)(q) 12 q 2 a
For (b) above, P.E. = = ´ 0.707
4pe 0 2 a 4 pe0 a W = ( q0 E ) a cos 45o = q0 E ×
2

4(q )( -q ) 4q 2 q0 (s1 - s 2 ) a
For (c) above, P.E. = =- ´ 0.577 \ W=
4 pe0 3a 4 pe 0 a 2 e0
Total potential energy of system = U 27. LIQUID BUBBLE : The potential of the liquid bubble is V.
1 q
æ q2 ö Þ V= ... (1)
\ U = -5.824 ç ÷ 4pe 0 a
è 4pe 0 a ø where q is the charge on the liquid bubble.
æ q2 ö
\ Binding energy of system = 5.824 ç ÷
è 4pe 0 a ø

æ q2 ö
\ Workdone to separate the charges = 5.824 ç ÷
è 4pe 0 a ø
25. (a) Potential energy of the dipole-charge system
Ui = 0 (since the charge is far away) LIQUID DROPLET
The volume of liquid droplet = Volume (of the liquid)
1 p in liquid bubble.
Uf = – Q × [at a point (d, w)
4pe 0 d 2 4 3 4 4
p r = p ( a + t )3 - p a 3
1 pQ 3 3 3
\ K.E. = | Uf – Ui | = or, r3 = a3 + t3 + 3a2t + 3at2 – a3
4pe 0 d 2
or, r3 = 3a2t
(b) Electric field at origin due to dipole (Q t is very small as compared to a)
ur 1 2p $ or, r = [3a2t]1/3 ... (iii)
E= i
4pe 0 d 3
Thus, force on charge Q is given by NOTE : By charge conservation we can conclude that
charge on liquid bubble is equal to charge on liquid droplet
ur ur 2 pQ $ Let charge on liquid droplet is q.
F = QE = i \ Potential on liquid droplet
4pe 0 d 3
1 q
s Vdroplet =
26. Electric field due to S1, E1 = 1 4pe 0 r
e0
1 4pe 0V ´ a
or, Vdroplet = ´ [From (i) and (ii)]
s 4pe 0 [3a 2t ]1/ 3
Electric field due to S2, E2 = 2
e0
1/ 3
éaù
or, Vdroplet = V ê ú
ë 3t û
MATCH THE FOLLOWING :

1. + – Q

– +
M

P+ –
The electric field at M due to the charges at the corners of
regular hexagon is as shown
E–
E+
120°
\ E = E1 – E2 E– 120°
E+ 120° E+
s1 - s 2
=
e0
(Q s1 > s2 ) E–
P-316 Topic-wise Solved Papers - PHYSICS
Here | E+ | = | E- | . The symmetry of the situation shows (B) is the correct option.
that E = 0 at M. When the system of charges rotate, we get a current I1 due
Therefore (A) is the correct option. to negative charges and another current I due to positive
The electric potential due to all the charges at M is zero. charges. The magnitude of the magnetic at M due to the
Therefore (B) is incorrect option. currents is different. Therefore B ¹ 0 and m ¹ 0.
When the system of charges is rotated about line PM, the (C) is incorrect option
net current will be zero. (D) is the correct option.
Therefore the magnetic field at M is zero. (s) 2a
(C) is the correct option. – –
+
When magnetic field is zero, then m = 0
a
(D) is incorrect option. M
P Q
(q) P – + –

– + – + – + The electric field at M due to all the charges is zero because


the electric field due to different charges cancel out in pairs.
M (A) is the correct option.
Q The potential at M due to the charges is
The electric field due to the inner most positive and negative é +q q æ q öù
V =kê + -4 ¹0
é qù a / 2 a / 2 ç 5a ÷ ú
charges at M is E1 = 2 ê k 2 ú towards left. The electic field ê ç ÷ú
ë r û ëê è 4 ø úû
due to the next positive and negative charges at M is (B) is the correct option.
é When the whole system is set into rotation with a constant
q ù
E2 = 2 ê k towards right. The electric field due to angular velocity about the line PQ we get three loops in

ëê (2r ) ûú which current is flowing.
the outermost positive and negative charges at M is The magnetic field due to these currents produce a resultant
magneic field at M which is not equal to zero. Therefore a
é q ù net magnetic dipole moment will be produced.
E3 = 2 ê k 2ú
towards left. Clearly the vector sum of
(C) is an incorrect option.
ëê (3r ) ûú
(D) is correct option.
these three electric field is not zero.
(A) is incorrect option. P
The electric potential due to the charges at M + –
é +q -q q q q q ù (t) + +
=kê + - + - =0 M– –
ë r r 2r 2r 3r 3r úû Q
(B) is incorrect option.
There will be a net electric field due to the arrangement of
The net current due to the innermost positive and negative
charges is zero. Similarly the net current due to other charges charges at M towards the right side.
in pairs is zero. Therefore the magnetic field at M is zero. (A) is an incorrect option.
Also the magnetic moment is zero. The electic potential at M will can out in pairs by positive
(C) is the correct option and negative charges, due to symmetrical arrangement of
(D) is incorrect option. charges.
(B) is an incorrect option.
(r) When the system of charges rotates about PQ, the net
+ Q
– current is zero due to symmetrical arrangement of charges.
Therefore B = 0 and m = 0

M – + (C) is the correct option.
(D) is the incorrect option.
P
+ COMPREHENSION BASED Q UESTIONS :
The net electric field due to negative charges in the inner
circle is zero. Similarly the net electric field due to positive 1. (a) When the point of observation is on the surface of
charges in the outercircle is zero. sphere then the whole charge inside the sphere (when
(A) is the correct option. distributed symmetrically about the centre) behaves as
The electric potential due to negative charges at M is a point charge on the centre. Therefore until the charge
different from the electric potential due to positive charges distribution is symmetrical about the centre it does not
at M. Therefore the electric potential at M is not equal to matter what is the ratio a/R. The electric field remains
zero. constant and is equal to
ELECTROSTATICS P-317

1 Ze For r = R :
E= . . The total charge enclosed in the sphere of radius R is
4π Îo R
R
R 3+ a
P
R
ò0
Q = 4pk x 2 + a dx = 4pk
3+ a .
a
\ The electric field at r = R is

1 4pkR3+ a 1 4pk 1+ a
E1 = = R
4pe 0 (3 + a ) R 2 4 pe0 3 + a
2. (b) For a = 0, the graph is as shown. The equation for the For r = R/2 :
graph line is
The total charge enclosed in the sphere of radius R/2 is
r(r)
R/2
4pk ( R / 2)3+ a
d Q' = ò0 4pk x 2 + a dx =
3+ a

\ The electric field at r = R/2 is


1+ a
r 1 4pk ( R / 2)3+ a 1 4 pk æ R ö
O
Fig (1)
R E2 = = ç ÷
4 pe 0 3 + a ( R / 2) 2 4pe 0 3 + a è 2 ø
r
r = d– dr 1
R Given, E2 = E1
The charge in the dotted element shown in Fig (2) is 8
1+ a
1 4 pk æ R ö 1 1 4 pk 1+ a
\ ç ÷ = ´ R
r dr
4pe0 (3 + a ) è 2 ø 2 3 4pe 0 3 + a
Þ 1+a=3 Þ a=2
2. 3

Fig (2) kq 2 é kq 2 1 ù q 2
dq = r × 4pr2dr Felectric = + 2ê ´ ú = ´ constant
2a 2 êë a
2 2 úû a 2
R R
æ ö
d 4πd 3
\ dq = èç d - r÷ø 4pr2dr Þ Ze = ò 4πdr dr – ò
2
r dr
R o R
o 2
kq kq
2
3 4
R 4πd R 2
Ze = 4πd - 2a a
2
3 R 4 2
kq
Ze. 1 1 1 3Ze 2
\ 3
= – = \d= a
4πdR 3 4 12 πR 3
3. (c) If the volume charge density is constant then E µ r.
ASSERTION & REASON TYPE QUESTIONS :
1. (a) Both the statements are true and statement-2 is the
correct explanation of statement-1 As the system of charges and planar film is in equlibrium,

INTEGERVALUECORRECTTYPE: therefore

1. Let us consider a spherical shell q2


´ constant = ga ´ constant
of radius x and thickness dx. The a2
volume of this shell is 4px2(dx).
dx
The charge enclosed in this 1/ 3
x æ q2 ö
spherical shell is a =kç ÷
\ ç g ÷
dq = (4px 2 )dx ´ kx a è ø
\ N =3
\ dq = 4pkx 2 + a dx .
P-318 Topic-wise Solved Papers - PHYSICS
3. 6 We suppose that the cavity is filled up by a positive as (b) The electric field created due to the spherical
well as negative volume charge of r. So the electric negative charge density
field now produced at P is the superposition of two rR
electric fields. E2 =
(a) The electric field created due to the infinitely long 96e 0 directed towards the –Y direction.
solid cylinder is \ The net electric field is
rR 1 é 23rR ù
E1 = directed towards the +Y direction E = E1 - E2 = ê ú
4e 0 6 ë 16e 0 û

4. (d) For equilibrium of charge Q


WAB
1. (a) We know that = VB - V A Q´Q Qq
q K +K =0
2
(2x) x2
2J
\ VB - V A = = 0.1J/C = 0.1V x x
20 C
Q q Q
2. (b) The equivalent capacitance of n identical capacitors of
capacitance C is equal to nC. Energy stored in this Q
Þ q=-
capacitor 4
1 1 5. (a) For an isolated sphere, the capacitance is given by
E= (nC )V 2 = nCV 2
2 2 1
C = 4p Î0 r = ´ 1 = 1.1 ´ 10-10 F
C 9 ´ 109
C 6. (a) The flux entering an enclosed surface is taken as
n times negative and the flux leaving the surface is taken as
C positive, by convention. Therefore the net flux leaving
V the enclosed surface = f2 - f1
V \ the charge enclosed in the surface by Gauss’s law is
Alternatively q =Î0 (f 2 - f1 )
Each capacitor has a potential difference of V between 7. (b) The capacitance of a parallel plate capacitor in which a
the plates. metal plate of thickness t is inserted is given by
1 eo A eo A
So energy stored in each capacitor = CV 2 . C= . Here t ® 0 \ C =
2 d -t d
é1 2ù
8. (c) Electric potential due to charge Q placed at the centre
\ Energy stored in n capacitor = ê CV ú ´ n of the spherical shell at point P is
ë2 û
3. (b) Both the charges are identical and placed symmetrically
about ABCD. The flux crossing ABCD due to each R/2
Q P
1é q ù
charge is 6 ê Î ú but in opposite directions. Therefore R q
ë 0û
the resultant is zero.
1 Q 1 2Q
V1 = =
C 4pe o R / 2 4pe o R
D Electric potential due to charge q on the surface of the
spherical shell at any point inside the shell is
.
q 1 q
V2 =
q
. 4pe o R
\ The net electric potential at point P is
B
1 2Q 1 q
A V = V1 + V2 = +
4pe o R 4pe o R
ELECTROSTATICS P-319
9. (d) The work done is stored as the potential energy. The x is distance between the spheres. After first operation
potential energy stored in a capacitor is given by Q
charge on B is halved i.e and charge on third sphere

U=
1 Q 2 1 8 ´ 10
= ´
( )
-18 2
= 32 ´ 10 –32 J Q
2

2 C 2 100 ´ 10-6 becomes . Now it is touched to C, charge then


2
qq equally distributes them selves to make potential same,
10. (b) Force on charge q1 due to q2 is F12 = k 1 2
b2 hence charge on C becomes æç Q + ö÷ =
Q 1 3Q
.
è 2ø 2 4
q1q3
Force on charge q1 due to q3 is F13 = k
a2 æ 3Q ö æ Q ö
çè ÷ø çè ÷ø
The X - component of the force (Fx) on QC' QB' 4 2 3 Q2
\ Fnew µ = =
2 2
q1 is F12 + F13 sin q x x 8 x2

q1q2 q1q2 3
\ Fx = k +k sin q or Fnew = F
8
b2 a2
1 2 kQq 1 kqQ r
q2 q3 13. (d) mv = Þ m(2v )2 = Þ r'=
\ Fx µ + sin q 2 r 2 r' 4
b2 a2 14. (b) Net field at A should be zero
2 E1 + E2 = E3

kQ ´ 2 kQ kq
\ + =
F12 a2 ( 2 a)2 2
æ a ö
çè ÷
F13 sin q 2ø
q
E3 –Q –Q B
E1
A
E2
F13
F13cos q 2 E1 A –Q –Q
E
11. (d) The total volume remains the same before and after Q 2 Q Q
stretching. Þ + = 2q Þ q = (2 2 + 1)
1 2 4
Therefore A ´ l = A ' ´ l ' 15. (c) At equilibrium, electric force on drop balances weight
of drop.
Here l ' = 2l
mg
A´l A´l A . qE = mg Þ q =
\ A' = = = E
l' 2l 2
Percentage change in resistance 9.9 ´ 10-15 ´ 10
q= 4
= 3.3 ´ 10 -18 C
l' l 3 ´ 10
R f - Ri r -
= ´ 100 = A ' A ´ 100 - K 2q K 8q 1 4
l 16. (b) + =0 Þ =
Ri ( x - L) 2
x 2
( x - L) 2
x2
r
A
1 2
or = Þ x = 2 x - 2 L or x = 2L
éæ l ' A ö ù é æ 2l A ö ù x-L x
= êç ´ ÷ - 1ú ´ 100 = êç A ´ ÷ - 1ú ´ 100
ëè A ' l ø û ëêè 2 l ø úû 17. (a) q q
= 300%
C
B
12. (d) ×
r r R R
1 2
QAQC
Fµ d
x2
P-320 Topic-wise Solved Papers - PHYSICS
At (1) using, potential (V1 ) = Vself + Vdue to (2) 1 2
22. (a) eV = mv
2
1 éq q ù
Þ V1 = ê - ú
4pe 0 êë R R 2 + d 2 úû 2eV 2 ´ 1.6 ´ 10-19 ´ 20
Þv= =
m 9.1 ´ 10 -31
At (2) using potential (V2 ) = Vself + Vdue to (1)
= 2.65 ´10 6 m / s
1 é-q q ù
Þ V2 = ê + ú
4pe 0 êë R R 2 + d 2 úû +Q1 +Q2

DV = V1 - V2 23. (c)
r1 r2
1 éq q q q ù
= ê + - - ú A B
4pe 0 êë R R R2 + d 2 R2 + d 2 úû
After connection, V1 = V2
1 éq q ù Q1 Q Q Q
= ê - ú ÞK =K 2 Þ 1= 2
2pe 0 êë R R 2 + d 2 úû r1 r2 r1 r2
18. (b) As n plates are joined, it means (n – 1) combination The ratio of electric fields
joined in parallel.
\ resultant capacitance = (n – 1) C Q1
K
E1 r12 E Q r2
19. (c) P = Þ 1 = 21 ´ 2
E2 Q E2 r1 Q2
T cos q K 22
r2
q T
s
F = Eq = q
e0 K E r ´ r2 E r 2
Þ 1 = 1 2 Þ 1 = 2 =
T sin q 2
E2 r1 ´ r2 E2 r1 1

Since the distance between the spheres is large as


mg compared to their diameters, the induced effects may
be ignored.
s
T sin q = .q .... (i) 24. (c) Y
e0 K
T cos q = mg .... (ii)
Dividing (i) by (ii), A(Ö2,Ö2)
®
r1
sq
tan q = \ s µ tan q
e 0 K . mg
O X
20. (c) Applying conservation of energy, (0,0) ®
r2 B (2,0)
1 2m. s. Dt
CV 2 = m. s Dt ; V =
2 C The distance of point A ( )
2, 2 from the origin,

21. (c) ur 2 2
+q OA = | r1 | = ( 2) + ( 2) = 4 = 2 units.
F1 The distance of point B(2, 0) from the origin,
E1 uur 2 2
OB = | r2 | = (2) + (0) = 2 units.
F2
E2 1 Q
Now, potential at A, VA = .
–q 4p Î0 (OA)

The electric field will be different at the location of the 1 Q


two charges. Therefore the two forces will be unequal. Potential at B, VB = .
4p Î0 (OB)
This will result in a force as well as torque.
ELECTROSTATICS P-321
\ Potential difference between the points A and B is 27. (a) The potential energy of a charged capacitor before
given by
Q2
removing the dielectric slat is U = .
1 Q 1 Q 2C
VA – VB = . - .
4p Î0 OA 4p Î0 OB The potential energy of the capacitor when the
dielectric slat is first removed and the reinserted in the
Q æ 1 1 ö Q æ 1 1ö
= ç - ÷ = ç - ÷ Q2
4p Î0 è OA OB ø 4p Î0 è 2 2 ø gap between the plates is U =
2C
Q There is no change in potential energy, therefore work
= ´ 0 = 0. done is zero.
4p Î0
28. (b) Electronic charge does not depend on acceleration due
25. (a) As shown in the figure, the resultant electric fields to gravity as it is a universal constant.
before and after interchanging the charges will have So, electronic charge on earth
the same magnitude, but opposite directions. = electronic charge on moon
Also, the potential will be same in both cases as it is a \ Required ratio = 1.
scalar quantity. 29. (c)

q q
A B

®
E The given capacitance is equal to two capacitances
D C
connected in series where
-q -q

-q -q and
A ® B
E
The equivalent capacitance Ceq is
1 1 1 d d 2d
= + = + =
Ceq C1 C2 9 Î0 A 9 Î0 A 9 Î0 A
D C
q q 9 Î0 A 9
\ Ceq = = ´ 9 pF = 40.5 pF
2 d 2
30. (a) The electric field inside a thin spherical shell of radius
20 R has charge Q spread uniformly over its surface is
26. (a) Here, V(x) = 2 volt
x -4 zero.

dV d æ 20 ö Q +
We know that E = - =- ç ++ + +
dx è x 2 - 4 ÷ø
+

dx R
+

+ + +
+++

Q
40 x E=k
or, E = + r2
++

( x 2 - 4) 2
+ +

E=0
+

At x = 4 mm ,
+

+ + +
++

40 ´ 4 160 10
E= + =+ = + volt / mm. Q
2
(4 - 4) 2 144 9 Outside the shell the electric field is E = k 2 . These
r
r
Positive sign indicates that E is in +ve x-direction. characteristics are represented by graph (a).
P-322 Topic-wise Solved Papers - PHYSICS
\ The electric field at point p inside the sphere at a
WPQ
= (VQ - VP ) distance r1 from the centre of the sphere is
31. (c)
q
é Q 4ù
r
Þ WPQ = q (VQ - VP ) 1 êë R 4 1 úû = 1 Q r 2
E= 1
= (– 100 × 1.6 × 10–19) (– 4 – 10) 4p Î0 r12 4p Î0 R 4
= +2.24 × 10–16J
32. (d) Let F be the force between Q and Q. The force between 35. (c) Let us consider a differential element dl. charge on this
q and Q should be attractive for net force on Q to be element.
j
zero. Let F ¢ be the force between Q and q . The resultant

+
+
of F ¢ and F ¢ is R. For equilibrium

+
+ dl

+
+ dq
+
Q q + dE q
+
cos q
i
O
dE
R = 2F' dE sin q
l F¢ æ q ö
dq = ç ÷ dl
è pr ø
q
= (rd q) (Qdl = rd q)
pr
q Q
F¢ æqö
= ç ÷dq
F è pø
Electric field at O due to dq is
r r
R + F = 0, 2 F ' = -F 1 dq 1 q
dE = . = . dq
4p Î0 r 2 4 p Î0 pr 2
Qq Q2 Q
2 ´ k 2 = -k Þ = -2 2 The component dE cos q will be counter balanced by
l ( 2 l) 2 q
another element on left portion. Hence resultant field at O is
33. (a) Statement 1 is true.
the resultant of the component dE sinq only.
Statement 2 is true and is the correct explanation of (1)
p
q
\ E = ò dE sin q = ò 4 p2 r 2 Î sin qd q
34. (b) 0 0
R
q
=
2 2
[ - cos q]p0 = q
(+1 + 1)
4p r Î0 2 2
4p r Î0
q
x dx
= 2 2
2p r Î0
The direction of E is towards negative y-axis.
r q
\ E=- 2 2 ˆj
2p r Î0
Let us consider a spherical shell of thickness dx and 36. (d) Let us consider a spherical shell of radius x and
radius x. The volume of this spherical shell = 4px 2 dx . thickness dx.
Charge on this shell
The charge enclosed within shell
æ5 xö 2
é Q.x ù 2 4Q 3 dq = r.4px 2 dx = r0 ç 4 - R ÷ .4px dx
= ê 4 ú[4px dx] = 4 x dx è ø
ë pR û R \ Total charge in the spherical region from centre to r
The charge enclosed in a sphere of radius r1 is (r < R ) is
r1 r1 r
4Q é x 4 ù æ5 xö
4Q Q 4 q = ò dq = 4 pr0 ò ç - ÷ x 2 dx
ò
3
= x dx = ê ú = 4 r1 è4 Rø
R4 4 4
R ëê ûú 0
R 0
0
ELECTROSTATICS P-323
By Gauss's theorem
1 q
E= ....(ii)
dx 4pe 0 r 2
x
From (i) and (ii),
Q = –8 pe0ar3
Þ dq = – 24pe0ar2 dr
dq
Charge density, r = = – 6e0a
4pr 2dr
39. (d) Potential at point A,
é 5 r3 1 r 4 ù
3æ5 r ö
= 4pr0 ê 4 . 3 - R . 4 ú = pr0r ç - ÷ 2kq 2kq
ëê ûú è3 Rø VA = -
a a 5
1 q
\ Electric field at r, E = . kq
4p Î0 r 2 \ (potential due to each q = and
a

1 pr0 r 3 æ 5 r ö r r æ5 r ö - kq
= . 2 ç - ÷= 0 ç - ÷ potential due to each – q = )
4pÎ0 r è 3 R ø 4 Î0 è 3 R ø a 5
37. (d) At any instant
2a
T cosq = mg ....(i) q q
A
T sin q = Fe ....(ii)
sin q Fe
Þ = Þ Fe = mg tan q
cos q mg
2a
B
kq 2
Þ = mg tan q Þ q2 µ x2 tan q
x2

x –q –q
sin q =
2l
For small q, sin q » tan q
\ q2 µ x 3 Potential at point. B,
VB = 0
( Q Point B is equidistant from all the four charges)
\ Using work energy theorem,
l l (WAB )electric = Q (VA - VB )
cosq
2 kqQ é 1 ù
= 1-
q
sinq a êë ú

æ 1 ö 2Qq é 1 ù
=ç ÷ ê1- ú
è 4pe 0 ø a ë 5û
r
40. (c) E inside the charged sphere
dq dx
µ x2
RP
Þ q
dt dt

\
dq
= const. O P Q
dt

3
\ q µ x2.v Þ x ax 2 .v r
2 Ein = 0 …(i)
Þ v µ x–1/2 [Q q2 µ x3] r
38. (c) Electric field E on the surface of the charge sphere
r 1 q r 1
df Es = i.e., Es µ nˆ …(ii)
E=- = – 2ar ....(i) 2
dr 4p Î0 R R2
P-324 Topic-wise Solved Papers - PHYSICS
r
E on any point away from the uniformly charged C1 C2
+ – – +
sphere is given 43. (b)
r 1 q 120 V 200 V
E= nˆ For potential to be made zero, after connection
4p Î0 r 2

r 1 é qù
E µ 2 nˆ … (iii)
120 C1 = 200 C2
êëQ C = v úû
r
Q R is the radius of the sphere, which is constant, thus Þ 3C1 = 5C2
r
E is maximum and constant at the surface of the sphere. y
But decreases on moving away from the surface of the 44. (a)
uniformally charged sphere.
F F
42. (c) The potential energy at the centre of the sphere
x
q a a q
3 KQq
Uc =
2 R
The potential energy at the surface of the sphere Þ F sin q F sin q Þ Fnet = 2F cosq
KqQ
Us =
R 2Fcos q
Now change in the energy
æ qö
DU = Uc - U s 2kq ç ÷
è 2ø y
Fnet = .
( )
KQq é 3 ù KQq 2
= -1 = y2 + a2 y2 + a2
R êë 2 úû 2R
4 3
Where Q = r.V = r. pR æ qö
3 2kq ç ÷ y
è 2ø kq2 y
Fnet = Þ
2 K pR 3 rq ( y 2 + a 2 )3 / 2 a3
DU =
3 R
So, F µ y
3
2 1 pR rq 45. (d) L L
DU = ´
3 4p Î0 R
O A B
dx
2
R rq
DU = Electric potential is given by,
6 Î0
æ qö
dx
1 çè L ÷ø
2L 2L q
kdq = ln(2)
V= ò x
= ò 4 pe 0 x 4pe 0 L
L L
13 Current Electricity
FILL IN THE BLANKS : MCQ's WITH ONE CORRECT ANSWER :

1. An electric bulb rated for 500 watts at 100 volts is used in a 1. The temperature coefficient of resistance of a wire is 0.00125
circuit having a 200 volts supply. The resistance R that per °C. At 300 K, its resistance is 1 ohm. This resistance of
the wire will be 2 ohm at.
must be put in series with the bulb, so that the bulb delivers
(a) 1154 K (b) 1100 K (1980)
500 watt is ..........ohm. (1987 - 2 Marks) (c) 1400 K (d) 1127 K
2. The equivalent resistance between points A and B of the 2. A constant voltage is applied between the two ends of a
circuit given below is ......W . (1997 - 2 Marks) uniform metallic wire. Some heat is developed in it. The heat
developed is doubled if (1980)
(a) both the length and the radius of the wire are halved.
2R 2R R B (b) both the length and the radius of the wire are doubled.
A
(c) the radius of the wire is doubled.
(d) the length of the wire is doubled.
3. The electrostatic field due to a point charge depends on the
3. In the circuit shown below, each battery is 5V and has an
1
internal resistance of 0.2 ohm. distance r as 2 . Indicate which of the following quantities
r
shows same dependence on r. (1980)
(a) Intensity of light from a point source.
V (b) Electrostatic potential due to a point charge.
(c) Electrostatic potential at a distance r from the centre of a
charged metallic sphere. Given r < radius of the sphere.
(d) None of these
The reading in the ideal voltmeter V is ..... V. (1997 - 2 Marks) 4. In the circuit shown in fig the heat produced in the 5 ohm
resistor due to the current flowing through it is 10 calories
TRUE / FALSE :
per second. (1981- 2 Marks)
1. In an electrolytic solution the electric current is mainly due
4W 6W
to the movement of free electrons. (1980)
2. Electrons in a conductor have no motion in the absence of
a potential difference across it. (1982 - 2 Marks) 5W
3. The current –voltage graphs for a given metallic wire at The heat generated in the 4 ohms resistor is
two different temperatures T1 and T2 are shown in the figure. (a) 1 calorie / sec (b) 2 calories /sec
(1985 - 3 Marks) (c) 3 calories /sec (d) 4 calories /sec
5. The current i in the circuit (see Fig) is (1983 - 1 Mark)
T1
i
I T2
30W 30W
2V
V 30W
The temperature T2 is greater than T1.
P-326 Topic-wise Solved Papers - PHYSICS
1 1 11. In the given circuit, with steady current, the potential drop
(a) amps (b) amps across the capacitor must be (2001S)
45 15
1 1
(c) amps (d) amps V R
10 5
6. A piece of copper and another of germanium are cooled
from room temperature to 80° K. The resistance of C
(a) each of them increases (1988 - 1 Mark)
V
(b) each of them decreases
(c) copper increases and germanium decreases 2V 2R
(d) copper decreases and germanium increases
7. A battery of internal resistance 4W is connected to the
network of resistances as shown. In order that the maximum (a) V (b) V/2
power can be delivered to the network, the value of R in W (c) V/3 (d) 2V/3
should be (1995S) 12. A wire of length L and 3 identical cells of negligible internal
R
resistances are connected in series. Due to the current, the
R
temperature of the wire is raised by DT in a time t. A number
E R 6R N of similar cells is now connected in series with a wire of
R
the same material and cross-section but of length 2L. The
4W R 4R temperature of the wire is raised by the same amount DT in
the same time t. the value of N is (2001S)
(a) 4 (b) 6
(c) 8 (d) 9
4
(a) (b) 2 13. In the given circuit, it is observed that the current I is
9
independent of the value of the resistance R6. Then the
8 resistance values must satisfy (2001S)
(c) (d) 18
3
8. A steady current flows in a metallic conductor of non-
uniform cross-section. The quantity/quantities constant I
R5
R1 R6 R3
along the length of the conductor is/are (1997C - 1 Mark)
(a) current, electric field and drift speed.
(b) drift speed only R2 R4
(c) current and drift speed
(d) current only
9. A parallel combination of 0.1 M W resistor and a 10 mF (a) R1R2R5 = R3R4R6
capacitor is connected across a 1.5 V source of negligible
resistance. The time required for the capacitor to get charged 1 1 1 1
up to 0.75 V is approximately (in seconds) (b) + = +
R 5 R6 R1 + R2 R3 + R 4
(1997C - 1 Marks)
(a) ¥ (b) loge2 (c) R1R4 = R2R3
(c) log102 (d) Zero (d) R1R3 = R2R4 = R5R6
10. In the circuit P ¹ R , the reading of the galvanometer is 14. The effective resistance between points P and Q of the
same with switch S open or closed. Then (1999 - 2 Marks) electrical circuit shown in the figure is (2002S)
2R 2R

P Q 2R
P Q
r r
S 2R 2R
2R

2 Rr 8R ( R + r )
R (a) (b)
G R+r 3R + r
5R
(c) 2r + 4R + 2r
(d)
2
15. A 100 W bulb B1, and two 60 W bulb B2 and B 3, are
V connected to a 250 V source, as shown in figure. Now W1,
(a) IR = IG (b) IP = IG W2 and W3 are the output powers of the bulbs B1, B2 and B3,
(c) IQ = IG (d) IQ = IR respectively. Then (2002S )
CURRENT ELECTRICITY P-327
18. The three resistance of equal value are arranged in the
different combinations shown below. Arrange them in
B1 B2
increasing order of power dissipation. (2003S)

B3 i
i

250V (I) (II)

(a) W1 > W2 = W3 (b) W1 > W2 > W3


i i
(c) W1 < W2 = W3 (d) W1 < W2 < W3
16. Express which of the following set ups can be used to verify
Ohm’s law? (2003S) (III) (IV)
(a) III < II < IV < I (b) II < III < IV < I
(c) I < IV < III < II (d) I < III < II < IV
19. Shown in figure is a Post Office box. In order to calculate
the value of external resistance, it should be connected
between (2004S)
(a)
C B A

C' B'
(b)
(a) B' and C' (b) A and D
(c) C and D (d) B and D
20. Six identical resistors are connected as shown in the figure.
The equivalent resistance will be (2004S)
R
P Q

(c)
R R
R
R R

R
(a) Maximum between P and R
(d) (b) Maximum between Q and R
(c) Maximum between P and Q
(d) All are equal
21. A capacitor is charged using an external battery with a
17. In the shown arrangement of the experiment of the meter resistance x in series. The dashed line shows the variation
bridge if AC corresponding to null deflection of of ln I with respect to time. If the resistance is changed to
galvanometer is x, what would be its value if the radius of 2x, the new graph will be (2004S)
the wire AB is doubled? (2003S)
S
R
R1 R2 1n I

G Q
P
A x C B t
(a) x (b) x /4 (a) P (b) Q
(c) 4 x (d) 2 x (c) R (d) S
P-328 Topic-wise Solved Papers - PHYSICS
22. Find out the value of current through 2W resistance for the (a) 0 (b) 54 mC
given circuit. (2005S) (c) 27mC (d) 81 mC
29. Figure shows three resistor configurations R1, R2 and R3
5W 10 W connected to 3V battery. If the power dissipated by the
10 V 20 V configuration R1, R2 and R3 is P1, P2 and P3, respectively,
2W then –
(a) zero (b) 2 A
(c) 5 A (d) 4 A
23. A 4 mF capacitor, a resistance of 2.5 MW is in series with
12 V battery. Find the time after which the potential difference
across the capacitor is 3 times the potential difference across
the resistor. [Given ln(2) = 0.693] (2005S)
(a) 13.86s (b) 6.93s 3V 3V
(c) 7s (d) 14s
24. A moving coil galvanometer of resistance 100 W is used as
an ammeter using a resistance 0.1 W. The maximum deflection
current in the galvanometer is 100 mA. Find the minimum
current in the circuit so that the ammeter shows maximum
deflection (2005S)
(a) 100.1 mA (b) 1000.1 mA R1
R2
(c) 10.01 mA (d) 1.01 mA
25. An ideal gas is filled in a closed rigid and thermally insulated
container. A coil of 100 W resistor carrying current 1 A for 5
minutes supplies heat to the gas. The change in internal
energy of the gas is (2005S)
(a) 10 kJ (b) 30 kJ
(c) 20 kJ (d) 0 kJ
26. If a steady current I is flowing through a cylindrical element
ABC. Choose the correct relationship 3V
A 2r B
r C
I

l/2
l/2
(a) V AB = 2VBC R3
(b) Power across BC is 4 times the power across AB
(c) Current densities in AB and BC are equal
(d) Electric field due to current inside AB and BC are equal (2008)
27. A resistance of 2W is connected across one gap of a metre- (a) P1 > P2 > P3 (b) P1 > P3 > P2
bridge (the length of the wire is 100 cm) and an unknown
resistance, greater than 2W, is connected across the other (c) P2 > P1 > P3 (d) P3 > P2 > P1
gap. When these resistances are interchanged, the balance 30. Incandescent bulbs are designed by keeping in mind that
point shifts by 20 cm. Neglecting any corrections, the the resistance of their filament increases with the increase
unknown resistance is in temperature. If at room temperature, 100 W, 60 W and
(a) 3W (b) 4W (2007) 40 W bulbs have filament resistances R100, R60 and R40,
(c) 5W (d) 6W
28. A circuit is connected as shown in the figure with the switch respectively, the relation between these resistances is
S open. When the switch is closed, the total amount of
1 1 1
charge that flows from Y to X is (2007) (a) R = + (b) R100 = R40 + R60 (2010)
100 R40 R60
3mF 6m F
X
1 1 1
(c) R100 > R60 > R40 (d) > >
R100 R60 R40
S 31. To verify Ohm’s law, a student is provided with a test resistor
3W 6W RT, a high resistance R1, a small resistance R2, two identical
Y galvanometers G1 and G2, and a variable voltage source V.
The correct circuit to carry out the experiment is (2010)
9V
CURRENT ELECTRICITY P-329

G1
1. Capacitor C1 of capacitance 1 micro-farad and capacitor C2
R2
of capacitance 2 microfarad are separately charged fully by
a common battery. The two capacitors are then separately
(a) RT R1
allowed to discharge through equal resistors at time t = 0.
(1989 - 2 Marks)
V (a) The current in each of the two discharging circuits is
zero at t = 0.
(b) The currents in the two discharging circuits at t = 0 are
equal but not zero.
(c) The currents in the two discharging circuits at t = 0 are
(b) unequal.
(d) Capacitor C1, losses 50% of its initial charge sooner
than C2 loses 50% of its initial charge.
2. Read the following statements carefully: (1993-2 Marks)
Y : The resistivity of a semiconductor decreases with
increase of temperature.
Z : In a conducing solid, the rate of collisions between
free electrons and ions increases with increase of
(c) temperature
Select the correct statement(s) from the following;
(a) Y is true but Z is false (b)Y is false but Z is true
(c) Both Y and Z are true (d) Y is true and Z is the
correct reason for Y
3. In the circuit shown in Figure the current through
(1998S - 2 Marks)
(d) 3W 2W 2W

32. Consider a thin square sheet of side L and thickness t, made


of a material of resistivity r. The resistance between two
opposite faces, shown by the shaded areas in the figure is 9V 8W 8W 4W
(2010)

2W 2W 2W
t (a) the 3 W resistor is 0.50 A.
L (b) the 3 W resistor is 0.25 A.
(c) the 4 W resistor is 0.50 A
(a) directly proportional to L (d) the 4 W resistor is 0.25 A.
(b) directly proportional to t 4. When a potential difference is applied across, the current
(c) independent of L passing through (1999S - 3 Marks)
(d) independent of t (a) an insulator at 0 K is zero
33. A meter bridge is set up as shown, to determine an unknown (b) a semiconductor at 0 K is zero
resistance ‘X’ using a standard 10 ohm resistor. The (c) a metal at 0 K is finite
(d) a p-n diode at 300K is finite, if it is reverse biased
galvanometer shows null point when tapping-key is at 52
5. For the circuit shown in the figure (2009)
cm mark. The end-corrections are 1 cm and 2 cm respectively
for the ends A and B. The determined value of ‘X’ is
(2011)
2kW R1
I

24 V 6kW R2 RL 1.5kW
(a) 10.2 ohm (b) 10.6 ohm
(c) 10.8 ohm (d) 11.1 ohm
P-330 Topic-wise Solved Papers - PHYSICS
(a) the current I through the battery is 7.5 mA 5. A copper wire having cross-sectional area of 0.5 mm2 and a
(b) the potential difference across RL is 18 V length of 0.1 metre is initially at 25°C and is thermally
(c) ratio of powers dissipated in R1 and R2 is 3 insulated from the surrounding. If a current of 10 amperes is
(d) if R1 and R2 are interchanged, magnitude of the power set up in this wire, (i) find the time in which the wire will start
dissipated in RL will decrease by a factor of 9 melting. The change of resistance with the temperature of
6. For the resistance network shown in the figure, choose the the wire may be neglected. (ii) What will this time be, if the
correct option(s) (2012- I) length of the wire is doubled? (1979)
Melting point of copper = 1075°C.
Specific resistance of copper = 1.6 × 10–8 W m
Density of copper = 9 × 10–3 kg/m3
Specific heat of copper = 9 × 10–2 cal/kg°C
6. A 25 watt and a 100 watt bulb are joined in series and
connected to the mains. Which bulb will grow brighter?
(1979)
7. A battery of emf 2 volts and internal resistance 0.1 ohm is
being charged with a current of 5 amps. (1980)
In what direction will the current flow inside the battery?
What is the potential difference between the two terminal of
(a) The current through PQ is zero. the battery?
(b) I1 = 3A 8. State ohm’s law.
(c) The potential at S is less than that at Q. In the circuit shown in figure, a voltmeter reads 30 volts
(d) I2 = 2A when it is connected across 400 ohm resistance. Calculate
SUBJECTIVE PROBLEMS : what the same voltmeter will read when it is connected across
the 300 ohm resistance. ( 1980)
1. A heater is designed to operate with a power of 1000 watts
in a 100 volt line. It is connected in a combinations with a
resistance of 10 ohms and a resistance R to a 100 volts 300W 400W
mains as shown in the figure. What should be the value of
R so that the heater operates with a power of 62.5 watts.
(1978) 60V
Heater
10W R 9. In the circuit shown in fig E1 =3 volts, E2 = 2 volts, E3 = 1
volt and R = r1 = r2 = r3 = 1 ohm. (1981 - 6 Marks)
r1 + –
100V
i1 E1
2. If a copper wire is stretched to make it 0.1% longer what is i2 r2
A R + – B
the percentage change in its resistance? (1978)
3. All resistances in the diagram below are in ohms. Find the C E2 D
effective resistance between the points A and B. (1979) i3 r3 + –
3 E3
(i) Find the potential difference between the points A and
3 3
B and the currents through each branch.
6
6 (ii) If r2 is short circuited and the point A is connected to
6 point B, find the currents through E1, E2 E3 and the
3 3 resistor R.
A 3 B 10. Calculate the steady state current in the 2-ohm resistor
shown in the circuit in the figure. The internal resistance of
4. In the diagram shown find the potential difference between the battery is negligible and the capacitance of the
the points A and B and between the points B and C in the condenser C is 0.2 microfarad. (1982 - 5 Marks)
steady state. (1979)
2W
B
3µf 1µf
3W
3µf 1µf
C 4W
1µf
10 W
20 W
100V 2.8W
A C V=6VOLTS
CURRENT ELECTRICITY P-331
Or The 6 volt battery between A and B has negligible internal
Two resistors, 400 ohms, and 800 ohms are connected in series resistance :
with a 6-volt battery. It is desired to measure the current in (i) Show that the effective resistance between A and B is
the circuit. An ammeter of a 10 ohms resistance is used for 2 ohms.
this purpose. What will be the reading in the ammeter ? (ii) What is the current that passes through the 2 ohm
Similarly, If a voltmeter of 10,000 ohms resistance is used to resistance nearest to the battery ?
measure the potential difference across the 400-ohms resistor, 15. In the given circuit (1988 - 5 Marks)
What will be the reading in the voltmeter.
E1 = 3E2 = 2 E3 = 6 volts R1 = 2 R4 = 6 ohms
11. A steady current passes through a cylindrical conductor. Is
there an electric field inside the conductor? (1982 - 2 Marks) R3 = 2 R2 = 4 ohms C = 5m f .
12. In the circuit shown in figure E, F, G, H are cells of emf 2, 1, 3
Find the current in R3 and the energy stored in the capacitor.
and 1 volt respectively, and their internal resistances are 2,
R1 E1
1, 3 and 1 ohm respectively. (1984 - 6 Marks)
Calculate :
C
(i) the potential difference between B and D and
(ii) the potential difference across the terminals of each
cells G and H
E
E2 R2 R3
A B

F 2W H E3 R4
16. An electrical circuit is shown in Fig. Calculate the potential
D C difference across the resistor of 400 ohm, as will be measured
G by the voltmeter V of resistance 400 ohm, either by applying
13. A part of ciucuit in a steady state along with the currents Kirchhoff’s rules or otherwise.
flowing in the branches, the values of resistances etc., is (1996 - 5 Marks)
shown in the figure. Calculate the energy stored in the V
capacitor C (4µF) (1986 - 4 Marks)
400W
1A
100W 100W 200W
3W l2
100W
l1
4V 3W 5W
l 10V
2A
17. Find the emf (V) and internal resistance (r) of a single battery
C 1W which is equivalent to a parallel combination of two batteries
4m F of emfs V1 and V 2 and internal resistance r1 and r 2
3V 1W respectively, with polarities as shown in figure
2A 4W (1997C - 5 Marks)
2W
r2
V2
3W + –
A B

r1 V
1A 18. A leaky parallel plate capacitor1 is filled completely with a
14. An infinite ladder network of resistances is constructed with material having dielectric constant k = 5 and electrical
conductivity s = 7.4 × 10–12 W–1m–1. If the charge on the
a1 ohm and 2 ohm resistances, as shown in fig.
plate at instant t = 0 is q = 8.85mC, then calculate the leakage
(1987 - 7 Marks)
1W 1W current at the instant t = 12s. (1997C - 5 Marks)
1W 1W
A 19. In the circuit shown in Figure, the battery is an ideal one,
with emf V. The capacitor is initially uncharged. The switch
6V S is closed at time t = 0. (1998 - 8 Marks)
2W 2W 2W 2W
(a) Find the charge Q on the capacitor at time t.
(b) Find the current in AB at time t. What is its limiting
B value as t ® ¥ :
Fig. 8.
P-332 Topic-wise Solved Papers - PHYSICS
S A R C

A R B
V R C
22. Draw the circuit diagram to verify Ohm’s Law with the help
of a main resistance of 100 W and two galvanometers of
resistances 106 W and 10–3 W and a source of varying emf.
B Show the correct positions of voltmeter and ammeter.
R
(2004 - 4 Marks)
20. A thin uniform wire AB of length 1m, an unknown resistance
23. An unknown resistance X is to be determined using
X and a resistance of 12 W are connected by thick conducting
resistances R1, R2 or R3. Their corresponding null points
strips, as shown in the figure. A battery and a galvanometer
are A, B and C. Find which of the above will give the most
(with a sliding jockey connected to it) are also available. accurate reading and why? (2005 - 2 Marks)
Connections are to be made to measure the unknown
resistance X using the principle of Wheatstone bridge.
Answer the following questions. (2002 - 5 Marks )

X R R = R1 or R2 or R3
G
X 12W
A B C
A B C D
24. In the given circuit, the switch S is closed at time t = 0. The
(a) Are there positive and negative terminals on the charge Q on the capacitor at any instant t is given by
galvanometer? Q(t) = Q(1 – e–at). Find the value of Q0 and a in terms of
(b) Copy the figure in your answer book and show the given parameters as shown in the circuit.
battery and the galvanometer (with jockey) connected (2005 - 4 Marks)
at appropriate points. R1
(c) After appropriate connections are made, it is found
that no deflection takes place in the galvanometer when
the sliding jockey touches the wire at a distance of 60 S
cm from A. Obtain the value of the resistance of X. C R2
21. How a battery is to be connected so that the shown rheostat +
will behave like a potential divider? Also indicate the points V
about which output can be taken. (2003 - 2 Marks) –

MATCH THE FOLLOWING :


MUTLIPLE CHOICE QUESTIONS WITH ONE CORRECT
Each question contains statements given in two columns, which have to be matched. The statements in Column-I are labelled A,
B, C and D, while the statements in Column-II are labelled p, q, r, s and t. Any given statement in Column-I can have correct
matching with ONE OR MORE statement(s) in Column-II. The appropriate bubbles corresponding to the answers to these
questions have to be darkened as illustrated in the following example :
If the correct matches are A-p, s and t; B-q and r; C-p and q; and D-s then the correct darkening of bubbles will look like the given.
p q r s t
A p q r s t
B p q r s t
C p q r s t
D p q r s t

1. Column I gives some devices and Column II gives some processes on which the functioning of these devices depend.
Match the devices in Column I with the processes in Column II and indicate your answer by darkening appropriate bubbles
in the 4 × 4 matrix given in the ORS. (2007)
Column I Column II
(A) Bimetallic strip (p) Radiation from a hot body
(B) Steam engine (q) Energy conversion
(C) Incandescent lamp (r) Melting
(D) Electric fuse (s) Thermal expansion of solids
CURRENT ELECTRICITY P-333

COMPREHENSION BASED Q UESTIONS : ASSERTION & REASON TYPE QUESTIONS :

Electrical resistance of certain materials, known as 1. STATEMENT-1 : In a Meter Bridge experiment, null point
superconductors, changes abruptly from a nonzero value for an unknown resistance is measured. Now, the unknown
to zero as their temperature is lowered below a critical resistance is put inside an enclosure maintained at a higher
temperature T C (0). An in teresting pr operty of temperature. The null point can be obtained at the same
superconductors is that their critical temperature becomes point as before by decreasing the value of the standard
smaller than TC (0) if they are placed in a magnetic field, i.e., resistance.
the critical temperature TC (B) is a function of the magnetic and
field strength B. The dependence of TC (B) on B is shown in STATEMENT-2 : Resistance of a metal increases with
the figure. (2010) increase in temperature. (2008)
(a) STATEMENT-1 is True, STATEMENT-2 is True;
TC (B) STATEMENT-2 is a correct explanation for
STATEMENT-1
TC (0) (b) STATEMENT-1 is True, STATEMENT-2 is True;
STATEMENT-2 is NOT a correct explanation for
STATEMENT-1
(c) STATEMENT -1 is True, STATEMENT-2 is False
O B (d) STATEMENT -1 is False, STATEMENT-2 is True

1. In the graphs below, the resistance R of a superconductor is


INTEGERVALUECORRECTTYPE:
shown as a function of its temperature T for two different 1. When two identical batteries of internal resistance 1W each
magnetic fields B1 (solid line) and B2 (dashed line). If B2 is are connected in series across a resistor R, the rate of heat
larger than B1 which of the following graphs shows the produced in R is J1. When the same batteries are connected
correct variation of R with T in these fields? in parallel across R, the rate is J2. If J1 = 2.25 J2 then the
value of R in W is (2010)
2. At time t = 0, a battery of 10 V is connected across points
A and B in the given circuit. If the capacitors have no
(a) (b) charge initially, at what time (in sceonds) does the voltage
across them become 4 V? [Take : ln5 =1.6, ln3 = 1.1] (2010)

2MW 2m F

A B
(c) (d)
2MW
2m F

2. A superconductor has TC (0) = 100 K. When a magnetic 3. Two batteries of different emfs and different internal
field of 7.5 Tesla is applied, its TC decreases to 75 K. For this resistances are connected as shown. The voltage across
material one can definitely say that when AB in volts is (2011)
(a) B = 5 Tesla, TC (B) = 80 K
(b) B = 5 Tesla, 75 K < TC (B) < 100 K
(c) B = 10 Tesla, 75K < TC (B) < 100 K
(d) B = 10 Tesla, TC (B) = 70K
P-334 Topic-wise Solved Papers - PHYSICS

1. If an ammeter is to be used in place of a voltmeter, then we


8. The thermo e.m.f. of a thermo -couple is 25 mV / o C at room
must connect with the ammeter a [2002]
temperature. A galvanometer of 40 ohm resistance, capable
(a) low resistance in parallel
(b) high resistance in parallel of detecting current as low as 10 -5 A , is connected with
(c) high resistance in series the thermo couple. The smallest temperature difference that
(d) low resistance in series. can be detected by this system is [2003]
2. A wire when connected to 220 V mains supply has power (a) 16 o C (b) 12 o C
dissipation P1. Now the wire is cut into two equal pieces (c) 8 o C (d) 20 o C
which are connected in parallel to the same supply. Power 9. The negative Zn pole of a Daniell cell, sending a constant
dissipation in this case is P2. Then P2 : P1 is [2002] current through a circuit, decreases in mass by 0.13g in 30
(a) 1 (b) 4 minutes. If the electeochemical equivalent of Zn and Cu are
32.5 and 31.5 respectively, the increase in the mass of the
(c) 2 (d) 3
positive Cu pole in this time is [2003]
3. If a current is passed through a spring then the spring will (a) 0.180 g (b) 0.141g
(a) expand (b) compress [2002] (c) 0.126 g (d) 0.242 g
(c) remains same (d) none of these. 10. An ammeter reads upto 1 ampere. Its internal resistance is
4. If in the circuit, power dissipation is 150 W, then R is 0.81ohm. To increase the range to 10 A the value of the
required shunt is [2003]
R
(a) 0.03 W (b) 0.3 W
2W (c) 0.9 W (d) 0.09 W
11. A 3 volt battery with negligible internal resistance is
15 V connected in a circuit as shown in the figure. The current I,
in the circuit will be [2003]
(a) 2 W (b) 6 W [2002]
(c) 5 W (d) 4 W
5. The mass of product liberated on anode in an electrochemical
cell depends on [2002] 3W
1/2 3W
(a) (It) (b) It 3V
(c) I/t (d) I2t
(where t is the time period for which the current is passed).
6. If qi , is the inversion temperature, q n is the neutral 3W
temperature, qc is the temperature of the cold junction, (a) 1 A (b) 1.5 A
then [2002] (c) 2 A (d) 1/3 A
12. A 220 volt, 1000 watt bulb is connected across a 110 volt
(a) qi + q c = q n (b) qi - qc = 2q n
mains supply . The power consumed will be [2003]
qi + qC (a) 750 watt (b) 500 watt
(c) = qn (d) qc - qi = 2qn (c) 250 watt (d) 1000 watt
2
13. The total current supplied to the circuit by the battery is
7. The length of a wire of a potentiometer is 100 cm, and the e. [2004]
m.f. of its standard cell is E volt. It is employed to measure
2W
the e.m.f. of a battery whose internal resistance is 0.5W. If 6V
6W 3W
the balance point is obtained at l = 30 cm from the positive
end, the e.m.f. of the battery is [2003] 1.5W

30 E 30 E
(a) (b)
100.5 (100 - 0.5) (a) 4 A (b) 2 A
(c) 1 A (d) 6 A
30 ( E - 0.5i ) 30 E 14. The resistance of the series combination of two resistances
(c) (d) is S. when they are joined in parallel the total resistance is P.
100 100
If S = nP then the Minimum possible value of n is
where i is the current in the potentiometer wire. (a) 2 (b) 3 [2004]
(c) 4 (d) 1
CURRENT ELECTRICITY P-335
15. An electric current is passed through a circuit containing 23. In the circuit , the galvanometer G shows zero deflection. If
two wires of the same material, connected in parallel. If the the batteries A and B have negligible internal resistance, the
value of the resistor R will be - [2005]
4 2
lengths and radii arein the ratio of and , then the ratio
3 3 500 W
G
of the current passing through the wires will be
[2004] 2V
(a) 8/9 (b) 1/3 12V B R A
(c) 3 (d) 2
16. In a meter bridge experiment null point is obtained at 20 cm.
from one end of the wire when resistance X is balanced
against another resistance Y. If X < Y, then where will be the (a) 100 W (b) 200 W
new position of the null point from the same end, if one (c) 1000 W (d) 500 W
decides to balance a resistance of 4 X against Y
24. A moving coil galvanometer has 150 equal divisions. Its
(a) 40 cm (b) 80 cm [2004] current sensitivity is 10-divisions per milliampere and voltage
(c) 50 cm (d) 70 cm sensitivity is 2 divisions per millivolt. In order that each
17. The termistors are usually made of [2004] division reads 1 volt, the resistance in ohms needed to be
(a) metal oxides with high temperature coefficient of connected in series with the coil will be - [2005]
resistivity (a) 105 (b) 103
(b) metals with high temperature coefficient of resistivity (c) 9995 (d) 99995
(c) metals with low temperature coefficient of resistivity 25. Two sources of equal emf are connected to an external
(d) semiconducting materials having low temperature resistance R. The internal resistance of the two sources are
coefficient of resistivity R1and R2 (R1 > R1). If the potential difference across the
18. Time taken by a 836 W heater to heat one litre of water from source having internal resistance R 2 is zero, then
10°C to 40°C is [2004]
(a) 150 s (b) 100 s (a) R = R2 - R1 [2005]
(c) 50 s (d) 200 s (b) R = R2 ´ ( R1 + R2 ) /( R2 - R1 )
19. The thermo emf of a thermocouple varies with the temperature
(c) R = R1R2 /( R2 - R1 )
q of the hot junction as E = aq + bq 2 in volts where the
ratio a/b is 700°C. If the cold junction is kept at 0°C, then the (d) R = R1R2 /( R1 - R2 )
neutral temperature is [2004] 26. Two voltameters, one of copper and another of silver, are
(a) 1400°C joined in parallel. When a total charge q flows through the
(b) 350°C voltameters, equal amount of metals are deposited. If the
(c) 700°C electrochemical equivalents of copper and silver are Z1 and
(d) No neutral temperature is possible for this termocouple.
Z 2 respectively the charge which flows through the silver
-7
20. The electrochemical equivalent of a metal is 3.35109 kg voltameter is [2005]
per Coulomb. The mass of the metal liberated at the cathode q
q
when a 3A current is passed for 2 seconds will be (a) (b)
Z Z
(a) 6.6×1057kg (b) 9.9×10–7 kg [2004] 1+ 2 1+ 1
–7 Z1 Z2
(c) 19.8×10 kg (d) 1.1×10–7 kg
21. Two thin, long, parallel wires, separated by a distance ‘d’ Z1
Z2
carry a current of ‘i’ A in the same direction. They will (c) q (d) q
Z1 Z2
[2005]
27. In a potentiometer experiment the balancing with a cell is at
(a) repel each other with a force of m 0 i 2 /(2pd ) length 240 cm. On shunting the cell with a resistance of 2W,
the balancing length becomes 120 cm. The internal
(b) attract each other with a force of m 0 i 2 /(2pd )
resistance of the cell is [2005]
(c) repel each other with a force of m 0 i 2 /(2pd 2 ) (a) 0.5W (b) 1W
(c) 2W (d) 4W
(d) attract each other with a force of m 0 i 2 /(2pd 2 ) 28. The resistance of hot tungsten filament is about 10 times
22. A heater coil is cut into two equal parts and only one part is the cold resistance. What will be the resistance of 100 W
now used in the heater. The heat generated will now be and 200 V lamp when not in use ? [2005]
(a) four times (b) doubled [2005] (a) 20 W (b) 40 W
(c) halved (d) one fourth (c) 200 W (d) 400 W
P-336 Topic-wise Solved Papers - PHYSICS
29. An energy source will supply a constant current into the 36. An electric bulb is rated 220 volt - 100 watt. The power
load if its internal resistance is [2005] consumed by it when operated on 110 volt will be
(a) very large as compared to the load resistance (a) 75 watt (b) 40 watt [2006]
(b) equal to the resistance of the load (c) 25 watt (d) 50 watt
(c) non-zero but less than the resistance of the load 37. A battery is used to charge a parallel plate capacitor till the
(d) zero potential difference between the plates becomes equal to
30. The Kirchhoff's first law (Si = 0) and second law (SiR = SE), the electromotive force of the battery. The ratio of the energy
where the symbols have their usual meanings, are stored in the capacitor and the work done by the battery will
respectively based on [2006] be [2007]
(a) conservation of charge, conservation of momentum (a) 1/2 (b) 1
(b) conservation of energy, conservation of charge
(c) 2 (d) 1/4
(c) conservation of momentum, conservation of charge
38. The resistance of a wire is 5 ohm at 50°C and 6 ohm at 100°C.
(d) conservation of charge, conservatrion of energy
The resistance of the wire at 0°C will be [2007]
31. A material 'B' has twice the specific resistance of 'A'. A circular
wire made of 'B' has twice the diameter of a wire made of 'A'. (a) 3 ohm (b) 2 ohm
then for the two wires to have the same resistance, the ratio (c) 1 ohm (d) 4 ohm
lB/lA of their respective lengths must be [2006] 39. Shown in the figure below is a meter-bridge set up with null
deflection in the galvanometer.
1
(a) 1 (b) 55W R
2
1
(c) (d) 2
4 G
32. A thermocouple is made from two metals, Antimony and
Bismuth. If one junction of the couple is kept hot and the 20 cm
other is kept cold, then, an electric current will
[2006]
(a) flow from Antimony to Bismuth at the hot junction
(b) flow from Bismuth to Antimony at the cold junction
(c) now flow through the thermocouple The value of the unknown resistor R is [2008]
(d) flow from Antimony to Bismuth at the cold junction (a) 13.75 W (b) 220 W
33. The current I drawn from the 5 volt source will be (c) 110 W (d) 55 W
[2006]
10W
DIRECTIONS : Question No. 40 and 41 are based on the following
5W 10W 20W paragraph.
Consider a block of conducting material of resistivity ‘r’ shown in
the figure. Current ‘I’ enters at ‘A’ and leaves from ‘D’. We apply
I superposition principle to find voltage ‘DV’ developed between
10W
‘B’ and ‘C’. The calculation is done in the following steps:
+– (i) Take current ‘I’ entering from ‘A’ and assume it to spread
5 volt over a hemispherical surface in the block.
(ii) Calculate field E(r) at distance ‘r’ from A by using Ohm’s law
(a) 0.33 A (b) 0.5 A E = r j, where j is the current per unit area at ‘r’.
(c) 0.67 A (d) 0.17 A
(iii) From the ‘r’ dependence of E(r), obtain the potential V(r) at
34. The resistance of a bulb filmanet is 100W at a temperature of
r.
100°C. If its temperature coefficient of resistance be 0.005
(iv) Repeat (i), (ii) and (iii) for current ‘I’ leaving ‘D’ and superpose
per °C, its resistance will become 200 W at a temperature of
(a) 300°C (b) 400°C [2006] results for ‘A’ and ‘D’.
(c) 500°C (d) 200°C
35. In a Wheatstone's bridge, three resistances P, Q and R I DV I
connected in the three arms and the fourth arm is formed by
two resistances S1 and S2 connected in parallel. The
condition for the bridge to be balanced will be a b a
[2006] A B C D
P 2R P R ( S1 + S2 )
(a) = (b) =
Q S1 + S2 Q S1 S2

P R ( S1 + S2 ) P R
(c) = (d) =
Q 2S1S2 Q S1 + S2
CURRENT ELECTRICITY P-337
40. DV measured between B and C is [2008] a1 + a 2 a1 + a 2
(a) , a1 + a 2 (b) a1 + a 2 ,
rI rI rI rI 2 2
(a) – (b) –
pa p(a + b) a (a + b)
a1a 2 a1 + a 2 a1 + a 2
(c) a1 + a 2 , (d) ,
rI rI rI a1 + a 2 2 2
(c) – (d)
2pa 2p(a + b) 2p(a - b) 45. If a wire is stretched to make it 0.1% longer, its resistance
41. For current entering at A, the electric field at a distance ‘r’ will : [2011]
from A is [2008] (a) increase by 0.2% (b) decrease by 0.2%
(c) decrease by 0.05% (d) increase by 0.05%
rI rI
(a) 2 (b) 46. If 400 W of resistance is made by adding four 100 W
8pr r2 resistances of tolerance 5%, then the tolerance of the
rI rI combination is [2011RS]
(c) 2 (d) (a) 5% (b) 10%
2pr 4pr 2
(c) 15% (b) 20%
42. A 5V battery with internal resistance 2W and a 47. The current in the primary circuit of a potentiometer is
2V battery with internal resistance 1W are connected to a 0.2 A. The specific resistance and cross-section of the
10W resistor as shown in the figure. [2008] potentiometer wire are 4 × 10–7 ohm metre and 8 × 10–7 m2,
respectively. The potential gradient will be equal to
P2 [2011RS]
(a) 1 V /m (b) 0.5 V/m
(c) 0.1 V/m (d) 0.2 V/m
48. Two electric bulbs marked 25W – 220 V an d
100W – 220V are connected in series to a 440 V supply.
5V 2V Which of the bulbs will fuse? [2012]
2W 10W 1W (a) Both(b) 100 W
(c) 25 W (d) Neither
49. The supply voltage to room is 120V. The resistance of the
lead wires is 6W. A 60 W bulb is already switched on. What
is the decrease of voltage across the bulb, when a 240 W
heater is switched on in parallel to the bulb? [JEE 2013 M]
The current in the 10W resistor is (a) zero (b) 2.9 Volt
(a) 0.27 A P2 to P1 (b) 0.03 A P1 to P2 (c) 13.3 Volt (d) 10.04 Volt
(c) 0.03 A P2 to P1 (d) 0.27 A P1 to P2 50. This questions has Statement I and Statement II. Of the four
43. Let C be the capacitance of a capacitor discharging through choices given after the Statements, choose the one that
a resistor R. Suppose t1 is the time taken for the energy best describes into two Statements. [JEE 2013 M]
stored in the capacitor to reduce to half its initial value and Statement-I : Higher the range, greater is the resistance of
t2 is the time taken for the charge to reduce to one-fourth its ammeter.
initial value. Then the ratio t1/ t2 will be [2010] Statement-II : To increase the range of ammeter, additional
shunt needs to be used across it.
1
(a) 1 (b) (a) Statement-I is true, Statement-II is true, Statement-II is
2 the correct explanation of Statement-I.
1 (b) Statement-I is true, Statement-II is true, Statement-II is
(c) (d) 2 not the correct explanation of Statement-I.
4
44. Two conductors have the same resistance at 0°C but their (c) Statement-I is true, Statement-II is false.
(d) Statement-I is false, Statement-II is true.
temperature coefficients of resistance are a1 and a 2 . The
respective temperature coefficients of their series and parallel
combinations are nearly [2010]
P-338 Topic-wise Solved Papers - PHYSICS

Solutions & Explanations


Section-A : JEE Advanced/ IIT-JEE

A 1. 20 2. R/2 3. 0

B 1. F 2. F 3. T

C 1. (d) 2. (b) 3. (a) 4. (b) 5. (c) 6. (d) 7. (b)


8. (d) 9. (b) 10. (a) 11. (c) 12. (b) 13. (c) 14. (a)
15. (d) 16. (a) 17. (a) 18. (a) 19. (b) 20. (c) 21. (b)
22. (a) 23. (a) 24. (a) 25. (b) 26. (b) 27. (a) 28. (c)
29. (c) 30. (d) 31. (c) 32. (c) 33. (b)

D 1. (b, d) 2. (c) 3. (d) 4. (a, b, d) 5. (a, d) 6. (a, b, c, d)

E 1. 5W 2. 0.2% 3. 2W 4. VAB = 25 V, VBC = 75 V


5. 9 min. 16 sec., 9 min. 16 sec. 6. 25 W 7. Positive to negative terminal, 2.5 V
8. 22.5 V 9. (i) 2V, 1A, 0A, 1A (ii) 1A, 2A, 1A; 2A 10. 0.9A or 4.96 × 10–3 A, 1.95 V 11. Yes

2 21 19
12. (i) V (ii) V, V 13. 8 × 10–4 J 14. (ii) 1.5A 15. 1.5 A, 1.44 × 10–5 J 16. 6.67 V
13 13 13

V1r2 - V2 r1 r1 r2
17. r1 + r2
,
r1 + r2 18. 0.199 mA 19. CV (1 - e -2t / 3 RC ); V - V e -2t / 3 RC ; V
2 2R 6R 2R

CVR2 R1 + R2
20. (a) No (b) 8W 24. Q0 = R + R ; a = CR R
1 2 1 2

F 1. A ® s; B ® q; C ® p, q; D ® q, r
G 1. (a) 2. (b)
H 1. (d)
I 1. 4 2. 2 3. 5

Section-B : JEE Main/ AIEEE

1. (c) 2. (b) 3. (b) 4. (b) 5. (b) 6. (c) 7. (d)


8. (a) 9. (c) 10. (d) 11. (b) 12. (c) 13. (a) 14. (c)
15. (b) 16. (c) 17. (a) 18. (a) 19. (d) 20. (c) 21. (b)
22. (b) 23. (a) 24. (c) 25. (a) 26. (a) 27. (c) 28. (b)
29. (d) 30. (d) 31. (d) 32. (d) 33. (b) 34. (b) 35. (b)
36. (c) 37. (a) 38. (d) 39. (b) 40. (a) 41. (c) 42. (c)
43. (c) 44. (d) 45. (a) 46. (a) 47. (c) 48. (c) 49. (d)
50. (d)
CURRENT ELECTRICITY P-339

3. For a given voltage, current is more in case of T1.


FILL IN THE BLANKS : Since, V = IR
2 V
V \ R=
1. We know that P = I
R
2 T1
V 100 ´ 100
\ R= = = 20 W
P 500 I
NOTE : For the bulb to deliver 500 W, it should have a p.d. T2
of 100 V across it. This would be possible only when
R = 20 W is in series with the bulb because in that case both
resistances will share equal p.d.
20W R V

Resistance is less in case of T1 and more in T2.


NOTE : For a metallic wire, resistance increases with
200V
temperature, therefore T2 > T1
2. The given circuit may be redrawn as shown in the figure.
Thus, the resistances 2R, 2R and R are in parallel. MCQ's WITH ONE CORRECT ANSWER :
1 1 1 1 2 1. (d) R1 = R0 (1 + a t1) Þ 1 = R0 [1 + 0.00125 × 27]
Hence, = + + =
R AB 2 R 2 R R R R2 = R0 (1 + a t2) Þ 2 = R0 [1 + 0.00125 × t2]
On solving we get
2R T2 = 854°C = 1127 K
2. (b) KEY CONCEPT : The heat produced is given by
A 2R B
2 æ 2ö
2 pr
D C V rl \ H = V ç ÷
H= and R =
R R pr 2 è rl ø

R æ pV 2 ö r 2
Hence, RAB = or H=ç ÷
2 è r ø l
3. Let a current I flow through the circuit. Net emf of the circuit
= 8 (5V) = 40 V Thus heat (H) is doubled if both length (l) and radius
Net resistance in the circuit = 8 (0.2 W) = 1.6 W (r) are doubled.
Current flowing through the circuit, 1 1
40 V 3. (a) I µ 2
;V µ ;V µ r 0
I= = 25 A r r
1.6 W
The voltmeter reading would be 4W 6W
V = E – IR = (5V) – (25A) (0.2 W) 4. (b) A B
= 5V – 5V = 0 (I-I1)
ALTERNATE SOLUTION
We apply Kirchoff's law in the loop and find the current.
I
TRUE / FALSE :
5W
1. NOTE : An electrolyte solution is formed by mixing an C D
I1
electrolyte in a solvent. The electrolyte on dissolution
furnishes ions. The preferred movement of ions under the
Let I1 be the current flowing in 5W resistance and
influence of electric field is responsible for electric current.
(I – I1) in 4W and 6W resistance. The heat generated in
2. NOTE : Billions of electrons in a conductor are free and
5W resistor is 10 cal/s = 4.2 × 10 J/s
have thermal velocities. The electrons have motion in random
directions even in the absence of potential difference. 2
\ 4.2 × 10 = I1 R
P-340 Topic-wise Solved Papers - PHYSICS
The circuit represents balanced Wheatstone Bridge.
4.2 ´ 10 Hence 6R W resistance is ineffective
\ I1 = = 8.4 = 2.9 amp ....(i)
5
R 2R
Since AB and CD are in parallel. 3R
\ The potential dfference remains the same between C 2R 4R 6R
and D; and between A and B.
\ (I – I1) (4 + 6) = I1 × 5 on solving using I1 from (i) we get 4W
E
(I – 2.9) 10 = 2.9 × 5 \ I – 2.9 = 1.45 E 4W
\ I = 4.35 A
Heat released/sec in 4 W resistance will be 1 1 1 (3R )(6 R)
= (4.35 – 2.9)2 × 4 = 8.4 J/s = 2 cal/s = + , Req = = 2R
Req 3R 6 R (3R) + (6R)
ALTERNATE SOLUTION
Since RAB = 2RCD therefore, current in AB will be half 2R

as compared to current in CD.


4W
P4 (i / 2) 2 4 1 10 E
= = Þ P4 = = 2 cal/s For Max. Power
P5 2
i ´5 5 5
External Resistance = Internal Resistance
Here P4 = Power dissipation in 4W
2R = 4 W
P5 = Power dissipation in 5W
5. (c) BC and AC are in series R=2W
\ RBCA = 30 + 30 = 60 W 8. (d) KEY CONCEPT : When a steady current flows in a
Now BA and DC are in parallel. metallic conductor of non uniform cross section then
drift speed
1 1 1 90
= + = I I
Req 30 60 30 ´ 60 vd = and Electric field E =
neA sA
Req = 20 W ; V = IR
Drift velocity (vd) and electric field (E) vary because
2 cross section of the conductor is non-uniform.
Þ I= = 0.1 Amp.
20 Hence only current remains constant along the length
B of the conductor.
9. (b) For the capacitor to get charged upto 0.75 V, the charge
2V 30W on the plates should be
30W
R=105W C = 10-5F
A C
30W
I B D I

30W 60W 20W


2V 2V E=1.5V

A C q = CV
= 10–5 × 0.75 = 0.75 × 10–5 C
6. (d) Copper is a metal whereas Germanium is Semi-
conductor. Using the equation of charging of capacitor
NOTE : Resistance of metal decreases and - t / RC
q = CE [1 - e ]
semiconductor increases with decrease in temperature.
7. (b) The equivalent circuits are shown in the figure. - t /105 ´ 10 -5
0.75 × 10–5 = 10–5 × 1.5 [1 - e ]

R
R
R 2R =
0.75
1.5
(
= 1- e
-t
) or
1
2
= 1- e
-t

R 6R R 6R
E 4W 1 1
-t -t
2R 4R or - = -e or =e
R 4R 2 2
Taking log on both sides
E 4W Balanced
Wheat stone – t = – ln 2
Bridge Þ t = 0.693 sec
CURRENT ELECTRICITY P-341
Let the current be flowing in clockswise direction.
10. (a) A Applying Kirchoff's law in the outer loop, we get
V – IR – I (2R) – 2V = 0
\ I = – V/3R
P Q
NOTE : The minus sign here indicates that the current
is in the opposite direction to what we have assumed.
S Applying Kirchoff's law from A to E via B, we get
V 4V
VA + V + IR = VE \ 0+ V+ ´ R = VE =
R G 3R 3
Again applying Kirchoff's law from A to E via C, we get
B
4V
VA + V + Vcap = VE Þ Vcap = -V
3
V
\ Vcap =
Since the opening or closing the switch does not affect 3
the current through G, it means that in both the cases 12. (b) Let R be the resistance of wire.
there is no current passing through S. Thus potential l
at A is equal to potential at B and it is the case of
balanced wheatstone bridge..
IP = IQ and IR = IG
11. (c) There will be no current flowing in branch BE in steady
condition. V V V
Let I be the current flowing in the loop ABCDEFA. (3V )2
Applying Kirchoff's law in the loop moving in Energy released in t second = ´t
R
anticlockwise direction starting from C.
+ 2V – I (2R) – I (R) – V = 0 9V 2
\ V = 3IR \ Q= ´t
R
Þ I = V/3R ... (1)
Applying Kirchoff's law in the circuit ABEFA we get on But Q = mcDT (where m = mass of wire)
moving in anticlockwise direction starting from B 9V 2
\ mcDT = ´t .... (i)
R R
A F
V I Let R' be the resistance of the second wire
2l
B E
V C

C D
2V 2R I
V V n times V
+ V + Vcap – IR – V = 0 (where Vcap is the p.d. across
capacitor). Þ R' = 2R (Q length is twice)

æV ö V ( NV )2
\ Vcap = IR = ç ÷ ´ R = \ Energy released in t-seconds = ´t
è 3R ø 3 2R
ALTERNATE SOLUTION Also Q' = m'cDT = (2m) CDT
Let us consider A to be at 0V. Then point B, C and D
N 2V 2
will be at V, V and 2V volt respectively. 2 mcDT = ´t .... (ii)
2R
V R Dividing (i) by (ii)
B
I mcDT 9V 2 ´ t / R 1 9´ 2
= 2 2 or,, =
A V 2mc DT N V t / 2 R 2 N2
0
V C C E or, N 2 = 18 × 2 \ N = 6
2V 13. (c) Since current I is independent of R6, it follows that the
resistance R1, R2, R3 and R4 must form the balanced
D 2R I
Wheatstone bridge.\ R1 R4 = R2R3
P-342 Topic-wise Solved Papers - PHYSICS
14. (a) The circuit is symmetrical about the axis POQ. or W3 : W2 : W1
The circuit above the axis POQ represents balanced
100 ´ 100 ´ 60 ´ 60 100 ´ 100 ´ 60 ´ 60
wheatstone bride. Hence the central resistance 2R is = 60 : :
ineffective. Similarly in the lower part (below the axis 160 ´ 160 ´ 60 160 ´ 160 ´ 100
POQ) the central resistance 2R is ineffective. = 64 : 25 : 15
2R 2R 16. (a) In ohm's law, we check V = IR where I is the current
flowing through a resistor and V is the potential
2R difference across that resistor. Only option (a) fits the
r O r
above criteria.
P Q
2R NOTE : Remember that ammeter is connected in series
with resistance and voltmeter parallel with the net
2R 2R resistance.
Therefore the equivalent circuit is drawn.
2R 2R A A
V V

P r r Q

2R 2R
1 1 1 1 r + r + 2r
\ = + + = 17. (a)
RPQ 4 R 4 R 2r 4 Rr

2 Rr R1 G R2
RPQ =
R+r x
V2 A R3 C R4 B
15. (d) KEY CONCEPT : R =
P At null point
2 2
V V R1 R3 x
\ R1 = ,R = = R3 ; = =
100 2 60 R2 R4 100 - x
R1 R2
If radius of the wire is doubled, then the resistance of
V1 V2 AC will change and also the resistance of CB will
V1= VR 1 V2= VR 2 R1 R
R 1+R 2 R 1+R 2 change. But since does not change so, 3 should
R2 R4
R3
also not change at null point. Therefore the point C
250V does not change.
2
W3= V =
2
(250) 18. (a)
R3 R3 III < II < IV < I.

I R R R I 3R
V = 250V
Power dissipation, P1 = I 2 (3R) = 3I 2 R
V12 V 2 R1 V22 V 2 R2
W1 = = , W = =
R1 ( R1 + R2 )2 2 R2 ( R1 + R2 ) 2 R R

I
V2 I 2R
and W3 = 3
R3
R

(250) 2 (250) 2 R2 (250)2 æ 2R ö


W3 : W2 : W1 = : R2 : R1 Power dissipation, P2 = I2 çè ÷ø = 0.67I2R
R3 ( R1 + R2 )2 ( R1 + R2 ) 2 3
or W3 : W2 : W1
R
(250) 2 (250) 2 V2 (250)2V 2
= ´ 60 : ´ :
V2 é 1 1ù 4
2 60 é 1 2
1ù 4 I R I R/3
êë100 + 60 úû V êë100 60 úû V ´ 1000
+
R
CURRENT ELECTRICITY P-343

Power dissipation, P3 = I2 ( R / 3) = 0.33I 2 R


Q ( )
V0 1 - e -t / RC = 3V0 .e - t / RC

Þ 1– e–t/RC = 3e–t/RC Þ 1 = 4e–t/RC


R Taking log on both sides
I R I 3R/2
æ t ö
loge 1 = 2loge2 + ç -
R è RC ÷ø

æ3 ö Þ 0 = 2 × 2.303 log10 2 –
t
Power dissipation, P4 = I 2 çè R÷ø = 1.5 I2R RC
2
19. (b) Total external resistance will be the total resistance of Þ t = [2 × 2.303 log10 2] × 2.5 × 106 × 4 × 10–6
whole length of box. It should be connected between A = 13.86 sec.
and D. 24. (a) KEY CONCEPT :
IgG = (I – Ig) S
20. (c) For various combinations equivalent resistance is
Here, Ig = 100 × 10–6 A
maximum between P and Q.
G = 100 W
21. (b) KEY CONCEPT : The current in RC circuit is given by
S = 0.1 W
I = I0e–t/RC
Ig
t æ -t ö G
or lnI = lnI0– or lnI = çè ÷ + lnI 0 I
RC RC ø
I-Ig
æ -t ö æE ö
lnI = ç ÷ + ln ç 0 ÷ S
è RC ø è Rø
On comparing with y = mx + C æG ö -6 æ 100 ö
\ I = Ig ç + 1÷ = 100 ´ 10 çè + 1÷
èS ø 0.1 ø
æE ö 1
Intercept = ln ç 0 ÷ and slope = – = 100 × 10–6 × 1000.1 = 100.01 mA
è Rø RC
25. (b) The heat supplied under these conditions is the change
When R is changed to 2R then slope increases and in internal energy
current becomes less. New graph is Q. Q = DU
22. (a) The current in 2W resistor will be zero because it is not The heat supplied Q = i2RT
a part of any closed loop. = 1 × 1 × 100 × 5 × 60 = 30,000 J = 30 kJ
26. (b) A 2r B
C
10V 5W
20V I r
10W l/2
l/2
VAB I AB R AB R
2W (a) = = AB
VBC I BC RBC RBC
23. (a) KEY CONCEPT : At any instant of time t during
charging process, the transient current in the circuit is l
given by r
2[p ´ 4r 2 ] 1
= =
V0 -t / RC l 4
I= e r
R 2[pr 2 ]
\ Potential difference across resistor R is [IAB = IBC, wire is of same material]
é V0 -t / RC ù Therefore option (a) is incorrect.
VR = ê e ú´R
ëR û l
r
PBC I 2 RBC 2[p ´ 4r 2 ] 1
= V0e - t / RC ...(i) (b) = 2 = =
PAB I RAB l 4
\ Potential diff. across C r
2[pr 2 ]
Vc = V0 – V0e–t/RC = V0 (1– e–t/RC) ...(ii)
\ PAB = 4PBC; Therefore (b) is correct.
Q Vc = 3VR (given)
P-344 Topic-wise Solved Papers - PHYSICS
I
\ Potential difference across 3W resistance = 3V and
potential difference across 6W resistance = 6V
J AB p ´ 4r 2 1
(c) = = ; Therefore (c) is incorrect. \ p.d. across 3 µF capacitor = 3V
J BC I 4
and p.d. across 6 µF capacitor = 6V
p ´ r2
\ Charge on 3 µF capacitor Q1 = 3 × 3 = 9 µC
éV AB ù Charge on 6 µF capacitor Q2 = 6 × 6 = 36 µC
E AB êë l / 2 úû 1 The total charge enclosed in the dotted portion =
(d) = = ; Therefore (d) is incorrect.
E BC é VBC ù 4 \ Charge passing the switch = 36 – 9 = 27 µC
ê l/2 ú
ë û
V2
27. (a) Given X is greater than 2W when the bridge is balanced 29. (c) We know that P =
R
R X
= For constant value of potential difference (V) we have
l 100 - l
1
R = 2W X Pµ
R
Case (i)

l 100-l
G 1W 1W
1W 1W
2W 2W 1W
1W 1W
B Switch 1W
1W 1W
or, 100 R – R l = l X or,, 200 – 2 l = l X
200
or, l=
X +2 This is a case of balanced Wheatstone bridge R1 = 1W
When the resistances are interchanged the jockey shifts Case (ii)
20 cm. Therefore
X 2 1W 1W
=
l + 20 80 - l 2W 2W
1W
80 X – l X = 2 l + 40 1W
X R = 2W 1W 1W

Clearly the equivalent resistance (R2) will be less than


1W.
Case (iii)
l+20 80-l
G
1W 1W
or, 80 X = l (X + 2) + 40 2W 2W
æ 200 ö
or, 80 X = çè ÷ ( X + 2) + 40 1W
X + 2ø
240 1W 1W
or, X = = 3W.
80
28. (c) The total charge enclosed in the dotted portion when
the switch S is open is zero. When the switch is closed
and steady state is reached, the current I coming from 1W
the battery is
9 = I (3 + 6) Þ I = 1A 2W
3 mF 6 mF 1W
+Q1 –Q1 +Q2 –Q2
3W 6W
Thus R3 = 2W
Since, R2 < R1 < R3
\ P2 > P1 > P3
9V
CURRENT ELECTRICITY P-345

V2 X 10
30. (d) We know that P = =
R l1 l 2
For a given potential difference at a particular Here l1 = 52 + End correction = 52 + 1 = 53 cm
temperature
l 2 = 48 + End correction = 48 + 2 = 50 cm
1

R X 10
\ 53 = 50
It is given that the powers of the bulbs are in the order
100W > 60 W > 40W
53
1 1 1 \X = = 10.6W
> > 5
\ R100 R60 R40
31. (c) The following points should be considered while making
the circuit : 1. (b, d) KEY CONCEPT : During decay of charge in R.C.
(i) An ammeter is made by connecting a low resistance R2 circuit,
in parallel with the galvanometer G2. I = I0e–t/RC

G2 q0
where I0 =
RC
q0
R2 When t = 0, I = I0 =
RC
(ii) A voltmeter is made by connecting a high resistance R1
in series with the galvanometer G1. C1 C2

R1
G1
(iii) Voltmeter is connected in parallel with the test resistor V V
RT. Since potential difference between the plates is same initially
(iv) Ammeter is connected in series with the test resistor therefore I is same in both the cases at t = 0 and is equal to
RT.
q0 V
(v) A variable voltage source V is connected in series with I= = (R is same for C1 and C2)
the test resistor RT. RC R
Also, q = q0e–t/RC
l
32. (c) We know that R = ρ q0 q
a When q = then 0 = q0 e–t/RC
Where l is the length of 2 2
the conductor through or e + t/RC = 2.
which the current flows t
and a is the area of cross = log e 2
L RC
section. \ t = RC loge 2
Here l = L and a = L × t t \ t µ C.
rL r L
\ R= = t1 C1 1
\ = = 0.5
L´t t t 2 C2 = 2
\ R is independent of L
33. (b) At Null point or, t1 = 0.5 t2
Therefore time taken for the first capacitor (1 µF) for
discharging 50% of initial charge will be less.
X 10W 2. (c) NOTE : The conductivity of a semiconductor increases
with increase in temperature i.e. the resistivity decreases
with increase in temperature.
In a conducting solid, the collisions become more
frequent with increase of temperature.
B 3. (d) The net resistance of the circuit is 9W as shown in the
A
52 cm 48 cm following figures.
P-346 Topic-wise Solved Papers - PHYSICS

3W 2W 2W 3W 2W 24 24 ´ 7.5
\ I= mA = = 7.5 mA
9 24
9V 8W 8W 4W 8W 8W 8W +2
9V 7.5
2W 2W 2W 2W 2W Þ option (a) is correct.
The potential difference accros RL = potential
3W 3W 2W difference accros Rp
8W 8W 8W 4W æ 9 ö
9V 9V = (7.5 mA) ç k W÷ = 9V
è 7.5 ø
2W 2W 2W
Þ option (b) is incorrect.
3W
(15)2
4W Power dissipation across R1
= 2
9V 9V 9W
Now,
Power dissipation across R2 (9)2
2W
6
V 9V
I= = = 1.0 A 15 ´ 15 6
R 9W = ´ = 8.33
2 9´9
The flow of current in the circuit is as follows.
Þ option (c) is incorrect.
1ampere 3W 0.5A
The magnitude of power dissipated accross R 2 is
0.5A
(9)2
9V 9W 9V
8W 8W .
1.5
2W Now when R1 and R2 are interchanged the equivalent
1A 2W 2W 3W 2W 0.25A 2 ´ 1.5 3
3W
0.25A resistance between R1 and RL = = kW
0.25A 2 + 1.5 3.5
8W 8W 4W 8W 8W 8W
9V 9V

2W 2W 2W 2W 2W
R2 = 6 k W
NOTE : The current divides into two equal parts if passes
through two equal resistances in parallel.
Thus current through 4W resistor is 0.25 A.
24 V RL = 1.5 k W
4. (a,b,d) At 0K an insulator does not permit any current to 6kW R1
flow through it. Option (a) is correct.
At 0K a semiconductor behaves as an insulator. Option (b)
is correct. \ Potential drop across this equivalent resistance
In reverse biasing at 300 K, a very small current.
flows through a p-n junction diode. Option (d) is correct. 3
In case of metal, the current flowing will be very-very high 3.5 3
= ´ 24 = ´ 24 = 3V
because a metal becomes super conductor at 0K. Option (c) 3 24
+6
is incorrect 3.5

5. (a, d) 32 1 é 92 ù
\ Potential difference accros RL = = ê ú
1.5 9 ëê1.5 ûú
2k W R1 R1
I I
\ The magnitude of the power dissipation in RL will
24 V decrease by a factor 9 if R1 and R2 are interchanged.
RL = 1.5 kW

24 V (d) is the correct option.


6k W R2 Rp 6. (a, b, c, d)
The given circuit is an extension of wheatstone bridge,
therefore points P and Q are at the same potential and point
S and T are also at the same potential. Therefore no current
R ´ RL 6 ´ 1.5 9 passes through PQ and ST and the circuit reduces to as
Rp = 2 = = kW shown
R2 + RL 6 + 1.5 7.5
CURRENT ELECTRICITY P-347
Let the initial length of the wire be 100 cm, then the new
6W
0.1
I2 length is 100 + ´ 100
100
I1 – I2 lf = 100.1 cm ...(ii)
12 W Let Ai and Af be the initial and final area of cross-section.
I1 Then
100 ×Ai = 100.1 Af
100
12V Þ Af = A ... (iii)
100.1 i
12 é 6 ´ 12 ù From (i), (ii) and (iii)
I1 =
4 êëQ Req = 6 + 12 úû
(100.1)2 100
= 3A -
R f - Ri 100 Ai Ai
´ 100 = ´ 100
é 12 ù Ri 100
\ I2 = 3 ê = 2A
ë 6 + 12 úû Ai
As P and Q are equipotential and potential at S is less
than the potential at P (potential drops across a (100.1) 2 - (100) 2 200.1 ´ 0.1
resistance as current passes through it), therefore VS < = ´ 100 = ´ 100
(100) 2 100 ´ 100
VQ.
= 0.2 %
SUBJECTIVE PROBLEMS : Thus the resistance increases by 0.2%.
1. The resistance of the heater is DR DA Dl
Alternatively for small change = +
V 2
100 ´ 100 R A l
R= = = 10 W
P 100 3W 3W 3W
The power on which it operates is 62.5 W 3. 3W
3W 3W 6W 3W
6W 6W 6W 3W
\ V = R ´ P ' = 10 ´ 62.5 = 625 = 25 6W
6W Þ Þ
3W 3W
Since the voltage drop across the heater is 25V hence 6W
6W
3W 6W 3W
voltage drop across 10W resistor is (100 – 25) = 75V. 3W 3W 3W

V 75 3W 6W 3W
3W 6W 3W
\ The current in AB = I = = = 7.5 A 3W 6W 6W
R 10 3W 3W
6W 3W
This current divides into two parts. Let I1 be the current 3W 3W 3W
3W
that passes through the heater. Therefore 6W
Þ 2W
25 = I1 × 10 3W
I1 = 2.5 A
Thus current through R is 5A. 4. Applying Kirchoff's law in loop AQBRC
A B q q
Heater – - + 100 = 0
6 2
R Þ q = 150 µC
150
\ Potential difference between AB = = 25V
100 V 6
\ Potential difference between BC
Applying Ohm's law across R, we get = 100 – 25 = 75V
25 = 5 × R
Þ R = 5W 3 mF 1 mF 3 mF 1 mF

lf li 3 mF 1 mF
r -r 3 mF 1 mF
R f - Ri Af Ai 1 mF
2. ´ 100 = ´ 100 1 mF
Ri li 10 W 10 W
r
Ai 20 W 100 V 20 W 100 V

6 mF 2 mF
lf
B
l Q +q -q R
- i
+q -q

A f Ai
= ´ 100 A C
li ... (i) 100 V

Ai D E
14mF
P-348 Topic-wise Solved Papers - PHYSICS
5. A = 0.5 mm2 = 0.5 × 10–6 m2, Let the resistance of voltmeter be RV and the current flowing
l = 0.1m, T1 = 25°C, I = 10 amp, T2 = 1075 k through it be IV.
(i) Mass of the wire (m) = A × l × d Then IV RV = 30 ...(i)
= 0.5 × 10–6 × 0.1 × 9 × 103 (I – IV) × 400 = 30 ...(ii)
= 45 × 10–5kg. Also 300 × I = 30
Rise in temperature DT = T2 – T1 \ I = 0.1 A
= 1075°C – 25°C = 1050°C 3
\ 0.1 – IV = [from equation (ii)]
l 40
Resistance of wire R = r
A 3 4-3 1
or IV = 0.1 – Þ IV = = A
If a current I flows in wire of resistance R for time t,the 40 40 40
heat energy produced is From equation (i)
I 2 Rt \
1
´ RV = 30
Q= cal.
4.2 40
Let the wire melts in time t then Þ RV = 1200 W.
For fig. 2
I 2 Rt
= m cDT (where C is specific heat of
4.2 V
IV
copper wire)
4.2(mc DT ) A I 400W 400W
I 2 ´ rl ´ t
or, = mcDT \ t=
4.2 ´ A I 2 rl 60 V
IV × RV = (I – IV) 300
4.2 ´ 45 ´ 10 -5 ´ 9 ´ 10-2 ´ 1050 ´ 0.5 ´ 10 -6 IV × 1200 = (I – IV) 300
=
10 ´ 10 ´ 1.6 ´ 10-8 ´ 0.1 4IV = I – IV ... (1)
From Kirchoff's loop law
= 556 sec = 9 minutes 16 seconds
300 (I – IV) + 400 I = 60
(ii) When the length of the wire is doubled, its resistance
Þ 300 I – 300 IV = 60
R as well as its mass m are doubled.
3500 IV – 300 IV = 60
4.2 ms DT 60 3
t= Þ IV = = A
I 2R 3200 160
As t is proportional to m/R and as both m and R are 3
doubled, t will remain the same, i.e. the wire will start \ Reading of voltmeter = ´ 1200 = 22.5 V..
160
melting in the same time.
6. 25 watt bulb will glow brighter. This is because P = I2R ALTERNATE SOLUTION : Potential difference across the
where I is the current flowing and R is the resistance of the 400 W resistance = 30 V. Therefore, potential difference
appliance. When I is same, P µ R. The resistance of 25 W is across the 300 W resistance = 60 – 30V = 30 V. Let R be the
resistance of the voltmeter. As the voltmeter is in the parallel
æ V2ö with the 400 W resistance, their combined resistance is
çQ R =
more
è P ÷ø than the resistance of 100 W bulb. Thus 400 R
R'=
(400 + R )
power dissipation in 25W bulb is more hence it will glow
brighter. As the potential difference of 60 V is equally shared between
7. NOTE : The current will flow from the positive terminal to the 300 W and 400 W resistance. R' should be equal to 300 W.
the negative terminal inside the battery. Thus
During charging the potential difference 400 R
300 =
V = E + Ir = 2 + 5 × 0.1 = 2.5 V (400 + R )
8. NOTE : According to Ohm's law the current flowing in a
which gives R = 1200W, is the resistance of the voltmeter.
metallic wire is directly proportional to the potential
When the voltmeter is connected across the 300W resistance,
difference applied across the ends of wire provided other
their combined resistance is
physical conditions like temperature, strain etc. remains
constant. 300R 300 ´ 1200
R '' = = = 240W
For fig. 1 (300 + R ) (300 + 1200)
\ Total resistance in the ciruit = 400 + 240 = 640 W
V
IV \ Current in the circuit is
I
300W A 400W B 60V 3
I= = A
640W 32
60 V
CURRENT ELECTRICITY P-349
\ Voltmeter reading NOTE : The – ve sign of I3 indicates that the direction of
= Potential difference across 240 W resistance current in branch UTSR is opposite to that assumed.
Applying Kirchoff's law in AURBA starting from A moving
3
= ´ 240 = 22.5V clockwise.
32
(I1 + I2 + I3) R – E2 = 0 or (1 + I2 – 1) R = 2
P
r1
Q
or I2 = 2 amp
E1 Current through R is I1 + I2 + I3 = 2A
R U I1 r2
9. (i) 10. KEY CONCEPT : When the current becomes steady then
A I2 E2 RB the branch containing capacitor will be ineffective as no
I3 r3
(I1+I2) T current will be flowing through it.
E3 S The circuit can be redrawn as
Applying Kirchoff's law in PQRUP starting from P moving I1
2W
clockwise
I1r1 – E1 + E2 – I2r2 = 0 A A B
G
3W I2
or I1 × 1 – 3 + 2 – I2 × 1 = 0
or I1 – I2 = 1 ...(i) C I
4W
Applying Kirchoff's law in URSTU starting from U moving F C
clockwise.
I2r2 – E2 + E3 + I3r3 = 0 2.8W
E D
or I2 × 1 – 2 + 1 + I3 = 0 or I2 + I3 = 1
V=6 Volt
or I2 + I1 + I2 = 1
or I1 + 2I2 = 1 ... (ii) It is clear from the figure that resistance 2W and 3W are in
Subtracting (i) from (ii) parallel
I1 + 2I2 – I1 + I2 = 0 \ 2I1 = 3I2 (as p.d. across the two resistors will be same)
or I2 = 0 or I1 = 1 amp. 2
\ Current through branch PQ is 1 amp. I2 = I1 ... (i)
3
Current through branch UR is 0 amp. Applying Kirchoff's law in loop ABCDEFGA starting from
Current through branch TS is 1 amp. A in the clockwise direction
potential difference from A to B – I1 × 2 – I × 2.8 + 6 = 0
VA – 0 × R + I1r1 – E1 = VB or – I1 × 2 – (I1 + I2) × 2.8 + 6 = 0
\ VA – VB = E1 – I1r1
= 3 – 1 = 2 Volt é 2 ù
or – 2I1 – 2.8 ê I1 + I1 ú + 6 = 0
ALTERNATE SOLUTION : ë 3 û
E1 E2 E3 é 5ù
+ + or 6 = ê 2 + 2.8 ´ ú I1 or I1 = 0.9 A
SE / r r1 r2 r3 ë 3û
VAB = =
1 1 1 1 ALTERNATE SOLUTION
S + +
r r1 r2 r3
2´3
(Req)AB = = 1.2 W
(ii) r1 2+3
P Q
Total current through the battery
I1 E1
6 3
A R U I2 E2 R B = = 1.5A \ I1 = ´ 1.5 = 0.9 A
1.2 + 2.8 5
I3
T r3 E3 S
I1+I2+I3 400W 800W
OR

Applying Kirchoff's law in PQRUP starting from P moving


10W A
clockwise
I1r1 – E1 + E2 = 0 or I1 – 3 + 2 = 0
or I1 = 1 amp
Applying Kirchoff's law in URSTU starting from U moving 6 Volt
clockwise
– E2 + E3 – I3r3 = 0 Applying Kirchoff's law moving in clockwise direction
or – 2 + 1 – I3 = 0 starting from battery, we get
or I3 = – 1 amp + 6 – 10 I – 400 I – 800 I = 0
P-350 Topic-wise Solved Papers - PHYSICS
6 To find the p.d. between B and D, we move from B to D
\ 6 = 1210 I \ I = = 4.96 × 10–3 A
1210 é5 6ù 2
VB + ê - ú ´ 2 = VD \ VB – BD = volt
ë13 13 û 13
10,000W
F V G 6 39 - 18 21
p.d. across G = 3 - ´3= = volt
C 13 13 13
B
I2 400W I 800W
D [Q the cell is in discharging mode]
I 6 19
p.d. across H = 1 + 1 × = volt
13 13
[Q cell is in charging mode]
A E
6 Volt 13. Applying Kirchoff's first law at junction M, we get the current
i1 = 3A
The voltmeter and 400 W resistor are in parallel and hence
p.d. will be same
\ 10,000 I1 = 400 I2 ... (i) 1A
Applying Kirchoff's law in loop ABCDEA starting from A in 3W
4V 3W
clockwise direction. M i1 5W N
– 400 I2 – 800 I + 6 = 0 2amp
C 4mF 1W
\ 6 = 400 I2 + 800 (I1 + I2) 3V 1W
\ 6 = 400 I2 + 800 [0.04 I2 + I2] 2A P i2 2W O 4W
3W
\ I2 = 4.87 × 10–3 Amp. 1A
\ Potential drop across 400W resistor
= I2 × 400 Applying Kirchoff's first law at junction P, we get current
= 4.87 × 10–3 × 400 i2 = 1A
= 1.948 volt » 1.95 volt NOTE : No current flows through capacitor at steady state.
\ The reading measured by voltmeter = 1.95 volt Moving the loop along MNO to P
11. Yes. Current flows in a conductor only when electric field is \ VM – 5 × i1 – 1 × I1 – 2 × i2 = VP
established within the conductor which exerts force on free \ VM – VP = 6i1 + 2i2 = 6 × 3 + 2 × 1 = 20 V
electrons.
\ V = 6 × 3 + 2 × 1= 20 V
NOTE : In electrostatic condition electric field inside a
Energy stored in the capacitor
conductor is zero.
12. Let I2 current flow through the branch DCB 1 1
\ By Kirchoff's junction law, current in branch DB will be = CV 2 = × 4 × 10–6 × 20 × 20 = 8 × 10–4 J
2 2
I2 – I1 as shown in the figure.
14. Let the effective resistance between points C and D be R
I2 2V then the circuit can be redrawn as shown
A B The effective resistance between A and B is
E
2W 2´R
Req = 1 + .
R+2
1V 1V
1W
2W

1W
F H A 1W C 1W 1W
A 1W
I2 3W I1 R
6V 2W 2W 2W to¥ 6V 2W
I2-I1
D G C B
3V I1 D B

Applying Kirchoff's law in loop BDAB This resistance Req can be taken as R because if we add one
+ 2 (I2 – I1) + 1 + 1 × I2 – 2 + 2I2 = 0 identical item to infinite items then the result will almost be
Þ 2I1 – 5I2 = – 1 ... (i) the same.
Applying Kirchoff's law in loop BCDB, we get 2´R
– 2(I2 – I1) + 1 + I1 – 3 + 3I1 = 0 \ 1+ =R
R+2
Þ 3I1 – I2 = 1 ... (ii)
Solving (i) and (ii), we get I1 = 6/13 amp Þ R + 2 + 2R = R2 + 2R Þ R2 – R – 2 = 0
Þ R2 – 2R + R – 2 = 0
5
and I2 = amp Þ R (R – 2) + 1 (R – 2) = 0
13 Þ [R + 1] [R – 2] = 0 Þ R = 2W.
CURRENT ELECTRICITY P-351
(ii) Applying Kirchoff's law in the two loops, we get 16. Applying Kirchoff's law in loop JMGDJ, we get

A 1W i C F L
i-i1 K V
I3
6V 2W 2W
i1 2
J M
B D I3 400W I3
I2-I3
6 – i – 2i1 = 0 C E I1+I2-I3
G
i = 6 – 2ii ... (i) I2 100W D 100W 200W
– 2 (i – i1) + 2i1 = 0 ...(ii) B F I1+I2
From (i) and (ii), we get I1 100W
– 2 (6 – 2i1) + 2i1 + 2i1 = 0
A
Þ – 12 + 4i1 + 4i1 = 0 I 10V

12 3
Þ i1 = = = 1.5 A I3
8 2 - × 400 + (I1 + I2 – I3) 200 + (I2 – I3) 100 = 0
2
Thus current through nearest 2 ohm resistance to the battery
– 200 I3 + 200 I1 + 300 I2 – 300 I3 = 0
is 1.5 A.
Þ 2I1 + 3I2 – 5I3 = 0 ... (i)
ALTERNATE SOLUTION
Applying Kirchoff's law in CDEFBC
6 – 100 I2 – 100 (I2– I3) + 100 I1 = 0
RAB= 1W + 1W = 2W \ IAB = = 3 Amp. Þ I1 – 2I3 + I3 = 0 ... (ii)
2
Further, iCD = iCF as resistances RCD = RCF Multiplying eq. (ii) by 2 and subtracting from (i)
2I1 + 3I2 – 5I3 – 2I1 + 4I2 – 2I3 = 0
\ iCD = iCF = 1.5 A
Þ 7I2 – 7I3 = 0
15. R 1=6W E1=6V Þ I2 = I3 ... (iii)
G A Applying Kirchoff's law in loop ABFEGHA
i1 – 100 I1 – 200 (I1 + I2 – I3) + 10 = 0
5mF=C i1
i2+i1
i2 Þ – 3I1 – 2I2 + 2I3 + 0.1 = 0 ... (iv)
E B
R =2W F R3 =4W Multiplying (ii) by (iv) and adding it with (iv)
2V=E2 2 i2
i2 Þ – 8I2 + 5I3 + 0.1 = 0
D E =3V C Þ – 8I3 + 5I3 + 0.1 = 0
R4=3W i2
3
0.1
Þ 3I3 = 0.1 Þ I3 =
Applying Kirchoff's law in ABFGA 3
6 – (i1 + i2) 4 = 0 ... (i) \ Potential difference across JM
Applying Kirchoff's law in BCDEFB
0.1/ 3 0.1 20
i2 × 3 – 3 – 2 + 2i2 + (i2 + i1) 4 = 0 ... (ii) = ´ 400 = ´ 400 = = 6.67V
2 2´3 3
Putting the value of 4 (i1 + i2) = 6 in (ii)
3i2 – 5 + 2i2 + 6 = 0 ALTERNATE SOLUTION
We can redraw the circuit as.
1
\ i2 = - A E
5
200W 100W
Substituting this value in (i), we get
I/2
æ 1ö G 100W B
i1 = 1.5 – ç - ÷ = 1.7 A M 400W
è 5ø I/2
Therefore current in R3 V 100W
D
= i1 + i2 = 1.7 – 0.2 = 1.5 A 400W
To find the p.d. across the capacitor
VE – 2 – 0.2 × 2 = VG
\ VE – VG = 2.4 V 10V
or V = 24 V
The equivalent resistance between G and D is
1 2
\ Energy stored in capacitor = CV 400 ´ 400
2 RGD = = 200 W
400 + 400
1
= × 5 × 10–6 × (2.4)2 = 1.44 × 10–5 J RGE REB
2 Since, =
RGD RDB
P-352 Topic-wise Solved Papers - PHYSICS
\ It is a case of balanced wheatstone bridge. \ VA - VB = V1 - ir1
The equivalent resistance across G and B is
300 ´ 300 æV +V ö
RGB = = 150 W or VA - VB = V1 - ç 1 2 ÷ r1
300 + 300 è r1 + r2 ø

V 10 1 V1r2 - V2 r1
\ Current I = = = Amp. or VA - VB =
RGB 150 15 r1 + r2
NOTE : Since RGEB = RGDB the current is divided at G into \ Equivalent emf of the battery = V
æ Iö I V1r2 - V2 r1
two equal parts çè ÷ø . The current further divides into \ V=
2 2 r1 + r2
two equal parts at M. 18. q0 = 8.85 × 10–6 C at t = 0
Therefore the potential difference across the voltmeter q = q at t = 12 sec
I 1 400 20 V AV
= ´ 400 = ´ = Volt = 6.67 V Now, I= = ... (i)
4 15 4 3 R rl
17. KEY CONCEPT : The equivalent internal resistance R = Resistance
1 1 1 r1r2 V = Potential difference at t sec.
= +
req r1 r2 , req = r + r
1 2

V K=5 -12
2 E
D s =7.4x10
I r2 _
+ C q
A B A
V1
H r1 G

This is the internal resistance of the single battery which is


equivalent to a parallel combination of the two batteries.
Now applying Kirchoff's law in loop HCDEFGH moving l
clockwise starting from D. V
V1 + V2
– Ir2 + V2 + V1 – Ir1 = 0 Þ I =
r1 + r2 dq AV
\ - =
Now applying Kirchoff's law in branch ACHGB dt rl
VA + Ir1 – V1 = VB
Þ VA – VB = V1 – Ir1 dq A q
or - = ´ (Q q = CV)
(V + V )r dt rl C
Þ VA – VB = V1 – 1 2 1
r1 + r2 dq A l dt æ k e0A ö
or - = çèQ C = ÷
V1r1 + V1r2 - V1r1 - V 2 r1 V1r2 - V 2 r1 q rl k e 0 A l ø
= =
r1 + r2 r1 + r2 dq s æ 1ö
This is the potential difference of the new battery.
or -
q
=
k e0
dt çèQ s = rø÷
ALTERNATE SOLUTION
(i) KEY CONCEPT : EMF of battery is equal to potential s 7.4 ´ 10-12
difference across the terminals, when no current is drawn = = 0.1672
k e0 5 ´ 8.85 ´ 10 -12
from battery (for external circuit)
Current in internal circuit = i dq
or = – 0.1672 dt
Net emf q
\ i=
Total resistance On integrating
q dq t
r2 V2
òq0 q
= - 0.1672ò dt
0

V1 + V2 q
or i= + – loge = – 0.1672 t
r1 + r2 A B
q0
i r1 V1
Þ q = q0e–0.1672 t
CURRENT ELECTRICITY P-353
When t = 12 sec CV
This equation gives Q = (1 – e–2t/3RC)
-6 2
q0 8.85 ´ 10 8.85
q = 0.1672t = 0.1672 ´12 = ´ 10-6 dQ
e e 7.439 (b) i1 =
dt
= 1.1896 × 10–6 C
From (i) d é CV
i1 = ê
dt ë 2
( )
1 - e-2t / 3RC ú
ù
û
sA ql s
\ I= ´ = ´ q = 0.1672 ×1.1896 × 10–6 CV 2 V -2t / 3RC
l K e0 A K e0 = ´ ´ e -2t / 3 RC = e
2 3 RC 3R
I = 0.199 µA From equation (i)
ALTERNATE SOLUTION
V
KEY CONCEPT : The problem can be treated as a V + e-2t /3RC
discharging of R–C circuit. V + i1R 3
i= =
For which q = q0 e–t/t where q0 = Initial charge 2R 2R
q = Charge at time t \ Current through AB
tC = time constant V
V + e -2t / 3 RC
K e0 A p l 3 V
where tC = CR = ´ i2 = i – i1 = - e -2t / 3 RC
l A 2R R
V V -2t / 3 RC
K e 0 5 ´ 8.85 ´ 10-12 i2 = - e
= 2R 6R
= = 5.98 s
s 7.4 ´ 10-12 V
i2 = as t ® ¥
\ q = q0e–t/5.98 2R
æ dq ö q0 - t / 5.89 20. (a) No. There are no positive and negative terminals on
\ Current I = ç - ÷ = e the galvanometer.
è dt ø 5.98
NOTE : Whenever there is no current, the pointer of the
galvanometer is at zero. The pointer swings on both side of
8.85 ´ 10 -6 -12 / 5.89
= e = 0.199 µA zero depending on the direction of current.
5.98
R 0.6 r 12 W
19. Let at any time t charge on capacitor C be Q. Let currents are (b)Q Bridge is balanced AJ = =
RJB 0.4 r X
as shown in fig. Since charge Q will increase with time 't'
Þ x=8W
dQ where r is the resistance per unit length.
therefore i1 =
dt
B X C
A i1 R
S
A JÝ D
N i 12W
i-i1=i2 G
Q+
V R C 21. Battery should be connected across A and B. Out put can
R
be taken across the terminals A and C or B and C.
M B T C
(a) Applying Kirchoff's second law in the loop MNABM
A B
V = (i – i1) R + iR R
or V = 2iR – i1R ... (i)
Similarly, applying Kirchoff's second law in loop MNSTM,
we have 22. For the experimental verification of Ohm's law, ammeter and
voltmeter should be connected as shown in the figure.
Q
V = i1 R + + iR ...(ii) A voltmeter is a high resistance galvanometer (106W) which
C is connected in parallel with the main resistance of 100W.
Eliminating i from equation (1) and (2), we get An ammeter is a low resistance galvanometer (10–3W) which
2Q 2Q is connected in parallel with the main resistance.
V = 3i1R + or 3i1R = V –
C C 100W Ammeter
1 æ 2Q ö dQ 1 æ 2Q ö
çèV -
–3
or i1 = ÷ or = çV - ÷ 10 W
3R C ø dt 3R è Cø
6
dQ dt Q dQ t dt 10 W
or
æ
=
2Q ö 3 R
or ò0 æ 2Q ö = ò0 3R Voltmeter
çè V - ÷ çè V - ÷
Cø C ø
P-354 Topic-wise Solved Papers - PHYSICS
23. KEY CONEPT : At all null points the wheatstone bridge will NOTE : This is a first order linear differential equation whose
be balanced æ R1 + R2 ö
X R çè CR R ÷ø t
\ = integrating factor is e 1 2
r1 r2
Given that at t = 0, Q = 0
X R
r CVR2
Þ X= R 1 \ C1 =
r2 G R1 + R2
r1 r2
where R is a constant
r1 and r2 are variable. M A B C N é æ R +R ö ù
- ç 1 2 ÷t
R=R1 R=R2 R=R3 CVR2 ê CR R ú
The maximum fractional or, Q= ë1 - e è 1 2 ø û
error is R1 + R2
DX D r1 D r2 On comparing with
= +
X r1 r2 Q = Q0 [1 – e–at], we get
Here, Dr1 = Dr2 = y (say) then CVR2 R + R2
DX Q0 = and a = 1
For to be minimum r1 × r2 should be max R1 + R2 CR1 R2
X
[Q r1 + r2 = c (constt.)] ALTERNATE SOLUTION
Let E = r1 × r2 Given Q = Q0[1 – e–at]
Þ E = r1 × (r1 – c) Here Q0 = Maximum change and
dE 1 1
\ = (r1 – c) + r1 = 0
dr1 a= t = C
c Re q
c c
Þ r1 = Þ r2 = Þ r1 = r2 Now the maximum charge
2 2
Þ R2 gives the most accurate value. Q0 = C [V0 ] where V0 = max potential difference across C
24. At any instant of time t, the current flowing the loops are
given in the figure. Applying Kirchoff's law in loop ADEFA. é V ù
Q = C ê R + R ´ R2 ú
V – I R1 – = 0 ... (i) ë 1 2 û
C
R1 and t c = C Req
I D I-I1 C
A
C = 4mF
I1 é RR ù 1 R + R2
+ Q R2 =Cê 1 2 ú \ a = = 1
C_ R
ë 1 + R2û t c CR1R2
V

MATCH THE FOLLOWING :


F E B
Applying Kirchoff's law in loop BCDEB 1. A®s
Reason : Bimetallic strip is based on thermal expansion of
Q solids.
- + (I – I1) R2 = 0 ... (ii)
C B®q
Q Steam engine is based on energy conversion.
From (i) IR1 = V – C ® p, q
C
Substituting in (ii) Incandescent lamp is based on energy conversion and
radiation from a hot body.
Q R2 æ Qö dQ D ® q, r
- + ç V - ÷ø - R2 =0
C R1 è C dt Electric fuse is based on melting point of the fuse material
which is turn depends on the heating effect of current.
Q VR2 R dQ
or, - + - Q 2 - R2 =0
C R1 R1C dt COMPREHENSION BASED Q UESTIONS :

dQ V Q é R2 ù V é R1 + R2 ù 1. (a) From the given graph it is clear that with increase of


= - ê1 + ú = - Q ê ú the magnitude of magnetic field (B), the critical
dt R1 CR2 ë R1 û R1 ë CR1 R2 û temperature TC (B) decreases.
Given B2 > B1. Therefore for B2, the temperature at
dQ æ R1 + R2 ö V
or, + Q= which the resistance becomes zero should be less. The
dt çè CR1 R2 ÷ø R1
above statement is true for graph (a).
CURRENT ELECTRICITY P-355
2. (b) We know that as B increases, TC decreases but the 2
exact dependence is not known. æ E ö
J2 = I 2R = ç ´R
Given at B = 0, TC = 100 K r÷ ...(2)
çR+ ÷
and at B = 7.5T, TC = 75 K è 2ø
\ At B = 5T, TC should be between 75 K and 100 K.
Given J1 = 2.25 J2
ASSERTION & REASON TYPE QUESTIONS :
(2 E )2 E2
1. (d) When the temperature of metal increases; its resistance .R = 2.25 .R
(2r + R ) 2 r
increases. ( R + )2
2
Therefore statement - 2 is correct.
(Unknown resistance) (Standard resistance) 4 2.25
\ =
X S (2r + R )2 æ rö
2

çè R + ÷ø
N 2

\ 4[ R + 0.5]2 = 2.25[2 + R]2 [Qr = 1W]


\ 2 (R + 0.5) = 1.5 (2 + R)
l 100 l \ R = 4W
N
2. The equivalent circuit is shown in the figure.
G R = 1MW, C = 4mF

For a meter bridge when null point N is obtained we get 1MW 4m F


X S
=
l 100 - l
When the unknown resistance is put inside an
enclosure, maintained at a high temperature, then X
increases. To maintain the ratio of null point l should
also increase. But if we want to keep the null point at
10V
the initial position (i.e., if we want no change in the
value of l) there to maintain the ratio, S should be \ The time constant t = RC = 4 sec
increased.
The potential across 4mF capacitor at any time ‘t’ is given as
Therefore statement - 1 is false.
é -t ù
INTEGER VALUE CORRECT TYPE : V = V0 ëê1 - e t ûú
1. Cells connected in series -t -t
é t ù
Þ 0.4 = 1 - e 4 Þ e 4 = 0.6
R R 4 = 10 ëê1 - e 4 ûú
-t
Þ = 2.303log10 0.6 Þ t = 2 sec
I 4
3. 5
Let i be the current flowing in the circuit. Apply Kirchhoff’s
r law in the loop we get
r
E E 2E 2r
2 6v
æ 2E ö 1W
J1 = I 2 R = ç .R
è 2r + R ÷ø
...(1)
i
Cells connected in parallel
A B
R
R
2W i
3v
I
E r
–3 – 2i – i + 6 = 0
\ 3i = 3
E r
E r 2 \ i = 1 Amp
P-356 Topic-wise Solved Papers - PHYSICS
Now let us travel in the circuit from A to B through battery Alternatively, when dissimilar battery are corrected in parallel
of 6V, we get
VA – 6 + 1 × 1 = VB E1 E2
+
\ VA – VB = 5 volt. r1 r2
then the emf of the combination is E = 1 1
+
r1 r2

1. (c) KEY CONCEPT : To convert a galvanometer into a 5. (b) According to Faraday's first law of electrolysis
voltmeter we connect a high resistance in series with m = ZIt Þ m µ It
the galvanometer. qi + qc
The same procedure needs to be done if ammeter is to 6. (c) qn = .
2
be used as a voltmeter.
7. (d) From the principle of potentiometer, V µ l
R
V l
2 Þ = ; where
V E L
2. (b) Case 1 : P1 =
R V = emf of battery, E = emf of standard cell.
V L = length of potentiometer wire
Case 2 : The wire is cut into two equal pieces. Therefore El 30E
V= =
R L 100
the resistance of the individual wire is . These are
2 E
connected in parallel
R/2 i
R/4
R/2 i
R/2 R =
\ Req = = r
2 4
V V E'

V2 æ V2 ö NOTE In this arrangement, the internal resistance


\ P2 = R / 4 = 4 ç R ÷ = 4P1 of the battery E does not play any role as current is not
è ø
passing through the battery.
3. (b) When current is passed through a spring then current 8. (a) Let q be the smallest temperature difference that can
flows parallel in the adjacent turns.
be detected by the thermocouple, then I × R
NOTE : When two wires are placed parallel to each
= (25 × 10–6) q
other and current flows in the same direction, the wires
attract each other. where I is the smallest current which can be detected
by the galvanometer of resistance R.
Similarly here the various turns attract each other and
the spring will compress. Here I = 10–5 A, R = 40 W
\ 10–5 × 40 = 25 × 10 – 6 × q
\ q = 16°C.
9. (c) According to Faraday’s first law of electrolysis
2´R m = Z×q
4. (b) The equivalent resistance is Req = For same q, mµ Z
2+ R
mCu ZCu
V2 \ =
\ Power dissipation P = mZn Z Zn
Re q
Z Cu 31.5
Þ mCu = ´ mZn = ´ 0.13 = 0.126 g
15 ´ 15 15 3 Z Zn 32.5
\ 15 0 = R \ Req = =
eq 10 2 10. (d) ig × G = (i – ig) S

2R 3 ig ´ G 1 ´ 0.81
Þ = Þ 4 R = 6 + 3R Þ R = 6W \ S= = = 0.09W
2+ R 2 i - ig 10 - 1
CURRENT ELECTRICITY P-357

3 ´ 6 18 R1 i1
11. (b) Rp = = = 2W
3+ 6 9
R2 i2
V 3 15. (b)
\ V = IR Þ I = = = 1.5 A
R 2
V
3W 3W
3V rl1 rl2
R1 = ; R2 =
pr12 pr22
i1R1 = i2R2 (same potential difference)
3W
i1 R2 l 2 r12 3 4 1
I \ = = ´ = ´ =
i2 R1 l1 r2 2
4 9 3
3V 6W 3V
3W 2W
R1 l1
=
16. (c) R2 l 2 where l2 = 100 – l1

2
Vrated (220)2 In the first case X = 20
12. (c) We know that R = = Y 80
Prated 1000
In the second case 4 X = l
When this bulb is connected to 110 volt mains supply Þ l = 50
we get Y 100 - l
17. (a) Thermistors are usually made of metaloxides with high
V 2 (110)2 ´ 1000 1000 temperature coefficient of resistivity.
P= = = = 250W
R (220)2 4 18. (a) DQ = mC ´ DT
= 1 × 4180 × (40 – 10) = 4180 × 30
6V ( \ DQ = heat supplied in time t for heating 1L water
from 10°C to 40°C)
2W
1.5W 4180 ´ 30
13. (a) also DQ = 836 ´ t Þ t = = 150 s
836
6W 3W 19. (d) Neutral temperature is the temperature of a hot junction
at which E is maximum.
6V 6V dE -a
Þ = 0 or a + 2bq = 0 Þ q = = -350
dq 2b
3/2W 3/2W 3W
d2E
Þ = 2b
3W 3W d q2
hence no q is possible for E to be maximum no neutral
6 temperature is possible.
hence Req = 3/2; \I = = 4A
3/ 2 20. (c) The mass liberated m, electrochemical equivalent of a
R1 R2 metal Z, are related as
14. (c) m = Zit
Resistance of the series combination, S = R1 + R2
Resistance of the parallel combination, Þ m = 3.3 ´ 10 -7 ´ 3 ´ 2 = 19.8 ´ 10 -7 kg

R1 R2 21. (b) i i
P=
R1 + R2 F
n( R1R2 )
S = nP Þ R1 + R2 =
( R1 + R2 )

Þ ( R1 + R2 )2 = nR1 R2
F m 0 i1 i2 m 0 i 2
Minimum value of n is 4 = =
l 2 pd 2 pd
for that ( R1 + R2 )2 = 4 R1 R2 Þ ( R1 - R2 )2 = 0 (attractive as current is in the same direction)
P-358 Topic-wise Solved Papers - PHYSICS

V 2t Also q = q1 + q2 .... (ii)


22. (b) H =
R q q
Þ = 1 +1 (Dividing (ii) by q2 )
R q2 q2
Resistance of half the coil =
2 q
\ As R reduces to half, ‘H’ will be doubled. Þ q2 = q
.... (iii)
1+ 1
500W q2
23. (a)
A
i
q
From equations (i) and (iii), q2 =
2V Z
R 1+ 2
12V Z1
27. (c) The internal resistance of the cell,
æl -l ö 240 - 120
10 1 r= ç 1 2÷ ´R = ´ 2 = 2W
12 – 2 = (500W)i Þ i = = è l 2 ø 120
500 50
12 1 V2
Again, i = = 28. (b) P = Vi =
500 + R 50 R
Þ 500 + R = 600 Þ R = 100 W V 2 200 ´ 200
24. (c) KEY CONCEPT : Resistance of Galvanometer, Rhot = = = 400 W
P 100
Current sensitivity 10
G= Þ G= = 5W 400
Voltage sensitivity 2 Rcold = = 40 W
10
150 E
Here i g = Full scale deflection current = = 15 mA 29. (d) I = , Internal resistance (r) is
10 R+r
V = voltage to be measured = 150 volts
E
(such that each division reads 1 volt) zero, I == constant.
R
150
Þ R= - 5 = 9995W 30. (d) NOTE : Kirchhoff's first law is based on conservation
15 ´ 10 -3 of charge and Kirchhoff's second law is based on
conservation of energy.
25. (a) 31. (d) rB = 2rA
R1 R2
I dB = 2dA
rB l B r A l A
RB = RA Þ A = A
B A

l r A d B2 r 4d 2
\ B = ´ = A ´ A =2
lA rB d A2 2r A d A2
R
32. (d) At cold junction, current flows from Antimony to
2e Bismuth (because current flows from metal occurring
I= later in the series to metal occurring earlier in the
R + R1 + R 2 thermoelectric series).
Potential difference across second cell = V = e - iR 2 = 0 33. (b) The network of resistors is a balanced wheatstone
bridge. The equivalent circuit is
2e 10W 20W
e – .R2 = 0
R + R1 + R 2 30W
10W
R + R1 + R 2 - 2R 2 = 0
R + R1 - R 2 = 0 \ R = R 2 - R1
15W
26. (a) Mass deposited 5W 10W
5V 5V 5V
1 Þ Z1 = q2
m = Zq Þ Zµ .... (i) 15 ´ 30 V 5
q Z 2 q1 Req = = 10 W Þ I = = = 0.5 A
15 + 30 R 10
CURRENT ELECTRICITY P-359
34. (b) R1 = R0 [1 + a × 100] = 100 ....(1) 40. (a) Let j be the current density.
R2 = R0 [1 + a × T] = 200 ....(2) I rI
On dividing we get Then j ´ 2pr 2 = I Þ j = \ E = rj =
2pr 2 2pr 2
200 1 + aT 1 + 0.005 T
= Þ2= Þ T = 400°C '
a r uur a
rI
100 1 + 100a 1 + 100 ´ 0.005 Now, DVBC =- ò E. dr = - ò 2
dr
NOTE : We may use this expression as an approximation a +b a + b 2 pr
because the difference in the answers is appreciable. a
rI é 1 ù rI rI
For accurate results one should use =- - = -
R = R0eaDT 2p ëê r ûú a + b 2pa 2p (a + b)
On applying superposition as mentioned we get
P R SS
35. (b) = where S = 1 2 ' rI rI
Q S S1 + S2 DVBC = 2 ´ DVBC = -
pa p(a + b)
V 2 (220) 2 rI
36. (c) The resistance of the bulb is R = = 41. (c) As shown above E=
P 100 2pr 2
The power consumed when operated at 110 V is 42. (c) Applying kirchoff's loop law in AB P2P1A we get
-2i + 5 - 10 i1 = 0 .....(i)
(110)2 100
P= = = 25 W i P2 i–i1
2 4 B C
(220) /100
i1
37. (a) Required ratio 2V
10W
5V 1W
1 2
Energy stored in capacitor 2 CV 2W
= =
Workdone by the battery Ce 2 A P1 D
where C = Capacitance of capacitor Again applying kirchoff's loop law in P2 CDP1P2 we
V = Potential difference, get, 10 i1 + 2 – i + i1= 0 .....(ii)
e = emf of battery
é 5 - 10i1 ù
From (i) and (ii) 11 i1 + 2 - ê ú=0
1 2 ë 2 û
Ce 1
= 2 = ( Q V = e) 1
Ce 2 2 Þ i1 = A from P2 to P1
32
38. (d) KEY CONCEPT : We know that
Rt = R0 (1 + at ), q2
43. (c) Initial energy of capacitor, E1 = 1
where Rt is the resistance of the wire at t ºC, 2C
2
R0 is the resistance of the wire at 0ºC æ q1 ö
1 q2
and a is the temperature coefficient of resistance. Final energy of capacitor, E2 = E1 = 1 = ç 2 ÷
Þ R50 = R0 (1 + 50 a ) 2 4C ç ÷
... (i) è 2C ø
R100 = R0 (1 + 100 a ) ... (ii) 1
\ t1 = time for the charge to reduce to 2
of its initial value
From (i), R50 – R0 = 50 aR0 ... (iii)
From (ii), R100 – R0 = 100 aR0 ... (iv) 1
and t2 = time for the charge to reduce to of its
4
Dividing (iii) by (iv), we get
initial value
R50 - R0 1 We have, q2 = q1e-t / CR
=
R100 - R0 2
1 ö -t1
Here, R50 = 5W and R100 = 6W Þ ln æç q2 ö÷ = - t \ ln æç ÷= ...(1)
è q1 ø CR è 2 ø CR
5 - R0 1
\ = æ 1 ö -t2
6 - R0 2 and ln ç ÷ = ...(2)
è 4 ø CR
or, 6 – R0 = 10 – 2 R0 or, R0 = 4W .
æ 1 ö æ1ö
39. (b) According to the condition of balancing
t1
ln ç ÷ 1 ln çè 2 ÷ø 1
By (1) and (2) , = è 2 ø = =
2ln æç ö÷
55 R 2 1 4
ln æç ö÷
t2 1
= Þ R = 220W
20 80 è4ø è 2 ø
P-360 Topic-wise Solved Papers - PHYSICS
44. (d) R1 = R0 [1 + a1Dt ] ; R2 = R0 [1 + a2 D t ] 440
I= , Reff = R1 + R2
Reff
In Series, R = R1 + R2

é æ a1 + a 2 ö ù V12 (220)2 V 2 (220)2


= R0 [ 2 + (a1 + a 2 )Dt ] = 2 R0 ê1 + ç ÷ Dt ú R1 = = ; R2 = 2 =
ë è 2 ø û P1 25 P 100

a1 + a 2 440 440
\ a eq = I= 2 2
=
2 (220) (220) é1 1 ù
+ (220)2 ê +
25 100 ë 25 100 ûú
1 1 1 1 1
In Parallel , = + = R 1 + a Dt + R 1 + a Dt
R R1 R2 0[ 1 ] 0[ 2 ] 40
I= Amp
220
1 1 1
Þ = +
R0 R0 (1 + a1Dt ) R0 (1 + a 2 Dt ) æ 25 ö æ 40 ö æ 100 ö
(1 + aeq Dt ) Q I1 ç = A÷ < I ç = A ÷ < I2 ç = A÷
2 è 220 ø è 220 ø è 200 ø
Thus the bulb marked 25W-220 will fuse.
a1 + a 2
2(1 - a eq Dt ) = (1 - a1Dt )(1 - a 2 Dt ) \ a eq =
2
45. (a) Resistance of wire
6W
rl rl 2
R= = (where Al = C) Bulb
A C (Lead)
\ Fractional change in resistance 49. (d)
DR Dl
=2
R l
\ Resistance will increase by 0.2%
46. (a) 120 V
R R R R
R = 100 ± 5 Þ 4R = 400 ± 20 Power of bulb = 60 W (given)
Thus, tolerance of combination is also 5%. é
120 ´ 120 V2 ù
47. (c) Potential gradient Resistance of bulb = = 240W êQ P = ú
60 ë R û
V IR I æ rl ö I r Power of heater = 240W (given)
x= = = ç ÷=
l l lè A ø A
120 ´120
Resistance of heater = = 60W
-7 240
0.2 ´ 4 ´ 100.8
x= =
= 0.1 V/m
8 ´ 10 -78 Voltage across bulb before heater is switched on,
48. (c) The current upto which bulb of marked 25W -220V, will 240
V1 = ´120 = 117.73 volt
W1 25 246
not fuse I1 = V = 220 Amp Voltage across bulb after heater is switched on,
1
48
W2 100 V2 = ´120 = 106.66 volt
Similarly, I2 = V = 220 Amp 54
2
Hence decrease in voltage
The current flowing through the circuit V1 – V2 = 117.073 – 106.66 = 10.04 Volt (approximately)
50. (d) Statements I is false and Statement II is true
B1 B2
IgG
For ammeter, shunt resistance, S =
I – Ig

R1 Therefore for I to increase, S should decrease, So


r2
additional S can be connected across it.

440V
14
Moving Charges &
Magnetism

FILL IN THE BLANKS : z


D
1. A neutron, a proton, and an electron and an alpha particle
C
enter a region of constant magnetic field with equal velocities. I
The magnetic field is along the inward normal to the plane
of the paper. The tracks of the particles are labelled in fig.
F
The electron follows track ...... and the alpha particle follows
E
track ..... (1984- 2 Marks) y

B A
B C
x
A
B
D Y
6.
E
G
F
2. A wire of length L metre, carrying a current i ampere is bent H
X
in the form of a circle. The magnitude of its magnetic moment
B I
is ........in MKS units. (1987 - 2 Marks)
A
3. In a hydrogen atom, the electron moves in an orbit of radius
C
0.5 Å making 1016 revolutions per second. The magnetic D
moment associated with the orbital motion of the electron is Z
............ (1988 - 2 Marks) A metallic block carrying current I is subjected to a uniform
ur
4. The wire loop PQRSP formed by joining two semicircular magnetic induction B as shown in Figure . The moving
ur
wires of radii R1 and R2 carries a current I as shown. The charges experience a force F given by .... which results in
magnitude of the magnetic induction at the centre C is the lowering of the potential of the face .... Assume the
.................... (1988 - 2 Marks) speed of the carriers to be v. (1996 - 2 Marks)
7. A uniform magnetic field with a slit system as shown in
figure is to be used as a momentum filter for high-energy
I charged particles. With a field B tesla, it is found that the
I filter transmits a- particles each of energy 5.3 MeV. The
magnetic field is increased to 2.3 B tesla and deuterons are

S R
R2
.
C
R1
Q P
passed into the filter. The energy of each deuteron
transmitted by the filter is ...... MeV. (1997C - 1 Mark)

B
5. A wire ABCDEF (with each side of length L) bent as shown
in figure and carrying a current I is placed in a uniform
magnetic induction B parallel to the positive y-direction.
The force experienced by the wire is ............ in the .............
Source Detector
direction. (1990 - 2 Marks)
1 Current Electricity
FILL IN THE BLANKS : MCQ's WITH ONE CORRECT ANSWER :

1. An electric bulb rated for 500 watts at 100 volts is used in a 1. The temperature coefficient of resistance of a wire is 0.00125
circuit having a 200 volts supply. The resistance R that per °C. At 300 K, its resistance is 1 ohm. This resistance of
the wire will be 2 ohm at.
must be put in series with the bulb, so that the bulb delivers
(a) 1154 K (b) 1100 K (1980)
500 watt is ..........ohm. (1987 - 2 Marks) (c) 1400 K (d) 1127 K
2. The equivalent resistance between points A and B of the 2. A constant voltage is applied between the two ends of a
circuit given below is ......W . (1997 - 2 Marks) uniform metallic wire. Some heat is developed in it. The heat
developed is doubled if (1980)
(a) both the length and the radius of the wire are halved.
2R 2R R B (b) both the length and the radius of the wire are doubled.
A
(c) the radius of the wire is doubled.
(d) the length of the wire is doubled.
3. The electrostatic field due to a point charge depends on the
3. In the circuit shown below, each battery is 5V and has an
1
internal resistance of 0.2 ohm. distance r as 2 . Indicate which of the following quantities
r
shows same dependence on r. (1980)
(a) Intensity of light from a point source.
V (b) Electrostatic potential due to a point charge.
(c) Electrostatic potential at a distance r from the centre of a
charged metallic sphere. Given r < radius of the sphere.
(d) None of these
The reading in the ideal voltmeter V is ..... V. (1997 - 2 Marks) 4. In the circuit shown in fig the heat produced in the 5 ohm
resistor due to the current flowing through it is 10 calories
TRUE / FALSE :
per second. (1981- 2 Marks)
1. In an electrolytic solution the electric current is mainly due
4W 6W
to the movement of free electrons. (1980)
2. Electrons in a conductor have no motion in the absence of
a potential difference across it. (1982 - 2 Marks) 5W
3. The current –voltage graphs for a given metallic wire at The heat generated in the 4 ohms resistor is
two different temperatures T1 and T2 are shown in the figure. (a) 1 calorie / sec (b) 2 calories /sec
(1985 - 3 Marks) (c) 3 calories /sec (d) 4 calories /sec
5. The current i in the circuit (see Fig) is (1983 - 1 Mark)
T1
i
I T2
30W 30W
2V
V 30W
The temperature T2 is greater than T1.
MOVING CHARGES & MAGNETISM P-363
15. The magnetic field lines due to a bar magnet are correctly
(a)
1
2
(
- ˆj + kˆ ) (b)
1
3
(- ˆj + kˆ + iˆ ) shown in (2002S)
N N

(c)
3
(
1 ˆ ˆ ˆ
i + j+k ) (d)
1 ˆ ˆ
2
i +k ( )
11. (a) (b)
Two particles A and B of masses mA and mB respectively
and having the same charge are moving in a plane. A uniform
S S
magnetic field exists perpendicular to this plane. The speeds
of the particles are vA and vB respectively and the trajectories
N N
are as shown in the figure. Then (2001S)

A
(c) (d)

B S S

16. For a positively charged particle moving in a x-y plane initially


along the x-axis, there is a sudden change in its path due to
(a) mAvA < mBvB (b) mAvA > mBvB the presence of electric and/or magnetic fields beyond P.
The curved path is shown in the x-y plane and is found to be
(c) mA < mB and vA < vB (d) mA = mB and vA = vB non-circular. Which one of the following combinations is
12. A coil having N turns is wound tightly in the form of a spiral possible? (2003S)
with inner and outer radii a and b respectively. When a
y
current I passes through the coil, the magnetic field at the
center is (2001S)

m o NI 2m o NI
(a) (b)
b a P x

m o NI b m0 IN a
(c) ln (d) ln
2(b - a ) a 2 (b - a ) b
uur uur uur uur
13. A particle of mass m and charge q moves with a constant (a)E = 0; B = biˆ + ckˆ (b) E = aiˆ; B = ckˆ + aiˆ
velocity v along the positive x-direction. It enters a region uur uur uur uur
(c) E = 0; B = cjˆ + bkˆ (d) E = aiˆ; B = ckˆ + bjˆ
containing a uniform magnetic field B directed along the
negative z-direction, extending from x = a to x = b. The 17. A conducting loop carrying a current I is placed in a uniform
magnetic field pointing into the plane of the paper as shown.
minimum value of v required so that the particle can just
The loop will have a tendency to (2003S)
enter the region x > b is (2002S)
B Y
qbB q (b - a ) B
(a) (b)
m m
x
qaB q (b + a ) B I
(c) (d)
m 2m
14. A long straight wire along the Z-axis carries a current I in the
r
negative Z-direction. The magnetic vector field B at a point (a) contract (b) expand
having coordinates (x, y) in the Z = 0 plane is (2002S) (c) move towards +ve x-axis (d) move towards -ve x-axis.
18. A current carrying loop is placed in a uniform magnetic field
(a)
(
m 0 I yiˆ - xjˆ ) (b)
(
m 0 I xiˆ + yjˆ ) in four different orientations, I, II, III & IV arrange them in

(
2p x 2 + y 2 ) (
2p x + y 2 2
) the decreasing order of Potential Energy (2003S)

(c)
(
m 0 I xjˆ - yiˆ ) (d)
(
m 0 I xiˆ - yjˆ ) I.
^
n
B

(
2p x + y 2 2
) (
2p x + y 2 2
)
P-364 Topic-wise Solved Papers - PHYSICS

II. B z

n^
(c) x
a 2a 3a
III B
n^
^
n
z
IV B
a 2a 3a
(d) x
(a) I > III > II > IV (b) I > II > III > IV
(c) I > IV > II > III (d) III > IV > I > II
19. An electron travelling with a speed u along the positive x-
axis enters into a region of magnetic field where B = –B0 k̂ 21. A thin flexible wire of length L is connected to two adjacent
(x > 0). It comes out of the region with speed v then (2004S) fixed points and carries a current I in the clockwise direction,
as shown in the figure. When the system is put in a uniform
×B magnetic field of strength B going into the plane of the
y
paper, the wire takes the shape of a circle. The tension in the
e– u
wire is (2010)

(a) v = u at y > 0 (b) v = u at y < 0


(c) v > u at y > 0 (d) v > u at y < 0
ur
20. A magnetic field B = B0 Jˆ , exists in the region a < x < 2a, IBL
uv (a) IBL (b)
and B = – B0 ˆj , in the region 2a < x < 3a, where B0 is a p
positive constant. A positive point charge moving with a IBL IBL
r (c) (d)
velocity v = v0iˆ , where v0 is a positive constant, enters the 2p 4p
magnetic field at x = a. The trajectory of the charge in this 22. A long insulated copper wire is closely wound as a spiral of
region can be like (2007) ‘N’ turns. The spiral has inner radius ‘a’ and outer radius
‘b’. The spiral lies in the XY plane and a steady current ‘I’
B0 flows through the wire. The Z-component of the magnetic
field at the centre of the spiral is (2011)
0 x
a 2a 3a

-B 0

(a) x
a 2a 3a

z
m0 NI æbö m0 NI æb+ aö
(a) ln ç ÷ (b) ln çè ÷
2(b - a) èaø 2(b - a) b-aø
(b) x
a 2a 3a
m0 NI æbö m0 NI æb+ aö
(c) ln ç ÷ (d) ln çè ÷
2b èaø 2b b-aø
MOVING CHARGES & MAGNETISM P-365
23. A loop carrying current I lies in the x-y plane as shown in 3. A rectangular loop carrying a current i is situated near a
long straight wire such that the wire is parallel to one of the
the figure. The unit vector k̂ is coming out of the plane of
the paper. The magnetic moment of the current loop is sides of the loop and is in the plane of the loop. If steady
(2012) current I is established in the wire as shown in the figure,
the loop will : (1985 - 2 Marks)

i
I

a 2 I kˆ æ p ö a 2 I kˆ (a) rotate about an axis parallel to the wire


(a) (b) çè + 1÷ø
2 (b) move away from the wire
(c) move towards the wire
p
(c) - æç + 1ö÷ a 2 I kˆ (d) (2p + 1)a2 I kˆ (d) remain stationary
è2 ø
4. Two thin long parallel wires seperated by a distance ‘b’ are
24. An infinitely long hollow conducting cylinder with inner carrying a current ‘i’ amp each. The magnitude of the force
radius R/2 and outer radius R carries a uniform current per unit lenght exerted by one wire on the other is
density along its length. The magnitude of the magnetic (1986 - 2 Marks)
r
field, B as a function of the radial distance r from the axis
m 0i 2 m 0i 2
is best represented by (2012) (a) (b)
b2 2pb
(a)
r
B m 0i m 0i
(c) (d)
2 pb 2pb 2
5. Two particles X and Y having equal charges, after being
accelerated through the same potential difference, enter a
(b) r region of uniform magnetic field and describe circular paths
B of radii R1 and R2 respectively. The ratio of the mass of X
to that of Y is (1988 - 2 Marks)
(a) ( R1 / R2 )1/ 2 (b) R2 / R1
(c) r
B (c) ( R1 / R2 )2 (d) R1 / R2 .
6. A particle of charge + q and mass m moving under the
influence of a uniform electric field Eiˆ and uniform magnetic
(d) r field B k̂ follows a trajectory from P to Q as shown in fig.
B
The velocities at P and Q are viˆ and – 2vjˆ . Which of the
following statement (s) is/are correct ? (1991 - 2 Marks)

v E
P
1. A magnetic needle is kept in a non uniform magnetic field. It
experiences (1982 - 3 Marks)
(a) a force and a torque aa B
(b) a force but not a torque
(c) a torque but not a force Q
(d) neither a force nor a torque x
2. A proton moving with a constant velocity passes through 2a 2v
a region of space without any change in its velocity. If E
an d B represent th e electric and magnetic fields
respectively, this region of space may have :
3 é mv 2 ù
(1985 - 2 Marks) (a) E = ê ú
(a) E = 0, B = 0 (b) E = 0, B ¹ 0 4 ëê qa ûú
(c) E ¹ 0, B = 0 (d) E ¹ 0, B ¹ 0
P-366 Topic-wise Solved Papers - PHYSICS
12. The following field line can never represent (2006 - 5M, –1)
3 é mv 3 ù
(b) Rate of work done by the electric field at P is ê ú
4 êë a úû
(c) Rate of work done by the electric field at P is zero
(d) Rate of work done by both the fields at Q is zero
7. A microameter has a resistance of 100 W and a full scale (a) induced electric field
(b) magnetostatic field
range of 50 mA . It can be used as a voltmeter or as a higher
(c) gravitational field of a mass at rest
range ammeter provided a resistance is added to it. Pick the (d) electrostatic field
correct range and resistance combination (s) 13. A long current carrying wire, carrying current I1 such that I1
(1991 - 2 Marks) is flowing out from the plane of paper is placed at O. A
(a) 50 V range with 10 kW resistance in series steady state current I2 is flowing in the loop ABCD
(b) 10 V range with 200 kW resistance in series
(2006 - 5M, –1)
(c) 5 mA range with 1W resistance in parallel
(d) 10 mA range with 1W resistance in parallel C
8. A current I flows along the length of an infinitely long, B
straight, thin-walled pipe. Then (1993-2 Marks) I2
(a) the magnetic field at all points inside the pipe is the O
same, but not zero. O'
(b) the magnetic field at any point inside the pipe is zero I1
(c) the magnetic field is zero only on the axis of the pipe A
(d) the magnetic field is different at different points inside D
the pipe.
9. H+, He+ and O++ all having the same kinetic energy pass (a) the net force is zero
through a region in which there is a uniform magnetic field (b) the net torque is zero
perpendicular to their velocity. The masses of H+, He+ and (c) as seen from O, the loop will rotate in clockwise along
O2+ are 1 amu, 4 amu and 16 amu respectively. Then OO' axis
(1994 - 2 Marks) (d) as seen from O, the loop will rotate in anticlockwise
(a) H+ will be deflected most direction along OO' axis
(b) O2+ will be deflected most 14. A particle of mass m and charge q, moving with velocity v
(c) He+ and O2+ will be deflected equally enters Region II normal to the boundary as shown in the
(d) all will be deflected equally figure. Region II has a uniform magnetic field B perpendicu-
10. Two particles, each of mass m and charge q, are attached to lar to the plane of the paper. The length of the Region II is l
the two ends of a light rigid rod of length 2R. The rod is Choose the correct choice(s). (2008)
rotated at constant angular speed about a pependicular axis
passing through its centre. The ratio of the magnitudes of Region II Region III
Region I
the magnetic moment of the system and its angular ××××××
momentum about the centre of the rod is (1998S - 2 Marks) ××××××
q q ××××××
(a) (b) V ××××××
2m m
××××××
2q q
(c) (d)
m pm
(a) The particle enters Region III only if its velocity
11. Two very long, straight, parallel wires carry steady currents
I & -I respectively. The distance between the wires is d. At qlB
v>
a certain instant of time, a point charge q is at a point m
equidistant from the two wires, in the plane of the wires. Its (b) The particle enters Region III only if its velocity
instantaneous velocity v is perpendicular to this plane. The
magnitude of the force due to the magnetic field acting on qlB
v<
the charge at this instant is (1998S - 2 Marks) m
(c) Path length of the particle in Region II is maximum when
m 0 Iqv m 0 Iqv
(a) (b) qlB
2 pd pd
velocity v =
m
2m0 Iqv
(c) (d) 0 (d) Time spent in Region II is same for any velocity v as
pd long as the particle returns to Region I
MOVING CHARGES & MAGNETISM P-367
15. Two metallic rings A and B, identical in shape and size but 19. A steady current I flows along an infinitely long hollow
having different resistivities rA and rB, are kept on top of cylindrical conductor of radius R. This cylinder is placed
two identical solenoids as shown in the figure. When current coaxially inside an infinite solenoid of radius 2R. The
I is switched on in both the solenoids in identical manner, solenoid has n turns per unit length and carries a steady
the rings A and B jump to heights hA and hB, respectively, current I. Consider a point P at a distance r from the common
with hA > hB. The possible relation(s) between their
axis. The correct statement(s) is (are) (JEE Adv. 2013)
resistivities and their masses mA and mB is(are) (2009)
(a) In the region 0 < r < R, the magnetic field is non-zero
A B (b) In the region R < r < 2R, the magnetic field is along the
common axis
(c) In the region R < r < 2R, the magnetic field is tangential
to the circle of radius r, centered on the axis
(d) In the region r > 2R, the magnetic field is non-zero

SUBJECTIVE PROBLEMS :
(a) rA > rB and mA = mB (b) rA< rB and mA = mB
1. A bar magnet with poles 25 cm apart and of strength 14.4
(c) rA > rB and mA > mB (d) rA < rB and mA < mB
amp-m rests with centre on a frictionless pivot. It is held in
16. An electron and a proton are moving on straight parallel
paths with same velocity. They enter a semi infinite region equilibrium at an angle of 60° with respect to a uniform
of uniform magnetic field perpendicular to the velocity. magnetic field of induction 0.25 Wb/m2, by applying a force
Which of the following statement(s) is / are true? (2011) F at right angles to its axis at a point 12 cm from pivot.
(a) They will never come out of the magnetic field region. Calculate F. What will happen if the force F is removed?
(b) They will come out travelling along parallel paths. (1978)
(c) They will come out at the same time. 2. A bar magnet is placed with its north pole pointing north
(d) They will come out at different times. and its south pole pointing south. Draw a figure to show
17. Consider the motion of a positive point charge in a region
the location of neutral points. (1979)
where there are simultaneous uniform electric and magnetic
r r 3. A potential difference of 600 volts is applied across the
fields E = E0 ˆj and B = B0 ˆj . At time t = 0, this charge has plates of a parallel plate condenser. The separation between
velocity vr in the in the x-y plane, making an angle q with the the plates is 3 mm. An electron projected vertically, parallel
x-axis. Which of the following option(s) is (are) correct for to the plates, with a velocity of 2 × 106 m/sec moves
time t > 0? (2012) undeflected between the plates. Find the magnitude and
(a) If q = 0°, the charge moves in a circular path in the x-z direction of the magnetic field in the region between the
plane. condenser plates. (Neglect the edge effects). (Charge of the
(b) If q = 0°, the charge undergoes helical motion with electron = – 1.6 × 10–19 coulomb) (1981- 3 Marks)
constant pitch along the y-axis.
(c) If q = 10°, the charge undergoes helical motion with its 600 VOLTS
pitch increasing with time, along the y-axis. +

(d) If q = 90°, the charge undergoes linear but accelerated


motion along the y-axis.
18. A particle of mass M and positive charge Q, moving with a
r
constant velocity u1 = 4iˆ ms-1 , enters a region of uniform +

static magnetic field, normal to the x-y plane. The region of


the magnetic field extends from x = 0 to x = L for all values of
y. After passing through this region, the particle emerges on 4. A particle of mass 1 × 10–26 kg and charge +1.6 × 10–19
the other side after 10 milliseconds with a velocity coulomb travelling with a velocity 1.28 × 106 m/s in the +X
r direction enters a region in which a uniform electric field E
u2 = 2( 3iˆ + ˆj ) ms -1 . The correct statement(s) is (are)
and a uniform magnetic field of induction B are present such
(JEE Adv. 2013) that Ex = Ey = 0, Ez = –102.4 kV/m and Bx = Bz = 0, By = 8 × 10–
(a) The direction of the magnetic field is – z direction 2 weber/m2 . The particle enters this region at the origin at
(b) The direction of the magnetic field is +z direction
time t = 0. Determine the location (x, y and z coordinates) of
50pM the particle at t = 5 × 10–6 s. If the electric field is switched
(c) The magnitude of the magnetic field units
3Q off at this instant (with the magnetic field still present), what
will be the position of the particle at t = 7.45 × 10–6 s?
100pM (1982 - 7 Marks)
(d) The magnitude of the magnetic field is units
3Q
P-368 Topic-wise Solved Papers - PHYSICS
5. A particle of mass m = 1.6 × 10–27 kg and charge q = 1.6 × 10–19 C
enters a region of uniform magnetic field of strength 1 tesla Y
along the direction shown in fig. The speed of the particle is ¥
107 m/s. (i) The magnetic field is directed along the inward
normal to the plane of the paper. The particle leaves the Ii
region of the field at the point F. Find the distance EF and Q
the angle q. (ii) If the direction of the field is along the L R M
¥ ¥ X
outward normal to the plane of the paper, find the time spent O S
Ii P Ii
by the particle in the region of the magnetic field after
entering it at E. (1984- 8 Marks)
Ii
q ¥
F
9. Two long parallel wires carrying current 2.5 amperes and I
E ampere in the same direction (directed into the plane of the
paper ) are held at P and Q respectively such that they are
45°
perpendicular to the plane of paper. The points P and Q are
6. A beam of protons with a velocity 4 × 105 m/sec enters a uniform located at a distance of 5 metres and 2 metres respectively
magnetic field of 0.3 tesla at an angle of 60° to the magnetic from a collinear point R (see figure) (1990 - 8 Marks)
field. Find the radius of the helical path taken by the proton
beam. Also find the pitch of the helix (which is the distance 5m
2m
travelled by a proton in the beam parallel to the magnetic field P Q R
during one period of rotation). (1986 - 6 Marks) - -O
X - - - -OX - - - - O - - ............. ® x
7. Two long straight parallel wires are 2 metres apart, 2.5A IA
perpendicular to the plane of the paper (see figure). The (i) An electron moving with a velocity of 4 × 105 m/s along
wire A carries a current of 9.6 amps, directed into the plane the positive x - direction experiences a force of
of the paper. The wire B carries a current such that the
magnitude 3.2 × 10–20 N at the point R. Find the value
magnetic field of induction at the point P, at a distance of
of I.
10 (ii) Find all the positions at which a third long parallel wire
metre from the wire B, is zero. (1987 - 7 Marks)
11 A × carrying a current of magnitude 2.5 amperes may be
placed so that the magnetic induction at R is zero.
10. A wire loop carrying a current I is placed in the x-y plane as
1.6

shown in fig. (1991 - 4 + 4 Marks)


m
2m

S M
m y
1.2 a +Q
B
120°

I P
O x
10/ m
11 N

P (a) If a particle with charge +Q and mass m is placed at the


Find : uur
(i) The magnitude and direction of the current in B . centre P and given a velocity v along NP (see figure),
(ii) The magnitude of the magnetic field of induction at the find its instantaneous acceleration.
point S. (b) If an external uniform magnetic induction field
(iii) The force per unit length on the wire B. uur
8. A pair of stationary and infinitely long bent wires are placed B = Biˆ is applied, find the force and the torque acting
in the XY plane as shown in fig. The wires carry currents of on the loop due to this field.
i = 10 amperes each as shown. The segments L and M are 11. A straight segment OC (of length L meter) of a circuit carrying
along the X-axis. The segments P and Q are parallel to the Y- a current I amp is placed along the x-axis (Fig.). Two infinetely
axis such that OS = OR = 0.02 m. Find the magnitude and long straight wires A and B, each extending from z = - ¥ to
direction of the magnetic induction at the origin O. +¥ , are fixed at y = – a meter and y = + a meter respectively,,
(1989 - 6 Marks) as shown in the figure.
MOVING CHARGES & MAGNETISM P-369

Y 15. Three infinitely long thin wires, each carrying current i in


the same direction, are in the x-y plane of a gravity free
B
X space. The central wire is along the y-axis while the other
I C two are along x = ± d.
O
(i) Find the locus of the points for which the magnetic
A field B is zero. (1997 - 5 Marks)
Z (ii) If the central wire is displaced along the Z-direction by
If the wires A and B each carry a current I amp into the plane a small amount and released, show that it will execute
of the paper, obtain the expression for the force acting on simple harmonic motion. If the linear density of the
the segment OC. What will be the force on OC if the current wires is l, find the frequency of oscillation.
in the wire B is reversed ? (1992 - 10 Marks) 16. A particle of mass m and charge q is moving in a region
12. An electron gun G emits electrons of energy 2keV travelling where uniform, constant electric and magnetic fields E and
in the positive x-direction. The electrons are required to hit B are present. E and B are parallel to each other. At time t =
the spot S where GS = 0.1m, and the line GS makes an angle 0, the velocity v0 of the particle is perpendicular to E.
of 60° with the x-axis as shown in the fig. A uniform magnetic (Assume that its speed is always <<c, the speed of light in
® r vacuum.) Find the velocity v of the particle at time t. You
field B parallel to GS exists. Find B parallel to GS exists in must express your answer in terms of t, q, m, the vectors v0,
the region outside the electron gun. Find the minimum value E and B and their magnitudes v0, E and B. (1998 - 8 Marks)
of B needed to make the electrons hit S. (1993-7 Marks) 17. A uniform, constant magnetic field B is directed at an angle
of 45° to the x axis in the xy-plane. PQRS is a rigid, square
S wire frame carrying a steady current I0, with its centre at the
origin O. At time t = 0, the frame is at rest in the position as
B shown in Figure, with its sides parallel to the x and y axes.
Each side of the frame is of mass M and length L.
60° x (a) What is the torque t about O acting on the frame due
v G to the magnetic field?
(b) Find the angle by which the frame rotates under the
13. A long horizontal wire AB, which is free to move in a vertical action of this torque in a short interval of time Dt, and
plane and carries a steady current of 20A, is in equilibrium at the axis about this rotation occurs. (Dt is so short that
a height of 0.01 m over another parallel long wire CD which any variation in the torque during this interval may be
is fixed in a horizontal plane and carries a steady current of neglected.) Given : the moment of inertia of the frame
30A, as shown in figure . Show that when AB is slightly about an axis through its centre perpendicular to its
depressed, it executes simple harmonic motion. Find the
4
period of oscillations. (1994 - 6 Marks) plane is ML2 . (1998 - 8 Marks)
B 3
A

C D y
14. An electron in the ground state of hydrogen atom is
S l0 R
revolving in anticlock-wise direction in a circular orbit of
radius R. (1996 - 5 Marks)

x
O

P Q

18. The region between x = 0 and x = L is filled with uniform,


(i) Obtain an expression for the orbital magnetic dipole
moment of the electron. steady magnetic field B kˆ . A particle of mass m, positive
0
ur
(ii) The atom is placed in a uniform magnetic induction B charge q and velocity n0iˆ travels along x-axis and enters
such that the plane-normal of the electron-orbit makes
the region of the magnetic field. Neglect gravity throughout
an angle of 30° with the magnetic induction. Find the
the question. (1999 - 10 Marks)
torque experienced by the orbiting electron.
P-370 Topic-wise Solved Papers - PHYSICS
(a) Find the value of L if the particle emerges from the (a) Find the magnetic field produced by this circuit at the
region of magnetic field with its final velocity at angle center. (2001-10 Marks)
30° to its initial velocity. (b) An infinitely long straight wire carrying a current of 10
(b) Find the final velocity of the particle and the time spent A is passing through the center of the above circuit
by it in the magnetic field, if the magnetic field now vertically with the direction of the current being into
extends up to 2.1L. the plane of the circuit. What is the force acting on the
19. A circular loop of radius R is bent along a diameter and wire at the center due to the current in the circuit?
given a shape as shown in the figure. One of the semicircles What is the force acting on the arc AC and the straight
(KNM) lies in the x-z plane and the other one (KLM) in the y- segment CD due to the current at the center?
z plane with their centres at the origin. Current I is flowing 21. A wheel of radius R having charge Q, uniformly distributed
through each of the semi circles as shown in figure. on the rim of the wheel is free to rotate about a light horizontal
rod. The rod is suspended by light inextensible strings and
a magnetic field B is applied as shown in the figure. The
L y initial tensions in the strings are T0. If the breaking tension
I M 3T0
of the strings are , find the maximum angular velocity
x 2
w0 with which the wheel can be rotated. (2003 - 4 Marks)
N z d
K I
(a) A particle of charge q is released at the origin with a
ur uur
velocity v = –v iˆ . Find the instantaneous force F
0
on the particle. Assume that space is gravity free. w0
(b) If an external uniform magnetic field Bo ĵ is applied,
uur uur B
determine the force F1 and F2 on the semicircles KLM
uur
and KNM due to the field and the net force F on the 22. A proton and an a-particle are accelerated with same
loop. (2000 - 10 Marks) potential difference and they enter in the region of constant
20. A current of 10 A flows around a closed path in a circuit magnetic field B perpendicular to the velocity of particles.
which is in the horizontal plane as shown in the figure. The Find the ratio of radius of curvature of proton to the radius
circuit consists of eight alternating arcs of radii r1 = 0.08 m of curvature of a - particle. (2004 - 2 Marks)
and r1 = 0.12 m. Each arc subtends the same angle at the 23. In a moving coil galvanometer, torque on the coil can be
center. expressed as t = ki , where i is current through the wire and
D k is constant. The rectangular coil of the galvanometer having
number of turns N, area A and moment of inertia I is placed
r2 C in magnetic field B. Find (2005 - 6 Marks)
(a) k in terms of given parameters N, I, A and B
A
(b) the torsion constant of the spring, if a current i 0
r1
produces a deflection of p/2 in the coil.
(c) the maximum angle through which the coil is deflected,
if charge Q is passed through the coil almost
instantaneously. (ignore the damping in mechanical
oscillations).
MATCH THE FOLLOWING :
MUTLIPLE CHOICE QUESTIONS WITH ONE CORRECT
Each question contains statements given in two columns, which have to be matched. The statements in Column-I are labelled A,
B, C and D, while the statements in Column-II are labelled p, q, r, s and t. Any given statement in Column-I can have correct
matching with ONE OR MORE statement(s) in Column-II. The appropriate bubbles corresponding to the answers to these
questions have to be darkened as illustrated in the following example :
If the correct matches are A-p, s and t; B-q and r; C-p and q; and D-s then the correct darkening of bubbles will look like the given.
p q r s t
A p q r s t
B p q r s t
C p q r s t
D p q r s t
MOVING CHARGES & MAGNETISM P-371
1. Match the following columns : (2006, 6M)
Column I Column II Column I
(A) Dielectric ring uniformly charged (p) Constant electrostatic field out of system
(B) Dielectric ring uniformly charged (q) Magnetic field strength
rotating with angular velocity w
(C) Constant current in ring i (r) Electric field (induced)
(D) i = i0coswt (s) Magnetic dipole moment
2. Column I gives certain situations in which a straight metallic wire of resistance R is used and Column II gives some resulting
effects. Match the statements in Column I with the statements in Column II and indicate your answer by darkening appropriate
bubbles in the 4 × 4 matrix given in the ORS. (2007)
Column I Column II
(A) A charged capacitor is connected to the ends of (p) A constant current flows through the wire
the wire
(B) The wire is moved perpendicular to its length with (q) Thermal energy is generated in the wire
a constant velocity in a uniform magnetic field
perpendicular to the plane of motion
(C) The wire is placed in a constant electric field that (r) A constant potential difference develops between the ends
has a direction along the length of the wire of the wire
(D) A battery of constant emf is connected to the (s) charges of constant magnitude appear at the ends of the
ends of the wire. wire
3. Two wires each carrying a steady current I are shown in four configurations in Column I. Some of the resulting effects are
described in Column II. Match the statements in Column I with the statements in column II and indicate your answer by
darkening appropriate bubbles in the 4 × 4 matrix given in the ORS. (2007)
Column I Column II
(A) Point P is situated midway between (p) The magnetic fields (B) at P due to the currents
the wires. in the wires are in the same direction.

P
(B) Point P is situated at the mid-point of the (q) The magnetic fields (B) at P due to the currents
line joining the centers of the circular in the wires are in opposite directions.
wires, which have same radii.

(C) Point P is situated at the mid-point of the (r) There is no magnetic field at P.
line joining the centers of the circular
wires, which have same radii.

(D) Point P is situated at the common center (s) The wires repel each other.
of the wires.

P
P-372 Topic-wise Solved Papers - PHYSICS

COMPREHENSION BASED Q UESTIONS : INTEGERVALUECORRECTTYPE:


Advanced countries are making use of powerful electromagnets 1. A steady current I goes through a wire loop PQR having
to move trains at very high speed. These trains are called maglev shape of a right angle triangle with PQ = 3x, PR = 4x and QR
trains (abbreviated from magnetic levitation). These trains float = 5x. If the magnitude of the magnetic field at P due to this
on a guideway and do not run on steel rail tracks.
m I
Instead of using a engine based on fossil fuels, they make use of loop is k æç 0 ö÷ , find the value of k. (2009)
è 48 px ø
magnetic field forces. The magnetized coils are arranged in the
guide way which repels the strong magnets placed in the train's 2. A long circular tube of length 10 m and radius 0.3 m carries
under carriage. This helps train move over the guideway , a technic a current I along its curved surface as shown. A wire-loop of
called electro-dynamic suspension. When current passes in the resistance 0.005 ohm and of radius 0.1 m is placed inside the
coils of guideway , a typical magnetic field is set up between the tube with its axis coinciding with the axis of the tube. The
undercarriage of train and guideway which pushes and pull the current varies as I = I0cos(300 t) where I0 is constant. If the
train along the guideway depending on the requirement. magnetic moment of the loop is Nm0I0sin(300 t), then ‘N’ is
The lack of friction and its aerodynamic style allows the train to (2011)
more at very high speed.
1. The levitation of the train is due to (2006 - 5M, –2)
(a) Mechanical force (b) Electrostatic attraction
(c) Electrostatic repulsion (d) Magnetic repulsion
2. The disadvantage of maglev trains is that (2006 - 5M, –2)
(a) More friction (b) Less pollution
(c) Less wear & tear (d) High initial cost
3. The force which makes maglev move (2006 - 5M, –2) 3. A cylindrical cavity of diameter a exists inside a cylinder of
(a) Gravitational field (b) Magnetic field diameter 2a as shown in the figure. Both the cylinder and
the cavity are infinity long. A uniform current density J flows
(c) Nuclear forces (d) Air drag
along the length. If the magnitude of the magnetic field at
ASSERTION & REASON TYPE QUESTIONS : N
the point P is given by m 0 aJ , then the value of N is
12
1. Statement-1 : The sensitivity of a moving coil galvanometer
(2012)
is increased by placing a suitable magnetic material as a
core inside the coil.
and
Statement-2 : Soft iron has a high magnetic permeability
and cannot be easily magnetized or demagnetized. (2008)
(a) Statement-1 is True, Statement-2 is True; Statement-2
is a correct explanation for Statement-1
(b) Statement-1 is True, Statement-2 is True; Statement-2
is NOT a correct explanation for Statement-1
(c) Statement -1 is True, Statement-2 is False
(d) Statement -1 is False, Statement-2 is True
MOVING CHARGES & MAGNETISM P-373

1. If in a circular coil A of radius R, current I is flowing and in 7. A thin rectangular magnet suspended freely has a period of
another coil B of radius 2R a current 2I is flowing, then the oscillation equal to T. Now it is broken into two equal halves
ratio of the magnetic fields BA and BB, produced by them (each having half of the original length) and one piece is
will be [2002] made to oscillate freely in the same field. If its period of
(a) 1 (b) 2
(c) 1/2 (d) 4 T'
oscillation is T ' , the ratio is [2003]
2. If an electron and a proton having same momenta enter T
perpendicular to a magnetic field, then [2002]
1 1
(a) curved path of electron and proton will be same (a) (b) 1 (c) 2 (d)
(ignoring the sense of revolution) 2 2 2 4
(b) they will move undeflected 8. A magnetic needle lying parallel to a magnetic field requiers
(c) curved path of electron is more curved than that of the
proton W units of work to turn it through 600 . The torque needed
(d) path of proton is more curved. to maintain the needle in this position will be [2003]
3. Wires 1 and 2 carrying currents i1 and i2 respectively are
(a) 3 W (b) W (c)3
inclined at an angle θ to each other. What is the force on a W (d) 2 W
small element dl of wire 2 at a distance of r from wire 1 (as 2
shown in figure) due to the magnetic field of wire 1? 9. The magnetic lines of force inside a bar magnet [2003]
[2002] (a) are from north-pole to south-pole of the magnet
(b) do not exist
1 2
(c) depend upon the area of cross-section of the bar magnet
i1 (d) are from south-pole to north-pole of the Magnet
r i2
10. Curie temperature is the temperature above which
q dl [2003]
(a) a ferromagnetic material becomes paramagnetic
(b) a paramagnetic material becomes diamagnetic
m0 m0 (c) a ferromagnetic material becomes diamagnetic
(a) i1i2 dl tan q (b) i1i2 dl sin q
2pr 2 pr (d) a paramagnetic material becomes ferromagnetic
m0 m0 11. A current i ampere flows along an infinitely long straight
(c) i1i2 dl cos q (d) i1i2 dl sin q thin walled tube, then the magnetic induction at any point
2pr 4 pr
4. The time period of a charged particle undergoing a circular inside the tube is [2004]
motion in a uniform magnetic field is independent of its m 0 2i
(a) speed (b) mass [2002] (a) . tesla (b) zero
4p r
(c) charge (d) magnetic induction
5. A particle of mass M and charge Q moving with velocity 2i
r (c) infinite (d) tesla
v describe a circular path of radius R when subjected to a r
uniform transverse magnetic field of induction B. The work 12. A long wire carries a steady current. It is bent into a circle of
done by the field when the particle completes one full circle one turn and the magnetic field at the centre of the coil is B.
is [2003] It is then bent into a circular loop of n turns. The magnetic
æ Mv 2 ö field at the centre of the coil will be [2004]
(a) ç ÷ 2pR (b) zero (a) 2n B (b) n2 B
è R ø (c) nB (d) 2 n2 B
(c) B Q 2p R (d) B Qv2p R 13. The magnetic field due to a current carrying circular loop of
radius 3 cm at a point on the axis at a distance of 4 cm from the
6. A particle of charge - 16 ´ 10 -18 coulomb moving with centre is 54 mT. What will be its value at the centre of loop?
[2004]
velocity 10ms -1 along the x-axis enters a region where a
(a) 125 mT (b) 150 mT
magnetic field of induction B is along the y-axis, and an
(c) 250 mT (d) 75 mT
electric field of magnitude 10 4 V / m is along the negative 14. Two long conductors, separated by a distance d carry current
z-axis. If the charged particle continues moving along the I1 and I2 in the same direction. They exert a force F on each
x-axis, the magnitude of B is [2003] other. Now the current in one of them is increased to two
times and its direction is reversed. The distance is also
(a) 103Wb / m2 (b) 105 Wb / m 2
increased to 3d. The new value of the force between them is
(c) 1016 Wb / m 2 (d) 10 -3Wb / m 2 [2004]
P-374 Topic-wise Solved Papers - PHYSICS
21. Needles N1, N2 and N3 are made of a ferromagnetic, a
2F F
(a) - (b) paramagnetic and a diamagnetic substance respectively. A
3 3
magnet when brought close to them will [2006]
F (a) attract N1 and N2 strongly but repel N3
(c) –2 F (d) -
3 (b) attract N1 strongly, N2 weakly and repel N3 weakly
15. The length of a magnet is large compared to its width and (c) attract N1 strongly, but repel N2 and N3 weakly
breadth. The time period of its oscillation in a vibration (d) attract all three of them
magnetometer is 2s. The magnet is cut along its length into 22. In a region, steady and uniform electric and magnetic fields
three equal parts and these parts are then placed on each are present. These two fields are parallel to each other. A
other with their like poles together. The time period of this charged particle is released from rest in this region. The
combination will be [2004] path of the particle will be a [2006]
2 (a) helix (b) straight line
(a) 2 3s (b) s
3 (c) ellipse (d) circle
2 23. A long solenoid has 200 turns per cm and carries a current i.
(c) 2 s (d) s The magnetic field at its centre is 6.28 × 10–2 Weber/m2.
3 Another long solenoid has 100 turns per cm and it carries a
16. The materials suitable for making electromagnets should
have [2004] i
current . The value of the magnetic field at its centre is
(a) high retentivity and low coercivity 3
(b) low retentivity and low coercivity (a) 1.05 × 10–2 Weber/m2 [2006]
(c) high retentivity and high coercivity (b) 1.05 × 10–5 Weber/m2
(d) low retentivity and high coercivity (c) 1.05 × 10–3 Weber/m2
17. Two concentric coils each of radius equal to 2 p cm are placed
(d) 1.05 × 10–4 Weber/m2
at right angles to each other. 3 ampere and 4 ampere are the
currents flowing in each coil respectively. The magnetic 24. A long straight wire of radius a carries a steady current i.
The current is uniformly distributed across its cross section.
induction in Weber / m 2 at the centre of the coils will be The ratio of the magnetic field at a/2 and 2a is

(m 0 = 4p ´ 10 -7 Wb / A.m ) [2005]
(a) 1/2
(c) 4
(b) 1/4
(d) 1
[2007]

25. A current I flows along the length of an infinitely long,


(a) 10 -5 (b) 12 ´ 10 -5
straight, thin walled pipe. Then [2007]
(c) 7 ´ 10 -5 (d) 5 ´ 10 -5 (a) the magnetic field at all points inside the pipe is the
18. A charged particle of mass m and charge q travels on a same, but not zero
circular path of radius r that is perpendicular to a magnetic (b) the magnetic field is zero only on the axis of the pipe
field B. The time taken by the particle to complete one (c) the magnetic field is different at different points inside
revolution is [2005] the pipe
2 pq 2 B 2 pmq (d) the magnetic field at any point inside the pipe is zero
(a) (b)
m B 26. A charged particle with charge q enters a region of constant,
ur ur
2pm 2pqB uniform and mutually orthogonal fields E and B with a
(c) (d) r ur ur
qB m velocity v perpendicular to both E and B , and comes out
r
19. A magnetic needle is kept in a non-uniform magnetic field. It without any change in magnitude or direction of v . Then
experiences [2005] r ur ur r ur ur
(a) neither a force nor a torque (a) v = B ´ E / E 2 (b) v = E ´ B / B 2 [2007]
(b) a torque but not a force r ur ur 2 r ur ur 2
(c) v = B ´ E / B (d) v = E ´ B / E
(c) a force but not a torque
27. A charged particle moves through a magnetic field
(d) a force and a torque
perpendicular to its direction. Then [2007]
20. A uniform electric field and a uniform magnetic field are
acting along the same direction in a certain region. If an (a) kinetic energy changes but the momentum is constant
electron is projected along the direction of the fields with a (b) the momentum changes but the kinetic energy is
certain velocity then [2005] constant
(a) its velocity will increase (c) both momentum and kinetic energy of the particle are
(b) Its velocity will decrease not constant
(c) it will turn towards left of direction of motion (d) both momentum and kinetic energy of the particle are
(d) it will turn towards right of direction of motion constant
MOVING CHARGES & MAGNETISM P-375
28. Two identical conducting wires AOB and COD are placed at 32. Due to the presence of the current I1 at the origin:
right angles to each other. The wire AOB carries an electric (a) The forces on AD and BC are zero.
current I1 and COD carries a current I2. The magnetic field (b) The magnitude of the net force on the loop is given by
on a point lying at a distance d from O, in a direction
I1 I
perpendicular to the plane of the wires AOB and COD, will m o [2(b - a ) + p 3 (a + b] .
be given by [2007] 4p
1 (c) The magnitude of the net force on the loop is given by
m0 m 0 æ I1 + I 2 ö 2 m o II1
(a) ( I12 + I 2 2 ) (b) ç ÷ (b - a).
2 pd 2p è d ø 24 ab
1 (d) The forces on AB and DC are zero.
(c)
m0
2pd
(
I12 + I 22 2 (d) ) m0
2 pd
( I1 + I 2 ) 33. Two long parallel wires are at a distance 2d apart. They
carry steady equal currents flowing out of the plane of the
29. A horizontal overhead powerline is at height of 4m from the paper as shown. The variation of the magnetic field B along
ground and carries a current of 100A from east to west. the line XX' is given by [2010]
The magnetic field directly below it on the ground is
(m0 = 4p×10 –7 Tm A–1) [2008] B
(a) 2.5 × 10–7 T southward
(b) 5 × 10–6 T northward
(c) 5 × 10–6 T southward (a) X X¢
(d) 2.5 × 10–7 T northward
30. Relative permittivity and permeability of a material e r and
d d
m r , respectively. Which of the following values of these
quantities are allowed for a diamagnetic material? [2008]
B
(a) e r = 0.5, m r = 1.5 (b) e r = 1.5, mr = 0.5
(c) e r = 0.5, m r = 0.5 (d) e r = 1.5, mr = 1.5
Directions : Question numbers 31 and 32 are based on the X X¢
(b)
following paragraph.
A current loop ABCD is held fixed on the plane of the paper as
d d
shown in the figure. The arcs BC (radius = b) and DA (radius = a)
of the loop are joined by two straight wires AB and CD. A steady
current I is flowing in the loop. Angle made by AB and CD at the B
origin O is 30°. Another straight thin wire with steady current I1
flowing out of the plane of the paper is kept at the origin.
X X¢
B (c)
a A
d d
I1 30° I
O B
D
b C X X¢
(d)
[2009]
31. The magnitude of the magnetic field (B) due to the loop d d
ABCD at the origin (O) is :
m o I (b - a ) 34. A current I flows in an infinitely long wire with cross section
(a) in the form of a semi-circular ring of radius R. The magnitude
24ab
of the magnetic induction along its axis is: [2011]
mo I é b - a ù m0 I
(b) m0 I
4p êë ab úû (a)
2p2 R
(b)
2p R
mo I
(c) [2(b - a) + p (a + b)] m0 I m0 I
4p 3 (c) (d)
4p R p2R
(d) zero
P-376 Topic-wise Solved Papers - PHYSICS
r
35. An electric charge +q moves with velocity v = 3iˆ + 4 ˆj + kˆ
ur
in an electromagnetic field given by E = 3i$ + $j + 2k$ and
ur B B
B = iˆ + ˆj - 3kˆ The y - component of the force experienced
(a) (b)
by + q is : [2011RS]
(a) 11 q (b) 5 q R R
(c) 3 q (d) 2 q
36. A thin circular disc of radius R is uniformly charged with
density s > 0 per unit area. The disc rotates about its axis
with a uniform angular speed w.The magnetic moment of
the disc is [2011RS] B B
(c) (d)
pR 4
(a) pR 4 sw (b) sw
2 R R
4
pR 38. Proton, deuteron and alpha particle of same kinetic energy
(c) sw (d) 2pR 4 sw
4 are moving in circular trajectories in a constant magnetic
37. A charge Q is uniformly distributed over the surface of non- field. The radii of proton, deuteron and alpha particle are
conducting disc of radius R. The disc rotates about an axis respectively rp, rd and ra. Which one of the following relation
perpendicular to its plane and passing through its centre is correct? [2012]
with an angular velocity w. As a result of this rotation a (a) ra = rp = rd (b) ra = rp < rd
magnetic field of induction B is obtained at the centre of the
disc. If we keep both the amount of charge placed on the (c) ra > rd > rp (d) ra =rd > rp
disc and its angular velocity to be constant and vary the 39. Two short bar magnets of length 1 cm each have magnetic
radius of the disc then the variation of the magnetic induction moments 1.20 Am2 and 1.00 Am2 respectively. They are
at the centre of the disc will be represented by the figure : placed on a horizontal table parallel to each other with their
[2012] N poles pointing towards the South. They have a common
magnetic equator and are separated by a distance of 20.0
cm. The value of the resultand horizontal magnetic induction
at the mid-point O of the line joining their centres is close to
(Horizontal component of earth.s magnetic induction is 3.6×
10.5Wb/m2) [JEE Main 2013]
(a) 3.6 × 10.5 Wb/m2 (b) 2.56 × 10.4 Wb/m2
(c) 3.50 × 10.4 Wb/m2 (d) 5.80 × 10.4 Wb/m2
MOVING CHARGES & MAGNETISM P-377

Solutions & Explanations


Section-A : JEE Advanced/ IIT-JEE

iL2 m0I é 1 1 ù
A 1. D, B 2. 3. 1.25 × 10–23 Am2 4. ê - ú
4p 4 ë R1 R 2 û

5. IlB; +Z direction 6. evB; ABCD 7. 14.0185 MeV


B 1. T 2. T 3. F 4. F
C 1. (c) 2. (d) 3. (a) 4. (d) 5. (a) 6. (c) 7. (b)
8. (c) 9. (c) 10. (d) 11. (b) 12. (c) 13. (b) 14. (a)
15. (d) 16. (b) 17. (b) 18. (a) 19. (b) 20. (a) 21. (c)
22. (a) 23. (b) 24. (d)
D 1. (a) 2. (a,b,d) 3. (c) 4. (b) 5. (c) 6. (a,b,d) 7. (b, c)
8. (b) 9. (a, c) 10. (a) 11. (d) 12. (c, d) 13. (a,c) 14. (a, c, d)
15. ( b, d) 16. (b, d) 17. (c, d) 18. (a, c) 19. (a, d)
E 1. 25.98 N 3. 0.1T, Directed perpendicular to the plane of paper inwards.
4. (6.4 m, 0, 0); (6.4 m, 0, 2 m) 5. (i) 0.1414 m, 45° (ii) 4.71 × 10–8 sec.
6. 0.012 m, 0.044 m 7. (i) 3A, upward direction (ii) 1.3 × 10–6 T (iii) 28.8 × 10–7 N
8. –4
10 Tesla; directed towards the reader perpendicular to the plane of paper.
9. (i) 4 Amp. (ii) r = 1m where r is the distance from R
0.11m 0 IQv
10. (a) directed 30° with the negative X-axis (b) zero, 0.614 BIa2 ĵ
ma

m é a 2 + L2 ù
11. F = 0 2 I 2 êloge ú directed toward –Z direction, zero.
4p êë a 2 úû
12. 4.737 × 10–3 T
he heB uur
13. 0.2 sec. 14. (i) M = (ii) directed perpendicular to the plane containing n̂ and B .
4 pm 8pm

uur uur uur


d i m0 æ qBt ö uur q æ v0 ´ B ö æ qB ö
15. (i) ± (ii) n = 16. v0 cos ç ÷+ E mt + çç ÷÷ sin ç t ÷
3 2 pd pl è m ø è B ø è m ø

ˆ pm
( )
2 3 I0 B 2 mv0
17. (a) I 0 L B ˆj - î (b) Dt 18. (a) L = (b) -v0i,
4 M 2qB qB0
2

æ -m qv I ö uur uur
19. (a) ç 0 0 ÷ kˆ (b) F1 = 2 BIRiˆ, F2 = 2 BIRiˆ, 4 BIRiˆ
è 4R ø
20. (a) 6.54 × 10–5T (b) 0, Force on arc AC = 0, 8.1 × 10–6N

DT0 1 2Ni0 AB NABp


21. w = 2 22. 23. (a) k = NAB (b) (c) Q
BQr 2 π 2 Ii0
F 1. A-p; B-q, s; C-q, s; D-q, r, s 2. A-q; B-r, s; C-s; D-p, q, r 3. A-q, r; B-p; C-q, r; D-q, s
G 1. (d) 2. (d) 3. (b)
H 1. (c)
I 1. Q=7 2. 6 3. 5
P-378 Topic-wise Solved Papers - PHYSICS

Section-B : JEE Main/ AIEEE


1. (a) 2. (a) 3. (c) 4. (a) 5. (b) 6. (a) 7. (b)
8. (a) 9. (d) 10. (a) 11. (b) 12. (b) 13. (c) 14. (a)
15. (b) 16. (b) 17. (d) 18. (c) 19. (d) 20. (b) 21. (b)
22. (b) 23. (a) 24. (d) 25. (d) 26. (b) 27. (b) 28. (c)
29. (c) 30. (b) 31. (a) 32. (a) 33. (a) 34. (d) 35. (a)
36. (c) 37. (a) 38. (b) 39. (b)

FILL IN THE BLANKS : e


1. According to Fleming's left hand rule, the force on electrons
will be towards right (D).
Also, by the same rule we find that the force on proton and +
a-particle is towards left. Now since the magnetic force will
behave as centripetal force, therefore
mv 2
\ = qvB
r 4. The magnetic field at C due to current in PQ and RS is zero.
mv m Magnetic field due to current in semi-circular arc QAR
\ =r or rµ
qB q 1 é µ0 I ù
= ê ú
1 4 2 ë 2 R1 û
For proton r µ = 1 ; For a-particle r µ = 2
1 2 directed towards reader perpendicular to the plane of paper.
\ radius will be more for a-particle
\ a-particle will take path B. I B
2. L
I A

r S R Q P
Magnetic field due to current in semi-circular arc
1 é µ0 I ù
SBP = 2 ê 2 R ú
Wire of length L is bent in the form of a circle. Then the ë 2û
perimeter of the circle directed away from reader perpendicular to the plane of
L paper.
2pr = L Þ r =
2p 1 é m0 I ù 1 é m0 I ù
\ Net Magnetic field = ê ú- ê ú
L2 pL2 2 ë 2 R1 û 2 ë 2 R2 û
\ Area of the circle = pr2 = =
4p 2 4p (directed towards the reader perpendicular to plane of paper).
Magnetic moment of a loop in which current i flows is given m0 I é 1 1 ù
by = ê - ú.
4 ë R1 R2 û
iL2 5. KEY CONCEPT : Use the formula
M = iA = .
4p ur r ur
F = I ( L ´ B) Þ F = I L B sin q
q ne 1016 For FE and BA sides, the angle will be 180° and 0° respectively
3. i= = = × 1.6 × 10–19 = 1.6 × 10–3 A.
t t 1 \ F=0
M = i × A = i × pr2 For sides ED and CB, the forces will be ILB each but in +Y
= 1.6 × 10–3 × 3.14 × 0.5 × 10–10 × 0.5 × 10–10 and – Y directions respectively. The net force due to these
= 1.25 × 10–23 Am2 two sides will be zero.
MOVING CHARGES & MAGNETISM P-379

+Z D F RQ
R
I
B
Q

C B
E F b

+Y B
I S
B A B
+X P F PS

l
The force due to current inside DC will be ILB sin 90° = ILB
in + Z direction, according to Flemings Left hand rule.
ALTERNATE SOLUTION : We may assume current to be Force on section RQ will be in upward direction and equal
flowing in segment EB in both directions. to I l b sin q. These two forces will cancel out. Similarly the
Net force on the loop EDCBE will be zero. Also force due to
segment FE and BA will be zero. Force due to segment EB forces on lengths RS and QP will be equal and opposite and
ur cancel out in pairs.
F = I [ Li$ ´ B $j ] = ILBk$ The net force is, therefore, zero.
ur r ur ALTERNATE SOLUTION : A current carrying coil is a
6. F = q(v ´ B) Þ qvB sin q = F magnetic dipole. The net force on a magnetic dipole placed
Þ F = evB sin 90° = evB in uniform magnetic field is zero.
The direction of this force is in + Z direction (by Fleming's 2. NOTE : The magnetic force acts in a direction perpendicular
left hand rule). Since electrons move towards the side ABDC, to the direction of velocity and hence it cannot change the
therefore, this side has low potential. speed of the charged particle.
ALTERNATE SOLUTION
ur r ur æ 1 ö
F = q(v ´ B) = ( - e) ( - vi$ ´ B $j ) = evB k$ Therefore, the kinetic energy ç = mv2 ÷ does not change.
è 2 ø
NOTE : The direction of flow of electrons is opposite to
that of current. 3. The velocity component v2 will be responsible in moving
7. The positively charged particle enters the uniform magnetic the charged particle in a circle.
field at right angles. Therefore, the force acting on the
charged particle in the magnetic field acts as centripetal B
force v
v2 = vsinq q
mv 2
)

= qvB v1= v cos q


r
Þ mv = qrB Þ m2v2 = q2 r2 B2 Þ p2 = q2 r2 B2

p2 The velocity component v1 will be responsible in moving


But K.E. = Þ K.E. × 2 m = p2 the charged particle in horizontal direction. Therefore the
2m
charged particle will travel in a helical path.
(K.E.)a 2ma qa2 r 2 B 2 4. When a charged particle passes through a uniform magnetic
Þ = field perpendicular to the direction of motion, a force acts
(K.E.)d 2md qd2 r 2 (2.3B )2
on the particle perpendicular to the velocity. This force acts
5.3 4 4 1 as a centripetal force
Þ ´ = ´
(K.E.) d 2 1 2.3 ´ 2.3 mv 2
\ = qvB
Þ (K.E.)d = 14.0185 M eV r
TRUE / FALSE :
mv 2 mv p
\ r= = = . ... (i)
1. Let us consider a rectangular loop PQRS (l ´ b) having qvB qB qB
current I in anticlockwise direction placed in a uniform
ur p2
magnetic field B . The uniform magnetic field is directed But K.E. (K) =
2m
from left to right.
Force on section PS according to Fleming's Left hand rule \ p= 2m K ... (ii)
will be in downward direction and equal to I l b sin q.
P-380 Topic-wise Solved Papers - PHYSICS
From (i) and (ii) Therefore net magnetic field at the centre
2 mK m 0 I1 q m0 I 2
r= B= - (2p - q)
qB 4 p r p 4p r
E E EA
m Also, I1 = = =
\ rµ [for const. K.E. and B] R1 rl1 / A rr q
q
Here, q is same for electron and proton E E EA
and I2 = = =
\ rµ m R2 rl 2 / A rr (2p - q)
Radius of proton will be more. m0 é EA q EA (2p - q) ù
\ B= ê ´ - ´ ú=0
MCQ's WITH ONE CORRECT ANSWER : 4p ë r r q r rr (2p - q) r û
1. (c) The magnetic field is perpendicular to the plane of the m
paper. Let us consider two diametrically opposite 3. (a) KEY CONCEPT : r µ
q
elements. By Fleming's left hand rule, on element AB
the direction of force will be leftwards and the 1 2 4
magnitude will be \ rp : rd : ra = : : = 1: 2 :1
1 1 1
dF = I ( d l ) B sin 90° = I ( d l ) B Þ ra = rp < rd
4. (d) The magnetic lines of force created due to current will
X X X X X X X
be in such a way that on x – y plane these lines will be
perpendicular. Further, these lines will be in circular
B CX
X X X X
dl X X loops. The number of lines moving downwards in x – y
dF dl dF
A D plane will be same in number to that coming upwards
X X X X X X X of the x – y plane. Therefore, the net flux will be zero.
I One such magnetic line is shown in the figure.
X X X X X X X
y
On element CD, the direction of force will be towards
right on the plane of the paper and the magnitude Magnetic line of
will be force
dF = I ( d l ) B.
These two forces will cancel out. x
NOTE : Similarly, all forces acting on the diametrically
opposite elements will cancel out in pair. The net force Current loop
acting on the loop will be zero.

D 5. (a) FE = qE (Force due to electric field)


2. (d)
FB = evB sin q = qvB sin 0 = 0
I2 (Force due to magnetic field)
2p-q E

A q B
+q
B
C Force due to electric field will make the charged particle
I1 released from rest to move in the straight line (that of
I electric field). Since the force due to magnetic field is
E zero, therefore, the charged particle will move in a
straight line.
Magnetic field at the centre due to current in arc 6. (c) The angular momentum L of the particle is given by
ABC is L = mr2w where w = 2pn.
m 0 I1 w wq
B1 = q (Directed upwards) \ Frequency n = ; Further i = q × n =
4p r 2p 2p
Magnetic field at the centre due to current in arc wq
ADB is Magnetic moment, M = iA = ´ pr 2 ;
2p
µ0 I 2
B2 = ( 2p - q) (Directed downwards) wqr 2 M wqr 2 q
4p r \M= So, = =
2 L 2mr 2 w 2m
MOVING CHARGES & MAGNETISM P-381
7. (b) The wires at A and B are perpendicular to the plane of Case of negatively charged particle.
paper and current is towards the reader. Let us consider Two forces are acting on the negatively charged particle
certain points. (a) due to electric field in the negative X-direction.
Point C (mid point between A and B) : The magnetic (b) due to magnetic field
ur ur r ur
field at C due to A ( B CA ) is in upward direction but F = - q(v ´ B)
magnetic field at C due to B is in downward direction. ur
Net field is zero. F = - q[v (-$i ) ´ B (k$ )]
Point E : Magnetic field due to A is upward and ur ur
magnetic field due to B is downward but F = - qvB [i$ ´ k$ ] , F = qvB (- $j )
ur ur
| B EA | < | B EB | . Same direction as that of positive charge.
\ Net magnetic field is in downward direction. (c) is the correct answer.
ur ur 10. (d) NOTE : If we take individual length for the purpose of
Point D : | B DA | > | B DB | . Net field upwards. Similarly,, calculating the magnetic field in a 3-Dimensional figure
other points can be considered. then it will be difficult.
Here a smart choice is divide the loop into two loops.
BDA BGB One loop is ADEFA in y-z plane and the other loop will
BCA
be ABCDA in the x – y plane.
BEA
BGA
FB A D C E B G Z
FB E Y
BDB
BFA BCB
I
I
BEB D C
F
8. (c) Case 1 : Magnetic field at M due to PQ and QR is X
I
1 é m0 I ù m I
H1 = +0= 0
2 êë 2 pR úû 4pR A B

We actually do not have any current in the segment


M
AD. By choosing the loops we find that in one loop we
R
have to take current from A to D and in the other one
from D to A. Hence these two cancel out the effect of
to µ to µ each other as far as creating magnetic field at the
P I Q S concerned point P is considered.
The point (a, 0, a) is in the X-Z plane.
The magnetic field due to current in ABCDA will be in
to µ
R + ve Z-direction.
NOTE : Due to symmetry the y-components and x-
Case 2 : When wire QS is joined.
H2 = (Magnetic field at M due to PQ) + (magnetic field components will cancel out each other.
at M due to QR) + (Magnetic field at M due to QS) Similarly the magnetic field due to current in ADEFA
will be in x-direction.
1 é m0 I ù 1 é m I / 2 ù 3m 0 I H 2 \ The resultant magnetic field will be
= ê ú +0+ ê 0 = \ 1 =
2 ë 2pR û 2 ë 2pR ûú 8pR H2 3 ur 1
NOTE : The magnetic field due to an infinitely long B= ($i + k$ ).
wire carrying current at a distance R from the end point 2
is half that at a distance R from the middle point. 11. (b) KEY CONCEPT : When a charged particle is moving
9. (c) Case of positively charged particle : at right angles to the magnetic field then a force acts
Two forces are acting on the positively charged particle on it which behaves as a centripetal force and moves
(a) due to electric field in the positive x-direction. the particle in circular motion.
(b) Force due to magnetic field.
ur r ur m Av 2A m Av A
F = q(v ´ B) \ = q.v A B \ = qB
rA rA
ur ur
Þ F = q(vi$ ´ Bk$ ) Þ F = qvB ( - $j )
mB vB
This forces will move the positively charged particle = qB
towards Y-axis. rB
P-382 Topic-wise Solved Papers - PHYSICS
14. (a) The wire carries a current I in the negative z-direction.
B We have to consider the magnetic vector field
ur
B at (x, y) in the z = 0 plane.
ur
Magnetic field B is perpendicular to OP.
ur
\ B = B sin qiˆ - B cos qˆj
A
y x m I
B sin q = , cos q = B= 0
r r 2 pr
y
ur m0 I
Þ
m A v A m B vB
= \ B=
2pr 2
(
yiˆ - xjˆ ) P(x, y)
rA rB r q ®
B
Since rA > rB
Þ mAvA > mBvB or B=
(
ur m 0 I yiˆ - xjˆ )
. O×
q
x
y
x
12. (c) Let us consider a thickness dx of wire. Let it be at a
2
(
2p x + y 2
)
distance x from the centre O.
15. (d) NOTE : Magnetic lines of force form closed loops.
Inside a magnet, these are directed from south to north
pole.
dx
16. (b) The velocity at P is in the X-direction (given).
r
x Let v = ki$.
a After P, the positively charged particle gets deflected
O in the x – y plane toward – y direction and the path is
b non-circular.
ur r ur
Now, F = q(v ´ B)
ur
Þ F = q [ki$ ´ (ckˆ + ai$)] for option (b)
ˆ $ ´ $i ]
= q [kci$ ´ kˆ + kai
N
Number of turns per unit length = = kcq ( - $j )
b-a ur
N Since in option (b), electric field is also present E = ai$,
\ Number of turns in thickness dx = dx therefore it will also exert a force in the + X direction.
b-a
The net result of the two forces will be a non-circular
Small amount of magnetic field is produced at O due to path.
thickness dx of the wire. Only option (b) fits for the above logic. For other option,
m 0 NI dx we get some other results.
\ dB = 17. (b) KEY CONCEPT : Use Fleming's left hand rule. We find
2 (b - a) x
that a force is acting in the radially outward direction
On integrating, we get, throughout the circumference of the conducting loop.
b m 0 NI dx m NI uur ur
B = òa = 0 18. (a) U = - M . B = – MB cos q
2 b-a x 2 (b - a )
In case I, q = 180°, U = + MB
b dx m0 NI In case II, q = 90°, U = 0
òa x
=
2 (b - a )
[loge x] ba
In case III, q = acute, U = + ve (less than + MB)
In case IV, q = obtuse, U = – ve
m 0 NI b \ I > III > II > IV.
B= log
2 (b - a) e a 19. (b) The force acting on electron will be perpendicular to
13. (b) the direction of velocity till the electron remains in the
magnetic field. So the electron will follow the path as
q X given.
x=a v x=b
y X B

Width of the magnetic field region (b – a) < R; where 'R' e


u x
is its radius of curvature inside magnetic field,
mv (b - a ) qB
\ R= ³ (b – a) Þ vmin =
qB m
MOVING CHARGES & MAGNETISM P-383
20. (a) For a < x < 2a;
ur æ dq ö L
At equilibrium 2T sin ç ÷ = BI dq
B = B0 ˆj è 2 ø 2p
r
The initial velocity is v = v0 $i dq L é dq ù
Þ 2T
2
= BI
2p
dq êëQ 2 = small úû
Z
Y'
BIL
F B0 ÞT =
2p
22. (a) Let us consider a thickness dx of wire. Let it be at a
X distance x from the centre O.
x=a V0
From the diagram it is clear that the force on the particle
is towards positive Z-axis (apply Fleming's left hand
rule) at x = a, which shifts the particle as shown in the dx
X-Z plane.
For 2a < x < 3a x
ur a
B = - B $j
0 O
The direction of velocity is shown at x = 2a. Again b
using Fleming's left hand rule, we get the direction of
force. This changes the profile of the particle as shown
by the dotted line.
ALTERNATE SOLUTION :
Use the vector form of B and v in the formulae
ur r ur
F = q(v ´ B) to get the instantaneous direction of N
Number of turns per unit length =
force at x = a and x = 2a. b-a
21. (c) Let us consider an elemental length dl subtending an
angle dq at the centre of the circle. Let FB be the N
\ Number of turns in thickness dx = dx
magnetic force acting on this length. Then b-a
Small amount of magnetic field is produced at O due to
T cos dq T cos dq thickness dx of the wire.
2 2
dq dq
m 0 NI dx
2 2 \ dB =
T Tsin dq T 2 (b - a) x
2 Tsin dq
2 On integrating, we get,
I R I
dq dq b m 0 NI dx m NI b dx
2 2 B = òa = 0 òa
2 b-a x 2 (b - a ) x

m0 NI
= [loge x] ba
2 (b - a )
FB = BI (dl) directed upwards as shown m 0 NI b
B= loge
é arc(dl ) ù 2 (b - a) a
= BI (Rdq) êQ angle (dq) = ú
ë radius R û 23. (b) The magnetic moment of a current carrying loop is
r r
æ Lö é Lù given by M = NIA
= BI ç ÷ d q êQ 2pR = L Þ R = 2p ú
è 2p ø ë û 2
æ aö é pù
Let T be the tension in the wire acting along both ends Here N = 1, A = a2 + 2p ç ÷ = a2 ê1 + ú , the direction
of the elemental length as shown. On resolving T, we è2ø ë 2û
is towards positive z-axis.
æ dq ö
find that the components. T cos ç ÷ cancel out and
è 2 ø r é pù
\ M = Ia 2 ê1 + ú kˆ
ë 2û
æ dq ö
the components. T sin ç ÷ add up to balance FB.
è 2 ø
P-384 Topic-wise Solved Papers - PHYSICS
3. (c) AB part of the rectangular loop will get attracted to the
R
24. (d) For r < , B=0 long straight wire as the currents are parallel and in the
2 same direction whereas CD part will be repelled. But
1
since this force F µ where r is the distance between
r
the wires. Therefore, there will be a net attractive force
on the rectangular loop. Force on BC is equal and
R opposite to that on AD.

R
r P B I C
2
I

A D
||
B 4. (b) The magnetic field due to current
1 2
in wire 1 in the region of wire 2
will be
m 0 2i i i
B1 =
4p b
Since wire 2 having current i is
R r placed in a magnetic field B1, it
R
2 will experience a force given by
F = i (l B1 sin 90°)
b

R µ0 é R2 ù
For £ r < R, B= êr - úJ F m 2i m i 2
2 2 2r ûú \ force per unit length =i´ 0 ´ = 0
ëê l 4p b 2p b

µ0 i é m 0 2i ù
For r > R, B= êQ B = 4 p ´ b ú
2 pr ë û
1
5. (c) K.E. of first particle = m1v12 = qV ... (i)
2
ur
1. (a) The force on north pole = mB1 1
K.E. of second particle = m2 v22 = qV ... (ii)
ur 2
The force on south pole = mB 2 NOTE : After entering the magnetic field, a magnetic force
acts on the charged particle which moves the charged particle
in circular path of radius
B
2m K
R=
qB
N Here, K, q, B are equal
\ R2 µ m
S
m1 R12
Þ = From (i) and (ii)
Since the forces will be unequal and are not having same m2 R22
line of action therefore, the magnetic needle experiences a 6. (a, b, d)
force as well as a torque. Considering the activity from P to Q (Horizontal)
2. (a,b,d)
There is no change in velocity. It can be possible when v E
P
• Electric and magnetic fields are absent, i.e., E = 0, B = 0
B
• Or when electric and magnetic fields are present but
a
force due to electric field is equal and opposite to the
magnetic force, (i.e., E ¹ 0, B ¹ 0).
• Or when E = 0. B ¹ 0 provided
F = qvB sin q = 0
2a Q
sin q = 0, i.e., q = 0 Þ v and B are in the same direction. 2v
MOVING CHARGES & MAGNETISM P-385
u1 = v, v1 = 2v, s1 = 2a, Acc = A Magnetic moment,
Þ 4v2 – v2 = 2A(2a)
æ qw ö 2 2
2 M = (i) (A) = çè ÷ø (pR ) = (qwR )
3v p
Þ A=
4a Angular momentum, L = 2 Iw = 2(mR2) w
Force acting in the horizontal direction is
M qw R 2 q
F = qE = mA \ = =
2
L 2( mR ) w 2m
mA 3 é mv 2 ù 11. (d) Net magnetic field due to the wires will be downward
Þ E= = ê ú r
q 4 êë qa úû as
ur shown below in the figure. Since angle between v and
Rate of doing work at P B is 180°,

3 é mv 3 ù v Z
Power = F × v = mA × v = ê ú
4 ëê a ûú Y

Rate of doing work by the magnetic field is throughout zero.


The rate of doing work by electric field is zero at Q. Because I -I
at Q, the angle between force due to electric field and
displacement is zero. X
B Wires are in X-Y plane
7. (b, c) and velocity in Z-direction
d
For V = Ig (G + R) = 5 × 10–5 [100 + 200,000] = 10V
ur r ur
æG ö é100 ù Therefore, magnetic force F m = q(v ´ B) = 0
For I = Ig ç + 1÷ = 5 ´ 10-5 ê + 1ú = 5mA.
èS ø ë 1 û 12. (c, d)
8. (b) Let us consider any point P Out of the given options only induced electric field and
inside the thin walled pipe. Let magnetostatic field form closed loops of field lines.
us consider a circular loop and P 13. (a, c) Net force on the loop :
apply Ampere's circuital law, Force on AB : The magnetic field due to current I1 is
along AB.
ur uur
Ñò B.d l =µ0I dF = I (d l ´ B ´ sin 0°) = 0
Since current inside the loop is zero.
ur C
\ B =0
B
9. (a, c) KEY CONCEPT : When the charged particles enter a I2
magnetic field then a force acts on the particle which will act
as a centripetal force. We know that when kinetic energy
O
and magnetic field are equal then
O'
m I1

q
A
1 4 16
r µ ;r +µ ; r ++ µ D
H+ 1 He 1 O 2
Þ r µ 1; r µ 2; r µ2 Force on CD : Similarly the magnetic field due to current I1
H+ He + O+ + is along DC. Because q = 180° here, therefore force on DC is
He+ and O++ will be deflected equally. zero.
H+ will be deflected the most since its radius is smallest.
B1 I2
10. (a) w C

dl
B
R R
Force on BC : Consider a small element d l.
(q,m) (q,m)
dF = I 2 d l B1sin 90° Þ dF = I 2 d l B1
æ wö qw By Fleming's left hand rule, the direction of this force is
Current, i = (frequency) (charge) = çè ÷ø (2q) = perpendicular to the plane of the paper directed outwards.
2p p
P-386 Topic-wise Solved Papers - PHYSICS
Force on AD : dF = I 2 d l B1sin 90° = I 2 d l B1 15. (b, d) When current I is switched on in both the solenoids
in identical manner, eddy currents are setup in metallic
B1 rings A and B in such a way that rings A and B are
D repelled.

dl

A I2
hA
By Fleming's left hand rule, the direction of this force is rA r B hB
perpendicular to the plane of paper directed inwards. Since A B
the current elements are located symmetrical to current I1,
therefore force on BC will cancel out the effect of force on
AD.
Þ Net force on loop ABCD is zero.
Net Torque on the loop : The force on BC and AD will create
a torque on ABCD in clockwise direction about OO' as seen
by the observer at O.
14. (a, c, d)
As the particle enters the magnetic field, a force acts on it Given hA > hB . This shows that eddy currents produced in
due to the magnetic field which moves the particle in a circular
path of radius A are greater than in B. This is possible when r A < rB (the
rate of change of flux is same in both the rings, therefore
mv induced emf is same).
r=
qB 16. (b,d)
r
Figure shows that the megnetic field B is present on the
Region I right hand side of AB. The electron (e) and proton (p) moving
Region II Region III
on straight parallel paths with the same velocity enter the
×××× region of uniform magnetic field.
The entry and exit of electron & proton in the magnetic field
×××× makes the same angle with AB as shown.

× r× × ×
Therefore both will come out travelling in parallel paths.
A
× ×r × × q × × × × ×
××××
×××× 2q × × × ×
B
×
××××
× v× × q
× × × × ×
× ×l × × p
e– q
×2q × × × ×
• For the particle to enter region III, r > l (path shown by q
dotted line)
mv ql B × × × × ×
Þ >l Þv>
qB m
• For maximum path length in region II, r = l B
The time taken by proton
mv qlB
\ l= Þv = distance arc angle × radius 2q ´ R p
qB m tp = = = =
• The time taken by the particle to move in region II speed speed speed v
before coming back in region I is given by
2q æ m p v ö 2q m p
pm = v ´ çç eB ÷÷ = eB
t= which is independent of v.. è ø
qB
MOVING CHARGES & MAGNETISM P-387
The time taken by electron is
p 4 ´ 10 ´ 10-3 é Mv ù
(2p - 2q) R e (2p - 2q) æ me v ö (2p - 2q) me \ = êQ radius = QB ú
te = = çè ÷= 6 M ´ 4 / QB ë û
v v eB ø eB
clearly te is not equal to tp as mp >> me 50p M
\B =
\ (b) , (d) are correct options 3Q
17. (c, d)When q = 0º, the charged particle is projected along x- (a) and (c) are the correct options
ur 19. (a, d)In the region O < r < R, the magnetic field is present due
axis, due to B the charged particle will tend to
move in a circular path in y-z plane but due to to current in solenoid.
force of electric field, the particle will move in a
helical path with increasing pitch. Therefore
options (A) and (B) are incorrect.
y

E B
In the region r > 2 R, the magnetic field is present due
to the current in the cylinder.
For the region R < r < 2R, the magnetic field is neither
along the common axis, nor tangential to the circle of
x radius r. (a) and (d) are correct options.
SUBJECTIVE PROBLEMS :

When q = 10º, we can resolve velocity into two 1. 2l = 0.25 m


rectangular components. One along x-axis Also, m × 2l = 14.4
(v cos 10º) and one along y-axis (v sin 10º). Due to v cos 14.4
10º, the particle will move in circular path and due to v Þ m= = 57.6 A-m2
0.25
sin 10º plus the force due to electric field, the particle
Torque due to magnetic field
will undergo helical motion with its pitch increasing.
= pm × B × sin 60°
If q = 90º, the charge is moving along the magnetic
field. Therefore the force due to magnetic field is zero. 3
But the force due to electric field will accelerate the = 14.4 × 0.25 ×
2
particle along y-axis.
The torque due to the force
18. (a, c)
= F × 0.12
The magnetic field should be in the –z direction
(Fleming’s left hand rule) 3
For equilibrium F × 0.12 = 14.4 × 0.25 ×
vy 2 2
tan q = = Þ F = 25.98 N
vx 2 3
If the force F is removed, the torque due to magnetic field
p will move the bar magnet. It will start oscillating about the
\q = ur
6 mean position where the angle between p m and
ur
y B is 0.
O z x u2 = v 2.
q
× × × × × ×
vy = 2
× × × × q × × N
× × × × vx = ×
2 3 × H H
× × × × × ×
W E
Q,M×+ × ×
ui = u = 4iˆ
× × ×
× × × × × × F F S
× × × × × ×

arc speed ´ time


angle = =
radius radius
P-388 Topic-wise Solved Papers - PHYSICS
The time period of revolution is
3. B 2pr 2 p ´1
x x x - T= = = 4.9 × 10–6 sec ... (i)
+ x v 1.28 ´ 106
x x x x We have to find the position of the particle after time
E (7.45 × 10–6 s – 5.0 × 10–6 s) = 2.45 × 10–6 sec ... (ii)
x x x x On comparing (i) and (ii), we find that the particle makes half
the revolution.
x x x x Therefore, its co-ordinates after time 7.45 × 10–6 sec will be
v
(6.4 m, 0, 2 m)
+ x x x x - 5. m = 1.6 × 10–27 kg, q = 1.6 × 10–19 C
B=1T
e v = 107 m/s
The force on electron will be towards the left plate due to F = q . v B sin a
electric field and will be equal to (acting towards O by Fleming's left hand rule)
Fe = eE Þ F = qvB [Q a = 90°]
NOTE : For the electron to move undeflected between the But F = ma
plates there should be a force (magnetic) which is equal to qvB
the electric force and opposite in direction. The force should \ qvB = ma \ a =
m
be directed towards the right as the electric force is towards
the left. 1.6 ´ 10 -19 ´ 107 ´ 1
On applying Fleming's left hand rule we find the magnetic = = 1015 m/s2
field should be directed perpendicular to the plane of paper 1.6 ´ 10-27
inwards. Therefore,
q ´ ´
Force due to electric field = Force due to magnetic field.
eE = evB F ´ ´

E V /d é Vù ´ ´
\ B= = êQE= ú
v v ë dû ´ ´
O
where V = p.d. between plates V ´ ´
V2= ¾
d = distance between plates Ö2
´ o
45
´
-3 V
600 / 3 ´ 10 600 E´ V 1= ¾
Ö2
´
\ B= =
6 -3 6
2 ´ 10 3 ´ 10 ´ 2 ´ 10
B = 0.1 Tesla Ð OEF = 45° (Q OE act as a radius)
4. m = 10–26 kg, q = + 1.6 × 10–19 C By symmetry Ð OFE = 45°
r ur \ Ð EOF = 90° (by Geometry)
v = (1.28 × 106 m/s) $i , E = -102.4 ´ 103 m/s k$ This is the centripetal acceleration
ur
B = 8 ´ 10-2 $j v2 1014
\ = 1015 Þ r = 15 = 0.1m.
The force on the charged particle due to electric field. r 10
F1 = qE in the – z direction Therefore EF = 0.141 m.
= 1.6 × 10–19 × 102.4 × 103 = 163.84 × 10–16 N If the magnetic field is in the outward direction and the
The force on the charged particle due to magnetic field. particle enters in the same way at E, then according to
F2 = qvB in the + z direction Fleming's left hand rule, the particle will turn towards
= 1.6 × 10–19 × 1.28 × 106 × 8 × 10–2 = 163.84 × 10–16 N clockwise direction and cover 3/4th of a circle as shown in
\ The net force in z-direction is zero and thus the net the figure.
force on the charged particle is zero and hence no
acceleration. O

45
Displacement of particle = v × t E
= 1.28 × 106 × 5 × 10–6 = 6.4 m
If the electric field is switched off at this instant then there
will be a force due to magnetic field which is in the + ve O
z-direction. O
45
This force is acting perpendicular to the velocity throughout 45O
and thus makes the particle to move in a circular path of F
radius
3 é 2p r ù
mv 10-26 ´ 1.28 ´ 106 \ Time required = ´ = 4.71 × 10–8 sec.
\ r= = = 1m 4 êë v úû
qB 1.6 ´ 10 -19 ´ 8 ´ 10-2
MOVING CHARGES & MAGNETISM P-389
(ii) The dimensions given shows that
6. SA2 + SB2 = AB2 Þ ÐASB = 90°
Magnetic field due to A at S
m 0 2I A m0 2 ´ 9.6
BSA = . = ´ (Directed S to B)
4p rSA 4p 1.6
Magnetic field due to B at S
v1 is responsible for horizontal motion of proton m0 2 I B m0 2 ´ 3
v2 is responsible for circular motion of proton BSB = . = (Directed S to A)
4p rSB 4p 1.2
mv22 The resultant magnetic field
\ = qv2 B
r
2 2 µ0 æ 9.6 ö 2 æ 3 ö 2
mv2 1.76 ´ 10-27 ´ 4 ´ 105 ´ 3 B= BSA + BSB = ç ÷ + çè ÷
r= = = 0.012 m 4p è 0.8 ø 0.6 ø
qB 1.6 ´ 10-19 ´ 0.3 ´ 2
= 10–7 × 13 = 1.3 × 10–6 T
Pitch of helix = v1 × T (iii) Force per unit length on wire B
2p r 2p r m0 2I A I B
where T= =
v2 v sin q =
4p rAB
2p r
Þ Pitch of helix = v cos q ´ 10-7 ´ 2 ´ 9.6 ´ 3
v sin q = = 28.8×10–7 N/m
2
= 2p r cot q = 2 × 3.14 × 0.012 × cot 60° = 0.044 m
This force will be repulsive in nature.
7. (i) The magnetic field at P due to current in wire A.
8. KEY CONCEPT : The magnetic field due to an infinitely
m 0 2I A m0 2 ´ 9.6 long current carrying conductor at one end of the conductor
BA = = ´ (Direction P to M) ...(i) at a distance r is given by
4p rAP 4p æ 10 ö
çè 2 + ÷ø
11 µ0 i
B=
NOTE : The current in wire B should be in upward direction 4p r
so as to cancel the magnetic field due to A at P. (By right
hand Thumb rule) ¥
The magnetic field at P due to current in wire B
i
A ´ Q
¥ L R M
i O S i ¥
1.

P
6m

10 m i
2m S 11
M to¥
1.2m
BA P BB N
\ Magnetic field due to current carrying conductor P at
B point O is
µ0 i
m0 2I B B1 =
BB = ´ ... (ii) 4p (OR)
4p æ 10 ö
çè ÷ø directed towards the reader perpendicular to the plane of
11
paper.
From (i) and (ii) Magnetic field due to current carrying conductor Q at point
m0 2 ´ 9.6 m 2I O is directed towards the reader perpendicular to the plane
´ = 0´ B of paper.
4p æ 10 ö 4p æ 10 ö
çè 2 + ÷ø çè ÷ø µ0 i
11 11
B2 =
4p (OS )
9.6 ´ 11 I B ´ 11 96
Þ = Þ IB = = 3A Magnetic field due to current carrying conductors L and M
32 10 32 at O is zero.
P-390 Topic-wise Solved Papers - PHYSICS
\ Resultant magnetic field at O 10. The magnetic field at the centre P due to current in wire
B = B1 + B2 NM is
(directed towards the reader perpendicular to the plane of
paper) y

m0 i m i m é 1 1 ù x
Þ B= + 0 = 0 iê + M
4p OR 4p OS 4p ë OR OS úû
v
30o a v sin60° v
é 1 1 ù

120°
= 10–7 × 10 × ê + –4 Q Q
0.02 0.02 ú = 10 tesla.
ë û 30o
P 60°
v cos60°
9. (i) The magnetic field (due to current in wire P) at R
N
m 0 2I p m0 2 ´ 2.5
= ´ = ´
4p rPR 4p 5
µ0 I
B1 = [sin 60° + sin 60°]
µ0 4p r
= [in the plane of paper downwards]
4p
µ0 I é 3 3ù µ0 2 I 3
B1 = ´ê + ú , B1 =
2.5 amp I amp 4X105m/s 4 p a/2 ë 2 2 û 4p a
× × e v=
P Q R +X directed away from the reader perpendicular to the plane of
2m paper.
rPR=5m r a M
sin 30° = Þ r= a
a 2 30°
Similarly, the magnetic field (due to current is wire Q) at R
S r P
µ 2 ´ I µ0 MS
= 0´ = I cos 30° =
a a
4p 2 4p
N
[in the plane of paper downwards] 3a
The total magnetic field at R [due to P and Q] \ MS = \ MN = 3a
2
µ0 µ0 µ The magnetic field at the centre P due to current in arc MN
B= + I = 0 (1 + I )
4p 4 p 4p is
[in the plane of paper downwards] m 0 2pI æ q ö m0 2pI æ 2p / 3ö m 0 2pI
The force experienced by the electron B2 = ç ÷= ç ÷=
2p a è 2p ø 4p a è 2p ø 4p 3a
F = qvB sin q directed towards the reader perpendicular to the plane of
µ0 paper
= evB sin 90° = 1.6 × 10–19× 4 × 105 × (1 + I) The net magnetic field
4p
But F = 3.2 × 10–20 N (Given) m 0 2 3I m 0 2pI
\ 3.2 × 10–20 = 1.6 × 10–19 × 4 × 105 × 10–7 (1 + I) B = B1 – B2 = -
4p a 4p 3a
Þ I = 4 amp.
(ii) Let us consider a position between Q and R. The m0 2I é pù m 2I
= ê 3 - ú = 0 ´ × (0.68)
magnetic field produced should be equal to 5 × 10–7 T in the 4pa ë 3p û 4p a
plane of paper acting upwards. (directed away from the reader perpendicular to the plane of
For this let the wire having current 2.5 amp be placed at a paper)
distance r from R and current flowing outwards the plane of The force acting on the charged particle Q when it has a
paper. velocity v and is instantaneously at the centre is
µ0 2 ´ 2.5 F = QvB sin q = QvB sin 90° = QvB
\ 5 × 10–7 = ´ or r = 1 m
4p r The acceleration produced
Let us consider another position beyond R collinear with P,
Q and R. Let it be placed at a distance r' from R, having F QvB Qv é m 0 2 I ù
A= = = ´ ´ (0.68) ú
current in the plane of paper. M m m êë 4 p a û
m0 2 ´ 2.5 0.11 m 0 IQv
\ 5 × 10–7 = ´ or r' = 1 m A=
4p r' ma
MOVING CHARGES & MAGNETISM P-391
11. The magnetic field produced at different points on OC will
y be different. Let us consider an arbitrary point P on OC
which is at a distance x from the origin. Let the magnetic
x field due to currents in A and B at P be B1 and B2 respectively,
M ®
v both being in the X-Y plane.
®
A R Y

Q 120o P B

Öa
2
a

+x
xq

2
B1sinq
N P
O X
B 2si nq C
The direction of acceleration is given by the vector product q
r ur -a q
v ´ B or by applying Fleming's left hand rule
\ Ð MPR = 120° – 90° = 30° B2 B1
Z A B2cosq
Since, Ð MPQ = 60 B1cosq
\ Ð RPQ = 30°
i.e., the acceleration vector makes an angle of 30° with the
negative x-axis. Let Ð BPO = ÐAPO = q
ALTERNATE SOLUTION ur µ 2I ur
| B1 | = 0 = | B2 |
ur ur 4p a 2 + x 2
m 2I 3 $ m 2pI $
B1 = 0 ( -k ); B2 = 0 k
4p a 4p 3a On resolving B1 and B2 we get that the sin q components
ur ur ur cancel out and the cos q components add up. Therefore,
m I é2 ù -m 2 I
B = B1 + B 2 = 0 ê - 2 3 ú k$ = 0 (1.4) (k$ ); the total magnetic field at P is
4p a ë 3 û 4p a B = 2B1 cos q
r
v = v cos 60$i + v sin 60 $j 2µ0 2I
´
m0x 4 Ix
=
= 4p 2 2 2 4p (a + x 2 )
2 2
ur ur ur a +x a +x
év 3 ˆù é -m0 2.8I ˆ ù
F = Q (V ´ B ) = Q ê $i + vjú ´ k (towards – Y direction)
ë2 2 û êë 4p a úû Let us consider a small portion of wire OC at P of length dx.
ur The small amount of force acting on that small portion
r F ur ur ur
Now apply a = dF = I (dx ´ B) \ dF = I dx B sin 90°
m
(b) KEY CONCEPT : The torque acting on the loop in the m0 4 Ix
Þ dF = I dx × ´
magnetic field is given by 4p ( a + x 2 )
2
r uur ur
t=M ´B where M = IA m0 2 xdx
Þ dF = 4I
A = (area of PMQNP) – (area of triangle PMN) 4p (a 2 + x 2 )
1 1 The total force
= (p a 2 ) - ´ MN ´ PS
3 2 m0 L xdx
F= ´ 4 I2ò
4p 0 (a 2 + x 2 )
p a2 1 a ép 3ù
= - ´ 3a ´ = a 2 ê - ú L
3 2 2 ë3 4 û m0 é1 ù
= ´ 4 I 2 ê log e (a 2 + x 2 ) ú
4p ë2 û0
ur 2 ép 3ù $
A = a ê - úk m0 é a2 + L2 ù
ë3 4 û Þ F= ´ 2 I 2 êloge ú
4p ëê a2 ûú
r ép 3ù $ $
\ t = Ia 2 ê - ú k ´ iB To find the direction of force we can use Fleming's left hand
ur
ë3 4 û rule. The direction of F is towards – Z direction.
When the current in wire B is reversed, the resultant magnetic
r æp 3ö $ 2$ field at any arbitrary point P on OC will be in the X-direction.
t = BIa 2 ç - ÷ j = 0.614 BIa j
è3 4 ø Since the current is also in X-direction, therefore force acting
The force acting on the loop is zero. will be zero (F = I l B sin q and q = 180°).
P-392 Topic-wise Solved Papers - PHYSICS
12. (a) Let us resolve the velocity into two rectangular When x is small i.e., x << r then r = x » r
components v 1 (= vcos 60°) and v 2 (= vsin 60°). v 1
m 0 2 I1I 2
component of velocity is responsible to move the charge Restoring force/length F = x
particle in the direction of the magnetic field whereas v2 4p r 2
component is responsible for revolving the charged particle Since, F µ x and directed to equilibrium position.
in circular motion. The overall path is helical. The condition \ The motion is simple harmonic
for the charged particle to strike S with minimum value of B
µ0 2I1 I 2
is that Pitch of Helix = GS \ = (mass per unit length) w2 ... (ii)
4p r 2
2p m S B
T × v1 = GS Þ × v cos 60° = 0.1 m 0 2 I1 I 2
qB From (i), (Mass per unit length) × g =
4p r
2 p mv cos 60° r = 0.1m
B= v1 = v cos 60° m 0 2I1I 2
q ´ 0.1 Mass per unit length = ... (iii)
v 4p rg
1 2E G
But mv2 = E Þ v = v2 = v sin 60o From (ii) and (iii)
2 m
m 0 2 I1I 2 m0 2I1I 2 g
2pm 2E = ´ w2 Þ w =
\ B= ´ ´ cos 60° 4p r 2 4p rg r
q ´ 0.1 m
2p g
2p 2 ´ 3.14 Þ =
= ´ 2mE ´ cos 60° = T r
q ´ 0.1 1.6 ´ 10-19 ´ 0.1
r 0.01
1 Þ T = 2p = 2p = 0.2sec
= 2 ´ 9.1 ´ 10 -31 ´ 2 ´ 103 ´ 1.6 ´ 10 -19 ´ g 9.8
2
14. (i) KEY CONCEPT : Orbital magnetic dipole moment M = IA
149.8
= ×0.316 × 10–23 = 47.37 × 10–4 where I is the current due to orbital motion of electron and A
10-19 is the area of loop made by electron.
= 4.737 × 10–3 T
e ew
13. When AB is steady, Þ M= ´ pR 2 Þ M = ´ pR 2
Weight per unit length = Force per unit length T 2p
m0 2 I1I 2 1 R
Weight per unit length = ... (i) Þ M= ewR 2 w
4p r 2
But according to Bohr's postulate e-
NOTE : When the rod is depressed by a distance x, then the
force acting on the upper wire increases and behaves as a nh nh
restoring force mRw2 = Þ Rw2 =
2p 2 pm
Fmag e nh nhe eh
Þ M= ´ = = (Q n = 1 for ground state)
A B 2 2pm 4 pm 4 pm
I1 = 20A x NOTE : The direction of magnetic momentum is same as
A'
mg the direction of area vector, i.e., perpendicular to the plane
r = 0.01m B'
of orbital motion.
(ii) KEY CONCEPT : We know that torque
r uur ur
C I2 = 30A D t = M ´ B Þ t = MB sin q
where q is the angle between M and B
m 0 2 I1I 2 m0 2 I1I 2
Restoring force/length = -
4p r - x 4p r B
m0 é 1 1ù n̂ o t
= 2 I1I 2 ê - ú 30
4p ë r - x rû

m0 é r - (r - x ) ù
Þ Restoring force/length = 2 I1I 2 ê ú
4p ë ( r - x) r û
e–
m0 2I1I2 x
=
4p r (r - x)
MOVING CHARGES & MAGNETISM P-393

he heB F m 2i i z é zù
Þ t= ´ B sin 30° = = 2 f cos q = 2 0 1 2 ´ êë as cos q = r úû
4pm 8pm l 4p r r
NOTE : The direction of torque can be found by right hand
thumb rule. F m0 4i 2 z
or = . [as I1 = I2 and r2 = d2 + z2]
The direction of torque is perpendicular to the plane l 4p ( d 2 + z 2 )
ur
containing n$ and B as shown. 2
15. (i) KEY CONCEPT : Magnetic field due to an infinitely F m æ 2i ö
or = - 0 ç ÷ z [as d >> z and F is opposite to z] ...(1)
long current carrying wire at distance r is given by l 4p è d ø
m0 æ 2i ö Since F µ – z, the motion is simple harmonic.
B= ç ÷ Comparing eq. (1) with the standard equation of S.H.M.
4p è r ø which is
The direction of B is given by right hand palm rule.
F m
Hence, in case of three identical wires, resultant field can be F = – mw2z i.e., = - w2 z
zero only if the point P is between the two wires, otherwise l l
field B due to all the wires will be in the same direction and = – l w2 z, we get
so resultant B cannot be zero. Hence, if point P is at a
distance x from the central wire as shown in figure, then, µ0 4i 2 µ0i 2
l w2 = ´ Þ w=
ur ur ur ur 4p d 2 pd 2 l
B P = B PA + B PB + B PC
ur
where B PA = magnetic field at P due to A m0i i m0
ur Þ 2pn= Þ n=
pld 2 pd p l
B PB = Magnetic field at P due to B
ur 16. Because the forces due to parallel electric and magnetic
B PC = Magnetic field at P due to C. fields on a charged particle moving perpendicular to the
ur m é 1 1 1 ù $ fields will be at right angles to each other, electric force
B P = 0 2i ê + - (- k ). ur
4p ë d + x x d - x ûú being along the direction of E while magnetic force
r ur
ur perpendicular to the plane containing v and B , so magnetic
For B P = 0 , we get x = ± d 3
force will not affect the motion of charged particle in the
(ii) KEY CONCEPT : The force per unit length between
direction of electric field and vice-versa. So, the problem is
two parallel current carrying wires is given by equivalent to superposition of two independent motion as
m 0 2i1i2 shown in the adjoining figures.
= f (say)
4p r
Y
and is attractive if currents are in the same direction.
Y B E Fe

i C
>

j
A B

P
X
O x
>

i
Z k X
d d Fm vo
Z
d d So, for motion of the particle under electric field alone,
A B
X qE dv y qE
r ay = i.e., =
Z m dt m
f f
q q y t qE qE
B
or ò0 dv y =ò0 m m
t
dt i.e., vy = ... (1)

Z
While at the same instant, the charged particle under the
action of magnetic field will describe a circle in the x-z plane
So, when the wire B is displaced along Z-axis by a small with
F mv0 v0 qB
distance Z, the restoring force per unit length on the r= i.e., w = =
l qB r m
wire B due to wires A and C will be
P-394 Topic-wise Solved Papers - PHYSICS
So, in vector from
v0
X ur æ B ö $æ B ö uur
v0 cosq B = i$ ç ÷ + jç ÷ and M = I = I 0 L2 k$
q q è 2ø è 2ø
v0 sinq v0 2
r uur ur 2$ æ B $ B $ ö I0 L B ˆ ˆ
So, t = M ´ B = I0 L k ´ ç
è 2
i+ j÷ = ( j -i)
2 ø 2

Z i.e., torque has magnitude I 02 L2 B and is directed along


line QS from Q to S.
So, angular position of the particle at time t in the x-z plane (b) According to the theorem of perpendicular axes,
will be given by moment of inertia of the frame about QS.
qB 1 1æ4 ö 2
q = wt = t I z = ç ML2 ÷ = ML2
m IQS =
2 2è3 ø 3
and therefore, in accordance with figure Also t = Ia,
æ qB ö
vx = v0 cos q = v0 cos wt = v0cos çè t ÷ø ... (2) t I 0 L2 B ´ 3 3 I 0 B
m \ a= = =
I 2ML2 2 M
æ qB ö Here a is constant, therefore we can apply
and vz = v0 sin q = v0 sin wt = v0sin ç t ÷ ... (3)
è m ø 1
q = w 0t + a t 2 with w0 = 0, we have
From equations (1), (2) and (3), we get 2
r
$ + $jv + kv
v = iv $ 1 2 1 æ 3I 0 B ö
x y z
q= at = ç ( Dt ) 2
2 2 è 2 M ÷ø
$ æ qB ö $ æ qE ö $ æ qB ö
= iv0 cos çè t÷ + j ç t ÷ + k v0 sin ç t ÷
m ø è m ø è m ø 3 I0B
or q= ( Dt ) 2
r ur ur 4 M
v E B 18. KEY CONCEPT : This question involves a simple
But because here, i$ = 0 , $j = =
v0 E B understanding of the motion of charged particle in a magnetic
field.
r r
$ v0 ´ B
and k =
v0 B (a) × × × × ×
r ur A × × × × ×
r æ v0 ö æ qBt ö æ E ö qE vo
So, v 0 = ç ÷ v0 cos ç + t 30° × × × ×
è v0 ø è m ÷ø çè E ÷ø m M × × × × ×
P

r ur × × × × × B = B0 kˆ
æ v0 ´ B ö æ qB ö × × × × ×
+ç v0 sin ç t B X
è v0 B ø
÷ è m ÷ø O × × × × ×
ur × × × × ×
17. (a) As the magnetic field B is in x – y plane and subtends × × × × ×
an angle of 45° with the x-axis, hence,
× × × × ×
Bx = B cos 45° = B / 2 and × × × × ×
By = B sin 45° = B / 2 × × × × ×
X=0 X=L
Y
Io Let the particle emerge out from the region of magnetic field
S R r
at point P. Then the velocity vector v 0 makes an angle 30°
B with x-axis. The normal to circular path at P intersects the
t
45o
negative y-axis at point A.
A X Hence, AO = AP = R = radius of circular path, which can be
found as
mv02 mv0
P
= B0 qv0 Þ R = qB ... (i)
Q R 0
MOVING CHARGES & MAGNETISM P-395

R (b) Force acting on a current carrying conductor placed in


In DAPM, R sin 30° = L Þ =L ... (ii) a magnetic field is given by
2 ur r ur
F = I (l ´ B) = I lB sin q
mv0
From (i) and (ii), L = 2qB . (i) For force acting on the wire at the centre
0 In this case q = 180°
\ F=0
(b) - v0 iˆ (ii) On arc AC due to current at the centre
ur m0 I
| B | on AC will be B = 2 p r
1
The direction of this magnetic field on any small
segment of AC will be tangential
v0 iˆ X \ q = 180° Þ F = 0
2.1mv0 /2qB0 (iii) On segment CD.
As the new region of magnetic field is 2.1 L Force on a small segment dx distant r from O
2.1R
= which is obviously > R.
2
r2= 0.12m D
Thus, the required velocity = -v $i. 0 dx
D
Since the time period for complete revolution = 2pm/qB0.
The time taken by the particle to cross the region of magnetic O
field = pm/qB0. r1= 0.08m B
ur x
C
19. (a) Magnetic field ( B) at the origin = Magnetic field due
to semicircle KLM + Magnetic field due to other semicircle
KNM.
ur m 0 I m I dF = I dxB
Therefore, B = ( -i$) + 0 ( $j )
4R 4R m0 I 5m 0 I dx
= 10 × dx × =
ur m I m I m I 2p x p x
Þ B = - 0 $i + 0 $j = 0 ( -$i + $j )
4R 4R 4R On integrating
ur
[NOTE : The magnetic field B due to a circular current 5 µ0 I r2 dx 5 m0 I
\ F=
p òr1 x
\ F=
p
[ log e x ] rr12
m0 I
carrying loop is \ For semicircle it is half]
2R 5 m0 I r 5m ´ 10 æ 0.12 ö
log e 2 = 0 log e ç
Therefore, magnetic force acting on the particle. \ F= p r1 p è 0.08 ÷ø
ur r ur ì m Iü = 8.1 × 10–6 N
F = q (v ´ B) = q í( - v0 i$) ´ (-$i + $j ) ´ 0 ý directed downwards (By Fleming left hand rule).
î 4R þ
21. When the ring is not rotating
-m 0 qv0 I $ Wt. of ring = Tension in string
= k mg = 2T0
4R
ur ur ur mg
(b) F KLM = F KNM = F KM \ T0 = .... (i)
ur 2
and F KM = BI (2 R )$i = 2 BIRi$ d
ur ur
Therefore, F1 = F 2 = 2BIRi$ or total force on the loop,
ur ur ur ur
F = F 1 + F 2 Þ F = 4B I R iˆ T1 T2
20. For finding the magnetic field produced by this circuit at the
centre we can consider it to contain two semicircles of radii, w0
r1 = 0.08 m and r2 = 0.12 m. Since current is flowing in the O P
same direction, the magnetic field created by circular arcs
will be in the same direction and therefore will be added. B

m0 i m0 i m 0i é 1 1 ù
\ B1 = and B2 = 4r \ B = 4 ê r + r ú When the ring is rotating, we can treat it as a current carrying
4r1 2 ë 1 2û loop. The magnetic moment of this loop
\ B = (6.54 × 10–5) T Directed outwards. Q Q
(Right hand thumb rule) M = iA = × pr2 = w × pR2
T 2p
P-396 Topic-wise Solved Papers - PHYSICS
This current carrying loop will create its own magnetic field But M = N i A and q = 90° (for moving coil galvanometer)
which will interact with the given vertical magnetic field in \ t = N i A B sin 90°
such a way that the tensions in the strings will become Þ t=NiAB
unequal. Let the tensions in the strings be T1 and T2.
But t = k i (given)
For translational equilibrium
\ k i = NiAB
T1 + T2 = mg ... (ii)
Þ k = NAB
Torque acting on the ring about the centre of ring
r uur ur (b) The torsion constant is given by
t=M ´B
t = M × B × sin 90° t NiAB
C= =
q q
Q QwBR 2
= w ´ pR 2 ´ B = Here given that when i = i0, q = p / 2
2p 2
NOTE : For rotational equilibrium, the torque about the 2 N i0 A B
\ C= ... (i)
centre of ring should be zero. p
(c) We know that angular Impulse
D D QwBR 2
\ T1 × - T2 ´ =
2 2 2 = ò tdt = ò NiAB dt
QwBR 2
Þ T1 – T2 = ... (iii) = NAB ò i dt
D
On solving (ii) and (iii), we get = NABQ ... (ii)
This angular impulse creates an angular momentum
mg QwBR 2
T1 =
2
+
2D ò t dt = I w ... (iii)
From (ii) and (iii)
3T0
But the maximum tension is NABQ
2 I w = NABQ Þ w =
I
3T0 Qw max BR 2 é mg ù
\ = T0 + êëQ T0 = 2 úû This is the instantaneous angular momentum due to which
2 2D the coil starts rotating. Let us apply the law of energy
DT0 conservation to find the angle of rotation.
\ wmax = Rotational kinetic energy of coil
BQR 2
22. KEY CONCEPT : 1 2 1 I N 2 A2 B 2Q 2 N 2 A2 B 2Q 2
= Iw = =
1 2 1 2 2 I2 2I
eV = mv p and eV = mva2
2 2
V is the potential difference 1 2 N 2 A2 B 2Q 2
C qmax =
vp = velocity of proton 2 2I
va = velocity of a-particle
m = mass of proton, mass of a-particle = 4 m N 2 A2 B 2Q 2 N 2 A2 B 2Q 2
Þ q2max = = ´p
2eV 2eV CI 2 Ni0 ABI
Þ vp = , va =
m 4m
p NAB Q 2 NAB p
Now when the particles enter in magnetic field, the force on Þ q2max = Þ qmax = Q .
proton is 2i0 I 2 Ii0

mv 2p mv p m MATCH THE FOLLOWING :


evpB = or rp = Þ ra =
rp eB eB 1. (A) Charge on ring will create electric field which is time
independent.
2eV 1 2mV 1 4mV
= and ra = (B) The rotating charge is like a current. This will create a
m B e B e magnetic field and a magnetic moment.
rp 1 (C) Since net charge is zero there will be no time
\ = independent electric field. The current produces
ra 2 magnetic field and magnetic moment.
23. (a) The torque acting on a rectangular coil placed in a (D) A changing magnetic field will be produced. This will
uniform magnetic field is given by, create a induced electric field. Also a changing magnetic
r uur ur
t = M ´ B Þ t = MB sin q moment will be produced.
MOVING CHARGES & MAGNETISM P-397
2. A: q
Reason : When a charged capacitor is connected to the ASSERTION & REASON TYPE QUESTIONS :
ends of the wire, a variable current (decreasing in magnitude
with time) passes through the wire (shown as resistor) and θ NBA
thermal energy is generated. The potential difference across 1. (c) Statement-1 is true. Sensitivity = = . If B
I C
the wire also decreases with time. The charge on the
capacitor plate also decreases with time. θ
B : r. s increases, increases. Statement-2 is wrong because
I
Reason : e = Blv soft iron can be easily magnetised and de magnetized.
When B, l, v are constant, e is constant
Þ A constant potential difference develops across the INTEGERVALUECORRECTTYPE:
ends of the wire and charges of constant magnitude appear
at the ends of the wire. 1. (k = 7)
C:s The right angled triangle is shown in the figure. Let us drop
Reason : The free electrons move under the influence of a perpendicular from P on QR which cuts QR at M.
electric field opposite to the direction of electric field. This The magnatic field due to currents in PQ and RP at P is zero.
movement of e– continues till the electric field inside the The magnetic field due to current in QR at P is
wire is zero.
Þ Changes of constant magnitude appear at the ends of
the wire. Q a
D : p, q, r q1
M
Reason : Since, E, R are constant, a constant current flows
I (5x – a)
in the wire. Due to heating effect of current, thermal energy
is generated in the wire. Also a constant potential difference
develops between the ends of the wire. 3x
3. A : q, r
Reason : The magnetic field at P due to current flowing in q2
P R
AB is perpendicular to the plane of paper acting vertically 4x
downward. And the magnetic field at P due to current flowing
in CD is perpendicular to the plane of paper acting vertically
m0 I
upwards. B= (cos q1 + cos q2 ) …(i)
Therefore, q is correct. 4p PM
As P is the mid point, the two magnetic fields, cancel out In DPQM,
each other. Therefore, r is correct. 9x2 = PM2 + a2 … (ii)
B:p
In DPRM,
Reason : The magnetic field at P due to current in loop A is
along the axial line towards right. Similarly, the magnetic 16x2 = PM 2 + (5x – a)2 … (iii)
field at P due to current in loop B is also along the axial line Þ 18x2
7 x 2 = 25 x 2 - 10 xa Þ 10xa =
towards right.
C : q, r Þ a = 1.8 x … (iv)
Reason : The magnetic field due to current in loop A at P is From (ii) & (iv),
equal and opposite to the magnetic field due to current in
loop B at P. 9 x 2 = PM 2 + (1.8 x)2
D : q, s
2
Reason : The direction of magnetic field at P due to current Þ PM = 9 x 2 - 3.24 x 2 = 5.76 x = 2.4 x … (v)
in loop A is perpendicular to the plane of paper directed
vertically upwards. a 1.8 x
Also cos q1 = = = 0.6 … (vi)
The direction of magnetic field at P due to current in loop B 3x 3x
is perpendicular to the plane of paper directed vertically
downward. 5 x - a 5 x - 1.8 x 3.2
cos q2 = = = = 0.8 …(vii)
Since the current are in opposite direction the wires repel 4x 4x 4
each other. But net force on each wire is zero.
From (i), (v), (vi) and (vii),
COMPREHENSION BASED Q UESTIONS :
m0 I m0 I ém I ù
1. (d) The magnetised coils running along the track repel large B= ´
4p 2.4 x
[ 0.6 + 0.8] = ´
4 p 2.4 x
´ 1.4 = 7 ê 0 ú
ë 48px û
magnets on the train's under carriage.
2. (d) Initial cost will be more.
3. (b) The magnetic force will pull the vehicle. é m I ù
Comparing it with B = k ê 0 ú , we get, k = 7.
ë 48p x û
P-398 Topic-wise Solved Papers - PHYSICS
2. (6) Let us consider an amperian loop ABCD which is a Comparing it with the expression given in the
rectangle as shown in the figure. question we get
Applying ampere’s circuital law we get
300p2 r 4 1 300(3.14)2 ´ (0.1)4
N= ´ = =6
R L 0.005 ´10

D C current I I'
3. (5) Current density J = = =
L
l
area p(2a) 2
pa 2 ( )
A B I
Þ I' =
4
r uur Let us consider the cavity to have current I' flowing in
Ñò B.d l = mo × (current passing through the loop) both the directions.
r uur The magnetic field at P due to the current flowing
æIö
\ Ñò B.d l = mo ç ÷ ´ l
è Lø
through the cylinder
µ0 2 I
I B1 =
\ B × l = mo ´ l 4p a
L
The magnetic field at P due to the current (I’) flowing
mo I mo in opposite direction is
\ B= = Io cos (300 t)
L L
µ 0 3I ' µ 2( I /4) µ0 I
The magnetic moment of the loop B2 = = 0 =
= (current in the loop) × pr2 4 p 3 a /2 4 p 3 a /2 4 p 3a
1 æ dfö 2
\ The net magnetic field is
= çè - ÷ø ´ pr
R dt µ0 I é 1ù µ I 5
B = B1 – B2 = ê 2- ú = 0 ´
1 éd ù p 2 r 4 dB 4p a ë 3 û 4p a 3
=- ê ( B ´ pr 2 ) ú ´ pr 2 = -
R ë dt û R dt
µ 0 J p a2 5 5 Ja
\ B= ´ = µ0
é p2 r 4 mo ù 4p a 3 12
= ê R ´ L I o sin(300 t ) ú ´ 300
êë úû

1. (a) KEY CONCEPT : We know that the magnetic field Here p, q and B are constant for electron and proton,
produced by a current carrying circular coil of radius r therefore the radius will be same.
m0 I 3. (c) Magnetic field due to current in wire 1 at point P distant
at its centre is B = ´ 2p r from the wire is
4p r
m0 I m0 2I
Here B A = ´ 2p and BB = ´ 2p
4p R 4p 2 R
q i2
BA
Þ =1 i1 P
BB m 0 i1 r
2. (a) KEY CONCEPT : When a charged particle enters B= [ cos q + cos q] dl
4p r
perpendicular to a magnetic field, then it moves in a q
circular path of radius.
p
r=
qB m 0 i1 cos q
B= (directed perpendicular to the plane
where q = Charge of the particle 2p r
p = Momentum of the particle of paper, inwards)
B = Magnetic field The force exerted due to this magnetic field on current
element i2 dl is
MOVING CHARGES & MAGNETISM P-399
dF = i2 dl B sin 90°
I 1 2
é m 0 i1 cos q ù m0 Case 1 : T = 2p where I = M l
mBH 12
\ dF = i2 dl ê ú= i1 i2 dl cos q
ë 2 p r û 2 pr
Case 2 : Magnet is cut into two identical pieces such
4. (a) KEY CONCEPT : The time period of a charged particle that each piece has half the original length. Then

(m, q) moving in a magnetic field (B) is T = 2pm I'


qB T ' = 2p
m ' BH
The time period does not depend on the speed of the
particle. 2
1 æ M ö æ lö I m
5. (b) The workdone, dW = Fds cosq where I ' = ç ÷ ç ÷ = and m ' =
12 è 2 ø è 2 ø 8 2
The angle between force and displacement is 90° .
Therefore work done is zero.
T' I' m I /8 m 1 1
\ = ´ = ´ = =
× × × T m' I m/2 I 4 2

8. (a) W = MB (cos q1 - cos q2 )


× × ×
1 MB
= MB (cos 0° - cos 60°) = MB(1 - ) =
F 2 2
MB
\ t = MB sin q = MB sin 60° = 3 = 3W
× × 2
S 9. (d) As shown in the figure, the magnetic lines of force are
6. (a) The situation is shown in the figure. directed from south to north inside a bar magnet.
FE= Force due to electric field
FB = Force due to magnetic field
It is given that the charged particle remains moving
along X-axis (i.e. undeviated). Therefore FB = FE

E 104 N S
Þ qvB = qE Þ B = = = 103 weber/m2
v 10
Y
Z'

10. (a) KEY CONCEPT : The temperature above which a


B ferromagnetic substance becomes paramagnetic is
E called Curie’s temperature.
11. (b) Using Ampere’s law at a distance r from axis, B is same
FB from symmetry.
X
v ò B.dl = m0i i.e., B ´ 2pr = m 0i
E
Here i is zero, for r < R, whereas R is the radius
\ B=0
Z 12. (b) KEY CONCEPT : Magentic field at the centre of a
7. (b) KEY CONCEPT : The time period of a rectangular m0i
circular coil of radius R carrying current i is B =
magnet oscillating in earth’s magnetic field is given by 2R
I Given : n ´ (2pr ') = 2pR
T = 2p
mBH Þ nr ' = R ...(1)
where I = Moment of inertia of the rectangular n.m 0i
B' = ...(2)
magnet 2r '
m = Magnetic moment
nm 0i.n
BH = Horizontal component of the earth’s magnetic from (1) and (2), B ' = = n2 B
field 2 pR
P-400 Topic-wise Solved Papers - PHYSICS
13. (c) The magnetic field at a point on the axis of a circular 18. (c) Equating magnetic force to centripetal force,
loop at a distance x from centre is,
mv 2
m 0i a 2
m0 i = qvB sin 90º
B= B' = r
2 2 3/2 2a
2( x + a ) Time to complete one revolution,

B.( x 2 + a2 )3/ 2 2pr 2pm


\ B' = T= =
v qB
a3
19. (d) A magnetic needle kept in non uniform magnetic field
54(53 ) experience a force and torque due to unequal forces
Put x = 4 & a = 3 Þ B ' = = 250 µT
3´3´ 3 acting on poles.
14. (a) Force between two long conductor carrying current, 20. (b) Due to electric field, it experiences force and accelerates
i.e. its velocity decreases.
m0 2I1I 2
F= ´l 21. (b) Ferromagnetic substance has magnetic domains
4p d whereas paramagnetic substances have magnetic
µ0 2(2I1 ) I 2 F ' -2 dipoles which get attracted to a magnetic field.
F'= - l \ =
4p 3d F 3 Diamagnetic substances do not have magnetic dipole
but in the presence of external magnetic field due to
I 1 their orbital motion of electrons these substances are
15. (b) T = 2p where I = ml 2
M ´B 12 repelled.
When the magnet is cut into three pieces the pole 22. (b) The charged particle will move along the lines of electric
strength will remain the same and field (and magnetic field). Magnetic field will exert no
2
force. The force by electric field will be along the lines
1 æ m öæ l ö I of uniform electric field. Hence the particle will move in
M.I. (I¢) = ´3 =
12 çè 3 ø÷ èç 3 ø÷ 9 a straight line.
We have, Magnetic moment (M)
i
= Pole strength (m) × l B2 m0 n2i2 100 ´
= B2 3
\ New magnetic moment, 23. (a) B1 m 0 n1i1 Þ =
6.28 ´ 10 -2 200 ´ i
æ lö T 2
M ' = m ´ ç ÷ ´ 3 = ml = M \ T ' = = s.
è 3ø 9 3 6.28 ´ 10-2
Þ B2 = = 1.05 ´ 10 -2 Wb / m2
16. (b) NOTE : Electro magnet should be amenable to 6
magnetisation & demagnetization. 24. (d) Here, current is uniformly distributed across the cross-
\ retentivity should be low & coercivity should be section of the wire, therefore, current enclosed in the
low
æ aö
amperean path formed at a distance r1 ç = ÷
17. (d) (1) è 2ø
®
B2
®
B1 (2)

a/2
P1 P2

The magnetic field due to circular coil 1 and 2 are

µ0 i1 m 0 i1 m ´ 3 ´ 102
B1 = = = 0
2r 2 (2 p ´ 10 -2 ) 4p

m0 i2 m0 ´ 4 ´ 10 2 æ p r2 ö
B2 = = = ç 12 ÷ ´ I , where I is total current
2(2p ´ 10 -2 ) 4p èpa ø
m0 \ Magnetic field at P1 is
B= B12 + B22 = . 5 × 102
4p m0 ´ current enclosed
B1 =
Þ B = 10 -7 2 Þ B = 5 × 10–5 Wb / m2 Path
´ 5 ´ 10
MOVING CHARGES & MAGNETISM P-401

æ p r2 ö \ Resultant field, B = B12 + B22


m 0 ´ ç 12 ÷ ´ I
è pa ø m ´ I r1 m 0 I1 m I
Þ B1 = = 0 But B1 = and B2 = 0 2
2p r1 2p a 2 2pd 2pd

Now, magnetic field at point P2, 2

m0 I m I
æ m ö
\ B= ç 0 ÷
è 2pd ø (I 2
1 + I 22 )
B2 = . = 0 .
2p (2a) 4pa
m0 2
( )
1/ 2
or, B = I1 + I 22
B1 m 0 Ir1 4pa 2pd
\ Required ratio = = ´
B2 2pa 2 m 0 I 29. (c) The magnetic field is
m0 2I 2 ´ 100
a B= = 10 -7 ´ = 5 × 10–6 T
2 r1 2´ 4p r 4
= = 2 = 1.
a a
25. (d) There is no current inside the pipe. Therefore W N
ur uur
oò B . d l = µo I
100A
I=0
\ B=0
r r
26. (b) Here, E and B are perpendicular to each other and
r 4m
the velocity v does not change; therefore
E
S
E
qE = qvB Þ v = Ground
B
Also, B
r r
E´B E B sin q E B sin 90° E r
= = = = |v| = v According to right hand palm rule, the magnetic field is
2 2 2 B
B B B directed towards south.
27. (b) NOTE : When a charged particle enters a magnetic 30. (b) For a diamagnetic material, the value of µr is less than
field at a direction perpendicular to the direction of
one. For any material, the value of Îr is always greater
motion, the path of the motion is circular. In circular
motion the direction of velocity changes at every point than 1.
(the magnitude remains constant). 31. (a) The magnetic field at O due to current in DA is
Therefore, the tangential momentum will change at mo I p
every point. But kinetic energy will remain constant as B1 = ´ (directed vertically upwards)
4p a 6
1
it is given by mv 2 and v 2 is the square of the The magnetic field at O due to current in BC is
2
magnitude of velocity which does not change. mo I p
B2 = ´ (directed vertically downwards)
28. (c) Clearly, the magnetic fields at a point P, equidistant 4p b 6
from AOB and COD will have directions perpendicular The magnetic field due to current AB and CD at O is
to each other, as they are placed normal to each other. zero.
Therefore the net magnetic field is
A P D
B = B1 - B2 (directed vertically upwards)
B1 B2
mo I p m o I p m I æ 1 1ö m I
= - ´ = o ç - ÷ = o (b - a )
I1 d I2 4 p a 6 4p b 6 è
24 a b ø 24ab
r r r
32. (a) KEY CONCEPT : F = I ( l ´ B)
O
The force on AD and BC due to current I1 is zero. This
uur
is because the directions of current element I d l and
r
magnetic field B are parallel.
C B
P-402 Topic-wise Solved Papers - PHYSICS
33. (a) The magnetic field varies inversely with the distance \ Magnetic dipole moment (M)
1
for a long conductor. That is, B µ . According to the q æ mR 2 ö 1
d M = .ç 4
÷ .w = s.pR w.
magnitude and direction shown graph (1) is the correct 2m è 2 ø 4
one.
37. (a) The magnetic field due a disc is given as
dq
34. (d) Current in a small element, dI = I
H 0 wQ 1
p B= i.e., B µ
Magnetic field due to the element 2 pR R
m 0 2dI mv2
dB = mv
4p R 38. (b) = qvB Þ r =
r qB
The component dB cos q, of the field is cancelled by
another opposite component. m p vp md vd m v
Therefore, Þ rp = ; rd = ; ra = a a
qpB qd B qa B

ma = 4mp , md = 2mp

qa = 2qp , q d = qp
From the problem
1 1 1
dB Ep = Ed = Ea = mp vp 2 = md vd 2 = ma va2
2 2 2
p
m0 I m0 I
Bnet = ò dB sin q = ò sin qd q = Þ vp2 = 2vd2 = 4mv22
2p 2 R 0 p2 R

35. (a) Lorentz force acting on the particle Thus we have, ra = rp < rd
ur ur r ur
F = q éë E + v × B ùû 39. (b) Given : M 1 = 1.20 Am2

é kˆ ù
ˆi ˆj N
ê ú
= q ê 3i$ + $j + 2k$ + 3 4 1 ú BH
ê ú B1
ê 1 1 -3 ú
ë û S B2 S
O
= q éë3i$ + $j + 2kˆ + $i ( -12 - 1) - $j ( -9 - 1) + k$ (3 - 4 ) ùû
N N
r r

= q éë3iˆ + $j + 2k$ - 13i$ + 10$j - k$ ùû S

= q éë -10i$ + 11j$ + k$ ùû
M 2 = 1.00 Am2
Fy = 11qjˆ
Thus, the y component of the force. 20
r= cm = 0.1m
2
q Magnetic dipole moment
36. (c) = Bnet = B1 + B2 + BH
2m Angular momentum
m 0 ( M1 + M 2 )
Bnet = + BH
4p r3

10 -7 (1.2 + 1)
a = + 3.6 ´ 10 -5 = 2.56 ´ 10 -4 wb/m2
3
(0.1)
Electromagnetic Induction
15 & Alternating Current
FILL IN THE BLANKS : BR AB
(a) (b)
1. A uniformly wound solenoidal coil of self inductance A R
1.8 × 10–4 henry and resistance 6 ohm is broken up into two
identical coils. These identical coils are then connected in B2 A
parallel across a 15-volt battery of negligible resistance. The (c) ABR (d)
R2
time constant for the current in the circuit is ........... seconds
2. A thin semi-circular conducting ring of radius R is falling
and the steady state current through the battery is ............... ur
amperes. (1989 - 2 Marks) with its plane vertical in horizontal magnetic induction B .
2. In a straight conducting wire, a constant current is flowing At the position MNQ the speed of the ring is v, and the
from left to right due to a source of emf. When the source is potential difference developed across the ring is
switched off, the direction of the induced current in the wire
will be..... (1993 - 1 Mark)
3. The network shown in figure is part of a complete circuit.
If at a certain instant the current (I) is 5A, and is decreasing
at a rate of 103 A/s then VB – VA = .....V (1997C - 1 Mark)

TRUE / FALSE :
(a) zero (1996 - 2 Marks)
1. An e.m.f. can be induced between the two ends of a straight (b) BvpR2/2 and M is at higher potential
copper wire when it is moved through a uniform magnetic (c) pRBv and Q is at higher potential
field. (1980) (d) 2RBv and Q is at higher potential.
2. A coil of metal wire is kept stationary in a non-uniform 3. Two identical circular loops of metal wire are lying on a table
magnetic field. An e.m.f. is induced in the coil. without touching each other. Loop-A carries a current which
(1986 - 3 Marks) increases with time. In response, the loop-B
3. A conducting rod AB moves parallel to the x-axis (see Fig.) (a) remains stationary (1999S - 2 Marks)
in a uniform magnetic field pointing in the positive (b) is attracted by the loop-A
z-direction. The end A of the rod gets positively charged. (c) is repelled by the loop-A
(1987 - 2 Marks) (d) rotates about its CM, with CM fixed
4. A coil of inductance 8.4 mH and resistance 6 W is connected
to a 12 V battery. The current in the coil is 1.0 A at
approximately the time (1999S - 2 Marks)
(a) 500 s (b) 25 s
(c) 35 ms (d) 1 ms
5. A uniform but time-varying magnetic field B(t) exists in a
circular region of radius a and is directed into the plane of
the paper, as shown. The magnitude of the induced electric
field at point P at a distance r from the centre of the circular
region (2000S )
MCQ's WITH ONE CORRECT ANSWER :
1. A thin circular ring of area A is held perpendicular to a
uniform magnetic field of induction B. A small cut is made in
the ring and a galvanometer is connected across the ends
such that the total resistance of the circuit is R. When the
ring is suddenly squeezed to zero area, the charge flowing (a) is zero (b) decreases as 1/r
through the galvanometer is (1995S) (c) increases as r (d) decreases as 1/r2
P-404 Topic-wise Solved Papers - PHYSICS
6. A coil of wire having inductance and resistance has a 11. When an AC source of emf e = E0 sin(100t) is connected
conducting ring placed coaxially within it. The coil is across a circuit, the phase difference between the emf e and
connected to a battery at time t = 0, so that a time-dependent the current i in the circuit is observed to be p 4 , as shown
current l1(t) starts flowing through the coil. If I2(t) is the
current induced in the ring, and B(t) is the magnetic field at in the diagram. If the circuit consists possibly only of R-C
the axis of the coil due to I1(t), then as a function of time or R-L or L-C in series, find the relationship between the
(t > 0), the product I2(t) B(t) (2000S ) two elements (2003S)
(a) increases with time
(b) decreases with time i e
(c) does not vary with time
(d) passes through a maximum
7. A metallic square loop ABCD is moving in its own plane
with velocity v in a uniform magnetic field perpendicular to
its plane as shown in the figure. An electric field is induced
(2001S)
A B
(a) R = 1k W , C = 10 µF (b) R = 1k W , C = 1 µF
v (c) R = 1k W , L = 10 H (d) R = 1k W , L = 1 H
12. A small bar magnet is being slowly inserted with constant
D C
velocity inside a solenoid as shown in figure. Which graph
(a) in AD, but not in BC (b) in BC, but not in AD best represents the relationship between emf induced with
(c) neither in AD nor in BC (d) in both AD and BC time (2004S)
8. Two circular coils can be arranged in any of the three
situations shown in the figure. Their mutual inductance will
be (2001S)

(a) (b) (c)


(a) maximum in situation (a) (b) maximum in situation (b) (a) (b) Time
Time
(c) maximum in situation (c) (d) the same in all situations
9. As shown in the figure, P and Q are two coaxial conducting
loops separated by some distance. When the switch S is
closed, a clockwise current IP flows in P (as seen by E) and
an induced current IQ1 flows in Q. The switch remains closed (c) (d)
for a long time. When S is opened, a current IQ2 flows in Q. Time Time
Then the direction IQ1 and IQ2 (as seen by E) are 13. An infinitely long cylinder is kept parallel to an uniform
(2002S) magnetic field B directed along positive z-axis. The
direction of induced current as seen from the z-axis will
be (2005S)
(a) zero
(b) anticlockwise of the +ve z axis
(c) clockwise of the +ve z axis
(d) along the magnetic field
14. Find the time constant (in ms) for the given RC circuits in the
given order respectively (2006 - 3M, –1)
(a) respectively clockwise and anti-clockwise
(b) both clockwise
(c) both anti-clockwise
(d) respectively anti-clockwise and clockwise
10. A short-circuited coil is placed in a time-varying magnetic
field. Electrical power is dissipated due to the current induced
in the coil. If the number of turns were to be quadrupled and
the wire radius halved, the electrical power dissipated would
be (2002S )
(a) halved (b) the same
(c) doubled (d) quadrupled
ELECTROMAGNETIC INDUCTION & ALTERNATING CURRENT P-405

V V

R1 C1

R1

R2
The current induced in the loop is:
R2 C2 (a) BLv/R clockwise (b) BLv/R anticlockwise
(c) 2BLv/R anticlockwise (d) zero.
R1 = 1W, R2 = 2W, C1 = 4mF , C2 = 2mF 3. Two different coils have self-inductances L1 = 8 mH and L2
8 8 = 2 mH. The current in one coil is increased at a constant
(a) 18, 4, (b) 18, ,4 rate. The current in the second coil is also increased at the
9 9
same constant rate. At a certain instant of time, the power
8 8 given to the two coils is the same. At that time, the current,
(c) 4, 18, (d) 4, , 18
9 9 the induced voltage and the energy stored in the first coil
15. The figure shows certain wire segments joined together to are i1, V1 and W1 respectively. Corresponding values for the
form a coplanar loop. The loop is placed in a perpendicular second coil at the same instant are i2, V2 and W2 respectively.
magnetic field in the direction going into the plane of the Then: (1994 - 2 Marks)
figure. The magnitude of the field increases with time. I1 and i1 1 i1
I2 are the currents in the segments ab and cd. Then, (2009) (a) = (b) =4
i2 4 i2
c d W1 1 V1
a (c) = (d) =4
b W2 4 V2
4. A small square loop of wire of side l is placed inside a large
square loop of wire of side L(L>>l). The loops are co-planar
and their centres coincide. The mutual inductance of the
system is proportional to (1998S - 2 Marks)
(a) l/L (b) l2/L
(a) I1 > I2 (c) L/l (d) L2/l
(b) I1 < I2 5. The SI unit of inductance, the henry, can be written as
(c) I1 is in the direction ba and I2 is in the direction cd (1998S - 2 Marks)
(d) I1 is in the direction ab and I2 is in the direction dc (a) weber/ampere (b) volt-second/ampere
16. An AC voltage source of variable angular frequency w and (c) joule/(ampere)2 (d) ohm-second
fixed amplitude V0 is connected in series with a capacitance 6. A metal rod moves at a constant velocity in a direction
C and an electric bulb of resistance R (inductance zero). perpendicular to its length. A constant, uniform magnetic
When w is increased (2010) field exists in space in a direction perpendicular to the rod
(a) the bulb glows dimmer as well as its velocity. Select the correct statement(s) from
(b) the bulb glows brighter the following (1998S - 2 Marks)
(c) total impedance of the circuit is unchanged (a) The entire rod is at the same electric potential.
(d) total impedance of the circuit increases (b) There is an electric field in the rod.
(c) The electric potential is highest at the centre of the rod
and decreases towards its ends.
1. L, C and R represent the physical quantities, inductance, (d) The electric potential is lowest at the centre of the rod,
capacitance and resistance respectively. The combination(s) and increases towards its ends
which have the dimensions of frequency are 7. A series R– C circuit is connected to AC voltage source.
(1984- 2 Marks) Consider two cases; (A) when C is without a dielectric
(a) 1/RC (b) R/L medium and (B) when C is filled with dielectric of constant 4.
(c) 1/ LC (d) C/L The current IR through the resistor and voltage VC across
the capacitor are compared in the two cases. Which of the
2. A conducting square loop of side L and resistance R moves
following is/are true? (2011)
in its plane with a uniform velocity v perpendicular to one of
its sides. A magnetic induction B, constant in time and space, (a) I RA > I RB (b) I RA < I RB
pointing perpendicular and into the plane of the loop exists
everywhere. (1989 - 2 Marks) (c) VCA > VCB (d) VCA < VCB
P-406 Topic-wise Solved Papers - PHYSICS
8. In the given circuit, the AC source has w = 100 rad/s. 4. A square metal wire loop of side 10 cms and resistance
Considering the inductor and capacitor to be ideal, the correct 1 ohm is moved with a constant velocity v0 in a uniform
choice(s) is (are) (2012) magnetic field of induction B = 2 webers/m2 as shown in the
100 mF figure. The magnetic field lines are perpendicular to the
100 W plane of the loop (directed into the paper). The loop is
connected to a network of resistors each of value 3 ohms.
The resistances of the lead wires OS and PQ are negligible.
0.5 H 50 W What should be the speed of the loop so as to have a
steady current of 1 milliampere in the loop ? Give the
direction of current in the loop. (1983 - 6 Marks)

20 V

(a) The current through the circuit, I is 0.3 A.


(b) The current through the circuit, I is
(c) The voltage across 100 W resistor =
(d) The voltage across 50 W resistor = 10 V
5. Space is divided by the line AD into two regions. Region I
9. A current carrying infinitely long wire is kept along the
is field free and the Region II has a uniform magnetic field B
diameter of a circular wire loop, without touching it, the
directed into the plane of the paper. ACD is a semicircular
correct statement(s) is(are) (2012) conducting loop of radius r with centre at O, the plane of
(a) The emf induced in the loop is zero if the current is the loop being in the plane of the paper. The loop is now
constant. made to rotate with a constant angular velocity w about an
(b) The emf induced in the loop is finite if the current is axis passing through O and the perpendicular to the plane
constant. of the paper. The effective resistance of the loop is R.
(c) The emf induced in the loop is zero if the current (1985 - 6 Marks)
decreases at a steady rate.
(d) The emf induced in the loop is infinite if the current
decreases at a steady rate.
SUBJECTIVE PROBLEMS :
1. A current from A to B is increasing in magnitude. What is
the direction of induced current, if any, in the loop as shown
in the figure? (1979) (i) Obtain an expression for the magnitude of the induced
current in the loop.
(ii) Show the direction of the current when the loop is
entering into the Region II.
(iii) Plot a graph between the induced e.m.f and the time of
rotation for two periods of rotation.
6. Two long parallel horizontal rails, a distance d apart and
2. The two rails of a railway track, insulated from each other and each having a resistance l per unit length, are joined at
the ground, are connected to a milli voltmeter. What is the one end by a resistance R. A perfectly conducting rod MN
reading of the milli voltmeter when a train travels at a speed of of mass m is free to slide along the rails without friction
180 km/hour along the track, given that the vertical component (see figure). There is a uniform magnetic field of induction B
of earth’s magnetic field is 0.2 × 10–4 weber/m2 & the rails are normal to the plane of the paper and directed into the paper.
separated by 1 meter? (1981- 4 Marks) A variable force F is applied to the rod MN such that, as the
3. Three identical closed coils A, B and C are placed with their rod moves, a constant current flows through R.
planes parallel to one another. Coils A and C carry equal (1988 - 6 Marks)
currents as shown in Fig. Coils B and C are fixed in position
and coil A is moved towards B with uniform motion. Is there
any induced current in B ? If no, give reasons. If yes mark
the direction of the induced current in the diagram.
(1982 - 2 Marks)

(i) Find the velocity of the rod and the applied force F as
function of the distance x of the rod from R.
(ii) What fraction of the work done per second by F is
converted into heat ?
ELECTROMAGNETIC INDUCTION & ALTERNATING CURRENT P-407
7. A circuit containing a two position switch S is shown in fig. 10. A metal rod OA of mass ‘m’ and length ‘r’ is kept rotating
(1991 - 4 + 4 Marks) with a constant angular speed w in a vertical plane about a
horizontal axis at the end O. The free end A is arranged to
slide without friction along a fixed conducting circular ring
in the same plane as that of rotation. A uniform and constant
r
magnetic induction B is applied perpendicular and into the
plane of rotation as shown in the figure below. An inductor
L and an external resistance R are connected through a switch
S between the point O and a point C on the ring to form an
electrical circuit. Neglect the resistance of the ring and the
rod. Initially, the switch is open. (1995 - 10 Marks)

(a) The switch S is in position ‘1’. Find the potential


difference VA – VB and the rate of production of joule
heat in R1.
(b) If now the switch S is put in position 2 at t = 0 find
(i) steady current in R4 and
(ii) the time when current in R4 is half the steady value. (a) What is the induced emf across the terminals of the
Also calculate the energy stored in the inductor L at that switch?
time (b) The switch S is closed at time t = 0.
8. A rectangular frame ABCD, made of a uniform metal wire, (i) Obtain an expression for the current as a function
has a straight connection between E and F made of the of time.
same wire, as shown in Fig. AEFD is a square of side 1m, (ii) In the steady state, obtain the time dependence
and EB = FC = 0.5m. The entire circuit is placed in steadily of the torque required to maintain the constant
angular speed, given that the rod OA was along
increasing, uniform magnetic field directed into the plane of
the positive X-axis at t = 0.
the paper and normal to it. The rate of change of the
11. A solenoid has an inductance of 10 henry and a resistance
magnetic field is 1T/s. The resistance per unit length of the of 2 ohm. It is connected to a 10 volt battery. How long will
wire is 1W / m. Find the magnitudes and directions of the it take for the magnetic energy to reach 1/4 of its maximum
currents in the segments AE, BE and EF. value? (1996 - 3 Marks)
(1993-5 Marks) ®
12. An infinitesimally small bar magnet of dipole moment M is
pointing and moving with the speed v in the x̂ - direction.
A small closed circular conducting loop of radius a and
negligible self-inductance lies in the y-z plane with its center
at x = 0, and its axis coinciding with the x-axis. Find the force
opposing the motion of the magnet, if the resistance of the
loop is R. Assume that the distance x of the magnet from the
center of the loop is much greater than a. (1997C - 5 Marks)
9. Two parallel vertical metallic rails AB and CD are separated 13. A pair of parallel horizontal conducting rails of negligible
resistance shorted at one end is fixed on a table. The distance
by 1 m. They are connected at two ends by resistances R1
between the rails is L. A conducting massless rod of
and R2 as shown in Figure. A horizontal metallic bar L of
resistance R can slide on the rails frictionlessly. The rod is
mass 0.2 kg slides without friction vertically down the rails tied to a massless string which passes over a pulley fixed to
under the action of gravity. There is a uniform horizontal the edge of the table. A mass m, tied to the other end of the
magnetic field of 0.6 Tesla perpendicular to the plane of the string hangs vertically. A constant magnetic field B exists
rails. It is observed that when the terminal velocity is perpendicular to the table. If the system is released from
attained, the powers dissipated in R1 and R2 are 0.76 Watt rest, calculate. (1997 - 5 Marks)
and 1.2 watt respectively. Find the terminal velocity of the
bar L and the values of R1 and R2. (1994 - 6 Marks)
R1
A C
L

R2 (i) the terminal velocity achieved by the rod, and


(ii) the acceleration of the mass at the instant when the
B D velocity of the rod is half the terminal velocity.
P-408 Topic-wise Solved Papers - PHYSICS
14. An inductor of inductance 2.0 mH is connected across a
charged capacitor of capacitance 5.0 mF, and the resulting
LC circuit is set oscillating at its natural frequency. Let Q L
E
denote the instantaneous charge on the capacitor, and I the R1
current in the circuit. It is found that the maximum value of Q S R2
is 200 mC. (1998 - 8 Marks)
(a) When Q = 100 mC, what is the value of |d /dt|?
(b) When Q = 200 mC, what is the value of ? 18. A rectangular loop PQRS made from a uniform wire has
(c) Find the maximum value of . length a, width b and mass m. It is free to rotate about the
(d) When is equal to one half its maximum value, what is arm PQ, which remains hinged along a horizontal line taken
the value of |Q|? as the y-axis (see figure). Take the vertically upward
15. A magnetic field B = B0 (y/a) k̂ is into the paper in the direction as the z-axis. A uniform magnetic field
+z direction. B0 and a are positive constants. A square loop ur
B = (3iˆ + 4kˆ) B0 exists in the region. The loop is held in the
EFGH of side a, mass m and resistance R, in x – y plane,
starts falling under the influence of gravity see figure) Note x-y plane and a current I is passed through it. The loop is
the directions of x and y axes in figure. (1999 - 10 Marks)) now released and is found to stay in the horizontal position
in equilibrium. (2002 - 5 Marks)

Find
(a) the induced current in the loop and indicate its direction.
(b) the total Lorentz force acting on the loop and indicate
its direction, and
(c) an expression for the speed of the loop, v(t) and its
terminal value.
16. A thermocole vessel contains 0.5 kg of distilled water at
30°C. A metal coil of area 5 × 10-3m2, number of turns 100,
mass 0.06 kg and resistance 1.6 W is lying horizontally at the
bottom of the vessel. A uniform, time varying magnetic field (a) What is the direction of the current I in PQ?
is set up to pass vertically through the coil at time t = 0. The (b) Find the magnetic force on the arm RS.
field is first increased from zero to 0.8 T at a constant rate (c) Find the expression for I in terms of B0, a, b and m.
between 0 and 0.2 s and then decreased to 0 and the same 19. A metal bar AB can slide on two parallel thick metallic rails
rate between 0.2 and 0.4s. This cycle is repeated 12000 times. separated by a distance l . A resistance R and an
Make sketches of the current through the coil and the power inductance L are connected to the rails as shown in the
dissipated in the coil as functions of time for the first two figure. A long straight wire carrying a constant current I0 is
cycles. Clearly indicate the magnitudes of the quantities on placed in the plane of the rails and perpendicular to them as
the axes. Assume that no heat is lost to the vessel or the shown. The bar AB is held at rest at a distance x0 from the
surroundings. Determine the final temperature of the water long wire. At t = 0, it is made to slide on the rails away from
under thermal equilibrium. Specific heat of metal the wire. Answer the following questions. (2002 - 5 Marks)
= 500 Jkg –1 K –1 and the specific heat of water
= 4200 Jkg–1K–1. Neglect the inductance of the coil.
(2000 - 10 Marks)
17. An inductor of inductance L = 400 mH and resistors of
resistances R1 = 2W and R2 = 2 W are connected to a battery
of emf E = 12 V as shown in the figure. The internal resistance
of the battery is negligible. The switch S is closed at time
t = 0. What is the potential drop across L as a function of
time? After the steady state is reached, the switch is opened.
What is the direction and the magnitude of current through
R1 as a function of time? (2001-5 Marks)
ELECTROMAGNETIC INDUCTION & ALTERNATING CURRENT P-409

di df
(a) Find a relation among i, and , where i is the
dt dt
current in the circuit and f is the flux of the magnetic
field due to the long wire through the circuit.
(b) It is observed that at time t = T, the metal bar AB is at a
distance of 2x0 from the long wire and the resistance R
carries a current i1. Obtain an expression for the net 22. In the figure both cells A and B are of equal emf. Find R for
charge that has flown through resistance R from t = 0 which potential difference across battery A will be zero,
to t = T. long time after the switch is closed. Internal resistance of
(c) The bar is suddenly stopped at time T. The current batteries A and B are r1 and r2 respectively (r1 > r2).
i1 (2004 - 4 Marks)
through resistance R is found to be at time 2T. Find
4
L
the value of in terms of the other given quantities.
R
20. A square loop of side ‘a’ with a capacitor of capacitance C
is located between two current carrying long parallel wires
as shown. The value of I in the wires is given as I = I0 sinwt.
(2003 - 4 Marks)
23. A long solenoid of radius a and number of turns per unit
length n is enclosed by cylindrical shell of radius R. thickness
d (d << R ) and length L. A variable current i = i0 sinwt flows
through the coil. If the resistivity of the material of cylindrical
shell is r, find the induced current in the shell.
(2005 - 4 Marks)

(a) Calculate maximum current in the square loop.


(b) Draw a graph between charges on the upper plate of
the capacitor vs time.
21. In a series L–R circuit (L = 35 mH and R = 11 W), a variable
emf source (V = V0 sin wt) of Vrms = 220 V and frequency 50
Hz is applied. Find the current amplitude in the circuit and
phase of current with respect to voltage. Draw current-time
graph on given graph (p = 22/7). (2004 - 4 Marks)

MATCH THE FOLLOWING :


MUTLIPLE CHOICE QUESTIONS WITH ONE CORRECT
Each question contains statements given in two columns, which have to be matched. The statements in Column-I are labelled A,
B, C and D, while the statements in Column-II are labelled p, q, r, s and t. Any given statement in Column-I can have correct
matching with ONE OR MORE statement(s) in Column-II. The appropriate bubbles corresponding to the answers to these
questions have to be darkened as illustrated in the following example :
If the correct matches are A-p, s and t; B-q and r; C-p and q; and D-s then the correct darkening of bubbles will look like the given.
p q r s t
A p q r s t
B p q r s t
C p q r s t
D p q r s t

1. You are given many resistances, capacitors and inductors. These are connected to a variable DC voltage source (the first two
circuits) or an AC voltage source of 50 Hz frequency (the next three circuits) in different ways as shown in Column II. When a
current I (steady state for DC or rms for AC) flows through the circuit, the corresponding voltage V1and V2 , (indicated in circuits)
are related as shown in Column I. Match the two (2010)
P-410 Topic-wise Solved Papers - PHYSICS
Column I Column II
V1 V2

6mH 3m F
(A) I ¹ 0, V1 is proportional to I (p)

V1 V2

6mH 2W
(B) I ¹ 0, V2 > V1 (q)

V1 V2

6mH 2W
(D) V1 = 0, V2 = V (r)

V
V1 V2

6mH 3m F
(D) I ¹ 0, V2 is proportional to I (s)

V
V1 V2

1kW 3m F
(t)

COMPREHENSION BASED Q UESTIONS : 1. At the start, the capacitor was uncharged. When switch S1
PASSAGE 1 is closed and S2 is kept open, the time constant of this
In the given circuit the capacitor (C) may be charged through circuit is t. Which of the following is correct
resistance R by a battery V by closing switch S1. Also when S1 is (2006 – 5M, –2)
opened and S2 is closed the capacitor is connected in series with
inductor (L). CV
(a) after time interval t, charge on the capacitor is
2
(b) after time interval 2t, charge on the capacitor of CV
(1 – e–2)
(c) the work done by the voltage source will be half of the
heat dissipated when the capacitor is fully charged
(d) after time interval 2t, charge on the capacitor is
CV (1 – e–1)
ELECTROMAGNETIC INDUCTION & ALTERNATING CURRENT P-411
2. When the capacitor gets charged completely, S1 is opened 6. The magnitude of the induced electric field in the orbit at
and S2 is closed. Then, (2006 – 5M, –2) any instant of time during the time interval of the magnetic
(a) at t = 0, energy stored in the circuit is purely in the form field change is
of magnetic energy
BR BR
(b) at any time t > 0, current in the circuit is in the same (a) (b)
direction 4 2
(c) at t > 0, there is no exchange of energy between the (c) BR (d) 2BR
inductor and capacitor 7. The change in the magnetic dipole moment associated with
(d) at any time t > 0, instantaneous current in the circuit the orbit, at the end of the time interval of the magnetic field
change, is
C
may be V
L BQR 2
(a) –gBQR2 (b) -g
3. Given that the total charge stored in the LC circuit is Q0, for 2
t ³ 0, the charge on the capacitor is (2006 – 5M, –2)
BQR 2
æp t ö æp t ö (c) g (d) g BQR 2
(a) Q = Q0 cos ç + ÷ (b) Q = Q0 cos ç - ÷ 2
è2 LC ø è2 LC ø

d 2Q 1 d 2Q ASSERTION & REASON TYPE QUESTIONS :


(c) Q = - LC (d) Q = -
dt 2 LC dt 2 1. Statement-1 : A vertical iron rod has coil of wire wound over
PASSAGE 2 it at the bottom end. An alternating current flows in the coil.
A thermal power plant produces electric power of 600 kW at 4000 The rod goes through a conducting ring as shown in the
V, which is to be transported to a place 20 km away from the power figure. The ring can float at a certain height above the coil.
plant for consumers' usage. It can be transported either directly (2007)
with a cable of large current carrying capacity or by using a
combination of step-up and step-down transformers at the two
ends. The drawback of the direct transmission is the large energy
dissipation. In the method using transformers, the dissipation is
much smaller. In this method , a step-up transformer is used at the
plant side so that the current is reduced to a smaller value. At the
consumers' end, a step-down transformer is used to supply power
to the consumers at the specified lower voltage. It is reasonable
to assume that the power cable is purely resistive and the Statement-2 : In the above situation, a current is induced in
transformers are ideal with power factor unity. All the currents the ring which interacts with the horizontal component of
and voltages mentioned are rms values. (JEE Adv. 2013) the magnetic field to produce an average force in the upward
4. If the direct transmission method with a cable of resistance direction.
0.4 W km–1 is used, the power dissipation| (in %) during (a) Statement-1 is True, Statement-2 is True; Statement-2
transmission is is a correct explanation for Statement-1
(a) 20 (b) 30 (b) Statement-1 is True, Statement-2 is True; Statement-2
(c) 40 (d) 50 is NOT a correct explanation for Statement-1
5. In the method using the transformers, assume that the ratio (c) Statement-1 is True, Statement-2 is False
of the number of turns in the primary to that in the secondary (d) Statement-1 is False, Statement-2 is True.
in the step-up transformer is 1 : 10. If the power to the
consumers has to be supplied at 200 V, the ratio of the number INTEGER ANSWER QUESTIONS :
of turns in the primary to that in the secondary in the step- 1. A series R-C combination is connected to an AC voltage of
down transformer is angular frequency w = 500 radian/s. If the impedance of the
(a) 200 : 1 (b) 150 : 1
(c) 100 : 1 (d) 50 : 1 R-C circuit is R 1.25 , the time constant (in millisecond) of
PASSAGE 3 the circuit is z (2011)
A point charge Q is moving in a circular orbit of radius R in the x- 2. A circular wire loop of radius R is 45°
y plane with an angular velocity w. This can be considered as placed in the x-y plane centered at the
Qw origin O. A square loop of side a
equivalent to a loop carrying a steady current . A uniform a(a<<R) having two turns is placed 3R
2p with its centre at
magnetic field along the positive z-axis is now switched on, which R y
increases at a constant rate from 0 to B in one second. Assume z= along the axis of the O
that the radius of the orbit remains constant. The application of circular wire loop, as shown in x
the magnetic field induces an emf in the orbit. The induced emf is figure. The plane of the square loop makes an angle of 45° with
defined as the work done by an induced electric field in moving a respect to the z-axis. If the mutual inductance between the
unit positive charge around a closed loop. It is known that, for an
orbiting charge, the magnetic dipole moment is proportional to m
the angular momentum with a proportionality constant g. loops is given by then the value of p is (2012)
(JEE Adv. 2013)
P-412 Topic-wise Solved Papers - PHYSICS

1. The power factor of an AC circuit having resistance (R) and 10. In an LCR series a.c. circuit, the voltage across each of the
inductance (L) connected in series and an angular velocity components, L, C and R is 50V. The voltage across the LC
w is combination will be [2004]
(a) R/ w L (b) R/(R2 + w 2L2)1/2
(a) 100 V (b)
50 2 V
(c) w L/R (d) R/(R2 – w 2L2)1/2
2. A conducting square loop of side L and resistance R moves (c) 50 V (d) 0 V (zero)
in its plane with a uniform velocity v perpendicular to one of 11. A coil having n turns and resistance RW is connected with
its sides. A magnetic induction B constant in time and space, a galvanometer of resistance 4RW. This combination is
pointing perpendicular and into the plane at the loop exists moved in time t seconds from a magnetic field W1 weber to
everywhere with half the loop outside the field, as shown in W2 weber. The induced current in the circuit is [2004]
figure. The induced emf is [2002]
(W2 - W1 ) n (W 2 - W1 )
(a) - (b) -
Rnt 5 Rt
v (W2 - W1 ) n(W2 - W1 )
(c) - (d) -
5 Rnt Rt
12. In a uniform magnetic field of induction B a wire in the form
(a) zero (b) RvB of a semicircle of radius r rotates about the diameter of the
(c) vBL/R (d) vBL circle with an angular frequency w. The axis of rotation is
3. The inductance between A and D is [2002] perpendicular to the field. If the total resistance of the circuit
is R, the mean power generated per period of rotation is
( B pr w )2 ( B pr 2 w ) 2
3H 3H 3H (a) (b)
2R 8R
(a) 3.66 H (b) 9 H (c) 0.66 H (d) 1 H. B pr 2 w ( B pr w 2 ) 2
4. In a transformer, number of turns in the primary coil are 140 (c) (d) [2004]
2R 8R
and that in the secondary coil are 280. If current in primary
13. In a LCR circuit capacitance is changed from C to 2 C. For
coil is 4 A, then that in the secondary coil is [2002]
the resonant frequency to remain unchanged, the
(a) 4 A (b) 2 A (c) 6 A (d) 10 A.
5. Two coils are placed close to each other. The mitual inductance should be changed from L to [2004]
inductance of the pair of coils depends upon [2003] (a) L/2 (b) 2 L (c) 4 L (d) L/4
(a) the rates at which currents are changing in the two 14. A metal conductor of length 1 m rotates vertically about one
coils of its ends at angular velocity 5 radians per second. If the
(b) relative position and orientation of the two coils horizontal component of earth’s magnetic field is 0.2×10–4T,
(c) the materials of the wires of the coils then the e.m.f. developed between the two ends of the
(d) the currents in the two coils conductor is [2004]
6. When the current changes from +2 A to -2A in 0.05 second, (a) 5 mV (b) 50 mV (c) 5 mV (d) 50mV
an e.m.f. of 8 V is induced in a coil. The coefficient of self - 15. One conducting U tube can slide inside another as shown
induction of the coil is [2003] in figure, maintaining electrical contacts between the tubes.
(a) 0.2 H (b) 0.4 H (c) 0.8 H (d) 0.1 H The magnetic field B is perpendicular to the plane of the
7. In an oscillating LC circuit the maximum charge on the figure . If each tube moves towards the other at a constant
capacitor is Q. The charge on the capacitor when the energy speed v, then the emf induced in the circuit in terms of B, l
is stored equally between the electric and magnetic field is and v where l is the width of each tube, will be [2005]
[2003]
X A X X X
Q Q Q BX
(a) (b) (c) (d) Q
2 3 2 X v X v X
8. The core of any transformer is laminated so as to [2003] X
(a) reduce the energy loss due to eddy currents X X X
(b) make it light weight X C
X X
(c) make it robust and strong
(d) increase the secondary voltage (a) – Blv (b) Blv
9. Alternating current can not be measured by D.C. ammeter (c) 2 Blv (d) zero
because [2004] 16. The self inductance of the motor of an electric fan is 10 H. In
(a) Average value of current for complete cycle is zero order to impart maximum power at 50 Hz, it should be
(b) A.C. Changes direction connected to a capacitance of [2005]
(c) A.C. can not pass through D.C. Ammeter (a) 8 mF (b) 4 mF (c) 2 mF (d) 1 mF
(d) D.C. Ammeter will get damaged.
ELECTROMAGNETIC INDUCTION & ALTERNATING CURRENT P-413
17. The phase difference between the alternating current and 26. An ideal coil of 10H is connected in series with a resistance
p of 5W and a battery of 5V. 2second after the connection is
emf is . Which of the following cannot be the constituent made, the current flowing in ampere in the circuit is [2007]
2
of the circuit? [2005] (a) (1 – e–1) (b) (1 – e) (c) e (d) e–1
(a) R, L (b) C alone (c) L alone (d) L, C 27. Two coaxial solenoids are made by winding thin insulated
18. A circuit has a resistance of 12 ohm and an impedance of 15 wire over a pipe of cross-sectional area A = 10 cm2 and
ohm. The power factor of the circuit will be [2005] length = 20 cm. If one of the solenoid has 300 turns and the
(a) 0.4 (b) 0.8 (c) 0.125 (d) 1.25 other 400 turns, their mutual inductance is [2008]
19. A coil of inductance 300 mH and resistance 2 W is connected (m0 = 4p × 10 –7 Tm A–1)
to a source of voltage 2 V. The current reaches half of its (a) 2.4p × 10–5 H (b) 4.8p × 10–4 H
steady state value in [2005] (c) 4.8p × 10 H–5 (d) 2.4p × 10–4 H
(a) 0.1 s (b) 0.05 s (c) 0.3 s (d) 0.15 s
ML2 28. E
20. Which of the following units denotes the dimension 2
,
Q L
where Q denotes the electric charge? [2006] R1
(a) Wb/m2 (b) Henry (H)
(c) H/m2 (d) Weber (Wb)
21. In a series resonant LCR circuit, the voltage across R is 100 R2
volts and R = 1 kW with C = 2mF. The resonant frequency w
is 200 rad/s. At resonance the voltage across L is [2006] S
(a) 2.5 × 10–2 V (b) 40 V
(c) 250 V (d) 4 × 10–3 V An inductor of inductance L = 400 mH and resistors of
22. In an AC generator, a coil with N turns, all of the same area resistance R1 = 2W and R2 = 2W are connected to a battery
A and total resistance R, rotates with frequency w in a
of emf 12 V as shown in the figure. The internal resistance of
magnetic field B. The maximum value of emf generated in
the coil is [2006] the battery is negligible. The switch S is closed at t = 0. The
(a) N.A.B.R.w (b) N.A.B potential drop across L as a function of time is : [2009]
(c) N.A.B.R. (d) N.A.B.w
23. The flux linked with a coil at any instant 't' is given by (a)
12 -3t
t
e V (b) 6 1 - e (
-t / 0.2
V )
2
f = 10t - 50t + 250 (c) 12e–5t V (d) 6e–5t V
The induced emf at t = 3s is [2006] 29. A rectangular loop has a sliding connector PQ of length l
(a) –190 V (b) –10 V (c) 10 V (d) 190 V and resistance R W and it is moving with a speed v as
24. An inductor (L = 100 mH), a resistor (R = 100 W) and a shown. The set-up is placed in a uniform magnetic field
battery (E = 100 V) are initially connected in series as shown
in the figure. After a long time the battery is disconnected going into the plane of the paper. The three currents I1, I2
after short circuiting the points A and B. The current in the and I are [2010]
circuit 1 ms after the short circuit is [2006]
L l
P

RW RW v RW
R
I
A B I2
I1 Q
E
(a) 1/eA (b) eA (c) 0.1 A (d) 1 A
25. In an a.c. circuit the voltage applied is E = E0 sin wt. The Blv 2 Blv
(a) I1 = - I 2 = , I=
æ pö 6R 6R
resulting current in the circuit is I = I 0 sin ç wt - ÷ . The
è 2ø Blv 2 Blv
power consumption in the circuit is given by [2007] (b) I1 = I 2 = ,I =
3R 3R
E0 I 0
(a) P = 2 E0 I0 (b) P= Blv
2 (c) I1 = I 2 = I =
R
E0 I 0
(c) P = zero (d) P= Bl n Bl n
2 (d) I1 = I 2 = , I=
6R 3R
P-414 Topic-wise Solved Papers - PHYSICS
30. In the circuit shown below, the key K is closed at t = 0. The 36. A horizontal straight wire 20 m long extending from east to
current through the battery is [2010] west falling with a speed of 5.0 m/s, at right angles to the
V K horizontal component of the earth’s magnetic field
0.30 × 10–4 Wb/m2. The instantaneous value of the e.m.f.
induced in the wire will be [2011RS]
L R1 (a) 3 mV (b) 4.5 mV
(c) 1.5 mV (d) 6.0 mV
37. A coil is suspended in a uniform magnetic field, with the
R2 plane of the coil parallel to the magnetic lines of force. When
a current is passed through the coil it starts oscillating; It is
VR1R2 very difficult to stop. But if an aluminium plate is placed
V near to the coil, it stops. This is due to : [2012]
(a) at t = 0 and R at t = ¥
R12 + R22 2 (a) developement of air current when the plate is placed
(b) induction of electrical charge on the plate
V V ( R1 + R2 ) (c) shielding of magnetic lines of force as aluminium is a
(b) R2 at t = 0 and R1 R2 at t = ¥ paramagnetic material.
(d) electromagnetic induction in the aluminium plate giving
V VR1R2 rise to electromagnetic damping.
(c) R2 at t = 0 and at t = ¥ 38. A metallic rod of length ‘l’ is tied to a string of length 2l and
R12 + R22
made to rotate with angular speed w on a horizontal table
V ( R1 + R2 ) V with one end of the string fixed. If there is a vertical magnetic
(d) R1 R2 at t = 0 and R2 at t = ¥ field ‘B’ in the region, the e.m.f. induced across the ends of
the rod is [JEE Main 2013]
31. In a series LCR circuit R = 200W and the voltage and the
frequency of the main supply is 220V and 50 Hz respectively. 2 Bwl2
On taking out the capacitance from the circuit the current (a)
2
lags behind the voltage by 30°. On taking out the inductor 3Bwl 2
from the circuit the current leads the voltage by 30°. The (b)
power dissipated in the LCR circuit is [2010] 2
(a) 305 W (b) 210 W 4 Bwl2
(c) Zero W (d) 242 W (c)
2
32. A boat is moving due east in a region where the earth's
magnetic field is 5.0 × 10–5 NA–1 m–1 due north and horizontal.
5Bwl2
The boat carries a vertical aerial 2 m long. If the speed of the (d)
boat is 1.50 ms–1, the magnitude of the induced emf in the 2
39. A circular loop of radius 0.3 cm lies parallel to amuch bigger
wire of aerial is: [2011]
circular loop of radius 20 cm. The centre of the small loop is
(a) 0.75 mV (b) 0.50 mV
on the axis of the bigger loop. The distance between their
(c) 0.15 mV (d) 1mV
33. A fully charged capacitor C with initial charge q0 is centres is 15 cm. If a current of 2.0 A flows through the
connected to a coil of self inductance L at t = 0. The time at smaller loop, then the flux linked with bigger loop is
which the energy is stored equally between the electric and [JEE Main 2013]
the magnetic fields is: [2011] (a) 9.1 × 10–11 weber (b) 6 × 10–11 weber
(c) 3.3 × 10–11 weber (d) 6.6 × 10–9 weber
p 40. In an LCR circuit as shown below both switches are open
(a) LC (b) 2p LC
4 initially. Now switch S1 is closed, S2 kept open. (q is charge
(c) LC (d) p LC on the capacitor and t = RC is Capacitive time constant).
34. A resistor ‘R’ and 2µF capacitor in series is connected Which of the following statement is correct ?
through a switch to 200 V direct supply. Across the capacitor [JEE Main 2013]
is a neon bulb that lights up at 120 V. Calculate the value of V
R to make the bulb light up 5 s after the switch has been
closed. (log10 2.5 = 0.4) [2011]
(a) 1.7 × 105 W (b) 2.7 × 106 W R
(c) 3.3 × 107 W (d) 1.3 × 104 W S1
35. Combination of two identical capacitors, a resistor R and a
dc voltage source of voltage 6V is used in an experiment on
a (C-R) circuit. It is found that for a parallel combination of C S2
the capacitor the time in which the voltage of the fully
charged combination reduces to half its original voltage is L
10 second. For series combination the time for needed for (a) Work done by the battery is half of the energy
reducing the voltage of the fully charged series combination dissipated in the resistor
by half is [2011RS] (b) At t = t, q = CV/2
(a) 10 second (b) 5 second (c) At t = 2t, q = CV (1 – e–2)
(c) 2.5 second (d) 20 second (d) At t = 2 t, q = CV (1 – e–1)
ELECTROMAGNETIC INDUCTION & ALTERNATING CURRENT P-415

Solutions & Explanations


Section-A : JEE Advanced/ IIT-JEE
A 1. 0.3 × 10–4 sec, 10 A 2. Left to right 3. 15 V
B 1. T 2. F 3. T
C 1. (b) 2. (d) 3. (c) 4. (d) 5. (b) 6. (b) 7. (d)
8. (a) 9. (d) 10. (b) 11. (a) 12. (c) 13. (a) 14. (b)
15. (d) 16. (b)
D 1. (a, b, c) 2. (d) 3. (a, c, d) 4. (b) 5. (a, b, c, d) 6. (b) 7. (a, c)
8. (a, c) 9. (a)
E 1. Clockwise 2. 1 mV 3. Yes, opposite direction of A 4. 0.02 m/s, clockwise direction
-1
1 Br 2 w æ R + 2lx ö 2lmI 2 é 2 lmI ( R + 2lx ) ù
5. (i) (ii) anticlockwise 6. (i) V = ç ÷ I , F = BId + ( R + 2lx) , (ii) ê1 + ú
2 R è Bd ø ( Bd ) 2 ë B3 d 3 û

7 6 1
7. (a) –5V, 24.5 W (b) (i) 0.6 amp. (ii) 1.386 × 10–3 sec., 4.5 × 10–4 J 8. amp , amp , amp
22 22 22

Br 2 w 2é - ç ÷t ù
æRö
B 2 r 4 w mgr
9. 1 m/s, 0.47 W, 0.3 W 10. (a) (b) I = Bwr êë1 - e è L ø úû , t = + cos wt 11. 3.466 sec
2 2R 4R 2

21 m02 M 2 a 4v mgR g
12. F= 13. (i) (ii) 14. (a) 104 A/s (b) 0 (c) 2.0 A (d) 1.732 × 10–4 C
4 Rx8 B 2 L2 2

B0av(t ) B 2 a 2 v (t ) é - B02 a 2t ù
15. (a) , anticlockwise (b) - 0 , upward (c) mgR êë1 - e mR úû ; mgr
R R
B02 a 2 B02 a 2

16. 35.6ºC 17. 12e–5tV, 3e–10tA, clockwise

mg df di 1 é m 0 I 0l ù L T
18. (a) P to Q (b) IbB0 (3kˆ - 4iˆ) (c) I = 19. (a) = iR + L (b) ê loge 2ú - i1 (c)
6aB0 dt dt R ë 2 p û R 2log e 2

m 0 aI 0 w 2C ln(2) p 4 µ0na 2 Ldi0 w cos w t


20. (a) 21. 20A, 22. (r1 - r2 ) 23. I=
p 4 3 2rR
F 1. A-r, s, t; B-q, r, s, t; C-p, q; D-q, r, s, t
G 1. (b) 2. (d) 3. (c) 4. (a) 5. (b) 6. (b) 7. (b)
H 1. (a)
I 1. 4 2. 7

Section-B : JEE Main/ AIEEE


1. (b) 2. (d) 3. (d) 4. (b) 5. (b) 6. (d) 7. (c)
8. (a) 9. (a) 10. (d) 11. (b) 12. (b) 13. (a) 14. (b)
15. (c) 16. (d) 17. (a) 18. (b) 19. (a) 20. (b) 21. (c)
22. (d) 23. (b) 24. (a) 25. (c) 26. (a) 27. (d) 28. (c)
29. (b) 30. (c) 31. (d) 32. (c) 33. (a) 34. (b) 35. (c)
36. (a) 37. (d) 38. (d) 39. (a) 40. (c)
P-416 Topic-wise Solved Papers - PHYSICS
2. (d) Induced emf produced across MNQ will be same as the
FILL IN THE BLANKS : induced emf produced in straight wire MQ.
1. The coil is broken into two identical coils. \ e = Bvl = Bv × 2R with Q at higher potential.
L/2´ L/2 L 3. (c) When the current in the loop A increases, the magnetic
Leq = = = 0.45 × 10–4 H, lines of force in loop B also increases as loop A is
L/2+ L/2 4 placed near loop B. This induces an emf in B in such a
R/2´ R/2 R direction that current flows opposite in B (as compared
Req = = = 1.5 W to A).
R/2+ R/2 4
Since currents are in opposite direction, the loop B is
Leq 0.45 ´ 10 -4 repelled by loop A.
Time constant = = = 0.3 × 10–4s
Re q 1.5
E 15
Steady current I = = = 10 A.
Req 1.5
2. NOTE : As the source is switched off, the current
decreases to zero. The induced current opposes the cause B A
as per Lenz's law. Therefore, the induced current will direct 4. (d) KEY CONCEPT : Using I = I0 (1 –e–t/t)
from left to right.
V L
But I 0 = and t =
3. R R
V 12 -3
VB + e – 15 + I × 1 = VA \ I = (1 - e - Rt / L ) = é1 - e -6t / 8.4´10 ù
R 6 ëê úû
Þ VB – VA = 15 – e – I
= 1 (given)
df é -dI ù \ t = 0.97 ×10 – 3 s » 1ms
Here, I = 5A, e = = Lê
dt ë dt úû ur uur d f d ur ur d dB
= – 5 × 10–3 × 103 = – 5 V, VB – VA = 15 V 5. (b) Ñ
ò E.d l = dt = dt ( B. A) = dt ( BA cos 0°) = A dt
TRUE / FALSE : dB
Þ E (2pr ) = pa 2 for r ³ a
1. True. A copper wire consists of billions and billions of free dt
electrons. When the wire is at rest, the average velocity of
each electron is zero. But when the wire is in motion, the a 2 dB 1
Þ E= ÞEµ
electrons have a net velocity in the direction of motion. 2r dt r
NOTE : A charged particle moving in a magnetic field 6. (b) KEY CONCEPT : The magnetic field at the centre of
r r ur the coil
experiences a force given by F = q( n ´ B). B(t) = µ0nI1.
Here also each electron experiences a force and therefore, As the current increases, B will also increase with time
electrons will move towards one end creating an emf till it reaches a maximum value (when the current
between the two ends of a straight copper wire. becomes steady).
2. NOTE : For induced emf to develop in a coil, the magnetic The induced emf in the ring
flux through it must change.
But in this case the number of magnetic lines of force through df d ur ur d
e=- = - ( B. A) = - A ( µ0 nI1 )
the coil is not changing. Therefore the statement is false. dt dt dt
3. NOTE : When conduction rod AB moves parallel to x-axis \ The induced current in the ring
in a uniform magnetic field pointing in the positive | e | µ0 nA dI1
z-direction, then according to Fleming’s left hand rule, the I 2 (t ) = =
R R dt
electrons will experience a force towards B. Hence, the end
dI1
A will become positive. [NOTE : decreases with time and hence I2 also
dt
B decreases with time.]
Fm – –
Where I1 = Imax (1 – e –t/t)
e v The relevant graphs are

+ +
A
MCQ's WITH ONE CORRECT ANSWER :
1. (b) The current induced will be
|e| 1 df dq
i= Þ i= But i =
R R dt dt
dq 1 d f 1 BA
Þ = Þ ò dq = ò d f Þ q =
dt R dt R R
ELECTROMAGNETIC INDUCTION & ALTERNATING CURRENT P-417
7. (d) NOTE : Electric field will be induced, as ABCD moves, 12. (c) KEY CONCEPT : Initially, fB increases as magnet
in both AD and BC. The metallic square loop moves in approaches the solenoid
its own plane with velocity v. A uniform magnetic field \ e = – ve and increasing in magnitude. When
is imposed perpendicular to the plane of the square magnet is moving inside the solenoid, increase in fB
loop. AD and BC are perpendicular to the velocity as slow down and finally fB starts decreasing
well as perpendicular to applied. \ emf is positive and increasing.
Only graph (c) shows these characteristic.
13. KEY CONCEPT : For a current to induce in cylindrical
conducting rod,
(a) the conducting rod should cut magnetic lines of force
which will happen only when the conducting rod is
moving. Since conducting rod is at rest, no current will
be induced.
(b) the magnitude/direction of the magnetic field changes.
A changing magnetic field will create an electric field
8. (a) Clearly the flux linkage is maximum in case (a) due to which can apply force on the free electrons of the
the spatial arrangement of the two loops. conducting rod and a current will get induced.
But since the magnetic field is constant, no current will
be induced.

9. (d) When switch S is closed, a magnetic field is set-up in


the space around P. The field lines threading Q
increases in the direction from right to left. According
to Lenz's law, IQ1 will flow so as to oppose the cause
and flow in anticlockwise direction as seen by E.
Reverse is the case when S is opened. IQ2 will be
clockwise. 14. (b) KEY CONCEPT :
10. (b) KEY CONCEPT : Time constant of R – C circuit is t = Req Ceq
2
(i) t1 = (2 + 1) (2 + 4) = 18 µs
E 2 pr 2 æ d f ö pr 2 é d ù
P= = çè ÷ø = ( NBA)2 ú æ 2 ´1 öæ 2 ´ 4 ö 8
R rl dt rl êë dt û (ii) t2 = ç ÷ç ÷ = µs
è 2 + 1 øè 2 + 4 ø 9
2
pr 2 2 2 æ dB ö N 2r 2 æ 2 ´1 ö
= N A ç ÷ Þ Pµ
rl è dt ø l (iii) t3 = ç ÷ ´ (4 + 2) = 4µs
è 2 +1 ø
(4 N )2 ( r / 2) 2 15. (d) The magnetic field is increasing in the downward
N 2r 2
Case 1 : P1 µ , Case 2 : P2 µ direction. Therefore, according to Lenz’s law the current
l 4l I1 will flow in the direction ab and I2 in the direction dc.
NOTE : When we decrease the radius of the wire, its
length increases but volume remains the same] Vrms
16. (b) I rms =
2
P1 1 æ 1 ö
Þ = R2 + ç
P2 1 è wC ÷ø
\ Power remains the same.
C R
11. (a) NOTE : Since current leads emf (as seen from the
graph), therefore, this is an R – C circuit.
XC - XL
Irms
tan f = ~
R
Here f = 45° As w increases, Irms increases. Therefore the bulb glows
\ XC = R [XL = 0 as there is no inductor] brighter.
1
= R Þ RCw = 1
wC 1. (a,b,c)
1 -1 1
\ RC = s , R/L and 1/ LC have the dimensions of frequency..
100 RC
P-418 Topic-wise Solved Papers - PHYSICS
2. (d) NOTE : Since the rate of change of magnetic flux is The flux through small square loop is directly
zero, hence there will be no net induced emf and hence proportional to the current passing through big square
no current flowing in the loop. loop.
\ f2 µ I1 Þ f = MI1where M = Mutual Conductance
3. (a, c, d)
2 2 µ0 I1 2
´l
f 2 2 µ0
Rate of change of current = di1 = m( say) \ M2 = 2 = pL = ´ l2
dt I1 I1 pL
di1 l2
Induced emf V1 = - L1 = - 8 ´ 10 -3 ´ m Þ M µ .
dt L
V2 1 5. (a, b, c, d)
\ =
V1 4 f weber
(a) L= or henry =
Power P = V1 i1 = 8 × 10 – 3 × m ×i1 i ampere
di2 æ di ö
Rate of change of current = = m (given) (b) e = - Lç ÷
dt è dt ø
di2 e volt - second
Induced emf, V2 = - L2 = – 2 × 10 – 3 × m \ L= - or henry =
dt (di / dt ) ampere
Power P = V2 i2 = 2 × 10 –3 × m × i2 1 2
Since Power is equal (c) U= Li
2
\ 8 × 10 – 3 × mi1 = 2 ×10 – 3 mi2
2U joule
i1 1 \ L= or henry =
Þ = ...(i) i 2 (ampere)2
i2 4
1 2 2
1 (d) U = Li = i Rt
1 2 -3 2 2
Energy W1 = L1i1 = ´ 8 ´ 10 ´ i1 \ L = R.t or henry = ohm-second.
2 2
6. (b) A B
W2 10 -3 ´ i22 1
\ = = ´ 4 ´ 4 = 4 [from (i)]
-3
W1 4 ´ 10 ´ i1 4 2

4. (b) KEY CONCEPT : The magnetic field due to a current


flowing in a wire of finite length is given by l
v
µ I
B = 0 (sin a + sin b)
4 pR
Applying the above formula for AB for finding the field
B
at O, we get
A motional emf, e = Blv is induced in the rod. Or we can say
µ0 I1 a potential difference is induced between the two ends of
B= (sin 45° + sin 45°) = µ0 I1
4p( L / 2) the rod AB, with A at higher potential and B at lower potential.
2pL
Due to this potential difference, there is an electric field in
acting perpendicular to the plane of paper upwards
the rod.
\ The total magnetic field due to current flowing
through ABCD is 2
æ 1 ö
7. (a,c) We know that Z = R2 + ç
è WC ÷ø
The capacitance in case B is four times the capacitance in
case A
\ Impedance in case B is less then that of case A (ZB < ZA)
V
Now I =
Z
4µ0 I1 2 2 µ0 I1 \I RA < I RB . option (a) is correct.
B = 4 B1 = =
2 pL pL \VRA < VRB .
The total flux passing through the square EFGH
Þ VCA > VCB
2 2 µ0 I1 2 [Q If V is the applied potential difference access
f2 = B ´ l 2 = ´ l ... (i)
pL
[Q l > L and therefore, B can be assumed series R-C circuit then V = VR2 + VC2 ]
constant for l2] \ (c) is the correct option.
ELECTROMAGNETIC INDUCTION & ALTERNATING CURRENT P-419
8. (a,c) Impedance across AB
I
2
æ 1 ö 2
Z1 = Xc2 + R12 = ç ÷ + R1
è wC ø
®
= (100)2 + (100)2 = 100 2 B

V 20 ®
\ I1 = = [leads emf by f1]
Z1 100 2 B

R 100 1 Circular
where cos f1 = = = Þ q = 45º
Z1 100 2 2 loop

100µF When the current is decreasing at a steady rate then


the change in the flux (decreasing inwards) on the right
W half of the wire is equal to the change in flux (decreasing
A B outwards) on the left half of the wire such that Df
I1
through the circular loop is zero.

C
0.5H 50W D
SUBJECTIVE PROBLEMS :
I2 1. The magnetic lines of force due to current flowing in wire
AB is shown.
~ NOTE : As the current increases, the number of magnetic
20V lines of force passing through the loop increases in the
outward direction. To oppose this change, the current will
Impedance across CD is flow in the clockwise direction.

Z2 = X L2 + R22 = (wL)2 + R22

= (0.5 ´ 100)2 + (50)2 = 50 2 W

V 20
\ I2 = = [leads emf by f2]
Z2 50 2 2. KEY CONCEPT : This is based on motional emf.

180 km/h=50m/s
I1

45º
45º I 20V 1m

I2
e = vBl = 50 × 0.2 × 10–4 × 1 = 10–3 volt = 1 milli volt
3.
R 50 1
where cos f2 = = = Þ f2 = 45º
Z2 50 2 2
\ The current I from the circuit is
I = I1 + I2 ; 0.3 A
9. (a) Emf will be induced in the circular wire loop when flux
NOTE : When the coil A moves towards B, the number of
through it changes with time. magnetic lines of force passing through B changes.
Df Therefore, an induced emf and hence induced current is
e =- produced in B.
Dt
The direction of current in B will be such as to oppose the
when the current is constant, the flux changing through field change in B and therefore, will be in the opposite
it will be zero. direction of A.
P-420 Topic-wise Solved Papers - PHYSICS
Magnitude of curr ent remains constant at
4.
I = Bwr 2 / 2 R .
I
Bw r 2
2R

T/2 T 3T/2 2T t

For next half rotation, when the loop comes out of the
field, current of the same magnitude is set up clockwise.
Anticlockwise current is supposed to be positive. The
I-t graph is shown in the figure for two periods of
rotation.
6. (i) A variable force F is applied to the rod MN such that as
NOTE : The network behaves like a balanced wheatstone the rod moves in the uniform magnetic field a constant
bridge. current flows through R. Consider the loop MPQN.
The free electrons in the portion MN of the rod have a Let MN be at a distance x from PQ.
velocity v in the right direction. Applying Fleming’s left hand Length of rails in loop = 2x
rule, we find that the force on electron will be towards N.
Hence, M will be + ve and N will be negative. Current will
flow in clockwise direction.
The induced emf developed is given by
e = vBl = v × 2 ×0.1 = 0.2v ...(i)
Now, e = IR
e = 10 – 3 × 4 = 4 ×10–3 amp ...(ii)
From (i) and (ii),
0.2 v = 4 ×10 –3
4 ´10-3 \ Resistance of rails in loop = 2xl
\ v= = 0.02 m / s
0.2 \ Total resistance of loop = R + 2lx
5. (i) Induced emf Induced emf = Bvd
Bvd
df d \ Induced current (I) =
E =- = - ( B ´ A) R + 2 lx
dt dt So for constant I,
d é æ1 2 öù ( R + 2lx )
=- B ç r q÷ ú = - 1 Br 2 d q = - 1 Br 2 w v= I ...(i)
dt êë è 2 øû 2 dt 2
Bd
Furthermore, as due to induced current I the wire will
experience a force FM = BId opposite to its motion, the
E 1 Br 2 w 1 Br 2 w
\ I= = - Þ| I |= equation of motion of the wire will be
R 2 R 2 R F – FM = ma i.e., F = FM + ma
But as here FM = BId and from equation (i)
dv 2 lI dx 2 lIv 2 lI 2
a= = = = ( R + 2l x )
dt Bd dt Bd ( Bd ) 2
2 lmI 2
So, F = BId + ( R + 2 lx )
( Bd ) 2
(ii) As the work done by force F per sec.
dW é 2lmI 2 ù é R + 2lx ù
= P = Fv = ê BId + ( R + 2l x ) ú ê .Iú
dt 2
êë ( Bd ) úû ë Bd û
(ii) The loop is entering in the magnetic field and hence
magnetic lines of force passing through the loop is é 2lmI 3 ù
increasing in the downward direction. Therefore, current i.e., P = ê I 2 ( R + 2lx ) + ( R + 2lx )2 ú
3 3
will flow in the loop in such a direction which will ëê B d ûú
oppose the change. The current will flow in the anti- and heat produced per second, i.e., joule heat
clockwise direction. H = I 2 ( R + 2lx )
(iii) Graph between induced emf and period of rotation:
-1
For first half rotation, (t = T/2), when the loop enters H é 2 lmI ( R + 2lx) ù
So, f = = 1+
the field, the current is in anticlockwise direction. P êë B3 d 3
ú
û
ELECTROMAGNETIC INDUCTION & ALTERNATING CURRENT P-421
7. (a) (i) In this case S and I are connected. Taking log on both sides
R
- t 1
log e e L = log e
2
R L 0.6930 ´ 10 ´10-3
Þ t = 0.693 Þ t = 0.693 =
L R (2 + 3)
when R = R2 + R4
Þ t = 1.386 × 10 –3 sec. Thus this much time is required
for current to reach half of its steady value.
The energy stored by the inductor at that time is given
2
1 2 1 æ 0.6 ö
by E = LI = ´ 10 ´ 10-3 ´ ç ÷ = 4.5 ´ 10 -4 J
2 2 è 2ø
8. The equivalent circuit is drawn in the adjacent figure.

Using Kirchhoff’s law in ABCDGA


+ 3 – I2 × 2 – 12 + I1× 2 = 0
2 I1 – 2I2 = 9 ...(i)
Applying Kirchhoff ’s law in DEFGD
– 2I1 + 12 – (I1 + I2) 2 = 0
Þ 2I1 + I2 = 6 ...(ii) NOTE : As the magnetic field increases in the downward
direction, an induced emf will be produced in the AEFD as
21 well as in the circuit EBCF such that the current flowing in
From (i) and (ii) I1 = amp.
6 the loop creates magnetic lines of force in the upward
\ From (ii) I2 = –1 amp. direction (to the plane of paper).
To find potential difference between A and B Thus, the current should flow in the anticlockwise direction
VA + 3 – (– 1) × 2 = VB Þ VA – VB = – 5V in both the loops.
The rate of production of heat in R1 Induced emf in loop AEFD
2 df d dB
æ 21ö e=- = - BA = - A = -1 ´ 1 = -1volt
= I12 R1 = ç ÷ ´ 2 = 24.5W dt dt dt
è 6ø
Induced emf in loop EBCF
(b) (i) When the switch is put in position 2 then the active
circuit will be as shown in the figure. df d dB
e= - = - BA ' = - A ' = – 0.5 × 1 = – 0.5 volt
dt dt dt
Let the current flowing in the branch EADF be i1 and the
current flowing in the branch FCBE be i2. Applying junction
law at F, we get current in branch FE to be (i1 – i2)
Applying Kirchhoff ’s law in loop EADFE
– 1 × i1 – 1× i1 + 1 – 1 × i1 – 1 (i1 – i2) = 0
Þ 4i1 – i2 = 1 ... (i)
When the steady state current is reached then the Applying Kirchhoff 's law in loop EBCFE
inductor plays no role in the circuit + 0.5i1 – 0.5 + 1i2 + 0.5 i2 – 1(i1 – i2) = 0
E2 = I (R2 + R4) – i1 + 3i2 = 0.5 ... (ii)
Solving (i) and (ii)
3 11 i1 = 3.5
Þ I= = 0.6 amp.
5 7
(ii) KEY CONCEPT : The growth of current in L–R Þ i1 = 3.5/11 = A
circuit is given by the expression 22
Also 11i2 = 3
I = I0 é - tù
R
6
ëê1 - e L ûú Þ i2 = 3/11 A = A
22
I0 I é - tù
R
7
When I = , then 0 = I 0 ê1 - e L ú \ Current in segment AE = i1 = A
2 2 ê ú 22
ë û 6
Current in segment BE = i2 = A
-R R 22
1 t - t 1
Þ = 1- e L Þ e L = Current in segment EF = (i1 - i2 ) =
1
2 2 A
22
P-422 Topic-wise Solved Papers - PHYSICS
9. KEY CONCEPT : We can R1 (b) The above diagram can be reconstructed as the
understand the direction of flow A C adjacent figure. e is a constant. O will accumulate
i1
of induced currents by imagining positive charge and A negative. When the switch S is
B(i1+i2)l
a fictitious battery to be attached closed, transient current at any time t, when current I is
between E and F. The direction of e flowing in the circuit,
induced current can be found with E L i1+i2 F I = I0 (1– et / t )
the help of Lenz’s law. mg I
Here,
i2 S
NOTE : P.d. across parallel B D e B wr 2 O
R2 I0 = = R
combinations remains the same R 2R A
Also, P1 = ei1 = 0.76 W
L
and P2 = ei2 = 1.2 W and t =
R L
i1 1.76 1.76
\ = Þ i1 = i2 … (ii) æ Rö
i2 1.2 1.2 Bwr 2 -ç ÷ t
Therefore, I = [1 - e è L ø ]
The horizontal metallic bar L moves with a terminal velocity. 2R
This means that the net force on the bar is zero. (ii) In steady state,
\ B (i1 + i2 ) = mg
B wr 2 æ Rö
mg 0.2 ´ 9.8 49 I= [Q t has a large value and - çè L ÷ø t ® 0]
Þ i1 + i2 = = = amp. ......(iii) 2R e
Bl 0.6 ´ 1 15 NOTE : When current flows in the circuit in steady state,
From (ii) and (iii) there is a power loss through the resistor.
1.76 49 Also since the rod is rotating in a vertical plane, work needs
i2 + i2 = to be done to keep it at constant angular speed.
1.2 15
Power loss due to current I will be
19 0.76
Þ i2 = 2 amp. Þ i1 = amp. Þ e = = 0.6V
æ Br 2 w ö
2 t=t
15 19 /15 2
P=I R=ç ÷ R
The induced emf across L due to the movement of bar L in a è 2R ø r/2
magnetic field q
If torque required for this power r t =0
e 0.6 2 cosq
e = BvTL Þ vT = = = 1m/s is t1 then mg
BL 0.6 ´ 1
Also from (i), P = t1w
e 0.6 e 0.6 B 2 r 4w
R1 = = = 0.47W and R2 = = = 0.3 W Þ t1 =
i1 19 /15 i2 2 4R
10. (i) (a) Let us consider a small length of metal rod dx at a Torque required to move the rod in circular motion against
distance x from the origin. Small amount of emf (de) gravitational field
induced in this small length (due to metallic rod cutting
r
magnetic lines of force) is t 2 = mg ´ cos q
de = B (dx) v … (i) 2
where v is the velocity of small length dx The total torque
v = xw … (ii) t = t1 + t2 (Clockwise)
\ The total emf acoss the whole metallic rod OA is
B 2 r 4 w mgr
t= + cos wt
Y 4R 2
The required torque will be of same magnitude and in
A anticlockwise direction. The second term will change signs
w
S x dx as the value of cos q can be positive as well as negative.
q 11. KEY CONCEPT : Let I0 be the current at steady state. The
R O X magnetic energy stored in the inductor at this state will be
B
L=10H R=2W
L

10V
r
r é x2 ù Br 2 w
e = ò Bxw dx = Bw ê ú = 1 2
E= LI 0 … (i)
ëê 2 ûú 0 2
0
2
ELECTROMAGNETIC INDUCTION & ALTERNATING CURRENT P-423
This is the maximum energy stored in the inductor. The The magnetic moment of the loop
current in the circuit for one fourth of this energy can be
3m Ma 2 v
found as M '=I ´ A= 0 ´ ( pa 2 )
2x4 R
1 1
´ E = LI 2 … (ii) Now the potential energy of the loop placed in the magnetic
4 2 field is
Dividing equation (i) and (ii)
3m Ma 2 v ´ xa 2 m 0 M
1 2 U = – M' B cos 180° = 0 ´
E
LI 0 I 2x4 R 2px 3
= 2 ÞI = 0
E/4 1 2 2 3m 20 M 2 a 4v
LI \ U =
2
4 Rx 4
Also, V = I0 R
r dU
V 10 I0 5 Now, | F |= -
Þ I0 = = = 5 amp. \ I = = = 2.5 amp. dx
R 2 2 2
The equation for growth of current in L-R circuit is 21 m 20 M 2 a 4 v
I = I0 [1 – e – Rt/L] \ F =
4 Rx8
1
Þ 2.5 = 5 [1– e – 2t/10] Þ = 1 – e-t / 5 Since by Newton’s third law, action and reaction are equal.
2 Therefore, the above calculated force is acting on the magnet.
Þ t = 5 loge 2 = 2 × 2.303 × 0.3010 = 3.466 sec.
12. KEY CONCEPT : When the magnet moves, then the The direction of the force is in -iˆ direction by Lenz’s law..
magnetic flux passing through the circular loop changes. 13. KEY CONCEPT : If v is the velocity of the rod at any time
Therefore, an induced emf will be developed in the loop and t, induced emf is BvL and so induced current in the rod
an induced current flows. Due to this a magnetic field is Induced e.m.f. BvL
created by the loop which interacts with the magnetic field I= =
R R
of the dipole and therefore, a force acts on the dipole. Due to this current, the rod in the field B will experience a
Y force
B 2 L2 v
F = BIL = (opposite to its motion) … (1)
R R
So, equation of motion of the rod will be,
a
S N v T – F = 0 × a, i.e., T = F [as rod is massless]
X
O T B 2 L2v
x M mg – T = ma Þ a = g - =g- … (2)
m mR
So rod will acquire terminal velocity when its acceleration is
zero i.e.,
Z
B 2 L2 vT mgR
The coil is at an axial point of the bar magnet. Since the size g- = 0 i.e. vT = 2 2 ;
mR B L
of bar magnet is negligible, therefore the magnetic field
produced at O due to bar magnet when it is at a distance x vT
from the origin is For the case when velocity is
2
m M
B= 0 3
2p x
\ Flux passing through the coil = Ba2
\ The induced emf in the coil
-d f d é m Ma 2 1 ù
e= =- ê 0 ´ ú
dt dt êë 2 x3 úû

3m 0 Ma 2 dx é dx ù
= êëQ v = dt úû vT mgR
2 x 4 dt v= =
2 2 B 2 L2
3m 0 Ma 2
= ´v Substituting this value of velocity in eq. (2) we get
2x4
B 2 L2 1 mgR 1 g
e 3m 0 Ma 2 v a=g- ´ =g- g=
\ Current in the coil = I = = mR 2 B L 2 2 2 2
R 2 x4 R
P-424 Topic-wise Solved Papers - PHYSICS
14. Here, Q0 = maximum value of charge. df dy
Q = 200 µ C = 2 × 10–4 C Net emf., e = - = - B0 a = - B0 av(t )
dt dt
1 1 As total resistance = R
w= = = 10+4 s -1
LC -3 -6
(2 ´ 10 H ) (5.0 ´ 10 F) | e | B0 av(t )
\|i|= =
Let at t = 0, Q = Q0 then R R
Q (t) = Q0 cos wt … (1)
X
dQ
I (t ) = = - Q0 w sin wt … (2) F1
dt y
L = 2.0 mH
y+a
F2
F4

F3
Y
C = 5.0 mF
NOTE : Now as loop goes down, magnetic flux linked with
dI (t ) it increases, hence induced current flows in such a direction
= -Q0 w 2 cos (wt ) … (3) so as to reduce the magnetic flux linked with it. Hence,
dt
induced current flows in anticlockwise direction.
æ Q0 ö (b) Each side of the cube will experience a force as shown
(a) For Q = 100 µC çè or
2 ÷ø (since a current carrying segment in a magnetic field
experiences a force).
1
From (1), 100 = 200 cos wt or cos (wt) = , ur r ur æ B y ö
2 F 1 = i ( l ´ B ) = i ç - aiˆ ´ 0 kˆ÷ = B0 y (iˆ ´ ˆj );
From eqution (3) : è a ø

dI æ1ö ur æ B ( y + a ) ˆö
= (2.0 ´10-4 )(10+4 )2 ç ÷ = 10+4 A/s F 3 = i ç + aiˆ ´ 0 k ÷ = iB0 ( y + a ) ˆj
dt è2ø è a ø
(b) Q = 200 µC when ur ur
cos (wt) = 1 i.e. wt = 2p… NOTE : F 2 = - F 4 and hence will cancel out each other..
At this time, I (t) = – Q0 sin wt or I (t) = 0 ur ur ur ur ur B02 a 2v (t ) ˆ
(c) I (t) = Q0w sin wt Net force, F = F 1 + F 2 + F 3 + F 4 = -iB0 ajˆ = - j
R
\ Maximum value of I is Q0w
or Imax = Q0 w = (2.0 × 10 –4 C) (10 + 4 s –1) ur é B 2 a 2 v (t ) ù ˆ
Imax = 2.0 A (c) Total net force = mg ˆj + F = ê mg - 0 ú j;
(d) From energy conservation, ëê R ûú
1 2 1 1 Q2 dv B 2 a 2 v (t )
LI max = LI 2 + \ m = mg - 0
2 2 2 C dt R
2
or Q = LC ( I max - I2) v dv t
Integrating it, we get, ò0 B02 a 2 v (t )
= ò dt
0
I g-
I = max = 1.0 A mR
2
(v )t
\ Q= (2.0 ´ 10-3 )(5.0 ´ 10 -6 ) (2 2 - 12 ) é B02 a 2v (t ) ù
log ê g - ú
Q = 3 × 10–4 C or Q = 1.732 × 10–4 C êë mR úû
0
=t
15. Suppose at t = 0, y = 0 and t = t, y = y - B02 a 2
ur ur
(a) Total magnetic flux = B . A mR
ur ur B0 y
where A = a 2 k$ and B = k$ é B02 a2 v(t ) ù
a êg - ú 2 2
or log ê mR ú = - B0 a t
B0 y 2
\ f= .a = B0 ya ê g ú mR
a ê ú
ë û
ELECTROMAGNETIC INDUCTION & ALTERNATING CURRENT P-425
(ii) Power w.r.t time :
or 1-
B02 a 2 v (t )
=e
- ( B02a2t ) mR P(W)
mgR
Power (P ) versus time (t)

or 1- e
(
- B02 a 2t mR) B2 a 2
= 0 v (t );
2.5
mgR

\ v (t ) =
mgR é
ê1 - e
- ( B02a 2t ) mR ùú 0.8 t (s)
2 2 (iii)
B0 a ëê ûú Power P = i2 R
When terminal velocity is attained, v (t) does not depend = (1.25)2 (1.6) W
on t = 2.5 W ...(ii)
Since the power depends on square of the current, it
mgR does not depend on direction of current.
\ v (t) =
B02 a2 Figure (iii) displays power versus time graph for the
first two cycles.
16. (i) The current through the coil and the power dissipated
Q Time for one cycle = 0.4s
in the coil as functions of time. \ Time for 12000 cycles = 12000 × 0.4 = 4800 s
For the first two cycles are sketched below: \ Energy obtained = Power × time = 2.5 × 4800 J
The graph (i) denotes variation of magnetic field (B) = 12,000 J ......(iii)
with respect to time t. This energy heats the coil and water.
dB 0.8 Heat taken by coil = M C sC (q - 30)
= = 4T/s
dt 0.2 = 0.06 ´ 500 ´ (q - 30) = 30(q - 30)
Heat taken by water = MW sW (q - 30)
B(T)
Magnetic field (B) versus time ( t) = 0.5 ´ 4200 ´ (q - 30) = 2100(q - 30)
0.8 \ Total heat taken up = 30(q - 30) + 2100 (q - 30)
= 2130 q - 63900 ......(iv)
Equ. (iv) and (iii), 2130 q - 63900 = 12000
0 0.2 0.4 0.6 0.8 t(s)
(i) or 2130 q = 75900 or q = 75900 = 35.6°C
2130
Induced emf in coil = e \ Final temperature of water under thermal equilibrium
= 35.6°C.
df dB 17. This is a question on growth and rise of current.
\ e= = NA (100)(5 10 3 )(4) 2V
dt dt GROWTH OF CURRENT : Let at any instant of time t the
current be as shown in the figure.
e 2 I2
\ i= = = 1.25A ....... (i) A B C
R 1.6
I
i(a) I1
L
Current (i) versus time (t) 12V E
+1.25
R1
0
0.2 0.4 0.6 0.8 t (s)
R2
–1.25
(ii)
NOTE : Since the direction of current depends upon G F D
flux-change associated with the coil, it will be induced Applying Kirchoff’s law in the loop ABCDFGA
in opposite directions when the field increases or we get, starting from G moving clockwise
decreases. Field increases from 0 to 0.2 sec and from dI dI 2 R
0.4s to 0.6s. E - L 2 - I 2 R2 = 0 Þ = 2 dt
dt E L
Hence current flows in one direction. I2 -
R2
Field decreases from 0.2s to 0.4s and from 0.6s to 0.8s.
E éê - tù
R
Hence current direction is reversed.
The graph (ii) illustrates variation of (i) with time (t). or I 2 = 1- e L ú
R2 ê ú
ë û
P-426 Topic-wise Solved Papers - PHYSICS
Also we know that the emf (V) produced across the inductor Thus we see that force on QR is equal and opposite to that
df d dI on PS and balance each other.
V=– = - [ LI 2 ] = - L 2 The force on RS is
dt dt dt ur r ur
F RS = I (l ´ B ) = I [b (- $j ) ´ (3$i + 4k$ ) B ]
0
é æ - R2 ö ù
t
d E
= -L ê
dt ê R2
ç1 - e L ÷ ú = I bB0 [3k$ - 4i$] ... (i)
çè ÷ø ú
ë û The torque about PQ by this force is
R2 r r ur $ $ $
t t RS = r ´ F = (i a) ´ (3k - 4i ) IbB0
V = – E e - L . Here the negative sign shows the opposition
to the growth of current. = I abB0 (3 $j ) ... (ii)
2 The torque about PQ due to weight of the wire PQRS is
- t
\ V = 12 e 400 ´10 -3 = 12 e–5t volt æ aö
t = mg ç ÷ ... (iii)
DECAY OF CURRENT : When the switch is opened, the è 2ø
branch AG is out of the circuit. Therefore, the current decays
For the wire loop to be horizontal, we have to equate (ii) and
through the circuit CBFDC (in clockwise direction).
Applying Kirchhoff 's law C a
I (iii) 3IabB0 = mg
æ L dI ö 2
I (R1 + R2) – ç - =0
è dt ÷ø L mg
R1 Þ I= ... (iv)
6bB0
dI æ R + R2 ö
\ = -ç 1 dt
I è L ÷ø Therefore,
(a) The direction of current assumed is right. This is
\ On integrating, R2 because torque due to mg and current are in opposite
I dI ( R1 + R2 ) t directions. Therefore, current is from P to Q.
òI 0 I = - L ò0 dt ur
(b) From (i), F RS = IbB (3k$ - 4i$) 0
( R1 + R2 )t
-
\ I = I0 e L mg
(c) From (iv), I =
6aB0
R + R2 2+2
Here, 1 = = 10 19. (a) KEY CONCEPT : As the metal bar AB moves towards
L 400 ´ 10-3
the right, the magnetic flux in the loop ABCD increases
E 12 in the downward direction. By Lenz's law, to oppose
and I0 = = = 3A
R1 + R2 4 this, current will flow in anticlockwise direction as
shown in figure.
\ I = 3e–10t A, clockwise.
Alternatively, you may directly find the time constant A
D
L
t= and use the equation i = i0 e - t / t where i0 = 6A
R1 + R2
18. Let us consider the current in the clockwise direction in R
loop PQRS. Force on wire QR,
ur r ur I i
= I (l ´ B) = I [(ai$) ´ (3i$ + 4k$ ) B ]
v
F QR 0

= I B0 [3ai$ ´ i$ + 4ai$ ´ k$ ] = I B0 [0 + 4a ( – $j )] = -4aB0 I $j


L
Z
C
B B
x
>
Applying Kirchhoff 's loop law in ABCD, we get
3i

Q
+

>
P
4k

Y df di
>

4k = iR + L ... (i)
dt dt
(b) Let AB be at a distance x from the long straight wire at
X any instant of time t during its motion. The magnetic
S R field at that instant at AB due to long straight current
carrying wire is
Force on wire PS
ur r ur m 0 I0
F PS = I (l ´ B ) = I [a( -$i ) ´ (3$i + 4k$ ) B0 ] = 4aB0 I $j B=
2p x
ELECTROMAGNETIC INDUCTION & ALTERNATING CURRENT P-427
The change in flux through ABCD in time dt is The emf produced
df = B (dA) = B l d x df m aI w
Therefore, the total flux change when metal bar moves e= - = 0 0 ln (2) cos wt
dt p
from a distance x0 to 2x0 is
Charge stored in the capacitor
2x 2x m I m I l
Df = ò 0 B ldx = l ò 0 0 0 dx = 0 0 [log e x]2x x0 m 0 aI 0 w
x0 x0 2p x 2p 0
q=C×e=C× ln (2) cos wt ... (i)
m0 I0l p
= log e 2 ... (ii) \ Current in the loop
2p
The charge flowing through resistance R in time T is dq C ´ m0 aI 0 w 2
i= = ln (2)sin wt
T T 1 é di ù dt p
q = ò idt = ò E - L ú dt [from eq. (i)]
0 0 R êë induced dt û
m 0 aI 0 w 2 C ln (2)
1 T L i1 1 L \ imax =
= ò0 Einduced dt - ò0 d i = ( D f) - i1 p
R R R R (b) From (i), the graph between charge and time is
1 é m0 I0 l ù L Q
q= ê log e 2 ú - i1 from eq. (ii) Q0
R ë 2p û R
(c) When the metal bar AB is stopped, the rate of change
of magnetic flux through ABCD becomes zero.
From (i), p/w 2p/w t
di p/2w 3p/2w
iR = – L
dt
2T L i1 / 4 di
òT dt = R òi1 i – Q0
L i /4 L T C ´ m 0 aI 0 w ln (2)
T= - log e 1 Þ =
R i1 R 2 log e 2 Here, q0 =
p
20. (a) Let us consider a small strip of thickness dx as shown 21. Given, Vrms = 220 V
in the figure. n = 50 Hz, L = 35 mH, R = 11W
The magnetic field at this strip Impedance
B = BA + BB
(Perpendicular to the plane of paper directed upwards) Z= (wL)2 + R2 = 11 2 W
m0 I m0 I V0
= + also, I0 =
2p x 2p (3a - x ) Z
BA = Magnetic field due to current in wire A
V 2
m I é1 1 ù V0 = Vrms 2 \ I0 = rms = 20 A
= 0 ê + Z
2p ë x 3a - x úû
BB = Magnetic field due to current in wire B R 1 p
cos f = = \ f=
Z 2 4
\ graph is given by.
dx
V = V0 sin w t
V0
x
I I0
O t
I= I0 sin(w t- p /4)
Small amount of magnetic flux passing through the strip
of thickness dx is
m 0 Ia ´ 3a dx
df = B × adx = 22. NOTE : After a long time capacitor will be fully charged,
2 p x(3a - x )
hence no current will flow through capacitor and all the
Total flux through the square loop current will flow from inductor. Since current is D.C.,
2a m 0 I ´ 3a 2 dx m Ia resistance of L is zero.
f= òa 2p x (3a - x)
= 0 ln 2
p æR ö 1 3R
\ Req = ç + R÷ ´ + r1 + r2 = + r1 + r2
è2 ø 2 4
m 0 a ln (2)
= (I0 sin wt)
p
P-428 Topic-wise Solved Papers - PHYSICS
R Here, l = 2 p R and A = L × d
R L
A B R 2p R
\ R= r ... (ii)
Ld
r1 r2 R The induced current I will be
R C
R | e | [p µ0 na 2i0 w cos wt ] ´ Ld
I= =
R r ´ 2 pR
S m 0 na2 Ld i0 w cos wt
Þ I=
e+e 2e 2e 2 rR
I= Þ I= =
Req Req (3R / 4) + r1 + r2
MATCH THE FOLLOWING :
Potential drop across A is
1. A-r,s,t; B-q,r,s,t; C-p,q; D-q,r,s,t
2e
e – I r1 = 0 Þ e= r1 The following are the important concepts which are applied
(3R / 4) + r1 + r2 in the given situation.
4 (i) For DC circuit, in steady state, the current I through
Þ r1 = r2 + 3R/4 or (r1 - r2 )
R= the capacitor is zero. In case of L-C circuit, the potential
3
23. KEY CONCEPT : The magnetic field in the solenoid is given difference across the inductor is zero and that across
by the capacitor is equal to the applied potential difference.
B = µ0 ni In case of L-R circuit, the potential difference across
R inductor is zero across resistor is equal to the applied
a voltage.
(ii) For AC circuit in steady state, Irms current flows
d through the capacitor, inductor and resistor. The
potential difference across resistor, inductor and
capacitor is proportional to I.
L (iii) For DC circuit, for changing current, the potential
difference across inductor, capacitor or resistor is
proportional to the current.
COMPREHENSION BASED Q UESTIONS :
1. (b) For charging of R – C circuit, Q = Q0 [1 – e–t/t] when
the charging is complete, the potential difference
Þ B = µ0 ni0 sin wt [Q i = i0 = sin wt (given)]
between the capacitor plates will be V. The charge
The magnetic flux linked with the solenoid
ur ur stored in this case will be maximum.
f = B . A = B A cos 90° = (µ0 ni0 sin wt) (pa2)
\ The rate of change of magnetic flux through the solenoid V
df
= p m 0 na 2i0 w cos wt R
dt
The same rate of change of flux is linked with the cylindrical C S1
shell. By the principle of electromagnetic induction, the
induced emf produced in the cylindrical shell is
S2
L
x
x xx Therefore, Q0 = CV.
é -2 t ù
When t = 2t, Q = CV ê
ë1 = e t úû = CV [1 - e -2 ]
I
TOP VIEW 2. (d) The instantaneous charge on plates at any time t during
df discharging is
e=- = -p m0 na 2 i0 w cos wt ... (i)
dt Q = Q0 cos wt
The resistance offered by the cylindrical shell to the flow of \ Instantaneous current,
induced current I will be
dQ
l I= = Q0 w sin wt
R= r dt
A
ELECTROMAGNETIC INDUCTION & ALTERNATING CURRENT P-429

_ C I Q×B
+ But Dw = ...(2)
2m
2
æ QB ö 2 - gBQR
S2 From (1) and (2) DM = -g m ç ÷ R =
è 2m ø 2
The negative sign shows that change is opposite to
the direction of B.
L (b) is the correct option.
\ The magnitude of maximum current
ASSERTION & REASON TYPE QUESTIONS :
Imax = Q0w
1. (a) As shown in the figure the horizontal component of
1
Here Q0 = CV and w = the magnetic field interacts with the induced current
LC produced in the conducting ring which produces an
average force in the upward direction. (Fleming's left
1 C hand rule).
\ Imax = CV × =V
LC L
F I
3. (c) Apply Kirchhoff 's law in the circuit
I
Q dI Q dI
-L =0 Þ =L BH
C dt C dt

d æ dQ ö d 2Q
Þ Q = LC ç - ÷ = - LC Conducting ring
dt è dt ø dt 2
Ns Vs 10 V INTEGERANSWERQUESTIONS:
4. (a) For step up transformer = Þ = s
N p Vp 1 4000 1. (4) Time constant = RC
\ Vs = 40,000 V æ 1 ö
2
Impedance = R2 + ç
Np Vp è wC ÷ø
For step down transformer =
Ns Vs
Given impedance = R 1.25
40,000 200
= = 2
200 1 æ 1 ö
\ R 1.25 = R2 + ç
(a) is the correct option. è wC ÷ø
5. (b) We know that P = V × I
2
P 600 ´ 1000 2 æ 1 ö
\ R 2 (1.25) = R + ç
\ I=
V
=
4000 è wC ÷ø
\ I = 150 A 2
Total resistance = 0.4 × 20 = 8 W R2 æ 1 ö R 1
\ =ç \ =
\ Power dissipated as heat = I2R = (150)2 × 8 = 180,000W 4 è wC ÷ø 2 wC
= 180 kW
2 2
180 \ RC = = ´ 1000 ms
\ % loss = ´ 100 = 30% w 500
600 \ RC = 4 ms
(b) is the correct option. 2. (7) The magnetic field due to current carrying wire at the
® ® - df d dB location of square loop is
6. (b) ò E .dl = = - (BpR 2 ) = -pR 2
dt dt dt µ0 2piR2 µ i
B= = 0
= - pR 2 B 4p (R2 + 3R2 )3/2 16R
\ E × 2pR = – pR2B The mutual induction
- BR N f 2 é µ0 i 2 ù
\E= M= = ê ´ a cos 45º ú
2 i i ë 16R û
(b) is the correct option.
7. (b) Given M = g L µ0 a2
\ M=
\ M = g mwR2 7
\ M = g m (Dw) R2 ...(1) 22 R
P-430 Topic-wise Solved Papers - PHYSICS

1. (b) The impedance triangle for resistance (R) and inductor When half energy is with the capacitor in the form of
(L) connected in series is shown in the figure. electric field between the plates of the capacitor we get
2 E 1 Q '2
2
L = where Q ' is the charge on one plate of
2 +w 2 2 C
R XL= w L the capacitor
f 1 1 Q 2 1 Q '2 Q
R \ ´ = Þ Q' =
2 2 C 2 C 2
R 8. (a) Laminated core provide less area of cross-section for
Power factor cosf = the current to flow. Because of this, resistance of the
R2 + w 2 L2 core increases and current decreases thereby
2. (d) The induced emf is decreasing the eddy current losses.
r r 9. (a) D.C. ammeter measure average current in AC current,
-d f d ( B. A) - d ( BA cos 0º )
e= =- = average current is zero for complete cycle. Hence
dt dt dt reading will be zero.
× × × × 10. (d) Since the phase difference between L & C is p,
× × × × \ net voltage difference across LC = 50 – 50 = 0
× ×l × × V Df (W2 - W1 )
11. (b) =
× × × × Dt t
x
× × × × Rtot = ( R + 4 R )W = 5R W
dA d (l ´ x ) dx nd f -n(W2 - W1 )
\ = –B = -B = - Bl = - Blv i= =
dt dt dt Rtot dt 5 Rt
3. (d) These three inductors are connected in parallel. The
equivalent inductance Lp is given by (QW2 & W1 are magnetic flux)
r r
1 1 1 1 1 1 1 3 12. (b) f = B. A ; f = BA cos wt
= + + = + + = =1
L p L1 L2 L3 3 3 3 3 df wBA
e=- = wBA sin wt ; i = sin wt
\ Lp = 1 dt R
4. (b) Np = 140, Ns = 280, Ip = 4A, Is = ? 2
æ wBA ö
Np Pinst = i 2 R = ç ´ R sin 2 wt
Is è R ÷ø
For a transformer I = N
p s
T T
2
Þ
I s 140
= Þ Is = 2 A
ò Pinst ´ dt (wBA) 2
ò sin wtdt
4 280 Pavg = 0 = 0 1 ( wBA) 2
T R T =
5. (b) Mutual conductance depends on the relative position 2 R
and orientation of the two coils. ò dt ò dt
0 0
Df -D ( LI ) DI
6. (d) e=- = = -L é pr 2 ù
Dt Dt Dt (w B pr 2 ) 2 ê A = ú
\ Pavg = 2 ûú
DI 4 8R ëê
\ | e |= L Þ 8= L ´
Dt 0.05 13. (a) For resonant frequency to remain same
LC should be const. LC = const
8 ´ 0.05
Þ L= = 0.1H L
4 Þ LC = L' × 2C Þ L ' =
7. (c) When the capacitor is completely charged, the total 2
energy in the L.C circuit is with the capacitor and that 14. (b) l = 1m, w = 5 rad/s, B = 0.2 ´ 10 -4 T
energy is
Bwl 2 0.2 ´10-4 ´ 5 ´1
1Q 2 e= = = 50mV
E= 2 2
2 C 15. (c) Relative velocity = v + v = 2v
\ emf. = B.l (2v)
ELECTROMAGNETIC INDUCTION & ALTERNATING CURRENT P-431
16. (d) For maximum power, X L = X C , which yields Let E is short circuited at t = 0. Then
E
1 1 At t = 0, i0 =
C= = R
2 2
(2 pn) L 4 p ´ 50 ´ 50 ´ 10 Let during decay of current at any time the current
\ C = 0.1 ´ 10-5 F = 1mF flowing is - L di - iR = 0
dt
æ pö
17. (a) Phase difference for R–L circuit lies between ç 0, ÷ di R
i
di
t
R
è 2ø Þ
i
= - dt Þ
L ò i ò L
= - dt
R 12 4 i0 0
18. (b) Power factor = cos f = = = = 0.8
Z 15 5 R
i R - t
19. (a) KEY CONCEPT : The charging of inductance given Þ log e = - t Þ i = i0 e L
i0 L
æ - ö
Rt
by, i = i0 ç1 - e L ÷ R -100 ´10-3
çè ÷ø E - t 100 100 ´10 -3 1
Þi= e L = e =
R 100 e
Rt Rt 25. (c) KEY CONCEPT : We know that power consumed in
i0 - - 1
= i0 (1 - e L ) Þ e L = a.c. circuit is given by, P = Erms .I rms cos f
2 2
Taking log on both the sides, Here, E = E0 sinwt
Rt æ pö
- = log1 - log 2 I = I0 sin ç wt - ÷
L è 2ø

L 300 ´ 10 -3 p
Þ t= log 2 = ´ 0.69 which implies that the phase difference, f =
R 2 2
Þ t = 0.1 sec. p æ p ö
\ P = Erms .I rms .cos =0 çè cos = 0÷
f BA 2 2 ø
20. (b) Mutual inductance = =
I I
æ - tö
R

[MT Q L ]-1 -1 2 26. (a) KEY CONCEPT : I = I o ç1 - e L ÷


[Henry] = = ML2 Q -2 çè ÷ø
[QT -1 ]
(When current is in growth in LR circuit)
V 100
21. (c) Across resistor, I = = = 0.1 A æ - ´2 ö
5
Eæ - tö
R
R 1000 5
= ç1 - e L ÷ = ç1 - e 10 ÷ = (1 – e–1)
At resonance, R çè ÷ø 5 çè ÷ø
1 1
X L = XC = = = 2500
wC 200 ´ 2 ´ 10-6 m0 N1 N 2 A 4p ´ 10-7 ´ 300 ´ 400 ´ 100 ´ 10-4
27. (d) M= =
Voltage across L is l 0.2
I X L = 0.1 ´ 2500 = 250 V = 2.4p ´ 10 -4 H
ur ur
df d ( N B. A) 28. (c) Growth in current in LR2 branch when switch is closed
22. (d) e = - = - is given by
dt dt
R2t
d E di E R2 - R2t / L E - L
= - N ( BA cos wt ) = NBAw sin wt i= [1 - e - R2t / L ] Þ = . .e = e
dt R2 dt R2 L L

Þ e max = NBAw Hence, potential drop across


2t
æ E -R t / L ö -
23. (b) f = 10t 2 - 50t + 250 L = çè e 2 ÷ø L = Ee 2
-R t / L
= 12e 400 ´10
-3
L
df = 12e–5tV
e=- = -(20t - 50)
dt 29. (b) Due to the movement of resistor R, an emf equal to Blv
will be induced in it as shown in figure clearly,
et = 3 = -10 V
I = I1 + I 2
24. (a) Initially, when steady state is achieved,
Also, I1 = I2
E
i=
R
P-432 Topic-wise Solved Papers - PHYSICS
P
Solving the circuit, 1
we get l Blv Also q = q0 cos wt and w =
LC
Blv
I1 = I 2 = RW v RW p
3R RW On solving t = LC
4
2 Blv I
and I = 2 I1 = 34. (b) We have, V = V0 (1 – e–t/RC)
3R I2
I1 Q (
Þ 120 = 200 1 - e - t / RC )
30. (c) At t = 0 , no current will flow through L and R1 Þ t = RC in (2.5)
V Þ R = 2.71 × 106 W
\ Current through battery = R
2 35. (c) Time constant for parallel combination = 2RC
At t = ¥ , RC
RR Time constant for series combination =
effective resistance, Reff = 1 2 2
R1 + R2
t
- 1 V0
V V ( R1 + R2 ) In first case : V = V0 e 2 RC = ....(1)
\ Current through battery = R = R1R2 2
eff
31. (d) When capacitance is taken out, the circuit is LR. -
t2
( RC / 2) V0
wL In second case : V = V0 e = ....(2)
\ tan f = 2
R
From (1) and (2)
1 200
Þ wL = R tan f = 200 ´ = t1 t2 t 10
3 3 = Þ t2 = 1 = = 2.5 sec.
2 RC ( RC / 2) 4 4
Again , when inductor is taken out, the circuit is CR.
1 36. (a)
\ tan f = W E
w CR
eind = Bvl
1 1 200
Þ = R tan f = 200 ´ = = 0.3 ´ 10-4 ´ 5 ´ 20 = 3 × 10–3 V = 3 mV..
wc 3 3
37. (d) Because of the Lenz's law of conservation of energy.
2 38. (d) Here, induced e.m.f.
2 æ 1 ö
Now, Z = R + ç - wL ÷ w l
è w C ø 2l
dx
2 x
æ 200 200 ö
= (200) 2 + ç - ÷ = 200 W
[(3l)2 – (2l)2 ] = 5Bl w
3l 2
è 3 3ø
e = ò (wx) Bdx = Bw
2 2
Power dissipated = Vrms I rms cos f 2l

Vrms R æ Rö 39. (a) As we know, Magnetic flux, f = B. A


= Vrms . . çQ cos f = ÷
Z Z è Zø
m0 (2)(20 ´ 10 -2 ) 2
2 2 ´ p(0.3 ´ 10 -2 ) 2
V rms R (220) ´ 200
220 ´ 220 2[(0.2)2 + (0.15) 2 ]
= = 2 2 = = 242 W
Z (200) 200 On solving
32. (c) Induced emf = vBH l = 1.5 × 5 × 10–5 × 2 = 15 × 10–5 = 9.216 × 10–11 = 9.2 × 10–11 weber
= 0.15 mV 40. (c) Charge on he capacitor at any time t is given by
1 q = CV (1– et/t)
33. (a) Energy stored in magnetic field = Li2
2 at t = 2t
q = CV (1 – e–2)
1 q2
Energy stored in electric field =
2 C

1 2 1 q2
\ Li =
2 2 C
16 Ray & Wave Optics

FILL IN THE BLANKS : f


8. A thin rod of length is placed along the optic axis of a
1. A light wave of frequency 5 × 1014 Hz enters a medium of 3
refractive index 1.5. In the medium the velocity of the light concave mirror of focal length f such that its image which is
wave is ......... and its wavelength is ..........(1983 - 2 Marks) real and elongated, just touches the rod. The magnification
2. A convex lens A of focal length 20 cm and a concave lens B is .................... (1991 - 1 Mark)
of focal length 5 cm are kept along the same axis with a 9. A ray of light undergoes deviation of 30° when incident on
distance d between them. If a parallel beam of light falling an equilateral prism of refractive index 2 . The angle made
on A leaves B as a parallel beam, then d is equal to ...... cm.
(1985 - 2 Marks) by the ray inside the prism with the base of the prism is
3. A monochromatic beam of light of wavelength 6000 Å in .................... (1992 - 1 Mark)
vacuum enters a medium of refractive index 1.5. In the 10. The resolving power of electron microscope is higher that
medium its wavelength is ....., its frequency is ...... that of an optical microscope because the wavelength of
(1985 - 2 Marks) electrons is ..................... than the wavelength of visible light.
4. In Young’s double-slit experiment, the two slits act as (1992 - 1 Mark)
coherent sources of equal amplitude ‘A’ and of wavelength 11. If e 0 and m 0 are, respectively, the electric permittivity and
‘ l ’. In another experiment with the same set-up the two magnetic permeability of free space, e and m the
slits are sources of equal amplitude ‘A’ and wavelength ‘ l ’, corresponding quantities in a medium, the index of refraction
but are incoherent. The ratio of the intensity of light at the of the medium in terms of the above parameters is
midpoint of the screen in the first case to that in the second ...................... (1992 - 1 Mark)
case is ............. (1986 - 2 Marks) 12. A slit of width d is placed in front of a lens of focal length
5. A thin lens of refractive index 1.5 has a focal length of 15 cm 0.5 m and is illuminated normally with light of wavelength
in air. When the lens is placed in a medium of refractive
5.89 ×10–7m. The first diffraction minima on either side of
4 the central diffraction maximum are separated by 2×10–3 m.
index , its focal length will become ........cm.
3 The width d of the slit is .....m. (1997C - 1 Mark)
(1987 - 2 Marks) 13. A light of wavelength 6000Å in air, enters a medium with
6. A point source emits sound equally in all directions in a refractive index 1.5 Inside the medium its frequency is .... Hz
non-absorbing medium. Two points P and Q are at a distance and its wavelength is .... Å. (1997 - 2 Marks)
of 9 meters and 25 meters respectively from the source. The 14. Two thin lenses, when in contact, produce a combination of
ratio of amplitudes of the waves at P and Q is ............ power +10 diopters. When they are 0.25 m apart, the power
(1989 - 2 Marks) reduces to +6 diopters. The focal length of the lenses are ....
7. A slab of a material of refractive index 2 shown in fig. has a m and ... m. (1997 - 2 Marks)
curved surface APB of radius of curvature 10 cm and a plane 15. A ray of light is incident normally on one of the faces of a
surface CD. On the left of APB is air and on the right of CD
is water with refractive indices as given in the figure. An prism of apex angle 30° and refractive index 2 . The angle
object O is placed at a distance of 15 cm from the pole P as of deviation of the ray is... degrees. (1997 - 2 Marks)
shown. The distance of the final image of O from P, as viewed
from the left is ............... (1991 - 2 Marks) TRUE / FALSE :
1. The setting sun appears higher in the sky than it really is.
A C
n1 = 1.0 (1980)
n 2 = 2.0 4 2. The intensity of light at a distance ‘r’ from the axis of a long
n3 =
3 cylindrical source is inversely proportional to ‘r’.
P C O (1981- 2 Marks)
3. A convex lens of focal length 1 meter and a concave lens of
15 cm
focal length 0.25 meter are kept 0.75 meter apart. A parallel
beam of light first passes through the convex lens, then
B D through the concave lens and comes to a focus 0.5 m away
20 cm from the concave lens. (1983 - 2 Marks)
P-434 Topic-wise Solved Papers - PHYSICS
4. A beam of white light passing through a hollow prism give 5. Two coherent monochromatic light beams of intensities I
no spectrum. (1983 - 2 Marks) and 4 I are superposed. The maximum and minimum possible
5. The two slits in a Young’s double slit experiment are intensities in the resulting beam are (1988 - 1 Mark)
illuminated by two different sodium lamps emitting light of
(a) 5I and I (b) 5I and 3I
the same wavelength. No interference pattern will be
observed on the screen. (1984- 2 Marks) (c) 9I and I (d) 9I and 3I
6. In a Young’s double slit experiment performed with a source 6. Spherical aberration in a thin lens can be reduced by
of white light, only black and white fringes are observed. (1994 - 1 Mark)
(1987 - 2 Marks) (a) using a monochromatic light
7. A parallel beam of white light fall on a combination of a
concave and a convex lens, both of the same meterial. Their (b) using a doublet combination
focal lengths are 15 cm and 30 cm respectively for the mean (c) using a circular annular mark over the lens
wavelength in white light. On the other side of the lens (d) increasing the size of the lens.
system, one sees coloured patterns with violet colour at the
7. A beam of light of wave length 600 nm from a distance source
outer edge. (1988 - 2 Marks)
falls on a single slit 1 mm wide and a resulting diffraction
pattern is observed on a screen 2m away. The distance
MCQ's WITH ONE CORRECT ANSWER : between the first dark fringes on either side of central bright
1. When a ray of light enters a glass slab from air, fringe is (1994 - 1 Mark)
(a) its wavelength decreases. (1980) (a) 1.2 cm (b) 1.2 mm
(b) its wavelength increases. (c) 2.4 cm (d) 2.4 mm
(c) Its frequency decreases. 8. An isosceles prism of angle 120° has a refractive index 1.44.
(d) neither its wavelength nor its frequency changes.
Two parallel monochromatic rays enter the prism parallel to
2. A glass prism of refractive index 1.5 is immersed in water
(refractive index 4/3). A light beam incident normally on the each other in air as shown. The rays emerge from the opposite
face AB is totally reflected to reach on the face BC if faces (1995S)
(1981- 2 Marks)
A B
q

120°
C

8 2 8
(a) sin q ³ (b) < sin q <
9 3 9
2 (a) are parallel to each other
(c) sin q £
3 (b) are diverging
3. In Young’s double-slit experiment, the separation between (c) make an angle 2 [sin–1 (0.72) – 30°] with each other
the slits is halved and the distance between the slits and the
screen is doubled. The fringe width is (1981- 2 Marks) (d) make an angle 2 sin –1 (0.72) with each other
(a) unchanged. (b) halved. 9. A diminished image of an object is to be obtained on a
(c) doubled (d) quadrupled screen 1.0 m from it. This can be achieved by appropriately
4. A ray of light from a denser medium strike a rarer medium at placing (1995S)
an angle of incidence i (see Fig). The reflected and refracted (a) a concave mirror of suitable focal length
rays make an angle of 90° with each other. The angles of
reflection and refraction are r and r’ The critical angle is (b) a convex mirror of suitable focal length
(1983 - 1 Mark) (c) a convex lens of focal length less than 0.25 m
(d) a concave lens of suitable focal length
10. The focal lengths of the objective and the eye piece of a
i r compound microscope are 2.0 cm and 3.0 cm, respectively.
The distance between the objective and the eye piece is
15.0 cm. The final image formed by the eye piece is at infinity.
r' The two lenses are thin. The distance in cm of the object
and the image produced by the objective, measured from
the objective lens, are respectively (1995S)
(a) sin –1 (tan r ) (b) sin –1 (tan i )
(a) 2.4 and 12.0 (b) 2.4 and 15.0
(c) sin –1 (tan r' ) (d) tan –1 (sin i ) (c) 2.0 and 12.0 (d) 2.0 and 3.0
RAY & WAVE OPTICS P-435
11. Consider Fraunhoffer diffraction pattern obtained with a 17. A hollow double concave lens is made of very thin
single slit illuminated at normal incidence. At the angular transparent material. It can be filled with air or either of two
position of the first diffraction minimum the phase difference liquids L 1 or L 2 having refractive indices m1 and m 2
(in radians) between the wavelets from the opposite edges respectively (m2 > m1 > 1). The lens will diverge a parallel
of the slit is (1995S)
beam of light if it is filled with (2000S)
(a) p / 4 (b) p / 2 (c) 2p (d) p (a) air and placed in air (b) air and immersed in L1
12. In an interference arrangement similar to Young’s double- (c) L1 and immersed in L2 (d) L2 and immersed in L1
slit experiment, the slits S1 and S2 are illuminated with
coherent microwave sources, each of frequency 106 Hz. The 18. A point source of light B is placed at a distance L in front of
sources are synchronized to have zero phase difference. the centre of a mirror of width 'd' hung vertically on a wall.
The slits are separated by a distance d = 150.0 m. The A man walks in front of the mirror along a line parallel to the
intensity I (q) is measured as a function of q, where q is mirror at a distance 2L from it as shown in fig. The greatest
defined as shown. If I0 is the maximum intensity, then I (q) distance over which he can see the image of the light source
for 0 £ q £ 90 ° is given by (1995S) in the mirror is (2000S)
B
S1 d

d/2
L
2L
d/2
(a) d/2 (b) d
S2 (c) 2d (d) 3d
19. A diverging beam of light from a point source S having
divergence angle a , falls symmetrically on a glass slab as
(a) I (q) = I0 / 2 for q = 30° shown. The angles of incidence of the two extreme rays are
(b) I (q) = I0 / 4 for q = 90° equal. If the thickness of the glass slab is t and the refractive
index n, then the divergence angle of the emergent beam is
(c) I (q) = I0 for q = 0°
(d) I (q) is constant for all values of q. S
13. An eye specialist prescribes spectacles having combination
of convex lens of focal length 40 cm in contact with a concave i a i
lens of focal length 25 cm. The power of this lens combination
in diopters is (1997C - 1 Mark) m t (2000S)
(a) + 1.5 (b) – 1.5 (c) +6.67 (d) –6.67
14. A concave lens of glass, refractive index 1.5 has both
surfaces of same radius of curvature R. On immersion in a (a) zero (b) a
medium of refractive index 1.75, it will behave as a
æ 1ö æ 1ö
(1999S - 2 Marks) (c) sin -1 ç ÷ (d) 2sin -1 ç ÷
(a) convergent lens of focal length 3.5 R è nø è nø
(b) convergent lens of focal length 3.0 R 20. A rectangular glass slab ABCD of refractive index n1 is
(c) divergent lens of focal length 3.5 R immersed in water of refractive index n2(n1 > n2). A ray of
(d) divergent lens of focal length 3.0 R light is incident at the surface AB of the slab as shown. The
15. Yellow light is used in a single slit diffraction experiment
with slit width of 0.6 mm. If yellow light is replaced by maximum value of the angle of incidence amax such that the
X– rays, then the observed pattern will reveal, ray comes out only from the other surface CD is given by
(1999S - 2 Marks) A
(a) that the central maximum is narrower D
(b) more number of fringes n1 n2
amax
(c) less number of fringes (2000S)
B C
(d) no diffraction pattern
16. A thin slice is cut out of a glass cylinder along a plane
parallel to its axis. The slice is placed on a flat glass plate as én æ æ n ööù
shown in Figure. (a) sin-1 ê 1 cos ç sin-1 ç 2 ÷ ÷ ú
The observed interference fringes ëê n2 è è n1 ø ø ûú
from this combination shall be
(1999S - 2 Marks) é æ æ 1 öö ù
(b) sin-1 ên1 cos ç sin-1 ç ÷ ÷ ú
(a) straight ëê è è n2 ø ø ûú
(b) circular
(c) equally spaced æn ö æn ö
(d) having fringe spacing which increases as we go (c) sin -1 ç 1 ÷ (d) sin -1 ç 2 ÷
outwards è n2 ø è n1 ø
P-436 Topic-wise Solved Papers - PHYSICS
21. In a double slit experiment instead of taking slits of equal 27. An observer can see through a pin-hole the top end of a
widths, one slit is made twice as wide as the other. Then, in thin rod of height h, placed as shown in the figure. The
the interference pattern (2000S) beaker height is 3h and its radius h. When the beaker is
(a) the intensities of both the maxima and the minima filled with a liquid up to a height 2h, he can see the lower
end of the rod. Then the refractive index of the liquid is
increase
(2002S)
(b) the intensity of the maxima increases and the minima
has zero intensity
(c) the intensity of the maxima decreases and that of the
minima increases
(d) the intensity of the maxima decreases and the minima 3h
has zero intensity
22. In a compound microscope, the intermediate image is h
(a) virtual, erect and magnified (2000S) 2h
(b) real, erect and magnified 5 5
(c) real, inverted and magnified (a) (b)
2 2
(d) virtual, erect and reduced
23. Two beams of light having intensities I and 4I interfere to 3 3
produce a fringe pattern on a screen. The phase difference (c) (d)
2 2
between the beams is p/2 at point A and p at point B. Then 28. Which one of the following spherical lenses does not exhibit
the difference between the resultant intensities at A and B is dispersion? The radii of curvature of the surfaces of the
(a) 2I (b) 4I (2001S) lenses are as given in the diagrams. (2002S)
(c) 5I (d) 7I
24. In a Young’s double slit experiment, 12 fringes are observed
to be formed in a certain segment of the screen when light of (a) R1 R2 (b) R
wavelength 600 nm is used. If the wavelength of light is
changed to 400 nm, number of fringes observed in the same
segment of the screen is given by (2001S)
(a) 12 (b) 18
(c) 24 (d) 30 (c) R R (d) R
25. A ray of light passes through four transparent media with
refractive indices m1, m2, m3 and m4 as shown in the figure. 29. In the ideal double-slit experiment, when a glass-plate
The surfaces of all media are parallel. If the emergent ray CD (refractive index 1.5) of thickness t is introduced in the path
is parallel to the incident ray AB, we must have (2001S) of one of the interfering beams (wave-lenght l ), the
intensity at the position where the central maximum occurred
D previously remains unchanged. The minimum thickness of
m1 m2 m3 m4 the glass-plate is (2002S)
C (a) 2 l (b) 2 l /3
B
(c) l /3 (d) l
30. Two plane mirrors A and B are aligned parallel to each other,
as shown in the figure. A light ray is incident at an angle 30°
A at a point just inside one end of A. The plane of incidence
coincides with the plane of the figure. The maximum number
(a) m1 = m2 (b) m2 = m3 of times the ray undergoes reflections (including the first
(c) m3 = m4 (d) m4 = m1 one) before it emerges out is (2002S)
26. A given ray of light suffers minimum deviation in an B
equilateral prism P. Additional prism Q and R of identical 2 3m
shape and of the same material as P are now added as shown
in the figure. The ray will now suffer
(2001S)
0.2m
30°
Q

P R
A
(a) greater deviation (b) no deviation (a) 28 (b) 30
(c) same deviation as before(d) total internal reflection (c) 32 (d) 34
RAY & WAVE OPTICS P-437
31. In the adjacent diagram, CP represents a wavefront and AO 35. An equilateral prism is placed on a horizontal surface. A ray
& BP, the corresponding two rays. Find the condition on PQ is incident onto it. For minimum deviation
(2004S)
q for constructive interference at P between the ray BP and
reflected ray OP. (2003S) R
Q S
P
Q O R
(a) PQ is horizontal (b) QR is horizontal
(c) RS is horizontal
(d) Any one will be horizontal
36. Monochromatic light of wavelength 400 nm and 560 nm are
C incident simultaneously and normally on double slits
d apparatus whose slits separation is 0.1 mm and screen
distance is 1m. Distance between areas of total darkness
will be (2004S)
(a) 4 mm (b) 5.6 mm
A (c) 14mm (d) 28mm
37. A source emits sound of frequency 600 Hz inside water.
P
The frequency heard in air will be equal to (velocity of
sound in water = 1500 m/s, velocity of sound in air = 300 m/s)
B (2004S)
(a) 3000 Hz (b) 120 Hz
(c) 600 Hz (d) 6000 Hz
(a) cos q =3 l /2d (b) cos q = l /4d 38. A point object is placed at the centre of a glass sphere of
(c) sec q - cos q = l /d (d) sec q - cos q =4 l /d radius 6 cm and refractive index 1.5. The distance of virtual
image from the surface is (2004S)
32. The size of the image of an object, which is at infinity, as (a) 6 cm (b) 4 cm
formed by a convex lens of focal length 30 cm is 2 cm. If a (c) 12 cm (d) 9 cm
concave lens of focal length 20 cm is placed between the 39. In Young’s double slit experiment intensity at a point is (1/4)
of the maximum intensity. Angular position of this point is
convex lens and the image at a distance of 26 cm from the (2005S)
convex lens, calculate the new size of the image. (2003S) (a) sin–1(l/d) (b) sin–1(l/2d)
(a) 1.25 cm (b) 2.5 cm (c) sin–1(l/3d) (d) sin–1(l/4d)
40. A convex lens is in contact with concave lens. The magnitude
(c) 1.05 cm (d) 2 cm of the ratio of their focal length is 2/3. Their equivalent focal
33. A ray of light is incident at the glass-water interface at an length is 30 cm. What are their individual focal lengths?
(2005S)
angle i, it emerges finally parallel to the surface of water, (a) –15, 10 (b) –10, 15
then the value of m g would be (2003S) (c) 75, 50 (d) –75, 50
41. A container is filled with water (m = 1.33) upto a height of
33.25 cm. A concave mirror is placed 15 cm above the water
Air level and the image of an object placed at the bottom is
formed 25 cm below the water level. Focal length of the
r mirror is (2005S)
Water
r

Glass i
15 cm
(a) (4/3)sini (b) 1/sini
(c) 4/3 (d) 1
34. A beam of white light is incident on glass air interface from
glass to air such that green light just suffers total internal m=1.33 25 cm
reflection. The colors of the light which will come out to air 33.25 cm
are (2004S) I
O
(a) Violet, Indigo, Blue (b) All colors except green
(c) Yellow, Orange, Red (d) White light (a) 15 cm (b) 20 cm
(c) –18.31 cm (d) 10 cm
P-438 Topic-wise Solved Papers - PHYSICS
42. Focal length of the plano-convex lens is 15 cm. A small object 46. A ray of light traveling in water is incident on its surface
is placed at A as shown in the figure. The plane surface is open to air. The angle of incidence is q, which is less than
silvered. The image will form at (2006 - 3M, –1) the critical angle. Then there will be (2007)
(a) only a reflected ray and no refracted ray
(b) only a refracted ray and no reflected ray
(c) a reflected ray and a refracted ray and the angle between
them would be less than 180° – 2q
(d) a reflected ray and a refracted ray and the angle between
them would be greater than 180° – 2q
47. Two beams of red and violet colours are made to pass
A 20 cm separately through a prism (angle of the prism is 60°). In the
(a) 60 cm to the left of lens (b) 12 cm to the left of lens position of minimum deviation, the angle of refraction will
(c) 60 cm to the right of lens (d) 30 cm to the left of lens be (2008)
43. The graph shows relationship between object distance and (a) 30° for both the colours
image distance for a equiconvex lens. Then, focal length of (b) greater for the violet colour
the lens is (2006 - 3M, –1) (c) greater for the red colour
(d) equal but not 30° for both the colours
48. A light beam is travelling from Region I to IV (figure). The
31
n 0 n0
v cm refractive index in regionals I, II, III and IV are n 0 , ,
30
2 6

n0
and respectively. The angle of incidence q for which
8
10 the beam just misses entering region IV is – (2008)

Region I Region II Region III Region IV


0 (–9, +9)
u cm –31 –30 –20 –10
n0 n0 n0 n0
2 6 8
(a) 0.50 ± 0.05 cm (b) 0.50 ± 0.10 cm
(c) 5.00 ± 0.05 cm (d) 5.00 ± 0.10 cm 0 0.2m 0.6m
44. Rays of light from Sun falls on a biconvex lens of focal
(a) sin–1 (3/4) (b) sin–1 (1/8)
length f and the circular image of Sun of radius r is formed
–1
(c) sin (1/4) (d) sin–1 (1/3)
on the focal plane of the lens. Then
49. A ball is dropped from a height of 20 m above the surface of
(a) Area of image is pr2 and area is directly proportional of f
water in a lake. The refractive index of water is 4.3. A fish
(b) Area of image is pr2 and area is directly proportional to f 2 inside the lake, in the line of fall of the ball, is looking at the
(c) Intensity of image increases if f is increased ball. At an instant, when the ball is 12.8 m above the water
(d) If lower half of the lens is covered with black paper surface, the fish sees the speed of ball as [Take g = 10 m/s2.]
area will become half (2009)
45. In an experiment to determine the focal length (a) 9 m/s (b) 12 m/s
(f ) of a concave mirror by the u - v method, a student places (c) 16 m/s (d) 21.33 m/s
the object pin A on the principal axis at a distance x from the 50. A biconvex lens of focal length 15 cm is in front of a plane
pole P. The student looks at the pin and its inverted image mirror. The distance between the lens and the mirror is 10
from a distance keeping his/her eye in line with PA. When cm. A small object is kept at a distance of 30 cm from the
the student shifts his/her eye towards left, the image appears lens. The final image is (2010)
to the right of the object pin. Then, (2007) (a) virtual and at a distance of 16 cm from the mirror
(a) x < f (b) f < x < 2f (b) real and at a distance of 16 cm from the mirror
(c) virtual and at a distance of 20 cm from the mirror
(c) x = 2f (d) x > 2f
(d) real and at a distance of 20 cm from the mirror
RAY & WAVE OPTICS P-439
51. A light ray travelling in glass medium is incident on glass- 53. Young’s double slit experiment is carried out by using
air interface at an angle of incidence q. The reflected (R) and green, red and blue light, one color at a time. The fringe
transmitted (T) intensities, both as function of q, are plotted. widths recorded are bG, bR and bB, respectively. Then,
The correct sketch is (2011) (a) bG > bB > bR (b) bB > bG > bR (2012)
(c) bR > bB > bG (d) bR > bG > bB
1 ˆ
(a) 54. A ray of light travelling in the direction (i + 3 ˆj ) is incident
2
Intensity

on a plane mirror. After reflection, it travels along the direction


1 ˆ
(i - 3 ˆj ) . The angle of incidence is (JEE Adv. 2013)
2
(a) 30° (c) 60°
(b) 45° (d) 75°
55. In the Young’s double slit experiment using a monochromatic
(b)
light of wavelength l, the path difference (in terms of an
integer n) corresponding to any point having half the peak
intensity is (JEE Adv. 2013)
Intensity

l l
(a) (2 n + 1) (b) (2 n + 1)
2 4

l l
(c) (2 n + 1) (d) (2 n + 1)
8 16
(c)

1. In the Young’ s double slit experiment, the interference


pattern is found to have an intensity ratio between the bright
Intensity

and dark fringes as 9. This implies that (1982 - 3 Marks)


(a) the intensities at the screen due to the two slits are 5
units and 4 units respectively
(b) the intensities at the screen due to the two slits are 4
units and 1 units respectively
(c) the amplitude ratio is 3
(d) the amplitude ratio is 2
(d)
2. A convex lens of focal length 40 cm is in contact with a
concave lens of focal length 25 cm . The power of the
combination is (1982 - 3 Marks)
Intesitya

(a) – 1.5 dioptres (b) – 6.5 dioptres


(c) + 6.5 dioptres (d) + 6.67 dioptres
3. White light is used to illuminate the two slits in a Young’s
double slit experiment. The separation between the slits is b
and the screen is at a distance d (> b) from the slits. At a
52. A bi-convex lens is formed with two thin plano-convex point on the screen directly in front of one of the slits, certain
lenses as shown in the figure. Refractive index n of the wavelengths are missing. Some of these missing
first lens is 1.5 and that of the second lens is 1.2. Both wavelengths are (1984- 2 Marks)
the curved surface are of the same radius of curvature
R = 14 cm. For this bi-convex lens, for an object distance b2 2b 2
(a) l= (b) l=
of 40 cm, the image distance will be (2012) d d
b2 2b 2
(c) l= (d) l =
3d 3d
4. A converging lens is used to form an image on a screen.
When the upper half of the lens is covered by an opaque
screen (1986 - 2 Marks)
(a) half the image will disappear.
(b) complete image will be formed.
(a) -280.0 cm (b) 40.0 cm (c) intensity of the image will increase.
(c) 21.5 cm (d) 13.3 cm (d) intensity of the image will decrease.
P-440 Topic-wise Solved Papers - PHYSICS
5. A short linear object of length b lies along the axis of a 10. Two thin convex lenses of focal lengths f1 and f2 are separated
concave mirror of focal length f at a distanee u from the pole by a horizontal distance d (where d <f1, d< f2) and their centres
of the mirror. The size of the image is approximately equal to are displaced by a vertical separation D as shown in the fig.
(1993-2 Marks)
(1988 - 2 Mark)
y
1/ 2 1/ 2
æu - f ö æ f ö
(a) bç (b) bç D
è f ÷ø è u - f ø÷ o x

2 d
æu- f ö æ f ö
bç bç
è f ÷ø
(c) (d)
è u - f ø÷ Taking the origin of coordinates O, at the centre of the first
lens the x and y coordinates of the focal point of this lens
6. A beam of light consisting of red, green and blue colours is system, for a parallel beam of rays coming from the left, are
incident on a right angled prism, fig. The refractive indices given by:
of the material of the prism for the above red, green and blue f1 f 2
(a) x= ,y= D
wavelengths are 1.39, 1.44 and 1.47 respectively. The prism f1 + f 2
will (1989 - 2 Mark) f ( f + d) D
(b) x= 1 2 ,y=
f1 + f2 - d f1 + f2
f1 f 2 + d ( f1 - d ) D( f1 - d )
(c) x= ,y=
f1 + f 2 - d f1 + f2 - d

f1 f2 + d ( f1 - d )
(d) x= ,y = 0
f1 + f 2 - d
45°
11. Which of the following form(s) a virtual and erect image for
(a) separate part of the red colour from the green and blue all positions of the object ? (1996 - 2 Marks)
colours (a) Convex lens (b) Concave lens
(b) separate part of the blue colour from the red and green (c) Convex mirror (d) Concave mirror.
colours 12. A real image of a distant object is formed by a plano-convex
lens on its principal axis. Spherical aberration
(c) separate all the three colours from one another (1998 - 2 Marks)
(d) not separate even partially any colour from the other (a) is absent.
two colours. (b) is smaller if the curved surface of the lens faces the
7. An astronomical telescope has an angular magnification of object.
magnitude 5 for distant objects. The separation between the (c) is smaller if the plane surface of the lens faces the object.
objective and the eyepiece is 36 cm and the final image is (d) is the same whichever side of the lens faces the object
formed at infinity. The focal length f0 of the objective and the 13. A ray of light travelling in a transparent medium falls on a
focal length f0 of the eyepiece are (1989 - 2 Marks) surface separating the medium from air at an angle of incidence
(a) f0 = 45 cm and fe = –9 cm (b) f0 = 50 cm and fe = 10 cm of 45°. The ray undergoes total internal reflection. If n is the
refractive index of the medium with respect to air, select the
(c) f0 = 7.2 cm and fe = 5 cm (d) f0 = 30 cm and fe = 6 cm. possible value(s) of n from the following : (1998 - 2 Marks)
8. A thin prism P1 with angle 4° and made from glass of (a) 1.3 (b) 1.4 (c) 1.5 (d) 1.6
refractive index 1.54 is combined with another thin prism P2 14. A parallel monochromatic beam of light is incident normally
made from glass of refractive index 1.72 to produce on a narrow slit. A diffraction pattern is formed on a screen
dispersion without deviation. The angle of the prism P2 is placed perpendicular to the direction of the incident beam.
(1990 - 2 Marks) At the first minimum of the diffraction pattern, the phase
difference between the rays coming from the two edges of
(a) 5.33° (b) 4°
the slit is (1998 - 2 Marks)
(c) 3° (d) 2.6° (a) 0 (b) p/2 (c) p (d) 2p
9. A planet is observed by an astronomical refracting telescope 15. A concave mirror is placed on a horizontal table, with its axis
having an objective of focal length 16 m and an eyepiece of directed vertically upwards. Let O be the pole of the mirror
focal length 2 cm. (1992 - 2 Marks) and C its centre of curvature. A point object is placed at C.
It has a real image, also located at C. If the mirror is now
(a) The distance between the objective and the eyepiece
filled with water, the image will be. (1998 - 2 Marks)
is 16.02 m
(a) real, and will remain at C.
(b) The angular magnification of the planet is – 800 (b) real, and located at a point between C and ¥ .
(c) The image of the planet is inverted (c) virtual, and located at a point between C and O.
(d) The objective is larger then the eyepiece (d) real, and located at a point between C and O
RAY & WAVE OPTICS P-441
16. A spherical surface of radius of curvature R separates air 2. A ray of light is incident at an angle of 60° on one face of
(refractive index 1.0) from glass (refractive index 1.5). The prism which has an angle of 30°. The ray emerging out of
centre of curvature is in the glass. A point object P placed in the prism makes an angle of 30° with the incident ray. Show
air is found to have a real image Q in the glass. The line PQ that the emergent ray is perpendicular to the face through
cuts the surface at a point O, and PO = OQ. The distance PO which it emerges and calculate the refractive index of the
is equal to (1998 - 2 Marks) material of the prism. (1978)
(a) 5R (b) 3R 3. The radius of curvature of the convex face of a plane convex
(c) 2R (d) 1.5R lens is 12 cm and its refractive index is 1.5.
17. In a Young’s double slit experiment, the separation between (i) Find the focal length of this lens. (1979)
the two slits is d and the wavelength of the light is l. The (ii) Find the focal length if the plane surface of the lens is
intensity of light falling on slit 1 is four times the intensity of silvered.
light falling on slit 2. Choose the correct choice(s). (2008) (iii) At what distance from lens will parallel rays incident
(a) If d = l, the screen will contain only one maximum on the convex face converge?
(iv) Sketch the ray diagram to locate the image, when a
(b) If l < d < 2l, at least one more maximum (besides the
point object is placed on the axis, 20 cm from the lens.
central maximum) will be observed on the screen
(v) Calculate the image distance when the object is placed
(c) If the intensity of light falling on slit 1 is reduced so as in (iii)
that it becomes equal to that of slit 2, the intensities of 4. A rectangular block of glass is placed on a printed page
the observed dark and bright fringes will increase lying on a horizontal surface. Find the minimum value of the
(d) If the intensity of light falling on slit 2 is increased so refractive index of glass for which the letters on the page are
that it becomes equal to that of slit 1, the intensities of not visible from any of the vertical faces of the block. (1979)
the observed dark and bright fringes will increase 5. What is the relation between the refractive indices m1 and
18. A student performed the experiment of determination of focal m2, if the behaviour of light rays is as shown in the figure?
length of a concave mirror by u-v method using an optical (1979)
bench of length 1.5 meter. The focal length of the mirror
used is 24 cm. The maximum error in the location of the
m1 m m1 m2 m m2
image can be 0.2 cm. The 5 sets of (u, v ) values recorded by
the student (in cm) are :
(42, 56), (48, 48), (60, 40), (66, 33), (78, 39). The data set(s)
that cannot come from experiment and is (are) incorrectly
recorded, is (are) (2009) 6. An object is placed 21 cm in front of a concave mirror of
(a) (42, 56) (b) (48, 48) radius of curvature 10 cm. A glass slab of thickness 3 cm
(c) (66, 33) (d) (78, 39) and refractive index 1.5 is then placed close to the mirror in
19. A ray OP of monochromatic light is incident on the face AB the space between the object and the mirror.
of prism ABCD near vertex B at an incident angle of 60° (see Find the position of the final image formed. (You may take
figure). If the refractive index of the material of the prism is the distance of the near surface of the slab from the mirror to
3 , which of the following is (are) correct? (2010) be 1 cm. (1980)
7. A telescope has an objective of focal length 50 cm and an
eye piece of focal length 5 cm. The least distance of distinct
vision is 25 cm. The telescope is focussed for distinct vision
on a scale 200 cm away from the objective. Calculate
(i) the separation between the objective and the eye-piece.
(ii) the magnification produced. ( 1980)
8. The convex surface of a thin concavo-convex lens of glass
of refractive index 1.5 has a radius of curvature 20 cm. The
concave surface has a radius of curvature 60 cm. The convex
(a) The ray gets totally internally reflected at face CD side is silvered and placed on a horizontal surface.
(b) The ray comes out through face AD (1981- 6 Marks)
(c) The angle between the incident ray and the emergent
ray is 90°
(d) The angle between the incident ray and the emergent
ray is 120°
SUBJECTIVE PROBLEMS :
(i) Where should a pin be placed on the optic axis such
1. A pin is placed 10 cm in front of a convex lens of focal length that its image is formed at the same place?
20 cm, made a material of refractive index 1.5. The surface of (ii) If the concave part is filled with water of refractive
the lens farther away from the pin is silvered and has a index 4/3, find the distance through which the pin
radius of curvature are of 22 cm. Determine the position of should be moved so that the image of the pin again
the final image. Is the image real as virtual? (1978) coincides with the pin.
P-442 Topic-wise Solved Papers - PHYSICS
9. Screen S is illuminated by two point sources A and B. Another (i) n3 < n1 and (ii) n3 > n1 .
source C sends a parallel beam of light towards point P on
the screen (see figure). Line AP is normal to the screen and MEDIUM I
the lines AP, BP and CP are in one plane. The distance AP, (n1 )
D E
BP and CP are 3 m, 1.5 m and 1.5 m respectively. The radiant MEDIUM I
powers of sources A and B are 90 watts and 180 watts (n 3 ) F
G
respectively. The beam from C is of intensity 20 watts/m2. A B
Calculate the intensity at P on the screen.(1982 - 5 Marks) q
C MEDIUM II
(n 2 )

60°
× P 14. A right prism is to be made by selecting a proper material
A 60°
and the angles A and B ( B £ A) , as shown in Figure. It is
desired that a ray of light incident on the face AB emerges
parallel to the incident direction after two internal reflections.
B S
10. In Young’s double slit experiment using monochromatic
light the fringe pattern shifts by a certain distance on the
screen when a mica sheet of refractive index 1.6 and thickness A B
1.964 microns is introduced in the path of one of the
interfering waves. The mica sheet is then removed and the
distance between the slits and the screen is doubled. It is
found that the distance between successive maxima (or C
minima) now is the same as the observed fringe shift upon (i) What should be the minimum refractive index n for this
the introduction of the mica sheet. Calculate the wavelength to be possible ?
of the monochromatic light used in the experiment.
(1983 - 6 Marks) (ii) For n = 5 is it possible to achieve this with the angle
3
11. A plano convex lens has a thickness of 4 cm . When placed B equal to 30 degrees ? (1987 - 7 Marks)
on a horizontal table, with the curved surface in contact with 15. A parallel beam of light travelling in water (refractive index
it, the apparent depth of the bottom most point of the lens is = 4/3) is refracted by a spherical air bubble of radius 2 mm
found to be 3 cm. If the lens is inverted such that the plane situated in water. Assuming the light rays to be paraxial
face is in contact with the table, the apparent depth of the (1988 - 6 Marks)
centre of the plane face is found to be 25/8 cm. Find the focal (i) Find the position of the image due to refraction at the first
length of the lens. (1984- 6 Marks) surface and the position of the final image.
12. A beam of light consisting of two wavelengths, 6500Å and (ii) Draw a ray diagram showing the positions of both the
5200Å, is used obtain interference fringes in a Young’s images.
double slit experiment : (1985 - 6 Marks) 16. In a modified Young’s double slit experiment, a
(i) Find the distance of the third bright fringe on the screen monochromatic uniform and parallel beam of light of
from the central maximum for wavelength 6500Å. wavelength 6000 Å and intensity (10/ p ) W m–2 is incident
(ii) What is the least distance from the central maximum normally on two circular apertures A and B of radii 0.001 m
and 0.002 m respectively. A perfectly transparent film of
where the bright fringes due to both the wavelengths
thickness 2000 Å and refractive index 1.5 for the wavelength
coincide?
of 6000 Å is placed in front of aperture A, see fig. Calculate
The distance between the slits is 2 mm and the distance the power (in watts) received at the focal spot F of the lens.
between the plane of the slits and the screen is The lens is symmetrically placed with respect to the
120 cm. apertures. Assume that 10% of the power received by each
13. Monochromatic light is incident on a plane interface AB aperture goes in the original direction and is brought to the
between two media of refractive indices n1 and n2 (n2 > n1) focal spot. (1989 - 8 Mark)
at an angle of incidence q as shown in fig. The angle q is
infinitesimally greater than the critical angle for the two media
so that total internal reflection takes place. Now if a A
transparent slab DEFG of uniform thickness and of refractive F
index n3 is introduced on the interface (as shown in the
figure), show that for any value of n3 all light will ultimately B
be reflected back again into medium II. Consider separately
the cases (1986 - 6 Marks)
RAY & WAVE OPTICS P-443
17. A narrow monochromatic beam of light of intensity I is 20. In Fig., S is a monochromatic point source emitting light of
incident on a glass plate as shown in figure. Another identical wavelength l = 500nm. A thin lens of circular shape and
glass plate is kept close to the first one and parallel to it. focal length 0.10 m is cut into two identical halves L1 and L2
Each glass plate reflects 25 per cent of the light incident on by a plane passing through a diameter. The two halves are
placed symmetrically about the central axis SO with a gap of
it and transmits the remaining. Find the ratio of the minimum 0.5 mm. The distance along the axis from S to L1 and L2 is
and the maximum intensities in the interference pattern 0.15 m while that from L1 and L2 to O is 1.30 m. The screen at
formed by the two beams obtained after one reflection at O is normal to SO. (1993 - 5+1 Marks)
each plate. Glass (1990 - 7 Mark)
plate L1
0.5mm A
I S
O
Screen
L2
0.15m 1.30m
Glass
plate (i) If the third intensity maximum occurs at the point A on
18. Two parallel beams of light P and Q (separation d) containing the screen, find the distance OA.
(ii) If the gap between L1 and L2 is reduced from its original
radiations of wavelengths 4000 Å and 5000 Å (which are
value of 0.5mm, will the distance OA increase, decrease,
mutually coherent in each wavelength separately) are or remain the same?
incident normally on a prism as shown in fig. The refractive 21. An image Y is formed of point object X by a lens whose
index of the prism as a function of wavelength is given by optic axis is AB as shown in figure. Draw a ray diagram to
b locate the lens and its focus. If the image Y of the object X
the relation. m(l) = 1.20 + where l is in Å and b is is formed by a concave mirror (Having the same axis as AB)
l2 instead of lens, draw another ray diagram to locate the mirror
positive constant. The value of b is such that the condition and its focus. Write down the steps of construction of the
for total reflection of the face AC is just satisfied for one ray diagrams. (1994 - 6 Marks)
wave length and is not satisfied for the other. X
A (1991 - 2 + 2 + 4 Marks)
A B
P q sin q = 0.8
Y
d
22. A ray of light travelling in air is incident at grazing angle
90° (incident angle @ 90° ) on a long rectangular slab of a
Q transparent medium of thickness t = 1.0 m (see figure below).
C
B The point of incidence is the origin A(0, 0). The medium has
(a) Find the value of b. a variable index of refraction n(y) given by
-3 / 2
(b) Find the deviation of the beams transmitted through n( y ) = [ky 3 / 2 + 1]1/ 2 , where k = 1.0 (metre)
the face AC y
(c) A convergent lens is used to bring these transmitted
beams into focus. If the intensities of transmission form AIR P( x1 , y1 )
the face AC, are 41 and I respectively, find the resultant
intensity at the focus. t =1.0m B(x, y)
19. Light is incident at an angle a on one planar end of a MEDIUM
transparent cylindrical rod of refractive index m. Determine x
A(0, 0) AIR
the least value of m so that the light entering the rod does
The refractive index of air is 1.0. (1995 - 10 Marks)
not emerge from the curved surface of rod irrespective of (a) Obtain a relation between the slope of the trajectory of
the value of a (1992 - 8 Marks) the ray at a pint B(x, y) in the medium and the incident
angle at that point.
(b) Obtain an equation for the trajectory y(x) of the ray in
m the medium.
(c) Determine the coordinates ( x1 , y1 ) of the point P,
a where the ray intersects the upper surface of the slab-
air boundary.
(d) Indicate the path of the ray subsequently.
P-444 Topic-wise Solved Papers - PHYSICS
23. Angular-width of central maximum in the Fraunhofer 28. A thin equiconvex lens of glass of refractive index m = 3/2
diffraction pattern of a slit is measured. The slit is illuminated and of focal length 0.3 m in air is sealed into an opening at
by light of wavelength 6000 Å. When the slit is illuminated one end of a tank filled with water (m = 4/3). On the opposite
by light of another wavelength, the angular-width decreases side of the lens, a mirror is placed inside the tank on the tank
by 30%. Calculate the wavelength of this light. The same wall perpendicular to the lens axis, as shown in figure. The
decrease in the angular-width of central maximum is obtained separation between the lens and the mirror is 0.8 m. A small
when the original apparatus is immersed in a liquid. Find object is placed outside the tank in front of the lens at a
refractive index of the liquid. (1996 -2 Marks) distance of 0.9 m from the lens along its axis. Find the position
24. A right angled prism (45° –90°–45°) of refractive index n has
(relative to the lens) of the image of the object formed by the
a plate of refractive index n1(n1 < n) cemented to its diagonal
face. The assembly is in air. A ray is incident on AB. system. (1997C - 5 Marks)
0.9 m 0.8 m
A

n1
n

29. In Young’s experiment, the upper slit is covered by a thin


B B glass plate of refractive index 1.4 while the lower slit is
(i) Calculate the angle of incidence at AB for which the covered by another glass plate, having the same thickness
ray strikes the diagonal face at the critical angle. as the first one but having refractive index 1.7. Interference
(ii) Assuming n = 1.352 calculate the angle of incidence at pattern is observed using light of wavelength 5400 Å. It is
AB for which the refracted ray passes through the found that the point P on the screen where the central
diagonal face undeviated. (1996 - 3 Marks) maximum (n = 0) fells before the glass plates were inserted
25. A double-slit apparatus is immersed in a liquid of refractive now has 3/4 the original intensity. It is further observed that
index 1.33. It has slit separation of 1mm, and distance what used to be the fifth maximum earlier, lies below the
between the plane of slits and screen is 1.33 m. The slits are point P while the sixth minimum lies above P. Calculate the
illuminated by a parallel beam of light whose wavelength in thickness of the glass plate. (Absorption of light by glass
air is 6300 Å. (1996 - 3 Marks) plate may be neglected.) (1997 - 5 Marks)
(i) Calculate the fringe-width. 30. A prism of refractive index n1 and another prism of refractive
(ii) One of the slits of the apparatus is covered by a thin index n2 are stuck together without a gap as shown in Figure.
glass sheet of refractive index 1.53. Find the smallest The angles of the prisms are as shown. n1 and n2 depend on
thickness of the sheet to bring the adjacent minimum l, the wavelength of light, according to
on the axis.
26. A thin plano-convex lens of focal length f is split into two 10.8 ´ 10 4 1.80 ´ 10 4
halves: one of the halves is shifted along the optical axis . n1 = 1.20 + an d n 2 = 1.45 +
The separation between object and image planes is 1.8 m. l2 l2
The magnification of the image formed by one of the half- where l is in nm. (1998 - 8 Marks)
lenses is 2. Find the focal-length of the lens and separation
between the two halves. Draw the ray diagram for image D
formation. (1996 - 5 Marks)

C
70°

O n2

1.8 m n1
20
27. In young’s experiment, the source is red light of wavelength °
7 × 10–7 m. When a thin glass plate of refractive index 1.5 at 60° 40°
this wavelength is put in the path of one of the interfering A
beams, the central bright fringe shifts by 10–3 m to the B
position previously occupied by the 5th bright fringe. Find (a) Calculate the wavelength l0 for which rays incident at
the thickness of the plate. When the source is now changed any angle on the interface BC pass through without
to green light of wavelength 5 ×10–7 m, the central fring bending at that interface.
shifts to a position initially occupied by the 6th bright (b) For light of wavelength l0, find the angle of incidence
fringe due to red light. Find the refractive index of glass for
i on the face AC such that the deviation produced by
green light. Also estimate the change in fringe width due to
the change in wavelength. (1997C - 5 Marks) the combination of prisms is minimum.
RAY & WAVE OPTICS P-445
31. A coherent parallel beam of microwaves of wavelength l = 34. A quarter cylinder of radius R and refractive index 1.5 is
0.5 mm falls on a Young’s double slit apparatus. The placed on a table. A point object P is kept at a distance of mR
separation between the slits is 1.0 mm. The intensity of
from it. Find the value of m for which a ray from P will emerge
microwaves is measured on a screen placed parallel to the
plane of the slits at a distance of 1.0 m from it as shown in parallel to the table as shown in Figure. (1999 - 5 Marks)
Fig.
y

d = 1.0 mm
30° x P
mR R
D = 1.0m
35. (a) A convex lens of focal length 15 cm and a cancave
Screen mirror of focal length 30 cm are kept with their optic
(a) If the incident beam falls normally on the double slit axes PQ and RS parallel but separated in vertical
apparatus, find the y-coordinates of all the interference direction by 0.6 cm as shown. The distance between
minima on the screen. the lens and mirror is 30 cm. An upright object AB of
(b) If the incident beam makes an angle of 30° with the x height 1.2 cm is placed on the optic axis PQ of the lens
axis (as in the dotted arrow shown in Figure), find the
y-coordinate of the first minima on either side of the at a distance of 20 cm from the lens. If A' B ' is the
central maximum. (1998 - 8 Marks) image after refraction from the lens and reflection from
32. The Young’s double slit experiment is done in a medium of the mirror, find the distance of A' B ' from the pole of
refractive index 4/3. A light of 600 nm wavelength is falling
on the slits having 0.45 mm separation. The lower slit S2 is the mirror and obtain its magnification. Also locate
covered by a thin glass sheet of thickness 10.4 mm and position of A ' and B ' with respect to the optic axis
refractive index 1.5. The interference pattern is observed on RS. (2000 - 6 Marks)
a screen placed 1.5 m from the slits as shown in Figure.
(1999 - 10 Marks)

S1
S* O
S2

(b) A glass plate of refractive index 1.5 is coated with a


thin layer of thickness t and refractive index 1.8. Light
of wavelength l travelling in air is incident normally on
(a) Find the location of the central maximum (bright fringe the layer. It is partly reflected at the upper and the
with zero path difference) on the y – axis. lower surface of the layer and the two reflected rays
(b) Find the light intensity at point O relative to the interfere. Write the condition for their constructive
maximum fringe intensity. interference. If l = 648 nm, obtain the least value of t
(c) Now, if 600 nm light is replaced by white light of range for which the rays interfere constructively.
400 to 700 nm, find the wavelengths of the light that
form maxima exactly at point O. (2000 - 4 Marks)
[All wavelengths in this problem are for the given 36. The refractive indices of the crown glass for blue and red
medium of refractive index 4/3. Ignore dispersion] lights are 1.51 and 1.49 respectively and those of flint glass
33. The x – y plane is the boundary between two transparent
are 1.77 and 1.73 respectively. An isosceles prism of angle
media. Medium –1 with z ³ 0 has a refractive index 2 6o is made of crown glass. A beam of white light is incident
and medium –2 with z £ 0 has a refractive index 3 . A ray at a small angle on this prism. The other flint glass isosceles
prism is combined with the crown glass prism such that
of light in medium –1 given by the vector A = 6 3i + 8 3 j
there is no deviation of the incident light. Determine the
–10 k is incident on the plane of separation. Find the unit
angle of the flint glass prism. Calculate the net dispersion of
vector in the direction of the refracted ray in medium –2.
(1999 - 10 Marks) the combined system. (2001 - 5 Marks)
P-446 Topic-wise Solved Papers - PHYSICS
37. A vessel ABCD of 10 cm width has two small slits S1 and S2 40. Find the focal length of the lens shown in the figure. The
sealed with identical glass plates of equal thickness. The radii of curvature of both the surfaces are equal to R.
distance between the slits is 0.8 mm. POQ is the line
perpendicular to the plane AB and passing through O, the
middle point of S1 and S2. A monochromatic light source is m1 < m 2 < m 3
kept at S, 40 cm below P and 2 m from the vessel, to illuminate
the slits as shown in the figure below. Calculate the position m1 m3
m2
of the central bright fringe on the other wall CD with respect
to the line OQ. Now, a liquid is poured into the vessel and
filled upto OQ. The central bright fringe is found to be at Q. R (2003 - 2 Marks)
R
Calculate the refractive index of the liquid.(2001-5 Marks)
A
D

S1
P 41. Shown in the figure is a prism of angle 30º and refractive
Q
O
40 cm S2 index m p = 3 . Face AC of the prism is covered with a thin
S 2m 10 cm
film of refractive index mf = 2.2. A monochromatic light of
B C wavelength l = 550 nm fall on the face AB at an angle of
38. A thin biconvex lens of refractive index 3/2 is placed on a incidence of 60º. (2003 - 4 Marks)
A
horizontal plane mirror as shown in the figure. The space
between the lens and the mirror is then filled with water of
refractive index 4/3. It is found that when a point object is
placed 15 cm above the lens on its principal axis, the object
coincides with its own image. On repeating with another 30º
liquid, the object and the image again coincide at a distance 60º
25 cm from the lens. Calculate the refractive index of the
liquid. (2001-5 Marks)

mp = 3
B C m f = 2.2
39. A point source S emitting light of wavelength 600 nm is
placed at a very small height h above a flat reflecting surface Calculate
AB (see figure). The intensity of the reflected light is 36% (a) angle of emergence.
of the incident intensity. Interference fringes are observed (b) minimum value of thickness t so that intensity of
on a screen placed parallel to the reflecting surface at a emergent ray is maximum.
very large distance D from it. (2002 - 5 Marks) 42. A ray is incident on a medium consisting of two boundaries,
one plane and other curved as shown in the figure. The
P plane surface makes an angle 60º with horizontal and curved
Screen
surface has radius of curvature 0.4 m. The refractive indices
of the medium and its environment are shown in the figure.
If after refraction at both the surfaces the ray meets principle
axis at P, find OP. (2004 - 2 Marks)
D

n=Ö2
45º O P
n=1 n=1.514

h S 60º
A B 43. In YDSE a light containing two wavelengths 500 nm and
700 nm are used. Find the minimum distance where maxima
(a) What is the shape of the interference fringes on the of two wavelengths coincide. Given D/d = 103, where D is
screen?
the distance between the slits and the screen and d is the
(b) Calculate the ratio of the minimum to the maximum
distance between the slits. (2004 - 4 Marks)
intensities in the interference fringes formed near the
point P (shown in the figure). 44. An object is moving with velocity 0.01 m/s towards a convex
lens of focal length 0.3 m. Find the magnitude of rate of
(c) If the intensity at point P corresponds to a maximum,
calculate the minimum distance through which the separation of image from the lens when the object is at a
reflecting surface AB should be shifted so that the distance of 0.4 m from the lens. Also calculate the magnitude
intensity at P again becomes maximum. of the rate of change of the lateral magnification.
(2004 - 4 Marks)
RAY & WAVE OPTICS P-447
45. What will be the minimum angle of incidence such that the
B D
total internal reflection occurs on both the surfaces?
60° 60°
m1 = 2
m2 = 2 60° 60°
(2005 - 2 Marks)
m3 = 3 A C E
(a) the angle of incidence, so that the emergent ray from
46. Two identical prisms of refractive index 3 are kept as the first prism has minimum deviation.
shown in the figure. A light ray strikes the first prism at face (b) through what angle the prism DCE should be rotated
AB. Find, (2005 - 4 Marks) about C so that the final emergent ray also has
minimum deviation.
MATCH THE FOLLOWING :
MUTLIPLE CHOICE QUESTIONS WITH ONE CORRECT
Directions (Q. No. 1 to Q. No. 4) Each question contains statements given in two columns, which have to be matched. The
statements in Column-I are labelled A, B, C and D, while the statements in Column-II are labelled p, q, r, s and t. Any given
statement in Column-I can have correct matching with ONE OR MORE statement(s) in Column-II. The appropriate bubbles
corresponding to the answers to these questions have to be darkened as illustrated in the following example :
If the correct matches are A-p, s and t; B-q and r; C-p and q; and D-s then the correct darkening of bubbles will look like the given.
p q r s t
A p q r s t
B p q r s t
C p q r s t
D p q r s t

1. A simple telescope used to view distant objects has eyepiece and objective lens of focal lengths fe and fo, respectively. Then
(2006 - 6M)
Column I Column II
(A) Intensity of light received by lens (p) Radius of aperture
(B) Angular magnification (q) Dispersion of lens
(C) Length of telescope (r) Focal length of objective lens and eyepiece lens
(D) Sharpness of image (s) Spherical aberration
2. An optical component and an object S placed along its optic axis are given in Column I. The distance between the object and the
component can be varied. The properties of images are given in Column II. Match all the properties of images from Column II with
the appropriate components given in Column I. Indicate your answer by darkening the appropriate bubbles of the 4 × 4 matrix
given in the ORS. (2008)
Column I Column II

S
(A) (p) real image

S
(B) (q) virtual image

S
(C) (r) magnified image

S
(D) (s) image at infinity
P-448 Topic-wise Solved Papers - PHYSICS
3. Column-I shows four situations of standard Young’s double slit arrangement with the screen placed far away from the slits S1
and S2. In each of these cases S1P0 = S2P0, S1P1-S2P1= l/4 and S1P2– S2P2= l/3, where l is the wavelength of the light used. In
the cases B, C and D, a transparent sheet of refractive index m and thickness t is pasted on slit S2. The thicknesses of the sheets
are different in different cases. The phase difference between the light waves reaching a point P on the screen from the two slits
is denoted by d (P) and the intensity by I(P). Match each situation given in Column-I with the statetment(s) in Column-II valid
for that situation. (2009)
Column-I Column-II
(A) (p) d(P0) = 0

P2
S2 P1
P0 (q) d (P1) = 0
S1

P2
S2 P1
P0
(B) (m – 1) t = l/4 (r) I (P1) = 0
S1

P2
S2
P1
(C) (m – 1) t = l/2 P0 (s) I(P0) > I (P1)
S1

P2
S2 P1
P0
(D) (m – 1) t = 3l/4 (t) I(P2) > I (P1)
S1

4. Two transparent media of refractive indices m1 and m3 have a solid lens shaped transparent material of refractive index m 2 between
them as shown in figures in Column II. A ray traversing these media is also shown in the figures. In Column I different relationships
between m1, m 2, and m3 are given. Match them to the ray diagrams shown in Column II. (2010)
Column I Column II

(A) m1 < m 2 (p) m3 m2 m1

(B) m1 > m 2 (q) m 3 m2 m1


RAY & WAVE OPTICS P-449

(C) m 2 = m3 (r)
m3 m2 m1

(D) m 2 > m 3 (s)


m3 m2 m1

(t)
m3 m2 m1

DIRECTION (Q. NO. 5) Following question has matching lists. The codes for the lists have choices (a), (b), (c) and (d) out of which
ONLY ONE is correct.
5. A right angled prism of refractive index m1 is placed in a rectangular block of refractive index m2, which is surrounded by a medium
of refractive index m3, as shown in the figure. A ray of light 'e' enters the rectangular block at normal incidence. Depending upon
the relationships between m1, m2 and m3, it takes one of the four possible paths 'ef', 'eg', 'eh' or 'ei'.

e 45° g

m1 h
i

m3
m2

Match the paths in List I with conditions of refractive indices in List II and select the correct answer using the codes given below the
lists: (JEE Adv. 2013)
List I List II
P. e®f 1. m1 > 2m 2
Q. e®g 2. m2 > m1 and m2 > m3
R. e®h 3. m1 = m2
S. e®i 4. m2 < m1< 2m 2 and m2 > m3
Codes:
P Q R S
(a) 2 3 1 4
(b) 1 2 4 3
(c) 4 1 2 3
(d) 2 3 4 1
P-450 Topic-wise Solved Papers - PHYSICS
4. For light incident from air on a meta-material, the
COMPREHENSION BASED Q UESTIONS : appropriate ray diagram is
PASSAGE - 1
The figure shows a surface XY separating two transparent media,
medium-1 and medium-2. The line ab and cd represent waveforms
of a light wave travelling in medium-1 and incident on XY. The q1
lines ef and gh represent wavefronts of the light wave in medium-
2 after refraction. (2007)
Air
b d (a) Meta-material

q2

medium-1
a c
X Y
f h
medium-2

e g (b)
1. Light travels as a
(a) parallel beam in each medium
(b) convergent beam in each medium
(c) divergent beam in each medium
(d) divergent beam in one medium and convergent beam
in the other medium.
2. The phases of the light wave at c, d, e and f are fc, fd, fe and q1
ff respectively. It is given that fc ¹ ff.
(a) fc cannot be equal to fd Air
(b) fd can be equal to fe (c)
(c) (fd – ff) is equal to (fc – fe) Meta-material
(d) (fd – fc) is not equal to (ff – fe) q2
3. Speed of light is
(a) the same in medium-1 and medium-2
(b) larger in medium-1 than in medium-2
(c) larger in medium-2 than in medium-1
(d) different at b and d.
PASSAGE-2 q1
Most materials have the refractive index, n > 1. So, when
a light ray from air enters a naturally occurring material, Air
sin q1 n2 (d) Meta-material
then by Snell’s law, = , it is understood that the q2
sin q2 n1
refracted ray bends towards the normal. But it never emerges
on the same side of the normal as the incident ray. According
to electromagnetism, the refractive index of the medium is 5. Choose the correct statement.
(a) The speed of light in the meta-material is v = c|n|
given by the relation, n = c/ v = ± er mr , where c is the speed
c
of electromagnetic waves in vacuum, v its speed in the (b) The speed of light in the meta-material is v =
medium, e r and mr are the relative permittivity and n
permeability of the medium respectively. (c) The speed of light in the meta-material is v = c.
In normal materials, both e r and mr , are positive, (d) The wavelength of the light in the meta-material
(lm ) is given by lm = lair |n|, where lair is
implying positive n for the medium. When both e r and mr wavelength of the light in air.
are negative, one must choose the negative root of n. Such
negative refractive index materials can now be artificially ASSERTION & REASON TYPE QUESTIONS :
prepared and are called meta-materials. They exhibit 1. STATEMENT-1 (2007)
significantly different optical behavior, without violating
The formula connecting u, v and f for a spherical mirror is
any physical laws. Since n is negative, it results in a change
in the direction of propagation of the refracted light. However, valid for mirrors whose sizes are very small compared to
similar to normal materials, the frequency of light remains their radii of curvature.
unchanged upon refraction even in meta-materials. (2012) because
RAY & WAVE OPTICS P-451
STATEMENT-2 3. Image of an object approaching a convex mirror of radius of
Laws of reflection are strictly valid for plane surfaces, but curvature 20 m along its optical axis is observed to move
not for large spherical surfaces. 25 50
(a) Statement-1 is True, Statement-2 is True; Statement-2 from m to m in 30 seconds. What is the speed of the
3 7
is a correct explanation for Statement-1 object in km per hour? (2010)
(b) Statement-1 is True, Statement-2 is True; Statement-2 4
is NOT a correct explanation for Statement-1 4. Water (with refractive index = ) in a tank is 18 cm deep. Oil
3
(c) Statement-1 is True, Statement-2 is False
(d) Statement-1 is False, Statement-2 is True 7
of refractive index lies on water making a convex surface
4
INTEGER VALUE CORRECT TYPE : of radius of curvature ‘R = 6 cm’ as shown. Consider oil to
1. The focal length of a thin biconvex lens is 20 cm. When an act as a thin lens. An object ‘S’ is placed 24 cm above water
object is moved from a distance of 25 cm in front of it to 50 surface. The location of its image is at ‘x’ cm above the
bottom of the tank. Then ‘x’ is (2011)
cm, the magnification of its image changes from m25 to m50.
m25
The ratio is (2010)
m50
2. A large glass slab (m = 5 / 3) of thickness 8 cm is placed over
a point source of light on a plane surface. It is seen that light
emerges out of the top surface of the slab from a circular
area of radius R cm. What is the value of R? (2010)

1. An astronomical telescope has a large aperture to 7. To demonstrate the phenomenon of interference, we require
(a) reduce spherical aberration [2002] two sources which emit radiation [2003]
(b) have high resolution (a) of nearly the same frequency
(c) increase span of observation (b) of the same frequency
(d) have low dispersion. (c) of different wavelengths
2. If two mirrors are kept at 60° to each other, then the number (d) of the same frequency and having a definite phase
of images formed by them is [2002] relationship
8. The image formed by an objective of a compound
(a) 5 (b) 6
microscope is [2003]
(c) 7 (d) 8 (a) virtual and diminished
3. Electromagnetic waves are transverse in nature is evident (b) real and diminished
by [2002] (c) real and enlarged
(a) polarization (b) interference (d) virtual and enlarged
(c) reflection (d) diffraction 9. To get three images of a single object, one should have two
4. Wavelength of light used in an optical instrument are plane mirrors at an angle of [2003]
l1 = 4000 Å and l 2 = 5000 Å , then ratio of their (a) 60º (b) 90º (c) 120º (d) 30º
10. A light ray is incident perpendicularly to one face of a 90°
respective resolving powers (corresponding to l1 and l 2 ) prism and is totally internally reflected at the glass-air
is [2002] interface. If the angle of reflection is 45°, we conclude that
(a) 16 : 25 (b) 9 : 1 the refractive index n [2004]
(c) 4 : 5 (d) 5 : 4.
5. Which of the following is used in optical fibres?
(a) total internal reflection [2002]
(b) scattering
45°
(c) diffraction 45°
(d) refraction.
6. Consider telecommunication through optical fibres.
Which of the following statements is not true? [2003] 45°
(a) Optical fibres can be of graded refractive index
(b) Optical fibres are subject to electromagnetic 1
interference from outside (a) n> (b) n> 2
2
(c) Optical fibres have extremely low transmission loss
(d) Optical fibres may have homogeneous core with a 1
(c) n< (d) n< 2
suitable cladding. 2
P-452 Topic-wise Solved Papers - PHYSICS
11. A plano convex lens of refractive index 1.5 and radius of 20. When an unpolarized light of intensity I 0 is incident on a
curvature 30 cm. Is silvered at the curved surface. Now this
polarizing sheet, the intensity of the light which does not
lens has been used to form the image of an object. At what
distance from this lens an object be placed in order to have a get transmitted is [2005]
real image of size of the object [2004] 1 1
(a) 60 cm (b) 30 cm (a) I0 (b) I0
4 2
(c) 20 cm (d) 80 cm
12. The angle of incidence at which reflected light is totally (c) I 0 (d) zero
polarized for reflection from air to glass (refractive index n), 21. The refractive index of a glass is 1.520 for red light and 1.525
is [2004] for blue light. Let D1 and D2 be angles of minimum deviation
for red and blue light respectively in a prism of this glass.
(a) tan -1 (1/ n) (b) sin -1 (1/ n) Then, [2006]
(c) sin -1 (n) (d) tan -1 (n) (a) D1 < D2
(b) D1 = D2
13. The maximum number of possible interference maxima for (c) D1 can be less than or greater than D2 depending upon
slit-separation equal to twice the wavelength in Young’s the angle of prism
double-slit experiment is [2004] (d) D1 > D2
(a) three (b) five 22. In a Young’s double slit experiment the intensity at a point
(c) infinite (d) zero
14. An electromagnetic wave of frequency r = 3.0 MHz l
where the path difference is (l being the wavelength of
passes from vacuum into a dielectric medium with 6
permittivity Î= 4.0 . Then [2004] I
(a) wave length is halved and frequency remains light used) is I. If I0 denotes the maximum intensity, is
I0
unchanged
(b) wave length is doubled and frequency becomes half equal to [2007]
(c) wave length is doubled and the frequency remains 3 1
unchanged (a) (b)
4 2
(d) wave length and frequency both remain unchanged.
15. A fish looking up through the water sees the outside world 3 1
contained in a circular horizon. If the refractive index of (c) (d)
2 2
4 23. Two lenses of power –15 D and +5 D are in contact with
water is and the fish is 12 cm below the surface, the
3 each other. The focal length of the combination is
radius of this circle in cm is [2005] (a) + 10 cm (b) – 20 cm [2007]
(c) – 10 cm (d) + 20 cm
36 24. In an experiment, electrons are made to pass through a narrow
(a) (b) 36 7
7 slit of width ‘d’ comparable to their de Broglie wavelength.
They are detected on a screen at a distance ‘D’ from the slit
(c) 4 5 (d) 36 5 (see figure).
16. Two point white dots are 1 mm apart on a black paper. They
are viewed by eye of pupil diameter 3 mm. Approximately,
what is the maximum distance at which these dots can be d
y=0
resolved by the eye? [Take wavelength of light = 500 nm]
(a) 1 m (b) 5 m [2005]
(c) 3 m (d) 6 m D
17. A thin glass (refractive index 1.5) lens has optical power of Which of the following graphs can be expected to represent
– 5 D in air. Its optical power in a liquid medium with refractive the number of electrons ‘N’ detected as a function of the
index 1.6 will be [2005] detector position ‘y’(y = 0 corresponds to the middle of the
(a) – 1D (b) 1 D slit) [2008]
(c) – 25 D (d) 25 D y y
18. A Young’s double slit experiment uses a monochromatic
source. The shape of the interference fringes formed on a
screen is [2005] (a) N d (b) N d
(a) circle (b) hyperbola
(c) parabola (d) straight line
19. If I 0 is the intensity of the principal maximum in the single
y y
slit diffraction pattern, then what will be its intensity when
the slit width is doubled? [2005]
(a) 4 I 0 (b) 2 I 0 (c) N d (d) N d
I0
(c) (d) I0
2
RAY & WAVE OPTICS P-453
25. A student measures the focal length of a convex lens by Directions : Questions number 30-32 are based on the following
putting an object pin at a distance ‘u’ from the lens and paragraph. An initially parallel cylindrical beam travels in a medium
measuring the distance ‘v’ of the image pin. The graph
between ‘u’ and ‘v’ plotted by the student should look like of refractive index m (I) =m0 + m2 I, where m0 and m2 are positive
[2008] constants and I is the intensity of the light beam. The intensity of
v(cm) the beam is decreasing with increasing radius.
v(cm)
30. As the beam enters the medium , it will [2010]
(a) (b) (a) diverge
O O u(cm) (b) converge
u(cm)
v(cm)
(c) diverge near the axis and converge near the periphery
v(cm) (d) travel as a cylindrical beam
31. The initial shape of the wavefront of the beam is [2010]
(c) (d) (a) convex
O O u(cm)
u(cm) (b) concave
26. An experment is performed to find the refractive index of
(c) convex near the axis and concave near the periphery
glass using a travelling microscope. In this experiment
distances are measured by [2008] (d) planar
(a) a vernier scale provided on the microscope 32. The speed of light in the medium is [2010]
(b) a standard laboratory scale
(c) a meter scale provided on the microscope (a) minimum on the axis of the beam
(d) a screw gauge provided on the microscope (b) the same everywhere in the beam
27. A mixture of light, consisting of wavelength 590 nm and an (c) directly proportional to the intensity I
unknown wavelength, illuminates Young’s double slit and
gives rise to two overlapping interference patterns on the (d) maximum on the axis of the beam
screen. The central maximum of both lights coincide. Further, 33. Let the x-z plane be the boundary between two transparent
it is observed that the third bright fringe of known light
coincides with the 4th bright fringe of the unknown light. media. Medium 1 in z ³ 0 has a refractive index of 2 and
From this data, the wavelength of the unknown light is:
[2009] medium 2 with z < 0 has a refractive index of 3 . A ray of
r
(a) 885.0 nm (b) 442.5 nm light in medium 1 given by the vector A = 6 3iˆ + 8 3 ˆj - 10kˆ
(c) 776.8 nm (d) 393.4 nm
28. A transparent solid cylindrical rod has a refractive index of is incident on the plane of separation. The angle of refraction
in medium 2 is: [2011]
2
. It is surrounded by air. A light ray is incident at the (a) 45° (b) 60°
3
mid-point of one end of the rod as shown in the figure. (c) 75° (d) 30°
34. This question has a paragraph followed by two statements,
Statement – 1 and Statement – 2. Of the given four
q alternatives after the statements, choose the one that
describes the statements.
The incident angle q for which the light ray grazes along the
wall of the rod is : [2009] A thin air film is formed by putting the convex surface of a
plane-convex lens over a plane glass plate. With
æ ö æ 2 ö
(a) sin -1 ç 3 2 ÷ (b) sin -1 ç ÷ monochromatic light, this film gives an interference pattern
è ø è 3ø due to light reflected from the top (convex) surface and the
æ 1 ö
( )
bottom (glass plate) surface of the film. [2011]
(c) sin -1 ç ÷ (d) sin
-1 1
è 3 ø 2 Statement – 1 : When light reflects from the air-glass plate
29. In an optics experiment, with the position of the object fixed, interface, the reflected wave suffers a phase change of p.
a student varies the position of a convex lens and for each Statement – 2 : The centre of the interference pattern is
position, the screen is adjusted to get a clear image of the
object. A graph between the object distance u and the image dark.
distance v, from the lens, is plotted using the same scale for (a) Statement – 1 is true, Statement – 2 is true, Statement –
the two axes. A straight line passing through the origin and 2 is the correct explanation of Statement – 1.
making an angle of 45° with the x-axis meets the experimental
curve at P. The coordinates of P will be : [2009] (b) Statement – 1 is true, Statement – 2 is true, Statement –
2 is not the correct explanation of Statement – 1.
æ f fö
(a) çè 2 , 2 ÷ø (b) ( f, f ) (c) Statement – 1 is false, Statement – 2 is true.
(d) Statement – 1 is true, Statement – 2 is false.
(c) ( 4 f, 4 f ) (d) ( 2 f, 2 f )
P-454 Topic-wise Solved Papers - PHYSICS
35. A car is fitted with a convex side-view mirror of focal length 39. When monochromatic red light is used instead of blue
20 cm. A second car 2.8 m behind the first car is overtaking light in a convex lens, its focal length will [2011RS]
the first car at a relative speed of 15 m/s. The speed of the (a) increase
image of the second car as seen in the mirror of the first one (b) decrease
is : [2011]
(c) remain same
1 (d) does not depend on colour of light
(a) m/s (b) 10 m/s
15 40. Statement - 1: On viewing the clear blue portion of the
sky through a Calcite Crystal, the intensity of transmitted
1
(c) 15 m/s (d) m/s light varies as the crystal is rotated.
10
Statement - 2: The light coming from the sky is polarized
36. At two points P and Q on screen in Young’s double slit due to scattering of sun light by particles in the atmo-
experiment, waves from slits S1 and S2 have a path sphere. The scattering is largest for blue light. [2011RS]
(a) Statement -1 is true, statement-2 is false.
l
difference of 0 and , respectively. The ratio of intensities (b) Statement-1 is true, statement-2 is true, statement-2
4
is the correct explanation of statement-1
at P and Q will be : [2011RS]
(c) Statement-1 is true, statement-2 is true, statement-2
(a) 2 : 1 (b) 2 :1 is not the correct explanation of statement-1
(c) 4 : 1 (d) 3 : 2 (d) Statement-1 is false, statement-2 is true.
37. In a Young’s double slit experiment, the two slits act as 41. An electromagnetic wave in vacuum has the electric
r r
coherent sources of wave of equal amplitude A and and magnetic field E and B , which are always
wavelength l . In another experiment with the same perpendicular to each other. The direction of
arrangement the two slits are made to act as incoherent r
polarization is given by X and that of wave
sources of waves of same amplitude and wavelength. If r
propagation by k . Then [2012]
the intensity at the middle point of the screen in the first
r r r r r
I1 (a) X || B and k || B ´ E
case is I1 and in the second case is I 2 , then the ratio r r r r r
I2
(b) X || E and k || E ´ B
is [2011RS] r r r r r
(c) X || B and k || E ´ B
(a) 2 (b) 1
r r r r r
(c) 0.5 (d) 4 (d) X || E and k || B ´ E
38. A beaker contains water up to a height h1 and kerosene of 42. In Young's double slit experiment, one of the slit is wider
height h2 above water so that the total height of ( water + than other, so that amplitude of the light from one slit is
kerosene) is (h1 + h2). Refractive index of water is m1and double of that from other slit. If Im be the maximum
that of kerosene is m2. The apparent shift in the position of intensity, the resultant intensity I when they interfere
the bottom of the beaker when viewed from above is at phase difference f is given by : [2012]
[2011RS] Im Im æ 2 fö
(a) (4 + 5 cos f) (b) ç 1 + 2 cos ÷
æ
9 3 è 2ø
1 ö æ 1 ö
(a) ç1 + ÷ h1 - ç 1 + ÷ h2
è m1ø è m 2 ø Im æ 2 fö Im æ 2 fö
(c) ç 1 + 4 cos ÷ (d) ç 1 + 8 cos ÷
5 è 2ø 9 è 2ø
æ 1 ö æ 1 ö
(b) ç1 - ÷ h1 + ç 1 - ÷ h2 43. An object 2.4 m in front of a lens forms a sharp image
è m1 ø è m2 ø on a film 12 cm behind the lens. A glass plate 1 cm
thick, of refractive index 1.50 is interposed between lens
æ 1 ö æ 1 ö and film with its plane faces parallel to film. At what
(c) ç 1 + ÷ h2 - ç1 + ÷ h1
è m1ø è m 2 ø distance (from lens) should object shifted to be in sharp
focus of film? [2012]
æ 1 ö æ 1 ö (a) 7.2 m (b) 2.4 m
(d) ç1 - ÷ h2 + ç1 - ÷ h1 (c) 3.2 m (d) 5.6 m
è m1 ø è m2 ø
RAY & WAVE OPTICS P-455
44. Diameter of a plano-convex lens is 6 cm and thickness at 47. The graph between angle of deviation (d) and angle of
the centre is 3 mm. If speed of light in material of lens is incidence (i) for a triangular prism is represented by
2× 108 m/s, the focal length of the lens is [JEE Main 2013]
[JEE Main 2013]
(a) 15 cm (b) 20 cm (c) 30 cm (d) 10 cm
d d
45. Abeam of unpolarised light of intensity I0 is passed through
a polaroidAand then through another polaroid B which is (a) (b)
oriented so that its principal plane makes an angle of 45°
relative to that of A. The intensity of the emergent light is o o
i i
[JEE Main 2013]
(a) I0 (b) I0/2
(c) I0/4 (d) I0/8
d d
46. Two coherent point sources S1 and S2 are separated by a
small distance 'd' as shown. The fringes obtained on the (c) (d)
screen will be [JEE Main 2013]
o o
i i

S1 S2 Screen
D

(a) points (b) straight lines


(c) semi-circles (d) concentric circles
P-456 Topic-wise Solved Papers - PHYSICS

Solutions & Explanations


Section-A : JEE Advanced/ IIT-JEE
A 1. 2 × 108 m/s, 0.4 × 10–6 m 2. d = +15 cm 3. 4000Å, 5 × 1014 Hz 4. 2
25
5. 60 cm 6. 7. 30 cm 8. 1.5 9. zero 10. smaller
9

me
11. 12. 2.945 × 10–4 m 13. 5 × 1014 Hz, 4000Å 14. 0.125 m, 0.5 m
m0 e 0
15. 15°
B 1. T 2. T 3. F 4. T 5. T 6. F 7. T
C 1. (c) 2. (a) 3. (d) 4. (a) 5. (c) 6. (c) 7. (d)
8. (c) 9. (c) 10. (a) 11. (c) 12. (c) 13. (b) 14. (a)
15. (d) 16. (a) 17. (d) 18. (d) 19. (b) 20. (a) 21. (a)
22. (c) 23. (b) 24. (b) 25. (d) 26. (c) 27. (b) 28. (c)
29. (a) 30. (b) 31. (b) 32. (b) 33. (b) 34. (c) 35. (b)
36. (d) 37. (c) 38. (a) 39. (c) 40. (a) 41. (c) 42. (b)
43. (c) 44. (b) 45. (b) 46. (c) 47. (a) 48. (b) 49. (c)
50. (b) 51. (c) 52. (b) 53. (d) 54. (a) 55. (b)
D 1. (b, d) 2. (a) 3. (a, c) 4. (b, d) 5. (d) 6. (a)
7. (d) 8. (c) 9. (a, b, c, d) 10. (c) 11. (b, c) 12. (b)
13. (c, d) 14. (d) 15. (d) 16. (a) 17. (a, b) 18. (c, d) 19. (a, b, c)
E 1. 11 cm, Real 2. 1.732 3. (i) 24 cm, (ii) 12 cm, (iii) 12 cm, (iv) – 30 cm 4. 1.41 5. m1 < m2
6. 7.67 cm 7. (i) 70.8 cm, (ii) –2 8. (i) 15 cm (ii) 1.15 cm 9. 13.9
10. 5892Å 11. 75 cm 12. (i) 1.17 × 10–3 m, (ii) 1.56 × 10–3 m
1
14. (i) 2 (ii) No 15. (i) –6 mm, –5 mm 16. 7 × 10–6 W 17.
49
18. (a) 0.8 × 10–14 m2 (b) 27.2° (c) 9I 19. 2 20. (i) 10–3 m (ii) increase
4
dy 2 æ xö
22. (a) = cot i (b) y = k ç ÷ (c) (4m, 1m) 23. 4200Å, 1.43
dx è 4ø

24. (i) sin -1 ê


é 1
ë 2
{ }ù
n2 - n12 - n1 ú (ii) 72.94°
û
25. (i) 6.3 × 10–4 m (ii) 1.575 × 10–6 m 26. 0.4 m, 0.6 m

æ 3ö
27. 7 × 10–6 m, 1.6, 5.7 × 10–5 m 28. 0.4 m 29. 9.3 × 10–6 30. (a) 600 nm (b) sin–1 çè ÷ø
4
31. (a) ± 0.26 m, ±1.13 m (b) 1.13 m, 0.26 m 32. (a) 4.33 × 10–3 m (b) 0.75 (c) 0.65 × 10–6 m, 0.433 × 10–6 m
1 4 1
33. é3iˆ + 4 ˆj - 5kˆù 34. 35. (a) 15 cm, (b) 90 nm 36. 4°, – 0.04°
5 2ë û 3 2

m3 R
37. 2 cm, 1.0016 38. 1.6 39. (a) circular (b) 1/16 (c) 300 nm 40. 41. (a) zero (b) 125 nm
m 3 - m1
42. 6.056 m 43. 3.5 mm 44. 0.09 m/sec, 0.35 s–1 45. 60º 46. (a) 60º, (b) 60º, anticlockwise
F 1. A-p; B-r; C-r; D-p, q, s 2. A-p, q, r, s; B-q; C-p, q, r, s; D-p, q, r, s 3. A-p, s; B-q; C-t; D-r, s, t
4. A-p, r; B-q, s, t; C-p, r, t; D-q, s 5. (d)
G 1. (a) 2. (c) 3. (b) 4. (c) 5. (b)
H 1. (c)
I 1. 6 2. 6 3. 3 4. 2
RAY & WAVE OPTICS P-457

Section-B : JEE Main/ AIEEE


1. (b) 2. (a) 3. (a) 4. (d) 5. (a) 6. (b) 7. (d) 8. (c)
9. (b) 10. (b) 11. (c) 12. (d) 13. (b) 14. (a) 15. (a) 16. (b)
17. (b) 18. (d) 19. (a) 20. (b) 21. (a) 22. (a) 23. (c) 24. (d)
25. (c) 26. (a) 27. (b) 28. (c) 29. (d) 30. (b) 31. (d) 32. (a)
33. (a) 34. (b) 35. (a) 36. (a) 37. (a) 38. (b) 39. (a) 40. (b)
41. (b) 42. (d) 43. (d) 44. (c) 45. (c) 46. (d) 47. (c)

FILL IN THE BLANKS : m gm 1.5


5. gm = = = 1.125
mm 43
3 ´ 108
1. V2 = = 2 ´ 108 m / s ; æ 1
1.5 1 1ö
= (1.5 - 1) ç - ÷ [Lensmaker's formula]
8
15 è 1
R R2ø
V2 2 ´ 10
l2 = = = 0.4 ´ 10-6 m
v 5 ´ 1014 1 æ 1 1 ö
and = (1.125 - 1) ç - ÷
2. f2 è R1 R2 ø
On dividing we get
f2 1.5 - 1 0.5
Parallel = = =4 \ f2 = 60 cm
beam 15 1.125 - 1 0.125

I1 A12
6. = ...(i)
I 2 A22

1 I r2
5 cm But I µ Þ 1 = 2 ...(ii)
20 cm r2 I 2 r12

From the diagram it is clear that the focus of both the lenses From (i) and (ii),
should coincide as shown in the diagram. A1 r2 25
Therefore d = 15 cm. = = .
A2 r1 9
3. KEY CONCEPT :
7. For refraction at APB
a Speed of light in med1 nl a l a
mm = = = µ2 µ1 µ1 - µ2
Speed of light in med 2 nl m l m - + =
u v R
[Q v does not change with the medium]
-2 1 1 - 2
la 6000 Þ + = Þ v = – 30 cm
\ lm = = = 4000Å -15 v -10
a
mm 1.5 Þ Image of O will be formed at 30 cm to the right at P.
8. Since the image formed is real and elongated, the situation
c 3 ´ 108 is as shown in the figure. Since the image of B is formed at B'
\ na = a = = 5 ´ 1014 Hz
l a 6000 ´ 10 -10 itself
4. For coherent sources, for constructive interference
The amplitude at the mid point = A + A = 2A
B' A'
Þ I1 µ (2A)2 Þ I2 µ 4I0 ... (i)
NOTE : For incoherent sources, the intensity add up P A B
normally (no interference). f/3
Therefore, the total intensity I2 = 2I0 ... (ii)
From (i) and (ii) \ B is situated at the centre of curvature that is at a
I1 4 I 0 distance at 2f from the pole.
= =2
I2 2I0 f 5f
\ PA = 2 f - =
3 3
P-458 Topic-wise Solved Papers - PHYSICS
5f 15. Using Snell's law for the refraction at AC, we get
Let us find the image of A. For point A, u = - ,v = ?
3 µ sin i = (1) sin r
1 1 1 1 1 1
Applying, + = Þ + = 2 sin 30° = sin r Þ r = 45°
u v f -5 f v - f
3 Angle of deviation at face AC = 45° – 30° = 15°
1 1 3
Þ =- + Þ v = – 2.5f
v f 5f A
Image length = 2.5 f – 2f = 0.5f
30o
\ Magnification = 0.5 f = 1.5
f 3
æ A + dm ö æ 60 + dm ö
sin ç ÷ sin ç
è 2 ø è 2 ÷ø 60o
9. m= , 2= r = 45o
sin A 2 sin 60 2 30o
60 + dm
\ = 45° Þ dm = 30°
2
Þ The condition is for minium deviation. In this case the B C
ray inside the prism becomes parallel to base. Therefore the
angle made by the ray inside the prism with the base of the
prism is zero. TRUE / FALSE :
10. KEY CONCEPT :The resolving power of a microscope
device is inversely proportional to the wavelength used. 1. This is due to atmospheric refraction. The light coming from
Þ The resolving power of an electron microscope is sun bends towards the normal. Therefore, sun appears
higher than that of an optical microscope because the
wavelength of electrons is smaller than the wavelength of higher.
visible light. 2. KEY CONCEPT :Formula for intensity of a line source of
1 power (P) at a distance r from the source is
11. Velocity of light in vacuum c =
µ0 e 0 P
I=
1 2prl
and the velocity of light in a medium v =
µe 3. The image formed by the convex lens at the focus of the
Velocity light in vacuum c 1/ µ0e0 µe concave lens. Therefore I will act as a virtual object for
n= = = = concave lens and angle will be formed at infinity.
Velocity light in medium v 1/ µe µ0e0
2 lD
12. = 2 ´ 10-3
d
2 ´ 5.89 ´ 10-7 ´ 0.5
Þ d= = 2.945 ´ 10-4 m I
2 ´ 10-3
13. Frequency remains the same
Virtual
3 ´ 108
object
c for concave
f = = = 5 ´ 1014 Hz lens
l 6000 ´ 10 -10 0.75m 0.25m
l 6000Å
and l 2 = 1 = = 4000Å
µ 1.5 4. NOTE : For the light to split, the material through which the
14. P1 + P2 = 10 m– 1 light passes should have refractive index greater than 1.
P1 + P2 – (0.25) P1P2 = 6m – 1 Since the prism is hollow, we get no spectrum. The thickness
From these two expressions, we get
of glass slabs through which the prism is made can be
P1 P2 = 16m –2
neglected.
P1 - P2 = ( P1 + P2 ) 2 - 4 P1 P2 5. When the two slits of Young's double slit experiment are
illuminated by two different sodium lamps, then the sources
= (10-1 )2 - 4(16-1 ) = 6m -1 are not coherent and hence sustained interference pattern
\ P1 = 8m – 1 and P2 = 2 m – 1, Hence will not be achieved. It will change so quickly that there will
1 1 2 1 be general illumination and hence interference pattern will
f1 = = m = 0.125 m and f 2 = = m = 0.5m
P1 8 P2 2 not be observed .
RAY & WAVE OPTICS P-459
6. In Young's double slit experiment if source is of white light 5. (c) Let I1 = I and I2 = 4I
than the central fringe is white with coloured fringes on
( ) =( ) = (3 I )
2 2 2
either side. I max = I1 + I 2 I + 4I = 9I
1 1 1
I min = ( I2 ) = ( 4I ) = I
= + 2 2
7. F f1 f 2 I1 - I-
6. (c) Spherical aberration occurs due to the inability of a
1 1 1 -2 + 1
Þ = + = Þ F = – 30 cm. lens to converge marginal rays of the same wavelength
F -15 30 30 to the focus as it converges the paraxial rays. This can
This combination behaves as a concave lens of focal length be done by using a circular annular mask over the lens.
30 cm. 7. (d) The distance between the first dark fringe on either
SinceFv < Fr. side of the central maximum = width of central maximum
\ One sees coloured pattern with violet colour at the
outer edge. 2 Dl 2 ´ 2 ´ 600 ´ 10-9
= = = 2.4 × 10 – 3 m = 2.4 mm
MCQ's WITH ONE CORRECT ANSWER : a 10-3
8. (c) Applying Snell's law at P, 30°
1. (a) NOTE :
When the ray enters a glass slab from air, its frequency sin r
m=
remains unchanged. sin 30° P r
Since glass slab in an optically denser medium, the d
1.44 30°
velocity of light decreases and therefore we can sin r = = 0.72
conclude that the wavelength decreases. 2
(Q n = nl) \ d = r – 30 ° = sin – 1 ( 0.72) – 30°
2. (a) The phenomenon of total internal reflection takes place \ The rays make an angle of
during reflection at P.
2d = 2 [sin – 1(0.72) – 30 °] with each other..
1 9. (c) NOTE : A convex mirror and a concave lens always
sin q =
w ...(i) produce virtual image.

Therefore, option (b) and (d) are not correct. The image
l by a convex lens is diminished when the object is placed
beyond 2f.
B A Let u = 2f + x
q
1 1 1 1 1 1
- =
v u f Þ v - -(2 f + x) = f
q Using
R P
1 1 1 2f + x- f ( f + x)
Þ = + = =
v f 2f +x f (2 f + x ) f (2 f + x )
But u + v = 1 (given)
C f (2 f + x)
(2 f + x ) + £1
f +x
a
w gm 1.5 1 8
Now, gµ = = = 1.125 \ sin q = = é f ù (2 f + x) 2
a 4/3 1.125 9 2 f + x ê1 + £1 Þ £1

ë f + x úû f +x
8 Þ (2f + x)2 £ f + x. The above is true for f < 0.25 m.
\ sin q should be greater than .
9 10. (a) Here fo = 2 cm and fe = 3 cm.
lD l (2 D) lD Using lens formula for eye piece
3. (d) b= , b' = =4 = 4b -1 1 1
d d /2 d Þ + = Þ ue = – 3 cm
ue µ 3
æ 1ö
4. (a) C = Sin -1 ç 1 ÷ ..... (i) But the distance between objective and eye piece is
è 2µø 15 cm (given)
Applying Snell's law at P, we get \ Distance of image formed by the objective
= v = 15 – 3 = 12 cm.
1 sin r ' sin (90 - r ) Let u be the object distance from objective, then for
m= = [Q i = r; r' + r = 90°]
2 sin i sin r objective lens
1 cos r 1 1 1 -1 1 1
\ m= ...(ii) - + = or + =
2 sin r u0 v0 f 0 u 12 2
From (i) and (ii)
C = sin –1 (tan r) -1 1 1 5 12
Þ = - = , u = - = -2.4 cm
u 2 12 12 5
P-460 Topic-wise Solved Papers - PHYSICS
11. (c) Path difference between the opposite edges is l. R1= – R and R2 = + R
For a phase difference of 2p we get a path diff. of l.
gm
12. (c) We know that mm
d 2pd tan q
I (q) = I0 cos 2 where d=
2 l R1 R2
MEDIUM
2 æ pd tan q ö 2 æ p ´ 150 ´ tan q ö mm
I (q) = I 0 cos ç ÷ø = I 0 cos çè ÷
è l 3 ´ 108 /106 ø
1 æ 1.5 ö æ 1 1ö 0.25 ´ 2
\ =ç - 1÷ ç - - ÷ = +
2æp
ö f è 1.75 ø è R R ø 1.75 R
= I0 cos ç tan q÷
è2 ø Þ f = + 3.5 R
NOTE : The positive sign shows that the lens behaves
as convergent lens.
15. (d) For diffraction pattern to be observed, the dimension
y of slit should be comparable to the wave length of
S1 rays. The wavelength of X-rays (1 – 100 Å) is less than
d/2 0.6 mm.
q
d/2 D 16. (a) Locus of equal path difference are lines running parallel
S2 to axis of the cylinder. Hence straight fringes will be
observed.
1 æ µ2 ö æ 1 1ö
= -1 -
17. (d) f çè µ1 ÷ø çè R1 R2 ÷ø m 1 m 2 m3
æ p ö If m2 > m1 , the concave lens
For q = 30°; I(q) = Io cos2 çè ÷ maintains its nature otherwise the
2 3ø
nature of th e lens will be
For q = 90° ; I(q) = Io cos2 (¥) reversed.
For q = 0° So, the lens should be filled with L2 and immerse in L1.
I (q) = I0 18. (d) From the ray diagram.
I (q) is not constant. M
Alternatively, when q is zero the path difference A 2L X N
between wave originating from S1 and that from S2 will D L B d/2
be zero. This corresponds to a maxima. d/2
13. (b) f1 = + 40 cm = 0.4 m (for convex lens)
X
f2 = – 25 cm = – 0.25 m (for concave lens)
In DANM and DADB
ÐADB = ÐANM = 90°
ÐMAN = ÐBAN (laws of reflection)
Also ÐBAN = ÐABD Þ ÐMAN = ÐABD
\ DANM is similar to DADB
x d /2
\ = or x = d
2L L
\ Focal length (f) of the combination So, required distance = d + d + d = 3d.
1 1 1 1 1 0.25 - 0.4
= + = - =
f f1 f 2 0.40 0.25 0.40 ´ 0.25 2
n1 a/
a/2
=-
0.15 1
= -1.5dioptre. Þ P = = -1.5 dioptre
n2
0.1 f
r
19. (b) a/2

14. (a)
1
f
= m( æ 1
g µ -1 ç )1 ö
è R1 R2 ø g
µ
- ÷ Now, m µ = g = 1.5
a /2
m µ 1.75
For concave lens as shown in figure in this case
The incident and emergent ray of a glass slab are
parallel therefore, the angle remains the same.
RAY & WAVE OPTICS P-461
20. (a) See figure. The ray will come out from CD if it suffers Applying Snell's law at Q,
total internal reflection at surface AD, i.e., it strikes the
surface AD at critical angle C ( the limiting case). 2 sin r1 m3
m3 = = ...(2)
sin r2 m 2
A P D
Again applying Snell's law at R
C n n2
1
Q 90°–C 3 sin r2 m 4
m4 = = ...(3)
a sin i m3
Multiplying (i), (ii) and (iii), we get
B C µ4 = µ1
Applying Snell's law at P NOTE : If the emergent ray is parallel to incident ray
n2 after travelling a number of parallel interfaces then the
n1 sin C = n2 or sin C =
n1 refractive index of the first and the last medium is always
Applying Snell's law at Q same.
n2 sin a = n1 cos C
26. (c) Q
n1 ìï æ n ö ïü
Þ sin a = cos ísin -1 ç 2 ÷ ý
n2 îï è n1 ø þï

én ìï æ n ö üïù
or a = sin -1 ê 1 cos ísin -1 ç 2 ÷ ýú P
ëê n2 îï è n1 ø þïûú
R
21. (a) When slits are of equal width.
Imax µ (a + a)2 ( = 4a2)
Imin µ (a – a)2 ( = 0) There will be no refraction from P to Q and then from Q
When one slit's width is twice that of other to R (all being identical). Hence the ray will now have
the same deviation.
I1 W1 a 2 W a2
= = Þ = Þ b = 2a 27. (b) For the image of point P to be seen by the observer, it
I 2 W2 b 2 2W b2
should be formed at point Q.
\ Imax µ (a + 2a )2 = (5.8 a2) In DQNS, N
Imin µ ( 2a – a)2 = (= 0.17 a2) NS = QS = 2h
22. (c) NOTE : The intermediate image in compound \ ÐNQS = 45°
r
microscope is real, inverted and magnified. \ r = 45° M 2h
23. (b) I = I1 + I2 +2 I1 I 2 cos f ...(1) Now in DQMA,
i
Applying eq. (1) when phase difference is p/2 ÐMQA = 45° 45°
Q S 2h
Ip/2 = I + 4I Þ Ip/2 = 5I \ MA = QA = h A
Again applying eq. (1) when d phase difference is p h
1 sin r sin 45°
I p = I + 4I + 2 I 4 I cos p m= = P B ...(i)
2 sin i sin i
\ Ip = I \ Ip/2 – Ip = 4I 2h
In D PMB,
12l1 D k l 2 D 12 ´ 600
24. (b) = , k= = 18 PM2 = 4h2 + h2 = 5h2
d d 400
25. (d) Applying Snell's law at P, h 1
\ sin i = = ...(ii)
1 sin i µ2 5h 5
µ2 = = ...(1)
sin r1 µ1 From (i) and (ii)

m1 m2 m3 m4 1 5
µ=
r1 r2 i 2 2
r2 R
Q 28. (c) Since both surfaces have same radius of curvature on
i1 P r1
the same side, no dispersion will occur.
29. (a) Path difference = (µ – 1) t = nl;
For minimum t, n = 1; \ t = 2 l
P-462 Topic-wise Solved Papers - PHYSICS
For concave lens,
30. (b) Maximum number of reflection =
2 3
x 1 1 1 1 1 1 4 1
- =- or = - + = =
where x = 0.2 tan 30° = 0.2 3. v 4 20 v 20 4 20 5
or v = 5 cm = Distance of I2 from concave lens.
l
x v size of image 5
\ Magnification = = =
u size of object 4

0.2m 30° 30° size of image


or = 1.25
2
or size of image due to concave lens = 2.5 cm
d 33. (b) µg sin i = m air sin 90°
31. (b) In D OPM, OP =
cos q
d cos 2q 1
In DCOP, OC = mg =
cos q sin i

O M 1 1
Q R 34. (c) KEY CONCEPT : sin C = andµ µ
µ l
q q \ sin C µ l
For higher value of l, the angle C also increases
C
d
l increases Air
(Green light)
q V I B G Y O R C
P Glass

35. (b) NOTE : For minimum deviation, incident angle is equal


Path difference between the two rays reaching P is to emerging angle and QR is parallel to base.
36. (d) At the area of total darkness minima will occur for both
l d cos 2q d l
= CO + OP + = + + the wavelengths.
2 cos q cos q 2
d l l (2n + 1) (2 m + 1)
= (cos 2q + 1) + = 2d cos q + \ l1 = l 2 Þ (2n + 1)l1 = ( 2m +1) l2
cos q 2 2 2 2
For constructive interference, path difference should
be nl (2n + 1) 560 7
or = = or 10n = 14m + 2
l (2n - 1) l (2m + 1) 400 5
\ 2d cos q + = nl Þ cos q =
2 4 d by inspection for m = 2, n =3 and for m = 7, n = 10, the
l distance between them will be the distance between
For n = 1, cos q = such points.
4d
32. (b) Convex lens forms the image at I1. I1 is at the second
Dl1 ì (2n2 + 1) - (2 n1 + 1) ü
focus of convex lens. Size of I1 = 2 cm. i.e., Ds = í ý
I1 acts as virtual object for concave lens. Concave lens d î 2 þ
forms the image of I1 and I2. put n2 = 10, n1 = 3
On solving we get, Ds = 28mm.
37. (c) NOTE : Frequency does not change with change of
I1
I2 medium.
38. (a) The rays coming from the point object fall on the glass-
air interface normally and hence pass undeviated.
26 cm 4 cm Therefore if we retrace the path of the refracted rays
backwards, the image will be formed at the centre only.
30 cm
RAY & WAVE OPTICS P-463
39. (c) 43. (c) We know that in case of a convex lens when object is
P placed at C ', the image is obtained at C. This situation
is represented in the graph by the point corresponding
to u = –10 cm, v = 10 cm.
A R
Therefore R = 10 cm Þ = 5cm = f
d q 2
q
in
B ds 31
v cm
pdsin q 30
I = Imax cos 2
l
Imax æ pd sin q ö
Þ = Imax cos 2 ç ÷
4 è l ø 10

pd sin q 1 pd sin q p
\ cos = \ = 0 (–9, +9)
l 2 l 3 u cm –31 –30 –20 –10
æl ö Lens formula is
\ q = sin -1 ç ÷
è 3d ø 1 1 1 Df Dv Du
| f1 | 2 = - Þ = +
2
40. (a) = f v u f v2 u 2
| f2 | 3
(for maximum error in f)
f 1 : focal length of convex lens.
1 1 2 Df 0.1 0.1
1 1 1 Þ = + [Du = Dv = 0.1 from graph)
= - Þ = - 25 (10) 2
(10) 2
f f1 f 2 30 f1 3 f1
f1 = 10 cm, f2 = –15 cm Þ Df = 25 × 0.1 × 2 × 0.01 = 0.05
41. (c) The image I ' for first refraction (i.e., when the ray comes Therefore, the focal length = (5.00 ± 0.05) cm.
out of liquid) is at a depth of
AB
é Re al depth ù 44. (b) From the figure in D ABC, tan b =
33.25 AC
= = 25cm êQ Apparent depth = ú
1.33 ë m û Þ AB = AC tan b, 2r = f tanb
Now, reflection will occur at concave mirror. For this I' Þ Area of image = pr2µ f 2
behaves as an object.
\ u = – (15 + 25) = – 40 cm Parallel
Rays from
é 25 ù Focal
object placed
and v = - ê15 +
ë 1.33 úû plane
at infinity
f
25 A
Where is the real depth of the image. a b
1.33
on focal plane

2r C
Image formed

Using mirror formula we get


1 1 1 B
= + , f = – 18.31 cm
f v u Biconvex lens
42. (b) The focal length f of the equivalent mirror is
1 2 1 2 1 15 45. (b) As shown in the figure, when the object (A) is placed
= + = +
f f1 f m 15 ¥ Þ f = 2 cm between F and C, the image (I) is formed beyond C. It
Since f has a positive value, the combination behaves is in this condition that when the student shifts his
as a converging mirror. eyes towards left, the image appears to the right of the
object pin. (Image distance > object distance)
15
Here u = – 20cm, f = - cm , v = ?
2 Movement towards left
1 1 1
According to mirror formula + = F
v u f
1 1 1 I C A P
Þ - = Þ v = – 12 cm
v -20 -15 / 2
Negative sign indicates that the image is 12 cm in front
of mirror.
P-464 Topic-wise Solved Papers - PHYSICS
46. (c) The ray is partly reflected and partly refracted 49. (c) Consider the activitiy A to B
ÐMOB = 180 – 2q
Applying v 2 - u 2 = 2 as

ray v 2 - 0 2 = 2 ´ 10 ´ 7.2
M ac ted
ef r
tly
r P Þ v = 12 m/s
Par
The velocity of ball as perceived by fish is
O
180–2q 4
q q v ' = wm ´ v = ´ 12 = 16 m/s
3
Pa
rtl
ay

y
tr

re
fle
en
cid

c t ed
In

A u=0
ra

A B
y

2
a = 10 m/s
s = 7.2 m
But the angle between refracted and reflected rays is 20 m
ÐPOB. Clearly is ÐPOB is less than ÐMOB. B
47. (a) v
12.8 m
60°
60°
iv r=
30
º fish

Medium 1 Medium 2 50. (b) Focal length of the biconvex lens is 15 cm. A small
object is placed at a distance of 30cm from the lens i.e.
at a distance of 2f. Therefore the image should form at
30cm from the lens at I1.
60°
60°
ir r=
30º
O I3 I2 I1

Medium 1 Medium 2
10cm
For minimum deviation the ray in the prism is parallel to
the base of the prism. This condition does not depend But since the ray strike the plane mirror before reaching
on the colour (or wave length) of incident radiation. So I1, the image I1 acts as the virtual object for reflection
in both the cases, by geometry, r = 30º. So (a) is correct on plane mirror kept at a distance of 20 cm from it.
option. It should produce an image I2 but as the ray encounters
48. (b) For refraction at parallel interfaces the lens, it gets refracted and the final image is formed
n0 n0 n0 at I3. For the last refraction from the biconvex lens,
n0 sinq = sin a = sinb = sin 90º u = 10 cm.
2 6 8
1 1 1
Applying lens formula - =
v u f
b
1 1 1 1 1 1 25
Þ - = Þ = + =
b v 10 15 v 15 10 150
1 a
\ sinq = Þ v = 6 cm.
8 a
q
Therefore a real image is formed at a distance of 16 cm
from the plane mirror.
RAY & WAVE OPTICS P-465
51. (c) When the light is incident on glass - an interface at an 55. (b) The intensity I is given as
angle less than critical angle a small part of light will be f
reflected and most part will be transmitted. I = Io cos 2 where Io is the peak intensity
2
When the light is incident greater than the critical angle,
it gets completed reflected (total internal reflection) Io I f p
Here I = , \ o = I o cos 2 \ f = (2n + 1)
These characteristics are depicted in option (c). 2 2 2 2
52. (b) The focal length (f1) of the lens with n = 1.5 is given by For a phase difference of 2p the path difference is l
1 é 1 1 ù p
= (n1 - 1) ê - \ For a phase difference of (2n+1) the path
ú 2
f1 ë R1 R2 û
difference
é 1 1ù 1 l
= (1.5 - 1) ê - ú = is (2n +1) . option (b) is correct.
ë 14 ¥ û 28 4
The focal length (f 2 ) of the lens with n = 1.2 is
given by

1
f2
é 1
= (n2 - 1) ê -
1 ù
ú 1.
I
(b, d) max =
( I1 + I 2 )2 = 9
ë R1 R2 û
I min
( I1 - I2 )
2 1
é1 1 ù 1
= (1.2 - 1) ê - úû = 70
ë ¥ -14 I1 a2 a
\ =4= 2 Þ =2
The focal length F of the combination is I2 b b
1 1 1 1
= + = 1 1 1 1 1
F f1 f 2 20 2. (a) P= = + = + = -1.5 dioptre.
f f1 f 2 0.4 -0.25
Applying lens formula for the combination of lens
1 1 1 1 1 1 lD ld
- = Þ - = 3. (a, c)Here y = (2n - 1) = (2n - 1)
V U F V -40 20 2 d 2b
Þ V = 40 cm (Q d = b and D = d)

lD b
But y =
53. (d) We know that b = 2
d
Now, lR > lG > lB
\ bR > bG > bB b ld
\ = (2n - 1)
2 2b
æ iˆ 3 ˆö æ ˆi 3 ˆö
ç 2 + 2 j÷ .ç 2 - 2 j÷ b2
è ø è ø Þ l= when n = 1, 2
54. (a) cos(180° - 2a ) = (2n - 1)d
2 2 2
æ 1ö æ 3ö æ 1ö æ 3ö
çè ÷ø + ç ÷ çè ÷ø + ç - ÷
2 è 2 ø 2 è 2 ø b2 b2
l= , ,...
d 3d
1 4. (b, d) The image formed will be complete because light rays
\ cos(180° - 2a ) = -
2 from all parts of the object will strike on the lower half.
But since the upper half light rays are cut off, the intensity
will reduce.
180°-2a
1 1 1 dv du
5. (d) + = \- 2
- =0
a a v u f v u2
2
dv - v 2 æ f ö
\ = 2 = -ç
du u è u - f ÷ø
\ 180° – 2a = 120° 2
image length = çæ
\ a = 30° f ö
\ ÷ ´b
option (a) is correct èf -uø
P-466 Topic-wise Solved Papers - PHYSICS
6. (a) For total internal reflection
1 1 1 f ( f - d)
1 1 Þ - = Þ v2 = 2 1
m= = = 1.414 v2 f1 - d f 2 f 2 + f1 - d
sin C sin 45°
i.e. for an angle of incidence of 45°, that colour will suffer \ The horizontal distance of the image I from O is
total internal reflection for which the refractive index is less
than 1.414. f 2 ( f1 - d ) f f + d ( f1 - d )
x=d+ = 1 2
A f 2 + f1 - d f1 + f 2 - d
To find the y-coordinate, we use magnification formula for
lens 2

i=45o f 2 ( f1 - d )
v f + f -d f2
m= 2 = 1 2 =
B u2 f1 - d f1 + f 2 - d
Therefore, red light will be refracted at interface AB whereas
blue and green light will suffer total internal reflection. h2 D ´ f2
7. (d) In an astronomical telescope when the object and final Also m = Þ h2 =
D f1 + f 2 - d
image are at infinity, M and L are given as shown:
Angular magnification M = fo/fe \ The y-coordinate y = D – h2
Seperation between lenses, L = fo +fe
Df 2 D ( f1 - d )
fo = D- =
\ = 5 or f = 5 f . ... (i) f1 + f 2 - d f1 + f 2 - d
fe o e
11. (b, c) NOTE : Concave lens and convex mirror are diverging
fo + f e = 36 or 5 f e + f e = 36
in nature. Therefore the refracted/reflected rays do not meet.
or fe = 6 cm ... (ii)
These rays are produced backwards to make them meet.
\ f o = 5 f e or fo = 30 cm Therefore the image formed is virtual and erect.
Hence fo = 30 cm, fe = 6 cm 12. (b) Spherical aberation is smaller when the curved surface
8. (c) The angle of deviation for the first prism P1
d1= (µ1 – 1) A1 is facing the object because the total deviation is shared
The angle of deviation for the second prism P2 between the two surfaces.
d2= (µ2 – 1) A2 13. (c, d) KEY CONCEPT : For total internal reflection to take
Since total deviation is to be zero place :
\ d1+ d2= 0
Þ (µ1 – 1) A1 + (µ2 – 1) A2= 0 Angle of incidence i > critical angle, qc
-(1.54 - 1) é 1ù
Þ A2 = 4° = -3°
(1.72 - 1) êë where sin qc = n úû
9. (a, b, c, d) In case of an astronomical telescope the distance
between the objective lens and eyepiece lens 1 1 1
= f0 +fe = 16 + 0.02 = 16.02 m or sin 45° > or > or n > 2 or n > 1.414.
n 2 n
f objective -16
The angular magnification = – = = -800 Therefore, possible values of n can be 1.5 or1.6 in the
f eye piece 0.02
given options.
NOTE : The image seen by the astronomical telescope is 14. (d) For first minima the path difference between the rays
inverted. Also the objective lens is larger than eye piece coming from the two edges should be l which
lens.
10. (c) The image I1 of parallel rays formed by lens 1 will act as corresponds to a phase difference of 2p.
virtual object. 15. (d) The ray diagram is shown in figure. Therefore, the
2 image will be real and between C and O.

1 Normal at P will
pass through C
C C
I
I1 Image
I
P
d O O
f1
Applying lens formula for lens 2
RAY & WAVE OPTICS P-467
16. (a) The formula for spherical refracting surface is 18. (c, d)Given f = – 24 cm
- µ1 µ2 µ2 - µ1 1 1 1
+ = Applying + =
u v R v u f
m1 m2 For (66, 33)
Glass
Air 1 1 1 1 1 -66 + 24 -42
Þ = - = + = =
P O C Q (Real Image) v f u -24 66 24 ´ 66 24 ´ 66
u v
24 ´ 66
Þ v=- = -37.7
42
Here u = –x, v = + x, R = + R, µ1 = 1, µ2 = 1.5
But the value of v = 33. The absolute error is 37.7 – 33
-1 1.5 1.5 - 1 = 4.7 cm which is greater than 0.2 cm. Therefore a wrong
+ = Þ x = 5R
-x x R reading.
17. (a,b) The condition to obtain maxima in the phenomenon For (78, 39) when u = 78 then
observed in young's double slit experiment is 1 1 1
+ =
d sin q = nl where n is an integer v -78 -24
Þ v = – 34.67
The absoluate error is 39 – 34.67 = 4.33 which is greater
than 0.2 cm.
19. (a,b,c)
Applying Snell’s law at P
n1 sin 60° = n 2 sinq
4I
Þ sin 60° = 3 sin q
Þ q = 30°
q
d In quadrilateral BCQP,
B
inq
I ds 60° 60° C
P 30°= q 135°

i
45° 45°
30°
When d = l 75°
Screen
l sin q = nl A aM D
Þ sin q = n
When n = 0, q = 0 60° + (90° + 30°) + 135° + ÐPQC = 360°
When n = 1, q = 90° (This will be a point on the screen Þ ÐPQC = 45° Þ i = 45°
which will be at infinity and therefore not practical) The critical angle for prism - air pair of media is
Other values of n are invalid as –1 £ sin q £ 1 . -1 æ 1 ö
ÞThe screen will have only one maxima. C = sin çè ÷ø which is less than 45°.
3
When λ < d < 2λ
Therefore total internal reflection takes place at face
nl é nl ù CD.
Þl <
sin q
< 2l êëQ d = sin q úû option (a) is correct.
In DQDM, ÐQMD = 180° – (45° + 75°) = 60°
n Therefore the angle of incidence of ray QM on AD is
Þ1< <2
sin q 30°.
The possible values of n are 0, + 1, –1. This angle is less than the critical angle. Therefore the
ÞThere is at least one more maxima (besides the central ray emerges out of face AD.
maxima, option [B] is correct. Option (b) is correct.
We know that Applying Snell’s law at M, we get

( ) ( )
2 2 3
Imax = I1 + I2 , Imin = I1 - I2 3 sin 30° = 1´ sin a Þ a = sin -1 = 60°
2
Initially I1 = 4 I and I2 = I In quadrilateral PQMN, ÐPNM = 360° – [60° + 90°
\ Imax = 9 I and Imin = I +120°] = 90°
When I1 = I2 = I then Imax = 4 I and Imin= 0 \ The angle between the incident ray and the emergent
i.e., when the intensities become equal, Imin reduces to ray is 90°
zero. Options [C] and [D] are incorrect. Option (c) is correct.
P-468 Topic-wise Solved Papers - PHYSICS
(iii) Since the focal length of the mirror is 12 cm, a parallel
SUBJECTIVE PROBLEMS : beam of light will converge at a distance of 12 cm from the
1. The focal length of the equivalent mirror is silvered surface.
1 2 1 1 1 1
= + (iv) + =
F f fm v u f

2 2 1 1 21 1 1 1
= + = + = Þ + =
20 22 10 11 110 v -20 -12
Þ v = – 30 cm
110
Þ F=
21
NOTE : Since the focal length is positive it is a converging
I O
mirror
20 cm
1 1 1 1 1 1 30 cm
Now, + = Þ + =
u v f -10 v -110 / 21
1 1 21
Þ = - Þ v = – 11cm
v 10 110
NOTE : The negative sign indicates the image is real. 4. A C
Vertical
2. The situation can be shown as in the figure. side
C
A 90–C
q 2m
B D
M 1m
d
i e For a grazing incident ray at BD for which i » 90° the angle of
r r' refraction (90 – C) is maximum. For this C is least. Let C is
greater than the critical angle.
B C Applying Snell's law at M
Here, i = 60°, A = 30°, d = 30°, e = ? sin 90°
1 1 1
We know that, A + d = i + e ....(1) µ= Þ µ = ...(i)
Also, A = r + r' ....(2)
2 sin(90 - C ) 2 cos C
From (1), 1 1
e = A + d – i = 30° + 30° – 60° = 0 Also µ= ...(ii)
sin C
2
As the angle of emergence (e) is 0, hence the emergent ray When C is the critical angle.
is normal to the face from which it emerges.
1 1
When e = 0, r' = 0 From (i) and (ii), = Þ C = 45°
\ From (2), A = r = 30°. cos C sin C
From Snell's law, refractive index of prism, 1 1
\ µ = = 2 = 1.41
sin i sin 60° 3/2
2 sin 45°
µ= = = = 3 = 1.732. 5. For case (i), there is no refraction. Therefore µ1 = µ
sin r sin 30° 1/ 2 NOTE : Here the convex lens behaves as a diverging lens.
Therefore, µ < µ2.
1 æ 1 1ö
3. (i) = (µ - 1) ç - ÷ 6. The rays originating from A (the point object) suffer
f è R1 R2 ø refraction before striking the concave mirror.
For the mirror the rays are coming from A'
R1

O
A' A
R2 = a

1 æ 1 1ö
= (1.5 - 1) ç - ÷ Þ f = 24 cm
f è 12 ¥ ø
æ 1ö
(ii) When the plane surface is silvered such that AA ' = shift = t ç1 - ÷
è µø
1 2 1 1 2 1 Therefore the object distance
= + Þ = + Þ F = 12 cm
F fl f m F 24 ¥
RAY & WAVE OPTICS P-469

æ 1ö 1 2 1 4
u = OA ' = OA - AA ' = 21 - t ç1 - ÷ \ = + =-
è mø f -30 -10 30
\ f = – 7.5 cm
æ 1 ö Using mirror formula
= 21 - 3 ç1 - ÷ = 20cm
è 1.5 ø 1 1 1 1 1 1
= + Þ = +
uf 20 ´ 5 20 f v u -7.5 - x - x
\ v= = = cm = 6.67cm x = 15 cm
u - f 20 - 5 3
(ii) Let the object distance be u. When water is poured
The reflected rays again pass through the glass slab. The over the concave surface the apparent object distance will
image should have formed at B is the absence of glass slab. be v then
But. due to its presence the image is formed at B'.
m m m - m1
- 1+ 2 = 2
u v R
For flat surface R = ¥
O
B B'
µ1 µ2
\ - + =0
u v
Therefore image distance = OB + BB' m 1 4
Þ v =u 2 =u´ m = u´
m1 2 3
20 æ 1 ö 20 23
+ t ç1 - ÷ , +1 = = 7.67 cm Since the ray enters the lens from water into glass
3 è µø 3 3
7. (i) f0 = + 50 cm; fe = + 5 cm ; D = 25 cm ; u0 = – 200 cm -m w m g m g - m w
+ =
For objective lens u v R
1 1 1 1 1 4 -1 3 -4 / 3 1.5 1.5 - 4 / 3
= + = - = = Þ + = Þ u = – 13.85 cm
v0 f 0 u0 50 200 200 200 4 -20 -60
u
3
200 \ Downward shift = 15 – 13.85 = 1.15 cm.
Þ v0 = cm
3 9. The total intensity at point P will be
For eyepiece lens = IA + IB + I C
C
1 1 1 1 1 6 -25 (Illumination power) ´ cos q 20W
= - = - =- Þ ue = cm IA =
ue ve f e -25 5 25 6 4pr 2
NOTE : [ve is take negative because the image is virtual 90 ´ cos 0 60O
\ Separation between objective and eyepiece = 2 P
4p´ 3 A 90W
200 25 400 + 25 425
O
60
| v o | + | ue | =+ = = = 70.8cm 10
3 6 6 6 = watt / m 2
Total magnification = mo × me 4p
v v 200 / 3 -25 180 ´ cos 60° 10 B 180W
= o´ e = ´ = -2 IB = = watt / m 2
uo ue -200 -25 / 6 4 p ´ (1.5) 2 p
8. (i) KEY CONCEPT :The given silvered concavo-convex IC = 20 cos 60° = 10
lens behaves like a mirror whose focal length can be 10 10
\ Ip = + + 10 = 13.9W / m 2
1 2 1 4p p
calculated by the formula f = f + f 10. KEY CONCEPT :Fringe shift when a sheet of thickness t
1 2
and refractive index µ is introduced in path of one of
f1 = focal length of concave surface. interfering waves is
f2 = focal length of concave mirror
Dt ( µ - 1) D ´ 1.964 ´ 10-6 (1.6 - 1)
Dx = = ...(i)
d d
The distance between two maxima where mica sheet is
remove and the distance between the slits and the screen is
l (2 D)
doubled = ...(ii)
d
R1=60 cm Given that the value in eq. (i) and eq. (ii) are equal
D ´ 1.964 ´ 10 -6 0.6 l ´ 2 D
R 2=20 cm \ =
d d
1.964 ´ 10-6 ´ 0.6
Þ l= = 0.5892 ´10-6 m = 5892Å
2
P-470 Topic-wise Solved Papers - PHYSICS
11. Here R = µ i.e., plane surface is the refracting surface Case (ii) : n3 > n1 but n3 < n2
The ray will get refracted in medium III as the angle q will
now be less than the critical angle required for medium II
and medium III pair.
v
u
I Med I R
O Med III
r
µ µ µ -µ µ µ
- 1+ 2 = 2 1 Þ - 1 + 2 =0 r
u v R -4 -3
G P Med II F
µ2 3
\ = ...(i) q
µ1 4
Again applying sin q n3
\ = (Applying Snell's law at P)
µ µ µ -µ 1 µ /µ (µ / µ ) - 1 sin r n2
- 1+ 2 = 2 1 Þ - + 2 1= 2 1
u v R u v R n2
\ sin r = sin q
1 3/4 3 / 4 -1 n3
Þ - + =
-4 -25 / 8 R As n2 > n3 So, r > q
When the refracted ray PR meets the boundary DE, it is
travelling from a denser medium to a rarer medium. Therefore
the ray will be totally internally reflected at DE if its angle of
v incidence r is more than the critical angle for med III and I.
u
I n1
O sin i'' = n
3
On solving we get R = – 25cm. n1
Applying Len's maker formula, Since, sin r > n Þ sin r > sin i'' Þ r > i''
3
1 æ 1 1ö
= (m - 1) ç - ÷ = æç - 1öæ
4 1 1ö Therefore ray PR will be totally internal reflected along RQ.
- \ f = 75cm
f è R1 R2 ø è 3 øè 25 µ ÷ø
֍ On reaching Q, the ray will be refracted in med II. Thus, the
ray will ultimately be reflected back in medium II.
12. (i) The distance of the nth bright fringe from the central 14. (i) Let x is the incident angle for reflection at AC. For total
maxima is given by the expression internal reflection x > iC (critical angle)
n lD
yn = , For 3rd bright fringe n = 3
d
3 ´ 6500 ´ 10-10 ´ 120 ´ 10-2 A x y B
\ y= = 1.17 × 10–3 m x
2 ´ 10-3 90–x
y
y 90–y
(ii) Let nth bright fringe of wavelength 6500 Å coincide M
with mth bright fringe of wavelength 5200Å. Their N
C
distance will be same from the central bright. Therefore, Let y be the incident angle of the ray on face CB. For total
nl1 D ml 2 D n 5200 4 internal reflection
= \ = = y > iC
d d m 6500 5
i.e., at the least distance 4th bright fringe of 6500 Å will \ x + y > 2iC
But x = ÐA and y = ÐB (from geometry)
coincide with 5th bright fringe of 5200 Å. Its distance
\ x + y = 90°
from the central maxima will be
Þ 90 > 2iC Þ iC < 45°
4 ´ 6500 ´ 10-10 ´ 120 ´ 10-2 The refractive index of the medium for this to happen.
yn = = 1.56 ´ 10 -3 m
2 ´ 10 3 1 1
µ= = = 2
13. KEY CONCEPT : For total internal reflection, the conditions sin iC sin 45°
are 5
· The object should be in the denser medium. (ii) µ =
· The angle of incidence should be greater than the 3
critical angle 1 1 3
Case (i) : When n3 < n1 Þ sin iC' = = = Þ iC' = 37°
µ 5/3 5
Obviously n3 < n2 and the angle q is greater than the critical y = 30° (Given) \ x = 60°
angle required for the ray passing from medium II to medium x > iC' but y < iC'
III. Therefore total internal reflection will also take place Þ Total internal reflection will take place on face AC but
when a ray strikes with the same angle at the interface of not on CB.
medium II and medium III.
RAY & WAVE OPTICS P-471
15. (i) Initially the object is in denser medium and 17. As shown in the figure, the interference will be between
u = ¥ using the formula of refraction at a spherical 0.25 I = I1 and 0.14 I = I2
surface for AB
µ2 µ1 µ1 - µ2 -4 / 3 1 1 - 4 / 3 Glass Glass
- + = Þ + = Plate Plate I
u v R -¥ v 2
Þ v = – 6 mm
NOTE : This is the position of the image due to refraction at 0.75I 0.25I
the first surface. This image will behave as a virtual object
for the refraction at the second surface.
u = – 6 – 4 = – 10 mm
Again using the formula of refraction at a spherical surface
for CD
0.1875I
4
µ1 µ2 +µ2 - µ1 -1 0.14I
- + = 1 4 / 3
u' v' R
, - + =3
10 v ' -2
Þ v' = – 5 mm.
The is the position of final image. I max
=
( I1 + I 2 )2
(ii) Ray Diagram. I min
( I1 - I2 )
2

A
C [ 0.25 I + 0.14 I ]2 49
= 2
= .
V' [ 0.25 I - 0.14 I ] 1
I' I 2mm 2mm 18. (a) l1 = 4000 Å and l2 = 5000 Å
V AIR
BUBBLE

WATER B D q

16. The power transmitted through A l1 M


q
é æ 10 ö ù d q
= ê10% of ç ÷ ú ´ p (0.001) 2 N r
ë è p øû l2 i
Q q=i d
10 10
= ´ ´ p ´ (0.001)2 = 10–6 W
100 p For total internal reflection to take place, q should be
The power transmitted through B greater than C. For smaller values of C, the values of m
should be high or in other words the value of l should
é æ 10 ö ù 2
= ê10% of ç ÷ ú ´ p ´ (0.002) be small.
ë è p ø û Therefore, total internal reflection will be given by
10 10 l1 = 4000 Å
= ´ ´ p ´ (0.002)2 = 4 × 10–6 W Here, sin q = 0.8 (given) Þ q = 53.1°
100 p
Let Df be the phase difference introduced by film 1 1
\ µ= = = 1.25
2p sin q 0.8
\ Df = (path difference introduced by the film)
l b
\ µ = 1.2 + = 1.25
2p 2p (4000 ´ 10 -10 ) 2
= ´ (µ - 1) t = [1.5 –1]×2000×10–10
l 6000 ´ 10-10 Þ b = 0.8 × 10–14 m2
(b) Applying Snell's law at N for wavelength l2
p
=
radian
3 sin r 0.8 ´ 10 -14
The power received at F µ= where µ = 1.5 + = 1.232
sin i (5000 ´ 10-10 ) 2
P = P1 + P2 + 2 P1 P2 cos D f
sin r
p Þ 1.232 = Þ sin r = 1.232 × 0.8 = 0.9856
= 10–6 + 4 × 10–6 + 2 10-6 ´ 4 ´ 10 -6 cos 0.8
3 Þ r = 80.3°
= 7 × 10–6 W. From the figure it is clear that the deviation,
d = r – i = 80.3° – 53.1° = 27.2°
P-472 Topic-wise Solved Papers - PHYSICS
(c) The intensities of transmitted beams are 4I and I 20. (i) In this case, the two identical halves of convex lens
respectively. will create two seperate images S1 and S2 of the source
S. These Images (S1 and S2) will behave as two
coherent sources and the further dealing will be in
A r accordance to Young's double slit experiment.
i N
i r
u v
M B L1
S1
A

Path diff = m( MB) - AN O1 d


S O2 O
sin r 0.3m
= ( AB sin i ) - AB sin q
sin i
= 0 S2
L2
Since both the radiations are mutually coherent and D
0.15m
while coming to focus these travel equal paths, 1.3m
therefore, these two beams will arrive in phase at focus.
\ Resultant Intensity
For lens L1
( ) =( )
2 2
I max = I1 + I 2 4I + I The object is S
u = – 0.15 m, v = ?, f = + 0.1 m
2
= (3 I ) = 9I. 1 1 1 1 1 1 1 1
19. The light entering the rod does not emerge from the curved - = Þ = + = +
surface of the rod when the angle (90° – b) is greater than v u f v f u 0.1 -0.15
the critical angle.
0.1 ´ 0.15
Þ v= = 0.3 m
0.05
D SO1O2 and D SS1S2 are similar. Also the placement of O1
b 90 – b
and O2 are symmetrical to S
a
S1S2 u + v
\ =
O1O2 u
1
i.e., µ < where C is the critical angle.
sin C (u + v ) (O1O2 ) (0.15 + 0.3)
Here, C = 90 – b Þ S1S2 = = × 0.5 × 10–3
u (0.15)
1 1
Þ µ < sin (90° - b) Þ µ < Þ S1S2 = d = 1.5 × 10–3 m \ D = 1.3 – 0.3 = 1m
cos b The fringe width
1
As a limiting case, µ = ... (i) lD 500 ´ 10-9 ´ 1 1
cosb b= = -3
= ´ 10-3 m
Applying Snell's law at A d 1.5 ´ 10 3

sin a sin a \ Therefore,


µ= Þ sin b = ... (ii)
sin b µ 1
OA = 3b = 3 × ´ 10-3 m = 10–3
NOTE : The smallest angle of incidence on the curved 3
p (ii) If the gap between L1 and L2 i.e., O1O2 is reduced.
surface is when a = . This can be taken as a limiting case
2 Then d will be reduced. Then the fringe width will
for angle of incidence on plane surface. increase and hence OA will increase.
From (ii) 21. (i) Since Y is below of optic axis, therefore the image is
sin p / 2 1 real and inverted.
sin b = Þ µ= ... (iii) (i) STEPS OF CONSTRUCTION OF DIAGRAM.
µ sin b
From (i) and (ii), sin b = cos b For convex lens
Þ b = 45° (1) Join XY. This represents the ray originating from the
1 1 source and meeting the image Y. Since the ray is
Þ µ= = Þ µ= 2 undeviated after passing through the lens, therefore O
cos 45° 1/ 2
is the optical centre of the lens. Draw Y 1 OY 2
This is the least value of the refractive index of rod for light
entering the rod and not leaving it from the curved surface. perpendicular to AB.
RAY & WAVE OPTICS P-473
(2) Draw a ray from X, parallel to AB. It strikes Y1OY2 at M. (b) EQUATION OFTRAJECTORY
Join MY. It cuts AB at F. This is the focus of the convex According to Snell's law, when light propagates
lens. through a series of parallel layers of different media,
Y1 then
M n sin i = constant
X Let us consider the rectangular state to be made up of
parallel layers such that as we move in the + Y direction,
F the refractive index increases as given by the
A B relationship
n (y) = [ky3/2 + 1]1/2 ...(iv)
Y Applying Snell's law at O, we get 1 × sin 90° = constant
Y2 = 1.
(ii) For concave mirror Again applying Snell's law at B, we get
As the image is real and inverted, the concave mirror n sin i = const. = 1 (from above equation)
has to be placed towards the left of X. To find the exact 2
position of the concave mirror, we draw a line YY'
1 2 æ dy ö
\ n = = cosec
= i 1 + cot= i 1+ ç ÷ ,
perpendicular to AB such that BY = BY' sin i è dx ø

Y' 2
N æ dy ö
ky 3 / 2 +=
1 1+ ç ÷ from (iv)
è dx ø
M X
F dy dy 1/ 2
A O C B Þ = [ky 3 / 2 ]1/ 2 Þ 3 / 4 = k dx = dx (Q k = 1)
dx y
dy
Y Þ ò y3 / 4 = ò dx
Join Y'X and extend the line to meet AB at O. If the concave
mirror is placed at O then after reflection at O, this line will Þ 4y1/4 = x + C where C is an integration constant.
meet Y. But at x = 0, y = 0
To find the radius of curvature of the mirror 4
æ xö
Join X and Y. Let it cut AB at C. This C should be the centre \ C=0\ 4y1/4 = x Þ y = ç ÷
of curvature of the concave mirror. With OC as radius, draw è 4ø
a part of sphere. This is the concave mirror. (c) CO-ORDINATES (x1, y1) OF THE POINT P
To find the focus of the concave mirror At P, y = 1m \ x = 4 y1/ 4 = 4
Draw XM parallel to the principal axis. Join M to Y. Let it cut
AD at F. Therefore, F is the focus of concave mirror. The coordinates of P are (4m, 1m)
22. (a) SLOPE AT P (d) The refractive index at P
To find the slope at B, we draw a tangent to the trajectory np = [ky3/2 + 1]1/2 = [1 (1)3/2 + 1]1/2 = 2
at B. The trajectory is such that as the ray passes If ip is angle of incidence at P then according to Snell's
through the rectangular transparent medium, the ray law,
continuously deviates towards the normal. The tangent
1
at B makes an angle q with the x-axis. Therefore, the np sin ip = 1 Þ sin ip =
2
dy Also by Snell's law, nair sin rp = np sin ip
slope at point B is tan q = ... (i)
dx p
1
Y 1 sin rp = 2´ Þ sin rp = 1 Þ rp =
2 2
AIR Þ After emerging from the rectangular glass slab, the light
P ray becomes parallel to slab length.
B(x,y)
2l
23. (a) Angular width =
i TRANSPARENT d
q MEDIUM
O Q M AIR X
(Origin)
Dark
i is the angle of incidence at B then according to DBQM
p 2q
CENTRAL
i+q + =p ... (ii) d
MAXIMA
2 angular
I
width
Substituting the value of q from (ii) in (i)
æ p ö dy dy Dark
tan ç - i÷ =
è 2 ø dx Þ dx
= cot i ... (iii) D
P-474 Topic-wise Solved Papers - PHYSICS
For wave length 6000 Å
2 ´ 6000 ´ 10-10 A
Angular width =
d 45°
For wavelength l' : Angular width decreases by 30%. The P'
remaining angular width is 70%. New angular width
2l ' 2 ´ 6000 ´ 10-10 45° n1
= 70% of Þ l' = 4200 Å M r=45°
d d n
i'
Let the refractive index of the liquid be µ.
B C
a velocity of light in air ca
mµ = =
velocity of light in medium cm
sin i '
f l l 6000 Applying Snell's law at M, we get n =
= a a = a = = 1.43 sin r
f m lm l m 4200 Since ÐAP'M = 90°
24. (i) The ray incident on AB at M makes an angle of \ ÐAMP' = 45° Þ r = 45°
incidence i. It gets refracted at M. The angle of
refraction is r. Applying Snell's law at M 1.352
\ sin i ' = n sin r = n sin 45° = = 0.956
sin i 2
n= ...(i)
sin r Þ i ' = 72.94°

A a la la
mm =
o
25. (i) l m Þ lm = a µ
45 m

P l D 6300 ´ 10-10 ´ 1.33


\ Fringe width = a a =
90°–r
n1 m µd 1.33 ´ 10-3
rC
i M N n = 6.3 × 10–4m
o
45 (ii) KEY CONCEPT : The shift of fringes when one slit is
B C
covered with thin glass sheet is
From fig
ÐAPM = 180° – (45° + 90° – r) = 45° + r Dt é g m ù
and C = 90° – (45° + r) = 45° – r = d ê m - 1ú
The ray after refraction at M enter the prism and strikes its ëm û
diagonal face AC making an angle C with the normal at P. where, t = thickness of glass sheet.
Here C is the critical angle, therefore, the ray after refraction The shift has to be such that the minima shifts to the axis.
at P makes angle of refraction 90°
Applying Snell's law at P b
For this the shifting of the fringes should be where b is
n1 2
n sin 90°
= Þ sin C = ... (ii) fringe width.
n1 sin C n
From (i), sin i = n sin r = n sin (45° – C) t
= n [sin 45° cos C – cos 45° sin C] m
n
= [ 1 - sin 2 C - sin C ] d
2 b /2
Minima
n é n2 n ù D
sin i = ê 1- 1 - 1 ú [From (ii)]

ë n2 n2 úû
Dt é g m -m m ù b
é 1 ù \ ê ú=
Þ i = sin–1 ê { n 2 - n12 - n1}ú d ë mm û 2
ë 2 û
(ii) Angle of incidence at AB for which the refracted ray
passes through the diagonal face undeviated. For this to
b dm µ 6.3 ´ 10 -4 ´ 10 -3 ´ 1.33
Þ t= =
happen, the angle of incidence of ray MP on diagonal face 2( g µ - m µ) ´ D 2 (1.53 - 1.33) ´ 1.33
should be zero. It means that the ray should strike normal to
= 15.75 × 10–7 m = 1.575 × 10–6 m
AC.
RAY & WAVE OPTICS P-475
26. Dt (m '- 1) 6l R D
=
d d
u2– d v2 + d 6l R 6 ´ 7 ´ 10-7
1 Þ µ' = +1 = +1
t 7 ´ 10-6
µ' = 1.6
New Fringe Width with Green Light
O lg D lR D
b¢= For red light b =
d d
\ Change in fringe width
d (b – b¢) =
D
(l – l ) ... (i)
2 Image d R g
u2 It is given that the central bright fringe shifts by 10–3 to the
v2 position occupied by the 5th dark fringe.
D D 10-3
\ 10–3 = 5b = 5lR Þ =
d d 5l R
Given u2 + v2 = 1.8 m ... (i) Substituting the above value in (i)
The magnification of lens (1) is 2 10-3 æ l g ö 10-3 æ 5 ´ 10-7 ö
(b – b¢) = 1- = ç1 - ÷
v2 + d 5 çè l R ÷ø 5 è 7 ´ 10-7 ø
\ 2= ... (ii)
u2 - d = 5.7 × 10–5 m
From (i) and (ii) 28.
u2 = 0.6 + d , v2 = 1.2 – d v'
Applying lens formula
1 1 1
+ = .... (iii) for lens (1)
v2 + d u2 - d f
O C I M I" I'
1 1 1
+ = ... (iv) for lens (2)
v2 u2 f
From (iii) and (iv) u v
1 1 1 1 The first refraction of the ray coming from the object is
+ = +
v2 + d u2 - d v2 u2 suffered on the left side of convex lens. For this we can
apply the equation
1 1 1 1
Þ + = + ma m g mg - ma
1.2 - d + d 0.6 + d - d 1.2 - d 0.6 + d - + = .... (i)
u v' R
On solving, we get
The image formed by this can be treated as a virtual object
Þ d = 0.6 m for the refracting surface to the right of the convex lens.
Substituting this value in (iv)
mg mw m w - m g
1 1 1 - + = .... (ii)
+ = v' v -R
1.2 - 0.6 0.6 + 0.6 f Adding (i) and (ii)
\ f = 0.4 m
ma m g m g m w m g - ma mw - m g
27. For Red Light - + - + = +
u v' v' v R -R
Dt (µ - 1) Dt (m - 1) 5l R D
d
= 5b Þ
d
=
d
m a mw -mw - m a + 2m g
Þ - + = ... (iii)
u v R
5l 5 ´ 7 ´ 10-7 But according to lens formula
t= = = 7× 10–6 m
(µ - 1) 1.5 - 1 1 æ 1 1 ö
= ( ga m - 1) ç - ÷
For Green Light : Let µ' be the refractive index of glass f è R1 R2 ø
plate of thickness t for green light.
lg = 5 × 10 –7 m 1 æ 2ö
Þ = (1.5 - 1) ç ÷ Þ R = 0.3 m
0.3 è Rø
P-476 Topic-wise Solved Papers - PHYSICS
Substituting the value of R in equation (iii), we get ALTERNATE SOLUTION
4 3 2p 2p
- - 1+ 2 ´ The phase difference f = Dx = (5l + D )
1 4/3 3 2 l l
- + = Þ v = 1.2 m
(0.9) v 0.3
æ fö
As shown in the figure this image will form as I'' behind the We know that I (f) = Imax cos2 ç ÷
è 2ø
mirror. But the ray will get reflected from the mirror in between
and the final image formed will be I. 3 f f p
Since CI '' = 1.2 m and CM = 0.8 m Þ I max = I max cos 2 Þ = 30° =
4 2 2 6
\ MI'' = 0.4 m
Þ MI = 0.4 m [Q MI = MI''] 2p 2p l
Þ = (5l + D ) Þ D x = = 0.3 t
\ CI = 0.4 m 6 l 6
29. Þ t = 9.3 × 10–6 m
E
P'
D
C 70°
t x
n2
S1 m1 n1

i 60°
d P r = 30°
30.

20
m2

°
S2 60° 60°
A B
For Minimum Deviation
D
(a) The rays of wavelength l0 incident at any angle on the
interface BC will pass through without bending,
\ Path difference of rays starting from S1 and S2 and provided the refractive indices n1 and n2 have the same
reaching P is
value for the wavelength l0. Equating the expressions
= t (µ2 – µ1) = t (1.7 – 1.4) = 0.3 t ... (i) of n1 and n2, we get
But the point P lies between the 5th maximum and 6th
minimum (given). 10.8 ´ 10-4 1.80 ´ 10-4
1.20 + = 1.45 +
Therefore, the path difference = 5l + D ...(ii) l 20 l 02
Equating equations (i) and (ii), we get
0.3t = 5l + D ...(iii) (where l0 is in nm)
The path difference D can be determined from the given 1/ 2
æ 9.0 ´ 104 ö
I 3 or l0 = ç ÷ = 600 nm
intensity at P, which is = . è 0.25 ø
I0 4
(b) For the wavelength 600 nm, the combination of prism
The expression I / I0 in terms of D is
acts as a single prism shaped like an isosceles triangle
I æ p Dö (ABE). At the minimum deviation, the ray inside the
= cos 2 ç
I0 è l ÷ø prism will be parallel to the base. Hence, the angle of
refraction on the face AC will be r = 30°.
æ p Dö 3 n
For I / I0 = 3/4, we get cos ç =
è l ÷ø 2 Now sin i = n sin r = n sin 30° =
2
... (1)

The value of n at 600 nm is


pD p l
or = or D =
l 6 6 10.8 ´ 10 4
n = 1.20 + = 1.50 ... (2)
Hence, the thickness of the glass plates (Eq. 3) is (600) 2
0.3 t = 5l + l/6
From (1) and (2),
æ 1 ö æ 31 ö æ 1 ö æ 31 ö
æ 3ö
or t = ç ÷ ç l÷ = çè ÷ø çè ´ 5400Å÷ø
è 0.3 ø è 6 ø 0.3 6 the angle of incidence is i = sin–1 çè ÷ø
4
= 9.3 × 104 Å = 9.3 × 10–6 m
RAY & WAVE OPTICS P-477
31. (a) The path difference (Dx) from the ray starting from S1 Path difference between ray 1 and 2 reaching P = S2M – NS1
and S2 and reaching a point P will be \ Dx1 = d sin q - d sin a (Case 1)
D x = d sin q
P

y S1
S1 M
Na d
q q q
Qq
d Q a
M O 1
y
S2 S2
P
2
D
(=1m)
Path difference between ray 1 and 2 reaching P = NS1 + S1M
We know that the path difference for minimum intensity is
Dx2 = d sin a + d sin q (Case 2)
l Position of Central maxima : Path difference should be
(2m – 1) where m = 1, 2, 3...
2 zero. Therefore Dx1 = 0 or Dx2 = 0
l Þ d sin a = d sin q
\ d sin q = (2m – 1)
2 1
Þ sin q = [Q a = 30°]
(2m - 1)l (2m - 1)0.5 2m - 1 2
Þ sin q = = =
2d 2 ´ 1.0 4 From equation (i), y = 0.58m
Also -1 £ sin q £ 1 . Therefore, possible values of m are ± 1, l
± 2, 0 For first minima; d sin q + d sin a =
2
From D POQ
D sin q l
y = D tan q = ... (i) Þ d sin q = + d sin a
2
1 - sin 2 q
Positions of minima l 0.5 1 1 3
\ sin q = + sin a = + sin 30° = + =
2d 2 ´1 4 2 4
1
For m = +1, sin q = and y = 0.26 From equation (i), y = 1.15m
4
For first minima on the other side
3
m = – 1, sin q = - and y = -1.13 m l
4 Þ sin q = -1
d sin a + d sin q =
2 4
3
m = + 2, sin q = \ y = + 1.13 m \ From (i), y = -0.26 m
4
32. (a) Let the central maxima is obtained at a distance x below
1 O. [This is because a glass sheet is present in front of
m = 0, sin q = - \ y = – 0.26 m
4 S2 which increases its path length to the screen.
(b) WHEN THE INCIDENT BEAM MAKES AN ANGLE Therefore the path length of ray from S1 to the screen
OF 30° WITH X-AXIS should also increase].
Two cases arise as shown by the following two figures.

S1
d/2
P O x
P
d/2
S1 y S2
Na q t
d q
aQ
1 M
Here,
S2
xd æ m g ö
Þ =ç - 1÷ t
2 D è mm ø
P-478 Topic-wise Solved Papers - PHYSICS
Unit vector in the direction of MOM' from fig. (1) is
æ mg ö D æ 1.5 ö (10.4 ´ 10-6 ) (1.5)
Þ x= ç - 1 t ´ = - 1
è mm ø
÷ ç
d è 4/3 ø
÷ ×
0.45 ´ 10-3 6 3$i + 8 3 $j
n$ = 3 4$
, n$ = i$ + j
–3 2 2 1/ 2
= 4.33 × 10 m [(6 3) + (8 3) ] 5 5
æ mg ö To find the angle of refraction, we use Snell's law
(b) For O, path difference = ç - 1÷ t
è mm ø
3 sin i sin 60°
\ Phase difference = = Þ r = 45°
2 sin r sin r
2p æ m g ö 2 ´ 3.14 æ 1.5 ö
f= ç - 1÷ t = ç - 1÷ (10.4 ´ 10 -6 )
l è mm ø 6 ´ 10 -7 è 4 / 3 ø Incident ray
= 6.8 rad
A (sin r) n^
f I i x-y plane
^
We know that I = I0 cos2 \ 2
I 0 = cos (6.8) = 0.75 M –(cos r)k
2 O M' ^r
(c) For maximum at O r
æ mg ö
Again path difference = ç - 1÷ t Refracted ray
m
è m ø
We know that for maxima, path difference = nl
æ mg ö Now, r$ = (sin r ) n$ - (cos r ) k$
\ nl = ç - 1÷ t
è mm ø é3 4 ù
= ( sin 45°) ê $i + $j ú - (cos 45°) k$
æ mg ö t æ 1.5 -6 ë5 5 û
ö 10.4 ´ 10
Þ l= ç - 1÷ = ç - 1÷
è mm ø n è 4 / 3 ø n
1
= [3i$ + 4 $j - 5k$ ]
-6 5 2
1.3 ´ 10 m
=
n 34. First of all, we consider the refraction at plane surface. Here
Putting different values of n for find the wave length in the the image of P will form at I' after refraction from I surface.
range of 0.4 × 10–6 m to 0.7 × 10–6 m we get
l = 0.65 × 10–6 m and 0.433 × 10–6 m
33. m1

Y Y
Z P
1
M 1
>

j M m2
Ö3
+8
>

3i

>

>

8Ö3 j X
Ö3j O I' u = mR
i +8
>

X 6Ö3
O 6Ö3i
>

>

M -10K
For plane surface :
A=6Ö
3i +8 Object distance u = – mR
>

M Ö3j -
>

10k Radius of curvature of the plane surface = ¥


>

(Fig. 1) (Fig. 2) The ray is coming from air and incident on the glas.
Here µ1 = 1, µ2 = 1.5.
Figure 1 shows vector OM ' = 6 3i + 8 3 $j
ur m 2 m1 m 2 - m1 m 2 m1
Figure 2 shows vector A = 6 3$i + 8 3 $j - 10k$ Apply
v
-
u
=
R
;
v
=
u
(as R = ¥)
The perpendicular to line MOM' is Z-axis which has a unit
m1 1.5
vector of k$ . \ Image distance v = u= (– mR) = – 1.5 mR
uuur uuur m2 1.0
Angle between vector MP and OP can be found by dot
product. Now we consider refraction at the curved surface.
uuur uuur Object distance, u = – (1.5 mR + R)
MP . OP = (MP) (OP) cos i
Here, µ2 = 1, µ1 = 1.5, Image distance, v = ¥,
(6 )
3 $i + 8 3 $j - 10k$ .( - k$ )
= cos i
Radius of curvature = – R

(6 3)2 + (8 3)2 + ( -10) 2 + ( -1) 2 1 1.5 1 - 1.5 4


Here, + = \ m=
Þ i = 60° ¥ (1.5m + 1) R -R 3
RAY & WAVE OPTICS P-479
35. (a) For the lens
36.
FLINT(µ¢)

A B
P
Q
B" B 0.6m
R B'
S
A'
A" R R R
ht
e Lig B
30cm hit
30cm 20cm W B

CROWN(µ)
1 1 1 1 1 1
- = Þ - =
v u fl v -20 15 For no deviation condition

1 1 1 1 v 60 é µ -1ù
Þ = - = Þ v = 60 cm, m = = = –3 A' = ê úA
v 15 20 60 u -20 ë µ '- 1 û
The image is formed to the left of the lens, real, inverted and 1.5 - 1
Þ A' = ´ 6° = 4°
three times the actual size (3.6 cm in height below PQ). 1.75 - 1
For the mirror, Now, the angular dispersion produced by crown glass prism
db – dr = A (µb – µr)
1 1 1 1 1 1 2
+ = Þ = - =- Also the angular dispersion produced by flint glass prism
v ' u ' fm v ' -30 30 30 db' – dr' = A' (µb' – µr')
\ Net deviation in blue light
Þ v' = – 15 cm
db = (mb1 - 1) A1 – (mb2 - 1) A2
v' -15 1
m= - = - = = (1.51 – 1) 6° – (1.77 – 1) 4° = – 0.02°
u' 30 2 Similarly Net deviation of red light
1 dr = (m r1 - 1) A1 – (m r2 - 1) A2
size of image = ´ 3.6 = 1.8 cm.
2 = (1.49 – 1) 6° – (1.73 – 1) 4° = 0.02
This image will be inverted w.r.t. the original image and its \ Net dispersion = db – dr = –0.04°
position will be 0.3 cm above RS and 1.5 cm below RS. The \ The magnitude of the net angular dispersion = 0.04
position of the image is 15 cm to the right of the mirror. 37. (i) O is the middle point of two slits S1 and S2.
(b) The path difference between the two rays reflected S1S2 = d = 0.8 mm
from the upper surface AB (shown by ray 1, single arrow
A C
upwards) and lower surface CD (shown by ray 2 double R
arrow pointing upwards) is S1
1 2 y2
A B P O q
n=1.8 a Q
l t C D 40 cm y1
D x = mn × 2t + S2
2
S B D
n=1.5
2m 10 cm
l
Here is the path difference as the ray 1 suffer reflection D1 D2
2
from a denser medium on surface AB y1 40 1
We known that for constructive interference tan a = = =
D1 200 5
Path difference = ml where m is 1, 2,....
1 1 1
l æ 1ö \ sin a = » = » tan a
\ mn × 2t + 2 = ml Þ 2 m nt = çè m - ÷ø l 26 5.1 1.5
2
Path difference DX1 = SS1 – SS2
l 648 æ 1ö
when m = 1, t = = = 90 nm. or DX1 = d sin a = ( 0.8 mm ) ç ÷ = 0.16 mm ... (i)
4 mn 4 ´ 1.8 è 5ø
P-480 Topic-wise Solved Papers - PHYSICS
Let R denotes the position of central bright fringe. Net path The focal length of the combination of two lenses will be
difference will be zero.
1 1 1 1 0.33 0.67
Now DX 2 = S2 R - S1R or DX 2 = d sin q ... (ii) = + = - = ... (i)
f f1 f 2 R R R
For central bright fringe
A convex lens placed on a plane mirror behaves like a
DX 2 - DX1 = 0 or d sin q - DX1 = 0
concave mirror. The image is formed at the object itself if the
or d sin q = DX 1 = 0.16 mm object is placed at centre of curvature of concave mirror.
0.16 1 After refraction through lens, the rays fall on the plane mirror
or (0.8)sin q = 0.16 or sin q = = normally and retrace their path to form image at the object
0.8 5 itself.
1 1 1 \ Focal length of system (f) = 15 cm ... (ii)
\ tan q = = » = sin q \ tan q = y2 From (i) and (ii)
24 4.9 5 D2
1 y2 1 0.67
D 10 = Þ R = 10.05 cm
or 5 = D or y2 = 2 = = 2 cm 15 R
2 5 5 The same situation is repeated with two differences
Hence position of central bright fringe is 2 cm above point
Q on side CD. (a) The object and image distance are now 25 cm and
(ii) m of liquid poured if central fringe is at Q: (b) In place of water there is a new liquid of refraction
index µ
A C
1 1 1 -(m - 1)
S1 Again = and = where f ' is the focal
fl R f' R
P O Q
length of new liquid lens.
y1
\ New combined lens,
S2 1 1 1 1 + (m - 1) 1 - m + 1 2 - m
= + = - = =
R .... (i)
S B D F fl f ' R R R
t
100 mm For new combined lens,
The liquid is poured into vessel upto OQ. 1 1
The central bright fringe is formed at Q. \ = ... (ii)
For central bright fringe net path difference = 0. F 25
(m – 1)t = DX1 or (m – 1)(100) = 0.16 From (i) and (ii)
or m – 1 = 0.0016 or m = 1.0016 2 -µ 1
38. The lens maker formula is = \ µ = 1.6
10.02 25
1 æ 1 1ö 39. (a) Because S is a point source, fringes will be circular.
= (m - 1) ç - ÷
f è R1 R2 ø 2 2
I min æ I - 0.36 I ö æ 0.4 ö 1
When the space between the lens and the mirror is filled (b) =ç ÷ = çè ÷ =
with water, a system of two lenses is formed. I max è I + 0.36 I ø 1.6 ø 16
(i) a glass lens
(ii) a plano concave water lens [Q If intensity of light falling on P directly from S is I, then
For glass lens Here R1 = + R and R2 = – R the intensity of light falling at P after reflection from AB is
0.36 I ]
1 æ1 1 ö 1 (c) For maximum at P, path difference = nl
= (1.5 - 1) ç - =
fl è R - R ÷ø R If AB is shifted by a distance x, it will cause an additional
O path difference of 2x.
I
P
Screen

15 cm
R1 = + R

D
R 2= –R

For water lens


1 æ 1 1 ö - 0.33
= (1.33 - 1) ç - ÷=
fw è - R -¥ ø R h S
A B
R1 = –R

l
R2 = ¥ 2x = l (for minimum value of x) Þ x = = 300 nm
2
RAY & WAVE OPTICS P-481
40. For an object placed at infinity the image after first refraction 42. Use Snell's law
will be formed at a distance v1 n1 sin i = n2 sin r
m 2 m1 m 2 - m1 Here, n1 = 1, n2 = 2 , i = 45°, r = ?
- = ... (i)
v1 - ¥ +R 1 ´ sin 45° 1
Þ sin r = =
Þ r = 30°
Image afte second refraction will be formed at a distance v2 2 2
m3 m 2 m3 - m 2 The angle made by refracted ray at B with normal is 30°.
- = ... (ii) \ Angle made by the first surface with refracted ray BC
v2 v1 +R
is 60°.
Hence the refracted ray at B is parallel to horizontal
arrow.
I2 I1
B
m1 m2 m3 45° C
30°
O
v2 P
A n1 = 1 n2 = 2 n=1.514
v1
Adding (i) and (ii),
60°
µ3 µ3 - µ1 m3 R
- Þ v2 = m - m
v2 R 3 1
\ For refraction at spherical surface, u = ¥
Final image is formed at the focus when incident rays are n2 n1 n2 - n1
Now, - =
parallel. v u R
µ3 R 1.514 1.514 - 1.414
Therefore, focal length will be Þ = or v = 6.056 m
µ3 - µ1 v 0.4
41. (a) Using Snell's law at surface AB \ OP = 6.056 m
43. At the place where maxima for both the wavelengths
3 coincide, y will be same for both the maxima, i.e.,
µair sin 60° = µp sin r Þ = 3 sin r Þ r = 30°
2 n1l1D n2 l 2 D n1 l1 700 7
Now, NN' is the normal to surface AB. = Þ = = =
d d n2 l 2 500 5
\ Ð AMN = 90°
Minimum integral value of n2 is 5.
But Ð QMN = 30° Þ Ð AMQ = 60°
\ Minimum distance of maxima of the two wavelengths
A from central fringe
n l D
30o = 2 2 = 5 × 700 × 10–9 × 103 = 3.5 mm.
d
44. f = 0.3 m, u = – 0.4 m
Q
N' o
Using lens formula
60
60o 30
o 1 1 1
M - = Þ v = 1.2 m
N v - 0.4 0.3
1 1 1
Now we have - = , differentiating w.r.t. t
v u f
B C 1 dv 1 du du
we have - 2 + 2 = 0 given = 0.01 m/s
In DAMQ v dt u dt dt
Ð AQM = 180° – (60° + 30°) = 90° 2
æ dv ö (1.20)
The refracted ray inside the prism hits the other face at 90°; Þ çè ÷ø = ´ 0.01 = 0.09 m/s
hence deviation produced by this face is zero and hence dt (0.4)2
angle of emergence is zero. So, rate of separation of the image (w.r.t. the lens)
(b) Multiple reflections occur in the film for minimum = 0.09 m/s
thickness. udv vdu
The intensity of emergent ray will be maximum if transmitted v -
dm
waves undergo constructive interference. Now, m = Þ = dt 2 dt
\ For minimum thickness, u dt u
Dx = l - (0.4) (0.09) - (1.2) (0.01)
Þ D x = 2mt = l, = = -0.35s -1
(0.4) 2
l Magnitude of rate of change of lateral magnification = 0.35
where t = thickness Þ t = = 125 nm
2µ s –1 .
P-482 Topic-wise Solved Papers - PHYSICS
45. For total internal reflection on interface AB 2. A-p, q, r, s
l When the object is at infinity, a real, inverted and
m1 = 2 B diminished image is formed at the focus of the concave
A
mirror.
i l As the object is brought closer to the mirror, the image
m2 = 2 moves farther, remains real and inverted and increases
i in size (but still it is diminished as compared to the object)
C D l When the object is at C, a real, inverted and same size
m3 = 3 image is formed at C.
l When the object is brought still closer, a real, inverted
1 1µ 2 1 and magnified image is formed beyond C.
sin i = = = = Þ i = 45° l When the object is at focus (F), the image is highly
2µ 2
1 2
µ
2 magnified, real and inverted and formed at infinity.
For total internal reflection in interface CD. l When the object is placed between pole and focus, a
virtual, erect and magnified image is formed behind the
1 m 3
sin i = 3 = 3 = Þ i = 60° mirror.
m 2m 2 ALTERNATE SOLUTION
2
Þ The minimum angle for total internal reflection for both f v
the interface is 60°. v= , m= –
f u
46. (a) For minimum deviation of emergent ray from the first 1-
u
prism. MN is parallel to AC
\ Ð BMN = 60° for different values of u, we get different values of v.
Þ Ð r = 30° (+ ve, –ve or ¥ ). Like wise m comes out to be positive,
Applying Snell's law at M negative, greater than 1 etc.
B
B-q
sin i In case of a convex mirror we always get virtual,
µ=
sin r diminished image between P and F.
sin i = µ sin r P C-p, q, r, s
i 60o N
r When object is at infinity, a real, inverted and dimished
3 M
sin i = 3 ´ sin 30° = Q image is formed at F. When the object is brought closer
2 to the lens, the image moves away from the lens and
Þ i = 60° 60o increases in size. (Image remains between F & 2F).
(b) When the prism DCE A is rotated about C C in When the object is at 2F¢, the image is formed at 2F. The
anticlockwise direction by 60°, as shown in the figure, image is real, inverted and of the same size.
then the final emergent ray SR becomes parallel to the When the object is moved still closer from 2F¢ to F¢, the
incident ray TM. Thus, the angle of deviation becomes image is real, inverted and magnified.
zero.
When the object is at F¢, the image is real, inverted,
B,D E highly magnified and is formed at infinity.
R When the object is between optical centre and F¢, the
M image is virtual erect and magnified.
T S

D-p, q, r, s
A C

MATCH THE FOLLOWING :


1. (A) ® (p).
More the radius of aperture more is the amount of light R1 R2
entering the telescope.
(B) ® (q).
f0
M= 1 é 1 1 ù é R 2 - R1 ù
fe = (m – 1) ê R - R ú = (m – 1) ê R R ú
f ë 1 2û ë 1 2 û
(C) ® (r).
L = f0 + fe Here R1 < R2
(D) ® (p), (q), (r). \ f is positive.
Depends on dispersion of lens, spherical aberration and Therefore it behaves effectively as a convex lens. So
radius of aperture. the case is same as column I (C).
RAY & WAVE OPTICS P-483
3. A-p, s; B-q; C-t; D-r, s, t (C)

S2 P2
P2 P1
S2
P0
P1
P0 S1

S1
Here d ( P0 ) = -l / 2; d ( P1 ) = -l / 4, d ( P2 ) = -l / 6
I ( P0 ) = I1 + I 2 + 2 I1 I2 cos(– p)
For path difference l / 4 , phase difference is p / 2 .
= I1 + I 2 - 2 I1 I 2 = I 0 + I 0 - 2 I 0 = 0
For path difference l / 3 , phase difference is 2p / 3 .
I ( P1 ) = I1 + I 2 + 2 I1 I 2 cos(-p / 2)
Here, S1P0 - S2 P0 = 0
= I1 + I 2 = I 0 + I 0 = 2 I 0
\ d ( P0 ) = 0
Therefore, (p) matches with (A). æ pö
I ( P2 ) = I1 + I 2 + 2 I1 I 2 cos ç - ÷
The path difference for P1 and P2 will not be zero. The è 3ø
intensities at P0 is maximum.
= I1 + I 2 + I1 I 2 = I 0 + I 0 + I0 = 3I 0
I ( P0 ) = I1 + I 2 + 2 I1 I 2 cos 0°
\ I ( P2 ) > I ( P1 )
= ( I1 + I 2 ) 2 = ( I 0 + I 0 ) 2 = 4 I 0 (t) matches (C).
(D)
p
I ( P1 ) = I1 + I 2 + 2 I1 I 2 cos
2 S2 P2
= I1 + I 2 = I 0 + I 0 = 2 I 0 P1
P0
I ( P2 ) = I1 + I 2 + 2 I1 I 2 cos(2 p / 3)
S1
= I1 + I 2 - I1 I 2 = I0 + I 0 - I0 = I 0

\ I ( P0 ) > I ( P1 )
Therefore, (s) matches with (A). Here dP0 = 3l / 4; dP1 = -l / 2; dP2 = -5l /12
(B) æ -3p ö
I ( P0 ) = I1 + I 2 + 2 I1 I 2 cos ç
è 2 ÷ø
S2 P2
P1 = I1 + I 2 = I 0 + I 0 = 2 I 0
P0 I ( P1 ) = I1 + I 2 + 2 I1 I 2 cos(-p)

S1 = I1 + I 2 - 2 I1 I 2 = I0 + I 0 - 2 I 0 I 0 = 0

I ( P2 ) = I1 + I 2 + 2 I1 I 2 cos [ -5p / 6 ]

dP0 =
l
, dP1 = 0, dP2 =
l = I1 + I 2 - 3 I1 I 2 = 2 - 3 I 0( )
4 12 (r), (s), (t) matches (D).
4. A-p, r; B-q,s,t; C-p,r,t, D-q,s
I ( P0 ) = I1 + I 2 + 2 I1 I 2 cos p / 2 (a) When m1 < m2, the ray of light while entering the lens
will bend towards the normal. Therefore p, r are the
= I1 + I 2 = I 0 + I 0 = 2 I 0
correct options
I ( P1 ) = I1 + I 2 + 2 I1 I 2 = 4I 0 (B) When m1> m2, the ray of light while entering the lens
will bend away from the normal. Therefore q,s,t are the
I ( P2 ) = I1 + I 2 + 2 I1 I 2 cos p / 6 correct options.
(C) When m2 = m3, the ray of light while coming out from
= I1 + I 2 + 3 I1 I 2 the lens does not deviate from its path. Therefore p,r,t
are the correct option.
= I0 + I0 + 3 I 0 = (2 + 3)I 0 (D) m2> m3, the ray of light coming out of the lens deviates
Therefore, q match with (B) away from the normal. Therefore q,s are the correct
options.
P-484 Topic-wise Solved Papers - PHYSICS
5. (d) e ® f. For the ray to bend towards the normal at the
prism surface m2 > m1. The ray then moves away 1 3 3
\ sin C = = \ tan C =
from the normal when it emerges out of the m 5 4
rectangular block. Therefore m2 > m3.
e ® g. As there is no deviation of the ray as it emerges OM R
out of the prism, m2 = m1. In D POM, tan C = =
PM 8
e ® h. As the ray emerges out of prism, it moves away
from the normal. Therefore m2 < m1 . As the ray 3
\R = ´ 8 = 6cm
moves away from the normal as it emerges out 4
of the rectangular block, therefore m2 > m3.
3. Using mirror formula for first position
e ® i. At the prism surface, total internal reflection
h2
has taken place. For this sin 45° >
h1
\ m1 > 2 m2. (d) is the correct option.
COMPREHENSION BASED Q UESTIONS : object Image
1. (a) For plane wave fronts the beam of light is parallel.
2. (c) Since points c and d are on the same wavefront,
therefore fd = fc
Similarly, fe = ff \ fd – ff = fc – ff
3. (b) The gap between consecutive wavefronts in medium 2 25
is less than that is medium 1. Therefore, wavelength of u1 = ?, v1 = cm, f = +10cm
3
light in medium 2 is less than that in medium 1. Therefore,
speed of light is more in medium 1 and less in medium 2. 1 1 1 3 1 1
4. (c) As n is negative, therefore direction charges + = + =
v1 u1 f , 25 u1 10
n1 sin q1 = n2 sin q2
5. (b) The physical characteristics remain unchanged.
1 1 3 -5
\ = - = \ u1 = – 50 m
ASSERTION & REASON TYPE QUESTIONS : u1 10 25 250
1. (c) Statement 1 : Using mirror formula for the second position
1 1 1
NOTE : The mirror (spherical) formula + = is 1 1 1 7 1 1 1 1 7
u v f + = Þ + = Þ = -
v2 u2 f 50 u2 10 u2 10 50
valid only for mirrors of small apertures where the size
of aperture is very small as compared to the radius of u2 = – 25m
curvature of the mirror. This statement is true. Change in position of object = 25 m
Statement 2 :
NOTE : Laws of mirror are valid for plane as well as 25 18
Speed of object = ´ = 3km h -1
large spherical surfaces. 30 5
Therefore, statement 2 is wrong.
4. 2
INTEGER VALUE CORRECT TYPE : For the convex spherical refracting surface of oil we apply

f -m1 m 2 m - m1
1. Given f = + 20cm Also m =
+ = 2
f +u u v R

20 7
-1
m25 20 - 25 -30 -1 7/4 4
\ = = =6 \ + = 0
20 ( -24) v 6
m50 -5
24 cm
20 - 50 \ v = 21 cm
2. In the figure, C represents the critical angle For water-oil interface oil
P R -7 4
4 + 3 =0 V¢
+21 V ¢ water
C C I
\ V¢ = 16 cm.
8 cm
C This is the image distance from water-oil interface. Therefore
C the distance of the image from the bottom of the tank is 2
cm.
R O
M
RAY & WAVE OPTICS P-485

1. (b) KEY CONCEPT : The resolving power of a telescope is an even number. The number of images formed is
given by
D
R.P =
1.22 l where D = diameter of the objective lens n=
360
-1 =
360
-1 = 4 - 1 = 3
q 90
l = wavelength of light.
10. (b) The incident angle is 45° .
Clearly, larger the aperture, larger is the value of D,
more is the resolving power or resolution. Incident angle > critical angle, i > ic
2. (a) KEY CONCEPT : When two plane mirrors are inclined 1
at each other at an angle q then the number of the \ sin i > sin ic or sin 45 > sin ic sin ic =
n
images of a point object placed between the plane
360° 360° 1 1 1
- 1 , if \ sin 45° > or > Þn> 2
mirrors is is even n 2 n
q q
11. (c) KEY CONCEPT : The focal length(F) of the final mirror
360°
\ Number of images formed = -1 = 5 1 2 1
60º is F = f + f
l m
3. (a) The phenomenon of polarisation is shown only by
transverse waves.
1 æ 1 1ö
4. (d) KEY CONCEPT : The resolving power of an optical Here = ( µ - 1) ç - ÷
fl è R1 R2 ø
instrument is inversely proportional to the wavelength
of light used.
é1 1 ù 1
= (1.5 - 1) ê - =
( R.P )1 l 2 5
= = ë a -30 úû 60
( R.P )2 l1 4
1 1 1 1
5. (a) In an optical fibre, light is sent through the fibre without \ = 2´ + = \ F = 10 cm
F 60 30 / 2 10
any loss by the phenomenon of total internal reflection
as shown in the figure. The combination acts as a converging mirror. For the
object to be of the same size of mirror,
n2 u = 2F = 20 cm
n1 12. (d) The angle of incidence for total polarization is given
n1> n 2
by tan q = n Þ q = tan -1 n
Where n is the refractive index of the glass.
13. (b) For constructive interference d sin q = nl
6. (b) Optical fibres form a dielectric wave guide and are free
from electromagnetic interference or radio frequency n
given d = 2 l Þ sin q =
interference. 2
7. (d) For the phenomenon of interference we require two n = 0, 1, – 1, 2, – 2 hence five maxima are possible
sources of light of same frequency and having a definite 14. (a) Frequency remains constant during refraction
phase relationship (a phase relationship that does not
change with time) 1 c
vmed = =
8. (c) A real, inverted and enlarged image of the object is µ0 Î0 ´4 2
formed by th e objective lens of a compound
microscope. l med vmed c / 2 1
= = =
l air vair c 2
360 360
9. (b) When q = 90° then = = 4 \ wavelength is halved and frequency remains
q 90
unchanged
P-486 Topic-wise Solved Papers - PHYSICS
18. (d) The shape of interference fringes formed on a screen
1 3 3 3 R
15. (a) sin qc = = or tan qc = = = in case of a monochromatic source is a straight line.
m 4 16 - 9 7 12 Remember for double hole experiment a hyperbola is
generated.

2
R æ sin f ö p
19. (a) I = I 0 çç ÷÷ and f = ( b sin q)
è f ø l

qc qc
12 cm When the slit width is doubled, the amplitude of the
wave at the centre of the screen is doubled, so the
intensity at the centre is increased by a factor 4.

20. (b) I = I 0 cos2 q

I0
Intensity of polarized light =
2
36
Þ R= cm
7 I0 I 0
Þ Intensity of untransmitted light = I 0 - =
2 2
16. (b) y l 21. (a) For a thin prism, D = (m – 1) A
³ 1.22
D d
Since lb < lr Þ mr < mb Þ D1 < D2

Þ D£ yd 10 - 3 ´ 3 ´ 10 - 3 = 30 22. (a) For path difference of l , the phase difference is 2 p


= » 5m
(1.22) l (1.22) ´ 5 ´ 10 - 7 6.1
l
For path difference of , the phase difference is
\ Dmax = 5m 6

2p ´ l / 6 p
1 æ 1.5 ö æ 1 1ö =
17. (b) =ç - 1÷ ç - ÷ .... (i) l 3
fa è 1 ø è R1 R2 ø
p
\ Intensity I = I1 + I 2 + 2 I1 I 2 cos
1 æ mg öæ 1 1ö 3
=ç -1 -
f m è m m ÷ø çè R1 R2 ÷ø
\ I = I1 + I 2 + I1 I 2
1 æ 1.5 ö æ 1 1ö .... (ii) when I1= I2 = I' (say) then I = 3I'
=ç - 1÷ ç - ÷
f m è 1.6 ø è R1 R2 ø
( )
2
Imax = I1 + I 2
æ ö
( ) = (2 I ')
f m ç 1.5 - 1 ÷ 2 2
Dividing (i) by (ii), = =–8 = I'+ I' = 4I '
f a ç 1.5 ÷
ç - 1÷
è 1.6 ø
I 3
\ =
Imax 4
1 1
Pa = - 5 = Þ fa = -
fa 5 ALTERNATE SOLUTION : The intensity of light at
any point of the screen where the phase difference due
1 8 to light coming from the two slits is f is given by
Þ f m = -8 ´ f a = - 8 ´ - =
5 5
æ fö
m 1.6 I = Io cos2 ç ÷ where I is the maximum intensity..
Pm = = ´ 5 = 1D è 2ø 0
fm 8
RAY & WAVE OPTICS P-487
NOTE : This formula is applicable when I1= I2. Here
Q
f = p3
n
90 – a
2 a
I p æ 3ö 3
\ = cos2 = ç ÷ = 28. (c) q P
I0 6 è 2 ø 4

23. (c) Power of combination is given by


P = P1 + P2 = (– 15 + 5) D = – 10 D.
Applying Snell’s law at Q
1 1 1 sin 90° 1 1
f Þ f = P = -10
Now, P = metre n= = cos a =
sin(90º -a ) cos a \
n

æ1 ö
\ f = - ç ´ 100÷ cm = -10 cm. 1 n2 - 1
è 10 ø 2
\ sin a = 1 - cos a = 1 - 2 = ...(1)
n n
24. (d) The electron beam will be diffracted and the maxima is
obtained at y = 0. Also the distance between the first Applying Snell’s Law at P
minima on both side will be greater than d.
sin q 2
25. (c) This graph suggest that when n= Þ sin q = n ´ sin a = n - 1 ; from (1)
sin a
u = – f, v = + µ
2
æ 2 ö 4 1
\ sin q = ç -1 = -1 =
v (cm) è 3 ÷ø 3 3

æ 1 ö
or q = sin -1 ç ÷
è 3ø

f
29. (d) Experimental
–f u (cm) |v| curve

Straight
When the object is moved further away from the lens, line
v decreases but remains positive. When u is at – µ ,
(2f, 2f)
v = f. P
This is how image formation takes place for different
positions of the object in case of a convex lens. 45°

ALTERNATE SOLUTION |u|


This graph obeys the lens equation

1 1 1 1 1 1
- = For a convex lens - =
v u f v u f
where f is a positive constant for a given convex lens. when u = -a , v = + f
26. (a) To find the refractive index of glass using a travelling
microscope, a vernier scale is provided on the when u = - f , v = +a
microscope
Then u = -2 f , v = 2 f
27. (b) Third bright fringe of known light coincides with the
4th bright fringe of the unknown light.
f
Also v =
3(590) D 4lD 3 f
= 1+
\ Þ l = ´ 590 = 442.5 nm u
d d 4
P-488 Topic-wise Solved Papers - PHYSICS
As |u| increases, v decreases for | u |> f . The graph
From Snell's law, 2 sin i = 3 sin r
between |v| and |u| is shown in the figure. A straight line
passing through the origin and making an angle of Ð r = 45°
45°with the x-axis meets the experimental curve at 34. (b) A phase change of p rad appears when the ray reflects
P (2f, 2f ). at the glass-air interface. Also, the centre of the
30. (b) In the medium, the refractive index will decrease from interference pattern is dark.
the axis towards the periphery of the beam. 35. (a) From mirror formula
Therefore, the beam will move as one move from the
axis to the periphery and hence the beam will converge. 1 1 1 dv v 2 æ du ö
+ = so, =- 2ç ÷
v u f dt u è dt ø

2
dv æ f ö du
Þ = -ç
dt è u - f ø÷ dt
Decreasing m
Axis
dv 1
Þ = m/s
dt 15

36. (a) Path difference at p


Dx1 = 0
31. (d) Initially the parallel beam is cylindrical . Therefore, the \ Phase difference at P
wavefront will be planar.
32. (a) The speed of light (c) in a medium of refractive index Df1 = 0°
(m) is given by Intensity at p

c0 I1 = I 0 + I 0 + 2 I0 cos 0° = 4 I 0
m= , where c0 is the speed of light in vacuum
c Path difference at Q

c0 c0 l
\c= = m + m (I ) Dx2 =
m 0 2 4

As I is decreasing with increasing radius, it is maximum \ Phase difference at Q


on the axis of the beam. Therefore, c is minimum on the
2p l æ p ö
axis of the beam. DfD2 = . =ç ÷
l 4 è 2ø
33. (a) Angle of incidence is given by
Intensity at Q.

cos (p–i) =
(6 )
3iˆ + 8 3 ˆj - 10kˆ .kˆ
p
20 I 2 = I 0 + I 0 + 2 I0 cos = 2I0
2
1
– cos i = – I1 4 I 0 2
2 Thus, = =
I 2 2I0 1
Ð i = 60 °
37. (a) For coherent sources :
I1 = 4 I 0
For incoherent sources
I 2 = 2I 0

I1 2
\ =
I2 1
RAY & WAVE OPTICS P-489
r r r r
38. (b) k is ^ H and k is also ^ to E
r r r r r
or In other words X || E and k || E ´ B
m2 Kerosene h2
42. (d) Let a1 = a, I1 = a12 = a2
a2 = 2a, I2 = a22 = 4a2
m1 Water h1
I2 = 4I1
Ir = a12 + a22 + 2a1a2 cos f

é 1ù = I1 + I 2 + 2 I1 I2 cos f
Apparent shift due to water = h1 ê1 - ú
ë m1û
Ir = I1 + 4 I1 + 2 4 I12 cos f
é 1 ù
Apparent shift due to kerosene = h2 ê1 - ú Þ Ir = 5I1 + 4 I1 cos f … (1)
ë m2 û
Now, Imax = ( a1 + a2 )2 = (a + 2 a)2 = 9a2
Thus, total apparent shift :

æ 1ö æ 1 ö Imax
Imax = 9I1 Þ I1 =
= h1 ç1 - ÷ + h2 ç1 - ÷ 9
è m1 ø è m2 ø
Substituting in equation (1)
39. (a) We know that m R < m B
5 Imax 4 I max
Ir = + cos f
1 æ 1 1ö 9 9
1 1
and = (m - 1) ç - ÷ Þ > Þ fR > fB.
f è R1 R2 ø fB fR Imax
Ir = [ 5 + 4 cos f]
40. (b) When viewed through a polaroid which is rotated then 9
the light from a clear blue portion of the sky shows a
rise and fall of intensity. Imax é 2f ù
Ir = êë 5 + 8 cos 2 - 4 úû
9
Incident sunlight
(unpolarised)
Imax é 2 fù
Ir = êë1 + 8 cos
9 2 úû
Sun 43. (d) The focal length of the lens
Scattered light 1 1 1
(polarised) = - = 1 + 1 = 20 + 1 = 21
f v u 12 240 240 240

240
f = cm
21
to observer
æ 1ö
41. (b) Q The E.M. wave are transverse in nature i.e., Shift = t ç 1 - ÷
è mø
r r
k ´E r
= =H …(i)
mw æ 1 ö 1
1ç 1 - ÷ = 1´ 3
è 3/2 ø
r
r
where H = B 1 35
m Now v' = 12 - = cm
3 3
r r
k ´H r Now the object distance u.
and = -E … (ii)
we
P-490 Topic-wise Solved Papers - PHYSICS
45. (c) Relation between intensities
1 3 21 1 é 3 21 ù
= - = -
u 35 240 5 êë 7 48 úû 45° B
I0 (I0/2)
1 1 é 48 - 49 ù (unpolarised) IR
=
u 5 êë 7 ´ 16 úû
A
u = –7 ×16 × 5 = – 560 cm = – 5.6 m

Velocity of light in vacuum æI ö I 1 I


44. (c) \ n = I r = ç 0 ÷ cos2 (45°) = 0 ´ = 0
Velocity of light in medium è 2ø 2 2 4

3 46. (d) It will be concentric circles.


\ n=
2 R = 3cm 47. (c) For the prism as the angle of incidence (i) increases,
32 + (R – 3mm)2 = R2 the angle of deviation (d) first decreases goes to
Þ 32 + R2 – 2R(3mm) + (3mm)2 = R2 3mm minimum value and then increases.
Þ R » 15 cm

1 æ 3 öæ 1 ö
= ç –1 ÷ ç ÷ Þ f = 30 cm
f è 2 ø è 15 ø
Modern Physics
17
FILL IN THE BLANKS : 11. The wavelength of the characteristic X-ray K a line emitted
1. To produce characteristic X-rays using a Tungsten target by a hydrogen like element is 0.32 Å. The wavelength of the
in an X-ray generator, the accelerating voltage should be K b line emitted by the same element will be ............
greater than ___________ volts and the energy of the (1990 - 2 Marks)
characteristic radiation is _____eV. 12. The Bohr radius of the fifth electron of phosphorous atom
(The binding energy of the innermost electron in Tungsten (atomic number = 15) acting as a dopant in silicon (relative
is 40 keV). (1983 - 2 Marks) dielectric constant = 12) is .................... Å (1991 - 1 Mark)
2. The radioactive decay rate of a radioactive element is found 13. For the given circuit shown in fig. to act as full wave rectifier,
to be 103 disintegration/second at a certain time. If the half the a.c. input should be connected across .................... and
life of the element is one second , the decay rate after one .................... and the d.c. output would appear across
second is _______ and after three seconds is _______. .................... and ................ (1991 - 1 Mark)
(1983 - 2 Marks)
B A
3. The maximum kinetic energy of electrons emitted in the
photoelectric effect is linearly dependent on the ...... of the
incident radiation. (1984- 2 Marks)
238
4. In the Uranium radioactive series the initial nucleus is 92 U
206
and the final nucleus is 82 Pb . When the Uranium nucleus C D
decays to lead, the number of a-particles emitted is ......and 14. In an X-ray tube, electrons accelerated through a potential
the number of b-particles emitted is ..... (1985 - 2 Marks) difference of 15, 000 volts strike a copper target. The speed
5. When the number of electrons striking the anode of an X- of the emitted X-ray inside the tube is ................... m/s
ray tube is increased, the ........ of the emitted X-rays (1992 - 1 Mark)
increases, while when the speeds of the electrons striking 15. In the Bohr model of the hydrogen atom, the ratio of the
the anode are increased, the cut-off wavelength of the kinetic energy to the total energy of the electron in a quantum
emitted X-rays......... (1986 - 2 Marks) state n is .................... (1992 - 1 Mark)
6. When Boron nucleus ( B) is bombarded by neutrons,
10
5 16. In the nuclear process, 6 C11 ¾¾ ® 5 B11 + b + + X , X
a -particles are emitted. The resulting nucleus is of the stands for ............. (1992 - 1 Mark)
element ........ and has the mass number ......(1986 - 2 Marks) 17. In a ..... biased p-n junction, the net flow of holes is from the
7. Atoms having the same ........... but different ........... are called n region to the p region. (1993 - 1 Mark)
isotopes. (1986 - 2 Marks) 18. A potential difference of 20 kV is applied across an X-ray
8. The binding energies per nucleon for deuteron (1H2) and tube. The minimum wavelength of X-rays generated
helium (2He4) are 1.1 MeV and 7.0 MeV respectively. The is...........Å. (1996 - 2 Marks)
energy released when two deuterons fuse to form a helium 19. The wavelength of Ka X-rays produced by an X-ray tube is
nucleus (2He4) is .......... (1988 - 2 Marks) 0.76Å. The atomic number of the anode material of the tube
9. In the forward bias arrangement of a p-n junction rectifier, is..... (1996 - 2 Marks)
the p end is connected to the ........terminal of the battery 20. Consider the following reaction :
and the direction of the current is from ............to ..................in 2 2 4
the rectifier. (1988 - 2 Marks) 1 H + 1 H =2 He + Q
10. ........... biasing of p-n junction offers high resistance to Mass of the deuterium atom = 2.0141 u
current flow across the junction. The biasing is obtained by Mass of helium atom = 4.0024 u
connecting the p- side to the .......... terminal of the battery. This is a nuclear ............. reaction in which the energy Q
(1990 - 2 Marks) released is ............. MeV. (1996 - 2 Marks)
P-492 Topic-wise Solved Papers - PHYSICS
21. The recoil speed of a hydrogen atom after it emits a photon 6. Consider the spectral line resulting from the transition
in going from n = 5 state to n = 1 state is ....... m/s. n = 2 ® n = 1 in the atoms and ions given below. The
(1997C - 1 Mark) shortest wavelength is produced by (1983 - 1 Mark)
(a) Hydrogen atom (b) Deuterium atom
TRUE / FALSE : (c) Singly ionized Helium (d) Doubly ionised Lithium
1. The kinetic energy of photoelectrons emitted by a 7. The equation
photosensitive surface depends on the intensity of the 4 11 H + ¾¾
® 42 He 2 + + 2e – + 26 MeV represents
incident radiation. (1981- 2 Marks) (1983 - 1 Mark)
2. In a photoelectric emission process the maximum energy of
(a) b -decay (b) g-decay
the photo-electrons increases with increasing intensity of
(c) fusion (d) fission
the incident light. (1986 - 3 Marks)
8. Fast neutrons can easily be slowed down by
3. For a diode the variation of its anode current Ia with the
(1994 - 1 Mark)
anode voltage Va at two different cathode temperatures T1 (a) the use of lead shielding
and T2 is shown in the figure. The temperature T2 is greater (b) passing them through water
than T1. (1986 - 3 Marks) (c) elastic collisions with heavy nuclei
(d) applying a strong electric field.
T2
9. Consider a particles, b particles and g - rays, each having
T1 an energy of 0.5 MeV. In increasing order of penetrating
Ia
powers, the radiations are: (1994 - 1 Mark)
(a) a, b, g (b) a, g , b
(c) b, g , a (d) g , b, a
10. An energy of 24.6 eV is required to remove one of the
electrons from a neutral helium atom. The energy in (eV)
Va required to remove both the electrons from a neutral helium
atom is (1995S)
4. The order of magnitude of the density of nuclear matter is (a) 38.2 (b) 49.2
104 kg m–2. (1989 - 2 Marks) (c) 51.8 (d) 79.0
11. A radioactive material decays by simultaneous emission of
MCQ's WITH ONE CORRECT ANSWER :
two particles with respective half-lives 1620 and 810 years.
1. The plate resistance of a triode is 3 × 103 ohms and its The time, in years, after which one-fourth of the material
mutual conductance is 1.5 × 10–3 amp/volt. The amplification remains is (1995S)
factor of the triode is (1981- 2 - 3 Marks) (a) 1080 (b) 2430
(a) 5 × 10–5 (b) 4.5 (c) 3240 (d) 4860
(c) 45 (d) 2 × 10 5 12. The probability of electrons to be found in the conduction
2. The half life of radioactive Radon is 3.8 days. The time at the band of an intrinsic semiconductor at a finite temperature
(1995S)
1 (a) increases exponentially with increasing band gap
end of which th of the radon sample will remain
20 (b) decreases exponentially with increasing band gap
(c) decreases with increasing temperature
undecayed is (given log10 e = 0.4343 ) (1981- 2 Marks) (d) is independent of the temperature and the band gap
(a) 3.8 days (b) 16.5 days 13. A full-wave rectifier circuit along with the out-put is shown
(c) 33 days (d) 76 days. in Figure. The contribution (s) from the diode 1 is (are)
3. An alpha particle of energy 5 MeV is scattered through 180° (1996 - 2 Marks)
by a fixed uranium nucleus. The distance of closest approach
is of the order of (1981- 2 Marks) V
(a) 1 Å (b) 10–10 cm
(c) 10–12cm (d) 10–15 cm
4. Beta rays emitted by a radioactive material are
0 t
(1983 - 1 Mark) 1 out-put
(a) electromagnetic radiations +
(b) the electrons orbiting around the nucleus
(c) charged particles emitted by the nucleus
(d) neutral particles V
5. If elements with principal quantum number n > 4 were not
allowed in nature, the number of possible elements would A B C D
0 t
be (1983 - 1 Mark) out-put
(a) 60 (b) 32 (a) C (b) A,C
(c) 4 (d) 64 (c) B, D (d) A,B,C, D.
MODERN PHYSICS P-493
14. The K a X-ray emission line of tungsten occurs at 21. The maximum kinetic energy of photoelectrons emitted from
a surface when photons of energy 6 eV fall on it is 4 eV. The
l = 0.021nm. The energy difference between K and L levels stopping potential, in volt, is (1997 - 1 Mark)
in this atom is about (1997C - 1 Mark) (a) 2 (b) 4
(a) 0.51 MeV (b) 1.2 MeV (c) 6 (d) 10
(c) 59 keV (d) 13.6 eV 22. In hydrogen spectrum the wavelength of Ha line is 656 nm,
15. The electrical conductivity of a semiconductor increases whereas in the spectrum of a distant galaxy, Ha line
when em radiation of wavelength shorter than 2480 nm is wavelength is 706 nm. Estimated speed of the galaxy with
respect to earth is, (1999S - 2 Marks)
incident on it. The band gap (in eV) for the semiconductor is
(a) 2 × 108 m/s (b) 2 × 107 m/s
(a) 0.9 (b) 0.7 (1997C - 1 Mark)
(c) 2 × 106 m/s (d) 2 × 105 m/s
(c) 0.5 (d) 1.1 23. A particle of mass M at rest decays into two particles of
16. The dominant mechanisms for motion of charge carriers in masses m1 and m2, having non-zero velocities. The ratio of
forward and reverse biased silicon p-n junctions are the de Broglie wavelengths of the particles, l1/l2, is
(1997C - 1 Mark) (1999S - 2 Marks)
(a) drift in forward bias, diffusion in reverse bias (a) m1/m2 (b) m2/m1
(b) diffusion in forward bias, drift in reverse bias
(c) diffusion in both forward and reverse bias (c) 1.0 (d) m2 / m1
(d) drift in both forward and reverse bias 24. Which of the following is a correct statement?
17. Masses of two isobars 29Cu64 and 30Zn64 are 63.9298 u (1999S - 2 Marks)
and 63.9292 u respectively. It can be concluded from these (a) Beta rays are same as cathode rays
data that (1997C - 1 Mark) (b) Gamma rays are high energy neutrons
(a) both the isobars are stable. (c) Alpha particles are singly ionised helium atoms
(d) Protons and neutrons have exactly the same mass
(b) Zn 64 is radioactive, decaying to Cu 64 through
25. Order of magnitude of density of uranium nucleus is, [mp =
b - decay. 1.67 × 10–27kg] (1999S - 2 Marks)
(c) Cu64 is radioactive, decaying to Zn 64 through g-decay. (a) 1020kg/m3 (b) 1017kg/m3
(d) Cu64 is radioactive, decaying to Zn64 through b-decay. (c) 1014kg/m3 (d) 1011kg/m3
18. As per Bohr model, the minimum energy (in eV) required to 26. 22Ne nucleus, after absorbing energy, decays into two a-
remove an electron from the ground state of doubly ionized particles and an unknown nucleus. The unknown nucleus
Li atom (Z = 3) is (1997 - 1 Mark) is (1999S - 2 Marks)
(a) 1.51 (b) 13.6 (a) nitrogen (b) carbon
(c) 40.8 (d) 122.4 (c) boron (d) oxygen
19. The circuit shown in the figure contains two diodes each 27. Binding energy per nucleon vs mass number curve for nuclei
with a forward resistance of 50 ohms and with infinite is shown in the Figure. W, X, Y and Z are four nuclei indicated
backward resistance. If the battery voltage is 6V, the current on the curve. The process that would release energy is
through the 100 ohm resistance (in amperes) is (1999S - 2 Marks)
Binding Energy/nucleon in Mev

(1997 - 1 Mark)

150 W
Y
8.5 X
8.0
7.5 W
50 W
5.0 Z

0 30 60 90 120
6V 100 W Mass number of nuclei

(a) Y ® 2Z (b) W ® X + Z
(a) zero (b) 0.02 (c) W ® 2Y (d) X ® Y + Z
(c) 0.03 (d) 0.036. 28. Imagine an atom made up of a proton and a hypothetical
particle of double the mass of the electron but having the
20. Which of the following statements is not true?
same charge as the electron. Apply the Bohr atom model
(1997 - 1 Mark)
and consider all possible transitions of this hypothetical
(a) The resistance of intrinsic semiconductors decrease particle to the first excited level. The longest wavelength
with increase of temperature
photon that will be emitted has wavelength l (given in
(b) Doping pure Si with trivalent impurities give p-type
terms of the Rydberg constant R for the hydrogen atom)
semiconductors equal to (2000S)
(c) The majority carriers in n-type semiconductors are holes (a) 9/(5R) (b) 36/(5R)
(d) A p-n junction can act as a semiconductor diode (c) 18/(5R) (d) 4/R
P-494 Topic-wise Solved Papers - PHYSICS
29. The electron in a hydrogen atom makes a transition from an 35. A radioactive sample consists of two distinct species having
excited state to the ground state. Which of the following equal number of atoms initially. The mean life time of one
statements is true ? (2000S) species is t and that of the other is 5t. The decay products
(a) Its kinetic energy increases and its potential and total in both cases are stable. A plot is made of the total number
energies decreases. of radioactive nuclei as a function of time. Which of the
(b) Its kinetic energy decreases, potential energy increases following figures best represent the form of this plot?
and its total energy remains the same. (2001S)
(c) Its kinetic and total energies decrease and its potential
energy increases.
(d) Its kinetic, potential and total energies decrease.
30. Two radioactive materials X1 and X2 have decay constants
10l and l respectively. If initially they have the same
number of nuclei, then the ratio of the number of nuclei of
X1 to that of X2 will be 1/e after a time (2000S)
1 1
(a) (b)
10l 11l
11 1 (a) (b)
(c) (d)
10l 9l
31. Electrons with energy 80 keV are incident on the tungsten
target of an X-ray tube. K-shell electrons of tungsten have
72.5 keV energy. X-rays emitted by the tube contain only
(2000S)
(a) a continuous X-ray spectrum (Bremsstrahlung) with a
minimum wavelength of 0.155Å
(b) a continuous X-ray spectrum (Bremsstrahlung) with
all wavelengths (c) (d)
(c) the characteristic X-ray spectrum of tungsten 36. The potential difference applied to an X-ray tube is 5kV and
(d) a continuous X-ray spectrum (Bremsstrahlung) with a the current through it is 3.2mA. Then the number of electrons
minimum wavelength of 0.155Å and the characteristic striking the target per second is (2002S)
X-ray spectrum of tungsten.
32. The electron emitted in beta radiation originates from (a) 2 ´ 1016 (b) 5 ´ 10 6
(a) inner orbits of atoms (2001S) (c) 1´ 1017 (d) 4 ´ 1015
(b) free electrons existing in nuclei 37. ++
A Hydrogen atom and a Li ion are both in the second
(c) decay of a neutron in a nucleus
(d) photon escaping from the nucleus excited state. If l H and l Li are their respective electronic
33. The transition from the state n = 4 to n = 3 in a hydrogen-like angular momenta, and EH and ELi their respective energies,
atom results in ultraviolet radiation. Infrared radiation will then (2002S)
be obtained in the transition (2001S) (a) l H > l Li and |EH| > |ELi| (b) l H = l Li and |EH| < |ELi|
(a) 2 ® 1 (b) 3 ® 2
(c) 4 ® 2 (d) 5 ® 4 (c) l H = l Li and |EH| > |ELi| (d) l H < l Li and |EH| < |ELi|
34. The intensity of X-rays from a Coolidge tube is plotted 38. The half-life of 215At is 100 m s. The time taken for the
against wavelength l as shown in the figure. The minimum radioactivity of a sample of 215At to decay to 1/16th of its
wavelength found is lC and the wavelength of the Ka line is initial value is (2002S)
lK. As the accelerating voltage is increased (2001S) (a) 400 m s (b) 6.3 m s
I (c) 40 m s (d) 300 m s
39. Which of the following processes represents a g -decay ?

A A
(a) X z + g ¾¾
® X Z -1 + a + b (2002S)

A A- 3
(b) X z + 1no ¾¾
® X Z -2 + c
A
lC lK l (c) A
X z ¾¾
® XZ + f

(a) lK - lC increases (b) lK - lC decreases (d) A A


(c) lK increases (d) lK decreases X z + e-1 ¾¾
® X Z -1 + g
MODERN PHYSICS P-495
40. The electric potential between a proton and an electron is 48. A photon collides with a stationary hydrogen atom in ground
r state inelastically. Energy of the colliding photon is 10.2 eV.
given by V = V0 ln , where r0 is a constant. Assuming After a time interval of the order of micro second another
r0
photon collides with same hydrogen atom inelastically with
Bohr’s model to be applicable, write variation of rn with n, n an energy of 15 eV. What will be observed by the detector?
being the principal quantum number? (2003S) (2005S)
(a) rn µ n (b) rn µ 1/n (a) One photon of energy 10.2 eV and an electron of
(c) rn µ n2 (d) rn µ 1/n2 energy 1.4 eV
257
41. If the atom 100Fm follows the Bohr model and the radius (b) 2 photon of energy of 1.4 eV
of 100Fm257 is n times the Bohr radius, then find n. (2003S) (c) 2 photon of energy 10.2 eV
(a) 100 (b) 200
(d) One photon of energy 10.2 eV and another photon of
(c) 4 (d) 1/4
1.4 eV
42. For uranium nucleus how does its mass vary with volume?
4 9 . A beam of electron is used in an YDSE experiment. The slit
(2003S)
width is d. When the velocity of electron is increased, then
(a) m µ V (b) m µ 1/V
(a) no interference is observed (2005S)
(c) m µ V (d) m µ V2 (b) fringe width increases
43. A nucleus with mass number 220 initially at rest emits an a - (c) fringe width decreases
particle. If the Q value of the reaction is 5.5 MeV, calculate (d) fringe width remains same
the kinetic energy of the a -particle (2003S) 50. If a star can convert all the He nuclei completely into oxygen
(a) 4.4 MeV (b) 5.4 MeV nuclei, the energy released per oxygen nuclei is [Mass of
(c) 5.6 MeV (d) 6.5 MeV He nucleus is 4.0026 amu and mass of Oxygen nucleus is
44. In a photoelectric experiment anode potential is plotted 15.9994 amu] (2005S)
against plate current. (2004S)
(a) 7.6 MeV (b) 56.12 MeV
(c) 10.24 MeV (d) 23.9 MeV
I 221
51. 87 Ra is a radioactive substance having half life of 4 days.
Find the probability that a nucleus undergoes decay after
C B two half lives (2006 - 3M, –1)
A 1
(a) 1 (b)
2
V
3 1
(c) (d)
(a) A and B will have different intensities while B and C 4 4
will have different frequencies 52. In the options given below, let E denote the rest mass energy
(b) B and C will have different intensities while A and C of a nucleus and n a neutron.The correct option is
will have different frequencies (2007)
(c) A and B will have different intensities while A and C
will have equal frequencies æ 236 ö æ137 ö æ 97 ö
(a) Eç U÷ > Eç I ÷ + E ç Y ÷ + 2 E (n )
(d) B and C will have equal intensities while A and B will è 92 ø è 53 ø è 39 ø
have same frequencies
45. A 280 days old radioactive substance shows an activity of æ 236 ö æ137 ö æ 97 ö
(b) Eç U÷ < Eç I ÷ + E ç Y ÷ + 2 E (n )
6000 dps, 140 days later its activity becomes 3000 dps. è 92 ø è 53 ø è 39 ø
What was its initial activity? (2004S)
(a) 20000 dps (b) 24000 dps æ 236 ö æ140 ö æ 94 ö
(d) 12000 dps (d) 6000 dps (c) Eç U÷ < Eç Ba ÷ + E ç Kr ÷ + 2 E (n)
è 92 ø è 56 ø è 36 ø
46. A proton has kinetic energy E = 100 keV which is equal to
that of a photon. The wavelength of photon is l2 and that
æ 236 ö æ 140 ö æ 94 ö
of proton is l1. The ration of l2/l1 is proportional to (d) Eç U÷ = Eç Ba ÷ + E ç Kr ÷ + 2 E (n)
(a) E 2 (b) E 1/2 (2004S) è 92 ø è 56 ø è 36 ø
(c) E –1 (d) E –1/2 53. The largest wavelength in the ultraviolet region of the
47. Ka wavelength emitted by an atom of atomic number Z = 11 hydrogen spectrum is 122 nm. The smallest wavelength in
is l. Find the atomic number for an atom that emits Ka the infrared region of the hydrogen spectrum (to the nearest
radiation with wavelength 4l. (2005S) integer) is
(a) Z = 6 (b) Z = 4 (a) 802 nm (b) 823 nm (2007)
(c) Z = 11 (d) Z = 44 (c) 1882 nm (d) 1648 nm
P-496 Topic-wise Solved Papers - PHYSICS
54. Electrons with de-Broglie wavelength l fall on the target in
an X-ray tube. The cut-off wavelength of the emitted X-
rays is (2007) 1. The shortest wavelength of X-rays emitted from an X-ray
2 tube depends on (1982 - 3 Marks)
2mcl 2h
(a) l0 = (b) l0 = (a) the current in the tube
h mc
(b) the voltage applied to the tube
2m 2 c 2 l 3 (c) the nature of the gas in tube
(c) l0 = (d) l 0 = l
h2 (d) the atomic number of the target material
55. Which one of the following statements is WRONG in the 2. The threshold wavelength for photoelectric emission from
context of X-rays generated from a X-ray tube? (2008) a material is 5200 Å. Photoelectrons will be emitted when
(a) Wavelength of characteristic X-rays decreases when this material is illuminated with monochromatic radiation
the atomic number of the target increases. from a (1982 - 3 Marks)
(b) Cut-off wavelength of the continuous X-rays depends (a) 50 watt infrared lamp (b) 1-watt infra-red lamp
on the atomic number of the target (c) 50 watt ultraviolet lamp (d) 1-watt ultraviolet lamp
(c) Intensity of the characteristic X-rays depends on the 3. From the following equations pick out the possible nuclear
electrical power given to the X-ray tube fusion reactions (1984- 2 Marks)
(d) Cut-off wavelength of the continuous X-rays depends 13 1 14
(a) 6 C + 1H ® 6 C + 4.3MeV
on the energy of the electrons in the X-ray tube
56. A radioactive sample S1 having an activity 5µCi has twice (b) 12
+ 1H1 ® 7 N13 + 2 MeV
6C
the number of nuclei as another sample S2 which has an
14
activity of 10 µCi. The half lives of S1 and S2 can be (c) 7N + 1H1 ® 8 O15 + 7.3MeV
(a) 20 years and 5 years, respectively (2008)
235
(b) 20 years and 10 years, respectively (d) 92 U + 0 n1 ® 54 Xe140 + 38 Sr 94 + 0 n1
(c) 10 years each
(d) 5 years each + 0 n1 + g + 200MeV
57. Photoelectric effect experiments are performed using three 4. In Bohr’s model of the hydrogen atom (1984- 2 Marks)
different metal plates p, q and r having work functions fp = (a) the radius of the nth orbit is proportional to n2
2.0 eV, fq = 2.5 eV and fr = 3.0 eV, respectively. A light beam (b) the total energy of the electron in nth orbit is inversely
containing wavelengths of 550 nm, 450 nm and 350 nm with proportional to n
equal intensities illuminates each of the plates. The correct
(c) the angular momentum of the electron in an orbit is an
I-V graph for the experiment is [Take hc = 1240 eV nm]
(2009) h
integral multiple of
I 2p
I (d) the magnitude of potential energy of the electron in
p any orbit is greater than its K.E.
q p 5. Select the correct statement from the following
(a) (b)
r q (1984- 2 Marks)
r (a) A diode can be used as a rectifier
V V (b) A triode cannot be used as a rectifier
(c) The current in a diode is always proportional to the
I applied voltage
I
r (d) The linear portion of the I–V characteristic of a triode
q
is used for amplification without distortion
(c) (d) 6. For a given plate voltage, the plate current in a triode valve
p
r q is maximum when the potential of (1985 - 2 Marks)
p
V (a) the grid is positive and plate is negative
V (b) the grid is zero and plate is positive
58. The wavelength of the first spectral line in the Balmer series (c) the grid is negative and plate is positive
of hydrogen atom is 6561 A°. The wavelength of the second (d) the grid is positive and plate is positive
spectral line in the Balmer series of singly-ionized helium 7. The X-ray beam coming from an X-ray tube will be
atom is (2011)
(1985 - 2 Marks)
(a) 1215 A° (b) 1640 A°
(c) 2430 A° (d) 4687 A° (a) monochromatic
59. A pulse of light of duration 100 ns is absorbed completely (b) having all wavelengths smaller than a certain maximum
by a small object initially at rest. Power of the pulse is 30 mW wavelength
and the speed of light is 3×108 ms–1. The final momentum of (c) having all wavelengths larger than a certain minimum
the object is (JEE Adv. 2013) wavelength
(a) 0.3 × 10–17 kg ms–1 (c) 3.0 × 10–17 kg ms–1 (d) having all wavelengths lying between a minimum and
(b) 1.0 × 10–17 kg ms–1 (d) 9.0 × 10–17 kg ms–1 a maximum wavelength
MODERN PHYSICS P-497
8. The mass number of a nucleus is (1986 - 2 Marks) 15. The impurity atoms with which pure silicon should be doped
(a) always less than its atomic number to make a p-type semiconductor are those of
(b) always more than its atomic number (1988 - 2 Marks)
(c) sometimes equal to its atomic number (a) phosphorus (b) boron
(d) sometimes more than and sometimes equal to its atomic (c) antimony (d) aluminium
number 16. Two identical p-n junctions may be connected in series with
9. Four physical quantities are listed in Column I. Their values a battery in three ways, fig. The potential drops across the
are listed in Column II in a random order: (1987 - 2 Marks) two p – n junctions are equal in (1989 - 2 Marks)
Column I Column II
+ – – + + – + –
(a) Thermal energy of air p n n p p n p n
molecules at room temp (e) 0.02 eV
(b) Binding energy of heavy
nuclei per nucleon (f) 2 eV
(c) X-ray photon energy (g) 1k eV Circuit 1 Circuit 2
(d) Photon energy of visible light (h) 7 MeV – + – +
The correct matching of Columns I and II is given by n p n p
(a) a – e, b – h, c – g, d – f
(b) a – e, b – g, c – f, d – h
(c) a – f, b –e, c – g, d – h
Circuit 3
(d) a – f, b – h, c – e, d – g.
10. Photoelectric effect supports quantum nature of light (a) circuit 1 and circuit 2 (b) circuit 2 and circuit 3
because (1987 - 2 Marks) (c) circuit 3 and circuit 1 (d) circuit 1 only
(a) there is a minimum frequency of light below which no 17. The decay constant of a radioactive sample is l . The half-
photoelectrons are emitted life and mean-life of the sample are respectively given by
(b) the maximum kinetic energy of photo electrons depends (1989 - 2 Marks)
only on the frequency of light and not on its intensity (a) 1/ l and (ln 2)/ l (b) (ln 2)/ l and 1/ l
(c) even when the metal surface is faintly illuminated, the
(c) l (ln 2) and 1/ l (d) l /(ln 2) and 1/ l
photoelectrons leave the surface immediately
18. When a monochromatic point source of light is at a distance
(d) electric charge of the photoelectrons is quantized
of 0.2 m from a photoelectric cell, the cut off voltage and the
11. During a negative beta decay (1987 - 2 Marks) saturation current are respectively 0.6 V and 18.0 mA. If the
(a) an atomic electron is ejected same source is placed 0.6 m away from the photoelectric
(b) an electron which is already present within the nucleus cell, then (1992 - 2 Marks)
is ejected (a) the stopping potential will be 0.2 volt
(c) a neutron in the nucleus decays emitting an electron (b) the stopping potential will be 0.6 volt
(d) a part of the binding energy of the nucleus is converted (c) the saturation current will be 6.0 mA
into an electron (d) the saturation current will be 2.0 mA
12. During a nuclear fusion reaction (1987 - 2 Marks) 19. In an n-p-n transistor circuit, the collector current is 10 mA.
(a) a heavy nucleus breaks into two fragments by itself If 90% of the electrons emitted reach the collector,
(b) a light nucleus bombarded by thermal neutrons breaks (1992 - 2 Marks)
up (a) the emitter current will be 9 mA
(c) a heavy nucleus bombarded by thermal neutrons (b) the base current will be 1 mA
breaks up (c) the emitter current will be 11 mA
(d) two light nuclei combine to give a heavier nucleus and (d) the base current will be – 1 mA
possibly other products 20. A star initially has 1040 deuterons. It produces energy via
13. The potential difference applied to an X-ray tube is 2 2 3
the processes 1H +1 H ®1 H + p, and
increased. As a result, in the emitted radiation
2 3 4
(a) the intensity increases (1988 - 2 Marks) 1H +1 H ® 2 He + n. If the average power radiated by
(b) the minimum wavelength increases the star is 1016 W, the deuteron supply of the star is exhausted
(c) the intensity remain unchanged in a time of the order of (1993-2 Marks)
(d) the minimum wavelength decreases (a) 106s. (b) 108s.
14. A freshly prepared radioactive source of half life 2 hr emits (c) 1012 s. (d) 1016 s.
radiation of intensity which is 64 times the permissibe safe
The masses of the nuclei are as follows :
level. The minimum time after which it would be possible to
work safely with this source is (1988 - 2 Marks) M (H 2 ) = 2.014 amu;
(a) 6 hr (b) 12 hr
M (p) = 1.007 amu; M(n) = 1.008 amu; M (He4 ) = 4.001amu.
(c) 24 hr (d) 128 hr
P-498 Topic-wise Solved Papers - PHYSICS
21. When photons of energy 4.25 eV strike the surface of metal 28. In a p-n junction diode not connected to any circuit,
A, the ejected photoelectrons have maximum kinetic energy, (1998S - 2 Marks)
TA eV and de Broglie wavelength lA. The maximum kinetic (a) the potential is the same everywhere
energy of photoelectrons liberated from another metal B by (b) the p-type side is at a higher potential than the n-type
photons of energy 4.70 eV is TB = (TA - 1.50) eV. If the de side
Broglie wavelength of these photoelectr ons is (c) there is an electric field at the junction directed from
l B = 2l A , then (1994 - 2 Marks) the n-type side to the p-type side
(a) The work function of A is 2.25 eV (d) there is an electric field at the junction directed from
(b) The work function of B is 4.20 eV the p-type side to the n-type side
(c) TA = 2.00 eV 29. X-rays are produced in an X-ray tube operating at a given
(d) TB = 2.75 eV accelerating voltage. The wavelength of the continuous
22. Which of the following statement(s) is (are) correct? X-rays has values from (1998S - 2 Marks)
(1994 - 2 Marks) (a) 0 to ¥
(a) The rest mass of a stable nucleus is less than the sum
of the rest masses of its separated nucleons (b) l min to ¥ where l min > 0
(b) The rest mass of a stable nucleus is greater than the
sum of the rest masses of its separated nucleons (c) 0 to l max where l max < ¥
(c) In nuclear fission, energy is released by fusing two (d) l min to l max where 0 < l min < l max < ¥
nuclei of medium mass (approximately 100 amu)
(d) In nuclear fission, energy is released by fragmentation 30. The work function of a substance is 4.0 eV. The longest
of a very heavy nucleus wavelength of light that can cause photoelectron emission
23. Holes are charge carriers in (1996 - 2 Marks) from this substance is approximately (1998S - 2 Marks)
(a) intrinsic semiconductors (a) 540 nm (b) 400 nm
(b) ionic solids (c) 310 nm (d) 220 nm
(c) p-type semiconductors 31. The half-life period of a radioactive element X is same as the
(d) metals mean-life time of another radioactive element Y. Initially both
24. A transistor is used in the common emitter mode as an of them have the same number of atoms. Then
amplifier. Then (1998S - 2 Marks)
(1999S - 3 Marks)
(a) the base-emitter junction is forward-biased
(b) the base-emitter junction is reverse-biased (a) X and Y have the same decay rate initially
(c) the input signal is connected in series with the voltage (b) X and Y decay at the same rate always
applied to bias the base-emitter junction (c) Y will decay at a faster rate than X
(d) the input signal is connected in series with the voltage (d) X will decay at a faster rate than Y
applied to bias the base-collector junction
25. Let mp be the mass of a proton, mn the mass of a neutron, æ1ö
32. The graph between the stopping potential (V0) and ç ÷ is
M1 the mass of a 20 nucleus and M2 the mass of a èlø
10 Ne
40 nucleus. Then (1998S - 2 Marks) shown in the figure. f1 , f 2 and f3 are work functions,
20 Ca
which of the following is/are correct (2006 - 5M, –1)
(a) M 2 = 2 M1 (b) M 2 > 2 M1
(c) M 2 < 2 M1 (d) M1 < 10 ( mn + m p ) V0
26. The electron in a hydrogen atom makes a transition n1 ® n2 Metal 1 Metal 2 Metal 3
where n1 and n2 are the principal quantum numbers of the
two states. Assume the Bohr model to be valid. The time f1 f2 f3
period of the electron in the initial state is eight times that in
the final state. The possible values of n1 and n2 are
1 (nm–1)
(1998S - 2 Marks)
0.001 0.002 0.004 l
(a) n1 = 4, n2 = 2 (b) n1 = 8, n2 = 2
(a) f1 : f2 : f3 = 1 : 2 : 4
(c) n1 = 8, n2 = 1 (d) n1 = 6, n2 = 3
27. The half-life of I is 8 days. Given a sample of 131I at time
131 (b) f1 : f2 : f3 = 4 : 2 : 1
t = 0, we can assert that (1998S - 2 Marks)
(a) no nucleus will decay before t = 4 days hc
(c) tan q µ
(b) no nucleus will decay before t = 8 days e
(c) all nuclei will decay before t = 16 days (d) ultravioletlight can be used to emit photoelectrons from
(d) a given nucleus may decay at any time after t = 0 metal 2 and metal 3 only
MODERN PHYSICS P-499
33. Assume that the nuclear binding energy per nucleon (B/A) 2. Hydrogen atom in its ground state is excited by means of
versus mass number (A) is as shown in the figure. Use this monochromatic radiation of wavelength 975Å. How many
plot to choose the correct choice(s) given below. (2008) different lines are possible in the resulting spectrum ?
Calculate the longest wavelength amongst them. You may
B/A assume the ionization energy for hydrogen atom as 13.6 eV.
(1982 - 5 Marks)
3. How many electron, protons and neutrons are there in a
8 nucleus of atomic number 11 and mass number 24 ?
6 (1982 - 2 Marks)
(i) number of electrons = (ii) number of protons =
4 (iii) number of neutrons =
2 4. A uranium nucleus (atomic number 92, mass number 238)
0 A emits an alpha particle and the resultant nucleus emits b -
100 200 particle. What are the atomic number and mass number of
the final nucleus. (1982 - 2 Marks)
(a) Fusion of two nuclei with mass numbers lying in the (i) Atomic number = (ii) Mass number =
range of 1 < A < 50 will release energy. 5. Ultraviolet light of wavelengths 800Å and 700Å when
(b) Fusion of two nuclei with mass numbers lying in the allowed to fall on hydrogen atoms in their ground state is
range of 51 < A < 100 will release energy found to liberate electrons with kinetic energy 1.8 eV and
(c) Fission of a nucleus lying in the mass range of 100 < A 4.0 eV respectively. Find the value of Planck’s constant.
< 200 will release energy when broken into two equal (1983 - 4 Marks)
fragments 6. The ionization energy of a hydrogen like Bohr atom is 4
(d) Fission of a nucleus lying in the mass range of 200 < A Rydbergs. (i) What is the wavelength of the radiation
< 260 will release energy when broken into two equal emitted when the electron jumps from the first excited state
to the ground state ? (ii) What is the radius of the first orbit
fragments
for this atom? (1984- 4 Marks)
34. The radius of the orbit of an electron in a Hydrogen-like
7. A double ionised Lithium atom is hydrogen-like with atomic
atom is 4.5 a0, where a0 is the Bohr radius. Its orbital angular number 3. (1985 - 6 Marks)
3h (i) Find the wavelength of the radiation required to excite
momentum is . It is given that h is Planck constant and R the electron in Li++ from the first to the third Bohr
2p
orbit. (Ionisation energy of the hydrogen atom equals
is Rydberg constant. The possible wavelength(s), when the
13.6 eV.)
atom de-excites, is (are) (JEE Adv. 2013) (ii) How many spectral lines are observed in the emission
9 9 spectrum of the above excited system?
(a) (b) 8. There is a stream of neutrons with a kinetic energy of 0.0327
32R 16R
eV. If the half life of neutrons is 700 seconds, what fraction
9 4 of neutrons will decay before they travel a distance of
(c) (d) 10 m? (1986 - 6 Marks)
5R 3R
9. A triode has plate characteristics in the form of parallel
SUBJECTIVE PROBLEMS : lines in the region of our interest. At a grid voltage of – 1
volt the anode current I (in milli amperes) is given in terms
1. A single electron orbits around a stationary nucleus of of plate voltage V (in volts) by the algebraic relation :
charge + Ze. Where Z is a constant and e is the magnitude I = 0.125V – 7.5
of the electronic charge. It requires 47.2 eV to excite the For grid voltage of – 3 volts, the current at anode voltage
electron from the second Bohr orbit to the third Bohr orbit. of 300 volts is 5 milliampere. Determine the plate resistance
Find (1981- 10 Marks) (rp), transconductance (gm) and the amplification factor (µ)
(i) The value of Z. for the triode. (1987 - 7 Marks)
(ii) The energy required to excite the electron from the 10. A particle of charge equal to that of an electron, – e, and
third to the fourth Bohr orbit. mass 208 times the mass of the electron (called a mu-meson)
(iii) The wavelength of the electromagnetic radiation moves in a circular orbit around a nucleus of charge + 3e.
required to remove the electron from the first Bohr (Take the mass of the nucleus to be infinite). Assuming
orbit to infinity. that the Bohr model of the atom is applicable to this system.
(iv) The kinetic energy, potential energy, potential energy (1988 - 6 Marks)
and the angular momentum of the electron in the first (i) Derive an expression for the radius of the nth Bohr
orbit.
Bohr orbit.
(ii) Find the value of n for which the radius of the orbit is
(v) The radius of the first Bohr orbit.
approximately the same as that of the first Bohr orbit
(The ionization energy of hydrogen atom = 13.6 eV, Bohr for the hydrogen atom.
radius = 5.3 × 10–11 metre, velocity of light = 3 × 108 m/sec. (iii) Find the wavelength of the radiation emitted when the
Planck’s constant = 6.6 × 10–34 joules - sec). mu-meson jumps from the third orbit of the first orbit.
P-500 Topic-wise Solved Papers - PHYSICS
11. A beam of light has three wavelengths 4144 Å, 4972 Å and (a) Calculate the photon flux at the centre of the screen
6216 Å with a total intensity of 3.6 × 10 - 3 W m -2 equally and the photocurrent in the detector.
(b) If the concave lens L of focal length 0.6 m is inserted in
distributed amongst the three wavelengths. The beam falls
the aperture as shown, find the new values of photon
normally on an area 1.0 cm2 of a clean metallic surface of
flux and photocurrent. Assume a uniform average
work function 2.3 eV. Assume that there is no loss of light
transmission of 80% from the lens.
by reflection and that each energetically capable photon (c) If the work function of the photoemissive surface is
ejects one electron. Calculate the number of photo electrons
1eV, calculate the values of the stopping potential in
liberated in two seconds. (1989 - 8 Marks)
the two cases (without and with the lens in the
12. A gas of identical hydrogen-like atoms has some atoms in
aperture).
the lowest (ground) energy level A and some atoms in a
16. A nucleus X, initially at rest, undergoes alpha decay
particular upper (excited) energy level B and there are no
according to the equation,
atoms in any other energy level. The atoms of the gas make
A
transition to a higher energy level by absorbing 92 X ® 228
Z Y +a (1991 - 4 + 4 Marks)
monochromatic light of photon energy 2.7 eV. Subsequently,
(a) Find the values of A and Z in the above process.
the atoms emit radiation of only six different photon
(b) The alpha particle produced in the above process is
energies. Some of the emitted photons have energy 2.7 eV,
found to move in a circular track of radius 0.11 m in a
some have energy more and some have less than 2.7 eV.
uniform magnetic field of 3 Tesla. Find the energy (In
(1989 - 8 Marks)
MeV) released during the process and the binding
(i) Find the principal quantum number of the initially
energy of the parent nucleus X.
excited level B.
(iii) Find the ionization energy for the gas atoms.
(iii) Find the maximum and the minimum energies of the
( ) 1
Given that : m(Y) = 228.03 u; m 0 n = 1.009 u.

emitted photons. m ( He ) = 4.003 u; m ( H ) = 1.008 u


4
2
1
1
13. Electrons in hydrogen like atom (Z = 3) make transitions
from the fifth to the fourth orbit and from the fourth to the 17. Light from a discharge tube containing hydrogen atoms falls
third orbit. The resulting radiations are incident normally on the surface of a piece of sodium. The kinetic energy of the
fastest photoelectrons emitted from sodium is 0.73 eV. The
on a metal plate and eject photoelectrons. The stopping
work function for sodium is 1.82 eV. Find (1992 - 10 Marks)
potential for the photoelectrons ejected by the shorter
(a) the energy of the photons causing the photoelectric
wavelength is 3.95 volts. Calculate the work function of the
emission,
metal and the stopping potential for the photoelectrons
ejected by the longer wavelength. (1990 - 7 Marks) (b) the quantum numbers of the two levels involved in
the emission of these photons,
(Rydberg constant = 1.094 × 10 7 m -1 ) (c) the change in the angular momentum of the electron in
14. It is proposed to use the nuclear fusion reaction the hydrogen atom in the above transition, and
2 2 (d) the recoil speed of the emitting atom assuming it to be
1 H +1 H ® 42 He (1990 - 8 Marks)
at rest before the transition.
in a nuclear reactor of 200 MW rating. If the energy from (Ionization potential of hydrogen is 13.6 eV)
the above reaction is used with a 25 per cent efficiency in 18. A neutron of kinetic energy 65eV collides inelastically with
the reactor, how many grams of deuterium fuel will be needed a singly ionized helium atom at rest. It is scattered at an
per day. (The masses of 12 H and 4
He are 2.0141 atomic angle of 90° with respect of its original direction.
2
(1993 - 9 +1 Marks)
mass units and 4.0026 atomic mass units respectively) (i) Find the allowed values of the energy of the neutron
15. A monochromatic point source radiating wavelength 6000 and that of the atom after the collision.
Å, with power 2 watt, an aperture A of diameter 0.1 m and a (ii) If the atom get de-excited subsequently by emitting
large screen SC are placed as shown in fig. A photoemissive radiation, find the frequencies of the emitted radiation.
detector D of surface area 0.5 cm2 is placed at the centre of [Given : Mass of He atom = 4 × (mass of neutron), Ionization
the screen. The efficiency of the detector for the energy of H atom = 13.6eV]
photoelectron generation per incident photon is 0.9. 19. A small quantity of solution containing Na24 radio nuclide
(1991 - 2 + 4 + 2 Marks) (half life = 15 hour) of activity 1.0 microcurie is injected into
SC
A the blood of a person. A sample of the blood of volume 1
cm 3 taken after 5 hour show an activity of 296
S disintegrations per minute. Determine the total volume of
D the blood in the body of the person. Assume that
L
0.6 m radioactive solution mixes uniformly in the blood of the
person. (1 curie = 3.7 ×1010 disintegrations per second)
6m (1994 - 6 Marks)
MODERN PHYSICS P-501
20. A hydrogen like atom (atomic number Z) is in a higher excited 28. A hydrogen-like atom of atomic number Z is in an excited
state of quantum number n. The excited atom can make a state of quantum number 2n. It can emit a maximum energy
transition to the first excited state by successively emitting photon of 204 eV. If it makes a transition to quantum state
two photons of energy 10.2 and 17.0 eV respectively. n, a photon of energy 40.8 eV is emitted. Find n, Z and the
Alternately, the atom from the same excited state can make ground state energy (in eV) for this atom. Also calculate
a transition to the second excited state by successively the minimum energy (in eV) that can be emitted by this
emitting two photons of energies 4.25 eV and 5.95 eV atom during de-excitation. Ground state energy of hydrogen
respectively. (1994 - 6 Marks) atom is –13.6 eV. (2000 - 6 Marks)
Determine the values of n and Z. (Ionization energy of H- 29. When a beam of 10.6 eV photons of intensity 2.0 W/m 2 falls
atom = 13.6 eV) on a platinum surface of area 1.0 × 10-4 m2 and work function
21. An electron, in a hydrogen-like atom, is in an excited state. 5.6 eV, 0.53% of the incident photons eject photoelectrons.
It has a total energy of –3.4 eV. Calculate (i) the kinetic Find the number of photoelectrons emitted per second and
energy and (ii) the de Broglie wavelength of the electron. their minimum and maximum energies (in eV). Take 1eV =
(1996 - 3 Marks) 1.6 × 10-19 J. (2000 - 4 Marks)
22. At a given instant there are 25% undecayed radio-active 30. In a nuclear reaction 235U undergoes fission liberating 200
nuclei in a sample. After 10 seconds the number of MeV of energy. The reactor has a 10% efficiency and
undecayed nuclei reduces to 12.5%. Calculate (i) mean-life produces 1000 MW power. If the reactor is to function for
of the nuclei, and (ii) the time in which the number of 10 years, find the total mass of uranium required.
undecayed nuclei will further reduce to 6.25% of the
(2001 - 5 Marks)
reduced number. (1996 - 3 Marks)
31. A nucleus at rest undergoes a decay emitting an a particle
23. In an ore containing Uranium, the ratio of U238 to Pb206
of de-Broglie wavelength l = 5.76 ´ 10-15 m. If the mass of
nuclei is 3. Calculate the age of the ore, assuming that all
the lead present in the ore is the final stable product of the daughter nucleus is 223.610 amu and that of the a
U238. Take the half-life of U238 to be 4.5 × 109 years. particles is 4.002 amu, determine the total kinetic energy in
(1997C - 5 Marks) the final state. Hence, obtain the mass of the parent nucleus
24. Assume that the de Broglie wave associated with an electron in amu. (1 amu = 931.470 MeV/c2) (2001-5 Marks)
can form a standing wave between the atoms arranged in a 32. A radioactive nucleus X decays to a nucleus Y with a decay
one dimensional array with nodes at each of the atomic constant lx = 0.1 s-1. Y further decays to a stable nucleus Z
sites. It is found that one such standing wave is formed if with a decay constant lY = 1/30 s-1. Initially, there are only
the distance d between the atoms of the array is 2Å. A X nuclei and their number is No = 1020. Set up the rate
similar standing wave is again formed if d is increased to equations for the populations of X, Y and Z. The population
2.5 Å but not for any intermediate value of d. Find the of Y nucleus as a function of time is given by NY(t) = (NolX/
energy of the electrons in electron volts and the least value (lX - lY)){exp(-lYt)-exp(-lxt)}. Find the time at which NY is
of d for which the standing wave of the type described maximum and determine the populations X and Z at that
above can form. (1997 - 5 Marks) instant. (2001-5 Marks)
33. A hydrogen-like atom (described by the Bohr model) is
25. The element Curium 248 13
96 Cm has a mean life of 10 seconds. observed to emit six wavelengths, originating from all
Its pirmary decay modes are spontaneous fission and a- possible transitions between a group of levels. These levels
decay, the former with a probability of 8% and the latter have energies between -0.85 eV and -0.544 eV (including
with a probability of 92%. Each fission releases 200 MeV of both these values). (2002 - 5 Marks)
energy. The masses involved in a-decay are as follows: (a) Find the atomic number of the atom.
248 244 (b) Calculate the smallest wavelength emitted in these
96 Cm = 248.072220u, 94 Pu = 244.064100u and
4
transitions.
He=4.002603 u. Calculate the power output from a sample
2 (Take hc = 1240 eV-nm, ground state energy of hydrogen
of 1020 Cm atoms. (1 u = 931 MeV/c2.) (1997 - 5 Marks) atom = –13.6 eV)
26. Nuclei of a radioactive element A are being produced at a 34. Two metallic plates A and B, each of area 5 × 10–4 m2, are
constant rate a. The element has a decay constant l. At placed parallel to each other at a separation of 1 cm. Plate B
time t = 0, there are N0 nuclei of the element. carries a positive charge of 33.7 × 10–12 C. A monochromatic
(1998 - 8 Marks) beam of light, with photons of energy 5 eV each, starts
(a) Calculate the number N of nuclei of A at time t. falling on plate A at t = 0 so that 1016 photons fall on it per
(b) If a = 2N0l, calculate the number of nuclei of A after square meter per second. Assume that one photoelectron
one half-life of A, and also the limiting value of N as is emitted for every 106 incident photons. Also assume that
t ®¥. all the emitted photoelectrons are collected by plate B and
27. Photoelectrons are emitted when 40 nm radiation is incident the work function of plate A remains constant at the value
on a surface of work function 1.9 eV. These photoelectrons 2eV. Determine (2002 - 5 Marks)
pass through a region containing a-particles. A maximum (a) the number of photoelectrons emitted up to t = 10 s,
energy electron combines with an a–particle to form a He+ (b) the magnitude of the electric field between the plates
ion, emitting a single photon in this process. He+ ions thus A and B at t = 10 s, and
formed are in their fourth excited state. Find the energies in (c) the kinetic energy of the most energetic photoelectron
eV of the photons, lying in the 2 to 4 eV range, that are emitted at t = 10 s when it reaches plate B.
likely to be emitted during and after the combination. Neglect the time taken by the photoelectron to reach plate
[Take h = 4.14×10–15 eV.s.] (1999 - 5 Marks) B. Take e0 = 8.85 × 10–12 C2/N-m2
P-502 Topic-wise Solved Papers - PHYSICS
35. Frequency of a photon emitted due to transition of electron 39. The photons from the Balmer series in Hydrogen spectrum
of a certain element from L to K shell is found to be having wavelength between 450 nm to 700 nm are incident
4.2 × 1018 Hz. Using Moseley’s law, find the atomic number on a metal surface of work function 2 eV. Find the maximum
kinetic energy of ejected electron. (Given hc = 1242 eV nm)
of the element, given that the Rydberg’s constant R = 1.1 ×
(2004 - 4 Marks)
107 m–1. (2003 - 2 Marks) 40. The potential energy of a particle of mass m is given by
36. A radioactive sample emits n b-particles in 2 sec. In next 2
sec it emits 0.75 n b-particle, what is the mean life of the ì E ; 0 £ x £ 1ü
V ( x) = í 0 ý
sample? (2003 - 2 Marks) î 0; x >1 þ
37. In a photoelectric experiment set up, photons of energy 5 l1 and l2 are the de-Broglie wavelengths of the particle,
eV falls on the cathode having work function 3 eV. (a) If the when 0 £ x £ 1 and x > 1 respectively. If the total energy of
saturation current is iA = 4mA for intensity 10–5 W/m2, then particle is 2E0, find l1 / l2. (2005 - 2 Marks)
41. Highly energetic electrons are bombarded on a target of an
plot a graph between anode potential and current. (b) Also element containing 30 neutrons. The ratio of radii of nucleus
draw a graph for in tensity of incident r adiation to that of Helium nucleus is (14)1/3. Find (a) atomic number
2 × 10–5 W/m2. (2003 - 2 Marks) of the nucleus. (b) the frequency of Ka line of the X-ray
38. 238
A radioactive sample of U decays to Pb through a process produced. (R = 1.1 × 107 m–1 and c = 3 × 108 m/s)
for which the half-life is 4.5×109 years. Find the ratio of (2005 - 4 Marks)
number of nuclei of Pb to 238U after a time of 1.5×109 years. 42. In hydrogen-like atom (z = 11), nth line of Lyman series has
wavelength l. The de-Broglie's wavelength of electron in
Given (2)1/3 = 1.26. (2004 - 2 Marks)
the level from which it originated is also l. Find the value of
n? (2006 - 6M)
MATCH THE FOLLOWING :
MUTLIPLE CHOICE QUESTIONS WITH ONE CORRECT
Direction (Q.No. 1 to Q. No. 4) : Each question contains statements given in two columns, which have to be matched. The
statements in Column-I are labelled A, B, C and D, while the statements in Column-II are labelled p, q, r, s and t. Any given
statement in Column-I can have correct matching with ONE OR MORE statement(s) in Column-II. The appropriate bubbles
corresponding to the answers to these questions have to be darkened as illustrated in the following example :
If the correct matches are A-p, s and t; B-q and r; C-p and q; and D-s then the correct darkening of bubbles will look like the given.
p q r s t
A p q r s t
B p q r s t
C p q r s t
D p q r s t

1. In the following, column I lists some physical quantities and the column II gives approximate energy values associated with
some of them. Choose the appropriate value of energy from column II for each of the physical quantities in column I and write
the corresponding letter p, q, r, etc. against the number (A), (B), (C), (D) etc. of the physical quantity in the answer book. In your
answer, the sequence of column I should be maintained. (1997 - 4 Marks)
Column I Column II
(A) Energy of thermal neutrons (p) 0.025 eV
(B) Energy of X-rays (q) 0.5 eV
(C) Binding energy per nucleon (r) 3 eV
(D) Photoelectric threshold of a metal (s) 20 eV
(t) 10 k eV
(u) 8 M eV
2. Given below are certain matching type questions, where two columns (each having 4 items) are given. Immediately after the
columns the matching grid is given, where each item of Column I has to be matched with the items of Column II, by encircling
the correct match(es). Note that an item of column I can match with more than one item of column II. All the items of column II
must be matched. Match the following : (2006 - 6M)
Column I Column II
(A) Nuclear fusion (p) Converts some matter into energy
(B) Nuclear fission (q) Generally possible for nuclei with low atomic
number
(C) b-decay (r) Generally possible for nuclei with higher atomic
number
(D) Exothermic nuclear reaction (s) Essentially proceeds by weak nuclear forces
MODERN PHYSICS P-503
3. Some laws / processes are given in Column I. Match these with the physical phenomena given in Column II and indicate your
answer by darkening appropriate bubbles in the 4 × 4 matrix given in the ORS. (2007)
Column I Column II
(A) Transition between two atomic energy levels (p) Characteristic X-rays
(B) Electron emission from a material (q) Photoelectric effect
(C) Mosley's law (r) Hydrogen spectrum
(D) Change of photon energy into kinetic energy of electrons (s) b-decay
4. Column-II gives certain systems undergoing a process. Column-I suggests changes in some of the parameters related to the
system. Match the statements in Column-I to the approapriate process(es) from Column-II. (2009)
Column-I Column-II
(A) The energy of the system is increased (p) System : A capacitor, initially uncharged
Process : It is connected to a battery
(B) Mechanical energy is provided to the system, which is (q) System : A gas in an adiabatic container fitted with
converted into energy of random motion of its parts an adiabatic piston
Process: The gas is compressed by pushing the
piston
(C) Internal energy of the system is converted into its (r) System: A gas in a rigid container
mechanical energy Process: The gas gets cooled due to colder
atmosphere surrounding it
(D) Mass of the system is decreased (s) System: A heavy nucleus, initially at rest
Process: The nucleus fissions into two fragments
of nearly equal masses and some neutrons are
emitted
(t) System: A resistive wire loop
Process: The loop is placed in a time varying
magnetic field perpendicular to its plane

Direction (Q.No. 5) : Following question has matching lists. The codes for the lists have choices (a), (b), (c) and (d) out of which
ONLY ONE is correct.
5. Match List I of the nuclear processes with List II containing parent nucleus and one of the end products of each process and then
select the correct answer using the codes given below the lists: (JEE Adv. 2013-II)
List I List II
15 15
P. Alpha decay 1. 8 O ® 7 O + ...

b+ decay 138 234


Q. 2. 92 U ® 90 Th + ...
185 184
R. Fission 3. 83 Bi ® 82 Pb + ...
239
S. Proton emission 4. 94 Pu ® 140
57 La + ...
Codes:
P Q R S
(a) 4 2 1 3
(b) 1 3 2 4
(c) 2 1 4 3
(d) 4 3 2 1

COMPREHENSION BASED Q UESTIONS : 2. The wavelength of light emitted in the visible region by He+
ions after collisions with H atoms is –
PASSAGE-1
In a mixture of H-He+ gas (He+ is singly ionized He atom), H (a) 6.5 × 10–7 m
atoms and He+ ions are excited to their respective first excited (b) 5.6 × 10–7 m
states. Subsequently, H atoms transfer their total excitation energy (c) 4.8 × 10–7 m
to He+ ions (by collisions). Assume that the Bohr model of atom (d) 4.0 × 10–7 m
is exactly valid. (2008) 3. The ratio of the kinetic energy of the n = 2 electron for the H
1. The quantum number n of the state finally populated in He+ atom to that of He+ ion is –
ions is – (a) 1/4 (b) 1/2
(a) 2 (b) 3
(c) 1 (d) 2
(c) 4 (d) 5
P-504 Topic-wise Solved Papers - PHYSICS
PASSAGE-2
Scientists are working hard to develop nuclear fusion reactor. p2
E= . Thus, the energy of the particle can be denoted by a
2m
Nuclei of heavy hydrogen, 12 H, known as deuteron and denoted
quantum number ‘n’ taking values1, 2, 3, ... (n = 1, called the ground
by D, can be thought of as a candidate for fusion reactor. The state) corresponding to the number of loops in the standing wave.
2 2 3 Use the model described above to answer the following three
D-D reaction is 1 H + 1 H ®2
He + n + energy. In the core of
questions for a particle moving in the line x = 0 to x = a. Take h =
fusion reactor, a gas of heavy hydrogen is fully ionized into
6.6 × 10–34 J s and e = 1.6 × 10–19 C.
deuteron nuclei and electrons. This collection of 12 H nuclei and 7. The allowed energy for the particle for a particular value of
electrons is known as plasma. The nuclei move randomly in the n is proportional to (2009)
reactor core and occasionally come close enough for nuclear (a) a–2 (b) a–3/2
fusion to take place. Usually, the temperatures in the reactor core (c) a–1 (d) a2
are too high and no material wall can be used to confine the plasma. 8. If the mass of the particle is m = 1.0 × 10–30 kg and a = 6.6
Special techniques are used which confine the plasma for a time t0 nm, the energy of the particle in its ground state is closest
before the particles fly away form the core. If n is the density to (2009)
(number/volume) of deuterons, the product nt0 is called Lawson (a) 0.8 meV (b) 8 meV
number. In one of the criteria, a reactor is termed successful if (c) 80 meV (d) 800 meV
Lawson number is greater than 5 × 1014 s/cm3. 9. The speed of the particle, that can take discrete values, is
It may be helpful to use the following: Boltzmann constant proportional to (2009)
e2 (a) n–3/2 (b) n–1
k = 8.6 × 10 – 5 eV/K; = 1.44 × 10–9 eVm (2009) (c) n1/2 (d) n
4pe 0
PASSAGE-4
4. In the core of nuclear fusion reactor, the gas becomes plasma
The key feature of Bohr’s theory of spectrum of hydrogen atom is
because of
the quantization of angular momentum when an
(a) strong nuclear force acting between the deuterons
(b) coulomb force acting between the deuterons electron is revolving around a proton. We will extend this to a
(c) coulomb force acting between deuteron-electron pairs general rotational motion to find quantized rotational
(d) the high temperature maintained inside the reactor core energy of a diatomic molecule assuming it to be rigid. The rule to
5. Assume that two deuteron nuclei in the core of fusion reactor be applied is Bohr’s quantization condition. (2010)
at temperature T are moving towards each other, each with 10. A diatomic molecule has moment of inertia I. By Bohr’s
kinetic energy 1.5 kT, when the separation between them is quantization condition its rotational energy in the nth level
large enough to neglect coulomb potential energy. Also (n = 0 is not allowed) is
neglect any interaction from other particles in the core. The
minimum temperature T required for them to reach a 1 æ h2 ö 1 æ h2 ö
(a) ç ÷ (b) n çè 8p 2 I ÷ø
separation of 4 × 10–15 m is in the range n2 è 8p 2 I ø
(a) 1.0 × 109 K < T < 2.0 × 109 K
(b) 2.0 × 109 K < T < 3.0 × 109 K æ h2 ö æ h2 ö
(c) 3.0 × 109 K < T < 4.0 × 109 K (c) nç 2 ÷ (d) n2 ç 2 ÷
(d) 4.0 × 109 K < T < 5.0 × 109 K è 8p I ø è 8p I ø
6. Results of calculations for four different designs of a fusion 11. It is found that the excitation frequency from ground to the
reactor using D-D reaction are given below. Which of these first excited state of rotation for the CO molecule is close
is most promising based on Lawson criterion?
(a) deuteron density = 2.0 × 1012 cm–3, 4
to ´ 1011 Hz. Then the moment of inertia of CO molecule
confinement time = 5.0 × 10–3 s p
(b) deuteron density = 8.0 × 1014 cm–3, about its center of mass is close to
confinement time = 9.0 × 10–1 s
(c) deuteron density = 4.0 × 1023 cm–3, (Take h = 2p ´ 10 –34 J s)
confinement time = 1.0 × 10–11 s
(a) 2.76 ´ 10 –46 kg m2 (b) 1.87 ´ 10 –46 kg m2
(d) deuteron density = 1.0 × 1024 cm–3,
confinement time = 4.0 × 10–12 s (c) 4.67 ´ 10–47 kg m2 (d) 1.17 ´ 10 –47 kg m2
PASSAGE-3
12. In a CO molecule, the distance between C (mass = 12 a.m.u.)
When a particle is restricted to move along x - axis between x = 0
and x = a, where a is of nanometer dimension, its energy can take 5
and O (mass = 16 a.m.u.), where 1 a.m.u. = ´ 10 –27 kg , is
only certain specific values. The allowed energies of the particle 3
moving in such a restricted region, correspond to the formation of close to
standing waves with nodes at its ends x = 0 and x = a . The
wavelength of this standing wave is related to the linear momentum (a) 2.4 ´ 10 –10 m (b) 1.9 ´ 10 –10 m
p of the particle according to the de Broglie relation. The energy
of the particle of mass m is related to its linear momentum as (c) 1.3 ´ 10 –10 m (d) 4.4 ´ 10 –11 m
MODERN PHYSICS P-505
PASSAGE-5 (a) 0 £ K £ 0.8 ´ 106 eV (2012)
The b -decay process, discovered around 1900, is basically the 3.0 eV £ K £ 0.8 ´ 106 eV
(b)
decay of a neutron (n). In the laboratory, a proton (p) and an
(c) 3.0 eV £ K < 0.8 ´ 106 eV
electron (e–) are observed as the decay products of the neutron.
Therefore, considering the decay of a neutron as a two-body (d) 0 £ K < 0.8 ´ 106 eV
decay process, it was predicted theoretically that the kinetic energy 14. What is the maximum energy of the anti-neutrino? (2012)
of the electron should be a constant. But experimentally, it was (a) Zero
observed that the electron kinetic energy has continuous (b) Much less than 0.8 ´ 106 eV .
spectrum. Considering a three-body decay process, i.e. (c) Nearly 0.8 ´ 106 eV
(d) Much larger than 0.8 ´ 106 eV
n ® p + e - + n e , around 1930, Pauli explained the observed
PASSAGE-6
electron energy spectrum. Assuming the anti-neutrino (ne ) to be A
The mass of a nucleus ZX is less than the sum of the masses of
massless and possessing negligible energy, and the neutron to
be at rest, momentum and energy conservation principles are (A-Z) number of neutrons and Z number of protons in the nucleus.
applied. From this calculation, the maximum kinetic energy of the The energy equivalent to the corresponding mass difference is
electron is 0.8 × 106 eV. The kinetic energy carried by the proton is known as the binding energy of the nucleus. A heavy nucleus of
only the recoil energy. mass M can break into two light nuclei of masses m 1 and m2 only
13. If the anti-neutrino had a mass of 3 eV/c2 (where c is the if (m1 + m2) < M. Also two light nuclei of masses m3 and m4 can
speed of light) instead of zero mass, what should be the undergo complete fusion and form a heavy nucleus of mass M'
range of the kinetic energy, K, of the electron? (2012) only if (m3 + m4) > M'. The masses of some neutral atoms are
given in the table below:
1 1.007825 u 2 2.014102 u 3 3.016050 u 4 4.002603 u
1H 1H 1H 2 He
6 6.015123 u 7 7.016004 u 70 69.925325 u 82 81.916709 u
3 Li 3 Li 30 Zn 34 Se
152 151.919803 u 206 205.974455 u 209 208.980388 u 210 209.982876 u
64 Gd 82 Pb 83 Bi 84 Po
(1u = 932 MeV/c2)
(JEE Adv. 2013)
15. The kinetic energy (in keV) of the alpha particle, when the INTEGER VALUE CORRECT TYPE :
210
nucleus 84 Po at rest undergoes alpha decay, is 1. An a-particle and a proton are accelerated from rest by a
(a) 5319 (b) 5422 potential difference of 100 V. After this, their de Broglie
(c) 5707 (d) 5818
16. The correct statement is lp
wavelengths are la and lp respectively. The ratio , to
la
(a) The nucleus 63 Li can emit an alpha particle
the nearest integer, is (2010)
(b) The nucleus 210 can emit a proton
84 Po 2. To determine the half life of a radioactive element, a student
(c) Deuteron and alpha particle can undergo complete
dN (t ) dN (t )
fusion plots a graph of ln versus t. Here is the rate
dt dt
(d) The nuclei 70 and 82 can undergo complete
30 Zn 34 Se of radioactive decay at time t. If the number of radioactive
fusion nuclei of this element decreases by a factor of p after 4.16
ASSERTION & REASON TYPE QUESTIONS : years, the value of p is (2010)

1. STATEMENT-1 (2007)
If the accelerating potential in an X-ray tube is increased,
the wavelengths of the characteristic X-rays do not change.
because
STATEMENT-2
ln|dN(t)/dt|

When an electron beam strikes the target in an X-ray tube,


part of the kinetic energy is converted into X-ray energy.
(a) Statement-1 is True, Statement-2 is True; Statement-2 is
a correct explanation for Statement-1
(b) Statement-1 is True, Statement-2 is True; Statement-2
is NOT a correct explanation for Statement-1
(c) Statement-1 is True, Statement-2 is False
(d) Statement-1 is False, Statement-2 is True
P-506 Topic-wise Solved Papers - PHYSICS
10
3. The activity of a freshly prepared radioactive sample is 10 Broglie wavelength (in units of fm) of the proton at its start is:
disintegrations per second, whose mean life is 109 s. The (take the proton mass, mp= (5/3) ´ 10 – 27 kg; h/e = 4.2 ´ 10–
mass of an atom of this radioisotope is 10–25 kg. The mass 15 J.s / C;
(in mg) of the radioactive sample is (2011) 1
4. A silver sphere of radius 1 cm and work function 4.7 eV is = 9 × 109 m/F; 1 fm = 10–15 m (2012- I)
suspended from an insulating thread in freespace. It is under 4pe 0
continuous illumination of 200 nm wavelength light. As 6. The work functions of Silver and Sodium are 4.6 and 2.3 eV,
photoelectrons are emitted, the sphere gets charged and acquires respectively. The ratio of the slope of the stopping potential
a potential. The maximum number of photoelectrons emitted from versus frequency plot for Silver to that of Sodium is
the sphere is A ´ 10z (where 1 < A < 10). The value of ‘z’ is (JEE Adv. 2013-I)
(2011) 7. A freshly prepared sample of a radioisotope of half-life 1386
5. A proton is fired from very far away towards a nucleus with s has activity 103 disintegrations per second. Given that ln2
charge Q = 120 e, where e is the electronic charge. = 0.693, the fraction of the initial number of nuclei (expressed
It makes a closest approach of 10 fm to the nucleus. The de in nearest integer percentage) that will decay in the first 80 s
after preparation of the sample is (JEE Adv. 2013-I)

1. If 13.6 eV energy is required to ionize the hydrogen atom, 10. Which of the following are not electromagnetic waves?
then the energy required to remove an electron from n = 2 is (a) cosmic rays (b) gamma rays [2002]
(a) 10.2 eV (b) 0 eV [2002]
(c) b -rays (d) X-rays.
(c) 3.4 eV (d) 6.8 eV.
2. At absolute zero, Si acts as [2002] 11. A strip of copper and another of germanium are cooled from
(a) non-metal (b) metal room temperature to 80K. The resistance of [2003]
(c) insulator (d) none of these (a) each of these decreases
3. At a specific instant emission of radioactive compound is (b) copper strip increases and that of germanium decreases
deflected in a magnetic field. The compound can emit (c) copper strip decreases and that of germanium increases
(i) electrons (ii) protons (d) each of these increases
(iii) He2+ (iv) neutrons 12. Which of the following radiations has the least wavelength ?
The emission at instant can be [2002] (a) g -rays (b) b -rays [2003]
(a) i, ii, iii (b) i, ii, iii, iv
(c) iv (d) ii, iii (c) a -rays (d) X -rays
4. Sodium and copper have work functions 2.3 eV and 4.5 eV 13. When a U 238 nucleus originally at rest, decays by emitting
respectively. Then the ratio of the wavelengths is nearest to an alpha particle having a speed ‘u’, the recoil speed of the
(a) 1 : 2 (b) 4 : 1 [2002]
residual nucleus is [2003]
(c) 2 : 1 (d) 1 : 4.
5. Formation of covalent bonds in compounds exhibits 4u (b) 4u 4u
(a) - (c) (d) - 4u
(a) wave nature of electron [2002] 238 234 234 238
(b) particle nature of electron 14. The difference in th e variation of resistance with
(c) both wave and particle nature of electron temeperature in a metal and a semiconductor arises
(d) none of these
essentially due to the difference in the [2003]
6. If N0 is the original mass of the substance of half-life period
(a) crystal sturcture
t1/2 = 5 years, then the amount of substance left after 15
years is [2002] (b) variation of the number of charge carriers with
(a) N0/8 (b) N0/16 temperature
(c) N0/2 (d) N0/4 (c) type of bonding
7. By increasing the temperature, the specific resistance of a (d) variation of scattering mechanism with temperature
conductor and a semiconductor [2002] 15. A radioactive sample at any instant has its disintegration
(a) increases for both (b) decreases for both rate 5000 disintegrations per minute. After 5 minutes, the
(c) increases, decreases(d) decreases, increases rate is 1250 disintegrations per minute. Then, the decay
8. The energy band gap is maximum in [2002] constant (per minute) is [2003]
(a) metals (b) superconductors (a) 0.4 ln 2 (b) 0.2 ln 2
(c) insulators (d) semiconductors. (c) 0.1 ln 2 (d) 0.8 ln 2
9. The part of a transistor which is most heavily doped to 16. A nucleus with Z= 92 emits the following in a sequence:
produce large number of majority carriers is [2002]
a, b - , b - a, a, a, a, a, b - , b - , a, b + , b + , a
(a) emmiter
(b) base Then Z of the resulting nucleus is [2003]
(c) collector (a) 76 (b) 78
(d) can be any of the above three. (c) 82 (d) 74
MODERN PHYSICS P-507
17. Two identical photocathodes receive light of frequencies f1 25. According to Einstein’s photoelectric equation, the plot of
and f2. If the velocites of the photo electrons (of mass m ) the kinetic energy of the emitted photo electrons from a
coming out are respectively v1 and v2, then [2003] metal Vs the frequency, of the incident radiation gives as
2 2 2h straight the whose slope [2004]
(a) v1 - v2 = (f - f ) (a) depends both on the intensity of the radiation and the
m 1 2
metal used
1/ 2
é 2h ù (b) depends on the intensity of the radiation
(b) v1 + v2 = ê ( f1 + f 2 )ú
ëm û (c) depends on the nature of the metal used
(d) is the same for the all metals and independent of the
(c) 2 2 2h intensity of the radiation
v1 + v2 = (f + f )
m 1 2 26. The work function of a substance is 4.0 eV. The longest
1/ 2 wavelength of light that can cause photoelectron emission
é 2h ù from this substance is approximately. [2004]
(d) v1 - v2 = ê ( f1 - f 2 )ú
ëm û (a) 310 nm (b) 400 nm
18. Which of the following cannot be emitted by radioactive (c) 540 nm (d) 220 nm
substances during their decay ? [2003] 27. A nucleus disintegrated into two nuclear parts which have
(a) Protons (b) Neutrinoes their velocities in the ratio of 2 : 1. The ratio of their nuclear
(c) Helium nuclei (d) Electrons sizes will be [2004]
19. In the nuclear fusion reaction (a) 3½ : 1 (b) 1:21/3
2 3 4 (c) 21/3:1 (d) 1:3½
1 H + 1 H ® 2 He + n
given that the repulsive potential energy between the two 28. The binding energy per nucleon of deuteron 1 H and ( )
2

nuclei is ~ 7.7 ´ 10 -14 J , the temperature at which the gases


must be heated to initiate the reaction is nearly ( )
4
helium nucleus 2 He is 1.1 MeV and 7 MeV respectively..
[Boltzmann’s Constant k = 1.38 ´ 10 -23 J/K ] [2003] If two deuteron nuclei react to form a single helium nucleus,
then the energy released is [2004]
(a) 10 7 K (b) 10 5 K
(a) 23.6 MeV (b) 26.9 MeV
(c) 10 3 K (d) 10 9 K (c) 13.9 MeV (d) 19.2 MeV
20. Which of the following atoms has the lowest ionization 29. An a-particle of energy 5 MeV is scattered through 180º by a
potential ? [2003] fixed uranium nucleus. The distance of closest approach is of
14 133 40 16 the order of [2004]
(a) N (b) Cs (c) Ar (d) O
10 -12 cm (b) 10 -10 cm
7 55 18 8
(a)
21. The wavelengths involved in the spectrum of deuterium

( D) are slightly different from that of hydrogen spectrum,


2
1
(c)1A (d) 10 -15 cm
30. When npn transistor is used as an amplifier [2004]
because [2003] (a) electrons move from collector to base
(a) the size of the two nuclei are different (b) holes move from emitter to base
(b) the nuclear forces are different in the two cases (c) electrons move from base to collector
(c) the masses of the two nuclei are different (d) holes move from base to emitter
(d) the atraction between the electron and the nucleus is
31. For a transistor amplifier in common emitter configuration
differernt in the two cases
22. In the middle of the depletion layer of a reverse- biased for load impedance of 1k W (h fe = 50 and hoe = 25) the
p-n junction, the [2003] current gain is [2004]
(a) electric field is zero (a) – 24.8 (b) – 15.7
(b) potential is maximum
(c) – 5.2 (d) – 48.78
(c) electric field is maximum
32. A piece of copper and another of germanium are cooled
(d) potential is zero
23. If the binding energy of the electron in a hydrogen atom is from room temperature to 77K, the resistance of
13.6eV, the energy required to remove the electron from the (a) copper increases and germanium decreases
(b) each of them decreases [2004]
first excited state of Li + + is [2003] (c) each of them increases
(a) 30.6 eV (b) 13.6 eV (d) copper decreases and germanium increases
(c) 3.4 eV (d) 122.4 eV 33. The manifestation of band structure in solids is due to
24. A radiation of energy E falls normally on a perfectly reflecting (a) Bohr’s correspondence principle [2004]
surface. The momentum transferred to the surface is [004]
(b) Pauli’s exclusion principle
(a) Ec (b) 2 E / c (c) Heisenberg’s uncertainty principle
(c) E /c (d) E / c2 (d) Boltzmann’s law
P-508 Topic-wise Solved Papers - PHYSICS
34. When p-n junction diode is forward biased then [2004] 42. If the kinetic energy of a free electron doubles, it’s deBroglie
(a) both the depletion region and barrier height are reduced wavelength changes by the factor [2005]
(b) the depletion region is widened and barrier height is 1
reduced (a) 2 (b)
(c) the depletion region is reduced and barrier height is 2
increased 1
(d) Both the depletion region and barrier height are (c) 2 (d)
2
increased
35. If radius of the 27 nucleus is estimated to be 3.6 fermi 43. A nuclear transformation is denoted by X (n, a ) 73 Li . Which
13 Al
of the following is the nucleus of element X ?
then the radius of 125 nucleus be nearly [2005]
52 Te 10 12
(a) 5 Be (b) C6 [2005]
(a) 8 fermi (b) 6 fermi
(c) 5 fermi (d) 4 fermi (c) 11 (d) 95 B
4 Be
7
66 44. In a full wave rectifier circuit operating from 50 Hz mains
36. Starting with a sample of pure Cu ,
of it decays into
8 frequency, the fundamental frequency in the ripple would be
Zn in 15 minutes. The corresponding half life is [2005] (a) 25 Hz (b) 50 Hz [2005]
(a) 15 minutes (b) 10 minutes (c) 70.7 Hz (d) 100 Hz
45. In a common base mode of a transistor, the collector current
1 is 5.488 mA for an emitter current of 5.60 mA. The value of
(c) 7 minutes (d) 5 minutes
2 the base current amplification factor (b) will be [2006]
37. A photocell is illuminated by a small bright source placed 1 (a) 49 (b) 50
1 (c) 51 (d) 48
m away. When the same source of light is placed m away,, 46. The threshold frequency for a metallic surface corresponds
2 to an energy of 6.2 eV and the stopping potential for a
the number of electrons emitted by photocathode would radiation incident on this surface is 5 V. The incident radiation
(a) increase by a factor of 4 [2005] lies in [2006]
(b) decrease by a factor of 4 (a) ultra-violet region (b) infra-red region
(c) increase by a factor of 2 (c) visible region (d) X-ray region
(d) decrease by a factor of 2
38. The electrical conductivity of a semiconductor increases 1 2
47. An alpha nucleus of energy mv bombards a heavy
when electromagnetic radiation of wavelength shorter than 2
2480 nm is incident on it. The band gap in (eV) for the nuclear target of charge Ze. Then the distance of closest
semiconductor is [2005] approach for the alpha nucleus will be proportional to
(a) 2.5 eV (b) 1.1 eV 1
(c) 0.7 eV (d) 0.5 eV (a) v 2 (b) [2006]
39. The intensity of gamma radiation from a given source is I. m
I 1 1
On passing through 36 mm of lead, it is reduced to . The (c) (d)
8 v2 Ze
48. The time taken by a photoelectron to come out after the
I
thickness of lead which will reduce the intensity to will be photon strikes is approximately [2006]
2 (a) 10–4 s (b) 10–10 s
(a) 9 mm (b) 6mm [2005] (c) 10–16 s (d) 10–1 s
(c) 12mm (d) 18mm 49. When 3Li 7 nuclei are bombarded by protons, and the
40. In a common base amplifier, the phase difference between resultant nuclei are 4Be8, the emitted particles will be
the input signal voltage and output voltage is [2005] (a) alpha particles (b) beta particles [2006]
p p (c) gamma photons (d) neutrons
(a) p (b) (c) (d) 0 50. The energy spectrum of b-particles [number N(E) as a
4 2
function of b-energy E] emitted from a radioactive source is
41. The diagram shows the energy levels for an electron in a
[2006]
certain atom. Which transition shown represents the
emission of a photon with the most energy? [2005]
n =4 N(E) N(E)
n =3
(a) E(b) E
n =2 E0 E0

n =1 N(E) N(E)
I II III IV
(a) IV (b) III E E
(c) II (d) I (c) E0 (d) E0
MODERN PHYSICS P-509
51. A solid which is not transparent to visible light and whose
conductivity increases with temperature is formed by
(a) Ionic bonding [2006]
(b) Covalent bonding (a) I (b) I

(c) Vander Waals bonding


(d) Metallic bonding
O l O l
52. If the ratio of the concentration of electrons to that of holes
7 7
in a semiconductor is and the ratio of currents is ,
5 4
I I
then what is the ratio of their drift velocities? [2006]
(c) (d)
5 4 5 4
(a) (b) (c) (d)
8 5 4 7 O l O l
53. The circuit has two oppositively connected ideal diodes in 56. If the binding energy per nucleon in 73 Li and 42 He nuclei
parallel. What is the current flowing in the circuit? [2006]
are 5.60 MeV and 7.06 MeV respectively, then in the reaction
4W p + 73 Li ¾¾ ® 2 42 He
energy of proton must be [2006]
D1 D2
(a) 28.24 MeV (b) 17.28 MeV
(c) 1.46 MeV (d) 39.2 MeV
12V 57. The 'rad' is the correct unit used to report the measurement of
3W 2W
(a) the ability of a beam of gamma ray photons to produce
ions in a target [2006]
(b) the energy delivered by radiation to a target
(a) 1.71 A (b) 2.00 A (c) the biological effect of radiation
(c) 2.31 A (d) 1.33 A (d) the rate of decay of a radioactive source
54. In the following, which one of the diodes reverse biased? 58. If the lattice constant of this semiconductor is decreased,
then which of the following is correct? [2006]
+10 V
conduction
Ec
band width
band gap Eg
(a) R [2006]
+5 V valence
band width Ev
(b) (a) All Ec, Eg, Ev increase
–10 V (b) Ec and Ev increase, but Eg decreases
R (c) Ec and Ev decrease, but Eg increases
(d) All Ec, Eg, Ev decrease
–5 V 59. The rms value of the electric field of the light coming from
the Sun is 720 N/C. The average total energy density of
(c) the electromagnetic wave is [2006]
(a) 4.58 × 10–6 J/m3 (b) 6.37 × 10–9 J/m3
R (c) 81.35 × 10–12 J/m3 (d) 3.3 × 10–3 J/m3
–10 V 60. If MO is the mass of an oxygen isotope 8 O17 ,MP and MN
are the masses of a proton and a neutron respectively, the
+5 V nuclear binding energy of the isotope is [2007]
(a) (MO –17MN)c2
R (b) (MO – 8MP)c2
R (c) (MO– 8MP –9MN)c2
(d) (d) MOc 2
61. In gamma ray emission from a nucleus [2007]
(a) only the proton number changes
(b) both the neutron number and the proton number
55. The anode voltage of a photocell is kept fixed. The change
wavelength l of the light falling on the cathode is gradually (c) there is no change in the proton number and the
changed. The plate current I of the photocell varies as neutron number
follows [2006] (d) only the neutron number changes
P-510 Topic-wise Solved Papers - PHYSICS
62. If in a p-n junction diode, a square input signal of 10 V is 67. Electrons accelerated by potential V are diffracted from a
applied as shown [2007] crystal. If d = 1Å and i = 30°, V should be about
5V (h = 6.6 × 10 – 34 Js, me = 9.1 × 10–31 kg, e = 1.6 × 10 – 19 C)
[2008]
RL (a) 2000 V (b) 50 V
(c) 500 V (d) 1000 V
-5V
68. If a strong diffraction peak is observed when electrons are
Then the output signal across RL will be
incident at an angle ‘i’ from the normal to the crystal planes
10 V with distance ‘d’ between them (see figure), de Broglie
+5V
wavelength ldB of electrons can be calculated by the
(a) (b)
relationship ( n is an integer) [2008]
(a) d sin i = nldB (b) 2d cos i = nldB
(c) 2d sin i = nldB (d) d cos i = nldB
(c) (d) 69. This question contains Statement-1 and statement-2. Of the
-5V four choices given after the statements, choose the one
-10 V
63. Photon of frequency n has a momentum associated with it. that best describes the two statements. [2008]
If c is the velocity of light, the momentum is [2007] Statement-1:
(a) hn / c (b) n /c Energy is released when heavy nuclei undergo fission or
(c) h n c (d) hn / c2 light nuclei undergo fusion and
64. The half-life period of a radio-active element X is same as Statement-2 :
the mean life time of another radio-active element Y. Initially
they have the same number of atoms. Then [2007] For heavy nuclei, binding energy per nucleon increases with
(a) X and Y decay at same rate always increasing Z while for light nuclei it decreases with increasing
(b) X will decay faster than Y Z.
(c) Y will decay faster than X (a) Statement-1 is false, Statement-2 is true
(d) X and Y have same decay rate initially (b) Statement-1 is true, Statement-2 is true; Statement-2 is
65. Carbon, silicon and germanium have four valence electrons a correct explanation for Statement-1
each. At room temperature which one of the following
(c) Statement-1 is true, Statement-2 is true; Statement-2 is
statements is most appropriate ? [2007]
not a correct explanation for Statement-1
(a) The number of free electrons for conduction is
significant only in Si and Ge but small in C. (d) Statement-1 is true, Statement-2 is false
(b) The number of free conduction electrons is significant 70. A working transistor with its three legs marked P, Q and R is
in C but small in Si and Ge. tested using a multimeter. No conduction is found between
(c) The number of free conduction electrons is negligibly P and Q. By connecting the common (negative) terminal of
small in all the three. the multimeter to R and the other (positive) terminal to P or
(d) The number of free electrons for conduction is Q, some resistance is seen on the multimeter. Which of the
significant in all the three. following is true for the transistor? [2008]
66. Which of the following transitions in hydrogen atoms emit
(a) It is an npn transistor with R as base
photons of highest frequency? [2007]
(a) n = 1 to n = 2 (b) n = 2 to n = 6 (b) It is a pnp transistor with R as collector
(c) n = 6 to n = 2 (d) n = 2 to n = 1 (c) It is a pnp transistor with R as emitter
Directions: Question No. 67 and 68 are based on the following (d) It is an npn transistor with R as collector
paragraph. 71. Suppose an electron is attracted towards the origin by a
Wave property of electrons implies that they will show diffraction
effects. Davisson and Germer demonstrated this by diffracting k
force where ‘k’ is a constant and ‘r’ is the distance of the
electrons from crystals. The law governing the diffraction from a r
crystal is obtained by requiring that electron waves reflected from electron from the origin. By applying Bohr model to this
the planes of atoms in a crystal interfere constructively (see figure). system, the radius of the nth orbital of the electron is found
to be ‘rn’ and the kinetic energy of the electron to be ‘Tn’.
Then which of the following is true? [2008]
Inco m ng
i
Electr ng Outgoi s 1 2
i o n (a) Tn µ , rn µ n (b) Tn independent of n, rn µ n
ons Electr 2
n
d
1 1
(c) Tn µ , rn µ n (d) Tn µ , rn µ n 2
n n
Crystal plane
MODERN PHYSICS P-511
72. In the circuit below, A and B represent two inputs and C Output is
represents the output. [2008]
(a)
A

C (b)

B (c)

(d)
The circuit represents
(a) NOR gate (b) AND gate 77. A p-n junction (D) shown in the figure can act as a rectifier.
(c) NAND gate (d) OR gate An alternating current source (V) is connected in the circuit.
73. The transition from the state n = 4 to n = 3 in a hydrogen like
atom results in ultraviolet radiation. Infrared radiation will
D
be obtained in the transition from : [2009]
R
(a) 3 ® 2 (b) 4 ® 2
(c) 5 ® 4 (d) 2 ® 1
74. The surface of a metal is illuminted with the light of 400 nm.
The kinetic energy of the ejected photoelectrons was found V ~
to be 1.68 eV. The work function of the metal is : [2009]
(hc = 1240 eV.nm)
(a) 1.41 eV (b) 1.51 eV
(c) 1.68 eV (d) 3.09 eV The current (I) in the resistor (R) can be shown by :

75.

(a)

(b)

(c)
The above is a plot of binding energy per nucleon Eb, against
the nuclear mass M; A, B, C, D, E, F correspond to different
nuclei. Consider four reactions : [2009] I
(i) A + B ® C + e (ii) C ® A + B + e
(iii) D + E ® F + e and (iv) F® D + E + e,
where e is the energy released? In which reactions is e (d)
positive? t
(a) (i) and (iii) (b) (ii) and (iv)
78. Statement -1 : When ultraviolet light is incident on a
(c) (ii) and (iii) (d) (i) and (iv)
photocell, its stopping potential is V0 and the maximum
76. The logic circuit shown below has the input waveforms ‘A’
kinetic energy of the photoelectrons is Kmax .When the
and ‘B’ as shown. Pick out the correct output waveform. ultraviolet light is replaced by X-rays, both V0 and Kmax
[2009] increase.
A Statement -2 : Photoelectrons are emitted with speeds
Y ranging from zero to a maximum value because of the range
B of frequencies present in the incident light. [2010]
(a) Statement -1 is true, Statement -2 is true ; Statement -2
Input A is the correct explanation of Statement -1.
(b) Statement -1 is true, Statement -2 is true; Statement -2
is not the correct explanation of Statement -1
Input B (c) Statement -1 is false, Statement -2 is true.
(d) Statement -1 is true, Statement -2 is false.
P-512 Topic-wise Solved Papers - PHYSICS
DIRECTIONS: Questions number 79-80 are based on the 86. The half life of a radioactive substance is 20 minutes. The
following paragraph. approximate time interval (t2 – t1) between the time t2 when
A nucleus of mass M + Dm is at rest and decays into two daughter 2 1
M of it had decayed and time t1 when of it had decayed is :
3 3
nuclei of equal mass each. Speed of light is c.
2 (a) 14 min (b) 20 min [2011]
79. The binding energy per nucleon for the parent nucleus is E1 (c) 28 min (d) 7 min
and that for the daughter nuclei is E2. Then [2010] 87. This question has Statement – 1 and Statement – 2. Of the
(a) E2 = 2E1 (b) E1 > E2 four choices given after the statements, choose the one
(c) E2 > E1 (d) E1 = 2 E2 that best describes the two statements. [2011]
Statement – 1: A metallic surface is irradiated by a
80. The speed of daughter nuclei is [2010]
monochromatic light of frequency v > v0 (the threshold
Dm 2Dm frequency). The maximum kinetic energy and the stopping
(a) c (b) c potential are Kmax and V0 respectively. If the frequency
M + Dm M
incident on the surface is doubled, both the Kmax and V0 are
Dm Dm also doubled.
(c) c (d) c Statement – 2 : The maximum kinetic energy and the
M M + Dm
81. A radioactive nucleus (initial mass number A and atomic stopping potential of photoelectrons emitted from a surface
number Z emits 3 a - particles and 2 positrons. The ratio of are linearly dependent on the frequency of incident light.
number of neutrons to that of protons in the final nucleus (a) Statement–1 is true, Statement–2 is true, Statement – 2
will be [2010] is the correct explanation of Statement – 1.
(b) Statement–1 is true, Statement–2 is true, Statement – 2
A- Z -8 A- Z -4 is not the correct explanation of Statement – 1.
(a) (b)
Z -4 Z -8 (c) Statement – 1 is false, Statement – 2 is true.
A - Z - 12 A- Z -4 (d) Statement – 1 is true, Statement – 2 is false.
(c) (d) 88. The output of an OR gate is connected to both the inputs of
Z -4 Z -2 a NAND gate. The combination will serve as a: [2011RS]
82. The combination of gates shown below yields [2010] (a) NOT gate (b) NOR gate
(c) AND gate (d) OR gate
A 89. After absorbing a slowly moving neutron of mass mN
(momentum » 0) a nucleus of mass M breaks into two nuclei
X of masses m1 and 5m1 (6m1 = M + mN) respectively. If the
de Broglie wavelength of the nucleus with mass m1 is l, the
B de Broglie wavelength of the nucleus will be [2011RS]
(a) 5l (b) l / 5
(a) OR gate (b) NOT gate (c) l (d) 25l
(c) XOR gate (d) NAND gate 90. Which of the following four alternatives is not correct ?
83. If a source of power 4kW produces 1020 photons/second , We need modulation : [2011RS]
the radiation belongs to a part of the spectrum called [2010] (a) to reduce the time lag between transmission and
(a) X -rays (b) ultraviolet rays reception of the information signal
(c) microwaves (d) g -rays (b) to reduce the size of antenna
84. This question has Statement – 1 and Statement – 2. Of the (c) to reduce the fractional band width, that is the ratio
four choices given after the statements, choose the one of the signal band width to the centre frequency
that best describes the two statements. [2011] (d) to increase the selectivity
Statement – 1 : Sky wave signals are used for long distance 91. Statement - 1 : A nucleus having energy E1 decays by
radio communication. These signals are in general, less b– emission to daughter nucleus having energy E2, but the
stable than ground wave signals. b– rays are emitted with a continuous energy spectrum
Statement – 2 : The state of ionosphere varies from hour to having end point energy E1 – E2. [2011RS]
hour, day to day and season to season. Statement - 2 : To conserve energy and momentum in
(a) Statement–1 is true, Statement–2 is true, Statement–2 b – decay at least three particles must take part in the
is the correct explanation of Statement–1. transformation.
(b) Statement–1 is true, Statement–2 is true, Statement–2 (a) Statement-1 is correct but statement-2 is not correct.
is not the correct explanation of Statement – 1. (b) Statement-1 and statement-2 both are correct and
(c) Statement – 1 is false, Statement – 2 is true. statement-2 is the correct explanation of statement-1.
(d) Statement – 1 is true, Statement – 2 is false.
(c) Statement-1 is correct, statement-2 is correct and
85. Energy required for the electron excitation in Li++ from the
statement-2 is not the correct explanation of
first to the third Bohr orbit is : [2011]
statement-1
(a) 36.3 eV (b) 108.8 eV
(d) Statement-1 is incorrect, statement-2 is correct.
(c) 122.4 eV (d) 12.1 eV
MODERN PHYSICS P-513
92. Hydrogen atom is excited from ground state to another state
with principal quantum number equal to 4. Then the number D
of spectral lines in the emission spectra will be : [2012]
(a) 2 (b) 3
Signal C R
(c) 5 (d) 6
93. Truth table for system of four NAND gates as shown in
figure is : [2012]
(a) 10.62 MHz (b) 10.62 kHz
A (c) 5.31 MHz (d) 5.31 kHz
98. Themagnetic field in a travelling electromagnetic wave has
a peak value of 20 nT. The peak value of electric field strength
Y is : [JEE Main 2013]
(a) 3 V/m (b) 6 V/m
(c) 9 V/m (d) 12 V/m
B 99. The anode voltage of a photocell is kept fixed. The
wavelength l of the light falling on the cathode is gradually
AB Y AB Y changed. The plate current I of the photocell varies as
0 0 0 0 0 0 follows : [JEE Main 2013]

0 1 1 0 1 0
(a)
1 0 1 (b)
1 0 1 I I

1 1 0 1 1 1 (a) (b)
AB Y AB Y O l O l
0 0 1 0 0 1
0 1 1 0 1 0 I I
(c)
1 0 0 (d)
1 0 1
1 1 0 1 1 1 (c) (d)
94. A radar has a power of 1kW and is operating at a frequency
of 10 GHz. It is located on a mountain top of height 500 m. O l O l
The maximum distance upto which it can detect object
100. The I-V characteristic of an LED is [JEE Main 2013]
located on the surface of the earth (Radius of earth
= 6.4 × 106m) is : [2012]
(a) 80 km (b) 16 km B
(c) 40 km (d) 64 km G
I Y
95. Assume that a neutron breaks into a proton and an electron. R
(a) (b)
The energy released during this process is : (mass of neutron
= 1.6725 × 10–27 kg, mass of proton = 1.6725 × 10–27 kg, O V O V
mass of electron = 9 × 10–31 kg). [2012]
(a) 0.73 MeV (b) 7.10 MeV
(c) 6.30 MeV (d) 5.4 MeV V O
96. A diatomic molecule is made of two masses m1 and m2 which I R
are separated by a distance r. If we calculate its rotational I
Y
energy by applying Bohr's rule of angular momentum (c) (d) G
quantization, its energy will be given by : (n is an integer) B
O V
(m1 + m2 )2 n2 h2 n2 h 2
(a) (b) 101. In a hydrogen like atom electron make transition from an
2m12 m22 r 2 2(m1 + m2 )r 2
energy level with quantum number n to another with quantum
2n 2 h 2 (m1 + m2 )n2 h 2 number (n – 1). If n>>1, the frequency of radiation emitted is
(c) (d) proportional to : [JEE Main 2013]
(m1 + m2 )r 2 2m1m2 r 2
97. A diode detector is used to detect an amplitudemodulated 1 1
(a) (b)
wave of 60% modulation by using a condenser of capacity n n2
250 picofarad in parallel with a load resistance 100 kilo ohm.
1 1
Find the maximum modulated frequency which could be (c) (d)
3
detected by it. [JEE Main 2013] n n3
2
P-514 Topic-wise Solved Papers - PHYSICS

Solutions & Explanations


Section-A : JEE Advanced/ IIT-JEE

A 1. 30,000, 30,000 2. 500 disintegration/sec, 125 disintegration/sec


3. frequency 4. eight, six
5. intensity, decreases 6. lithium, 7 7. atomic number, mass number 8. 23.6 MeV
9. positive, p-part, n-part 10. reverse, negative terminal 11. 0.27Å 12. 3.81Å
13. B and D, A and C 14. 3 × 10 8 15. –1 16. neutrino 17. reverse
18. 0.62Å 19. 41 20. fusion, 24.03 21. 4.17
B 1. F 2. F 3. T 4. F
C 1. (b) 2. (b) 3. (c) 4. (c) 5. (a) 6. (d)
7. (c) 8. (b) 9. (a) 10. (d) 11. (a) 12. (b)
13. (b) 14. (c) 15. (c) 16. (b) 17. (d) 18. (d)
19. (b) 20. (c) 21. (b) 22. (b) 23. (c) 24. (a)
25. (b) 26. (b) 27. (c) 28. (c) 29. (a) 30. (d)
31. (d) 32. (c) 33. (d) 34. (a) 35. (d) 36. (a)
37. (b) 38. (a) 39. (c) 40. (a) 41. (d) 42. (a)
43. (b) 44. (d) 45. (b) 46. (d) 47. (a) 48. (a)
49. (c) 50. (c) 51. (c) 52. (a) 53. (b) 54. (a)
55. (b) 56. (a) 57. (a) 58. (a) 59. (b)
D 1. (b, d) 2. (c, d) 3. (b, c) 4. (a, c, d) 5. (a) 6. (d)
7. (c) 8. (c, d) 9. (a) 10. (a, b, c) 11. (c) 12. (d)
13. (c, d) 14. (b) 15. (b, d) 16. (b) 17. (b) 18. (b, d)
19. (b, c) 20. (c) 21. (a,b,c) 22. (a, d) 23. (a, c) 24. (a, c)
25. (c, d) 26. (a, d) 27. (d) 28. (c) 29. (b) 30. (c)
31. (c) 32. (a, c) 33. (b,d) 34. (a,c)
E 1. (i) 5 (ii) 16.53 eV (iii) 3.65 × 10–9 m (iv) 340 eV, –680 eV, 1.05 × 10–34 J.s (v) 1.05 × 10–11m 2. 6, 18.835 × 10–7 m
3. 0, 11, 13 4. 91, 234 5. 6.6 × 10–34 J.s. 6. (i) 300Å (ii) 2.5 × 10–11 m
7. –8
(i) 1.142 × 10 (ii) 3 8. 3.96 × 10–6 9. 8 mW, 12.5 × 10–3 s, 100

n2 h 2 Î0
10. (i) r = (ii) 25 (iii) 5.478 × 10–11 m 11. 11 × 1011 (approx.)
624pme 2
12. (i) 2 (ii) 14.46 eV (iii) 13.5 eV, 0.7 eV 13. 2 eV, 0.754 V 14. 119.6 gm

16. (a) 232, 90 (b) 5.34 MeV, 1823 MeV 17. (a) 2.55 eV (b) 4 ® 2 (c) - h (d) 0.814 m/s
p
18. (i) 6.36 eV, 17.84 eV; 0.312 eV, 16.328 eV (ii) 1.82 × 1015 Hz, 11.67 × 1015 Hz, 9.84 × 1015 Hz
19. 5.95 l 20. 6, 3 21. (i) 3.4 eV (ii) 0.66 × 10–9 m 22. (i) 14.43 sec. (ii) 40 sec.
23. 1.868 × 109 years 24. 151 eV, 0.5 Å 25. 3.32 × 10–5 W
1é 3
26. (a) a - (a - lN 0 )e -lt ù (b) N 0 , 2 N0 27. 3.4 eV, 3.84 eV and 2.64 eV 28. 2, 4, 10.5 eV
lë û 2
29. 6.25 × 1011, 0 eV, 5 eV 30. 38451 Kg 31. 10–12 J, 227.62 amu
20
10 20 æ 3 3 - 4ö
32. 15 loge 3, , 10 ç ÷ 33. (a) 3 (b) 4052 nm
3 3 è 3 3 ø
MODERN PHYSICS P-515

2
34. (a) 5 × 107 (b) 2000 N/C (c) 23 eV 35. 42 36.
log e (4 / 3)

38. 0.26 39. 0.55 eV 40. 2 41. (a) 56 (b) 1.546 × 1018 Hz. 42. 24

F 1. A-p; B-t; C-u; D-r 2. A-p, q; B-p, r; C-p, s; D-p, q, r


3. A-p, r; B-q, s; C-p; D-q 4. A-p, q, t; B-q; C-s; D-s 5. (c)
G 1. (c) 2. (c) 3. (a) 4. (d) 5. (a) 6. (b)
7. (a) 8. (b) 9. (d) 10. (d) 11. (b) 12. (c)
13. (d) 14. (c) 15. (a) 16. (c)
H 1. (b)
I 1. (3) 2. (8) 3. (1) 4. (7) 5. (7) 6. (1) 7. (4)

Section-B : JEE Main/ AIEEE

1. (c) 2. (c) 3. (a) 4. (c) 5. (a) 6. (a) 7. (c) 8. (c) 9. (a) 10. (c) 11. (c) 12. (a)
13. (c) 14. (b) 15. (a) 16. (b) 17. (a) 18. (a) 19. (d) 20. (b) 21. (c) 22. (c) 23. (a) 24. (b)
25. (d) 26. (a) 27. (b) 28. (a) 29. (a) 30. (d) 31. (d) 32. (d) 33. (b) 34. (a) 35. (b) 36. (d)
37. (a) 38. (d) 39. (c) 40. (d) 41. (b) 42. (d) 43. (a) 44. (d) 45. (a) 46. (a) 47. (b) 48. (b)
49. (c) 50. (c) 51. (b) 52. (c) 53. (b) 54. (d) 55. (b) 56. (b) 57. (c) 58. (c) 59. (a) 60. (c)
61. (c) 62. (a) 63. (a) 64. (c) 65. (a) 66. (d) 67. (b) 68. (b) 69. (d) 70. (N) 71. (b) 72. (d)
73. (c) 74. (a) 75. (d) 76. (d) 77. (b) 78. (d) 79. (c) 80. (b) 81. (b) 82. (a) 83. (a) 84. (b)
85. (b) 86. (b) 87. (c) 88. (b) 89. (c) 90. (a) 91. (b) 92. (d) 93. (a) 94. (a) 95. (a) 96. (d)
97. (b) 98. (b) 99. (d) 100. (a) 101. (d))

FILL IN THE BLANKS :


\ Minimum accelerating voltage,
E1
1. For minimum accelerating voltage, the electron should jump Vmin = = 30, 000 V
from n = 2 to n = 1 level. e
For characteristic X-rays n
æ 1ö
2. A = A0 ç ÷ where A0 = Initial activity = 1000 dps (given)
1 é 1ù E è 2ø
= Ra (Z – 1)2 ê1 - 2 ú = = A = Activity after n half lives
l ë n û hc
1
æ1ö
E1 2é 1ù At t = 1, n = 1 \ A = 1000 ç ÷ = 500 dps
\ = Ra ( Z – 1) ê1 – ú ......(i) è2ø
hc ë 22 û
3
The binding energy of innermost electron = 40 keV æ 1ö
At t = 3, n = 3 \ A = 1000 ç ÷ = 125 dps
\ Ionisation potential of tungsten = 40 kV = 40 × 103 V è 2ø
3. Note : According to law of photoelectric effect
E2 é 1 ù
Þ = Ra ( Z - 1) 2 ê1 - ú .....(ii) (K.E.)max = hn – hn0
hc ë ¥2 û i.e., the maximum kinetic energy of electrons emitted in the
photoelectric effect is linearly dependent on the frequency
é 1ù of incident radiation.
ê1 – 2 ú
E1 ë 2 û 238 206 4
y –10 e
\ = 4. 92 U ® 82 Pb + x 2 He +
E2 é 1 ù First we find the number of a- particles. The change in mass
ê1 – 2 ú number during the decay from uranium to lead = 238 – 206 =
ë ¥ û
32. Therefore, the number of a-particles (with mass no. 4)
3 3 32
Þ E1 = E2 = ´ 40, 000 eV = 30, 000 eV = =8
4 4 4
P-516 Topic-wise Solved Papers - PHYSICS
The change in atomic number (i.e, number of protons) taking 11 +
16. 6 C ® 11 11 11 0
5 B + b + X Þ 6 C ® 5 B + +1e + n (neutrino)
place when 8 a-particles are emitted and lead is formed is
= 92 – (82 + 2 × 8) = 92 – (82 + 16) = 92 – 98 = – 6 The balancing of atomic number and mass number is correct.
This change will take place by emitting of six b-particles. Therefore, X stands for neutrino.
5. Note : More the number of electrons striking the anode, 17. Reverse
more is the intensity of X-rays. –34
´ 3 ´ 108
hc 6.63 ´ 10
When the speed of the striking electrons on anode is 18. lmin = = = 0.62Å
increased, the emitted X-rays have greater energy. We know eV 1.6 ´ 10 –19 ´ 20 ´ 103

hc 1 é1 1ù
that energy, E = . Therefore, when E increases then l 19. = R( Z –1)2 ê 2 – 2 ú
l l êë n1 n2 úû
decreases.
Since for Ka, n2 = 2 and n1 = 1
10 1 4
6. 5 B + 0n ¾¾ ® 2 He + 37 Li 1 é1 1 ù
The resulting nucleus is of element lithium and mass number \ –10 = 1.097 (Z – 1)2 ê 2 – 2 ú
0.76 ´ 10 ë1 2 û
is 7.
7. Atomic number, mass number Þ z – 1 = 40 Þ Z = 41
2 2 n=2
8. 1 H + 1H ¾¾ ® 42 He
X-ray
Binding energy of two deuterons
= 2 [1.1 × 2] = 4.4 MeV
Binding energy of helium nucleus = 4 × 7.0 = 28 MeV n=1
The energy released = 28 – 4.4 = 23.6 MeV 20. This is a nuclear fusion reaction
9. Positive, p-part, n-part Energy released = (Dm) [931.5 MeV/u]
10. Reverse, negative terminal. = [2 × 2.0141 – 4.0024 ] × 931.5 MeV
11. We know that = 24.03 MeV
21. For photon emitted from hydrogen atom, the wavelength is
1 é1 1 ù
For Ka , = C ê – ú , where C is a constant æ 1
l 2 2 1 1ö
êë n1 n2 úû = Rç 2 – 2 ÷ ...(i)
l è n1 n2 ø
1 é1 1 ù 3C
Þ =Cê 2 – 2ú = ....(i) But according to de Broglie concept
0.32Å ë1 2 û 4
h 1 p
é1 1 ù 8C l= Þ = ...(ii)
1 p l h
For Kb, =Cê 2 – 2ú=
l ë1 3 û 9 From (i) and (ii),
On dividing, we get l = 0.27 Å.
12. The fifth valence electron of phosphorous is in its third p æ 1 1ö æ 1 1ö
= R ç 2 – 2 ÷ Þ p = Rh ç 2 – 2 ÷ ....(iii)
shell, i.e., n = 3. For phosphorous, Z = 15. The Bohr’s radius h è n1 n2 ø è n1 n2 ø
for nth orbit
Note : Since the momentum of the hydrogen atom initially
æ n2 ö 32 was zero, therefore the momentum of photon is finally equal
=ç e r ÷ r0 = ´ 12 ´ 0.529 Å = 3.81Å to momentum of hydrogen atom in magnitude. (By law of
è Z ø 15
conservation of momentum).
13. B and D is a.c. input and A and C is the d.c. output. Let the momentum of hydrogen atom be mHvH
Case (i) When B is –ve and D is +ve
æ 1 1ö
Current passes from D ® A ® C ® B Then from (iii), mHvH = Rh ç 2 – 2 ÷
Case (ii) When B is + ve and D is – ve è n1 n2 ø
Current passes from B ® A ® C ® B
Thus curve is always from A to C in output (a d.c. current) Rh é1 1ù
14. The speed of X-rays is always 3 × 108 m/s in vacuum. It Þ vH = ê 2 – 2ú
mH êë n1 n2 úû
does not depend on the potential differences through which
electrons are accelerated in an X-ray tube.
Note : All electromagnetic waves propagate at 3 × 108 m/s in 1.097 ´ 107 ´ 6.63 ´ 10 –34 æ 1 1 ö
= çè 2 – 2 ÷ø
vacuum. 1.67 ´ 10 –27 1 5
Þ vH = 4.178 m/s
kZe2
15. K.E. = and ALTERNATIVE SOLUTION :
2r Energy of photon
– kZ e 2 K.E. é1 1ù
Total energy T.E. = \ = -1 E = E5 – E1 = –13.6 ê 2 – 2 ú eV = 2.09 × 10–18J
2r T.E. ë5 2 û
MODERN PHYSICS P-517
According to momentum conservation,
Momentum of Recoil hydrogen atom=Momentum of photon 9 ´ 109 ´ 2 ´ 1.6 ´ 10 –19 ´ 92 ´ 1.6 ´ 10 –19
\ r=
–18 5 ´ 1.6 ´ 10 –13
E E = 2.09 ´ 10
\ mv = Þ v = = 4.17 m/s = 529.92 × 10–16 m
c mc (1.67 ´ 10 –27 )(3 ´ 108 ) = 529.92 × 10–14 cm = 5.2992 × 10–12 cm
4. (c) b-particles are charged particles emitted by the nucleus.
TRUE / FALSE : 5. (a) KEY CONCEPT : The maximum number of electrons
1. For photoelectric effect in an orbit is 2n2. n > 4 is not allowed.
hn – hn0 = (K.E.)max Therefore the number of maximum electron that can be
where h = Planck’s constt. in first four orbits are
n0 = Threshold frequency 2 (1)2 + 2 (2)2 + 2 (3)2 + 2 (4) 2
Þ (K.E.)max µ n = 2 + 8 + 18 + 32 = 60
K.E. does not depend on the intensity of incident radiation. Therefore, possible element are 60.
2. (K.E.)max = hn – hn0 Þ (K.E.)max µ n 6. (d) We know that
Thus maximum kinetic energy is proportional to frequency é1
1 1ù 1 2
and not intensity. = RZ 2 ê 2 – 2 ú Þ µ Z
l l
3. Note : When the cathode temperature is higher, then more ëê n2 n1 ûú
number of electrons will be emitted which in turn will increase
the anode current. 1
l is shortest when is largest i.e., when Z has a higher
l
m A ´ 1.67 ´ 10 –27 value. Z is highest for lithium.
4. Density = =
V 4 é 3
p R0 A1/ 3 ù 7. (c)411H + ® 42 He 2 + + 2e – + 26 MeV
3 ë û
represent a fusion reaction.
1.67 ´ 10 –27 8. (b) Fast neutrons can be easily slowed down by passing
= = 3 × 1017 kg/m3 them through water.
1.33 ´ 3.14 ´ (1.1 ´ 10 –15 ) 9. (a) Note : The penetrating power is dependent on velocity.
where A = mass number. For a given energy, the velocity of g radiation is highest
Note : The order of nuclear density is 1017 kg/m3. and a-particle is least.
10. (d) When one e– is removed from neutral helium atom, it
MCQ's WITH ONE CORRECT ANSWER : becomes a one e– species.
1. (b) KEY CONCEPT : For one e– species we know
We know that µ = gm × r0
–13.6Z 2
where µ = amplification factor, En = eV/atom
gm = mutual conductance n2
r0 = plate resistance For helium ion, Z = 2 and for first orbit n = 1.
\ µ = 3 × 103 × 1.5 × 10–3 = 4.5
2. (b) t1/2 = 3.8 day –13.6
\ E1 = ´ 22 = – 54.4 eV
2
0.693 0.693 (1)
\ l= = = 0.182 \ Energy required to remove this e– = + 54.4 eV
t1/ 2 3.8
\ Total energy required = 54.4 + 24.6 = 79 eV
If the initial number of atom is a = A0 then after time t
the number of atoms is a/20 = A. We have to find t. –dN
11. (a) = l1 N + l 2 N
dt
2.303 A0 2.303 a
t = log = log = 16.46 days N
l A 0.182 a / 20 Þ log e = – ( l1 + l 2 ) t
No
3. (c) One point charge is ( 235
92 U ) uranium nucleus when No is initial number of atoms
\ q1 = 92e 0.693 0.693
Here l1 = and l 2 = ;
+2e 1620 810
92e
N 1
= Þ log e = – æç
1 0.693 0.693ö
d + ÷t
No 4 4 è 1620 810 ø
The other point charge is a particle \ q2= + 2e
Here the loss in K.E. = Gain in P.E. (till a-particle reaches Þ t = 1 1080 years
the distance d) 12. (b) KEY CONCEPT : For a semi conductor n = n0e–Eg/kT
where n0 = no. of free electrons at absolute zero, n =
1 2 qq 2q1q2
Þ mv = k 1 2 Þ r = k no. of free electrons at T kelvin, Eg = Energy gap, k =
2 r 1 2 Boltzmann constant.
mv
2 As Eg increases, n decreases exponentially.
P-518 Topic-wise Solved Papers - PHYSICS
13. (b) As shown in the fig. (i) during one half cycle the polarity 22. (b) KEY CONCEPT :
of P and S are opposite such that diode (1) is reversed According to Doppler’s effect of light, the wavelength
biased and hence non conducting. shift is given by
+ I Forward biased Reversed biased v

Dl = ´l

p.n
+
p.n c
S S
Dl ´ c (706 – 656)
P P Þ v= = ´ 3 ´ 108 » 2 × 107 m/s
+ Output – Output l 656
p.n p.n
23. (c) Applying conservation of linear momentum,
– 1 + I 1 Initial momentum = Final momentum
Reverse bias Forward biased
0 = m1v1 – m2v2 Þ m1v1 = m2v2
fig. (i) fig. (ii)
During the other half cycle, diode (1) gets forward l1 h / m1v1
Now, = =1
biased and is conducting. Thus diode (1) conducts in l 2 h / m2 v2
one half cycle and does not conduct in the other so the 24. (a) Beta rays are same as cathode rays as both are stream
correct option is (b) (a and c.) of electrons.
12400 12400 25. (b) Nuclear density of an atom of mass number A,
14. (c) E = eV = eV = 59 keV
l (inÅ) 0.21 mass A(1.67 ´ 10 –27 )
d= =
volume 4
hc 6.63 ´ 10 –34 ´ 3 ´ 108 p[1.25 ´ 10 –15 A1/ 3 ]3
15. (c) E = = = 0.5eV 3
l 2480 ´ 10–9 ´ 1.6 ´ 10 –19
é 4 3 –15 ù
êëQ V = 3 pR , R = R0 A , R0 = 1.25 ´ 10 úû
16. (b) The charge carriers in forward bias diffuse and they 1/ 3
drift in reverse biased p-n junctions.
17. (d) The mass defect in 64Zn is more than 64Cu. Therefore, \ d = 2 × 1017 kg/m3.
Zn is more stable and 64Cu is radioactive and will decay 22 4 4 14
to 64Zn through b-decay as follows 26. (b) 10 Ne ® 2 He + 2 He + 6 X
64 64 The new element X has atomic number 6. Therefore, it
® 30
29 Cu Zn + –10 e is carbon atom.
18. (d) KEY CONCEPT : 27. (c) KEY CONCEPT : Energy is released when stability

En = –13.6
( Z ) eV
2 increases. This will happen when binding energy per
nucleon increases.
(n2 ) Reactant Product
Reaction (a) 60 × 8.5MeV = 510MeV 2 × 30 × 5 = 300 MeV
Therefore, ground state energy of doubly ionized Reaction (b) 120 × 7.5 = 900 MeV (90 × 8 + 30 × 5) = 870 MeV
lithium atom (Z = 3, n = 1) will be Reaction (c) 120 × 7.5 = 900 MeV 2 × 60 × 8.5 = 1020 MeV
Reaction (d) 90 × 8 = 720 MeV (60 × 8.5 + 30 × 5) = 600 MeV
(3) 2 28. (c) KEY CONCEPT :
E1 = (–13.6) = –122.4 eV
(1) 2 lµ
1
\ Ionization energy of an electron in ground state of m
doubly ionized lithium atom will be 122.4eV. For ordinary hydrogen atom, longest wavelength
19. (b) In the circuit, diode D1 is forward biased, while D2 is
1 é1 1 ù 5R 36
reverse biased. Therefore, current i (through D1 and = Rê – ú = or l =
100W resistance) will be l ë2 2
32 û 36 5R
6 With hypothetical particle, required wavelength
i= = 0.02 A
50 + 100 + 150 1 36 18
l' = ´ =
D1 150W 2 5R 5R
29. (a) NOTE :
D2 50W As the electron comes nearer to the nucleus the
i potential energy decreases

100W æ – k .Ze 2 ö
çQ = P.E. and r decreases÷
6V è r ø
Here, 50W is the resistance of D1 in forward biasing.
20. (c) In n-type semiconductors, electrons are the majority é 1 1 kZe2 ù
charge carriers. The K.E. will increase êQ K.E. = | P.E. | = ú
ë 2 2 r úû
21. (b) Note :
Stopping potential is the negative potential applied to é 1 kZe2 ù
stop the electrons having maximum kinetic energy. The total energy decreases êT.E. = – ú
Therefore, stopping potential will be 4 volt. ëê 2 r ûú
MODERN PHYSICS P-519
30. (d) N1 = N0e–10lt and N2 = N0e–lt. 36. (a) KEY CONCEPT :
N1 e –10 lt 1 I=
q ne
=
\ = =
N2 e – lt
e 9l t t t
No. of electrons striking the target per second
N1 1 1 1 I
Given = ; \ 9lt = = = 2 ´ 1016
N2 e e e e
æ 1ö nh
or, 9lt = 1 or t = ç ÷ 37. (b) l= ,| E |µ Z 2 / n2 ; n = 3
è 9l ø 2p
31. (d) KEY CONCEPT : Þ lH = lLi and |EH| < |EL i|
hc 38. (a) A = A0 (1/2)n; n = number of half lives.
lmin =
E n 4 n
A0 æ 1ö æ 1ö æ 1ö
12400 = A0 ç ÷ \ ç ÷ =ç ÷
\ lmin = Å = 0.155Å 16 è 2ø è 2ø è 2ø
80 ´ 103 Þ n=4
Energy of incident electrons is greater than the \ t = (4 ´ 100)ms = 400 µs
ionization energy of electrons in K-shell, the K-shell
electrons will be knocked off. Hence, characteristic 39. (c) In g-decay, the atomic number and mass number do
X-ray spectrum will be obtained. not change.
32. (c) Note : In a nucleus neutron converts into proton as 40. (a) Given potential energy between electron and proton
follows r
n ® p+ + e–1 = eV0 log [Q | U |= eV ]
r0
Thus, decay of neutron is responsible for b-radiation
origination d é r ù eV0 1
33. (d) For 2 to 1, 3 to 2 and 4 to 2 we get energy that n = 4 to \ |F| = êeV0 loge ú = ´
n = 3, dr ë r0 û r0 r
I.R. radiation has less energy than U.V. radiation. But this force acts as centripetal force
34. (a) KEY CONCEPT :
In case of Coolidge tube mv 2 eV0 eV
\ = Þ mv 2 = 0 ...(i)
hc r rr0 r0
lmin = = l (as given here)
eV nh
Thus the cut off wavelength is inversely proportional By Bohr’s postulate, mvr = ....(ii)
2p
to accelerating voltage. As V increases, lc decreases. From (i) and (ii),
lk is the wavelength of Kµ line which is a characteristic
of an atom and does not depend on accelerating voltage m2 v 2 r 2 n2 h2 r0
of bombarding electron since lk always refers to a =
photon wavelength of transition of e– from the target mv 2 4p 2 ´ V0 e
element from 2 ® 1.
The above two facts lead to the conclusion that n 2 h2 r0
Þ r2 = Þrµn
lk – lc increases as accelerating voltage is increased. 4pV0 me
t 41. (d) KEY CONCEPT : For an atom following Bohr’s

35. (d) N1 = N 0 e – l1t = N 0 e t ....(i) model, the radius is given by
1 r0 m 2
as t = rm = where r0 = Bohr’s radius and m = orbit
l1 Z
number.
t
– 1 For Fm, m = 5 (Fifth orbit in which the outermost electron
N2 = N 0 e – l 2t = N0 e 5t ....(i) as 5t = is present)
l2
Adding (i) and (ii) we get r0 52 1
\ rm = = nr0 (given) Þ n = 4
N = N1 + N2 = N0 (e – t / t + e – t / 5t ) 100
(a) is NOT the correct option as there is a time t for 42. (a) KEY CONCEPT : We know that radius of the nucleus
which N is constant which means for time t there is no R = R0A1/3, where A is the mass number.
process of radioactivity which does not makes sense.
\ R 3 = R03 A
(b) and (c) shows intermediate increase in the number
of radioactive atom which is IMPOSSIBLE as N will 4 3 4 3
only decrease exponentially. Þ pR = pR0 A Þ Volume µ mass.
3 3
P-520 Topic-wise Solved Papers - PHYSICS
43. (b) By conservation of momentum, p1 = p2 The electron of hydrogen atom will jump to second
orbit after absorbing the photon of energy 10.2 eV. The
2 K1m1 = 2 K 2 m2 electron jumps back to its original state in less than
microsecond and release a photon of energy 10.2 eV.
Þ 2 K1 (216) = 2 K 2 (4)
Another photon of energy 15 eV strikes the hydrogen
Þ K2 = 54K1 ...(i) atom inelastically. This energy is sufficient to knock
Also, K1 + K2 = 5.5 MeV ...(ii) out the electron from the atom as ionisation energy is
Solve equation (i) and (ii) 13.6 eV. The remaining energy of 1.4 eV is left with
44. (d) From the graph it is clear that A and B have the same electron as its kinetic energy.
stopping potential and therefore, the same frequency. 49. (c) Note : Since electron shows wave nature, it will show
Also, B and C have the same intensity. the phenomenon of interference.
45. (b) We know that h
For electron, l =
2.303 A0 mv
l= log where A0 is initial activity and
t A When speed of electron increases, l will decrease. The
A is the activity at time t. distance between two consecutive fringes
2.303 A0 2.303 A0 lD
\ l = log = log b=
280 6000 420 3000 d
On solving, we get, A0 = 24000 dps. As l decreases, b also decrease.
ALTERNATE SOLUTION 16
50. (c) 4 42 He ¾¾ ® 8O
1
In two half lives, the activity will remain of its initial B.E. = Dm × 931.5 MeV
4 = (4 × 4.0026 – 15.9994) × 931.5 = 10.24 MeV
activity. 51. (c) For a nucleus to disintegrate in two half life, the
46. (d) For photon,
3
hc probability is as 75% of the nuclei will disintegrate
l2 = ...(i) 4
E in this time.
52. (a) Iodine and Yttrium are medium sized nuclei and
For proton, p = 2mE therefore, have more binding energy per nucleon as
h h compared to Uranium which has a big nuclei and less
l1 = = ...(ii) B.E./nucleon.
p 2mE In other words, Iodine and Yttrium are more stable and
l2 hc therefore possess less energy and less rest mass. Also
= µ E -1/ 2 when Uranium nuclei explodes, it will convert into I
l1 h
E´ and Y nuclei having kinetic energies.
2mE 53. (b) The smallest frequency and largest wavelength in
47. (a) According to the Moseley’s law ultraviolet region will be for transition of electron from
orbit 2 to orbit 1.
f = a (Z – b) Þ f = a 2 (Z – b)2
1 æ 1 1ö
c \ = Rç 2 – 2÷
Þ = a 2 ( Z – b)2 ...(i) l è n1 n2 ø
l
For ka line, b = 1 1 é1 1ù é 1 ù 3R
Þ = R ê – ú = R ê1 – ú =
2 –9 2 2 ë 4û 4
l 2 ( Z1 –1)2 4l (11 –1) 122 ´ 10 m ë1 2 û
From (i), - Þ =
l1 ( Z 2 –1) 2 l ( Z 2 –1)2 4
Þ R= m –1
10 3 ´ 122 ´ 10–9
Þ Z2 – 1 = Þ Z2 = 6 The highest frequency and smallest wavelength for
2 infrared region will be for transition of electron from ¥
ALTERNATE SOLUTION to 3rd orbit.
1 æ 1
For Ka , µ (Z – 1) 2 1 1ö
l \ = Rç 2 – 2÷
l è n1 n2 ø
48. (a) Initially a photon of energy 10.2eV collides inelastically
with a hydrogen atom in ground state. For hydrogen
1 4 æ 1 1ö
atom, Þ = –
l 3 ´ 122 ´ 10 çè 32 ¥ ÷ø
–9
13.6
E1 = – 13.6 eV; E2 = – eV = – 3.4eV
4 3 ´ 122 ´ 9 ´ 10 –9
\ l= = 823.5nm
\ E2 – E1= 10.2 eV 4
MODERN PHYSICS P-521
54. (a) The cut off wavelength is given by For metal plate r :
hc fr = 3 eV
l0 = ...(i) Only wavelength of 350 nm will be able to eject electrons
eV and therefore, current is minimum. Also the stopping
According to de Broglie equation potential is least.
h h
l= = 1 é1 1ù
p 2meV 58. (a) We know that = RZ 2 ê 2 - 2 ú
l êë n1 n2 úû
h2 h2 ..(ii)
Þ l2 = ÞV = The wave length of first spectral line in the
2meV 2mel 2 Balmer series of hydrogen atom is 6561Å . Here n2 = 3
From (i) and (ii), and n1 = 2
hc ´ 2mel 2 2mcl 2 1 2æ1 1ö 5R
l0 = = \ 6561 = R(1) ç 4 - 9 ÷ = 36 ...(i)
eh 2 h è ø
55. (b) The continuous spectrum depends on the accelerating For the second spectral line in the Balmer series of
voltage. It has a definite minimum wavelength. singly ionised helium ion n2 = 4 and n1 = 2 ; Z = 2
Greater the accelerating voltage for electrons, higher 1 2 é1 1 ù 3R
will be the kinetic energy it attains before striking the \ = R (2) ê - ú = ...(ii)
target, higher will be the frequency of X - rays and l ë 4 16 û 4
smaller will be the wavelength. The wavelength of Dividing equation (i) and equation (ii) we get
continuous X - rays is independent of the atomic l 5R 4 5
number of target material. = ´ =
6561 36 3 R 27
56. (a) Sample S –1 Sample S – 2
Activity 5 mCi 10 mCi \ l = 1215 Å
No. of nuclei N1= 2N N2 = N
E P ´ t 30 ´ 10-3 ´ 100 ´ 10 -9
æ dN ö æ dN ö 59. (b) p= = = = 10 -17 kg ms -1
–ç = l1 N1 –ç = l 2 N2 c c 3 ´ 108
è dt ÷ø 1 è dt ÷ø 2
option (b) is correct
Þ – 5 = l1× 2N ...(i) – 10 = l2 × N ...(ii)
From (i) and (ii)
5 l1 ´ 2N l1 1 1. (b,d) Note : Shortest wavelength means highest frequency.
= Þ = This means highest energy.
10 l2 ´ N l2 4
The energy of X-rays depends on the accelerating
( T1/ 2 )2 1 é 1 ù voltage provided in the X-ray tube.
Þ = ê\ l µ ú
( T1/ 2 )1 4 ë T1/2 û Also, according to Moseley’s law n = a( Z – b) .
57. (a) The energy possessed by photons of wavelength Thus the frequency also depends on the atomic number.
2. (c,d) The threshold wavelength is 5200Å. For ejection of
1240 electrons, the wavelength of the light should be less
550 nm is = 2.25 eV
550 than 5200 Å, so that frequency increases and hence
The energy possessed by photons of wavelength the energy of incident photon increases. U.V light has
1240 less wavelength than 5200 Å.
450 nm is = 2.76 eV 3. (b,c) Nuclear fusion occurs when two or more lighter nuclei
450
combine to form a heavier nucleus with release of a
The energy possessed by photons of wavelength
huge amount of energy.
1240
350 nm is = 3.54 eV 4. (a,c,d) We know, rn µ n 2
350
For metal plate p : -13.6 Z 2
fp = 2 eV. En = eV
All the wavelengths are capable of ejecting electrons. n2
Therefore, the current is maximum. Also as the work nh
function is lowest in p, the kinetic energy of ejected Angular momentum, Ln =
electron will be highest and therefore, the stopping 2p
potential is highest. | P.E. | = 2 × | K.E. |
For metal plate q : 5. (a) A diode can be used as a rectifier.
fq = 2.5 eV. 6. (d) is the correct option. The electrons emitted by emitter
Photons of wavelength 550 nm will not be able to eject are collected to the maximum by the plate in this case.
electrons and therefore, the current is smaller than p. 7. (c) is the correct option.
The work function is greater than q therefore the hc
stopping potential is lower in comparison to p. l min =
eV
P-522 Topic-wise Solved Papers - PHYSICS
8. (c,d) In the case of hydrogen, atomic number = mass number 20. (c) 1 + 1 ® 1H3 + p
H2 H2
In the other atoms, atomic number < mass number. 2 3 4
1H + 1H ® 2He + n
9. (a) Net Reaction 31H ® 2He4 + p + n
2
10. (a,b,c) are correct options. Dm = 3 (2.014) – [4.001 + 1.007 + 1.008] = 0.026
11. (c) is the correct option. 3 deuterons release 3.87 × 10–12 J
12. (d) is the correct option.
13. (c,d) are correct options. 3.87 ´ 10 –12 ´ 1040
14. (b) Note: The intensity of radiation emitted is proportional \ 1040 deuterons release =
3
to the rate of decay which in turn is proportional to
= 1.29 × 1028J
number of atoms left (radioactive).
2.303 N0 E E 1.29 ´ 1028
t= log10 P= Þt = = 16
= 1.29 ´ 1012 sec
l N t P 10
2.303 N0 21. (a,b,c) For metal A
Þt= log10 N / 64 Þ t = 12 hours. 4.25 = WA + TA ...(i)
0.693 / 2 0
15. (b, d) are correct options. 2 2
1 2 1 m vA pA
2
h2
Boron and Aluminium are trivalent impurities. Also TA = mv A = = = ...(ii)
2 2 m 2m 2ml 2A
16. (b) Since the p-n junction arrangement are in series,
therefore the potential drop across a p-n junction will
be proportional to their resistances. When the é hù
êQ l = p ú
resistances will be equal, the potential drops will be ë û
equal. In circuit I, the two p-n junctions are attached
For metal B
such that one is forward biased (low resistance) and
4.7 = (TA – 1.5) + WB ...(iii)
other is reverse biased (high resistance). Whereas in
the other two circuits both are either forward biased or h2
reversed biased. Also TB = ...(iv) [as eq. (ii)]
2ml 2B
ln 2
17. (b) T1/ 2 = and Dividing equation (iv) by (ii),
l
1 TB h2 2ml 2A l 2A
Mean life, t = = ´ =
l TA 2ml 2B h2 l 2B
18. (b, d) Since the stopping potential depends on the frequency
and not on the intensity and the source is same, the TA –1.5 l 2A l2 1
stopping potential remains unaffected. The saturation Þ = = A =
current depends on the intensity of incident light on TA ( 2l A ) 4l A 4
2 2

the cathode of the photocell which in turn depends on


the distance of the source from cathode. The intensity [Q l B = 2l A given]
(I) of light is inversely proportional to the square of the Þ 4TA – 6 =TA Þ TA = 2 eV
distance between the light source and photocell. From (i), WA = 2.25 eV
1 From (iii), WB = 4.2 eV
Iµ and saturation current µ I Also TB = TA – 1.5 Þ TB = 0.5eV
r2 22. (a,d) are correct options.
1 23. (a,c) Holes are electron vacancies which participate in
Þ Saturation Current µ
r2 electrical conductivity. These are produced in
semiconductors.
(Saturation Current)final r2 24. (a,c) The circuit for a p-n-p transistor used in the common
Þ = Initial emitter mode as an amplifier is shown in figure. The
(Saturation Current)initial 2
rfinal
base emitter junction is forward-biased and the input
0.2 ´ 0.2 signal is connected in series with the voltage applied
Þ (Saturation Current)final = ´ 18 = 2mA to bias the base emitter junction.
0.6 ´ 0.6
19. (b,c) Ic = 10 mA
90% of electrons emitted produce a collector current of C
10 mA. The base current RB B RC Output
10 Input N
Ib = 10% of Ic = ´ 10 = 1mA
100 E
Now, Ie = Ib + Ic = 1 + 10 = 11mA
MODERN PHYSICS P-523
25. (c, d) KEY CONCEPT : Due to mass defect (which is finally All wavelengths > lmin are found.
responsible for the binding energy of the nucleus),
12400
mass of a nucleus is always less than the sum of masses where lmin = V (in volts) Å
of its constituent particles.
20 Here V is the applied voltage.
10 Ne is made up of 10 protons plus 10 neutrons.
–34
20 hc 6.63 ´ 10 ´ 3 ´ 108
Therefore, mass of 10 Ne nucleus 30. (c) lmin = = = 310 × 10–9m.
M1 < 10 (mp + mn) W 4(1.6 ´ 10 –19 )
Note : = 310 nm
Heavier the nucleus, more is the mass defect.
0.693 1
20 (mn + mp) – M2 > 10 (mp + mn) – M1 31. (c) (t1/2)x = (tmean)Y Þ =
Thus, 10 (mn + mp) > M2 – M1 lx lY
or M2 < M1 + 10 (mp + mn) \ lx = 0.693 lY
Now, since M1 < 10 (mp + mn) lx < lY. Now, rate of decay = lN
\ M2 < 2M1 Initially, number of atoms (N) of both are equal but
26. (a, d) The time period of the electron in a Bohr orbit is given since lY < lx, therefore Y will decay at a faster rate
2 pr than x.
by T =
v 32. (a, c) f1 : f2 : f3 = eV01 : eV02 : eV03
Since for the nth Bohr orbit, mvr = n (h/2p), the time
period becomes V01 : V02 : V03 = 0.001: 0.002 : 0.004 = 1: 2 : 4

æ 4p 2 m ö 2 Therefore option (a) is correct


2pr
T= =ç ÷r V0
nh /(2pmr ) è nh ø
Since the radius of the orbit r depends on n, we replace Metal 1 Metal 2 Metal 3
r. Bohr radius of a hydrogen atom is f1 f2 f3
æ 2 ö
2 h e0 q q q –1
r= n ç 1/l (nm )
2÷ 0.001 0.002 0.004
è pme ø
hc
æ 4p 2 m ö æ n4 h 4 e 20 ö æ 3 2ö
3 4h e 0 By Einstein’s photoelectric equation, – f = eV
Hence, T = ç ÷ç 2 2 4÷ = n ç l
4 ÷
è nh ø è p m e ø è me ø
hc f
ÞV= – ...(i)
3 el e
T1 æ n1 ö
For two orbits, = Comparing equation (i) by y = mx + c, we get the slope
T2 çè n2 ÷ø
hc
It is given that T1 / T2 = 8, hence, n1/n2 = 2. of the line m = = tan q
27. (d) The result follows from the formula based on laws of e
radioactive decay N = N0e–lt Þ Option (c) is correct.
The nucleus start decaying after time t = 0 From the graph it is clear that,
28. (c) At junction a potential barrier/depletion layer is formed 1
= 0.001( nm) Þ l = 1 = 1000mn
–1
as shown, with n-side at higher potential and p-side at
l 01 01
0.001
lower potential. Therefore, there is an electric field at
the junction directed from the n-side to p-side
1
= 0.002 ( nm ) Þ l 02 = 500nm
–1
p E Also
n l 02
– +
– +
and l 03 = 250nm

– + Note : Violet colour light will have wavelength less


than 400 nm.
– + Therefore, this light will be unable to show photoelectric
29. (b) The continuous X-ray spectrum is shown in figure. effect on plate 3 Þ Option (d) is wrong.
33. (b,d) Note : When binding energy per nucleon increases for
E a nuclear process, energy is released.
• When two nuclei of mass numbers between 51 to 100
fuse, the mass number of the resulting nuclei will come
out to be between 100 to 200. The graph shows that in
this process the binding energy per nucleon increases
l
l min and therefore energy is released.
P-524 Topic-wise Solved Papers - PHYSICS
• When nucleus of mass number 200 to 260 breaks; it
hc
will produce nuclei of mass numbers lying between But E =
100 to 200 if we assume that the two daughter nuclei l
are of nearly same mass. This in fact happens practically hc 6.6 ´ 10 –34 ´ 3 ´ 108
that when a heavy nucleus splits into two parts during \ l= = = 3.65 ´ 10 -19 m
E –19
nuclear fission, two moderate size nuclei are formed in 340 ´ 10 ´ 1.6
general. The graph shows that in this process also the (iv) Total Energy of 1st orbit = – 340 eV
binding energy per nucleon increases. Therefore energy We know that – (T.E). = K.E. [in case of electron revolving
is released. around nucleus]
and 2T.E. = P.E.
nh 3h
34. (a, c)Angular momentum = = . Therefore n = 3. \ K.E. = 340 eV ; P.E. = – 680 eV
2p 2p KEY CONCEPT :
ao n2 Angular momentum in 1st orbit :
Also rn = = 4.5a o According to Bohr’s postulate,
z
nh
n2 9 mvr =
\ = 4.5 Þ = 4.5 Þ z = 2 2p
z z For n = 1,
we know that
h 6.6 ´ 10–34
1 é1 1 ù mvr = = = 1.05 × 10–34J-s.
= R z2 ê 2 - 2 ú 2p 2p
l ëê n1 n 2 ûú (v) Radius of first Bohr orbit

1 é1 1 ù 5.3 ´ 10-11 5.3 ´ 10 -11


= 4R ê 2 - 2 ú r1 = =
\ l Z 5
êë n1 n 2 úû
= 1.06 × 10–11 m
9 9 12400
For n2 = 3, n1 = 1 we get l = = E=
12400
8 ´ 4R 32R 2. eV = = 12.75 eV ...(i)
l(inÅ) 975
36 9 Also
For n2 = 3, n1 = 2 we get l = =
5 ´ 4R 5R é1 1ù é ù
13.6 ê 2 – 2 ú = 12.75 Þ ê1 – 1 ú = 12.75 Þ n = 4
4 1 êë n1 n2 úû 2 2
For n2 = 2, n1 = 1 we get l = = ëê1 n2 ûú 13.6
3´ 4R 3R For every possible transition one downward arrow is shown
(a), (c) are correct options therefore the possibilities are 6.
SUBJECTIVE PROBLEMS : n=4
n=3
Increasing

n=2
Energy

13.6 2 13.6 2
1. (i) E2 = – Z , E3 = – Z
4 9 n=1
æ 1 1ö 13.6 ´ 5 2
E3 – E2 = –13.6 Z 2 ç – ÷ = + Z Note : For longest wavelength, the frequency should be
è 9 4ø 36
smallest.
But E3 – E2 = 47.2 eV (Given) This corresponds to the transition from n = 4 to n = 3, the
13.6 ´ 5 2 47.2 ´ 36 13.6 13.6
\ Z = 47.2 \ Z= =5 energy will be E4 = – ; E3 = -
36 13.6 ´ 5 4 2
32
–13.6 2
(ii) E4 = Z –13.6 æ -13.6 ö é1 1 ù
16 \ E4 – E3 = –ç = 13.6 ê – ú
2 è 2 ÷ø ë 9 16 û
4 3
2é1 1ù 2 é 9 –16 ù = 0.66 eV = 0.66 × 1.6 × 10–19J= 1.056 × 10–19J
\ E4 – E3 = –13.6 Z ê – ú = –13.6Z ê
ë16 9 û ë 9 ´ 16 úû
12400
Now, E = eV \ l = 18787 Å
+13.6 ´ 25 ´ 7 l (inÅ)
= = 16.53eV
9 ´ 16 3. (i) In a nucleus, number of electrons = 0 ( Q electrons don't
13.6 reside in the nucleus of atom).
(iii) E1 = – ´ 25 = –340 eV (ii) number of protons = 11
1
(iii) number of neutrons = 24 – 11 = 13
\ E = E ¥ – E1 = 340 eV = 340 × 1.6 × 10–19 J [E ¥ = 0 eV]
MODERN PHYSICS P-525
238 234 4 234 8. K.E. = 0.0327 eV = 0.0327 × 1.6 × 10–19J
4. 92 U ¾¾ ® 90 X + 2 He
234
® 91 Y + –10 e
90 X ¾¾ 1
(i) Atomic number = 91 mn vn2 = 0.0327 ´ 1.6 ´ 10 –19
(ii) Mass number = 234 2
1/ 2
hc hc é 2 ´ 0.0327 ´ 1.6 ´ 10 –19 ù
5. – = K.E.1 ...(i) Þ vn = ê ú
l1 l 0
ëê 1.675 ´ 10 –27 ûú
hc hc Þ vn = 0.25 × 104 m/s
and – = K.E.2 ...(ii)
l 2 l0 Time taken by the neutron to travel 10 m will be
hc hc d 10
Þ – = K.E.1 – K.E2 t= = = 4 ´ 10 –3 s
l1 l 2 vn 0.25 ´ 104
é l – l1 ù Let the number of neutron initially be a.
Þ hc ê 2 ú = K.E.1 – K.E2
ë l1l 2 û l=
0.693 0.693 –1
= s
(K.E.1 – K.E.2 ) l1l 2 t1/ 2 700
\h = We know that
c(l 2 – l1 )
2.303 a
(1.8 – 4) ´ 1.6 ´ 10 –19 ´ 800 ´ 10 –10 ´ 700 ´ 10 –10 t= log
= l a–x
3 ´ 108 ´ (700 – 800) ´ 10 –10
= 6.6 × 10–34 J.s. 4 ´ 10–3 0.693 a
Þ ´ = log10
I.E. 2.303 700 a –x
6. (i) En = – for Bohr’s hydrogen atom.
n2 a a
Þ log10 = 1.72 × 10–6 Þ = 1.000004
a–x a–x
– 4R
Here, I.E. = 4R \ En = x
n2 Þ = 3.96 ´ 10 –6
a
–4 R æ 4R ö 9. I = 0.125 V – 7.5
\ E2 – E1 = –ç– = 3R ..(i)
2 2 è 12 ÷ø dV 1
Þ dI = 0.125 dV or = =8
hc dI 0.125
E2 – E1 = hn = ...(ii)
l dV
From (i) and (ii) We know that plate resistance, rp = = 8mW
dI
hc
= 3R é dI ù
l The transconductance, gm = ê ú
êë dVg úûV = constt
hc 6.6 ´ 10 –34 ´ 3 ´ 108
\ l= = = 300Å At Vg = – 1 volt, V = 300 volt, the plate current
3R 2.2 ´ 10 –18 ´ 3
I = [0.125 × 300 – 7.5] mA = 30 mA
(ii) The radius of the first orbit
Also it is given that Vg = – 3V, V = 300 V and I = 5mA
Bohr’s radius of hydrogen atom = 5 × 10–11 m (given)
| En| = + 0.22 × 10–17Z2 = 4R = 4 × 2.2 × 10–18 é 30 – 5 ù 25
\Z =2 \ gm = ê ú = ´ 10 –3 = 12.5 × 10–3s
ë –1 – (–3) û 2
r0 5 ´ 10–11 5 ´ 10 –11 The characteristics are given in the form of parallel lines.
\ rn = = = = 2.5 × 10–11m Amplification factor
Z Z 2
= rp gm = 8 × 103 × 12.5 × 10–3 = 100
13.6 10. (i) Let m be the mass of electron. Then the mass of mu-
7. (i) En = – 2
Z 2 eV/atom
n meson is 208 m. According to Bohr’s postulate, the
angular momentum of mu-meson should be an integral
–13.6 ´ 9 multiple of h/2p.
For Li2+, Z = 3 \ En = eV/atom
n2 v e
13.6 ´ 9 13.6 ´ 9
\ E1 = – and E3 = – = –13.6
1 9 r
DE = E3 – E1 = – 13.6 – (–13.6 × 9)
= 13.6 × 8 = 108.8 eV/atom +3e
12400 12400
l= Å = = 114Å
E (in eV) 108.8
(ii) The spectral line observed will be three namely 3 ® 1, nh
3 ® 2, 2 ® 1. \ (208m) vr =
2p
P-526 Topic-wise Solved Papers - PHYSICS
nh 12. (i) The transition state of six different photon energies are
nh
\ v= = ...(i) shown.
2p ´ 208mr 416 pmr
Note: Since mu-meson is moving in a circular path, n = 4 [– 0.85 eV]

Increasing Energy
therefore, it needs centripetal force which is provided [0.65 eV]
by the electrostatic force between the nucleus and mu- n = 3 [– 1.5 eV]
meson. [+ 1.9 eV]
n = 2 [– 3.4 eV]
(208m)v 2 1 3e ´ e
\ = ´ [10.2 eV]
r 4pe 0 r2
n = 1 [– 13.6 eV]
3e 2
\ r= Since after absorbing monochromatic light, some of the
4pe 0 ´ 208mv 2 emitted photons have energy more and some have less than
Substituting the value of v from (1), we get 2.7 eV, this indicates that the excited level B is n = 2. (This is
because if n = 3 is the excited level then energy less than 2.7
3e 2 ´ 416pmr ´ 416pmr eV is not possible).
r=
4pe 0 ´ 208mn 2 h2 (ii) For hydrogen like atoms we have
–13.6
n2 h2 e 0 En = 2
Z 2 eV/atom
Þ r= ....(i) n
624pme 2
(ii) The radius of the first orbit of the hydrogen atom –13.6 2 æ –13.6 ö 2
E4 – E2 = Z –ç Z = 2.7
16 è 4 ÷ø
e 0 h2
= ...(ii)
pme 2 é1 1 ù
Þ Z 2 ´ 13.6 ê – ú = 2.7
To find the value of n for which the radius of the orbit ë 4 16 û
is approximately the same as that of the first Bohr orbit
for hydrogen atom, we equate eq. (i) and (ii) 2.7 4 ´ 16 æ1 1 ö
Þ Z2 = ´ Þ I.E. = 13.6 Z 2 ç 2 – 2 ÷
13.6 12 è1 ¥ ø
n2 h2 e 0 e 0 h2
= Þ n = 624 » 25
624pme 2 pme 2 2.7 4 ´ 16
= 13.6 ´ ´ = 14.46 eV
13.6 12
1 é1 1ù (iii) Max. Energy
(iii) = 208R ´ Z 2 ê 2 – 2 ú
l êë n1 n2 úû æ 1 1ö
E4 – E3 = –13.6 Z 2 ç 2 – 2 ÷
1 é1 1ù è4 1 ø
Þ = 208 ´ 1.097 ´ 10 7 ´ 32 ê – ú
l ë12 32 û 2.7 4 ´ 16 15
Þ l = 5.478 × 10–11m = 13.6 ´ ´ ´ = 13.5 eV
13.6 12 16
12400 12400 Min. Energy
11. E1 = = 2.99 eV, E2 = = 2.49 eV,,
4144 4972 æ 1 1ö
E4 – E3 = –13.6 Z 2 ç 2 – 2 ÷
12400 è4 3 ø
E3 = = 1.99 eV
6216
Þ Only first two wavelengths are capable of ejecting 2.7 4 ´ 16 7
= 13.6 ´ ´ ´ = 0.7eV
photoelectrons. 13.6 12 9 ´ 16
Energy incident per second 13. For hydrogen like atom energy of the nth orbit is
3.6 13.6
= ´ 10 –3 ´ 10 –4 = 1.2 ´ 10–7 J/s En = – Z 2 eV/atom
3 n 2

1.2 ´ 10 –7 For transition from n = 5 to n = 4,


\ n1 = = 2.5 ´ 1011
2.99 ´ 1.6 ´ 10 –19 é 1 1 ù 13.6 ´ 9 ´ 9
hn = 13.6 ´ 9 ê – ú = = 2.754 eV
ë16 25 û 16 ´ 25
1.2 ´ 10 –7
n2 = –19
= 3 ´ 1011 For transition from n = 4 to n = 3,
2.99 ´ 1.6 ´ 10
Total number of photons = 2(n1 + n2) é 1 1 ù 13.6 ´ 9 ´ 7
hn' = 13.6 ´ 9 ê – ú = = 5.95eV
= 3.01 × 1011 + 2.51 × 1011 = 5.52 × 1011 ë 9 16 û 9 ´ 16
\ Total number of photoelectrons ejected in two seconds For transition n = 4 to n = 3, the frequency is high and hence
= 11 × 1011. wavelength is short.
MODERN PHYSICS P-527
For photoelectric effect, hn' – W = eV0, where W = work \ Total number of photons incident per unit time on the
function aperture,
5.95 × 1.6 × 10–19 – W = 1.6 × 10–19 × 3.95
Þ W = 2 × 1.6 × 10–19 = 2 eV n3 = n2 s1 = (1.34 ´ 1018 ) (7.85 ´ 10 -3 ) / s
Again applying hn – W = eV'0 = 1.052 × 1016 / s
We get, 2.754 × 1.6 × 10–19 – 2 × 1.6 × 10–19 = 1.6 × 10–19 V'0 The aperture will become new source of light.
Þ V'0 = 0.754 V Now these photons are further distributed in all directions.
14. Energy required per day Hence, at the location of detector, photons incident per unit
E = P × t = 200 × 106 × 24 × 60 × 60 area per unit time :
= 1.728 × 1013J
n3 1.052 ´ 1016
Energy released per fusion reaction n4 = =
= [2 (2.0141) – 4.0026] × 931.5 MeV 4 p(6 - 0.6) 2 4p(5.4) 2
= 23.85 MeV = 23.85 × 106 × 1.6 × 10–19 = 2.87 × 1013 s–1 m–2
= 38.15 × 10–13J This is the photon flux at the centre of the screen. Area of
\ No. of fusion reactions required detector is 0.5 cm2 or 0.5 × 10–4 m2. Therefore, total number
of photons incident on the detector per unit time :
1.728 ´ 1013
= = 0.045 × 1026
38.15 ´ 10–13 n5 = (0.5 ´ 10-4 ) (2.87 ´ 1013 d ) = 1.435 ´ 109 s -1
\ No. of deuterium atoms required The efficiency of photoelectron generation is 0.9. Hence,
= 2 × 0.045 × 1026 = 0.09 × 1026 total photoelectrons generated per unit time :
Number of moles of deuterium atoms n6 = 0.9n5 = 1.2915 ´ 109 s -1
0.09 ´ 1026 or, photocurrent in the detector :
= = 14.95
6.02 ´ 1023 i = (e)n6 = (1.6 ´ 10 -19 ) (1.2915 ´ 109 ) = 2.07 ´ 10 -10 A
\ Mass in gram of deuterium atoms (b) Using the lens formula :
= 14.95 × 2 = 29.9 g
1 1 1
But the efficiency is 25%. - = or v = – 0.3 m
Therefore, the actual mass required = 119.6 g v -0.6 -0.6
i.e., image of source (say S', is formed at 0.3 m from the lens.)
hc (6.6 ´ 10-34 ) (3.0 ´ 108 )
15. Energy of one photon, E = =
l 6000 ´ 10 -10
= 3.3 × 10–19 J

S S'
L D
0.6 m 5.4 m
0.3 m
S A D

5.7 m
Total number of photons incident per unit time on the lens
are still n3 or 1.052 × 1016/s. 80% of it transmits to second
medium. Therefore, at a distance of 5.7 m from S' number of
Power of the source is 2 W or 2 J/s. Therefore, number of photons incident per unit are per unit time will be :
photons emitting per second,
(80 /100) (1.05 ´ 1016 )
n1 =
2
= 6.06 ´ 1018 / s n7 = = 2.06 ´ 1013 s -1m -2
2
3.3 ´ 10 -19 (4 p) (5.7)
At distance 0.6 m, number of photons incident per unit area This is the photon flux at the detector.
per unit time : New value of photocurrent is :
n1 i = (2.06 ´ 1013 ) (0.5 ´ 10 -4 ) (0.9) (1.6 ´ 10 -19 )
n2 = = 1.34 ´ 1018 / m 2 / s
4 p (0.6) 2 = 1.483 × 10–10 A
Area of aperture is, (c) For stopping potential

p 2 p hc
S1 = d = (0.1) 2 = 7.85 ´ 10-3 m 2 = ( EK )max + W = eV0 + W
4 4 l
P-528 Topic-wise Solved Papers - PHYSICS
By hit and trial we get n2 = 4 and n1 = 2
hc 3.315 ´ 10 –19
\ eV0 = –W = –1 = 1.07eV nh
l 1.6 ´ 10 –19 [angular momentum mvr = ]
\ V0 = 1.07 Volt 2p
Note : The value of stopping potential is not affected by the (c) Change in angular momentum
presence of concave lens as it changes the intensity and nh n h h h h
not the frequency of photons. The stopping potential = 1 – 2 = (2 – 4) = ´ ( -2) = -
2p 2p 2p 2p p
depends on the frequency of photons.
(d) The momentum of emitted photon can be found by de
A
16. (a) 92X ® 228 4
XY + 2He Broglie relationship
A = 228 + 4 = 232; 92 = Z + 2 Þ Z = 90
(b) Let v be the velocity with which a - particle is emitted. h h hn E 2.55 ´ 1.6 ´ 10 –19
l= Þ p= = = \ p=
Then p l c c 3 ´ 108
qrB 2 ´ 1.6 ´ 10 –19 ´ 0.11 ´ 3 Note : The atom was initially at rest the recoil momentum of
mv 2
= qvB Þ v = = the atom will be same as emitted photon (according to the
r m 4.003 ´ 10–27 conservation of angular momentum).
Þ v = 1.59 × 107ms–1. Let m be the mass and v be the recoil velocity of hydrogen
Applying law of conservation of linear momentum during atom then
a-decay we get
mY vY = mava ...(1) 2.55 ´ 1.6 ´ 10 –19
The total kinetic energy of a-particle and Y is m´v =
3 ´ 108
1 1
E = K.E.a + K.E.Y = ma va 2 + mY vY 2
2 2 2.55 ´ 1.6 ´ 10 –19
Þv = = 0.814 m/s
2 3 ´ 108 ´ 1.67 ´ 10 –27
1 1 ém v ù 1 ma2 va2
= ma va2 + mY ê a a ú = ma va 2 + ma va2 +
2 2 ë mY û 2 2mY K2
m
1 é ma ù y
= ma va2 ê1 + ú
2 ëê mY ûú 18. q x
1 é 4.003 ù m 4m
= ´ 4.033 ´ 1.6 ´ 10 –27 ´ (1.59 ´ 107 )2 ê1 + J 4m
2 ë 228.03 úû K1
= 8.55 × 10–13 J
= 5.34 MeV Applying conservation of linear momentum in horizontal
\ Mass equivalent of this energy direction
5.34 (Initial Momentum)x = (Final Momentum)x
= = 0.0051 a.m.u. (Pi)x = (Pf)x
931.5
Also, mx = mY + ma + mass equivalent of energy (E) Þ 2 Km = 2(4m) K1 cos q ...(i)
= 228.03 + 4.003 + 0.0057 = 232.0387 u. Now applying conservation of linear momentum in Y-
The number of nucleus = 92 protons + 140 neutron.
direction
\ Binding energy of nucleus X
(Pi)y = (Pf)y
= [92 × 1.008 + 140 × 1.009] – 232.0387 = 1.9571 u
= 1.9571 × 931.5 = 1823 MeV. 0 = 2 K2 m – 2(4 m) K1 sin q
17. (a) The energy of photon causing photoelectric emission
= Work function of sodium metal + KE of the fastest Þ 2 K2 m = 2(4m) K1 sin q ...(ii)
photoelectron Squaring and adding (i) and (ii),
= 1.82 + 0.73 = 2.55 eV 2Km + 2K2m = 2 (4m)K1 + 2 (4m)K1
–13.6 eV K1 + K2 = 4K1 Þ K = 4K1 – K2 Þ 4K1 – K2 = 65 ...(iii)
(b) We know that En = for hydrogen atom.
n 2 atom When collision takes place, the electron gains energy and
Let electron jump from n2 to n1 then jumps to higher orbit.
Applying energy conservation
–13.6 æ –13.6 ö K = K1 + K2 + DE
En2 – En1 = –ç 2 ÷ Þ 65 = K1 + K2 + DE ...(iv)
n22 è n1 ø
Possible value of DE for He+
æ 1 Case (1)

Þ 2.55 = 13.6 ç 2 – 2 ÷ DE1 = – 13.6 – (–54.4) = 40.8 eV
è n1 n2 ø Þ K1 + K2 = 24.2 eV from (4)
MODERN PHYSICS P-529
Solving with (3), we get
13.6 2 æ 13.6 2 ö
K2 = 6.36 eV ; K1 = 17.84 eV But E3 – E2 = – Z –ç– Z ÷
Case (2) 9 è 4 ø
DE2 = – 6.04 – (– 54.4) = 48.36 eV
é1 1ù
Þ K1 + K2 = 16.64 eV from (4) = –13.6Z 2 ê – ú
Solving with (3), we get ë9 4û
K2 = 0.312 eV; K1 = 16.328 eV
é 4 – 9 ù 13.6 ´ 5 2
= –13.6 Z 2 ê = Z
-3.4 eV n=4 ë 36 úû 36
-6.04 eV n=3
13.6 ´ 5 2
-13.6 eV n=2 \ Z = 17 Þ Z = 3
36

-54.4 eV n=1 13.6 2 é 13.6 2 ù


For He+ En – E2 = – ´ 3 – ê– ´3 ú
n2 ë 22 û
Case (3)
DE3 = – 3.4 – (–54.4) = 51.1eV é 9 9ù é 4 – n2 ù
Þ K1 + K2 = 14 eV = –13.6 ê 2 – ú = –13.6 ´ 9 ê 2
ú ...(ii)
ën 4û ëê 4n ûú
Solving with (3), we get
K2 = 15.8 eV ; K1 = – 1.8 eV From eq. (i) and (ii),
But K.E. can never be negative therefore case (3) is not
possible. é 4 – n2 ù
–13.6 ´ 9 ê ú = 27.2
Therefore, the allowed values of kinetic energies are only 2
êë 4n úû
that of case (1) and case (2) and electron can jump upto
n = 3 only. Þ – 122.4 (4 – n2) = 108.8n2
(ii) Thus when electron jumps back there are three 489.6
possibilities Þ n2 = = 36 Þ n = 6
13.6
n3 ® n1 or n3 ® n2 and n2 ® n1 21. (i) En = – 3.4 eV
The frequencies will be The kinetic energy is equal to the magnitude of total energy
E3 – E2 E3 – E1 E – E1 in this case.
n1 = ; n2 = ; n3 = 2 \ K.E. = + 3.4 eV
h h h
i.e., 1.82 × 1015 Hz ; 11.67 × 1015 Hz; 9.84 × 1015 Hz (ii) The de Broglie wavelength of electron
19. t1/2 = 15 hours
h 6.64 ´ 10 –34
Activity initially A0 = 10–6 Curie (in small quantity of solution l= = eV
of 24Na) = 3.7 × 104dps 2mK 2 ´ 9.1 ´ 10 –31 ´ 3.4 ´ 1.6 ´ 10 –19
Observation of blood of volume 1 cm3 = 0.66 × 10–9 m
After 5 hours, A = 296 dpm 22. (i) From the given information, it is clear that half life of
The initial activity can be found by the formula the radioactive nuclei is 10 sec (since half the amount is
2.303 A0 2.303 A consumed in 10 second. 12.5% is half of 25% pls. note).
t= log10 Þ5= ´ log10 0 Mean life
l A 0.693/15 296
1 1 t 10
A0 5 ´ 0.693 0.3010 t= = = 1/ 2 =
Þ log10 = = = 0.10033 l 0.693/ t1/ 2 0.693 0.693 = 14.43 sec.
296 2.303 ´ 15 3
A0 373 (ii) N = N 0 e -lt
Þ = 1.26 Þ A0 = 373 dpm = dps
296 60 N 6.25
This is the activity level in 1 cm3. Comparing it with the =
initial activity level of 3.7 × 104 dps we find the volume of N0 100
blood.
l = 0.0693s -1
4
3.7 ´ 10
V= = 5951.7 cm3 = 5.95l litre 6.25
373 / 60 = e -0.0693t
100
20. For hydrogen like atoms
100
13.6 e +0.0693t = = 16
En = – Z 2 eV/atom 6.25
n2 0.0693t = ln 16 = 2.773
Given En – E2 = 10.2 + 17 = 27.2 eV ..(i)
En – E3 = 4.24 + 5.95 = 10.2 eV 2.733
or t= = 40 sec.
\ E3 – E2 = 17 0.0693
P-530 Topic-wise Solved Papers - PHYSICS
238 Rate of decay at the moment when number of nuclei are
Number of atoms of U initially 4 a 1020 is
23. = =
238 3 (a – x)
Number of atoms of U finally dN
[ Q Initially one part lead is present with three parts Uranium] = lN = (10 –13 )(1020 ) = 107 dps
dt
a 2.303 ´ 4.5 ´ 109 4 Of these distintegrations, 8% are in fission and 92% are in
2.303
\t= log = log a-decay.
l ( a – x) 0.693 3
Therefore, energy released per second
= 1.868 × 109years. = (0.08 × 107 × 200 + 0.92 × 107 × 5.136) MeV
= 2.074 × 108 MeV
24. As nodes are formed at each of the atomic sites, hence \ Power output (in watt) = Energy released per second (J/s)
æ lö = (2.074 × 108) (1.6 × 10–13)
2Å = n ç ÷ ...(1) \ Power output = 3.32 × 10–5 watt.
è 2ø
[ Q Distance between successive nodes = l/2] A
o 26. Rate of production = a Rate of decay = lN
2A

N N N N N
n loops

N N N N N N
o (n+1) loops
2.5A t = time
N = Number of radioactive nuclei
l
and 2.5 Å = (n + 1) (a) Let at time ‘t’ number of radioactive nuclei are N.
2
Net rate of formation of nuclei of A.
2.5 n + 1 5 n + 1
\ = , = or n = 4 dN dN
2 n 4 n = a – l N or = dt
Hence, from equation (1), dt a – lN
t
l N dN
i.e., l = 1Å
2Å = 4
2
or ò N0 a – lN = ò dt
0
Now, de broglie wavelength is given by
Solving this equation, we get
l=
h h2 1é
or K = N= a - (a - lN 0 )e -lt ù
2mK l 2 .2m ........(1)
lë û

(6.63 ´ 10 –34 ) 2 (b) Substituting a = 2lN0 and


\ K= eV
–10 2 –31 –19
(1 ´ 10 ) ´ 2 ´ 9.1 ´ 10 ´ 1.6 ´ 10 ln(2)
t = t1/ 2 = in equation (1),
l
(6.63) 2
= ´ 102 eV = 151 eV 3
8 ´ 9.1 ´ 1.6 we get, N = N0
d will be minimum, when 2
(ii) Substituting a = 2l N0 and t ® ¥ in equation (1), we get
l 1Å
n = 1, dmin = = = 0.5Å a
2 2 N= = 2 N0 .
25. The reaction involved in a-decay is l
27. The energy of the incident photon is
248 244 4
96 Cm ® 94 Pu + 2 He
Mass defect hc (4.14 ´ 10 –15 eVs)(3 ´ 108 m / s)
E1 = = = 3.1eV
248 244 4
l (400 ´ 10 –9 m)
Dm = Mass of 96 Cm – Mass of 94 Pu – Mass of 2 He
The maximum kinetic energy of the emitted electrons is
= (248.072220 – 244.064100 – 4.002603) u
Emax = E1 – W = 3.1 eV – 1.9 eV = 1.2eV
= 0.005517u
It is given that,
Therefore, energy released in a-decay will be
Ea = (0.005517 × 931) MeV = 5.136 MeV æ Emitted electrons ö 2+
Similarly, Efission = 200 MeV (given) çè of maximum energy÷ø + 2 He ¾¾® He+
in 4th excited state
1 + photon
Mean life is given as tmean = 1013 s =
l The fourth excited state implies that the electron enters in
\ Disintegration constant l = 10–13 s–1 the n =5 state.
MODERN PHYSICS P-531
In this state its energy is For minimum energy = Transition from 4 to 3.
(13.6eV) Z 2 (13.6eV)(2)2 æ 1 1ö æ 7 ö
E5 = – =– E = 13.6 ´ 4 2 ç – ÷ = 13.6 ´ 42 ç
n2 52 è32
4 ø
2 è 9 ´ 16 ÷ø
= – 2.18 eV. = 10.5 eV.
The energy of the emitted photon in the above combination Hence n = 2, Z = 4, Emin = 10.5 eV
reaction is 29. No. of photons/sec
E = Emax + (– E5) = 1.2 eV + 2.18 eV = 3.4 eV Energy incident on platinum surface per sec ond
Note : After the recombination reaction, the electron may =
Energy of one photon
undergo transition from a higher level to a lower level
thereby emitting photons. No. of photons incident per second
The energies in the electronic levels of He+ are
2 ´ 10 ´ 10-4
= = 1.18 ´ 1014
=
( –13.6eV ) (2 2
)
= –3.4 eV 10.6 ´ 1.6 ´ 10 –19
E4
42 As 0.53% of incident photon can eject photoelectrons
\ No. of photoelectrons ejected per second
( –13.6eV) (22 ) = –6.04 eV
E3 = = 1.18 ´ 1014 ´
0.53
= 6.25 ´ 1011
32 100
( –13.6eV) (22 ) = –13.6 eV Minimum energy = 0 eV,
E2 = Maximum energy = (10.6 – 5.6) eV = 5 eV
22 30. The formula for h of power will be
The possible transitions are P
n=5®n=4 h = out
DE = E5 – E4 = [– 2.18 – (– 3.4)] eV = 1.28 eV Pin
n=5®n=3
Pout 1000 ´ 106
DE = E5– E3 = [– 2.18 – (– 6.04)] eV = 3.84 eV \ Pin = = = 1010 W
n=5®n=2 h 0.1
DE = E5 – E2 = [– 2.18 – (– 13.6)] eV = 11.4 eV Energy required for this power is given by
n=4®n=3 E =P×t
DE = E4 – E3 = [– 3.4 – (– 6.04)] eV = 2.64 eV = 1010 × 86,400 × 365 × 10
Hence, the photons that are likely to be emitted in the range = 3.1536 × 1018 J
of 2 eV to 4 eV are 3.4 eV, 3.84 eV and 2.64 eV. 200 × 1.6 × 10–13 J of energy is released by 1 fission
28. Energy for an orbit of hydrogen like atoms is \ 3.1536 × 1018 J of energy is released by

13.6 Z 2 3.1536 ´ 1018


En = – fission
n2 200 ´ 1.6 ´ 10–13
For transition from 2n orbit to 1 orbit = 0.9855 × 1029 fission
= 0.985 × 1029 of U235 atoms.
æ
2 1 1 ö 6.023 × 1023 atoms of Uranium has mass 235g
Maximum energy = 13.6Z ç – ÷
è 1 (2n)2 ø \ 0.9855 × 1029 atoms of Uranium has

æ1 1 ö 235 ´ 0.9855 ´ 1029


Þ 204 = 13.6 Z 2 ç – 2 ÷ ...(i) g = 38451 kg
è 1 4n ø 6.023 ´ 1023
Also for transition 2n ® n. 31. Let the reaction be
A A–4 4
æ 1
2 1 ö æ 3 ö Z X ® Z –2Y + 2 He
40.8 = 13.6 Z ç – ÷ Þ 40.8 = 13.6 Z 2 ç
èn 2
4n ø
2 è 4 n 2 ÷ø Here, my = 223.61 amu and ma = 4.002 amu
We know that
Z2
Þ 40.8 = 40.8 Þ 4n2 = Z2 or 2n = Z ....(ii) h h2
4n 2 l= Þ m2 v 2 = 2 = p 2
mv l
From (i) and (ii)
2æ 1 ö p2 h2
204 = 13.6 Z ç1 – = 13.6 Z 2 – 13.6 Þ But K.E. = . Therefore K.E. = ...(i)
è Z 2 ÷ø 2m 2ml 2
13.6Z 2 = 204 + 13.6 = 217.6 Applying eq. (i) for Y and a, we get
217.6 Z 4 (6.6 ´ 10 –34 )2
Z2 = = 16, Z = 4, n = = = 2 K.E.a =
13.6 2 2 2 ´ 4.002 ´ 1.67 ´ 10 –27 ´ 5.76 ´ 10 –15 ´ 5.76 ´ 10 –15
orbit no. = 2n = 4 = 0.0982243 × 10–11 = 0.982 × 10–12J
P-532 Topic-wise Solved Papers - PHYSICS
Similarly (K.E.)Y = 0.0178 × 10–12 J –1.5
Total energy = 10–12 J \ N x = N 0 e –0.1(15 log e 3) = N 0 elog e (3 )

We know that E = Dmc2


1020
E 10 –12 Þ N x = N0 3–1.5 =
\ Dm = = kg 3 3
c2
(3 ´ 108 )2
1.65 × 10–27kg = 1 amu dN y l x N x 1020
Since, = 0 at t = 15 loge 3, \ N y = =
dt ly 3
10 –12 10 –12 amu
Q kg =
(3 ´ 108 )2 1.67 ´ 10 –27 ´ (3 ´ 108 ) 2 and Nz = N0 – Nx – Ny
æ 1020 ö 1020 æ 3 3 – 4ö
10 –12 amu = 1020 – ç = 1020 ç
= = 0.00665 amu ÷– è 3 3 ø
÷
1.67 ´ 9 ´ 10 –27 ´ 1016 è3 3ø 3
The mass of the parent nucleus X will be 33. (a) If x is the difference in quantum number of the states
mx = my + ma + Dm than x+1C2 = 6 Þ x = 3
= 223.61 + 4.002 + 0.00665 = 227.62 amu
n+3
T1/ 2 = 10 sec T1/ 2 = 30sec
32. X ¾¾¾¾¾¾ ® Y ¾¾¾¾¾¾ ®Z
-1 1
l x = 0.1s l y = s -1
30
The rate of equation for the population of X, Y and Z will be
dN x n
= –l x N x ...(i) Smallest l
dt
dN y – z 2 (13.6eV)
= –l y N y + l x N x ...(ii) Now, we have = – 0.85eV ...(i)
dt n2
dN z
= –l y N y ...(iii) – z 2 (13.6eV)
dt and = –0.544 eV ...(ii)
Þ On integration, we get ( n + 3) 2
N x = N 0e – l xt ...(iv) Solving (i) and (ii) we get n = 12 and z = 3
(b) Smallest wavelength l is given by
Given
l x N 0 é –l y t hc
= (0.85 – 0.544) eV
Ny = e – el x t ù l
l x – lY êë úû
Solving, we get l = 4052 nm.
To determine the maximum NY, we find 34. (a) Number of electrons falling on the metal plate A
dNY = 1016 × (5 × 10–4)
=0
dt
From (ii) A B
-l y N y + l x N x = 0
Þ lx Nx = l y Ny .......(v)

( )
é l x N0 -l t ù
Þ l x ( N 0 e -l xt ) = l y ê e y - e l xt ú
ëê l x - l y úû
-l y t
lx - ly e - e -l xt lx ( l -l )t
d = 1 cm
Þ = Þ =e x y
ly e -l xt ly
\ Number of photoelectrons emitted from metal plate A upto
lx
Þ log e
ly
(
= lx - ly t ) 10 seconds is

(5 ´ 104 ) ´ 1016
ne = ´ 10 = 5 ´ 107
é æ 1 öù 10 6
log e ê0.1/ ç ÷ ú
log e ( l x / lg ) ë è 30 ø û (b) Charge on plate B at t = 10 sec
Þt= = = 15 log e 3
lx – lg 1 Qb = 33.7 × 10–12 – 5 × 107 × 1.6 × 10–19 = 25.7 × 10–12 C
0.1 –
30 also Qa = 8 × 10–12 C
MODERN PHYSICS P-533

sB s A 1 39. KEY CONCEPT :


E= – = (QB – QA ) The wavelength l, of photon for different lines of Balmer
2e 0 2e0 2 Ae0 series is given by
17.7 ´ 10–12 hc é1 1ù
= –4 –12 = 2000 N/C = 13.6 ê – ú eV, where n = 3, 4, 5
5 ´ 10 ´ 8.85 ´ 10 l ë2 2
n2 û
(c) K.E. of most energetic particles Using above relation, we get the value of l = 657nm, 487 nm
= (hn – f) + e (Ed) = 23 eV between 450 nm and 700 nm. Since 487 nm, is smaller than
Note : (hn – f) is energy of photoelectrons due to light e 657 nm, electron of max. K.E. will be emitted for photon
(Ed) is the energy of photoelectrons due to work done by corresponding to wavelength 487 nm with
photoelectrons between the plates.
35. According to Bohr’s model, the energy released during hc æ 1242 ö
(K.E.) = –W = ç – 2÷ = 0.55 eV
transition from n2 to n1 is given by l è 487 ø
é1 1ù 40. The de Broglie wave length is given by
DE = hn = Rhc( Z – b)2 ê 2 – 2 ú
h h
ëê n1 n2 ûú l= Þl=
For transition from L shell to K shell mv 2mK
b = 1, n2 = 2, n1 = 1 Case (i) 0 £ x £ 1
é1 1 ù For this, potential energy is E0 (given)
\ ( Z –1)2 Rhc ê – ú = hn Total energy = 2E0 (given)
ë1 4 û \ Kinetic energy = 2E0 – E0 = E0
On putting the value of R = 1.1 × 107 m –1 (given),
c = 3 × 108 m/s, we get h
l1 = ...(i)
Z = 42 2mE0
A Case (ii) x > 1
log e 0
36. l= A = 1 log n For this, potential energy = 0 (given)
t 2 e 0.75n Here also total energy = 2E0 (given)
\ Kinetic energy = 2E0
1 2
Þ Mean Life = =
h
l loge 4 / 3 \ l2 =
2m(2 E0 ) ....(ii)
37. (a) eV0 = hn – hn0 = 5 – 3 = 2 eV
\ V0 = 2 volt Dividing (i) and (ii)
(b) Note : When the intensity is doubled, the saturation
current is also doubled. l1 2 E0 l
= Þ 1 = 2
l2 E0 l2
8µA –5 2
41. (a) KEY CONCEPT : We know that radius of nucleus is
(b) I = 2 × 10 w/m
4µA –5 2
given by formula
(a) I = 10 w/m r = r0A1/3 where r0 = constt, and A = mass number.
For the nucleus r1 = r041/3
Vs = – 2V For unknown nucleus r2 = r0 (A)1/3
1/ 3 1/ 3
r2 æ A ö 1/ 3 æ A ö
\ = ç ÷ , (14) = çè ÷ø Þ A = 56
r1 è 4 ø 4
38. a = Initial Uranium atom \ No of proton = A – no. of neutrons = 56 – 30 = 26
(a – x) = Uranium atoms left \ Atomic number = 26
n
æ 1ö é1
( a – x) = a ç ÷ 1ù
è 2ø (b) We know that n = Rc (Z – b)2 ê 2 – 2 ú
ëê n1 n2 úû
t 1.5 ´ 109 1 Here, R = 1.1 × 107 , c = 3 × 108, Z = 26
and n = = 9
=
t1/ 24.5 ´ 10 3 b = 1 (for Ka), n1 = 1, n2 = 2
1 1/ 3 é1 1ù
\ a – x = a( ) \ n = 1.1 × 107 × 3 × 108 [26 – 1]2 ê – ú
2 ë1 4 û
a 1 21/ 3 3
Þ = = = 1.26 = 3.3 ´ 1015 ´ 25 ´ 25 ´= 1.546 ´ 1018 Hz
a – x (1/ 2)1/ 3 1 4
x 42. Note : nth line of Lyman series means electron jumping
Þ = 1.26 – 1 = 0.26
a–x from (n + 1)th orbit to 1st orbit.
P-534 Topic-wise Solved Papers - PHYSICS
For an electron to revolve in (n + 1)th orbit. (q) When a gas in an adiabatic container fitted with an
2pr = (n + 1)l adiabatic piston is compressed by pushing the piston
(i) the internal energy of the system increases
2p 2p 2
Þ l= ´r = é 0.529 ´ 10 –10 ù (n + 1) DU = Q - W = 0 - (- PdV ) = + PdV
(n + 1) (n + 1) ë û Z
(ii) Mechanical energy is proceeded to the piston
1 Z which is converted into kinetic energy of the gas
Þ = molecules.
l 2p é0.529 ´ 10 –10 ù ( n + 1) ..(i)
ë û (r) None of the options in column I matches. As the gas in
Also we know that when electron jumps from (n + 1)th orbit a rigid container gets cooled, the internal energy of the
to 1st orbit. system will decrease. The average kinetic energy per
molecule will decrease.
1 é1 1 ù é 1 ù (s) When a heavy nucleus initially at rest splits into two
= RZ 2 ê 2 – 7 2
ú = 1.09 ´ 10 Z ê1– 2ú nuclei of nearly equal masses and some neutrons are
l 2
êë1 (n + 1) úû êë (n + 1) úû
emitted then
...(ii) (i) Internal energy of the system is converted into
From (i) and (ii) mechanical energy (precisely speaking kinetic
energy) and
Z é 1 ù
= 1.09 ´ 107 Z 2 ê1– (ii) Mass of the system decreases which converts
–10 2ú
2p (0.529 ´ 10 )( n + 1) ëê (n + 1) ûú into energy.
On solving, we get n = 24 × × ×

MATCH THE FOLLOWING : × × ×


1. A ® p; B ® t; C ® u; D ® r
The correct match is as follows : × × ×
(A) Energy of thermal neutrons (p) 0.025 eV
(t) When a resistive wire loops is placed in a time varying
(B) Energy of X-rays (t) 10 k eV
magnetic field perpendicular to its palne.
(C) Binding energy per nucleon (u) 8 M eV
(i) Induced current shows in the loop due to which the
(D) Photoelectric threshold (r) 3 eV
energy of system is increased.
of a metal
15
2. A ® p, q; B ® p, r; C ® p, s; D ® p, q, r 5. (c) 8 O ¾® 15
7 N+
0
1b
In a nuclear fusion reaction matter is converted into energy +
b particle
and nuclei of low atomic number generally given this reaction.
238 234 4
In a nuclear fission reaction matter is converted into energy 92 U ¾® 90 Th + 2 He
and nuclei of high atomic number generally given this a- particle
reaction.
185
3. A ® p, r 83 Bi ¾® 184 1
82 Pb + 1 H
Reason : Characteristic X-ray are produced due to transition proton
of electrons from one energy level to another. 239
94 Pu ¾® 140 99
57 La + 37 X
Similarly the lines in the hydrogen spectrum is obtained due
to transition of electrons from one energy level to another. (c) is the correct option.
B ® q, s COMPREHENSION BASED Q UESTIONS :
Reason : In photoelectric effect electrons from the metal
surface are emitted out upon the incidence of light of -13.6 Z2
appropriate frequency. 1. (c) For hydrogen like atoms En = eV / atom
n2
Note : In b-decay, electrons are emitted from the nucleus of
an atom. For hydrogen atom
E1 = -13.6eV
C®p (Z = 1)
Moseley gave a law which related frequency of emitted X- E2 = – 3.4eV
ray with the atomic number of the target material \ DE = E2 – E1 = – 3.4 – (– 13.6) = 10.2 eV
n = a( Z – b) i.e., when hydrogen comes to ground state it will release
D®q 10.2 eV of energy.
In photoelectric effect, energy of photons of incident ray For He+ion
gets converted into kinetic energy of emitted electrons. E1 = – 13.6 × 4 eV = – 54.4 eV
(Z = 2)
4. A ® p, q, t; B ® q; C ® s; D ® s
(p) When an uncharged capacitor is connected to a battery, E2 = – 13.6 eV
it becomes charged and energy is stored in the E3 = – 6.04 eV
capacitor. (A) is the correct option. E4 = – 3.4eV
MODERN PHYSICS P-535
Here He+ ion is in the first excited state i.e., possessing The length in which the particle is restricted to move is
energy – 13.6 eV. After receiving energy of + 10.2 eV l
from excited hydrogen atom on collision, the energy of a. This length is a multiple of .
electron will be (–13.6 + 10.2) eV = –3.4 eV. This means 2
that the electron will jump to n = 4. l 2a
2. (c) After collision with hydrogen atom the He+ ion is in its Now, n =a Þ l=
2 n
third excited state (n = 4). After that the electron can
jump into n = 3. h 2 n2 n 2 h2
hc Þ E= =
DE = hn = = E4 - E3 2 m ´ 4a 2 8m a 2
l
é -13.6 ´ 4 æ -13.6 ´ 4 ö ù -19
Þ E µ a -2 for a particular value of n.
=ê -ç ÷ø ú ´ 1.6 ´ 10 8. (b) For ground state n = 1,
ë 16 è 9 û
Given m = 1.0 × 10–30 kg, a = 6.6 × 10–9 m
-34
\ 6.6 ´ 10 ´ 3 ´ 108 é 1 1ù
= -13.6 ´ 4 ê - ú 12 ´ (6.6 ´ 10-34 )2
l ë16 9 û \ E= J = 8 meV
8 ´1´10-30 ´ (6.6 ´10-9 ) 2
6.6 ´ 10 -34 ´ 3 ´ 108 ´ 9 ´ 16
l= = 4.68 ´ 10 -7 m h h h
7 ´ 13.6 ´ 4 ´ 1.6 ´ 10 -19 9. (d) l= Þ l= Þ mv =
p mv l
Since only one option is correct, we need not work out
the case of electron jumping from n = 4 to n = 2 . nl 2a
But =a Þ l=
+13.6 ´ 12 2 n
3. (a) K. E. for hydrogen atom (for n = 2) = eV
4 nh nh
\ mv = Þ v= Þ v µn
2 2 am
13.6 ´ 2 2a
K. E for He+ (for n = 2) = = 13.6 eV
22 nh
10. (d) According to Bohr’s quantisation principle L =
1 2p
\Ratio = 2
4 1 2 1 é Lù
Rotational kinetic energy = Iw = I ê ú
4. (d) The collection of 12 H nuclei and electron is known as 2 2 ëI û
plasma which is formed due to high temperature inside [Q L = Iw]
the reactor core.
2é h
2 ù
5. (a) Applying conservation of mechanical energy we get 1 L2 1 n2 h 2
= = ´ = n ê 2 ú ...(i)
Loss of kinetic energy of two deuteron nuclei 2 I 2I 4p 2 ë 8p I û
= Gain in their potential energy.
11. (b) Energy given = change in kinetic energy
1 e´e
2 ´ 1.5kT = h2
4pe0 r hn = K f - Ki = [22 - 12 ] [From (i)]
8p 2 I
æ eV ö (1.44 ´ 10-9 eVm) 3h 2
Þ 2 ´ 1.5 ´ ç 8.6 ´ 10-5 ÷ ´T = hn =
è k ø 4 ´ 10-15 m 8p 2 I

1.44 ´ 10-9 3h 3 ´ 2 p ´ 10 -34 3


Þ T= = 1.4 × 109K ÞI= = = ´ 10 -45
-15 2 4
8p n 8p 2 ´ ´ 1011 16
2 ´ 1.5 ´ 8.6 ´ 10-5 ´ 4 ´ 10
p
6. (b) For the reading B we get nto > 5 × 1014 which is the –46 2
Lawson criterion for a reactor to work successfully. = 1.87 × 10 kg m
12. (c) Centre of mass divides the distance between the point
h p2 masses in inverse ratio of their masses.
7. (a) l = and E =
p 2m m2 d m1d
\r1 = and r2 =
n loops m1 + m2 m1 + m2
x=0 x=a Also the moment of inertia of the system is
C
r1 r2
h2 c.m
Þ E= d
2ml 2
P-536 Topic-wise Solved Papers - PHYSICS

I = m1r12 + m2 r22 2. We know that N = N 0e-lt


2 dN
é 5 -27 ù \ = N 0 e –λt (–λ) = –N0 λ e –λt
ê16 ´ 3 ´ 10 ´ d ú dt
5 ê ú
Þ 1.87 ´ 10 -46 = 12 ´ ´ 10 -27 Taking log on both sides
3 ê 28 ´ 5 ´ 10-27 ú
ë 3 û dN
log e = loge (- N0 l) - lt
2
dt
é 5 ù Comparing it with the graph line,
12 ´ ´ 10-27 d ú
5 -2 ê 3
+ 16 ´ ´ 10 ê ú 1 -1 é AC 1 ù
3 ê 28 ´ 5 ´ 10-27 ú we get l = yr êë BC = 2 úû
ë 3 û 2

Þ d = 1.3 × 10–10 m
13. (d) K should be less than 0.8 × 106 eV as anti-neutrino will 6
have some energy.
5
14. (c) The energy shared between anti-neutrino and electron. A
If the energy of electron is almost zero then the 4
maximum energy of anti-neutrino is nearly 0.8 × 106 eV. B
3 C
210 206
15. (a) 84 Po ¾® 82 Pb + 42 He 2
Here Dm = [209.982876– (205.974455 + 4.002603)]× 932 MeV 1
= 5.422 MeV = 5422 keV 2 3 4 5 6 7 8
By conservation of linear momentum
Linear momentum of a-particle = linear momentum of 0.693
\ T1/ 2 = = 0.693 ´ 2 = 1.386 years
lead l
pa = plead
t 4.16
2ma K.E a = 2m lead K.E lead N æ 1 ö T1/ 2 1 æ 1 ö 1.386 1
Now =ç ÷ Þ =ç ÷ =
N0 è 2 ø p è 2ø 8
m lead ´ K.E lead 206 ´ K.E lead
\K.E = = ...(1) 3. 1
ma 4
dN 1
Also K.Ea + K.Elead = 5422 keV ...(2) We know that , = lN = N
On solving the above two equations we get dt Tmean
K.Ea = 5319 keV
1
(a) is the correct option. \ 1010 = ´N
16. (c) Only in case of c we have m3 + m4 > M¢ 109
(c) is the correct option. \ N = 1019
In other cases of fission m1 + m2 < M and in the other i.e. 1019 radioactive atoms are present in the freshly prepared
case of fusion m3 + m4 > M¢ sample.
The mass of the sample = 1019 × 10–25 kg = 10–6kg = 1 mg
ASSERTION & REASON TYPE QUESTIONS : 4. 7
1. (b) Statement 1 : The wavelength of characteristic X-rays 1 é hc ù
depends on the type of atoms of which the target material is Stopping potential = ê - f ú where hc = 1240eV –nm
made. It does not depend on the accelerating potential. eë l û
Therefore, statement I is true.
1 é 1240 ù 1
Statement 2 : When an electric beam strikes the target in an = ê - 4.7 ú = [6.2 - 4.7 ]
X-ray tube, part of the kinetic energy is converted into X- e ë 200 û e
ray energy. This statement is true. But statement 2 does not
1
explain statement 1. = ´ 1.5eV = 1.5V
e
INTEGER VALUE CORRECT TYPE :
1 q 1 ne
But V = =
h 4pe 0 r 4pe 0 r
1. We know that, l =
2mqV
Vr (4pe0 ) 1.5 ´ 10 -2
\n= =
l1 ma qa 4 2 e 9 ´ 109 ´ 1.6 ´ 10 -19
\ = = ´ = 8»3 \ n = 1.04 × 107
la mpqp 1 1
Comparing it with A × 10z we get, z = 7
MODERN PHYSICS P-537
5. (7) Loss in K.E. of proton = Gain in potential energy 7. (4) For a radioactive decay
of the proton – nucleus system
1 1 q1 q2 N = N 0 e- l t
mv2 =
2 4 p e0 r
N
\ = e-l t
p2 1 q1 q2 N0
\ =
2m 4 p Î0 r
N
1 æ h2 ö 1 q1 q2 \1 - = 1 - e-l t
ç 2÷ = N0
\ 2m è l ø 4p Î0 r
0.693
4 p Î0 r . h 2 N -N
-
t1 2
´t
\ l= = 7 fm \ 0 = 1- e
q1 q2 (2 m) N
6. (1) For photoelectric effect Vo
hn f o = 1 - e -0.04 = 1 - (1 - 0.04)
- = Vo
e e
The slope is » 0.04 = 4% [Q e- x = 1 - x x <<1]
q n
h
tan θ = = constant
e
\ The ratio will be 1.

1. (c) KEY CONCEPT : ALTERNATE SOLUTION


The energy of nth orbit of hydrogen is given by 15
13.6 Number of half lives n = =3
En = - eV /atom 5
2
n n 3
æ 1ö æ 1ö N0
-13.6 We know that N = N 0 ç ÷ = N 0 ç ÷ =
For n = 2, En = = -3.4 eV è 2ø è 2ø 8
4
Therefore the energy required to remove electron from 7. (c) Specific resistance is resistivity which is given by
n = 2 is + 3.4 eV.
m
2. (c) Pure silicon, at absolute zero, will contain all the r=
electrons in bounded state. The conduction band will ne 2 t
be empty. So there will be no free electrons (in where n = no. of free electrons per unit volume
conduction band) and holes (in valence band) due to and t = average relaxation time
thermal agitation. Pure silicon will act as insulator. For a conductor with rise in temperature n increases
3. (a) Charged particles are deflected in magnetic field. and t decreases. But the decrease in t is more dominant
4. (c) We know that work function is the energy required than increase in n resulting an increase in the value of
and energy E = hu r.
For a semiconductor with rise in temperature, n
E Na h u Na l Cu é 1 ù increases and t decreases. But the increase in n is more
\ = = êëQ u µ l for light úû
E Cu h uCu l Na dominant than decrease in t resulting in a decrease in
the value of r.
l Na E Cu 4.5 2
\ l = E = 2.3 » 1 ALTERNATE SOLUTION
Cu Na
r 2 = r1 (1 + aDT )
5. (a) Formation of covalent bond is best explained by
molecular orbital theory. For conductor a is positive
6. (a) After every half-life, the mass of the substance reduces \ r2 > r 1 for D T positive i.e., increase in temperature.
to half its initial value. For semi conductor a is negative
\ r2 < r 1 for D T positive.
5 years N0 5 years N0 / 2
N 0 ¾¾¾¾
® ¾¾¾¾
® 8. (c) The energy band gap is maximum in insulators. Because
2 2 of this the conduction band of insulators is empty.
N0 5years N0 / 4 N0 9. (a) Emitter sends the majority charge carrriers towards the
= ¾¾¾¾
® = collector. Therefore emitter is most heavily doped.
4 2 8
P-538 Topic-wise Solved Papers - PHYSICS
10. (c) b -rays are fast moving beam of electrons and are Subtracting (ii) from (i) we get
1 2 1
therefore not electromagnetic waves ( hf1 - W ) – ( hf 2 - W ) = mv - mv
2 1 2 2
2

11. (c) The resistance of metal (like Cu) decreases with


decrease in temperature whereas the resistance of a m 2 2
\ h( f1 - f 2 ) = (v 1 - v 2 )
semi-conductor (like Ge) increases with decrease in 2
temperature. 2 22h
\ v1 - v 2 = (f - f )
12. (a) The electromagnetic spectrum is as follows m 1 2
¾¾¾¾¾¾¾¾¾¾¾¾¾¾¾¾¾¾¾¾¾¾®
increasing wavelength 18. (a) The radioactive substances emit a -particles (Helium
g - rays × x-rays × UV rays × visible rays × IR rays × microwaves × radiowaves nucleus), b – particles (electrons) and neutrinoes.
\ g-rays has least wavelength 3
13. (c) Here, conservation of linear momentum can be applied 19. (d) The average kinetic energy per molecule = kT
2
This kinetic energy should be able to provide the
repulsive potential energy
4
He 3
kT = 7.7 ´ 10 -14
234 2
Th \
90 2

2 ´ 7.7 ´ 10 -14 9
ÞT = -23
= 3.7 ´ 10
3 ´ 1.38 ´ 10
238 × 0 = 4 u + 234 v 20. (b) The ionisation potential increases from left to right in a
period and decreases from top to bottom in a group.
4
\ v=- u Therefore ceasium will have the lowest ionisation
234 potential.
21. (c) The wavelength of spectrum is given by
r 4
\ speed =| v |= u
234 1 æ 1 1ö 1.097 ´ 10 7
= Rz 2 ç 2 - 2 ÷ where R =
14. (b) When the temperature increases, certain bounded çè n m
l
1 n2 ÷ø 1+
electrons become free which tend to promote M
conductivity. Simultaneously number of collisions where m = mass of electron
between electrons and positive kernels increases M = mass of nucleus.
1 A 1 5000 For different M, R is different and therefore l is different
15. (a) l = log e o = log e 22. (c) It can be seen from the following graph -
t A 5 1250

2
Electric field
=
log e 2 = 0.4 log e 2
5
p n
16. (b) The number of a - particles released =8
Therefore the atomic number should decrease by 16 Distance
The number of b - -particles released = 4
Therefore the atomic number should increase by 4. 13.6
23. (a) 2
En = - Z eV/atom
+ 2
Also the number of b particles released is 2, which n
should decrease the atomic number by 2. For lithium ion Z = 3 ; n = 2 ( for first excited state)
Therefore the final atomic number 13.6
= Z –16 + 4 – 2 = Z –14 En = - 2
´ 32 = -30.6 eV
= 92 – 14 = 78
2
17. (a) For one photocathode E
24. (b) Momentum of photon =
1 2 c
hf1 - W = mv ....(i)
2 1 2E
Change in momentum =
For another photo cathode c
= momentum transferred to the surface
1 2
hf 2 - W = mv ....(ii) (the photon will reflect with same magnitude of
2 2 momentum in opposite direction)
MODERN PHYSICS P-539
25. (d) From Equation K .E = hn - f 7
36. (d) of Cu decays in 15 minutes.
slope of graph of K.E & n is h (Plank's constant) 8
which is same for all metals 3
7 1 æ1ö
26. (a) For the longest wavelength to emit photo electron \ Cu undecayed = N = 1 – = =ç ÷
8 8 è2ø
hc hc
=fÞl = \ No. of half lifes = 3
l f
t 15
-34 8
n= or 3 =
6.63 ´ 10 ´ 3 ´ 10 T T
Þl= = 310 nm
40 ´ 1.6 ´ 10 -16 15
Þ T = half life period = = 5 minutes
27. (b) From conservation of momentum m1v1 = m2 v2 3
2
1 I1 ær ö 1
æ m ö æv ö v 37. (a) I µ ; = çç 2 ÷÷ =
Þ ç 1 ÷ = ç 2 ÷ given 1 = 2 r
2 I 2 è r1 ø 4
è 2ø è 1ø
m v v 2
I 2 ® 4 times I1
3 1/ 3
m1 1 r1 1 æ r ö æ 1ö When intensity becomes 4 times, no. of photoelectrons
Þ = Þ = Þç 1÷ =ç ÷ emitted would increase by 4 times, since number of
m2 2 r2
3 2 è r2 ø è 2 ø
electrons emitted per second is directly proportional
to intensity.
28. (a) The chemical reaction of process is 212 H ®42 He 38. (d) Band gap = energy of photon of wavelength 2480 nm.
Energy released = 4 × (7) – 4(1.1) = 23.6 MeV So,
29. (a) KEY CONCEPT : hc æ 6.63 ´ 10 - 34 ´ 3 ´ 108 ö 1
Distance of closest approach DE = =ç ÷´ eV
l ç 2480 ´ 10 -9 ÷ -19
è ø 1.6 ´ 10
Ze(2e) = 0.5 eV
r0 =
æ 1 2ö
4pe 0 ç mv ÷ 39. (c) KEY CONCEPT : Intensity I = I 0 . e - md ,
è2 ø
Applying logarithm on both sides,
Energy, E = 5 ´ 106 ´ 1.6 ´ 10-19 J æ I ö
9 -19 -19
- md = log ç ÷
\ r0 = 9 ´ 10 ´ (92 ´ 1.6 ´ 10 ) (2 ´ 1.6 ´ 10 ) è I0 ø
5 ´ 106 ´ 1.6 ´ 10 -19
æ I /8ö
-14
- m ´ 36 = logç ÷ ..........(i)
Þ r = 5.2 ´ 10 m = 5.3 × 10–12 cm è I ø
30. (d) Holes move from base to emmitter. æ I / 2ö
-m ´ d = log ç ...........(ii)
31. (d) In common emitter configuration current gain è I ÷ø
- hf e -50 Dividing (i) by (ii),
Ai = = = –48.78
1 + boe RL -6 3
1 + 25 ´ 10 ´ 1 ´ 10 æ 1ö æ 1ö
log ç ÷ 3log ç ÷
32. (d) Copper is a conductor, so its resistance decreases on 36 è 8ø è 2ø 36
= = = 3 or d = = 12 mm
decreasing temperature as thermal agitation decreases,; d æ 1ö æ 1ö 3
whereas germanium is semiconductor therefore on log ç ÷ log ç ÷
è 2ø è 2ø
decreasing temperature resistance increases.
33. (b) Pauli’s exclusion principle. 40. (d) Zero; In common base amplifier circuit, input and output
voltage are in the same phase.
34. (a) Both the depletion region and barrier height is reduced.
é 1 1 ù
35. (b) KEY CONCEPT : R = R0 ( A)1/ 3 41. (b) KEY CONCEPT : E = Rhc ê 2 - 2 ú
êë n1 n2 úû
1/ 3 1/ 3
R æA ö æ 27 ö 3 E will be maximum for the transition for which
\ 1 = ç 1÷ =ç =
R2 è A2 ø è 125 ÷ø 5 é 1 1 ù is maximum. Here n is the higher energy
ê - ú 2
5 êë n12 n2 2 úû
R2 = ´ 3.6 = 6 fermi
3 level.
P-540 Topic-wise Solved Papers - PHYSICS

é 1 Ie ne eAve 7 7 ve v 5
1 ù 52. (c) = Þ = ´ Þ e =
Clearly, ê - ú is maximum for the third Ih nh eAvh 4 5 vh vh 4
êë n12 n2 2 úû
53. (b) D2 is forward biased whereas D1 is reversed biased.
transition, i.e. 2 ® 1 . I transition represents the So effective resistance of the circuit
absorption of energy. R = 4 + 2 = 6W
42. (d) de-Broglie wavelength,
12
\i = =2A
h h 1 6
l= = \ lµ
p 2. m.(K.E) K. E 54. (d) p-side connected to low potential and n-side is
connected to high potential.
l 55. (b) As l decreases, y increases and hence the speed of
If K.E is doubled, wavelength becomes photoelectron increases. The chances of photo electron
2
to meet the anode increases and hence photo electric
A current increases.
43. (a) ZX + 0 n1 ¾¾
® 3 Li7 + 2 He4
56. (b) Let E be the energy of proton, then
On comparison,
E + 7 ´ 5.6 = 2 ´ [4 ´ 7.06]
A = 7 + 4 – 1 = 10, z = 3 + 2 – 0 = 5
It is boron 5B10 Þ E = 56.48 - 39.2 = 17.28MeV
57. (c) The risk posed to a human being by any radiation
1
44. (d) Input frequency, f = 50 Hz Þ T = exposure depends partly upon the absorbed dose, the
50 amount of energy absorbed per gram of tissue.
Absorbed dose is expressed in rad. A rad is equal to
T 1
For full wave rectifier, T1 = = Þ f1 = 100 Hz. 100 ergs of energy absorbed by 1 gram of tissue. The
2 100 more modern, internationally adopted unit is the gray
45. (a) IC = 5.488 mA, Ie = 5.6 mA (named after the English medical physicist L. H. Gray);
one gray equals 100 rad.
5.488 a
a= ,b= = 49 58. (c) A crystal structure is composed of a unit cell, a set of
5. 6 1- a atoms arranged in a particular way; which is periodically
repeated in three dimensions on a lattice. The spacing
46. (a) f = 6.2 eV = 6.2 ´ 1.6 ´10 -19 J
between unit cells in various directions is called its
hc lattice parameters or constants. Increasing these lattice
V = 5 volt, - f = eV0 constants will increase or widen the band-gap (Eg),
l which means more energy would be required by
electrons to reach the conduction band from the
6.6 ´ 10 - ´ 3 ´ 10
34 8
hc -7 valence band. Automatically Ec and Ev decreases.
Þl= = » 10 m
f + eV0 1.6 ´ 10 -19 (6.2 + 5) 59. (a) Erms = 720
This range lies in ultra violet range. The average total energy density
47. (b) Work done to stop the a particle is equal to K.E. 1 2 1 2 2
= Î0 E0 = Î0 [ 2 Erms ] =Î0 Erms
1 2 K (Ze) 1 2 2 2
\ qV = mv Þ q ´ = mv
2 r 2 = 8.85 ´10-12 ´ (720)2 = 4.58 ´10-6 J / m3
2(2e) K ( Ze) 4 KZe
2 60. (c) Binding energy
Þr= 2
= 2 = [ZMP + (A – Z)MN – M]c2
mv mv = [8MP + (17 – 8)MN – M]c2
1 1 = [8MP + 9MN – M]c2
Þrµ and r µ . = [8MP + 9MN – Mo]c2
2 m
v
61. (c) There is no change in the proton number and the
48. (b) The order of time is nano second. neutron number as the g-emission takes place as a result
7 of excitation or de-excitation of nuclei. g-rays have no
49. (c) 3 Li + 11p ¾
¾® 8
4 Be + 0
0g charge or mass.
50. (c) The range of energy of b-particles is from zero to some 62. (a) The current will flow through RL when the diode is
maximum value. forward biased.
51. (b) Van der Waal's bonding is attributed to the attractive 63. (a) Energy of a photon of frequency n is given by E = h n .
forces between molecules of a liquid. The conductivity
Also, E = mc2, mc2 = hn
of semiconductors (covalent bonding) and insulators
(ionic bonding) increases with increase in temperature hn hn
Þ mc = Þ p=
while that of metals (metallic bonding) decreases. C c
MODERN PHYSICS P-541
64. (c) According to question, Also by de-broglie concept
Half life of X, T1/2 = tav , average life of Y h h h
l= = =
0.693 1 p 2mK.E 2meV
Þ = Þ l X = (0.693). l Y
lX lY
nh
\ lX < lY . \ = 2d cos i
2meV
Now, the rate of decay is given by
2
h
æ dN ö Here n =1 : V =
-ç = l X N0
è dt ÷ø x 8med 2 cos 2 i

æ dN ö (6.6 ´10-34 )2
-ç = l y N0 =
è dt ÷ø y 8 ´ 9.1´ 10-31 ´ 1.6 ´10 -19 ´ (10 -10 ) 2 ´ cos2 30
Y will decay faster than X. = 50 V
65. (a) Si and Ge are semiconductors but C is an insulator. ALTERNATE SOLUTION
Also, the conductivity of Si and Ge is more than C Using Bragg's equation 2d sinq = nl
because the valence electrons of Si, Ge and C lie in
third, fourth and second orbit respectively. Here n = 1, q = 90 – i = 90 – 30 = 60°
66. (d) We have to find the frequency of emitted photons. For \ 2d sin q = l .......(i)
emission of photons the transition must take place from
a higher energy level to a lower energy level which are
given only in options (c) and (d).
Frequency is given by

æ 1 1ö
hn = -13.6 ç - ÷ q
çè n 2 n 2 ÷ø
2 1
For transition from n = 6 to n = 2,

-13.6 æ 1 1 ö 2 æ 13.6 ö
n1 = ç - 2÷ = ´ç ÷
2 ø 9 è h ø
h è6 2
12.27 -10
For transition from n = 2 to n = 1, Also, l = ´ 10 m .......(ii)
V
-13.6 æ 1 1 ö 3 æ 13.6 ö
n2 = ç - 2÷ = ´ç ÷. 12.27
h è2 1 ø 4 è h ø
2 From (i) & (ii) 2 ´ 10 -10 ´ sin 60o = ´ 10 -10
V
\ n1 > n2
2
67. (b) The path difference between the rays APB and CQD is (12.27)
V= = 50 V
Dx = MQ + QN = d cos i + d cos i 3
Dx = 2d cos i 68. (b) 2d cos i = nldB
A B 69. (d) We know that energy is released when heavy nuclei
undergo fission or light nuclei undergo fusion.
C D Therefore statement (1) is correct.
The second statement is false because for heavy nuclei
i the binding energy per nucleon decreases with
increasing Z and for light nuclei, B.E/nucleon increases
with increasing Z.
P 70. (N) It is a p-n-p transistor with R as base. None of the
d M i N option is correct.
Q 2
k k mv
We know that for constructive interference the path 71. (b) When F = = centripetal force, then =
r r r
difference is nl
Þ mv2 = constat Þ kinetic energy is constant
\ nl = 2d cos i
Þ T is independent of n.
P-542 Topic-wise Solved Papers - PHYSICS
72. (d) 78. (d) We know that
A
eV0 = K max = hn - f

C where, f is the work function .


Hence, as n increases (note that frequency of X-rays
B
is greater than that of U.V. rays), both V0 and Kmax
The truth table for the above logic gate is : increase. So statement - 1 is correct
79. (c) In nuclear fission, the binding energy per nucleon of
A B C daughter nuclei is always greater than the parent
1 1 1 nucleus.
1 0 1 80. (b) By conservation of energy,
0 1 1 2.M 2 1 2M 2
( M + Dm) c2 = c + . v ,
0 0 0 2 2 2
where v is the speed of the daughter nuclei
This truth table follows the boolean algebra C = A + B
which is for OR gate 2 M 2 2 Dm
73 (c) It is given that transition from the state n =4 to n = 3 in Þ Dmc = v \v=c
2 M
a hydrogen like atom result in ultraviolet radiation. For
81. (b) As a result of emission of 1 a-particle, the mass number
infrared radiation the energy gap should be less. The
decreases by 4 units and atomic number decreases by
only option is 5 ® 4 . 2 units. And by the emission of 1 positron the atomic
number decreases by 1 unit but mass number remains
Increasing constant.
Energy n=5 \ Mass number of final nucleus = A – 12
n=4 Atomic number of final nucleus = Z – 8
n=3 \ Number of neutrons = (A – 12) – (Z – 8)
n=2 =A–Z –4
n=1 Number of protons = Z – 8
74. (a) l = 400 nm, hc = 1240 eV.nm, K.E. =1.68 eV A-Z -4
\ Required ratio =
Z -8
hc hc
We know that - W = K .E Þ W = - K .E 82. (a) The final boolean expression is,
l l
1240 X = ( A . B ) = A + B = A + B Þ OR gate
ÞW= - 1.68 = 3.1 – 1.68 = 1.42 eV
400
83. (a) Power, P = nhn, n = no, of photons per second
75. (d) For A + B ® C + e, e is positive. This is because Eb
P
for C is greater than the E b for A and B. Þ n=
nh
Again for F ® D + E + e, e is positive. This is
because E b for D and E is greater than Eb for F . 4 ´ 103
= = 6 × 1016 Hz
10 20
´ 6.63 ´ 10 -34
76. (d) Here y = ( A + B ) = A.B = A × B . Thus it is an AND gate
84. (b) For long distance communication, sky wave signals
for which truth table is
are used.
Also, the state of ionosphere varies every time. So,
A B y both statements are correct.
0 0 0 85. (b) Energy of excitation,
0 1 0 æ 1 1ö
D E = 13.6 p2 çè h - h ÷ø eV
1 0 0 1 2
1 1 1 æ1 1ö
Þ DE = 13.6 (3)2 çè - ÷ = 108.8 eV
12 32 ø
77. (b) We know that a single p-n junction diode connected to
an a-c source acts as a half wave rectifier [Forward 86. (b) Number of undecayed atom after time t2 ;
biased in one half cycle and reverse biased in the other N0
= N 0 e -lt2 ...(i)
half cycle]. 3
MODERN PHYSICS P-543
Number of undecayed atom after time t1; 92. (d) For gr ound state, the prin cipal quantum no
2N0 (n) = 1. There is a 3rd excited state for principal quantum
= N 0 e -lt1 ...(ii) number.
3

-lt
From (i), e 2 =
1
4 3rdexcitedstate

Pincipalquantumno.(n)
3

Energystates
æ 1ö
Þ –lt2 = loge çè ÷ø
3
...(iii) 3 2ndexcitedstate
2
From (ii) – e -lt2 =
3 2 1stexcitedstate
æ 2ö
Þ –lt1 = loge çè 3 ÷ø ...(iv) 1 groundstate
Solving (iii) and (iv), we get
Pos
siblen
1444444
umberofs
p ec
tr
424444444
allin
3
es
t2 – t1 = 20 mm The possible number of the spectral lines is given
87. (c) By Einstein photoelectric equation,
Kmax = eV0 = hv – hv0 n(n - 1) 4(4 - 1)
= = =6
When v is doubled, Kmax and V0 become more than 2 2
double. 93. (a)
88. (b) ( A + B) = NOR gate A
Y2 = A.AB
When both inputs of NAND gate are connected, it
behaves as NOT gate Y = A.AB B.AB
Y1 = AB
OR + NOT = NOR.
B
A Y3 = B.AB
B (A+B) By expanding this Boolen expression
(A+B)
Y = A.B + B.A
89. (c) pi = 0
Thus the truth table for this expression should be (1).
p f = p1 + p 2 , p i = p f 94. (a) Let d is the maximum distance, upto it condetict the
objects
0 = p1 + p 2 , p1 = - p 2 C
From DAOC

h h OC 2 = AC 2 + AO2 h
l1 = , l2 = d
p1 p2 B
(h + R)2 = d 2 + R 2
l1 = l 2 A q R
Þ d 2 = ( h + R )2 - R 2
l1 = l 2 = l. R
2 2 O
90. (a) Low frequencies cannot be transmitted to long d = ( h + R) - R
distances. Therefore, they are super imposed on a high
frequency carrier signal by a process known as d = h2 + 2hR
modulation.
Speed of electro-magnetic waves will not change due d = 5002 + 2 ´ 6.4 ´ 10 6 = 80 km
to modulation. So there will be time lag between
1
transmission and reception of the information signal. 95. (a) 0n ® 11H + -1e 0 + n + Q
¾¾
91. (b) Statement-1: Energy of b-particle from 0 to maximum The mass defect during the process
so E1 - E2 is the continuous energy spectrum.
Dm = mn - mH - me
Statement-2: For energy conservation and momentum
= 1.6725 × 10–27 – (1.6725 × 10–27+ 9 × 10–31kg)
conservation at least three particles, daughter nucleus,
b–1 and anti-neutron are required. = – 9 × 10–31 kg
P-544 Topic-wise Solved Papers - PHYSICS
The energy released during the process The higher frequency whcih can be detected with
E = Dmc2 tolerable distortion is
E = 9 × 10–31× 9 × 1016 = 81 × 10–15 Joules 1 1
f = = Hz
81 ´ 10 -15 2pma RC 2p ´ 0.6 ´ 2.5 ´ 10 -5
E= = 0.511MeV
1.6 ´ 10 –19
100 ´ 104
96. (d) The energy of the system of two atoms of diatomic = Hz
25 ´ 1.2p
1 2
molecule E = Iw 4
2 = ´ 104 Hz
where I = moment of inertia 1.2 p
= 10.61 KHz
L
w = Angular velocity = , This condition is obtained by applying the condition
I that rate of decay of capacitor voltage must be equal or
L = Angular momentum less than the rate of decay modulated singnal voltage
1 for proper detection of mdoulated signal.
I = (m1r12 + m 2 r22 ) 98. (b) From question,
2
B0 = 20 nT = 20 × 10–9T
1 2 2 2 (Q velocity of light in vacuum C = 3 × 108 ms–1)
Thus, E = (m1r1 + m2 r2 )w … (i)
2 r r r
E0 = B0 ´ C
1 L2 r r r
E = (m1r12 + m2 r2 2 ) 2 | E 0 |=| B | . | C |= 20 ´ 10 -9 ´ 3 ´ 108
2 I
= 6 V/m.
L = n h (According Bohr's Hypothesis)
99. (d) As l is increased, there will be a value of l above
1 L2 which photoelectrons will be cease to come out so
E = (m1r12 + m2 r22 ) photocurrent will become zero. Hence (d) is correct
2 (m1r12 + m2 r22 )2
answer.
100. (a) For same value of current higher value of voltage is
1 L2 n 2 h2 required for higher frequency hence (1) is correct
E= =
2 (m1r12 + m2 r22 ) 8p2 (m1r12 + m2 r22 ) answer.
101. (d) DE = hv
(m1 + m2 )n 2 h 2
E= DE é 1 1 ù k(2n - 1)
8 p2 r 2 m1m2 n= =kê 2
- 2ú = 2
h ë (n - 1) n û n (n - 1)2
97. (b) Given : Resistance R = 100 kilo ohm = 100 × 103 W
Capacitance C = 250 picofarad = 250 × 10–12F 2k
»
t = RC = 100 × 103 × 250 × 10–12 sec n3
= 2.5 × 107 × 10–12 sec
= 2.5 × 10–5 sec 1
or, n µ 3
n

Вам также может понравиться